February 2014 MEE Questions and AnalysesPreface The Multistate Essay Examination (MEE) is developed...

38
February 2014 MEE Questions and Analyses National Conference of Bar Examiners 302 South Bedford Street | Madison, WI 53703-3622 Phone: 608-280-8550 | Fax: 608-280-8552 | TDD: 608-661-1275 www.ncbex.org e-mail: [email protected]

Transcript of February 2014 MEE Questions and AnalysesPreface The Multistate Essay Examination (MEE) is developed...

Page 1: February 2014 MEE Questions and AnalysesPreface The Multistate Essay Examination (MEE) is developed by the National Conference of Bar Examiners (NCBE). This publication includes the

February 2014 MEE Questions and Analyses

National Conference of Bar Examiners 302 South Bedford Street | Madison WI 53703-3622 Phone 608-280-8550 | Fax 608-280-8552 | TDD 608-661-1275

wwwncbexorg e-mail contactncbexorg

Copyright copy 2014 by the National Conference of Bar Examiners All rights reserved

Contents

Prefaceii

Description of the MEE ii

Instructionsiii

February 2014 Questions

Constitutional Law Question 3

Trusts and Future Interests Question 4

Secured Transactions Question 5

Federal Civil Procedure Question 6

Criminal Law and Procedure Question 7

Agency and Partnership Question 8

February 2014 Analyses

Constitutional Law Analysis 11

Trusts and Future Interests Analysis 14

Secured Transactions Analysis 17

Federal Civil Procedure Analysis 19

Criminal Law and Procedure Analysis 23

Agency and Partnership Analysis 27

i

Preface

The Multistate Essay Examination (MEE) is developed by the National Conference of Bar Examiners (NCBE) This publication includes the questions and analyses from the February 2014 MEE (In the actual test the questions are simply numbered rather than being identified by area of law) The instructions for the test appear on page iii

The model analyses for the MEE are illustrative of the discussions that might appear in excellent answers to the questions They are provided to the user jurisdictions to assist graders in grading the examination They address all the legal and factual issues the drafters intended to raise in the questions

The subjects covered by each question are listed on the first page of its accompanying analysis identified by roman numerals that refer to the MEE subject matter outline for that subject For example the Federal Civil Procedure question on the February 2014 MEE tested the following area from the Federal Civil Procedure outline IVD Pretrial proceduresmdashDiscovery (including e-discovery)

For more information about the MEE including subject matter outlines visit the NBCE website at wwwncbexorg

Description of the MEE

The MEE consists of six 30-minute essay questions and is a component of the Uniform Bar Examination (UBE) It is administered by participating jurisdictions on the Tuesday before the last Wednesday in February and July of each year The areas of law that may be covered by the questions on any MEE are Business Associations (Agency and Partnership Corporations and Limited Liability Companies) Conflict of Laws Constitutional Law Contracts Criminal Law and Procedure Evidence Family Law Federal Civil Procedure Real Property Torts Trusts and Estates (Decedentsrsquo Estates Trusts and Future Interests) and Uniform Commercial Code (Negotiable Instruments and Bank Deposits and Collections Secured Transactions) Some questions may include issues in more than one area of law The particular areas covered vary from exam to exam

The purpose of the MEE is to test the examineersquos ability to (1) identify legal issues raised by a hypothetical factual situation (2) separate material which is relevant from that which is not (3) present a reasoned analysis of the relevant issues in a clear concise and well-organized composition and (4) demonstrate an understanding of the fundamental legal principles relevant to the probable solution of the issues raised by the factual situation The primary distinction between the MEE and the Multistate Bar Examination (MBE) is that the MEE requires the examinee to demonstrate an ability to communicate effectively in writing

ii

Instructions The back cover of each test booklet contains the following instructions

You will be instructed when to begin and when to stop this test Do not break the seal on this booklet until you are told to begin

You may answer the questions in any order you wish Do not answer more than one question in each answer booklet If you make a mistake or wish to revise your answer simply draw a line through the material you wish to delete

If you are using a laptop computer to answer the questions your jurisdiction will provide you with specific instructions

Read each fact situation very carefully and do not assume facts that are not given in the question Do not assume that each question covers only a single area of the law some of the questions may cover more than one of the areas you are responsible for knowing

Demonstrate your ability to reason and analyze Each of your answers should show an understanding of the facts a recognition of the issues included a knowledge of the applicable principles of law and the reasoning by which you arrive at your conclusions The value of your answer depends not as much upon your conclusions as upon the presence and quality of the elements mentioned above

Clarity and conciseness are important but make your answer complete Do not volunteer irrelevant or immaterial information

Answer all questions according to generally accepted fundamental legal principles unless your testing jurisdiction has instructed you to answer according to local case or statutory law

NOTE Examinees testing in UBE jurisdictions must answer according to generally accepted fundamental legal principles rather than local case or statutory law

iii

February 2014 MEE

QUESTIONS Constitutional Law

Trusts and Future Interests Secured Transactions

Federal Civil Procedure Criminal Law and Procedure

Agency and Partnership

CONSTITUTIONAL LAW QUESTION ___________

A city ordinance required each downtown business to install high-powered halogen floodlights that would illuminate the property owned by that business and the adjoining sidewalks A study commissioned by the city estimated that installation of the floodlights would cost a typical business about $1000 but that increased business traffic due to enhanced public safety especially after dark would likely offset this cost

A downtown restaurant applied to the city for a building permit to construct an addition that would increase its seating capacity In its permit application the restaurant accurately noted that its current facility did not have sufficient seating to accommodate all potential customers during peak hours The city approved the permit on the condition that the restaurant grant the city an easement over a narrow strip of the restaurantrsquos property to be used by the city to install video surveillance equipment that would cover nearby public streets and parking lots The city based its permit decision entirely on findings that the increased patronage that would result from the increased capacity of the restaurant might also attract additional crime to the neighborhood and that installing video surveillance equipment might alleviate that problem

The restaurant has challenged both the ordinance requiring it to install floodlights and the easement condition imposed on approval of the building permit

1 Under the Fifth Amendment as applied to the states through the Fourteenth Amendment is the city ordinance requiring the restaurant to install floodlights an unconstitutional taking Explain

2 Under the Fifth Amendment as applied to the states through the Fourteenth Amendment is the cityrsquos requirement that the restaurant grant the city an easement as a condition for obtaining the building permit an unconstitutional taking Explain

3

TRUSTS AND FUTURE INTERESTS QUESTION _______________

Ten years ago a testator died survived by his only children a son age 26 and a daughter age 18

A testamentary trust was created under the testatorrsquos duly probated will The will specified that all trust income would be paid to the son during the sonrsquos lifetime and that upon the sonrsquos death the trust would terminate and trust principal would be distributed to the testatorrsquos ldquograndchildren who shall surviverdquo the son The testator provided for his daughter in other sections of the will

Five years ago the trustee of the testamentary trust purchased an office building with $500000 from the trust principal Other than this building the trust assets consist of publicly traded securities

Last year the trustee received $30000 in rents from the office building The trustee also received with respect to the securities owned by the trust cash dividends of $20000 and a stock dividend of 400 shares of Acme Corp common stock distributed to the trust by Acme Corp

Eight months ago the trustee sold the office building for $700000

Six months ago the son delivered a letter to the trustee stating ldquoI hereby disclaim any interest I may have in the income interest of the trustrdquo On the date the son delivered this letter to the trustee the son had no living children the daughter had one living minor child

A statute in this jurisdiction provides that ldquoa disclaimer of any interest created by will is valid only if made within nine months after the testatorrsquos death and if an interest is validly disclaimed the disclaiming party is deemed to have predeceased the testatorrdquo

1 How should the rents sales proceeds cash dividends and stock dividends received prior to the trusteersquos receipt of the sonrsquos letter have been allocated between trust principal and income Explain

2 How if at all does the sonrsquos letter to the trustee affect the future distribution of trust income and principal Explain

4

SECURED TRANSACTIONS QUESTION

On March 1 the owner of a manufacturing business entered into negotiations with a bank to obtain a loan of $100000 for the business The bank loan officer informed the business owner that the interest rate for a loan would be lower if the repayment obligation were secured by all the businessrsquos present and future equipment The loan officer also informed the business owner that the bank could not commit to making the loan until its credit investigation was completed but that funds could be advanced faster following loan approval if a financing statement with respect to the transaction were filed in advance Accordingly the business owner signed a form on behalf of the business authorizing the bank to file a financing statement with respect to the proposed transaction The bank properly filed a financing statement the next day correctly providing the name of the business as the debtor and indicating ldquoequipmentrdquo as the collateral

On March 15 the business owner had heard nothing from the bank about whether the loan had been approved so the business owner approached a finance company for a loan The finance company quickly agreed to lend $100000 to the business secured by all the businessrsquos present and future equipment That same day the finance company loaned to the business $100000 and the business owner signed an agreement obligating the business to repay the loan and granting the finance company a security interest in all the businessrsquos ldquopresent and future equipmentrdquo to secure the repayment obligation Also on that day the finance company properly filed a financing statement correctly providing the businessrsquos name as the debtor and indicating ldquoequipmentrdquo as the collateral

On March 21 the bank loan officer contacted the business owner and indicated that the loan application had been approved On the next day March 22 the bank loaned the business $100000 The loan agreement signed by the owner on behalf of the business granted the bank a security interest in all the businessrsquos ldquopresent and future equipmentrdquo

On April 10 the business sold an item of manufacturing equipment to a competitor for $20000 This was the first time the business had ever sold any of its equipment The competitor paid the purchase price in cash and took possession of the equipment that day The competitor acted in good faith at all times and had no knowledge of the businessrsquos prior transactions with the bank and the finance company

The business has defaulted on its obligations with respect to the loans from the bank and the finance company Each of them has asserted a claim to all the businessrsquos equipment as well as to the item of equipment sold to the businessrsquos competitor

Assume that the business owner had the authority to enter into all these transactions on behalf of the business

1 As between the bank and the finance company which has a superior claim to the businessrsquos equipment Explain

2 Do the claims of the bank and the finance company to the businessrsquos equipment continue in the item of equipment sold to the competitor Explain

5

FEDERAL CIVIL PROCEDURE QUESTION

A builder constructed a vacation house for an out-of-state customer on the customerrsquos land The house was completed on June 1 at which point the customer still owed $200000 of the $800000 contract price which was payable in full five days later

On June 14 the basement of the house was flooded with two inches of water during a heavy rainfall When the customer complained the builder told the customer ldquoThe flooding was caused by poorly designed landscaping Our work is fine and fully up to code Have an engineer look at the foundation If therersquos a problem wersquoll fix itrdquo

The customer pleased by the builderrsquos cooperative attitude immediately hired a structural engineer to examine the foundation of the house On June 30 the engineer provided the customer with a written report on the condition of the foundation which stated that the foundation was properly constructed

Unhappy with the conclusions in the engineerrsquos report the customer then hired a home inspector to evaluate the house The home inspectorrsquos report concluded that the foundation of the house had been poorly constructed and was inadequately waterproofed

On July 10 the customer sent the builder the home inspectorrsquos report with a note that said ldquoUntil you fix this problem you wonrsquot get another penny from merdquo The builder immediately contacted an attorney and directed the attorney to prepare a draft complaint against the customer for nonpayment Hoping to avoid litigation the builder sent several more requests for payment to the customer The customer ignored all these requests

On September 10 the builder filed suit in federal district court properly invoking the courtrsquos diversity jurisdiction and seeking $200000 in damages for breach of contract The customerrsquos answer denied liability on the basis of alleged defective construction of the housersquos foundation

Several months later the case is nearly ready for trial However two discovery disputes have not yet been resolved

First despite a request from the builder the customer has refused to provide a copy of the report prepared by the structural engineer who examined the foundation of the house The customer claims that the report is ldquowork productrdquo and not discoverable because the customer does not intend to ask the engineer to testify at trial The builder has asked the court to order the customer to turn over the engineerrsquos report

Second the customer has asked the court to impose sanctions for the builderrsquos failure to comply with the customerrsquos demand for copies of all emails concerning construction of the foundation of the house The builder has truthfully informed the customer that all such emails were destroyed on August 2 This destruction was pursuant to the builderrsquos standard practice of permanently deleting all project-related emails from company records 60 days after construction of a project is complete There is no relevant state records-retention law

1 Should the court order the customer to turn over the engineerrsquos report Explain

2 Should the court sanction the builder for the destruction of emails related to the case and if so what factors should the court consider in determining those sanctions Explain

6

CRIMINAL LAW AND PROCEDURE QUESTION _____

A defendant was charged under state law with felony theft (Class D) and felony residential burglary (Class C) The indictment alleged that the defendant entered his neighborsrsquo home without their consent and stole a diamond ring worth at least $2500

Defense counsel filed a pretrial motion to dismiss the charges on the ground that prosecuting the defendant for both burglary and theft would constitute double jeopardy The trial court denied the motion and the defendant was prosecuted for both crimes The only evidence of the ringrsquos value offered at the defendantrsquos jury trial was the ownerrsquos testimony that she had purchased the ring two years earlier for $3000

At trial the judge issued the following jury instruction on the burglary charge prior to deliberations

If after consideration of all the evidence presented by the prosecution and defense you find beyond a reasonable doubt that the defendant entered the dwelling without the ownersrsquo consent you may presume that the defendant entered with the intent to commit a felony therein

The jury found the defendant guilty of both offenses

At the defendantrsquos sentencing hearing an expert witness called by the prosecutor testified that the diamond ring was worth between $7000 and $8000 Over defense objection the judge concluded by a preponderance of the evidence that the value of the stolen ring exceeded $5000 The judge sentenced the defendant to four yearsrsquo incarceration on the theft conviction On the burglary conviction the defendant received a consecutive sentence of seven yearsrsquo incarceration

In this state residential burglary is defined as ldquoentry into the dwelling of another without the consent of the lawful resident with the intent to commit a felony thereinrdquo Residential burglary is a Class C felony for which the minimum sentence is five years and the maximum sentence is ten years of incarceration

In this state theft is defined as ldquotaking and carrying away the property of another with the intent to permanently deprive the owner of possessionrdquo Theft is a Class D felony if the value of the item(s) taken is between $2500 and $10000 The sentence for a Class D felony theft is determined by the value of the items taken If the value is between $2500 and $5000 the maximum sentence is three yearsrsquo incarceration If the value of the items exceeds $5000 the maximum sentence is five yearsrsquo incarceration

This state affords a criminal defendant no greater rights than those mandated by the United States Constitution

1 Did the trial court err when it denied the defendantrsquos pretrial motion to dismiss on double jeopardy grounds Explain

2 Did the trial court err in its instruction to the jury on the burglary charge Explain

3 Did the trial court err when it sentenced the defendant to an additional year of incarceration on the theft conviction based on the expertrsquos testimony Explain

7

AGENCY AND PARTNERSHIP QUESTION _____

Five years ago Adam and Ben formed a general partnership Empire Partnership (Empire) to buy and sell antique automobiles at a showroom in State A Adam contributed $800000 to Empire and Ben contributed $200000 Their written partnership agreement allocated 80 of profits losses and control to Adam and 20 to Ben No filings of any type were made in connection with the formation of Empire

Three years ago a collector purchased one of Empirersquos antique cars for $3400000 The collector was willing to pay this price because of Benrsquos false representation (repeated in the sales contract) that a famous movie star had once owned the car Without the movie-star connection the car was worth only $100000 One month later when the collector discovered the truth he sued Adam Ben and Empire for $3300000 in damages The lawsuit is still pending

Two years ago Adam and Ben admitted a new partner Diane to Empire in return for her contribution of $250000 The three agreed to allocate profits losses and control 75 to Adam 10 to Ben and 15 to Diane Before joining the partnership Diane learned of the collectorrsquos claim and stated her concern to Adam and Ben that she might become liable if the claim were reduced to a judgment

Following Dianersquos admission to Empire the three partners sought to convert Empire into a limited liability partnership (LLP) Adamrsquos lawyer proposed to file with State A a ldquostatement of qualificationrdquo making an LLP election and declaring the name of the partnership to be ldquoEmpire LLPrdquo Benrsquos lawyer stated that this would not work and that a new LLP had to be formed with the assets of the old partnership transferred to the new one In the end the conversion was done the way Adamrsquos lawyer suggested with the approval of all three partners

One year ago a driver purchased a vintage car from Empire LLP based on the representation that the car was ldquofully roadworthy and capable of touring at 70 mph all dayrdquo The driver took the car on the highway at 50 mph whereupon the front suspension collapsed resulting in a crash in which the car was destroyed and the driver killed The driverrsquos estate sued Adam Ben Diane and Empire LLP for $10000000 The lawsuit is still pending

Although profitable Empire LLP does not have resources sufficient to pay the collectorrsquos claim or the claim of the driverrsquos estate

Assume that the Uniform Partnership Act (1997) applies

1 Before the filing of the statement of qualification (a) was Adam personally liable on the collectorrsquos claim Explain (b) was Diane personally liable on the collectorrsquos claim Explain

2 After the filing of the statement of qualification was Adam Ben or Diane personally liable as a partner on (a) the collectorrsquos claim or (b) the driverrsquos estatersquos claim Explain

8

February 2014 MEE

ANALYSES Constitutional Law

Trusts and Future Interests Secured Transactions

Federal Civil Procedure Criminal Law and Procedure

Agency and Partnership

CONSTITUTIONAL LAW ANALYSIS (Constitutional Law IVD)

ANALYSIS

Legal Problems

(1) Is the city ordinance requirement that businesses install floodlights a taking

(2) Is conditioning the approval of a building permit on the grant of an easement to install surveillance equipment a taking of property

DISCUSSION

Summary

The ordinance requiring businesses to install floodlights is not a per se taking under Loretto because it does not force a private landowner to allow a third party to enter and place a physical object on the land Here the city ordinance requires the businessmdashnot a third partymdashto install the floodlights

The ordinance is likely not a regulatory taking under the Penn Central balancing test While the ordinance will impose a cost on business owners that cost may be offset by the expected increase in business due to the ordinance and the ordinance does not appear to interfere with the ownerrsquos primary use of the property as a restaurant

The permit condition however is likely an uncompensated taking of property While the condition has an essential nexus with the cityrsquos legitimate interest in promoting public safety the city has not made an individualized determination that the easement condition is roughly proportional to the possibility of increased crime due to the restaurantrsquos proposed addition Thus the permit condition likely violates the Fifth Amendment as applied to the states through the Fourteenth Amendment

Point One (50) The ordinance requiring that businesses install floodlights is not a per se taking under Loretto It is not a regulatory taking under the Penn Central balancing test because the cost of compliance with the ordinance may be offset by an expected increase in business and compliance does not interfere with the businessrsquos primary use of its property as a restaurant

The city ordinance requiring a business to install floodlights does not effect a per se taking of the sort described in Loretto v Teleprompter Manhattan CATV Corp 458 US 419 (1982) because no property is physically taken by the government and the ordinance does not involve a physical invasion of private property by a third party

Even though the ordinance does not constitute an occupation of the property by either the government or a third party it is still subject to the three-factor balancing test under Penn Central Transportation Co v City of New York 438 US 104 (1978) to determine whether it is a ldquoregulatory takingrdquo Under Penn Central a court must balance (1) ldquo[t]he economic impact of the regulation on the claimantrdquo (2) ldquothe extent to which the regulation has interfered with distinct investment-backed expectationsrdquo and (3) ldquothe character of the governmental actionrdquo Id at 124 Here each factor weighs against finding that the ordinance is a taking

11

Constitutional Law Analysis

First the ordinance requirement likely has a minimal economic impact on the restaurant Compliance with the ordinance is estimated to cost $1000 and the city has found that businesses will likely recoup that cost in increased sales Also because the ordinance does not interfere with the operation of the restaurant the owner may still earn a reasonable return on its investment in the property

Second the ordinance does not interfere with the businessrsquos investment-backed expectations As in Penn Central the challenged law does not interfere with the ownerrsquos ldquoprimary expectationrdquo for use of the propertymdashin Penn Central as a railroad terminal and here as a restaurant Further the ordinance does not prevent the restaurant from expanding to meet the changing business environment

Third the character of the government action does not weigh in favor of a taking While Penn Central does say that a ldquophysical invasionrdquo is more likely to pose a taking Loretto suggests that the Courtrsquos main concern is with physical invasions by third parties Also like the landmark law challenged in Penn Central the ordinance here ldquoadjust[s] the benefits and burdens of economic life to promote the common goodrdquo Id In Penn Central the landmark law restricted development of the railroad terminal to promote the common interest in preserving historic landmarks Here the ordinance requires the businesses to install floodlights to promote the common interest in crime prevention and public safety

Because the ordinance is clearly a valid exercise of the police power it satisfies the takings clausersquos public-use requirement Kelo v City of New London 545 US 469 (2005)

In sum all three factors weigh against finding a taking under the Penn Central balancing test

Point Two (50) The permit condition may be unconstitutional as an uncompensated taking of property because the city has not made an individualized determination that the easement condition is roughly proportional to the impact of the restaurantrsquos proposed addition

In Dolan v City of Tigard 512 US 374 (1994) the Supreme Court set forth the test for determining whether an exaction imposed by a government in exchange for a discretionary benefit conferred by the government such as a condition on the approval of a building permit in this case constitutes an uncompensated taking under the Fifth Amendment The exaction is not a taking if (1) there is an ldquoessential nexusrdquo between the ldquopublic need or burdenrdquo to which the proposed development contributes and ldquothe permit condition exacted by the cityrdquo id at 386 and (2) the government makes ldquosome sort of individualized determination that the required dedication is [roughly proportional] both in nature and extent to the impact of the proposed developmentrdquo Id at 391 see also Nollan v California Coastal Commission 483 US 825 (1987)

Here the city likely can meet the nexus requirement In Dolan the landowner sought to double the size of its business which would have increased traffic on nearby roadways In exchange for approving the development the city sought an easement for a bike and pedestrian path The Court found the required nexus between the easement and the cityrsquos ldquoattempt to reduce traffic congestion by providing for alternative means of transportationrdquo 512 US at 387 Here a similar nexus likely exists between the requested easement and the cityrsquos interest in crime prevention and public safety Increased patronage and economic activity at the restaurant might attract additional crime to the area and the requested easement to install surveillance equipment would attempt to address that increased crime

12

Constitutional Law Analysis

The exaction here however may fail the second prong of the Dolan testmdashthat the exaction be roughly proportional to the anticipated impact of the requested development As noted the city in Dolan claimed that a bike and pedestrian path was needed to offset the increase in traffic due to the proposed doubling of the business The Court explained that the government must demonstrate that the additional traffic reasonably was related to the requested exaction and that the government must ldquomake some effort to quantify its findings in support of the dedication for the pedestrianbicycle pathway beyond the conclusory statement that it could offset some of the traffic demand generatedrdquo Id at 395 Here the city did not carry its burden The city simply speculates that increased patronage of the restaurant ldquomightrdquo increase crime and that the surveillance equipment ldquomightrdquo alleviate this increased crime Because the city has not made ldquosome effort to quantify its findingsrdquo in support of the easement it has not shown that the burden of the easement is roughly proportional to the benefits thought to flow from it

Thus the exaction appears to be an uncompensated taking of property in violation of the Fifth Amendment as applied to the states through the Fourteenth Amendment

13

TRUSTS AND FUTURE INTERESTS ANALYSIS ____ (Trusts and Future Interests IE3 I5 IIIA amp B)

ANALYSIS

Legal Problems

(1) How should rents dividends and sales proceeds received by the trustee prior to receipt of the sonrsquos letter have been allocated between trust income and principal

(2)(a) Did the remainder interest in the trust accelerate and become immediately payable to the daughterrsquos minor child upon the trusteersquos receipt of the sonrsquos letter and if not how should the trustee handle the distribution of the principal in the future

(2)(b) Following the trusteersquos receipt of the sonrsquos letter how should the trustee distribute future receipts of income prior to the distribution of the principal

DISCUSSION

Summary

Prior to the trusteersquos receipt of the sonrsquos letter cash dividends and rents should have been allocated to trust income and were distributable to the son the income beneficiary of the trust sales proceeds and stock dividends should have been allocated to principal

Because the sonrsquos letter to the trustee did not result in a valid disclaimer under state law (having been made more than nine months after the testatorrsquos death) the son is not deemed to have predeceased the testator Because the son is still living the class gift to the testatorrsquos grandchildren who survive the son has not closed and is not possessory it will not become possessory until the son dies The daughterrsquos minor child being the testatorrsquos only living grandchild is not currently entitled to a distribution of trust principal Trust principal will instead be distributable upon the sonrsquos death to the testatorrsquos then-living grandchildren or if there are none to the testatorrsquos then-living heirs

As for future income the trustee should either distribute the trust income to the son and the daughter as the testatorrsquos heirs accumulate the income for future distribution to those individuals ultimately entitled to the trust principal or distribute it to those presumptively entitled to the principal upon the sonrsquos death ie the daughterrsquos minor child

Point One (45) Cash dividends and rents are allocable to income sales proceeds and stock dividends are allocable to principal Items allocable to income for the period prior to the sonrsquos attempted disclaimer were distributable to the son

Receipts earned during the administration of a trust are allocable either to income or to principal Almost all states have adopted the most recent or an earlier version of the Uniform Principal and Income Act (the Act) which specifies how such receipts should be allocated

Under the Act rents (UNIF PRIN amp INC ACT (2000) sect 405 UNIF PRIN amp INC ACT (1962) sect 3(a)(1)) and cash dividends received from a corporation (UNIF PRIN amp INC ACT (2000) sect 401(b) UNIF PRIN amp INC ACT (1962) sect 6(d)) are allocable to income and are distributable to the income beneficiary of the trust

14

Trusts and Future Interests Analysis Sales proceeds (UNIF PRIN amp INC ACT (2000) sect 404(2) UNIF PRIN amp INC ACT (1962)

sect 3(b)(1)) and dividends paid in the stock of the distributing corporation (UNIF PRIN amp INC ACT (2000) sect 401(c)(1) UNIF PRIN amp INC ACT (1962) sect 3(b)(4)) are allocable to principal and added to the principal of the trust

Here the cash dividends and office building rents should have been allocated to income and until the trustee received the sonrsquos letter should have been distributed to him as the sole income beneficiary of the trust The stock dividend and proceeds from the sale of the office building should have been allocated to principal and held by the trustee for future distribution to the ultimate remaindermen of the trust

[NOTE The 2000 Uniform Principal and Income Act has been adopted in Alabama Arkansas Colorado Connecticut the District of Columbia Hawaii Idaho Iowa Kentucky Missouri Montana Nebraska New Mexico North Dakota Oregon South Dakota Utah and West Virginia]

Point Two(a) (45) Because the son did not disclaim within nine months of the testatorrsquos death there is no valid disclaimer under state law Therefore the son is not deemed to have predeceased the testator Furthermore because of the express survivorship contingency in the will the remainder in the trust does not accelerate and become distributable until the son in fact dies When the son dies the trust principal will be distributable to the testatorrsquos then-living grandchildren or if none then to the testatorrsquos then-living heirs

When a trust remainder is given to a class the class closes (ie no new persons can join the class) when there is no outstanding income interest and at least one member of the class is then entitled to demand possession of his or her share of the remainder This principle is called the rule of convenience See generally HERBERT HOVENKAMP amp SHELDON F KURTZ PRINCIPLES OF PROPERTY LAW 199ndash200 (6th ed 2005) A class member may demand possession of his or her share of the remainder upon termination of the income interest only when the class memberrsquos interest is not otherwise subject to a condition precedent See id

When a beneficiary timely disclaims an interest in a trust that beneficiary is treated as if he had predeceased the testator Here had the son disclaimed within nine months of the testatorrsquos death as required by the state statute he would have been deemed to have predeceased the testator This would have closed the class of remaindermen and the testatorrsquos then-living grandchildren (ie the daughterrsquos child) would have been entitled to the trust principal However under the state statute the sonrsquos disclaimer was not timely because he did not disclaim within nine months of the testatorrsquos death Thus because the statute is inapplicable and the son is still alive the class of grandchildren entitled to share in trust principal did not close

Because here the statute is inapplicable due to the sonrsquos failure to comply with the statutory time requirements then presumably the common-law rule allowing disclaimers (aka renunciations) at any time should apply Under the common law if a life estate is renounced the remainder interest accelerates and becomes immediately distributable to the remaindermen of the trust if the remainder is vested but not if the remainder is contingent JESSE DUKEMINIER amp ROBERT H SITKOFF WILLS TRUSTS AND ESTATES 844ndash845 (9th ed 2013) Here because the remainder is contingent upon there being grandchildren who survive the son the remainder will not accelerate It will remain open until the son dies leaving open the possibility that additional grandchildren will be included in the class or the daughterrsquos child could fall out of the class because that child fails to survive the son

And if none of the testatorrsquos grandchildren survive the son the trust principal will be distributed to the testatorrsquos heirs living at the sonrsquos death

15

Trusts and Future Interests Analysis

Point Two(b) (10) Until the trust terminates the trustee must continue to hold the trust assets The distribution of income in the meantime is unclear There are at least three possibilities Income earned on the undistributed assets could be distributed to the son and daughter as the testatorrsquos heirs accumulated and added to principal for distribution to the ultimate remaindermen or distributed from time to time to those persons who are presumptively remaindermen

When trust principal is not immediately distributable the trustee must continue to hold trust assets until the ultimate remaindermen are ascertained During this period trust income will be distributed or retained according to any instructions contained in the trust instrument See WILLIAM M MCGOVERN JR SHELDON F KURTZ amp DAVID M ENGLISH WILLS TRUSTS amp ESTATES sect 102 (4th ed 2010)

Here the testator did not specify what the trustee should do with trust income in the event the sonrsquos disclaimer did not comply with the state statute There are at least three approaches One approach would have the trustee distribute the trust income to the testatorrsquos heirs on the theory that the income represents property that was not disposed of by the testatorrsquos will and which thus passes by partial intestacy to the testatorrsquos heirs A second approach would have the trustee accumulate trust income for distribution to the ultimate remaindermen Under this approach only those individuals ultimately entitled to the principal would share in the income A third approach would have the trustee distribute trust income to those individuals who would be the remaindermen if the trust were to terminate when the income is received by the trustee under this approach trust income would be distributed to the daughterrsquos minor child until another presumptive remainderman is born This approach could result in individuals not ultimately entitled to principal say because they do not survive the son receiving income It could also result in a disproportionate distribution of income among the individuals ultimately entitled to income

[NOTE Examinees should demonstrate a recognition and understanding of the income-allocation problem and the alternatives available to address that issue There is no widely accepted solution to the problem Examinees who cite any of these possible problem-solving approaches may receive credit]

16

SECURED TRANSACTIONS ANALYSIS (Secured Transactions IB IID E amp F IIIB IVA B amp F)

ANALYSIS

Legal Problems

(1)(a) What is the nature of the bankrsquos claim to the businessrsquos equipment

(1)(b) What is the nature of the finance companyrsquos claim to the businessrsquos equipment

(1)(c) As between the bank and the finance company whose claim to the businessrsquos equipment has priority

(2) Do the claims of the bank and the finance company continue in the item of equipment sold by the business to the competitor

DISCUSSION

Summary

The bank and the finance company both have perfected security interests in the businessrsquos equipment Even though the finance companyrsquos perfected security interest was created first the bankrsquos perfected security interest has priority because the bankrsquos financing statement was filed before the finance companyrsquos financing statement The security interests of the bank and the finance company continue in the item of equipment sold by the business to the competitor because their security interests were perfected and the competitor was not a buyer in ordinary course of business

Point One(a) (25) The bank has a perfected security interest in the businessrsquos equipment

The bank has met all criteria necessary for it to have an attached and enforceable security interest in the businessrsquos equipment First value must be given UCC sect 9-203(b)(1) This criterion is fulfilled by the loan made by the bank to the business Second the debtor must have rights in the collateral UCC sect 9-203(b)(2) Clearly the business has rights in its equipment Third either the secured party must take possession of the collateral or the debtor must authenticate a security agreement containing a description of the collateral UCC sect 9-203(b)(3) The agreement that the business owner signed is a ldquosecurity agreementrdquo because it is an agreement that creates or provides for a security interest UCC sect 9-102(a)(74) By signing the security agreement the business owner authenticated it UCC sect 9-102(a)(7) Therefore all three criteria are fulfilled and the bank has an enforceable and attached security interest

A security interest is perfected when it has attached and when any additional steps required for perfection have occurred UCC sect 9-308(a) Generally speaking the additional steps will either be possession of the collateral by the secured party or the filing of a financing statement with respect to the collateral See UCC sectsect 9-310 9-313 In this case the bank filed a financing statement naming the debtor and sufficiently indicating the collateral The collateral indication is sufficient because it identifies the collateral by type of property See UCC sectsect 9-504 9-108 The fact that the financing statement was filed before the security interest was created is

17

Secured Transactions Analysis

not a problem Even though the security agreement had not yet been signed the business had authorized the filing of the financing statement in an authenticated record UCC sect 9-509(a)(1) Moreover the financing statement may be filed before the security agreement is created UCC sect 9-502(d)

Point One(b) (10) The finance company also has a perfected security interest in the businessrsquos equipment

The finance companyrsquos security interest is enforceable and attached for the same reasons as the bankrsquos security interest The loan from the finance company to the business constitutes value the business has rights in the collateral and the business owner has authenticated a security agreement containing a description of the collateral The finance companyrsquos security interest is perfected because the finance company filed a financing statement with respect to it that provides that the business is the debtor and indicates that the collateral is equipment

Point One(c) (30) The bankrsquos security interest has priority over the finance companyrsquos security interest because the bankrsquos financing statement was filed first

As between two perfected security interests the general rule is that the security interest that was the earlier to be either perfected or the subject of a filed financing statement has priority UCC sect 9-322(a)(1) While the finance companyrsquos security interest was perfected before the bankrsquos (March 15 vs March 22) the bankrsquos financing statement was filed even earlier on March 2 Thus under the first-to-file-or-perfect rule of UCC sect 9-322(a)(1) the bankrsquos security interest has priority No exceptions to the general rule apply here

Point Two (35) A security interest in collateral continues notwithstanding its sale unless an exception applies Because the security interests of the bank and the finance company were perfected and the competitor was not a buyer in ordinary course of business no exception applies and the security interests of both creditors continue in the equipment sold to the competitor

As a general rule a security interest in collateral continues notwithstanding the fact that the debtor has sold the collateral to another person UCC sect 9-315(a)(1) Thus unless an exception applies the security interests of the bank and the finance company will continue in the item of equipment sold to the competitor

A buyer of goods will take free of an unperfected security interest in those goods See UCC sect 9-317(a)(2) However when the competitor bought the businessrsquos equipment both the bank and the finance company had perfected security interests in the equipment

A buyer can take free even of a perfected security interest in goods if the buyer is a ldquobuyer in ordinary course of businessrdquo See UCC sect 9-320(a) However the competitor was not a buyer in ordinary course of business To be a ldquobuyer in ordinary course of businessrdquo a buyer must buy goods from a seller that is in the business of selling goods of that kind See UCC sect 1-201(b)(9) The competitor bought this equipment from a seller that is not in the business of selling goods of this kind so the competitor was not a buyer in ordinary course of business with respect to these goods

Because no exception applies the security interests of the bank and the finance company continue even after the item of equipment was sold to the competitor

18

FEDERAL CIVIL PROCEDURE ANALYSIS (Federal Civil Procedure IVD)

ANALYSIS

Legal Problems

(1) Is a document prepared in the course of a contract dispute protected from discovery as ldquowork productrdquo when there is no evidence that the document was prepared in anticipation of litigation

(2)(a) Is a partyrsquos failure to provide relevant electronically stored information excused when the information was destroyed pursuant to a routine document retention scheme at a time when litigation was contemplated by the destroying party

(2)(b) What sanctions should be imposed on a party for allowing the destruction of evidence that is relevant to potential future litigation

DISCUSSION

Summary

The report prepared by the structural engineer is probably not work product and is thus discoverable The engineer examined the foundation of the house at the customerrsquos request and the engineerrsquos findings are potentially relevant to the customerrsquos claim that the foundation is defective The report was not prepared in anticipation of litigation The customer appears to have sought the engineerrsquos opinion in response to the builderrsquos offer to fix any problems with the foundation that an engineer might identify Because the report was not prepared in anticipation of litigation it is not protected by the work-product doctrine

The builder should have taken appropriate steps to preserve evidence including suspending its document retention program as soon as it began planning for litigationmdashie on July 10 Its destruction of potentially relevant material after that date was wrongful However a court is unlikely to impose severe sanctions on the builder because there are no facts indicating that the builder acted in bad faith and the customer can prove that the foundation is defective without the destroyed emails

Point One (40) The customer must turn over the engineerrsquos report because it was not prepared in anticipation of litigation

In general a party to a lawsuit in federal court ldquomay obtain discovery regarding any nonprivileged matter that is relevant to any partyrsquos claim or defenserdquo FED R CIV P 26(b)(1) (2009) This includes the right to inspect and copy documents in the other partyrsquos possession FED R CIV P 34(a)(1) Here the customer hired a structural engineer to examine the foundation of the house The engineerrsquos report on the foundation is likely to include information that would be relevant to the customerrsquos claim that the foundation was defectively constructed

The so-called ldquowork productrdquo rule allows a party to refuse to turn over ldquodocuments that are prepared in anticipation of litigation or for trialrdquo by that partyrsquos representative including

19

Federal Civil Procedure Analysis

a consultant Thus if the customer had hired the structural engineer to prepare a report ldquoin anticipation of litigationrdquo that report might not be discoverable See FED R CIV P 26(b)(3)

In this case however the customer hired the engineer to evaluate the foundation of the house as part of the customerrsquos negotiation with the builder concerning the housersquos flooding problem The builder told the customer that the housersquos landscaping was the reason for the flooding and the builder told the customer ldquoHave an engineer look at the foundation If therersquos a problem wersquoll fix itrdquo The customer appears to have acted in response to that statement There is no indication that the customer anticipated any kind of legal action at the time that the structural engineer was hired Accordingly the structural engineerrsquos report is discoverable and the court should order the customer to turn it over

[NOTE If an examinee concludes that the structural engineerrsquos report was prepared in anticipation of litigation then the examinee should also conclude that the report is not discoverable Documents prepared in anticipation of litigation do not need to be disclosed to an adverse party unless that party can demonstrate a ldquosubstantial needrdquo for the documents and an inability to obtain substantially equivalent information without ldquoundue hardshiprdquo FED R CIV P 26(b)(3)(A)(ii) Furthermore a report prepared by an expert who is not expected to testify is not discoverable in the absence of ldquoexceptional circumstancesrdquo making it ldquoimpracticablerdquo to obtain the information in another way FED R CIV P 26(b)(4)(D)(ii) The builder probably cannot make these showings here unless the engineerrsquos report deals with circumstances that have since changed There is no evidence that the structural engineer would have had access to any information or facts that the builder would not already know as a result of its construction and subsequent inspection of the house In addition if necessary the builder could ask the court for permission to arrange for a further inspection of the house by a structural engineer hired by the builder See FED R CIV P 34(a)(2) Accordingly if an examinee concludes that the report was prepared in anticipation of litigation the examinee should also conclude that the builder is not entitled to see the report]

Point Two(a) (30) Because the builder anticipated that it might be involved in litigation concerning its contract with the customer the builder acted wrongfully in destroying emails that were relevant to the housersquos construction even though the emails were destroyed pursuant to a routine document retention plan

As noted above a party to a lawsuit in federal court ldquomay obtain discovery regarding any nonprivileged matter that is relevant to any partyrsquos claim or defenserdquo FED R CIV P 26(b)(1) This includes emails and other electronically stored information FED R CIV P 34(a)(1)(A) Here the customer has requested all the builderrsquos emails pertaining to work done on the foundation of the house Ordinarily the builder would be obliged to turn over this information which is relevant to the customerrsquos defense that the housersquos foundation was poorly constructed

Unfortunately the emails in question no longer exist because the builder destroyed them on August 2

In general spoliation of evidence (destruction or alteration of evidence) is improper if the party who destroyed or altered the evidence ldquohas notice that the evidence is relevant to litigation or should have known that the evidence may be relevant to future litigationrdquo Fujitsu Ltd v Federal Express Corp 247 F3d 423 436 (2d Cir 2001) It is improper for a party to destroy electronic information relevant to pending litigation even if the destruction occurs before there is any request or order seeking the information See eg Leon v IDX Sys Corp 464 F3d 951 (9th Cir 2006) (plaintiffrsquos intentional destruction of computer files warranted dismissal even

20

In this case the builderrsquos destruction of the emails was pursuant to a routine document retention plan The Federal Rules provide expressly that in the absence of ldquoexceptional circumstancesrdquo parties should not be sanctioned for the loss of electronically stored information when the loss occurs pursuant to ldquoroutine good-faith operation of an electronic information systemrdquo FED R CIV P 37(e) However when a party anticipates litigation ldquoit must suspend its routine document retentiondestruction policy and put in place a lsquolitigation holdrsquo to ensure the preservation of relevant documentsrdquo Zubulake v UBS Warburg LLC 220 FRD 212 218 (SDNY 2003)

Federal Civil Procedure Analysis

though spoliation occurred before order compelling discovery) Similarly the duty to preserve evidence applies to a party who anticipates litigation even if litigation has not yet been commenced See THE SEDONA PRINCIPLES BEST PRACTICES RECOMMENDATIONS amp PRINCIPLES FOR ADDRESSING ELECTRONIC DOCUMENT PRODUCTION 70 cmt 14a (2d ed 2007)

The builder destroyed the emails on August 2 At that time the builder knew that litigation was a possibility because the builder had already directed its attorney to prepare a draft complaint for possible filing Knowing that litigation was a possibility the builder had a duty to take steps to preserve evidence including the emails in question See generally Fujitsu Ltd

Thus the builderrsquos destruction of potentially relevant emails at a time when it knew that litigation was a possibility was improper It had a duty to preserve evidence and it breached that duty

[NOTE Because courts have used different words to describe the test for when evidence must be preserved an examineersquos precise formulation of the test is not critical]

Point Two(b) (30) In determining appropriate sanctions for spoliation courts consider both the level of culpability of the spoliating party and the degree of prejudice the loss of evidence has caused the other party Here the builderrsquos destruction of evidence does not appear to have been willful nor is it likely to pose a significant obstacle to the customerrsquos defense Any sanctions imposed by the court should be modest

Federal courts have inherent power to control the litigation process and can sanction misbehavior including spoliation even when there has been no specific violation of the Federal Rules of Civil Procedure See generally Chambers v NASCO Inc 501 US 32 (1991) (discussing courtrsquos inherent power to control the litigation process) The range of available sanctions is broad It can include such sanctions as the payment of expenses incurred by the other party as a result of the destruction of the evidence an instruction to the jury authorizing it to draw an adverse inference from the destruction of the evidence a shifting of the burden of proof on the relevant issue or even judgment against the responsible party See eg Residential Funding Corp v DeGeorge Financial Corp 306 F3d 99 108 (2d Cir 2002) (adverse inference) Silvestri v General Motors Corp 271 F3d 583 593 (4th Cir 2001) (possibility of dismissal) Cf FED R CIV P 37(b)(2)(A) (listing remedies for failure to comply with discovery obligations)

In determining appropriate sanctions for spoliation courts consider both the level of culpability of the spoliating party and the degree of prejudice the loss of evidence has caused the other party Many courts impose severe sanctions (such as an adverse-inference instruction or the entry of judgment against the spoliating party) only when there is evidence of bad faith in the form of an intentional effort to hide information Eg Greyhound Lines Inc v Wade 485 F3d 1032 1035 (8th Cir 2007) (spoliation sanction requires intentional destruction out of desire ldquoto suppress the truthrdquo) However other courts have said that negligence in preserving evidence can

21

Federal Civil Procedure Analysis

support an adverse-inference instruction See Residential Funding 306 F3d at 108 (negligence enough under some circumstances)

Although a court might well order an evidentiary hearing on the issue of sanctions the facts presented do not seem appropriate for severe sanctions First the evidence was destroyed pursuant to the builderrsquos standard document retention plan and there is no evidence that the builder deliberately failed to suspend its usual procedures with the purpose of allowing the destruction of evidence Second the loss of this evidence will not severely hinder the customerrsquos presentation of his case The central issue is whether the foundation of the house was properly constructed If the construction job was poorly done the customer can present evidence derived from inspection of the premises to prove that point The customer can also depose witnesses about any issues that arose during construction

Under the circumstances a court is not likely to impose particularly severe sanctions although it might shift the burden to the builder to show that the foundation was properly constructed or it might require the builder to reimburse any expenses the customer incurs to discover and prove the facts about issues or disputes that arose during construction of the foundation

[NOTE The result reached by the examinee is less important than the examineersquos recognition that (a) a range of sanctions is available to the court and (b) the appropriate sanction depends both on the culpability of the builder and the prejudice suffered by the customer]

22

CRIMINAL LAW AND PROCEDURE ANALYSIS (Criminal Law and Procedure IIA amp D VE amp F)

ANALYSIS

Legal Problems

(1) Did charging the defendant with both theft and burglary constitute double jeopardy

(2) Did the jury instruction violate the due process clause either by relieving the prosecution of the burden of proving the element of intent or by shifting the burden to the defendant to disprove that element

(3) Did the sentence imposed in this case for the theft conviction unconstitutionally deprive the defendant of his right to a jury trial on the issue of the value of the stolen item

DISCUSSION

Summary

The trial court properly denied the defendantrsquos pretrial motion to dismiss the charges on double jeopardy grounds The defendant may be charged with and convicted of both theft and burglary Each of the charges has an element that the other does not Neither charge is a lesser-included offense nor are they multiplicitous Thus charging both theft and burglary does not violate double jeopardy

The jury instruction on the burglary charge was constitutionally flawed It could have been reasonably understood by the jury as either (1) an irrebuttable conclusive presumption (which relieved the prosecution of proving the element of intent and removed the issue from the jury) or (2) a rebuttable mandatory presumption (which unconstitutionally shifted the burden of proof on an element of a charged offense from the prosecution to the defendant)

Because the four-year sentence imposed by the judge was based on the judgersquos finding by a preponderance of the evidence that the value of the stolen ring exceeded $5000 the sentence violates the defendantrsquos right to a jury determination beyond a reasonable doubt of the value of the ring

Point One (30) Charging the defendant with theft and burglary did not constitute double jeopardy

The Double Jeopardy Clause of the Fifth Amendment provides that a person shall not be twice put in jeopardy for the ldquosame offenserdquo Thus the question is whether the elements of the theft charge are wholly contained in the burglary charge or vice versa If the elements of the lesser charge (theft) are not wholly contained in the greater charge (burglary)mdashie if each charge requires proof of a fact that the other does notmdashthen convicting the defendant of both crimes would not violate double jeopardy even when the two offenses occurred at the same time and are thus arguably part of the ldquosame transactionrdquo Blockburger v United States 284 US 299 304 (1932) See also Albernaz v United States 450 US 333 344 n3 (1981) United States v Dixon 509 US 688 704 (1993)

23

Criminal Law and Procedure Analysis

Here theft and burglary each require proof of an element not required for the other crime Burglary may be defined differently in different jurisdictions However it almost invariably requires entry into a building or dwelling of another with the specific intent to commit a felony therein and the crime of burglary is complete upon the entry into the building or dwelling with such intent See eg Cannon v Oklahoma 827 P2d 1339 1342 (Okla Crim App 1992) In contrast theft which also may be defined differently in different states almost invariably requires the taking and carrying away of an item of personal property belonging to another with the intent to steal or permanently deprive the owner of possession

Here the ldquotakingrdquo or ldquostealingrdquo element is not contained in the definition of burglary and the ldquoentryrdquo element of burglary is not contained in the definition of theft Because theft is not a lesser-included offense of burglary and burglary is not a lesser-included offense of theft charging the defendant for both burglary and theft did not violate double jeopardy and the court properly denied the defense motion on those grounds Yparrea v Dorsey 64 F3d 577 579ndash80 (10th Cir 1995) citing Blockburger 284 US at 304

Finally the defendantrsquos motion to dismiss all the charges on double jeopardy grounds was improper because if both charges were for the same offense the motion should have requested dismissal of one charge not both

Point Two (35) The jury instruction on the burglary charge violated the Due Process Clause because it created either (1) an irrebuttable conclusive presumption (which relieved the prosecution of proving the element of intent and removed that issue from the jury) or (2) a rebuttable mandatory presumption (which unconstitutionally shifted the burden of proof on an element of a charged offense to the defendant)

The Supreme Court has interpreted the Due Process Clause of the US Constitution to require that the prosecution prove all elements of an offense beyond a reasonable doubt See In re Winship 397 US 358 364 (1970) The burden of proof cannot be shifted to the defendant by presuming an essential element upon proof of other elements of the offense because shifting the burden of persuasion with respect to any element of a criminal offense is contrary to the Due Process Clause See Mullaney v Wilbur 421 US 684 (1975)

The crime of burglary includes entry into a building or dwelling with the specific intent to commit a felony therein The requirement that the prosecutor prove beyond a reasonable doubt that the defendant had this specific intent distinguishes burglary from general-intent crimes like trespass See Sandstrom v Montana 442 US 510 523 (1979)

Here the jury was instructed that if ldquoafter consideration of all the evidence presented by the prosecution and defense you find beyond a reasonable doubt that the defendant entered the dwelling without the ownersrsquo consent you may presume that the defendant entered with the intent to commit a felony thereinrdquo This instruction was unconstitutional because it created either an irrebuttable conclusive presumption or a rebuttable mandatory presumption

A conclusive presumption is ldquoan irrebuttable direction by the court to find intent once convinced of the facts triggering the presumptionrdquo Id at 517 Here the jurors were instructed that once the prosecutor established that the defendant entered the neighborsrsquo house without consent they ldquomay presumerdquo that he intended to commit a felony therein The jurors may have reasonably concluded from this instruction that if they found that the defendant intended to enter his neighborsrsquo home without permission they must further find that he entered with the specific intent to commit a felony therein Because this instruction could operate as a conclusive

24

Criminal Law and Procedure Analysis

irrebuttable presumption by eliminating intent ldquoas an ingredient of the offenserdquo it violated due process by relieving the prosecution of the burden of proof for this element Id at 522

In the alternative the jury instruction could have been reasonably understood to create a rebuttable mandatory presumption which ldquotells [the jury] they must find the elemental fact upon proof of the basic fact at least unless the defendant has come forward with some evidence to rebut the presumed connection between the two factsrdquo County Court of Ulster County New York v Allen 442 US 140 157 (1979) The due process problem created by rebuttable mandatory presumptions is that ldquo[t]o the extent that the trier of fact is forced to abide by the presumption and may not reject it based on an independent evaluation of the particular facts presented by the State the analysis of the presumptionrsquos constitutional validity is logically divorced from those facts and based on the presumptionrsquos accuracy in the run of casesrdquo Id at 159

Unlike irrebuttable conclusive presumptions rebuttable mandatory presumptions are not always per se violations of the Due Process Clause However the Supreme Court of the United States has held that jury instructions that could reasonably be understood as shifting the burden of proof to the defendant on an element of the offense are unconstitutional Francis v Franklin 471 US 307 (1985) Here the argument that the jury instruction operated as a rebuttable mandatory presumption is supported by the fact that the judge also instructed the jury to ldquoconsider[ ] all the evidence presented by the prosecution and defenserdquo However even if the instruction created a rebuttable mandatory presumption it would be unconstitutional because it shifted the burden to the defense on an element of the offense Sandstrom 442 US at 524 Mullaney 421 US at 686

[NOTE Whether an examinee identifies the jury instruction as containing a ldquoconclusiverdquo or ldquomandatoryrdquo presumption is less important than the examineersquos analysis of the constitutional infirmities]

Point Three (35) The trial court violated the defendantrsquos Sixth Amendment right to a jury trial on an essential element of the offense when it found by a preponderance of the evidence that the ring was worth over $5000 and increased the defendantrsquos sentence based on this finding

In the statutory scheme under which the defendant was tried and convicted a Class D felony theft is defined as theft of item(s) with a value between $2500 and $10000 The jury found that the value of the diamond ring was at least $2500 and convicted the defendant of felony theft However at sentencing the trial court made a separate finding by a preponderance of the evidence that the value of the ring was greater than $5000 Following the statutersquos two-tiered sentencing scheme the judge then imposed on the defendant a sentence that was one year longer than the maximum that would otherwise have been allowed

The judgersquos sentence was unconstitutional because it violated the defendantrsquos Sixth Amendment right to a jury trial on this question The Supreme Court held in Apprendi v New Jersey 530 US 466 (2000) that ldquo[o]ther than the fact of a prior conviction any fact that increases the penalty for a crime beyond the prescribed statutory maximum must be submitted to a jury and proved beyond a reasonable doubtrdquo because ldquo[i]t is unconstitutional for a legislature to remove from the jury the assessment of facts that increase the prescribed range of penalties to which a criminal defendant is exposed [because] such facts must be established by proof beyond a reasonable doubtrdquo Id The Court reaffirmed Apprendi in Blakely v Washington 542 US 296 (2004) holding that the ldquolsquostatutory maximumrsquo for Apprendi purposes is the maximum sentence a judge may impose solely on the basis of the facts reflected in the jury verdict or admitted by the defendantrdquo Id at 303 (emphasis in original) In United States v Booker 543 US 220 (2005)

25

Criminal Law and Procedure Analysis

the Court relied on Blakely and Apprendi to conclude that protecting a defendantrsquos Sixth Amendment right to a jury trial required that ldquo[a]ny fact which is necessary to support a sentence exceeding the maximum authorized by the facts established by a plea of guilty or a jury verdict must be admitted by the defendant or proved to a jury beyond a reasonable doubtrdquo Id at 244

Thus in order to constitutionally increase a sentence above the statutory maximum of three years the jury must have found beyond a reasonable doubt that the value of the ring exceeded $5000 Here the court made the finding based on an appraisal proffered by the prosecutor only at sentencing and the judgersquos finding was by a preponderance of the evidence rather than beyond a reasonable doubt

26

AGENCY AND PARTNERSHIP ANALYSIS __________ (Agency and Partnership VA amp C VI)

ANALYSIS

Legal Problems

(1) Is a partner in a general partnership personally liable on a claim arising from misrepresentations by another partner made in the course of the partnership business

(2) Does a newly admitted partner in a general partnership become personally liable on existing claims against the partnership

(3) After the filing by a general partnership of a statement of qualification as a limited liability partnership are the partners personally liable as partners on (a) an existing claim against the general partnership and (b) a claim against the partnership that arose after the filing

DISCUSSION

Summary

Adam and Ben formed a general partnership under which they were jointly and severally liable for obligations of the partnership Thus Adam was personally liable for misrepresentations by Ben made in the ordinary course of the partnership business

Upon joining the general partnership Diane became personally liable for the obligations of the partnership arising after her admission but not for obligations pre-existing her admission such as the collectorrsquos claim

By filing a statement of qualification the three partners properly elected limited liability partnership status As partners in an LLP none of the three partners is personally liable as a partner for partnership obligations arising after the election such as the claim by the driverrsquos estate The election however does not change their personal liability on pre-existing claims that arose before the election such as the collectorrsquos claim

Point One (30) As a general partner of Empire a general partnership Adam became personally liable on the collectorrsquos claim a valid claim against the partnership that arose because of Benrsquos wrongful act in the ordinary course of the partnership business

When the collectorrsquos claim arose Empire was a general partnership composed of Adam and Ben Under UPA (1997) sect 306(a) partners of a general partnership are liable jointly and severally for all obligations of the partnership Under UPA (1997) sect 305(a) the partnership could become obligated for the loss caused to the collector as a result of the misrepresentation by Ben provided he was acting in the ordinary course of the partnership business Because there was no statement that limited his partnership authority Ben as partner was ldquoan agent of the partnership for the purpose of its businessrdquo See UPA (1997) sect 301(1) Benrsquos misrepresentation to the collector even if intentional appears to be in the ordinary course of the partnershiprsquos business of dealing

27

Agency and Partnership Analysis

in antique cars Thus Benrsquos wrongful act created a partnership obligation for which Adam was jointly and severally liable

[NOTE Generally a partnership creditor must ldquoexhaust the partnershiprsquos assets before levying on a judgment debtor partnerrsquos individual property where the partner is personally liable for the partnership obligationrdquo as a result of his status as a partner UPA (1997) sect 307 cmt 4 As the UPA comments explain this places Adam more in the position of guarantor than principal debtor on the partnership obligation Id cmt 4 Although an examinee might discuss this point the call focuses on whether Adam is personally liable not how the liability might be enforced]

Point Two (30) Because the collectorrsquos claim arose before Diane joined Empire Diane did not become personally liable on the claim

Diane was admitted to Empire when it was a general partnership and after the collectorrsquos claim arose While the general rule under UPA (1997) sect 306(a) is that the partners of a general partnership are liable jointly and severally for all obligations of the partnership there is a special rule for partners who are admitted during the duration of the partnership Under UPA (1997) sect 306(b) a person admitted to an existing partnership is not personally liable for any partnership obligations incurred before the personrsquos admission Because Diane was admitted to Empire after the collectorrsquos claim arose Diane is not personally liable on the claim

Dianersquos knowledge of the pre-existing claim and her stated concern about becoming liable on the collectorrsquos claim do not change her personal nonliability to the collector Although partners who have a liability shield can assume liability to third parties through private contractual guarantees or modifications to the partnership agreement Dianersquos stated concern constituted neither a guaranty to the collector nor ldquoan intentional waiver of liability protectionsrdquo See UPA (1997) sect 306 cmt 3 (describing methods for waiver of liability protections under sect 306(c) applicable in limited liability partnerships)

At most Diane will lose her investment in the partnership as a result of the collectorrsquos claim Although Diane did not become personally liable on the collectorrsquos claim when she joined the partnership the $250000 she contributed to the partnership is ldquoat risk for the satisfaction of existing partnership debtsrdquo UPA (1997) sect 306 cmt 2

Point Three (40) Filing the statement of qualification was effective to elect limited liability partnership status Despite this new status Adam and Ben remain personally liable on the collectorrsquos claim which arose before the election But as partners in an LLP neither Adam Ben nor Diane is personally liable as a partner on the driverrsquos estatersquos claim which arose after the election

Under UPA (1997) sect 1001 a general partnership can make an election and become a limited liability partnershipmdashif the partners approve the conversion by a vote equivalent to that necessary to amend the partnership agreement and the partnership then files a statement of qualification that specifies the name of the partnership its principal office and its election to be an LLP Here the partners agreed unanimouslymdashsufficient to amend their agreement under UPA (1997) sect 401(j)mdashand the statement of qualification was filed In addition the name of Empire LLP properly included an appropriate ending ldquoLLPrdquo See UPA (1997) sect 1002

Although another way to effectuate a ldquoconversionrdquo (as suggested by Benrsquos lawyer) is to form a new LLP and transfer the assets of the old general partnership to the new LLP the

28

Agency and Partnership Analysis

method used here (approval by the partners and the filing of a statement of qualification) is also sufficient to create LLP status

Thus Empire became Empire LLP as of the date of filing of the statement of qualification See UPA (1997) sect 1001 What effect did this have on the collectorrsquos claim which predated the filing According to UPA (1997) sect 306(c) an obligation incurred while a partnership is an LLP is solely a partnership obligation As the collectorrsquos claim predated the LLP Adam and Ben remain personally liable on the collectorrsquos claim Diane on the other hand was not personally liable on the collectorrsquos claim either before or after the filing of the statement of qualification See Point Two above

The driverrsquos estatersquos claim arose after Empire became Empire LLP Under UPA (1997) sect 306(c) an obligation incurred while a partnership is an LLP is solely a partnership obligationThus Adam Ben and Diane as partners are all protected from personal liability on the driverrsquos estatersquos claim But there may be personal liability if any of them was negligent or otherwise acted wrongfully by not informing the buyer of the bad suspension that caused the accident

29

National Conference of Bar Examiners 302 South Bedford Street | Madison WI 53703-3622 Phone 608-280-8550 | Fax 608-280-8552 | TDD 608-661-1275

wwwncbexorg e-mail contactncbexorg

  • Preface
  • Description of the MEE
  • Instructions
  • February 2014 Questions
    • Constitutinal Law Question
    • Trusts and Future Interests Question
    • Secured Transactions Question
    • Federal Civil Procedure Question
    • Criminal Law and Procedure Question
    • Agency and Partnership Question
      • February 2014 Analyses
        • Constitutional Law Analysis
        • Trust and Future Interests Analysis
        • Secured Transactions Analysis
        • Federal Civil Procedure Analysis
        • Criminal Law and Procedure Analysis
        • Agency and Partnership Analysis
            • ltlt13 ASCII85EncodePages false13 AllowTransparency false13 AutoPositionEPSFiles true13 AutoRotatePages None13 Binding Left13 CalGrayProfile (Dot Gain 20)13 CalRGBProfile (sRGB IEC61966-21)13 CalCMYKProfile (US Web Coated 050SWOP051 v2)13 sRGBProfile (sRGB IEC61966-21)13 CannotEmbedFontPolicy Error13 CompatibilityLevel 1413 CompressObjects Tags13 CompressPages true13 ConvertImagesToIndexed true13 PassThroughJPEGImages true13 CreateJobTicket false13 DefaultRenderingIntent Default13 DetectBlends true13 DetectCurves 0000013 ColorConversionStrategy CMYK13 DoThumbnails false13 EmbedAllFonts true13 EmbedOpenType false13 ParseICCProfilesInComments true13 EmbedJobOptions true13 DSCReportingLevel 013 EmitDSCWarnings false13 EndPage -113 ImageMemory 104857613 LockDistillerParams false13 MaxSubsetPct 10013 Optimize true13 OPM 113 ParseDSCComments true13 ParseDSCCommentsForDocInfo true13 PreserveCopyPage true13 PreserveDICMYKValues true13 PreserveEPSInfo true13 PreserveFlatness true13 PreserveHalftoneInfo false13 PreserveOPIComments true13 PreserveOverprintSettings true13 StartPage 113 SubsetFonts true13 TransferFunctionInfo Apply13 UCRandBGInfo Preserve13 UsePrologue false13 ColorSettingsFile ()13 AlwaysEmbed [ true13 ]13 NeverEmbed [ true13 ]13 AntiAliasColorImages false13 CropColorImages true13 ColorImageMinResolution 30013 ColorImageMinResolutionPolicy OK13 DownsampleColorImages true13 ColorImageDownsampleType Bicubic13 ColorImageResolution 30013 ColorImageDepth -113 ColorImageMinDownsampleDepth 113 ColorImageDownsampleThreshold 15000013 EncodeColorImages true13 ColorImageFilter DCTEncode13 AutoFilterColorImages true13 ColorImageAutoFilterStrategy JPEG13 ColorACSImageDict ltlt13 QFactor 01513 HSamples [1 1 1 1] VSamples [1 1 1 1]13 gtgt13 ColorImageDict ltlt13 QFactor 01513 HSamples [1 1 1 1] VSamples [1 1 1 1]13 gtgt13 JPEG2000ColorACSImageDict ltlt13 TileWidth 25613 TileHeight 25613 Quality 3013 gtgt13 JPEG2000ColorImageDict ltlt13 TileWidth 25613 TileHeight 25613 Quality 3013 gtgt13 AntiAliasGrayImages false13 CropGrayImages true13 GrayImageMinResolution 30013 GrayImageMinResolutionPolicy OK13 DownsampleGrayImages true13 GrayImageDownsampleType Bicubic13 GrayImageResolution 30013 GrayImageDepth -113 GrayImageMinDownsampleDepth 213 GrayImageDownsampleThreshold 15000013 EncodeGrayImages true13 GrayImageFilter DCTEncode13 AutoFilterGrayImages true13 GrayImageAutoFilterStrategy JPEG13 GrayACSImageDict ltlt13 QFactor 01513 HSamples [1 1 1 1] VSamples [1 1 1 1]13 gtgt13 GrayImageDict ltlt13 QFactor 01513 HSamples [1 1 1 1] VSamples [1 1 1 1]13 gtgt13 JPEG2000GrayACSImageDict ltlt13 TileWidth 25613 TileHeight 25613 Quality 3013 gtgt13 JPEG2000GrayImageDict ltlt13 TileWidth 25613 TileHeight 25613 Quality 3013 gtgt13 AntiAliasMonoImages false13 CropMonoImages true13 MonoImageMinResolution 120013 MonoImageMinResolutionPolicy OK13 DownsampleMonoImages true13 MonoImageDownsampleType Bicubic13 MonoImageResolution 120013 MonoImageDepth -113 MonoImageDownsampleThreshold 15000013 EncodeMonoImages true13 MonoImageFilter CCITTFaxEncode13 MonoImageDict ltlt13 K -113 gtgt13 AllowPSXObjects false13 CheckCompliance [13 None13 ]13 PDFX1aCheck false13 PDFX3Check false13 PDFXCompliantPDFOnly false13 PDFXNoTrimBoxError true13 PDFXTrimBoxToMediaBoxOffset [13 00000013 00000013 00000013 00000013 ]13 PDFXSetBleedBoxToMediaBox true13 PDFXBleedBoxToTrimBoxOffset [13 00000013 00000013 00000013 00000013 ]13 PDFXOutputIntentProfile ()13 PDFXOutputConditionIdentifier ()13 PDFXOutputCondition ()13 PDFXRegistryName ()13 PDFXTrapped False1313 CreateJDFFile false13 Description ltlt13 ARA 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 BGR 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 CHS ltFEFF4f7f75288fd94e9b8bbe5b9a521b5efa7684002000410064006f006200650020005000440046002065876863900275284e8e9ad88d2891cf76845370524d53705237300260a853ef4ee54f7f75280020004100630072006f0062006100740020548c002000410064006f00620065002000520065006100640065007200200035002e003000204ee553ca66f49ad87248672c676562535f00521b5efa768400200050004400460020658768633002gt13 CHT ltFEFF4f7f752890194e9b8a2d7f6e5efa7acb7684002000410064006f006200650020005000440046002065874ef69069752865bc9ad854c18cea76845370524d5370523786557406300260a853ef4ee54f7f75280020004100630072006f0062006100740020548c002000410064006f00620065002000520065006100640065007200200035002e003000204ee553ca66f49ad87248672c4f86958b555f5df25efa7acb76840020005000440046002065874ef63002gt13 CZE 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 DAN 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 DEU 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 ESP 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 ETI 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 FRA 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 GRE ltFEFF03a703c103b703c303b903bc03bf03c003bf03b903ae03c303c403b5002003b103c503c403ad03c2002003c403b903c2002003c103c503b803bc03af03c303b503b903c2002003b303b903b1002003bd03b1002003b403b703bc03b903bf03c503c103b303ae03c303b503c403b5002003ad03b303b303c103b103c603b1002000410064006f006200650020005000440046002003c003bf03c5002003b503af03bd03b103b9002003ba03b103c42019002003b503be03bf03c703ae03bd002003ba03b103c403ac03bb03bb03b703bb03b1002003b303b903b1002003c003c103bf002d03b503ba03c403c503c003c903c403b903ba03ad03c2002003b503c103b303b103c303af03b503c2002003c503c803b703bb03ae03c2002003c003bf03b903cc03c403b703c403b103c2002e0020002003a403b10020005000440046002003ad03b303b303c103b103c603b1002003c003bf03c5002003ad03c703b503c403b5002003b403b703bc03b903bf03c503c103b303ae03c303b503b9002003bc03c003bf03c103bf03cd03bd002003bd03b1002003b103bd03bf03b903c703c403bf03cd03bd002003bc03b5002003c403bf0020004100630072006f006200610074002c002003c403bf002000410064006f00620065002000520065006100640065007200200035002e0030002003ba03b103b9002003bc03b503c403b103b303b503bd03ad03c303c403b503c103b503c2002003b503ba03b403cc03c303b503b903c2002egt13 HEB 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 HRV (Za stvaranje Adobe PDF dokumenata najpogodnijih za visokokvalitetni ispis prije tiskanja koristite ove postavke Stvoreni PDF dokumenti mogu se otvoriti Acrobat i Adobe Reader 50 i kasnijim verzijama)13 HUN ltFEFF004b0069007600e1006c00f30020006d0069006e0151007300e9006701710020006e0079006f006d00640061006900200065006c0151006b00e90073007a00ed007401510020006e0079006f006d00740061007400e100730068006f007a0020006c006500670069006e006b00e1006200620020006d0065006700660065006c0065006c0151002000410064006f00620065002000500044004600200064006f006b0075006d0065006e00740075006d006f006b0061007400200065007a0065006b006b0065006c0020006100200062006500e1006c006c00ed007400e10073006f006b006b0061006c0020006b00e90073007a00ed0074006800650074002e0020002000410020006c00e90074007200650068006f007a006f00740074002000500044004600200064006f006b0075006d0065006e00740075006d006f006b00200061007a0020004100630072006f006200610074002000e9007300200061007a002000410064006f00620065002000520065006100640065007200200035002e0030002c0020007600610067007900200061007a002000610074007400f3006c0020006b00e9007301510062006200690020007600650072007a006900f3006b006b0061006c0020006e00790069007400680061007400f3006b0020006d00650067002egt13 ITA 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 JPN ltFEFF9ad854c18cea306a30d730ea30d730ec30b951fa529b7528002000410064006f0062006500200050004400460020658766f8306e4f5c6210306b4f7f75283057307e305930023053306e8a2d5b9a30674f5c62103055308c305f0020005000440046002030d530a130a430eb306f3001004100630072006f0062006100740020304a30883073002000410064006f00620065002000520065006100640065007200200035002e003000204ee5964d3067958b304f30533068304c3067304d307e305930023053306e8a2d5b9a306b306f30d530a930f330c8306e57cb30818fbc307f304c5fc59808306730593002gt13 KOR ltFEFFc7740020c124c815c7440020c0acc6a9d558c5ec0020ace0d488c9c80020c2dcd5d80020c778c1c4c5d00020ac00c7a50020c801d569d55c002000410064006f0062006500200050004400460020bb38c11cb97c0020c791c131d569b2c8b2e4002e0020c774b807ac8c0020c791c131b41c00200050004400460020bb38c11cb2940020004100630072006f0062006100740020bc0f002000410064006f00620065002000520065006100640065007200200035002e00300020c774c0c1c5d0c11c0020c5f40020c2180020c788c2b5b2c8b2e4002egt13 LTH 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 LVI 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 NLD (Gebruik deze instellingen om Adobe PDF-documenten te maken die zijn geoptimaliseerd voor prepress-afdrukken van hoge kwaliteit De gemaakte PDF-documenten kunnen worden geopend met Acrobat en Adobe Reader 50 en hoger)13 NOR 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 POL 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 PTB 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 RUM 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 RUS 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 SKY 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 SLV ltFEFF005400650020006e006100730074006100760069007400760065002000750070006f0072006100620069007400650020007a00610020007500730074007600610072006a0061006e006a006500200064006f006b0075006d0065006e0074006f0076002000410064006f006200650020005000440046002c0020006b006900200073006f0020006e0061006a007000720069006d00650072006e0065006a016100690020007a00610020006b0061006b006f0076006f00730074006e006f0020007400690073006b0061006e006a00650020007300200070007200690070007200610076006f0020006e00610020007400690073006b002e00200020005500730074007600610072006a0065006e006500200064006f006b0075006d0065006e0074006500200050004400460020006a00650020006d006f0067006f010d00650020006f0064007000720065007400690020007a0020004100630072006f00620061007400200069006e002000410064006f00620065002000520065006100640065007200200035002e003000200069006e0020006e006f00760065006a01610069006d002egt13 SUO 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 SVE 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 TUR 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 UKR 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 ENU (Use these settings to create Adobe PDF documents best suited for high-quality prepress printing Created PDF documents can be opened with Acrobat and Adobe Reader 50 and later)13 gtgt13 Namespace [13 (Adobe)13 (Common)13 (10)13 ]13 OtherNamespaces [13 ltlt13 AsReaderSpreads false13 CropImagesToFrames true13 ErrorControl WarnAndContinue13 FlattenerIgnoreSpreadOverrides false13 IncludeGuidesGrids false13 IncludeNonPrinting false13 IncludeSlug false13 Namespace [13 (Adobe)13 (InDesign)13 (40)13 ]13 OmitPlacedBitmaps false13 OmitPlacedEPS false13 OmitPlacedPDF false13 SimulateOverprint Legacy13 gtgt13 ltlt13 AddBleedMarks false13 AddColorBars false13 AddCropMarks false13 AddPageInfo false13 AddRegMarks false13 ConvertColors ConvertToCMYK13 DestinationProfileName ()13 DestinationProfileSelector DocumentCMYK13 Downsample16BitImages true13 FlattenerPreset ltlt13 PresetSelector MediumResolution13 gtgt13 FormElements false13 GenerateStructure false13 IncludeBookmarks false13 IncludeHyperlinks false13 IncludeInteractive false13 IncludeLayers false13 IncludeProfiles false13 MultimediaHandling UseObjectSettings13 Namespace [13 (Adobe)13 (CreativeSuite)13 (20)13 ]13 PDFXOutputIntentProfileSelector DocumentCMYK13 PreserveEditing true13 UntaggedCMYKHandling LeaveUntagged13 UntaggedRGBHandling UseDocumentProfile13 UseDocumentBleed false13 gtgt13 ]13gtgt setdistillerparams13ltlt13 HWResolution [2400 2400]13 PageSize [612000 792000]13gtgt setpagedevice13

Page 2: February 2014 MEE Questions and AnalysesPreface The Multistate Essay Examination (MEE) is developed by the National Conference of Bar Examiners (NCBE). This publication includes the

Copyright copy 2014 by the National Conference of Bar Examiners All rights reserved

Contents

Prefaceii

Description of the MEE ii

Instructionsiii

February 2014 Questions

Constitutional Law Question 3

Trusts and Future Interests Question 4

Secured Transactions Question 5

Federal Civil Procedure Question 6

Criminal Law and Procedure Question 7

Agency and Partnership Question 8

February 2014 Analyses

Constitutional Law Analysis 11

Trusts and Future Interests Analysis 14

Secured Transactions Analysis 17

Federal Civil Procedure Analysis 19

Criminal Law and Procedure Analysis 23

Agency and Partnership Analysis 27

i

Preface

The Multistate Essay Examination (MEE) is developed by the National Conference of Bar Examiners (NCBE) This publication includes the questions and analyses from the February 2014 MEE (In the actual test the questions are simply numbered rather than being identified by area of law) The instructions for the test appear on page iii

The model analyses for the MEE are illustrative of the discussions that might appear in excellent answers to the questions They are provided to the user jurisdictions to assist graders in grading the examination They address all the legal and factual issues the drafters intended to raise in the questions

The subjects covered by each question are listed on the first page of its accompanying analysis identified by roman numerals that refer to the MEE subject matter outline for that subject For example the Federal Civil Procedure question on the February 2014 MEE tested the following area from the Federal Civil Procedure outline IVD Pretrial proceduresmdashDiscovery (including e-discovery)

For more information about the MEE including subject matter outlines visit the NBCE website at wwwncbexorg

Description of the MEE

The MEE consists of six 30-minute essay questions and is a component of the Uniform Bar Examination (UBE) It is administered by participating jurisdictions on the Tuesday before the last Wednesday in February and July of each year The areas of law that may be covered by the questions on any MEE are Business Associations (Agency and Partnership Corporations and Limited Liability Companies) Conflict of Laws Constitutional Law Contracts Criminal Law and Procedure Evidence Family Law Federal Civil Procedure Real Property Torts Trusts and Estates (Decedentsrsquo Estates Trusts and Future Interests) and Uniform Commercial Code (Negotiable Instruments and Bank Deposits and Collections Secured Transactions) Some questions may include issues in more than one area of law The particular areas covered vary from exam to exam

The purpose of the MEE is to test the examineersquos ability to (1) identify legal issues raised by a hypothetical factual situation (2) separate material which is relevant from that which is not (3) present a reasoned analysis of the relevant issues in a clear concise and well-organized composition and (4) demonstrate an understanding of the fundamental legal principles relevant to the probable solution of the issues raised by the factual situation The primary distinction between the MEE and the Multistate Bar Examination (MBE) is that the MEE requires the examinee to demonstrate an ability to communicate effectively in writing

ii

Instructions The back cover of each test booklet contains the following instructions

You will be instructed when to begin and when to stop this test Do not break the seal on this booklet until you are told to begin

You may answer the questions in any order you wish Do not answer more than one question in each answer booklet If you make a mistake or wish to revise your answer simply draw a line through the material you wish to delete

If you are using a laptop computer to answer the questions your jurisdiction will provide you with specific instructions

Read each fact situation very carefully and do not assume facts that are not given in the question Do not assume that each question covers only a single area of the law some of the questions may cover more than one of the areas you are responsible for knowing

Demonstrate your ability to reason and analyze Each of your answers should show an understanding of the facts a recognition of the issues included a knowledge of the applicable principles of law and the reasoning by which you arrive at your conclusions The value of your answer depends not as much upon your conclusions as upon the presence and quality of the elements mentioned above

Clarity and conciseness are important but make your answer complete Do not volunteer irrelevant or immaterial information

Answer all questions according to generally accepted fundamental legal principles unless your testing jurisdiction has instructed you to answer according to local case or statutory law

NOTE Examinees testing in UBE jurisdictions must answer according to generally accepted fundamental legal principles rather than local case or statutory law

iii

February 2014 MEE

QUESTIONS Constitutional Law

Trusts and Future Interests Secured Transactions

Federal Civil Procedure Criminal Law and Procedure

Agency and Partnership

CONSTITUTIONAL LAW QUESTION ___________

A city ordinance required each downtown business to install high-powered halogen floodlights that would illuminate the property owned by that business and the adjoining sidewalks A study commissioned by the city estimated that installation of the floodlights would cost a typical business about $1000 but that increased business traffic due to enhanced public safety especially after dark would likely offset this cost

A downtown restaurant applied to the city for a building permit to construct an addition that would increase its seating capacity In its permit application the restaurant accurately noted that its current facility did not have sufficient seating to accommodate all potential customers during peak hours The city approved the permit on the condition that the restaurant grant the city an easement over a narrow strip of the restaurantrsquos property to be used by the city to install video surveillance equipment that would cover nearby public streets and parking lots The city based its permit decision entirely on findings that the increased patronage that would result from the increased capacity of the restaurant might also attract additional crime to the neighborhood and that installing video surveillance equipment might alleviate that problem

The restaurant has challenged both the ordinance requiring it to install floodlights and the easement condition imposed on approval of the building permit

1 Under the Fifth Amendment as applied to the states through the Fourteenth Amendment is the city ordinance requiring the restaurant to install floodlights an unconstitutional taking Explain

2 Under the Fifth Amendment as applied to the states through the Fourteenth Amendment is the cityrsquos requirement that the restaurant grant the city an easement as a condition for obtaining the building permit an unconstitutional taking Explain

3

TRUSTS AND FUTURE INTERESTS QUESTION _______________

Ten years ago a testator died survived by his only children a son age 26 and a daughter age 18

A testamentary trust was created under the testatorrsquos duly probated will The will specified that all trust income would be paid to the son during the sonrsquos lifetime and that upon the sonrsquos death the trust would terminate and trust principal would be distributed to the testatorrsquos ldquograndchildren who shall surviverdquo the son The testator provided for his daughter in other sections of the will

Five years ago the trustee of the testamentary trust purchased an office building with $500000 from the trust principal Other than this building the trust assets consist of publicly traded securities

Last year the trustee received $30000 in rents from the office building The trustee also received with respect to the securities owned by the trust cash dividends of $20000 and a stock dividend of 400 shares of Acme Corp common stock distributed to the trust by Acme Corp

Eight months ago the trustee sold the office building for $700000

Six months ago the son delivered a letter to the trustee stating ldquoI hereby disclaim any interest I may have in the income interest of the trustrdquo On the date the son delivered this letter to the trustee the son had no living children the daughter had one living minor child

A statute in this jurisdiction provides that ldquoa disclaimer of any interest created by will is valid only if made within nine months after the testatorrsquos death and if an interest is validly disclaimed the disclaiming party is deemed to have predeceased the testatorrdquo

1 How should the rents sales proceeds cash dividends and stock dividends received prior to the trusteersquos receipt of the sonrsquos letter have been allocated between trust principal and income Explain

2 How if at all does the sonrsquos letter to the trustee affect the future distribution of trust income and principal Explain

4

SECURED TRANSACTIONS QUESTION

On March 1 the owner of a manufacturing business entered into negotiations with a bank to obtain a loan of $100000 for the business The bank loan officer informed the business owner that the interest rate for a loan would be lower if the repayment obligation were secured by all the businessrsquos present and future equipment The loan officer also informed the business owner that the bank could not commit to making the loan until its credit investigation was completed but that funds could be advanced faster following loan approval if a financing statement with respect to the transaction were filed in advance Accordingly the business owner signed a form on behalf of the business authorizing the bank to file a financing statement with respect to the proposed transaction The bank properly filed a financing statement the next day correctly providing the name of the business as the debtor and indicating ldquoequipmentrdquo as the collateral

On March 15 the business owner had heard nothing from the bank about whether the loan had been approved so the business owner approached a finance company for a loan The finance company quickly agreed to lend $100000 to the business secured by all the businessrsquos present and future equipment That same day the finance company loaned to the business $100000 and the business owner signed an agreement obligating the business to repay the loan and granting the finance company a security interest in all the businessrsquos ldquopresent and future equipmentrdquo to secure the repayment obligation Also on that day the finance company properly filed a financing statement correctly providing the businessrsquos name as the debtor and indicating ldquoequipmentrdquo as the collateral

On March 21 the bank loan officer contacted the business owner and indicated that the loan application had been approved On the next day March 22 the bank loaned the business $100000 The loan agreement signed by the owner on behalf of the business granted the bank a security interest in all the businessrsquos ldquopresent and future equipmentrdquo

On April 10 the business sold an item of manufacturing equipment to a competitor for $20000 This was the first time the business had ever sold any of its equipment The competitor paid the purchase price in cash and took possession of the equipment that day The competitor acted in good faith at all times and had no knowledge of the businessrsquos prior transactions with the bank and the finance company

The business has defaulted on its obligations with respect to the loans from the bank and the finance company Each of them has asserted a claim to all the businessrsquos equipment as well as to the item of equipment sold to the businessrsquos competitor

Assume that the business owner had the authority to enter into all these transactions on behalf of the business

1 As between the bank and the finance company which has a superior claim to the businessrsquos equipment Explain

2 Do the claims of the bank and the finance company to the businessrsquos equipment continue in the item of equipment sold to the competitor Explain

5

FEDERAL CIVIL PROCEDURE QUESTION

A builder constructed a vacation house for an out-of-state customer on the customerrsquos land The house was completed on June 1 at which point the customer still owed $200000 of the $800000 contract price which was payable in full five days later

On June 14 the basement of the house was flooded with two inches of water during a heavy rainfall When the customer complained the builder told the customer ldquoThe flooding was caused by poorly designed landscaping Our work is fine and fully up to code Have an engineer look at the foundation If therersquos a problem wersquoll fix itrdquo

The customer pleased by the builderrsquos cooperative attitude immediately hired a structural engineer to examine the foundation of the house On June 30 the engineer provided the customer with a written report on the condition of the foundation which stated that the foundation was properly constructed

Unhappy with the conclusions in the engineerrsquos report the customer then hired a home inspector to evaluate the house The home inspectorrsquos report concluded that the foundation of the house had been poorly constructed and was inadequately waterproofed

On July 10 the customer sent the builder the home inspectorrsquos report with a note that said ldquoUntil you fix this problem you wonrsquot get another penny from merdquo The builder immediately contacted an attorney and directed the attorney to prepare a draft complaint against the customer for nonpayment Hoping to avoid litigation the builder sent several more requests for payment to the customer The customer ignored all these requests

On September 10 the builder filed suit in federal district court properly invoking the courtrsquos diversity jurisdiction and seeking $200000 in damages for breach of contract The customerrsquos answer denied liability on the basis of alleged defective construction of the housersquos foundation

Several months later the case is nearly ready for trial However two discovery disputes have not yet been resolved

First despite a request from the builder the customer has refused to provide a copy of the report prepared by the structural engineer who examined the foundation of the house The customer claims that the report is ldquowork productrdquo and not discoverable because the customer does not intend to ask the engineer to testify at trial The builder has asked the court to order the customer to turn over the engineerrsquos report

Second the customer has asked the court to impose sanctions for the builderrsquos failure to comply with the customerrsquos demand for copies of all emails concerning construction of the foundation of the house The builder has truthfully informed the customer that all such emails were destroyed on August 2 This destruction was pursuant to the builderrsquos standard practice of permanently deleting all project-related emails from company records 60 days after construction of a project is complete There is no relevant state records-retention law

1 Should the court order the customer to turn over the engineerrsquos report Explain

2 Should the court sanction the builder for the destruction of emails related to the case and if so what factors should the court consider in determining those sanctions Explain

6

CRIMINAL LAW AND PROCEDURE QUESTION _____

A defendant was charged under state law with felony theft (Class D) and felony residential burglary (Class C) The indictment alleged that the defendant entered his neighborsrsquo home without their consent and stole a diamond ring worth at least $2500

Defense counsel filed a pretrial motion to dismiss the charges on the ground that prosecuting the defendant for both burglary and theft would constitute double jeopardy The trial court denied the motion and the defendant was prosecuted for both crimes The only evidence of the ringrsquos value offered at the defendantrsquos jury trial was the ownerrsquos testimony that she had purchased the ring two years earlier for $3000

At trial the judge issued the following jury instruction on the burglary charge prior to deliberations

If after consideration of all the evidence presented by the prosecution and defense you find beyond a reasonable doubt that the defendant entered the dwelling without the ownersrsquo consent you may presume that the defendant entered with the intent to commit a felony therein

The jury found the defendant guilty of both offenses

At the defendantrsquos sentencing hearing an expert witness called by the prosecutor testified that the diamond ring was worth between $7000 and $8000 Over defense objection the judge concluded by a preponderance of the evidence that the value of the stolen ring exceeded $5000 The judge sentenced the defendant to four yearsrsquo incarceration on the theft conviction On the burglary conviction the defendant received a consecutive sentence of seven yearsrsquo incarceration

In this state residential burglary is defined as ldquoentry into the dwelling of another without the consent of the lawful resident with the intent to commit a felony thereinrdquo Residential burglary is a Class C felony for which the minimum sentence is five years and the maximum sentence is ten years of incarceration

In this state theft is defined as ldquotaking and carrying away the property of another with the intent to permanently deprive the owner of possessionrdquo Theft is a Class D felony if the value of the item(s) taken is between $2500 and $10000 The sentence for a Class D felony theft is determined by the value of the items taken If the value is between $2500 and $5000 the maximum sentence is three yearsrsquo incarceration If the value of the items exceeds $5000 the maximum sentence is five yearsrsquo incarceration

This state affords a criminal defendant no greater rights than those mandated by the United States Constitution

1 Did the trial court err when it denied the defendantrsquos pretrial motion to dismiss on double jeopardy grounds Explain

2 Did the trial court err in its instruction to the jury on the burglary charge Explain

3 Did the trial court err when it sentenced the defendant to an additional year of incarceration on the theft conviction based on the expertrsquos testimony Explain

7

AGENCY AND PARTNERSHIP QUESTION _____

Five years ago Adam and Ben formed a general partnership Empire Partnership (Empire) to buy and sell antique automobiles at a showroom in State A Adam contributed $800000 to Empire and Ben contributed $200000 Their written partnership agreement allocated 80 of profits losses and control to Adam and 20 to Ben No filings of any type were made in connection with the formation of Empire

Three years ago a collector purchased one of Empirersquos antique cars for $3400000 The collector was willing to pay this price because of Benrsquos false representation (repeated in the sales contract) that a famous movie star had once owned the car Without the movie-star connection the car was worth only $100000 One month later when the collector discovered the truth he sued Adam Ben and Empire for $3300000 in damages The lawsuit is still pending

Two years ago Adam and Ben admitted a new partner Diane to Empire in return for her contribution of $250000 The three agreed to allocate profits losses and control 75 to Adam 10 to Ben and 15 to Diane Before joining the partnership Diane learned of the collectorrsquos claim and stated her concern to Adam and Ben that she might become liable if the claim were reduced to a judgment

Following Dianersquos admission to Empire the three partners sought to convert Empire into a limited liability partnership (LLP) Adamrsquos lawyer proposed to file with State A a ldquostatement of qualificationrdquo making an LLP election and declaring the name of the partnership to be ldquoEmpire LLPrdquo Benrsquos lawyer stated that this would not work and that a new LLP had to be formed with the assets of the old partnership transferred to the new one In the end the conversion was done the way Adamrsquos lawyer suggested with the approval of all three partners

One year ago a driver purchased a vintage car from Empire LLP based on the representation that the car was ldquofully roadworthy and capable of touring at 70 mph all dayrdquo The driver took the car on the highway at 50 mph whereupon the front suspension collapsed resulting in a crash in which the car was destroyed and the driver killed The driverrsquos estate sued Adam Ben Diane and Empire LLP for $10000000 The lawsuit is still pending

Although profitable Empire LLP does not have resources sufficient to pay the collectorrsquos claim or the claim of the driverrsquos estate

Assume that the Uniform Partnership Act (1997) applies

1 Before the filing of the statement of qualification (a) was Adam personally liable on the collectorrsquos claim Explain (b) was Diane personally liable on the collectorrsquos claim Explain

2 After the filing of the statement of qualification was Adam Ben or Diane personally liable as a partner on (a) the collectorrsquos claim or (b) the driverrsquos estatersquos claim Explain

8

February 2014 MEE

ANALYSES Constitutional Law

Trusts and Future Interests Secured Transactions

Federal Civil Procedure Criminal Law and Procedure

Agency and Partnership

CONSTITUTIONAL LAW ANALYSIS (Constitutional Law IVD)

ANALYSIS

Legal Problems

(1) Is the city ordinance requirement that businesses install floodlights a taking

(2) Is conditioning the approval of a building permit on the grant of an easement to install surveillance equipment a taking of property

DISCUSSION

Summary

The ordinance requiring businesses to install floodlights is not a per se taking under Loretto because it does not force a private landowner to allow a third party to enter and place a physical object on the land Here the city ordinance requires the businessmdashnot a third partymdashto install the floodlights

The ordinance is likely not a regulatory taking under the Penn Central balancing test While the ordinance will impose a cost on business owners that cost may be offset by the expected increase in business due to the ordinance and the ordinance does not appear to interfere with the ownerrsquos primary use of the property as a restaurant

The permit condition however is likely an uncompensated taking of property While the condition has an essential nexus with the cityrsquos legitimate interest in promoting public safety the city has not made an individualized determination that the easement condition is roughly proportional to the possibility of increased crime due to the restaurantrsquos proposed addition Thus the permit condition likely violates the Fifth Amendment as applied to the states through the Fourteenth Amendment

Point One (50) The ordinance requiring that businesses install floodlights is not a per se taking under Loretto It is not a regulatory taking under the Penn Central balancing test because the cost of compliance with the ordinance may be offset by an expected increase in business and compliance does not interfere with the businessrsquos primary use of its property as a restaurant

The city ordinance requiring a business to install floodlights does not effect a per se taking of the sort described in Loretto v Teleprompter Manhattan CATV Corp 458 US 419 (1982) because no property is physically taken by the government and the ordinance does not involve a physical invasion of private property by a third party

Even though the ordinance does not constitute an occupation of the property by either the government or a third party it is still subject to the three-factor balancing test under Penn Central Transportation Co v City of New York 438 US 104 (1978) to determine whether it is a ldquoregulatory takingrdquo Under Penn Central a court must balance (1) ldquo[t]he economic impact of the regulation on the claimantrdquo (2) ldquothe extent to which the regulation has interfered with distinct investment-backed expectationsrdquo and (3) ldquothe character of the governmental actionrdquo Id at 124 Here each factor weighs against finding that the ordinance is a taking

11

Constitutional Law Analysis

First the ordinance requirement likely has a minimal economic impact on the restaurant Compliance with the ordinance is estimated to cost $1000 and the city has found that businesses will likely recoup that cost in increased sales Also because the ordinance does not interfere with the operation of the restaurant the owner may still earn a reasonable return on its investment in the property

Second the ordinance does not interfere with the businessrsquos investment-backed expectations As in Penn Central the challenged law does not interfere with the ownerrsquos ldquoprimary expectationrdquo for use of the propertymdashin Penn Central as a railroad terminal and here as a restaurant Further the ordinance does not prevent the restaurant from expanding to meet the changing business environment

Third the character of the government action does not weigh in favor of a taking While Penn Central does say that a ldquophysical invasionrdquo is more likely to pose a taking Loretto suggests that the Courtrsquos main concern is with physical invasions by third parties Also like the landmark law challenged in Penn Central the ordinance here ldquoadjust[s] the benefits and burdens of economic life to promote the common goodrdquo Id In Penn Central the landmark law restricted development of the railroad terminal to promote the common interest in preserving historic landmarks Here the ordinance requires the businesses to install floodlights to promote the common interest in crime prevention and public safety

Because the ordinance is clearly a valid exercise of the police power it satisfies the takings clausersquos public-use requirement Kelo v City of New London 545 US 469 (2005)

In sum all three factors weigh against finding a taking under the Penn Central balancing test

Point Two (50) The permit condition may be unconstitutional as an uncompensated taking of property because the city has not made an individualized determination that the easement condition is roughly proportional to the impact of the restaurantrsquos proposed addition

In Dolan v City of Tigard 512 US 374 (1994) the Supreme Court set forth the test for determining whether an exaction imposed by a government in exchange for a discretionary benefit conferred by the government such as a condition on the approval of a building permit in this case constitutes an uncompensated taking under the Fifth Amendment The exaction is not a taking if (1) there is an ldquoessential nexusrdquo between the ldquopublic need or burdenrdquo to which the proposed development contributes and ldquothe permit condition exacted by the cityrdquo id at 386 and (2) the government makes ldquosome sort of individualized determination that the required dedication is [roughly proportional] both in nature and extent to the impact of the proposed developmentrdquo Id at 391 see also Nollan v California Coastal Commission 483 US 825 (1987)

Here the city likely can meet the nexus requirement In Dolan the landowner sought to double the size of its business which would have increased traffic on nearby roadways In exchange for approving the development the city sought an easement for a bike and pedestrian path The Court found the required nexus between the easement and the cityrsquos ldquoattempt to reduce traffic congestion by providing for alternative means of transportationrdquo 512 US at 387 Here a similar nexus likely exists between the requested easement and the cityrsquos interest in crime prevention and public safety Increased patronage and economic activity at the restaurant might attract additional crime to the area and the requested easement to install surveillance equipment would attempt to address that increased crime

12

Constitutional Law Analysis

The exaction here however may fail the second prong of the Dolan testmdashthat the exaction be roughly proportional to the anticipated impact of the requested development As noted the city in Dolan claimed that a bike and pedestrian path was needed to offset the increase in traffic due to the proposed doubling of the business The Court explained that the government must demonstrate that the additional traffic reasonably was related to the requested exaction and that the government must ldquomake some effort to quantify its findings in support of the dedication for the pedestrianbicycle pathway beyond the conclusory statement that it could offset some of the traffic demand generatedrdquo Id at 395 Here the city did not carry its burden The city simply speculates that increased patronage of the restaurant ldquomightrdquo increase crime and that the surveillance equipment ldquomightrdquo alleviate this increased crime Because the city has not made ldquosome effort to quantify its findingsrdquo in support of the easement it has not shown that the burden of the easement is roughly proportional to the benefits thought to flow from it

Thus the exaction appears to be an uncompensated taking of property in violation of the Fifth Amendment as applied to the states through the Fourteenth Amendment

13

TRUSTS AND FUTURE INTERESTS ANALYSIS ____ (Trusts and Future Interests IE3 I5 IIIA amp B)

ANALYSIS

Legal Problems

(1) How should rents dividends and sales proceeds received by the trustee prior to receipt of the sonrsquos letter have been allocated between trust income and principal

(2)(a) Did the remainder interest in the trust accelerate and become immediately payable to the daughterrsquos minor child upon the trusteersquos receipt of the sonrsquos letter and if not how should the trustee handle the distribution of the principal in the future

(2)(b) Following the trusteersquos receipt of the sonrsquos letter how should the trustee distribute future receipts of income prior to the distribution of the principal

DISCUSSION

Summary

Prior to the trusteersquos receipt of the sonrsquos letter cash dividends and rents should have been allocated to trust income and were distributable to the son the income beneficiary of the trust sales proceeds and stock dividends should have been allocated to principal

Because the sonrsquos letter to the trustee did not result in a valid disclaimer under state law (having been made more than nine months after the testatorrsquos death) the son is not deemed to have predeceased the testator Because the son is still living the class gift to the testatorrsquos grandchildren who survive the son has not closed and is not possessory it will not become possessory until the son dies The daughterrsquos minor child being the testatorrsquos only living grandchild is not currently entitled to a distribution of trust principal Trust principal will instead be distributable upon the sonrsquos death to the testatorrsquos then-living grandchildren or if there are none to the testatorrsquos then-living heirs

As for future income the trustee should either distribute the trust income to the son and the daughter as the testatorrsquos heirs accumulate the income for future distribution to those individuals ultimately entitled to the trust principal or distribute it to those presumptively entitled to the principal upon the sonrsquos death ie the daughterrsquos minor child

Point One (45) Cash dividends and rents are allocable to income sales proceeds and stock dividends are allocable to principal Items allocable to income for the period prior to the sonrsquos attempted disclaimer were distributable to the son

Receipts earned during the administration of a trust are allocable either to income or to principal Almost all states have adopted the most recent or an earlier version of the Uniform Principal and Income Act (the Act) which specifies how such receipts should be allocated

Under the Act rents (UNIF PRIN amp INC ACT (2000) sect 405 UNIF PRIN amp INC ACT (1962) sect 3(a)(1)) and cash dividends received from a corporation (UNIF PRIN amp INC ACT (2000) sect 401(b) UNIF PRIN amp INC ACT (1962) sect 6(d)) are allocable to income and are distributable to the income beneficiary of the trust

14

Trusts and Future Interests Analysis Sales proceeds (UNIF PRIN amp INC ACT (2000) sect 404(2) UNIF PRIN amp INC ACT (1962)

sect 3(b)(1)) and dividends paid in the stock of the distributing corporation (UNIF PRIN amp INC ACT (2000) sect 401(c)(1) UNIF PRIN amp INC ACT (1962) sect 3(b)(4)) are allocable to principal and added to the principal of the trust

Here the cash dividends and office building rents should have been allocated to income and until the trustee received the sonrsquos letter should have been distributed to him as the sole income beneficiary of the trust The stock dividend and proceeds from the sale of the office building should have been allocated to principal and held by the trustee for future distribution to the ultimate remaindermen of the trust

[NOTE The 2000 Uniform Principal and Income Act has been adopted in Alabama Arkansas Colorado Connecticut the District of Columbia Hawaii Idaho Iowa Kentucky Missouri Montana Nebraska New Mexico North Dakota Oregon South Dakota Utah and West Virginia]

Point Two(a) (45) Because the son did not disclaim within nine months of the testatorrsquos death there is no valid disclaimer under state law Therefore the son is not deemed to have predeceased the testator Furthermore because of the express survivorship contingency in the will the remainder in the trust does not accelerate and become distributable until the son in fact dies When the son dies the trust principal will be distributable to the testatorrsquos then-living grandchildren or if none then to the testatorrsquos then-living heirs

When a trust remainder is given to a class the class closes (ie no new persons can join the class) when there is no outstanding income interest and at least one member of the class is then entitled to demand possession of his or her share of the remainder This principle is called the rule of convenience See generally HERBERT HOVENKAMP amp SHELDON F KURTZ PRINCIPLES OF PROPERTY LAW 199ndash200 (6th ed 2005) A class member may demand possession of his or her share of the remainder upon termination of the income interest only when the class memberrsquos interest is not otherwise subject to a condition precedent See id

When a beneficiary timely disclaims an interest in a trust that beneficiary is treated as if he had predeceased the testator Here had the son disclaimed within nine months of the testatorrsquos death as required by the state statute he would have been deemed to have predeceased the testator This would have closed the class of remaindermen and the testatorrsquos then-living grandchildren (ie the daughterrsquos child) would have been entitled to the trust principal However under the state statute the sonrsquos disclaimer was not timely because he did not disclaim within nine months of the testatorrsquos death Thus because the statute is inapplicable and the son is still alive the class of grandchildren entitled to share in trust principal did not close

Because here the statute is inapplicable due to the sonrsquos failure to comply with the statutory time requirements then presumably the common-law rule allowing disclaimers (aka renunciations) at any time should apply Under the common law if a life estate is renounced the remainder interest accelerates and becomes immediately distributable to the remaindermen of the trust if the remainder is vested but not if the remainder is contingent JESSE DUKEMINIER amp ROBERT H SITKOFF WILLS TRUSTS AND ESTATES 844ndash845 (9th ed 2013) Here because the remainder is contingent upon there being grandchildren who survive the son the remainder will not accelerate It will remain open until the son dies leaving open the possibility that additional grandchildren will be included in the class or the daughterrsquos child could fall out of the class because that child fails to survive the son

And if none of the testatorrsquos grandchildren survive the son the trust principal will be distributed to the testatorrsquos heirs living at the sonrsquos death

15

Trusts and Future Interests Analysis

Point Two(b) (10) Until the trust terminates the trustee must continue to hold the trust assets The distribution of income in the meantime is unclear There are at least three possibilities Income earned on the undistributed assets could be distributed to the son and daughter as the testatorrsquos heirs accumulated and added to principal for distribution to the ultimate remaindermen or distributed from time to time to those persons who are presumptively remaindermen

When trust principal is not immediately distributable the trustee must continue to hold trust assets until the ultimate remaindermen are ascertained During this period trust income will be distributed or retained according to any instructions contained in the trust instrument See WILLIAM M MCGOVERN JR SHELDON F KURTZ amp DAVID M ENGLISH WILLS TRUSTS amp ESTATES sect 102 (4th ed 2010)

Here the testator did not specify what the trustee should do with trust income in the event the sonrsquos disclaimer did not comply with the state statute There are at least three approaches One approach would have the trustee distribute the trust income to the testatorrsquos heirs on the theory that the income represents property that was not disposed of by the testatorrsquos will and which thus passes by partial intestacy to the testatorrsquos heirs A second approach would have the trustee accumulate trust income for distribution to the ultimate remaindermen Under this approach only those individuals ultimately entitled to the principal would share in the income A third approach would have the trustee distribute trust income to those individuals who would be the remaindermen if the trust were to terminate when the income is received by the trustee under this approach trust income would be distributed to the daughterrsquos minor child until another presumptive remainderman is born This approach could result in individuals not ultimately entitled to principal say because they do not survive the son receiving income It could also result in a disproportionate distribution of income among the individuals ultimately entitled to income

[NOTE Examinees should demonstrate a recognition and understanding of the income-allocation problem and the alternatives available to address that issue There is no widely accepted solution to the problem Examinees who cite any of these possible problem-solving approaches may receive credit]

16

SECURED TRANSACTIONS ANALYSIS (Secured Transactions IB IID E amp F IIIB IVA B amp F)

ANALYSIS

Legal Problems

(1)(a) What is the nature of the bankrsquos claim to the businessrsquos equipment

(1)(b) What is the nature of the finance companyrsquos claim to the businessrsquos equipment

(1)(c) As between the bank and the finance company whose claim to the businessrsquos equipment has priority

(2) Do the claims of the bank and the finance company continue in the item of equipment sold by the business to the competitor

DISCUSSION

Summary

The bank and the finance company both have perfected security interests in the businessrsquos equipment Even though the finance companyrsquos perfected security interest was created first the bankrsquos perfected security interest has priority because the bankrsquos financing statement was filed before the finance companyrsquos financing statement The security interests of the bank and the finance company continue in the item of equipment sold by the business to the competitor because their security interests were perfected and the competitor was not a buyer in ordinary course of business

Point One(a) (25) The bank has a perfected security interest in the businessrsquos equipment

The bank has met all criteria necessary for it to have an attached and enforceable security interest in the businessrsquos equipment First value must be given UCC sect 9-203(b)(1) This criterion is fulfilled by the loan made by the bank to the business Second the debtor must have rights in the collateral UCC sect 9-203(b)(2) Clearly the business has rights in its equipment Third either the secured party must take possession of the collateral or the debtor must authenticate a security agreement containing a description of the collateral UCC sect 9-203(b)(3) The agreement that the business owner signed is a ldquosecurity agreementrdquo because it is an agreement that creates or provides for a security interest UCC sect 9-102(a)(74) By signing the security agreement the business owner authenticated it UCC sect 9-102(a)(7) Therefore all three criteria are fulfilled and the bank has an enforceable and attached security interest

A security interest is perfected when it has attached and when any additional steps required for perfection have occurred UCC sect 9-308(a) Generally speaking the additional steps will either be possession of the collateral by the secured party or the filing of a financing statement with respect to the collateral See UCC sectsect 9-310 9-313 In this case the bank filed a financing statement naming the debtor and sufficiently indicating the collateral The collateral indication is sufficient because it identifies the collateral by type of property See UCC sectsect 9-504 9-108 The fact that the financing statement was filed before the security interest was created is

17

Secured Transactions Analysis

not a problem Even though the security agreement had not yet been signed the business had authorized the filing of the financing statement in an authenticated record UCC sect 9-509(a)(1) Moreover the financing statement may be filed before the security agreement is created UCC sect 9-502(d)

Point One(b) (10) The finance company also has a perfected security interest in the businessrsquos equipment

The finance companyrsquos security interest is enforceable and attached for the same reasons as the bankrsquos security interest The loan from the finance company to the business constitutes value the business has rights in the collateral and the business owner has authenticated a security agreement containing a description of the collateral The finance companyrsquos security interest is perfected because the finance company filed a financing statement with respect to it that provides that the business is the debtor and indicates that the collateral is equipment

Point One(c) (30) The bankrsquos security interest has priority over the finance companyrsquos security interest because the bankrsquos financing statement was filed first

As between two perfected security interests the general rule is that the security interest that was the earlier to be either perfected or the subject of a filed financing statement has priority UCC sect 9-322(a)(1) While the finance companyrsquos security interest was perfected before the bankrsquos (March 15 vs March 22) the bankrsquos financing statement was filed even earlier on March 2 Thus under the first-to-file-or-perfect rule of UCC sect 9-322(a)(1) the bankrsquos security interest has priority No exceptions to the general rule apply here

Point Two (35) A security interest in collateral continues notwithstanding its sale unless an exception applies Because the security interests of the bank and the finance company were perfected and the competitor was not a buyer in ordinary course of business no exception applies and the security interests of both creditors continue in the equipment sold to the competitor

As a general rule a security interest in collateral continues notwithstanding the fact that the debtor has sold the collateral to another person UCC sect 9-315(a)(1) Thus unless an exception applies the security interests of the bank and the finance company will continue in the item of equipment sold to the competitor

A buyer of goods will take free of an unperfected security interest in those goods See UCC sect 9-317(a)(2) However when the competitor bought the businessrsquos equipment both the bank and the finance company had perfected security interests in the equipment

A buyer can take free even of a perfected security interest in goods if the buyer is a ldquobuyer in ordinary course of businessrdquo See UCC sect 9-320(a) However the competitor was not a buyer in ordinary course of business To be a ldquobuyer in ordinary course of businessrdquo a buyer must buy goods from a seller that is in the business of selling goods of that kind See UCC sect 1-201(b)(9) The competitor bought this equipment from a seller that is not in the business of selling goods of this kind so the competitor was not a buyer in ordinary course of business with respect to these goods

Because no exception applies the security interests of the bank and the finance company continue even after the item of equipment was sold to the competitor

18

FEDERAL CIVIL PROCEDURE ANALYSIS (Federal Civil Procedure IVD)

ANALYSIS

Legal Problems

(1) Is a document prepared in the course of a contract dispute protected from discovery as ldquowork productrdquo when there is no evidence that the document was prepared in anticipation of litigation

(2)(a) Is a partyrsquos failure to provide relevant electronically stored information excused when the information was destroyed pursuant to a routine document retention scheme at a time when litigation was contemplated by the destroying party

(2)(b) What sanctions should be imposed on a party for allowing the destruction of evidence that is relevant to potential future litigation

DISCUSSION

Summary

The report prepared by the structural engineer is probably not work product and is thus discoverable The engineer examined the foundation of the house at the customerrsquos request and the engineerrsquos findings are potentially relevant to the customerrsquos claim that the foundation is defective The report was not prepared in anticipation of litigation The customer appears to have sought the engineerrsquos opinion in response to the builderrsquos offer to fix any problems with the foundation that an engineer might identify Because the report was not prepared in anticipation of litigation it is not protected by the work-product doctrine

The builder should have taken appropriate steps to preserve evidence including suspending its document retention program as soon as it began planning for litigationmdashie on July 10 Its destruction of potentially relevant material after that date was wrongful However a court is unlikely to impose severe sanctions on the builder because there are no facts indicating that the builder acted in bad faith and the customer can prove that the foundation is defective without the destroyed emails

Point One (40) The customer must turn over the engineerrsquos report because it was not prepared in anticipation of litigation

In general a party to a lawsuit in federal court ldquomay obtain discovery regarding any nonprivileged matter that is relevant to any partyrsquos claim or defenserdquo FED R CIV P 26(b)(1) (2009) This includes the right to inspect and copy documents in the other partyrsquos possession FED R CIV P 34(a)(1) Here the customer hired a structural engineer to examine the foundation of the house The engineerrsquos report on the foundation is likely to include information that would be relevant to the customerrsquos claim that the foundation was defectively constructed

The so-called ldquowork productrdquo rule allows a party to refuse to turn over ldquodocuments that are prepared in anticipation of litigation or for trialrdquo by that partyrsquos representative including

19

Federal Civil Procedure Analysis

a consultant Thus if the customer had hired the structural engineer to prepare a report ldquoin anticipation of litigationrdquo that report might not be discoverable See FED R CIV P 26(b)(3)

In this case however the customer hired the engineer to evaluate the foundation of the house as part of the customerrsquos negotiation with the builder concerning the housersquos flooding problem The builder told the customer that the housersquos landscaping was the reason for the flooding and the builder told the customer ldquoHave an engineer look at the foundation If therersquos a problem wersquoll fix itrdquo The customer appears to have acted in response to that statement There is no indication that the customer anticipated any kind of legal action at the time that the structural engineer was hired Accordingly the structural engineerrsquos report is discoverable and the court should order the customer to turn it over

[NOTE If an examinee concludes that the structural engineerrsquos report was prepared in anticipation of litigation then the examinee should also conclude that the report is not discoverable Documents prepared in anticipation of litigation do not need to be disclosed to an adverse party unless that party can demonstrate a ldquosubstantial needrdquo for the documents and an inability to obtain substantially equivalent information without ldquoundue hardshiprdquo FED R CIV P 26(b)(3)(A)(ii) Furthermore a report prepared by an expert who is not expected to testify is not discoverable in the absence of ldquoexceptional circumstancesrdquo making it ldquoimpracticablerdquo to obtain the information in another way FED R CIV P 26(b)(4)(D)(ii) The builder probably cannot make these showings here unless the engineerrsquos report deals with circumstances that have since changed There is no evidence that the structural engineer would have had access to any information or facts that the builder would not already know as a result of its construction and subsequent inspection of the house In addition if necessary the builder could ask the court for permission to arrange for a further inspection of the house by a structural engineer hired by the builder See FED R CIV P 34(a)(2) Accordingly if an examinee concludes that the report was prepared in anticipation of litigation the examinee should also conclude that the builder is not entitled to see the report]

Point Two(a) (30) Because the builder anticipated that it might be involved in litigation concerning its contract with the customer the builder acted wrongfully in destroying emails that were relevant to the housersquos construction even though the emails were destroyed pursuant to a routine document retention plan

As noted above a party to a lawsuit in federal court ldquomay obtain discovery regarding any nonprivileged matter that is relevant to any partyrsquos claim or defenserdquo FED R CIV P 26(b)(1) This includes emails and other electronically stored information FED R CIV P 34(a)(1)(A) Here the customer has requested all the builderrsquos emails pertaining to work done on the foundation of the house Ordinarily the builder would be obliged to turn over this information which is relevant to the customerrsquos defense that the housersquos foundation was poorly constructed

Unfortunately the emails in question no longer exist because the builder destroyed them on August 2

In general spoliation of evidence (destruction or alteration of evidence) is improper if the party who destroyed or altered the evidence ldquohas notice that the evidence is relevant to litigation or should have known that the evidence may be relevant to future litigationrdquo Fujitsu Ltd v Federal Express Corp 247 F3d 423 436 (2d Cir 2001) It is improper for a party to destroy electronic information relevant to pending litigation even if the destruction occurs before there is any request or order seeking the information See eg Leon v IDX Sys Corp 464 F3d 951 (9th Cir 2006) (plaintiffrsquos intentional destruction of computer files warranted dismissal even

20

In this case the builderrsquos destruction of the emails was pursuant to a routine document retention plan The Federal Rules provide expressly that in the absence of ldquoexceptional circumstancesrdquo parties should not be sanctioned for the loss of electronically stored information when the loss occurs pursuant to ldquoroutine good-faith operation of an electronic information systemrdquo FED R CIV P 37(e) However when a party anticipates litigation ldquoit must suspend its routine document retentiondestruction policy and put in place a lsquolitigation holdrsquo to ensure the preservation of relevant documentsrdquo Zubulake v UBS Warburg LLC 220 FRD 212 218 (SDNY 2003)

Federal Civil Procedure Analysis

though spoliation occurred before order compelling discovery) Similarly the duty to preserve evidence applies to a party who anticipates litigation even if litigation has not yet been commenced See THE SEDONA PRINCIPLES BEST PRACTICES RECOMMENDATIONS amp PRINCIPLES FOR ADDRESSING ELECTRONIC DOCUMENT PRODUCTION 70 cmt 14a (2d ed 2007)

The builder destroyed the emails on August 2 At that time the builder knew that litigation was a possibility because the builder had already directed its attorney to prepare a draft complaint for possible filing Knowing that litigation was a possibility the builder had a duty to take steps to preserve evidence including the emails in question See generally Fujitsu Ltd

Thus the builderrsquos destruction of potentially relevant emails at a time when it knew that litigation was a possibility was improper It had a duty to preserve evidence and it breached that duty

[NOTE Because courts have used different words to describe the test for when evidence must be preserved an examineersquos precise formulation of the test is not critical]

Point Two(b) (30) In determining appropriate sanctions for spoliation courts consider both the level of culpability of the spoliating party and the degree of prejudice the loss of evidence has caused the other party Here the builderrsquos destruction of evidence does not appear to have been willful nor is it likely to pose a significant obstacle to the customerrsquos defense Any sanctions imposed by the court should be modest

Federal courts have inherent power to control the litigation process and can sanction misbehavior including spoliation even when there has been no specific violation of the Federal Rules of Civil Procedure See generally Chambers v NASCO Inc 501 US 32 (1991) (discussing courtrsquos inherent power to control the litigation process) The range of available sanctions is broad It can include such sanctions as the payment of expenses incurred by the other party as a result of the destruction of the evidence an instruction to the jury authorizing it to draw an adverse inference from the destruction of the evidence a shifting of the burden of proof on the relevant issue or even judgment against the responsible party See eg Residential Funding Corp v DeGeorge Financial Corp 306 F3d 99 108 (2d Cir 2002) (adverse inference) Silvestri v General Motors Corp 271 F3d 583 593 (4th Cir 2001) (possibility of dismissal) Cf FED R CIV P 37(b)(2)(A) (listing remedies for failure to comply with discovery obligations)

In determining appropriate sanctions for spoliation courts consider both the level of culpability of the spoliating party and the degree of prejudice the loss of evidence has caused the other party Many courts impose severe sanctions (such as an adverse-inference instruction or the entry of judgment against the spoliating party) only when there is evidence of bad faith in the form of an intentional effort to hide information Eg Greyhound Lines Inc v Wade 485 F3d 1032 1035 (8th Cir 2007) (spoliation sanction requires intentional destruction out of desire ldquoto suppress the truthrdquo) However other courts have said that negligence in preserving evidence can

21

Federal Civil Procedure Analysis

support an adverse-inference instruction See Residential Funding 306 F3d at 108 (negligence enough under some circumstances)

Although a court might well order an evidentiary hearing on the issue of sanctions the facts presented do not seem appropriate for severe sanctions First the evidence was destroyed pursuant to the builderrsquos standard document retention plan and there is no evidence that the builder deliberately failed to suspend its usual procedures with the purpose of allowing the destruction of evidence Second the loss of this evidence will not severely hinder the customerrsquos presentation of his case The central issue is whether the foundation of the house was properly constructed If the construction job was poorly done the customer can present evidence derived from inspection of the premises to prove that point The customer can also depose witnesses about any issues that arose during construction

Under the circumstances a court is not likely to impose particularly severe sanctions although it might shift the burden to the builder to show that the foundation was properly constructed or it might require the builder to reimburse any expenses the customer incurs to discover and prove the facts about issues or disputes that arose during construction of the foundation

[NOTE The result reached by the examinee is less important than the examineersquos recognition that (a) a range of sanctions is available to the court and (b) the appropriate sanction depends both on the culpability of the builder and the prejudice suffered by the customer]

22

CRIMINAL LAW AND PROCEDURE ANALYSIS (Criminal Law and Procedure IIA amp D VE amp F)

ANALYSIS

Legal Problems

(1) Did charging the defendant with both theft and burglary constitute double jeopardy

(2) Did the jury instruction violate the due process clause either by relieving the prosecution of the burden of proving the element of intent or by shifting the burden to the defendant to disprove that element

(3) Did the sentence imposed in this case for the theft conviction unconstitutionally deprive the defendant of his right to a jury trial on the issue of the value of the stolen item

DISCUSSION

Summary

The trial court properly denied the defendantrsquos pretrial motion to dismiss the charges on double jeopardy grounds The defendant may be charged with and convicted of both theft and burglary Each of the charges has an element that the other does not Neither charge is a lesser-included offense nor are they multiplicitous Thus charging both theft and burglary does not violate double jeopardy

The jury instruction on the burglary charge was constitutionally flawed It could have been reasonably understood by the jury as either (1) an irrebuttable conclusive presumption (which relieved the prosecution of proving the element of intent and removed the issue from the jury) or (2) a rebuttable mandatory presumption (which unconstitutionally shifted the burden of proof on an element of a charged offense from the prosecution to the defendant)

Because the four-year sentence imposed by the judge was based on the judgersquos finding by a preponderance of the evidence that the value of the stolen ring exceeded $5000 the sentence violates the defendantrsquos right to a jury determination beyond a reasonable doubt of the value of the ring

Point One (30) Charging the defendant with theft and burglary did not constitute double jeopardy

The Double Jeopardy Clause of the Fifth Amendment provides that a person shall not be twice put in jeopardy for the ldquosame offenserdquo Thus the question is whether the elements of the theft charge are wholly contained in the burglary charge or vice versa If the elements of the lesser charge (theft) are not wholly contained in the greater charge (burglary)mdashie if each charge requires proof of a fact that the other does notmdashthen convicting the defendant of both crimes would not violate double jeopardy even when the two offenses occurred at the same time and are thus arguably part of the ldquosame transactionrdquo Blockburger v United States 284 US 299 304 (1932) See also Albernaz v United States 450 US 333 344 n3 (1981) United States v Dixon 509 US 688 704 (1993)

23

Criminal Law and Procedure Analysis

Here theft and burglary each require proof of an element not required for the other crime Burglary may be defined differently in different jurisdictions However it almost invariably requires entry into a building or dwelling of another with the specific intent to commit a felony therein and the crime of burglary is complete upon the entry into the building or dwelling with such intent See eg Cannon v Oklahoma 827 P2d 1339 1342 (Okla Crim App 1992) In contrast theft which also may be defined differently in different states almost invariably requires the taking and carrying away of an item of personal property belonging to another with the intent to steal or permanently deprive the owner of possession

Here the ldquotakingrdquo or ldquostealingrdquo element is not contained in the definition of burglary and the ldquoentryrdquo element of burglary is not contained in the definition of theft Because theft is not a lesser-included offense of burglary and burglary is not a lesser-included offense of theft charging the defendant for both burglary and theft did not violate double jeopardy and the court properly denied the defense motion on those grounds Yparrea v Dorsey 64 F3d 577 579ndash80 (10th Cir 1995) citing Blockburger 284 US at 304

Finally the defendantrsquos motion to dismiss all the charges on double jeopardy grounds was improper because if both charges were for the same offense the motion should have requested dismissal of one charge not both

Point Two (35) The jury instruction on the burglary charge violated the Due Process Clause because it created either (1) an irrebuttable conclusive presumption (which relieved the prosecution of proving the element of intent and removed that issue from the jury) or (2) a rebuttable mandatory presumption (which unconstitutionally shifted the burden of proof on an element of a charged offense to the defendant)

The Supreme Court has interpreted the Due Process Clause of the US Constitution to require that the prosecution prove all elements of an offense beyond a reasonable doubt See In re Winship 397 US 358 364 (1970) The burden of proof cannot be shifted to the defendant by presuming an essential element upon proof of other elements of the offense because shifting the burden of persuasion with respect to any element of a criminal offense is contrary to the Due Process Clause See Mullaney v Wilbur 421 US 684 (1975)

The crime of burglary includes entry into a building or dwelling with the specific intent to commit a felony therein The requirement that the prosecutor prove beyond a reasonable doubt that the defendant had this specific intent distinguishes burglary from general-intent crimes like trespass See Sandstrom v Montana 442 US 510 523 (1979)

Here the jury was instructed that if ldquoafter consideration of all the evidence presented by the prosecution and defense you find beyond a reasonable doubt that the defendant entered the dwelling without the ownersrsquo consent you may presume that the defendant entered with the intent to commit a felony thereinrdquo This instruction was unconstitutional because it created either an irrebuttable conclusive presumption or a rebuttable mandatory presumption

A conclusive presumption is ldquoan irrebuttable direction by the court to find intent once convinced of the facts triggering the presumptionrdquo Id at 517 Here the jurors were instructed that once the prosecutor established that the defendant entered the neighborsrsquo house without consent they ldquomay presumerdquo that he intended to commit a felony therein The jurors may have reasonably concluded from this instruction that if they found that the defendant intended to enter his neighborsrsquo home without permission they must further find that he entered with the specific intent to commit a felony therein Because this instruction could operate as a conclusive

24

Criminal Law and Procedure Analysis

irrebuttable presumption by eliminating intent ldquoas an ingredient of the offenserdquo it violated due process by relieving the prosecution of the burden of proof for this element Id at 522

In the alternative the jury instruction could have been reasonably understood to create a rebuttable mandatory presumption which ldquotells [the jury] they must find the elemental fact upon proof of the basic fact at least unless the defendant has come forward with some evidence to rebut the presumed connection between the two factsrdquo County Court of Ulster County New York v Allen 442 US 140 157 (1979) The due process problem created by rebuttable mandatory presumptions is that ldquo[t]o the extent that the trier of fact is forced to abide by the presumption and may not reject it based on an independent evaluation of the particular facts presented by the State the analysis of the presumptionrsquos constitutional validity is logically divorced from those facts and based on the presumptionrsquos accuracy in the run of casesrdquo Id at 159

Unlike irrebuttable conclusive presumptions rebuttable mandatory presumptions are not always per se violations of the Due Process Clause However the Supreme Court of the United States has held that jury instructions that could reasonably be understood as shifting the burden of proof to the defendant on an element of the offense are unconstitutional Francis v Franklin 471 US 307 (1985) Here the argument that the jury instruction operated as a rebuttable mandatory presumption is supported by the fact that the judge also instructed the jury to ldquoconsider[ ] all the evidence presented by the prosecution and defenserdquo However even if the instruction created a rebuttable mandatory presumption it would be unconstitutional because it shifted the burden to the defense on an element of the offense Sandstrom 442 US at 524 Mullaney 421 US at 686

[NOTE Whether an examinee identifies the jury instruction as containing a ldquoconclusiverdquo or ldquomandatoryrdquo presumption is less important than the examineersquos analysis of the constitutional infirmities]

Point Three (35) The trial court violated the defendantrsquos Sixth Amendment right to a jury trial on an essential element of the offense when it found by a preponderance of the evidence that the ring was worth over $5000 and increased the defendantrsquos sentence based on this finding

In the statutory scheme under which the defendant was tried and convicted a Class D felony theft is defined as theft of item(s) with a value between $2500 and $10000 The jury found that the value of the diamond ring was at least $2500 and convicted the defendant of felony theft However at sentencing the trial court made a separate finding by a preponderance of the evidence that the value of the ring was greater than $5000 Following the statutersquos two-tiered sentencing scheme the judge then imposed on the defendant a sentence that was one year longer than the maximum that would otherwise have been allowed

The judgersquos sentence was unconstitutional because it violated the defendantrsquos Sixth Amendment right to a jury trial on this question The Supreme Court held in Apprendi v New Jersey 530 US 466 (2000) that ldquo[o]ther than the fact of a prior conviction any fact that increases the penalty for a crime beyond the prescribed statutory maximum must be submitted to a jury and proved beyond a reasonable doubtrdquo because ldquo[i]t is unconstitutional for a legislature to remove from the jury the assessment of facts that increase the prescribed range of penalties to which a criminal defendant is exposed [because] such facts must be established by proof beyond a reasonable doubtrdquo Id The Court reaffirmed Apprendi in Blakely v Washington 542 US 296 (2004) holding that the ldquolsquostatutory maximumrsquo for Apprendi purposes is the maximum sentence a judge may impose solely on the basis of the facts reflected in the jury verdict or admitted by the defendantrdquo Id at 303 (emphasis in original) In United States v Booker 543 US 220 (2005)

25

Criminal Law and Procedure Analysis

the Court relied on Blakely and Apprendi to conclude that protecting a defendantrsquos Sixth Amendment right to a jury trial required that ldquo[a]ny fact which is necessary to support a sentence exceeding the maximum authorized by the facts established by a plea of guilty or a jury verdict must be admitted by the defendant or proved to a jury beyond a reasonable doubtrdquo Id at 244

Thus in order to constitutionally increase a sentence above the statutory maximum of three years the jury must have found beyond a reasonable doubt that the value of the ring exceeded $5000 Here the court made the finding based on an appraisal proffered by the prosecutor only at sentencing and the judgersquos finding was by a preponderance of the evidence rather than beyond a reasonable doubt

26

AGENCY AND PARTNERSHIP ANALYSIS __________ (Agency and Partnership VA amp C VI)

ANALYSIS

Legal Problems

(1) Is a partner in a general partnership personally liable on a claim arising from misrepresentations by another partner made in the course of the partnership business

(2) Does a newly admitted partner in a general partnership become personally liable on existing claims against the partnership

(3) After the filing by a general partnership of a statement of qualification as a limited liability partnership are the partners personally liable as partners on (a) an existing claim against the general partnership and (b) a claim against the partnership that arose after the filing

DISCUSSION

Summary

Adam and Ben formed a general partnership under which they were jointly and severally liable for obligations of the partnership Thus Adam was personally liable for misrepresentations by Ben made in the ordinary course of the partnership business

Upon joining the general partnership Diane became personally liable for the obligations of the partnership arising after her admission but not for obligations pre-existing her admission such as the collectorrsquos claim

By filing a statement of qualification the three partners properly elected limited liability partnership status As partners in an LLP none of the three partners is personally liable as a partner for partnership obligations arising after the election such as the claim by the driverrsquos estate The election however does not change their personal liability on pre-existing claims that arose before the election such as the collectorrsquos claim

Point One (30) As a general partner of Empire a general partnership Adam became personally liable on the collectorrsquos claim a valid claim against the partnership that arose because of Benrsquos wrongful act in the ordinary course of the partnership business

When the collectorrsquos claim arose Empire was a general partnership composed of Adam and Ben Under UPA (1997) sect 306(a) partners of a general partnership are liable jointly and severally for all obligations of the partnership Under UPA (1997) sect 305(a) the partnership could become obligated for the loss caused to the collector as a result of the misrepresentation by Ben provided he was acting in the ordinary course of the partnership business Because there was no statement that limited his partnership authority Ben as partner was ldquoan agent of the partnership for the purpose of its businessrdquo See UPA (1997) sect 301(1) Benrsquos misrepresentation to the collector even if intentional appears to be in the ordinary course of the partnershiprsquos business of dealing

27

Agency and Partnership Analysis

in antique cars Thus Benrsquos wrongful act created a partnership obligation for which Adam was jointly and severally liable

[NOTE Generally a partnership creditor must ldquoexhaust the partnershiprsquos assets before levying on a judgment debtor partnerrsquos individual property where the partner is personally liable for the partnership obligationrdquo as a result of his status as a partner UPA (1997) sect 307 cmt 4 As the UPA comments explain this places Adam more in the position of guarantor than principal debtor on the partnership obligation Id cmt 4 Although an examinee might discuss this point the call focuses on whether Adam is personally liable not how the liability might be enforced]

Point Two (30) Because the collectorrsquos claim arose before Diane joined Empire Diane did not become personally liable on the claim

Diane was admitted to Empire when it was a general partnership and after the collectorrsquos claim arose While the general rule under UPA (1997) sect 306(a) is that the partners of a general partnership are liable jointly and severally for all obligations of the partnership there is a special rule for partners who are admitted during the duration of the partnership Under UPA (1997) sect 306(b) a person admitted to an existing partnership is not personally liable for any partnership obligations incurred before the personrsquos admission Because Diane was admitted to Empire after the collectorrsquos claim arose Diane is not personally liable on the claim

Dianersquos knowledge of the pre-existing claim and her stated concern about becoming liable on the collectorrsquos claim do not change her personal nonliability to the collector Although partners who have a liability shield can assume liability to third parties through private contractual guarantees or modifications to the partnership agreement Dianersquos stated concern constituted neither a guaranty to the collector nor ldquoan intentional waiver of liability protectionsrdquo See UPA (1997) sect 306 cmt 3 (describing methods for waiver of liability protections under sect 306(c) applicable in limited liability partnerships)

At most Diane will lose her investment in the partnership as a result of the collectorrsquos claim Although Diane did not become personally liable on the collectorrsquos claim when she joined the partnership the $250000 she contributed to the partnership is ldquoat risk for the satisfaction of existing partnership debtsrdquo UPA (1997) sect 306 cmt 2

Point Three (40) Filing the statement of qualification was effective to elect limited liability partnership status Despite this new status Adam and Ben remain personally liable on the collectorrsquos claim which arose before the election But as partners in an LLP neither Adam Ben nor Diane is personally liable as a partner on the driverrsquos estatersquos claim which arose after the election

Under UPA (1997) sect 1001 a general partnership can make an election and become a limited liability partnershipmdashif the partners approve the conversion by a vote equivalent to that necessary to amend the partnership agreement and the partnership then files a statement of qualification that specifies the name of the partnership its principal office and its election to be an LLP Here the partners agreed unanimouslymdashsufficient to amend their agreement under UPA (1997) sect 401(j)mdashand the statement of qualification was filed In addition the name of Empire LLP properly included an appropriate ending ldquoLLPrdquo See UPA (1997) sect 1002

Although another way to effectuate a ldquoconversionrdquo (as suggested by Benrsquos lawyer) is to form a new LLP and transfer the assets of the old general partnership to the new LLP the

28

Agency and Partnership Analysis

method used here (approval by the partners and the filing of a statement of qualification) is also sufficient to create LLP status

Thus Empire became Empire LLP as of the date of filing of the statement of qualification See UPA (1997) sect 1001 What effect did this have on the collectorrsquos claim which predated the filing According to UPA (1997) sect 306(c) an obligation incurred while a partnership is an LLP is solely a partnership obligation As the collectorrsquos claim predated the LLP Adam and Ben remain personally liable on the collectorrsquos claim Diane on the other hand was not personally liable on the collectorrsquos claim either before or after the filing of the statement of qualification See Point Two above

The driverrsquos estatersquos claim arose after Empire became Empire LLP Under UPA (1997) sect 306(c) an obligation incurred while a partnership is an LLP is solely a partnership obligationThus Adam Ben and Diane as partners are all protected from personal liability on the driverrsquos estatersquos claim But there may be personal liability if any of them was negligent or otherwise acted wrongfully by not informing the buyer of the bad suspension that caused the accident

29

National Conference of Bar Examiners 302 South Bedford Street | Madison WI 53703-3622 Phone 608-280-8550 | Fax 608-280-8552 | TDD 608-661-1275

wwwncbexorg e-mail contactncbexorg

  • Preface
  • Description of the MEE
  • Instructions
  • February 2014 Questions
    • Constitutinal Law Question
    • Trusts and Future Interests Question
    • Secured Transactions Question
    • Federal Civil Procedure Question
    • Criminal Law and Procedure Question
    • Agency and Partnership Question
      • February 2014 Analyses
        • Constitutional Law Analysis
        • Trust and Future Interests Analysis
        • Secured Transactions Analysis
        • Federal Civil Procedure Analysis
        • Criminal Law and Procedure Analysis
        • Agency and Partnership Analysis
            • ltlt13 ASCII85EncodePages false13 AllowTransparency false13 AutoPositionEPSFiles true13 AutoRotatePages None13 Binding Left13 CalGrayProfile (Dot Gain 20)13 CalRGBProfile (sRGB IEC61966-21)13 CalCMYKProfile (US Web Coated 050SWOP051 v2)13 sRGBProfile (sRGB IEC61966-21)13 CannotEmbedFontPolicy Error13 CompatibilityLevel 1413 CompressObjects Tags13 CompressPages true13 ConvertImagesToIndexed true13 PassThroughJPEGImages true13 CreateJobTicket false13 DefaultRenderingIntent Default13 DetectBlends true13 DetectCurves 0000013 ColorConversionStrategy CMYK13 DoThumbnails false13 EmbedAllFonts true13 EmbedOpenType false13 ParseICCProfilesInComments true13 EmbedJobOptions true13 DSCReportingLevel 013 EmitDSCWarnings false13 EndPage -113 ImageMemory 104857613 LockDistillerParams false13 MaxSubsetPct 10013 Optimize true13 OPM 113 ParseDSCComments true13 ParseDSCCommentsForDocInfo true13 PreserveCopyPage true13 PreserveDICMYKValues true13 PreserveEPSInfo true13 PreserveFlatness true13 PreserveHalftoneInfo false13 PreserveOPIComments true13 PreserveOverprintSettings true13 StartPage 113 SubsetFonts true13 TransferFunctionInfo Apply13 UCRandBGInfo Preserve13 UsePrologue false13 ColorSettingsFile ()13 AlwaysEmbed [ true13 ]13 NeverEmbed [ true13 ]13 AntiAliasColorImages false13 CropColorImages true13 ColorImageMinResolution 30013 ColorImageMinResolutionPolicy OK13 DownsampleColorImages true13 ColorImageDownsampleType Bicubic13 ColorImageResolution 30013 ColorImageDepth -113 ColorImageMinDownsampleDepth 113 ColorImageDownsampleThreshold 15000013 EncodeColorImages true13 ColorImageFilter DCTEncode13 AutoFilterColorImages true13 ColorImageAutoFilterStrategy JPEG13 ColorACSImageDict ltlt13 QFactor 01513 HSamples [1 1 1 1] VSamples [1 1 1 1]13 gtgt13 ColorImageDict ltlt13 QFactor 01513 HSamples [1 1 1 1] VSamples [1 1 1 1]13 gtgt13 JPEG2000ColorACSImageDict ltlt13 TileWidth 25613 TileHeight 25613 Quality 3013 gtgt13 JPEG2000ColorImageDict ltlt13 TileWidth 25613 TileHeight 25613 Quality 3013 gtgt13 AntiAliasGrayImages false13 CropGrayImages true13 GrayImageMinResolution 30013 GrayImageMinResolutionPolicy OK13 DownsampleGrayImages true13 GrayImageDownsampleType Bicubic13 GrayImageResolution 30013 GrayImageDepth -113 GrayImageMinDownsampleDepth 213 GrayImageDownsampleThreshold 15000013 EncodeGrayImages true13 GrayImageFilter DCTEncode13 AutoFilterGrayImages true13 GrayImageAutoFilterStrategy JPEG13 GrayACSImageDict ltlt13 QFactor 01513 HSamples [1 1 1 1] VSamples [1 1 1 1]13 gtgt13 GrayImageDict ltlt13 QFactor 01513 HSamples [1 1 1 1] VSamples [1 1 1 1]13 gtgt13 JPEG2000GrayACSImageDict ltlt13 TileWidth 25613 TileHeight 25613 Quality 3013 gtgt13 JPEG2000GrayImageDict ltlt13 TileWidth 25613 TileHeight 25613 Quality 3013 gtgt13 AntiAliasMonoImages false13 CropMonoImages true13 MonoImageMinResolution 120013 MonoImageMinResolutionPolicy OK13 DownsampleMonoImages true13 MonoImageDownsampleType Bicubic13 MonoImageResolution 120013 MonoImageDepth -113 MonoImageDownsampleThreshold 15000013 EncodeMonoImages true13 MonoImageFilter CCITTFaxEncode13 MonoImageDict ltlt13 K -113 gtgt13 AllowPSXObjects false13 CheckCompliance [13 None13 ]13 PDFX1aCheck false13 PDFX3Check false13 PDFXCompliantPDFOnly false13 PDFXNoTrimBoxError true13 PDFXTrimBoxToMediaBoxOffset [13 00000013 00000013 00000013 00000013 ]13 PDFXSetBleedBoxToMediaBox true13 PDFXBleedBoxToTrimBoxOffset [13 00000013 00000013 00000013 00000013 ]13 PDFXOutputIntentProfile ()13 PDFXOutputConditionIdentifier ()13 PDFXOutputCondition ()13 PDFXRegistryName ()13 PDFXTrapped False1313 CreateJDFFile false13 Description ltlt13 ARA 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 BGR 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 CHS ltFEFF4f7f75288fd94e9b8bbe5b9a521b5efa7684002000410064006f006200650020005000440046002065876863900275284e8e9ad88d2891cf76845370524d53705237300260a853ef4ee54f7f75280020004100630072006f0062006100740020548c002000410064006f00620065002000520065006100640065007200200035002e003000204ee553ca66f49ad87248672c676562535f00521b5efa768400200050004400460020658768633002gt13 CHT ltFEFF4f7f752890194e9b8a2d7f6e5efa7acb7684002000410064006f006200650020005000440046002065874ef69069752865bc9ad854c18cea76845370524d5370523786557406300260a853ef4ee54f7f75280020004100630072006f0062006100740020548c002000410064006f00620065002000520065006100640065007200200035002e003000204ee553ca66f49ad87248672c4f86958b555f5df25efa7acb76840020005000440046002065874ef63002gt13 CZE 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 DAN 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 DEU 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 ESP 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 ETI 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 FRA 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 GRE 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 HEB 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 HRV (Za stvaranje Adobe PDF dokumenata najpogodnijih za visokokvalitetni ispis prije tiskanja koristite ove postavke Stvoreni PDF dokumenti mogu se otvoriti Acrobat i Adobe Reader 50 i kasnijim verzijama)13 HUN 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 ITA 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 JPN ltFEFF9ad854c18cea306a30d730ea30d730ec30b951fa529b7528002000410064006f0062006500200050004400460020658766f8306e4f5c6210306b4f7f75283057307e305930023053306e8a2d5b9a30674f5c62103055308c305f0020005000440046002030d530a130a430eb306f3001004100630072006f0062006100740020304a30883073002000410064006f00620065002000520065006100640065007200200035002e003000204ee5964d3067958b304f30533068304c3067304d307e305930023053306e8a2d5b9a306b306f30d530a930f330c8306e57cb30818fbc307f304c5fc59808306730593002gt13 KOR ltFEFFc7740020c124c815c7440020c0acc6a9d558c5ec0020ace0d488c9c80020c2dcd5d80020c778c1c4c5d00020ac00c7a50020c801d569d55c002000410064006f0062006500200050004400460020bb38c11cb97c0020c791c131d569b2c8b2e4002e0020c774b807ac8c0020c791c131b41c00200050004400460020bb38c11cb2940020004100630072006f0062006100740020bc0f002000410064006f00620065002000520065006100640065007200200035002e00300020c774c0c1c5d0c11c0020c5f40020c2180020c788c2b5b2c8b2e4002egt13 LTH 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 LVI ltFEFF0049007a006d0061006e0074006f006a00690065007400200161006f00730020006900650073007400610074012b006a0075006d00750073002c0020006c0061006900200076006500690064006f00740075002000410064006f00620065002000500044004600200064006f006b0075006d0065006e007400750073002c0020006b006100730020006900720020012b00700061016100690020007000690065006d01130072006f00740069002000610075006700730074006100730020006b00760061006c0069007401010074006500730020007000690072006d007300690065007300700069006501610061006e006100730020006400720075006b00610069002e00200049007a0076006500690064006f006a006900650074002000500044004600200064006f006b0075006d0065006e007400750073002c0020006b006f002000760061007200200061007400760113007200740020006100720020004100630072006f00620061007400200075006e002000410064006f00620065002000520065006100640065007200200035002e0030002c0020006b0101002000610072012b00200074006f0020006a00610075006e0101006b0101006d002000760065007200730069006a0101006d002egt13 NLD (Gebruik deze instellingen om Adobe PDF-documenten te maken die zijn geoptimaliseerd voor prepress-afdrukken van hoge kwaliteit De gemaakte PDF-documenten kunnen worden geopend met Acrobat en Adobe Reader 50 en hoger)13 NOR 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 POL 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 PTB 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 RUM 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 RUS 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 SKY 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 SLV 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 SUO 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 SVE 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 TUR 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 UKR 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 ENU (Use these settings to create Adobe PDF documents best suited for high-quality prepress printing Created PDF documents can be opened with Acrobat and Adobe Reader 50 and later)13 gtgt13 Namespace [13 (Adobe)13 (Common)13 (10)13 ]13 OtherNamespaces [13 ltlt13 AsReaderSpreads false13 CropImagesToFrames true13 ErrorControl WarnAndContinue13 FlattenerIgnoreSpreadOverrides false13 IncludeGuidesGrids false13 IncludeNonPrinting false13 IncludeSlug false13 Namespace [13 (Adobe)13 (InDesign)13 (40)13 ]13 OmitPlacedBitmaps false13 OmitPlacedEPS false13 OmitPlacedPDF false13 SimulateOverprint Legacy13 gtgt13 ltlt13 AddBleedMarks false13 AddColorBars false13 AddCropMarks false13 AddPageInfo false13 AddRegMarks false13 ConvertColors ConvertToCMYK13 DestinationProfileName ()13 DestinationProfileSelector DocumentCMYK13 Downsample16BitImages true13 FlattenerPreset ltlt13 PresetSelector MediumResolution13 gtgt13 FormElements false13 GenerateStructure false13 IncludeBookmarks false13 IncludeHyperlinks false13 IncludeInteractive false13 IncludeLayers false13 IncludeProfiles false13 MultimediaHandling UseObjectSettings13 Namespace [13 (Adobe)13 (CreativeSuite)13 (20)13 ]13 PDFXOutputIntentProfileSelector DocumentCMYK13 PreserveEditing true13 UntaggedCMYKHandling LeaveUntagged13 UntaggedRGBHandling UseDocumentProfile13 UseDocumentBleed false13 gtgt13 ]13gtgt setdistillerparams13ltlt13 HWResolution [2400 2400]13 PageSize [612000 792000]13gtgt setpagedevice13

Page 3: February 2014 MEE Questions and AnalysesPreface The Multistate Essay Examination (MEE) is developed by the National Conference of Bar Examiners (NCBE). This publication includes the

Contents

Prefaceii

Description of the MEE ii

Instructionsiii

February 2014 Questions

Constitutional Law Question 3

Trusts and Future Interests Question 4

Secured Transactions Question 5

Federal Civil Procedure Question 6

Criminal Law and Procedure Question 7

Agency and Partnership Question 8

February 2014 Analyses

Constitutional Law Analysis 11

Trusts and Future Interests Analysis 14

Secured Transactions Analysis 17

Federal Civil Procedure Analysis 19

Criminal Law and Procedure Analysis 23

Agency and Partnership Analysis 27

i

Preface

The Multistate Essay Examination (MEE) is developed by the National Conference of Bar Examiners (NCBE) This publication includes the questions and analyses from the February 2014 MEE (In the actual test the questions are simply numbered rather than being identified by area of law) The instructions for the test appear on page iii

The model analyses for the MEE are illustrative of the discussions that might appear in excellent answers to the questions They are provided to the user jurisdictions to assist graders in grading the examination They address all the legal and factual issues the drafters intended to raise in the questions

The subjects covered by each question are listed on the first page of its accompanying analysis identified by roman numerals that refer to the MEE subject matter outline for that subject For example the Federal Civil Procedure question on the February 2014 MEE tested the following area from the Federal Civil Procedure outline IVD Pretrial proceduresmdashDiscovery (including e-discovery)

For more information about the MEE including subject matter outlines visit the NBCE website at wwwncbexorg

Description of the MEE

The MEE consists of six 30-minute essay questions and is a component of the Uniform Bar Examination (UBE) It is administered by participating jurisdictions on the Tuesday before the last Wednesday in February and July of each year The areas of law that may be covered by the questions on any MEE are Business Associations (Agency and Partnership Corporations and Limited Liability Companies) Conflict of Laws Constitutional Law Contracts Criminal Law and Procedure Evidence Family Law Federal Civil Procedure Real Property Torts Trusts and Estates (Decedentsrsquo Estates Trusts and Future Interests) and Uniform Commercial Code (Negotiable Instruments and Bank Deposits and Collections Secured Transactions) Some questions may include issues in more than one area of law The particular areas covered vary from exam to exam

The purpose of the MEE is to test the examineersquos ability to (1) identify legal issues raised by a hypothetical factual situation (2) separate material which is relevant from that which is not (3) present a reasoned analysis of the relevant issues in a clear concise and well-organized composition and (4) demonstrate an understanding of the fundamental legal principles relevant to the probable solution of the issues raised by the factual situation The primary distinction between the MEE and the Multistate Bar Examination (MBE) is that the MEE requires the examinee to demonstrate an ability to communicate effectively in writing

ii

Instructions The back cover of each test booklet contains the following instructions

You will be instructed when to begin and when to stop this test Do not break the seal on this booklet until you are told to begin

You may answer the questions in any order you wish Do not answer more than one question in each answer booklet If you make a mistake or wish to revise your answer simply draw a line through the material you wish to delete

If you are using a laptop computer to answer the questions your jurisdiction will provide you with specific instructions

Read each fact situation very carefully and do not assume facts that are not given in the question Do not assume that each question covers only a single area of the law some of the questions may cover more than one of the areas you are responsible for knowing

Demonstrate your ability to reason and analyze Each of your answers should show an understanding of the facts a recognition of the issues included a knowledge of the applicable principles of law and the reasoning by which you arrive at your conclusions The value of your answer depends not as much upon your conclusions as upon the presence and quality of the elements mentioned above

Clarity and conciseness are important but make your answer complete Do not volunteer irrelevant or immaterial information

Answer all questions according to generally accepted fundamental legal principles unless your testing jurisdiction has instructed you to answer according to local case or statutory law

NOTE Examinees testing in UBE jurisdictions must answer according to generally accepted fundamental legal principles rather than local case or statutory law

iii

February 2014 MEE

QUESTIONS Constitutional Law

Trusts and Future Interests Secured Transactions

Federal Civil Procedure Criminal Law and Procedure

Agency and Partnership

CONSTITUTIONAL LAW QUESTION ___________

A city ordinance required each downtown business to install high-powered halogen floodlights that would illuminate the property owned by that business and the adjoining sidewalks A study commissioned by the city estimated that installation of the floodlights would cost a typical business about $1000 but that increased business traffic due to enhanced public safety especially after dark would likely offset this cost

A downtown restaurant applied to the city for a building permit to construct an addition that would increase its seating capacity In its permit application the restaurant accurately noted that its current facility did not have sufficient seating to accommodate all potential customers during peak hours The city approved the permit on the condition that the restaurant grant the city an easement over a narrow strip of the restaurantrsquos property to be used by the city to install video surveillance equipment that would cover nearby public streets and parking lots The city based its permit decision entirely on findings that the increased patronage that would result from the increased capacity of the restaurant might also attract additional crime to the neighborhood and that installing video surveillance equipment might alleviate that problem

The restaurant has challenged both the ordinance requiring it to install floodlights and the easement condition imposed on approval of the building permit

1 Under the Fifth Amendment as applied to the states through the Fourteenth Amendment is the city ordinance requiring the restaurant to install floodlights an unconstitutional taking Explain

2 Under the Fifth Amendment as applied to the states through the Fourteenth Amendment is the cityrsquos requirement that the restaurant grant the city an easement as a condition for obtaining the building permit an unconstitutional taking Explain

3

TRUSTS AND FUTURE INTERESTS QUESTION _______________

Ten years ago a testator died survived by his only children a son age 26 and a daughter age 18

A testamentary trust was created under the testatorrsquos duly probated will The will specified that all trust income would be paid to the son during the sonrsquos lifetime and that upon the sonrsquos death the trust would terminate and trust principal would be distributed to the testatorrsquos ldquograndchildren who shall surviverdquo the son The testator provided for his daughter in other sections of the will

Five years ago the trustee of the testamentary trust purchased an office building with $500000 from the trust principal Other than this building the trust assets consist of publicly traded securities

Last year the trustee received $30000 in rents from the office building The trustee also received with respect to the securities owned by the trust cash dividends of $20000 and a stock dividend of 400 shares of Acme Corp common stock distributed to the trust by Acme Corp

Eight months ago the trustee sold the office building for $700000

Six months ago the son delivered a letter to the trustee stating ldquoI hereby disclaim any interest I may have in the income interest of the trustrdquo On the date the son delivered this letter to the trustee the son had no living children the daughter had one living minor child

A statute in this jurisdiction provides that ldquoa disclaimer of any interest created by will is valid only if made within nine months after the testatorrsquos death and if an interest is validly disclaimed the disclaiming party is deemed to have predeceased the testatorrdquo

1 How should the rents sales proceeds cash dividends and stock dividends received prior to the trusteersquos receipt of the sonrsquos letter have been allocated between trust principal and income Explain

2 How if at all does the sonrsquos letter to the trustee affect the future distribution of trust income and principal Explain

4

SECURED TRANSACTIONS QUESTION

On March 1 the owner of a manufacturing business entered into negotiations with a bank to obtain a loan of $100000 for the business The bank loan officer informed the business owner that the interest rate for a loan would be lower if the repayment obligation were secured by all the businessrsquos present and future equipment The loan officer also informed the business owner that the bank could not commit to making the loan until its credit investigation was completed but that funds could be advanced faster following loan approval if a financing statement with respect to the transaction were filed in advance Accordingly the business owner signed a form on behalf of the business authorizing the bank to file a financing statement with respect to the proposed transaction The bank properly filed a financing statement the next day correctly providing the name of the business as the debtor and indicating ldquoequipmentrdquo as the collateral

On March 15 the business owner had heard nothing from the bank about whether the loan had been approved so the business owner approached a finance company for a loan The finance company quickly agreed to lend $100000 to the business secured by all the businessrsquos present and future equipment That same day the finance company loaned to the business $100000 and the business owner signed an agreement obligating the business to repay the loan and granting the finance company a security interest in all the businessrsquos ldquopresent and future equipmentrdquo to secure the repayment obligation Also on that day the finance company properly filed a financing statement correctly providing the businessrsquos name as the debtor and indicating ldquoequipmentrdquo as the collateral

On March 21 the bank loan officer contacted the business owner and indicated that the loan application had been approved On the next day March 22 the bank loaned the business $100000 The loan agreement signed by the owner on behalf of the business granted the bank a security interest in all the businessrsquos ldquopresent and future equipmentrdquo

On April 10 the business sold an item of manufacturing equipment to a competitor for $20000 This was the first time the business had ever sold any of its equipment The competitor paid the purchase price in cash and took possession of the equipment that day The competitor acted in good faith at all times and had no knowledge of the businessrsquos prior transactions with the bank and the finance company

The business has defaulted on its obligations with respect to the loans from the bank and the finance company Each of them has asserted a claim to all the businessrsquos equipment as well as to the item of equipment sold to the businessrsquos competitor

Assume that the business owner had the authority to enter into all these transactions on behalf of the business

1 As between the bank and the finance company which has a superior claim to the businessrsquos equipment Explain

2 Do the claims of the bank and the finance company to the businessrsquos equipment continue in the item of equipment sold to the competitor Explain

5

FEDERAL CIVIL PROCEDURE QUESTION

A builder constructed a vacation house for an out-of-state customer on the customerrsquos land The house was completed on June 1 at which point the customer still owed $200000 of the $800000 contract price which was payable in full five days later

On June 14 the basement of the house was flooded with two inches of water during a heavy rainfall When the customer complained the builder told the customer ldquoThe flooding was caused by poorly designed landscaping Our work is fine and fully up to code Have an engineer look at the foundation If therersquos a problem wersquoll fix itrdquo

The customer pleased by the builderrsquos cooperative attitude immediately hired a structural engineer to examine the foundation of the house On June 30 the engineer provided the customer with a written report on the condition of the foundation which stated that the foundation was properly constructed

Unhappy with the conclusions in the engineerrsquos report the customer then hired a home inspector to evaluate the house The home inspectorrsquos report concluded that the foundation of the house had been poorly constructed and was inadequately waterproofed

On July 10 the customer sent the builder the home inspectorrsquos report with a note that said ldquoUntil you fix this problem you wonrsquot get another penny from merdquo The builder immediately contacted an attorney and directed the attorney to prepare a draft complaint against the customer for nonpayment Hoping to avoid litigation the builder sent several more requests for payment to the customer The customer ignored all these requests

On September 10 the builder filed suit in federal district court properly invoking the courtrsquos diversity jurisdiction and seeking $200000 in damages for breach of contract The customerrsquos answer denied liability on the basis of alleged defective construction of the housersquos foundation

Several months later the case is nearly ready for trial However two discovery disputes have not yet been resolved

First despite a request from the builder the customer has refused to provide a copy of the report prepared by the structural engineer who examined the foundation of the house The customer claims that the report is ldquowork productrdquo and not discoverable because the customer does not intend to ask the engineer to testify at trial The builder has asked the court to order the customer to turn over the engineerrsquos report

Second the customer has asked the court to impose sanctions for the builderrsquos failure to comply with the customerrsquos demand for copies of all emails concerning construction of the foundation of the house The builder has truthfully informed the customer that all such emails were destroyed on August 2 This destruction was pursuant to the builderrsquos standard practice of permanently deleting all project-related emails from company records 60 days after construction of a project is complete There is no relevant state records-retention law

1 Should the court order the customer to turn over the engineerrsquos report Explain

2 Should the court sanction the builder for the destruction of emails related to the case and if so what factors should the court consider in determining those sanctions Explain

6

CRIMINAL LAW AND PROCEDURE QUESTION _____

A defendant was charged under state law with felony theft (Class D) and felony residential burglary (Class C) The indictment alleged that the defendant entered his neighborsrsquo home without their consent and stole a diamond ring worth at least $2500

Defense counsel filed a pretrial motion to dismiss the charges on the ground that prosecuting the defendant for both burglary and theft would constitute double jeopardy The trial court denied the motion and the defendant was prosecuted for both crimes The only evidence of the ringrsquos value offered at the defendantrsquos jury trial was the ownerrsquos testimony that she had purchased the ring two years earlier for $3000

At trial the judge issued the following jury instruction on the burglary charge prior to deliberations

If after consideration of all the evidence presented by the prosecution and defense you find beyond a reasonable doubt that the defendant entered the dwelling without the ownersrsquo consent you may presume that the defendant entered with the intent to commit a felony therein

The jury found the defendant guilty of both offenses

At the defendantrsquos sentencing hearing an expert witness called by the prosecutor testified that the diamond ring was worth between $7000 and $8000 Over defense objection the judge concluded by a preponderance of the evidence that the value of the stolen ring exceeded $5000 The judge sentenced the defendant to four yearsrsquo incarceration on the theft conviction On the burglary conviction the defendant received a consecutive sentence of seven yearsrsquo incarceration

In this state residential burglary is defined as ldquoentry into the dwelling of another without the consent of the lawful resident with the intent to commit a felony thereinrdquo Residential burglary is a Class C felony for which the minimum sentence is five years and the maximum sentence is ten years of incarceration

In this state theft is defined as ldquotaking and carrying away the property of another with the intent to permanently deprive the owner of possessionrdquo Theft is a Class D felony if the value of the item(s) taken is between $2500 and $10000 The sentence for a Class D felony theft is determined by the value of the items taken If the value is between $2500 and $5000 the maximum sentence is three yearsrsquo incarceration If the value of the items exceeds $5000 the maximum sentence is five yearsrsquo incarceration

This state affords a criminal defendant no greater rights than those mandated by the United States Constitution

1 Did the trial court err when it denied the defendantrsquos pretrial motion to dismiss on double jeopardy grounds Explain

2 Did the trial court err in its instruction to the jury on the burglary charge Explain

3 Did the trial court err when it sentenced the defendant to an additional year of incarceration on the theft conviction based on the expertrsquos testimony Explain

7

AGENCY AND PARTNERSHIP QUESTION _____

Five years ago Adam and Ben formed a general partnership Empire Partnership (Empire) to buy and sell antique automobiles at a showroom in State A Adam contributed $800000 to Empire and Ben contributed $200000 Their written partnership agreement allocated 80 of profits losses and control to Adam and 20 to Ben No filings of any type were made in connection with the formation of Empire

Three years ago a collector purchased one of Empirersquos antique cars for $3400000 The collector was willing to pay this price because of Benrsquos false representation (repeated in the sales contract) that a famous movie star had once owned the car Without the movie-star connection the car was worth only $100000 One month later when the collector discovered the truth he sued Adam Ben and Empire for $3300000 in damages The lawsuit is still pending

Two years ago Adam and Ben admitted a new partner Diane to Empire in return for her contribution of $250000 The three agreed to allocate profits losses and control 75 to Adam 10 to Ben and 15 to Diane Before joining the partnership Diane learned of the collectorrsquos claim and stated her concern to Adam and Ben that she might become liable if the claim were reduced to a judgment

Following Dianersquos admission to Empire the three partners sought to convert Empire into a limited liability partnership (LLP) Adamrsquos lawyer proposed to file with State A a ldquostatement of qualificationrdquo making an LLP election and declaring the name of the partnership to be ldquoEmpire LLPrdquo Benrsquos lawyer stated that this would not work and that a new LLP had to be formed with the assets of the old partnership transferred to the new one In the end the conversion was done the way Adamrsquos lawyer suggested with the approval of all three partners

One year ago a driver purchased a vintage car from Empire LLP based on the representation that the car was ldquofully roadworthy and capable of touring at 70 mph all dayrdquo The driver took the car on the highway at 50 mph whereupon the front suspension collapsed resulting in a crash in which the car was destroyed and the driver killed The driverrsquos estate sued Adam Ben Diane and Empire LLP for $10000000 The lawsuit is still pending

Although profitable Empire LLP does not have resources sufficient to pay the collectorrsquos claim or the claim of the driverrsquos estate

Assume that the Uniform Partnership Act (1997) applies

1 Before the filing of the statement of qualification (a) was Adam personally liable on the collectorrsquos claim Explain (b) was Diane personally liable on the collectorrsquos claim Explain

2 After the filing of the statement of qualification was Adam Ben or Diane personally liable as a partner on (a) the collectorrsquos claim or (b) the driverrsquos estatersquos claim Explain

8

February 2014 MEE

ANALYSES Constitutional Law

Trusts and Future Interests Secured Transactions

Federal Civil Procedure Criminal Law and Procedure

Agency and Partnership

CONSTITUTIONAL LAW ANALYSIS (Constitutional Law IVD)

ANALYSIS

Legal Problems

(1) Is the city ordinance requirement that businesses install floodlights a taking

(2) Is conditioning the approval of a building permit on the grant of an easement to install surveillance equipment a taking of property

DISCUSSION

Summary

The ordinance requiring businesses to install floodlights is not a per se taking under Loretto because it does not force a private landowner to allow a third party to enter and place a physical object on the land Here the city ordinance requires the businessmdashnot a third partymdashto install the floodlights

The ordinance is likely not a regulatory taking under the Penn Central balancing test While the ordinance will impose a cost on business owners that cost may be offset by the expected increase in business due to the ordinance and the ordinance does not appear to interfere with the ownerrsquos primary use of the property as a restaurant

The permit condition however is likely an uncompensated taking of property While the condition has an essential nexus with the cityrsquos legitimate interest in promoting public safety the city has not made an individualized determination that the easement condition is roughly proportional to the possibility of increased crime due to the restaurantrsquos proposed addition Thus the permit condition likely violates the Fifth Amendment as applied to the states through the Fourteenth Amendment

Point One (50) The ordinance requiring that businesses install floodlights is not a per se taking under Loretto It is not a regulatory taking under the Penn Central balancing test because the cost of compliance with the ordinance may be offset by an expected increase in business and compliance does not interfere with the businessrsquos primary use of its property as a restaurant

The city ordinance requiring a business to install floodlights does not effect a per se taking of the sort described in Loretto v Teleprompter Manhattan CATV Corp 458 US 419 (1982) because no property is physically taken by the government and the ordinance does not involve a physical invasion of private property by a third party

Even though the ordinance does not constitute an occupation of the property by either the government or a third party it is still subject to the three-factor balancing test under Penn Central Transportation Co v City of New York 438 US 104 (1978) to determine whether it is a ldquoregulatory takingrdquo Under Penn Central a court must balance (1) ldquo[t]he economic impact of the regulation on the claimantrdquo (2) ldquothe extent to which the regulation has interfered with distinct investment-backed expectationsrdquo and (3) ldquothe character of the governmental actionrdquo Id at 124 Here each factor weighs against finding that the ordinance is a taking

11

Constitutional Law Analysis

First the ordinance requirement likely has a minimal economic impact on the restaurant Compliance with the ordinance is estimated to cost $1000 and the city has found that businesses will likely recoup that cost in increased sales Also because the ordinance does not interfere with the operation of the restaurant the owner may still earn a reasonable return on its investment in the property

Second the ordinance does not interfere with the businessrsquos investment-backed expectations As in Penn Central the challenged law does not interfere with the ownerrsquos ldquoprimary expectationrdquo for use of the propertymdashin Penn Central as a railroad terminal and here as a restaurant Further the ordinance does not prevent the restaurant from expanding to meet the changing business environment

Third the character of the government action does not weigh in favor of a taking While Penn Central does say that a ldquophysical invasionrdquo is more likely to pose a taking Loretto suggests that the Courtrsquos main concern is with physical invasions by third parties Also like the landmark law challenged in Penn Central the ordinance here ldquoadjust[s] the benefits and burdens of economic life to promote the common goodrdquo Id In Penn Central the landmark law restricted development of the railroad terminal to promote the common interest in preserving historic landmarks Here the ordinance requires the businesses to install floodlights to promote the common interest in crime prevention and public safety

Because the ordinance is clearly a valid exercise of the police power it satisfies the takings clausersquos public-use requirement Kelo v City of New London 545 US 469 (2005)

In sum all three factors weigh against finding a taking under the Penn Central balancing test

Point Two (50) The permit condition may be unconstitutional as an uncompensated taking of property because the city has not made an individualized determination that the easement condition is roughly proportional to the impact of the restaurantrsquos proposed addition

In Dolan v City of Tigard 512 US 374 (1994) the Supreme Court set forth the test for determining whether an exaction imposed by a government in exchange for a discretionary benefit conferred by the government such as a condition on the approval of a building permit in this case constitutes an uncompensated taking under the Fifth Amendment The exaction is not a taking if (1) there is an ldquoessential nexusrdquo between the ldquopublic need or burdenrdquo to which the proposed development contributes and ldquothe permit condition exacted by the cityrdquo id at 386 and (2) the government makes ldquosome sort of individualized determination that the required dedication is [roughly proportional] both in nature and extent to the impact of the proposed developmentrdquo Id at 391 see also Nollan v California Coastal Commission 483 US 825 (1987)

Here the city likely can meet the nexus requirement In Dolan the landowner sought to double the size of its business which would have increased traffic on nearby roadways In exchange for approving the development the city sought an easement for a bike and pedestrian path The Court found the required nexus between the easement and the cityrsquos ldquoattempt to reduce traffic congestion by providing for alternative means of transportationrdquo 512 US at 387 Here a similar nexus likely exists between the requested easement and the cityrsquos interest in crime prevention and public safety Increased patronage and economic activity at the restaurant might attract additional crime to the area and the requested easement to install surveillance equipment would attempt to address that increased crime

12

Constitutional Law Analysis

The exaction here however may fail the second prong of the Dolan testmdashthat the exaction be roughly proportional to the anticipated impact of the requested development As noted the city in Dolan claimed that a bike and pedestrian path was needed to offset the increase in traffic due to the proposed doubling of the business The Court explained that the government must demonstrate that the additional traffic reasonably was related to the requested exaction and that the government must ldquomake some effort to quantify its findings in support of the dedication for the pedestrianbicycle pathway beyond the conclusory statement that it could offset some of the traffic demand generatedrdquo Id at 395 Here the city did not carry its burden The city simply speculates that increased patronage of the restaurant ldquomightrdquo increase crime and that the surveillance equipment ldquomightrdquo alleviate this increased crime Because the city has not made ldquosome effort to quantify its findingsrdquo in support of the easement it has not shown that the burden of the easement is roughly proportional to the benefits thought to flow from it

Thus the exaction appears to be an uncompensated taking of property in violation of the Fifth Amendment as applied to the states through the Fourteenth Amendment

13

TRUSTS AND FUTURE INTERESTS ANALYSIS ____ (Trusts and Future Interests IE3 I5 IIIA amp B)

ANALYSIS

Legal Problems

(1) How should rents dividends and sales proceeds received by the trustee prior to receipt of the sonrsquos letter have been allocated between trust income and principal

(2)(a) Did the remainder interest in the trust accelerate and become immediately payable to the daughterrsquos minor child upon the trusteersquos receipt of the sonrsquos letter and if not how should the trustee handle the distribution of the principal in the future

(2)(b) Following the trusteersquos receipt of the sonrsquos letter how should the trustee distribute future receipts of income prior to the distribution of the principal

DISCUSSION

Summary

Prior to the trusteersquos receipt of the sonrsquos letter cash dividends and rents should have been allocated to trust income and were distributable to the son the income beneficiary of the trust sales proceeds and stock dividends should have been allocated to principal

Because the sonrsquos letter to the trustee did not result in a valid disclaimer under state law (having been made more than nine months after the testatorrsquos death) the son is not deemed to have predeceased the testator Because the son is still living the class gift to the testatorrsquos grandchildren who survive the son has not closed and is not possessory it will not become possessory until the son dies The daughterrsquos minor child being the testatorrsquos only living grandchild is not currently entitled to a distribution of trust principal Trust principal will instead be distributable upon the sonrsquos death to the testatorrsquos then-living grandchildren or if there are none to the testatorrsquos then-living heirs

As for future income the trustee should either distribute the trust income to the son and the daughter as the testatorrsquos heirs accumulate the income for future distribution to those individuals ultimately entitled to the trust principal or distribute it to those presumptively entitled to the principal upon the sonrsquos death ie the daughterrsquos minor child

Point One (45) Cash dividends and rents are allocable to income sales proceeds and stock dividends are allocable to principal Items allocable to income for the period prior to the sonrsquos attempted disclaimer were distributable to the son

Receipts earned during the administration of a trust are allocable either to income or to principal Almost all states have adopted the most recent or an earlier version of the Uniform Principal and Income Act (the Act) which specifies how such receipts should be allocated

Under the Act rents (UNIF PRIN amp INC ACT (2000) sect 405 UNIF PRIN amp INC ACT (1962) sect 3(a)(1)) and cash dividends received from a corporation (UNIF PRIN amp INC ACT (2000) sect 401(b) UNIF PRIN amp INC ACT (1962) sect 6(d)) are allocable to income and are distributable to the income beneficiary of the trust

14

Trusts and Future Interests Analysis Sales proceeds (UNIF PRIN amp INC ACT (2000) sect 404(2) UNIF PRIN amp INC ACT (1962)

sect 3(b)(1)) and dividends paid in the stock of the distributing corporation (UNIF PRIN amp INC ACT (2000) sect 401(c)(1) UNIF PRIN amp INC ACT (1962) sect 3(b)(4)) are allocable to principal and added to the principal of the trust

Here the cash dividends and office building rents should have been allocated to income and until the trustee received the sonrsquos letter should have been distributed to him as the sole income beneficiary of the trust The stock dividend and proceeds from the sale of the office building should have been allocated to principal and held by the trustee for future distribution to the ultimate remaindermen of the trust

[NOTE The 2000 Uniform Principal and Income Act has been adopted in Alabama Arkansas Colorado Connecticut the District of Columbia Hawaii Idaho Iowa Kentucky Missouri Montana Nebraska New Mexico North Dakota Oregon South Dakota Utah and West Virginia]

Point Two(a) (45) Because the son did not disclaim within nine months of the testatorrsquos death there is no valid disclaimer under state law Therefore the son is not deemed to have predeceased the testator Furthermore because of the express survivorship contingency in the will the remainder in the trust does not accelerate and become distributable until the son in fact dies When the son dies the trust principal will be distributable to the testatorrsquos then-living grandchildren or if none then to the testatorrsquos then-living heirs

When a trust remainder is given to a class the class closes (ie no new persons can join the class) when there is no outstanding income interest and at least one member of the class is then entitled to demand possession of his or her share of the remainder This principle is called the rule of convenience See generally HERBERT HOVENKAMP amp SHELDON F KURTZ PRINCIPLES OF PROPERTY LAW 199ndash200 (6th ed 2005) A class member may demand possession of his or her share of the remainder upon termination of the income interest only when the class memberrsquos interest is not otherwise subject to a condition precedent See id

When a beneficiary timely disclaims an interest in a trust that beneficiary is treated as if he had predeceased the testator Here had the son disclaimed within nine months of the testatorrsquos death as required by the state statute he would have been deemed to have predeceased the testator This would have closed the class of remaindermen and the testatorrsquos then-living grandchildren (ie the daughterrsquos child) would have been entitled to the trust principal However under the state statute the sonrsquos disclaimer was not timely because he did not disclaim within nine months of the testatorrsquos death Thus because the statute is inapplicable and the son is still alive the class of grandchildren entitled to share in trust principal did not close

Because here the statute is inapplicable due to the sonrsquos failure to comply with the statutory time requirements then presumably the common-law rule allowing disclaimers (aka renunciations) at any time should apply Under the common law if a life estate is renounced the remainder interest accelerates and becomes immediately distributable to the remaindermen of the trust if the remainder is vested but not if the remainder is contingent JESSE DUKEMINIER amp ROBERT H SITKOFF WILLS TRUSTS AND ESTATES 844ndash845 (9th ed 2013) Here because the remainder is contingent upon there being grandchildren who survive the son the remainder will not accelerate It will remain open until the son dies leaving open the possibility that additional grandchildren will be included in the class or the daughterrsquos child could fall out of the class because that child fails to survive the son

And if none of the testatorrsquos grandchildren survive the son the trust principal will be distributed to the testatorrsquos heirs living at the sonrsquos death

15

Trusts and Future Interests Analysis

Point Two(b) (10) Until the trust terminates the trustee must continue to hold the trust assets The distribution of income in the meantime is unclear There are at least three possibilities Income earned on the undistributed assets could be distributed to the son and daughter as the testatorrsquos heirs accumulated and added to principal for distribution to the ultimate remaindermen or distributed from time to time to those persons who are presumptively remaindermen

When trust principal is not immediately distributable the trustee must continue to hold trust assets until the ultimate remaindermen are ascertained During this period trust income will be distributed or retained according to any instructions contained in the trust instrument See WILLIAM M MCGOVERN JR SHELDON F KURTZ amp DAVID M ENGLISH WILLS TRUSTS amp ESTATES sect 102 (4th ed 2010)

Here the testator did not specify what the trustee should do with trust income in the event the sonrsquos disclaimer did not comply with the state statute There are at least three approaches One approach would have the trustee distribute the trust income to the testatorrsquos heirs on the theory that the income represents property that was not disposed of by the testatorrsquos will and which thus passes by partial intestacy to the testatorrsquos heirs A second approach would have the trustee accumulate trust income for distribution to the ultimate remaindermen Under this approach only those individuals ultimately entitled to the principal would share in the income A third approach would have the trustee distribute trust income to those individuals who would be the remaindermen if the trust were to terminate when the income is received by the trustee under this approach trust income would be distributed to the daughterrsquos minor child until another presumptive remainderman is born This approach could result in individuals not ultimately entitled to principal say because they do not survive the son receiving income It could also result in a disproportionate distribution of income among the individuals ultimately entitled to income

[NOTE Examinees should demonstrate a recognition and understanding of the income-allocation problem and the alternatives available to address that issue There is no widely accepted solution to the problem Examinees who cite any of these possible problem-solving approaches may receive credit]

16

SECURED TRANSACTIONS ANALYSIS (Secured Transactions IB IID E amp F IIIB IVA B amp F)

ANALYSIS

Legal Problems

(1)(a) What is the nature of the bankrsquos claim to the businessrsquos equipment

(1)(b) What is the nature of the finance companyrsquos claim to the businessrsquos equipment

(1)(c) As between the bank and the finance company whose claim to the businessrsquos equipment has priority

(2) Do the claims of the bank and the finance company continue in the item of equipment sold by the business to the competitor

DISCUSSION

Summary

The bank and the finance company both have perfected security interests in the businessrsquos equipment Even though the finance companyrsquos perfected security interest was created first the bankrsquos perfected security interest has priority because the bankrsquos financing statement was filed before the finance companyrsquos financing statement The security interests of the bank and the finance company continue in the item of equipment sold by the business to the competitor because their security interests were perfected and the competitor was not a buyer in ordinary course of business

Point One(a) (25) The bank has a perfected security interest in the businessrsquos equipment

The bank has met all criteria necessary for it to have an attached and enforceable security interest in the businessrsquos equipment First value must be given UCC sect 9-203(b)(1) This criterion is fulfilled by the loan made by the bank to the business Second the debtor must have rights in the collateral UCC sect 9-203(b)(2) Clearly the business has rights in its equipment Third either the secured party must take possession of the collateral or the debtor must authenticate a security agreement containing a description of the collateral UCC sect 9-203(b)(3) The agreement that the business owner signed is a ldquosecurity agreementrdquo because it is an agreement that creates or provides for a security interest UCC sect 9-102(a)(74) By signing the security agreement the business owner authenticated it UCC sect 9-102(a)(7) Therefore all three criteria are fulfilled and the bank has an enforceable and attached security interest

A security interest is perfected when it has attached and when any additional steps required for perfection have occurred UCC sect 9-308(a) Generally speaking the additional steps will either be possession of the collateral by the secured party or the filing of a financing statement with respect to the collateral See UCC sectsect 9-310 9-313 In this case the bank filed a financing statement naming the debtor and sufficiently indicating the collateral The collateral indication is sufficient because it identifies the collateral by type of property See UCC sectsect 9-504 9-108 The fact that the financing statement was filed before the security interest was created is

17

Secured Transactions Analysis

not a problem Even though the security agreement had not yet been signed the business had authorized the filing of the financing statement in an authenticated record UCC sect 9-509(a)(1) Moreover the financing statement may be filed before the security agreement is created UCC sect 9-502(d)

Point One(b) (10) The finance company also has a perfected security interest in the businessrsquos equipment

The finance companyrsquos security interest is enforceable and attached for the same reasons as the bankrsquos security interest The loan from the finance company to the business constitutes value the business has rights in the collateral and the business owner has authenticated a security agreement containing a description of the collateral The finance companyrsquos security interest is perfected because the finance company filed a financing statement with respect to it that provides that the business is the debtor and indicates that the collateral is equipment

Point One(c) (30) The bankrsquos security interest has priority over the finance companyrsquos security interest because the bankrsquos financing statement was filed first

As between two perfected security interests the general rule is that the security interest that was the earlier to be either perfected or the subject of a filed financing statement has priority UCC sect 9-322(a)(1) While the finance companyrsquos security interest was perfected before the bankrsquos (March 15 vs March 22) the bankrsquos financing statement was filed even earlier on March 2 Thus under the first-to-file-or-perfect rule of UCC sect 9-322(a)(1) the bankrsquos security interest has priority No exceptions to the general rule apply here

Point Two (35) A security interest in collateral continues notwithstanding its sale unless an exception applies Because the security interests of the bank and the finance company were perfected and the competitor was not a buyer in ordinary course of business no exception applies and the security interests of both creditors continue in the equipment sold to the competitor

As a general rule a security interest in collateral continues notwithstanding the fact that the debtor has sold the collateral to another person UCC sect 9-315(a)(1) Thus unless an exception applies the security interests of the bank and the finance company will continue in the item of equipment sold to the competitor

A buyer of goods will take free of an unperfected security interest in those goods See UCC sect 9-317(a)(2) However when the competitor bought the businessrsquos equipment both the bank and the finance company had perfected security interests in the equipment

A buyer can take free even of a perfected security interest in goods if the buyer is a ldquobuyer in ordinary course of businessrdquo See UCC sect 9-320(a) However the competitor was not a buyer in ordinary course of business To be a ldquobuyer in ordinary course of businessrdquo a buyer must buy goods from a seller that is in the business of selling goods of that kind See UCC sect 1-201(b)(9) The competitor bought this equipment from a seller that is not in the business of selling goods of this kind so the competitor was not a buyer in ordinary course of business with respect to these goods

Because no exception applies the security interests of the bank and the finance company continue even after the item of equipment was sold to the competitor

18

FEDERAL CIVIL PROCEDURE ANALYSIS (Federal Civil Procedure IVD)

ANALYSIS

Legal Problems

(1) Is a document prepared in the course of a contract dispute protected from discovery as ldquowork productrdquo when there is no evidence that the document was prepared in anticipation of litigation

(2)(a) Is a partyrsquos failure to provide relevant electronically stored information excused when the information was destroyed pursuant to a routine document retention scheme at a time when litigation was contemplated by the destroying party

(2)(b) What sanctions should be imposed on a party for allowing the destruction of evidence that is relevant to potential future litigation

DISCUSSION

Summary

The report prepared by the structural engineer is probably not work product and is thus discoverable The engineer examined the foundation of the house at the customerrsquos request and the engineerrsquos findings are potentially relevant to the customerrsquos claim that the foundation is defective The report was not prepared in anticipation of litigation The customer appears to have sought the engineerrsquos opinion in response to the builderrsquos offer to fix any problems with the foundation that an engineer might identify Because the report was not prepared in anticipation of litigation it is not protected by the work-product doctrine

The builder should have taken appropriate steps to preserve evidence including suspending its document retention program as soon as it began planning for litigationmdashie on July 10 Its destruction of potentially relevant material after that date was wrongful However a court is unlikely to impose severe sanctions on the builder because there are no facts indicating that the builder acted in bad faith and the customer can prove that the foundation is defective without the destroyed emails

Point One (40) The customer must turn over the engineerrsquos report because it was not prepared in anticipation of litigation

In general a party to a lawsuit in federal court ldquomay obtain discovery regarding any nonprivileged matter that is relevant to any partyrsquos claim or defenserdquo FED R CIV P 26(b)(1) (2009) This includes the right to inspect and copy documents in the other partyrsquos possession FED R CIV P 34(a)(1) Here the customer hired a structural engineer to examine the foundation of the house The engineerrsquos report on the foundation is likely to include information that would be relevant to the customerrsquos claim that the foundation was defectively constructed

The so-called ldquowork productrdquo rule allows a party to refuse to turn over ldquodocuments that are prepared in anticipation of litigation or for trialrdquo by that partyrsquos representative including

19

Federal Civil Procedure Analysis

a consultant Thus if the customer had hired the structural engineer to prepare a report ldquoin anticipation of litigationrdquo that report might not be discoverable See FED R CIV P 26(b)(3)

In this case however the customer hired the engineer to evaluate the foundation of the house as part of the customerrsquos negotiation with the builder concerning the housersquos flooding problem The builder told the customer that the housersquos landscaping was the reason for the flooding and the builder told the customer ldquoHave an engineer look at the foundation If therersquos a problem wersquoll fix itrdquo The customer appears to have acted in response to that statement There is no indication that the customer anticipated any kind of legal action at the time that the structural engineer was hired Accordingly the structural engineerrsquos report is discoverable and the court should order the customer to turn it over

[NOTE If an examinee concludes that the structural engineerrsquos report was prepared in anticipation of litigation then the examinee should also conclude that the report is not discoverable Documents prepared in anticipation of litigation do not need to be disclosed to an adverse party unless that party can demonstrate a ldquosubstantial needrdquo for the documents and an inability to obtain substantially equivalent information without ldquoundue hardshiprdquo FED R CIV P 26(b)(3)(A)(ii) Furthermore a report prepared by an expert who is not expected to testify is not discoverable in the absence of ldquoexceptional circumstancesrdquo making it ldquoimpracticablerdquo to obtain the information in another way FED R CIV P 26(b)(4)(D)(ii) The builder probably cannot make these showings here unless the engineerrsquos report deals with circumstances that have since changed There is no evidence that the structural engineer would have had access to any information or facts that the builder would not already know as a result of its construction and subsequent inspection of the house In addition if necessary the builder could ask the court for permission to arrange for a further inspection of the house by a structural engineer hired by the builder See FED R CIV P 34(a)(2) Accordingly if an examinee concludes that the report was prepared in anticipation of litigation the examinee should also conclude that the builder is not entitled to see the report]

Point Two(a) (30) Because the builder anticipated that it might be involved in litigation concerning its contract with the customer the builder acted wrongfully in destroying emails that were relevant to the housersquos construction even though the emails were destroyed pursuant to a routine document retention plan

As noted above a party to a lawsuit in federal court ldquomay obtain discovery regarding any nonprivileged matter that is relevant to any partyrsquos claim or defenserdquo FED R CIV P 26(b)(1) This includes emails and other electronically stored information FED R CIV P 34(a)(1)(A) Here the customer has requested all the builderrsquos emails pertaining to work done on the foundation of the house Ordinarily the builder would be obliged to turn over this information which is relevant to the customerrsquos defense that the housersquos foundation was poorly constructed

Unfortunately the emails in question no longer exist because the builder destroyed them on August 2

In general spoliation of evidence (destruction or alteration of evidence) is improper if the party who destroyed or altered the evidence ldquohas notice that the evidence is relevant to litigation or should have known that the evidence may be relevant to future litigationrdquo Fujitsu Ltd v Federal Express Corp 247 F3d 423 436 (2d Cir 2001) It is improper for a party to destroy electronic information relevant to pending litigation even if the destruction occurs before there is any request or order seeking the information See eg Leon v IDX Sys Corp 464 F3d 951 (9th Cir 2006) (plaintiffrsquos intentional destruction of computer files warranted dismissal even

20

In this case the builderrsquos destruction of the emails was pursuant to a routine document retention plan The Federal Rules provide expressly that in the absence of ldquoexceptional circumstancesrdquo parties should not be sanctioned for the loss of electronically stored information when the loss occurs pursuant to ldquoroutine good-faith operation of an electronic information systemrdquo FED R CIV P 37(e) However when a party anticipates litigation ldquoit must suspend its routine document retentiondestruction policy and put in place a lsquolitigation holdrsquo to ensure the preservation of relevant documentsrdquo Zubulake v UBS Warburg LLC 220 FRD 212 218 (SDNY 2003)

Federal Civil Procedure Analysis

though spoliation occurred before order compelling discovery) Similarly the duty to preserve evidence applies to a party who anticipates litigation even if litigation has not yet been commenced See THE SEDONA PRINCIPLES BEST PRACTICES RECOMMENDATIONS amp PRINCIPLES FOR ADDRESSING ELECTRONIC DOCUMENT PRODUCTION 70 cmt 14a (2d ed 2007)

The builder destroyed the emails on August 2 At that time the builder knew that litigation was a possibility because the builder had already directed its attorney to prepare a draft complaint for possible filing Knowing that litigation was a possibility the builder had a duty to take steps to preserve evidence including the emails in question See generally Fujitsu Ltd

Thus the builderrsquos destruction of potentially relevant emails at a time when it knew that litigation was a possibility was improper It had a duty to preserve evidence and it breached that duty

[NOTE Because courts have used different words to describe the test for when evidence must be preserved an examineersquos precise formulation of the test is not critical]

Point Two(b) (30) In determining appropriate sanctions for spoliation courts consider both the level of culpability of the spoliating party and the degree of prejudice the loss of evidence has caused the other party Here the builderrsquos destruction of evidence does not appear to have been willful nor is it likely to pose a significant obstacle to the customerrsquos defense Any sanctions imposed by the court should be modest

Federal courts have inherent power to control the litigation process and can sanction misbehavior including spoliation even when there has been no specific violation of the Federal Rules of Civil Procedure See generally Chambers v NASCO Inc 501 US 32 (1991) (discussing courtrsquos inherent power to control the litigation process) The range of available sanctions is broad It can include such sanctions as the payment of expenses incurred by the other party as a result of the destruction of the evidence an instruction to the jury authorizing it to draw an adverse inference from the destruction of the evidence a shifting of the burden of proof on the relevant issue or even judgment against the responsible party See eg Residential Funding Corp v DeGeorge Financial Corp 306 F3d 99 108 (2d Cir 2002) (adverse inference) Silvestri v General Motors Corp 271 F3d 583 593 (4th Cir 2001) (possibility of dismissal) Cf FED R CIV P 37(b)(2)(A) (listing remedies for failure to comply with discovery obligations)

In determining appropriate sanctions for spoliation courts consider both the level of culpability of the spoliating party and the degree of prejudice the loss of evidence has caused the other party Many courts impose severe sanctions (such as an adverse-inference instruction or the entry of judgment against the spoliating party) only when there is evidence of bad faith in the form of an intentional effort to hide information Eg Greyhound Lines Inc v Wade 485 F3d 1032 1035 (8th Cir 2007) (spoliation sanction requires intentional destruction out of desire ldquoto suppress the truthrdquo) However other courts have said that negligence in preserving evidence can

21

Federal Civil Procedure Analysis

support an adverse-inference instruction See Residential Funding 306 F3d at 108 (negligence enough under some circumstances)

Although a court might well order an evidentiary hearing on the issue of sanctions the facts presented do not seem appropriate for severe sanctions First the evidence was destroyed pursuant to the builderrsquos standard document retention plan and there is no evidence that the builder deliberately failed to suspend its usual procedures with the purpose of allowing the destruction of evidence Second the loss of this evidence will not severely hinder the customerrsquos presentation of his case The central issue is whether the foundation of the house was properly constructed If the construction job was poorly done the customer can present evidence derived from inspection of the premises to prove that point The customer can also depose witnesses about any issues that arose during construction

Under the circumstances a court is not likely to impose particularly severe sanctions although it might shift the burden to the builder to show that the foundation was properly constructed or it might require the builder to reimburse any expenses the customer incurs to discover and prove the facts about issues or disputes that arose during construction of the foundation

[NOTE The result reached by the examinee is less important than the examineersquos recognition that (a) a range of sanctions is available to the court and (b) the appropriate sanction depends both on the culpability of the builder and the prejudice suffered by the customer]

22

CRIMINAL LAW AND PROCEDURE ANALYSIS (Criminal Law and Procedure IIA amp D VE amp F)

ANALYSIS

Legal Problems

(1) Did charging the defendant with both theft and burglary constitute double jeopardy

(2) Did the jury instruction violate the due process clause either by relieving the prosecution of the burden of proving the element of intent or by shifting the burden to the defendant to disprove that element

(3) Did the sentence imposed in this case for the theft conviction unconstitutionally deprive the defendant of his right to a jury trial on the issue of the value of the stolen item

DISCUSSION

Summary

The trial court properly denied the defendantrsquos pretrial motion to dismiss the charges on double jeopardy grounds The defendant may be charged with and convicted of both theft and burglary Each of the charges has an element that the other does not Neither charge is a lesser-included offense nor are they multiplicitous Thus charging both theft and burglary does not violate double jeopardy

The jury instruction on the burglary charge was constitutionally flawed It could have been reasonably understood by the jury as either (1) an irrebuttable conclusive presumption (which relieved the prosecution of proving the element of intent and removed the issue from the jury) or (2) a rebuttable mandatory presumption (which unconstitutionally shifted the burden of proof on an element of a charged offense from the prosecution to the defendant)

Because the four-year sentence imposed by the judge was based on the judgersquos finding by a preponderance of the evidence that the value of the stolen ring exceeded $5000 the sentence violates the defendantrsquos right to a jury determination beyond a reasonable doubt of the value of the ring

Point One (30) Charging the defendant with theft and burglary did not constitute double jeopardy

The Double Jeopardy Clause of the Fifth Amendment provides that a person shall not be twice put in jeopardy for the ldquosame offenserdquo Thus the question is whether the elements of the theft charge are wholly contained in the burglary charge or vice versa If the elements of the lesser charge (theft) are not wholly contained in the greater charge (burglary)mdashie if each charge requires proof of a fact that the other does notmdashthen convicting the defendant of both crimes would not violate double jeopardy even when the two offenses occurred at the same time and are thus arguably part of the ldquosame transactionrdquo Blockburger v United States 284 US 299 304 (1932) See also Albernaz v United States 450 US 333 344 n3 (1981) United States v Dixon 509 US 688 704 (1993)

23

Criminal Law and Procedure Analysis

Here theft and burglary each require proof of an element not required for the other crime Burglary may be defined differently in different jurisdictions However it almost invariably requires entry into a building or dwelling of another with the specific intent to commit a felony therein and the crime of burglary is complete upon the entry into the building or dwelling with such intent See eg Cannon v Oklahoma 827 P2d 1339 1342 (Okla Crim App 1992) In contrast theft which also may be defined differently in different states almost invariably requires the taking and carrying away of an item of personal property belonging to another with the intent to steal or permanently deprive the owner of possession

Here the ldquotakingrdquo or ldquostealingrdquo element is not contained in the definition of burglary and the ldquoentryrdquo element of burglary is not contained in the definition of theft Because theft is not a lesser-included offense of burglary and burglary is not a lesser-included offense of theft charging the defendant for both burglary and theft did not violate double jeopardy and the court properly denied the defense motion on those grounds Yparrea v Dorsey 64 F3d 577 579ndash80 (10th Cir 1995) citing Blockburger 284 US at 304

Finally the defendantrsquos motion to dismiss all the charges on double jeopardy grounds was improper because if both charges were for the same offense the motion should have requested dismissal of one charge not both

Point Two (35) The jury instruction on the burglary charge violated the Due Process Clause because it created either (1) an irrebuttable conclusive presumption (which relieved the prosecution of proving the element of intent and removed that issue from the jury) or (2) a rebuttable mandatory presumption (which unconstitutionally shifted the burden of proof on an element of a charged offense to the defendant)

The Supreme Court has interpreted the Due Process Clause of the US Constitution to require that the prosecution prove all elements of an offense beyond a reasonable doubt See In re Winship 397 US 358 364 (1970) The burden of proof cannot be shifted to the defendant by presuming an essential element upon proof of other elements of the offense because shifting the burden of persuasion with respect to any element of a criminal offense is contrary to the Due Process Clause See Mullaney v Wilbur 421 US 684 (1975)

The crime of burglary includes entry into a building or dwelling with the specific intent to commit a felony therein The requirement that the prosecutor prove beyond a reasonable doubt that the defendant had this specific intent distinguishes burglary from general-intent crimes like trespass See Sandstrom v Montana 442 US 510 523 (1979)

Here the jury was instructed that if ldquoafter consideration of all the evidence presented by the prosecution and defense you find beyond a reasonable doubt that the defendant entered the dwelling without the ownersrsquo consent you may presume that the defendant entered with the intent to commit a felony thereinrdquo This instruction was unconstitutional because it created either an irrebuttable conclusive presumption or a rebuttable mandatory presumption

A conclusive presumption is ldquoan irrebuttable direction by the court to find intent once convinced of the facts triggering the presumptionrdquo Id at 517 Here the jurors were instructed that once the prosecutor established that the defendant entered the neighborsrsquo house without consent they ldquomay presumerdquo that he intended to commit a felony therein The jurors may have reasonably concluded from this instruction that if they found that the defendant intended to enter his neighborsrsquo home without permission they must further find that he entered with the specific intent to commit a felony therein Because this instruction could operate as a conclusive

24

Criminal Law and Procedure Analysis

irrebuttable presumption by eliminating intent ldquoas an ingredient of the offenserdquo it violated due process by relieving the prosecution of the burden of proof for this element Id at 522

In the alternative the jury instruction could have been reasonably understood to create a rebuttable mandatory presumption which ldquotells [the jury] they must find the elemental fact upon proof of the basic fact at least unless the defendant has come forward with some evidence to rebut the presumed connection between the two factsrdquo County Court of Ulster County New York v Allen 442 US 140 157 (1979) The due process problem created by rebuttable mandatory presumptions is that ldquo[t]o the extent that the trier of fact is forced to abide by the presumption and may not reject it based on an independent evaluation of the particular facts presented by the State the analysis of the presumptionrsquos constitutional validity is logically divorced from those facts and based on the presumptionrsquos accuracy in the run of casesrdquo Id at 159

Unlike irrebuttable conclusive presumptions rebuttable mandatory presumptions are not always per se violations of the Due Process Clause However the Supreme Court of the United States has held that jury instructions that could reasonably be understood as shifting the burden of proof to the defendant on an element of the offense are unconstitutional Francis v Franklin 471 US 307 (1985) Here the argument that the jury instruction operated as a rebuttable mandatory presumption is supported by the fact that the judge also instructed the jury to ldquoconsider[ ] all the evidence presented by the prosecution and defenserdquo However even if the instruction created a rebuttable mandatory presumption it would be unconstitutional because it shifted the burden to the defense on an element of the offense Sandstrom 442 US at 524 Mullaney 421 US at 686

[NOTE Whether an examinee identifies the jury instruction as containing a ldquoconclusiverdquo or ldquomandatoryrdquo presumption is less important than the examineersquos analysis of the constitutional infirmities]

Point Three (35) The trial court violated the defendantrsquos Sixth Amendment right to a jury trial on an essential element of the offense when it found by a preponderance of the evidence that the ring was worth over $5000 and increased the defendantrsquos sentence based on this finding

In the statutory scheme under which the defendant was tried and convicted a Class D felony theft is defined as theft of item(s) with a value between $2500 and $10000 The jury found that the value of the diamond ring was at least $2500 and convicted the defendant of felony theft However at sentencing the trial court made a separate finding by a preponderance of the evidence that the value of the ring was greater than $5000 Following the statutersquos two-tiered sentencing scheme the judge then imposed on the defendant a sentence that was one year longer than the maximum that would otherwise have been allowed

The judgersquos sentence was unconstitutional because it violated the defendantrsquos Sixth Amendment right to a jury trial on this question The Supreme Court held in Apprendi v New Jersey 530 US 466 (2000) that ldquo[o]ther than the fact of a prior conviction any fact that increases the penalty for a crime beyond the prescribed statutory maximum must be submitted to a jury and proved beyond a reasonable doubtrdquo because ldquo[i]t is unconstitutional for a legislature to remove from the jury the assessment of facts that increase the prescribed range of penalties to which a criminal defendant is exposed [because] such facts must be established by proof beyond a reasonable doubtrdquo Id The Court reaffirmed Apprendi in Blakely v Washington 542 US 296 (2004) holding that the ldquolsquostatutory maximumrsquo for Apprendi purposes is the maximum sentence a judge may impose solely on the basis of the facts reflected in the jury verdict or admitted by the defendantrdquo Id at 303 (emphasis in original) In United States v Booker 543 US 220 (2005)

25

Criminal Law and Procedure Analysis

the Court relied on Blakely and Apprendi to conclude that protecting a defendantrsquos Sixth Amendment right to a jury trial required that ldquo[a]ny fact which is necessary to support a sentence exceeding the maximum authorized by the facts established by a plea of guilty or a jury verdict must be admitted by the defendant or proved to a jury beyond a reasonable doubtrdquo Id at 244

Thus in order to constitutionally increase a sentence above the statutory maximum of three years the jury must have found beyond a reasonable doubt that the value of the ring exceeded $5000 Here the court made the finding based on an appraisal proffered by the prosecutor only at sentencing and the judgersquos finding was by a preponderance of the evidence rather than beyond a reasonable doubt

26

AGENCY AND PARTNERSHIP ANALYSIS __________ (Agency and Partnership VA amp C VI)

ANALYSIS

Legal Problems

(1) Is a partner in a general partnership personally liable on a claim arising from misrepresentations by another partner made in the course of the partnership business

(2) Does a newly admitted partner in a general partnership become personally liable on existing claims against the partnership

(3) After the filing by a general partnership of a statement of qualification as a limited liability partnership are the partners personally liable as partners on (a) an existing claim against the general partnership and (b) a claim against the partnership that arose after the filing

DISCUSSION

Summary

Adam and Ben formed a general partnership under which they were jointly and severally liable for obligations of the partnership Thus Adam was personally liable for misrepresentations by Ben made in the ordinary course of the partnership business

Upon joining the general partnership Diane became personally liable for the obligations of the partnership arising after her admission but not for obligations pre-existing her admission such as the collectorrsquos claim

By filing a statement of qualification the three partners properly elected limited liability partnership status As partners in an LLP none of the three partners is personally liable as a partner for partnership obligations arising after the election such as the claim by the driverrsquos estate The election however does not change their personal liability on pre-existing claims that arose before the election such as the collectorrsquos claim

Point One (30) As a general partner of Empire a general partnership Adam became personally liable on the collectorrsquos claim a valid claim against the partnership that arose because of Benrsquos wrongful act in the ordinary course of the partnership business

When the collectorrsquos claim arose Empire was a general partnership composed of Adam and Ben Under UPA (1997) sect 306(a) partners of a general partnership are liable jointly and severally for all obligations of the partnership Under UPA (1997) sect 305(a) the partnership could become obligated for the loss caused to the collector as a result of the misrepresentation by Ben provided he was acting in the ordinary course of the partnership business Because there was no statement that limited his partnership authority Ben as partner was ldquoan agent of the partnership for the purpose of its businessrdquo See UPA (1997) sect 301(1) Benrsquos misrepresentation to the collector even if intentional appears to be in the ordinary course of the partnershiprsquos business of dealing

27

Agency and Partnership Analysis

in antique cars Thus Benrsquos wrongful act created a partnership obligation for which Adam was jointly and severally liable

[NOTE Generally a partnership creditor must ldquoexhaust the partnershiprsquos assets before levying on a judgment debtor partnerrsquos individual property where the partner is personally liable for the partnership obligationrdquo as a result of his status as a partner UPA (1997) sect 307 cmt 4 As the UPA comments explain this places Adam more in the position of guarantor than principal debtor on the partnership obligation Id cmt 4 Although an examinee might discuss this point the call focuses on whether Adam is personally liable not how the liability might be enforced]

Point Two (30) Because the collectorrsquos claim arose before Diane joined Empire Diane did not become personally liable on the claim

Diane was admitted to Empire when it was a general partnership and after the collectorrsquos claim arose While the general rule under UPA (1997) sect 306(a) is that the partners of a general partnership are liable jointly and severally for all obligations of the partnership there is a special rule for partners who are admitted during the duration of the partnership Under UPA (1997) sect 306(b) a person admitted to an existing partnership is not personally liable for any partnership obligations incurred before the personrsquos admission Because Diane was admitted to Empire after the collectorrsquos claim arose Diane is not personally liable on the claim

Dianersquos knowledge of the pre-existing claim and her stated concern about becoming liable on the collectorrsquos claim do not change her personal nonliability to the collector Although partners who have a liability shield can assume liability to third parties through private contractual guarantees or modifications to the partnership agreement Dianersquos stated concern constituted neither a guaranty to the collector nor ldquoan intentional waiver of liability protectionsrdquo See UPA (1997) sect 306 cmt 3 (describing methods for waiver of liability protections under sect 306(c) applicable in limited liability partnerships)

At most Diane will lose her investment in the partnership as a result of the collectorrsquos claim Although Diane did not become personally liable on the collectorrsquos claim when she joined the partnership the $250000 she contributed to the partnership is ldquoat risk for the satisfaction of existing partnership debtsrdquo UPA (1997) sect 306 cmt 2

Point Three (40) Filing the statement of qualification was effective to elect limited liability partnership status Despite this new status Adam and Ben remain personally liable on the collectorrsquos claim which arose before the election But as partners in an LLP neither Adam Ben nor Diane is personally liable as a partner on the driverrsquos estatersquos claim which arose after the election

Under UPA (1997) sect 1001 a general partnership can make an election and become a limited liability partnershipmdashif the partners approve the conversion by a vote equivalent to that necessary to amend the partnership agreement and the partnership then files a statement of qualification that specifies the name of the partnership its principal office and its election to be an LLP Here the partners agreed unanimouslymdashsufficient to amend their agreement under UPA (1997) sect 401(j)mdashand the statement of qualification was filed In addition the name of Empire LLP properly included an appropriate ending ldquoLLPrdquo See UPA (1997) sect 1002

Although another way to effectuate a ldquoconversionrdquo (as suggested by Benrsquos lawyer) is to form a new LLP and transfer the assets of the old general partnership to the new LLP the

28

Agency and Partnership Analysis

method used here (approval by the partners and the filing of a statement of qualification) is also sufficient to create LLP status

Thus Empire became Empire LLP as of the date of filing of the statement of qualification See UPA (1997) sect 1001 What effect did this have on the collectorrsquos claim which predated the filing According to UPA (1997) sect 306(c) an obligation incurred while a partnership is an LLP is solely a partnership obligation As the collectorrsquos claim predated the LLP Adam and Ben remain personally liable on the collectorrsquos claim Diane on the other hand was not personally liable on the collectorrsquos claim either before or after the filing of the statement of qualification See Point Two above

The driverrsquos estatersquos claim arose after Empire became Empire LLP Under UPA (1997) sect 306(c) an obligation incurred while a partnership is an LLP is solely a partnership obligationThus Adam Ben and Diane as partners are all protected from personal liability on the driverrsquos estatersquos claim But there may be personal liability if any of them was negligent or otherwise acted wrongfully by not informing the buyer of the bad suspension that caused the accident

29

National Conference of Bar Examiners 302 South Bedford Street | Madison WI 53703-3622 Phone 608-280-8550 | Fax 608-280-8552 | TDD 608-661-1275

wwwncbexorg e-mail contactncbexorg

  • Preface
  • Description of the MEE
  • Instructions
  • February 2014 Questions
    • Constitutinal Law Question
    • Trusts and Future Interests Question
    • Secured Transactions Question
    • Federal Civil Procedure Question
    • Criminal Law and Procedure Question
    • Agency and Partnership Question
      • February 2014 Analyses
        • Constitutional Law Analysis
        • Trust and Future Interests Analysis
        • Secured Transactions Analysis
        • Federal Civil Procedure Analysis
        • Criminal Law and Procedure Analysis
        • Agency and Partnership Analysis
            • ltlt13 ASCII85EncodePages false13 AllowTransparency false13 AutoPositionEPSFiles true13 AutoRotatePages None13 Binding Left13 CalGrayProfile (Dot Gain 20)13 CalRGBProfile (sRGB IEC61966-21)13 CalCMYKProfile (US Web Coated 050SWOP051 v2)13 sRGBProfile (sRGB IEC61966-21)13 CannotEmbedFontPolicy Error13 CompatibilityLevel 1413 CompressObjects Tags13 CompressPages true13 ConvertImagesToIndexed true13 PassThroughJPEGImages true13 CreateJobTicket false13 DefaultRenderingIntent Default13 DetectBlends true13 DetectCurves 0000013 ColorConversionStrategy CMYK13 DoThumbnails false13 EmbedAllFonts true13 EmbedOpenType false13 ParseICCProfilesInComments true13 EmbedJobOptions true13 DSCReportingLevel 013 EmitDSCWarnings false13 EndPage -113 ImageMemory 104857613 LockDistillerParams false13 MaxSubsetPct 10013 Optimize true13 OPM 113 ParseDSCComments true13 ParseDSCCommentsForDocInfo true13 PreserveCopyPage true13 PreserveDICMYKValues true13 PreserveEPSInfo true13 PreserveFlatness true13 PreserveHalftoneInfo false13 PreserveOPIComments true13 PreserveOverprintSettings true13 StartPage 113 SubsetFonts true13 TransferFunctionInfo Apply13 UCRandBGInfo Preserve13 UsePrologue false13 ColorSettingsFile ()13 AlwaysEmbed [ true13 ]13 NeverEmbed [ true13 ]13 AntiAliasColorImages false13 CropColorImages true13 ColorImageMinResolution 30013 ColorImageMinResolutionPolicy OK13 DownsampleColorImages true13 ColorImageDownsampleType Bicubic13 ColorImageResolution 30013 ColorImageDepth -113 ColorImageMinDownsampleDepth 113 ColorImageDownsampleThreshold 15000013 EncodeColorImages true13 ColorImageFilter DCTEncode13 AutoFilterColorImages true13 ColorImageAutoFilterStrategy JPEG13 ColorACSImageDict ltlt13 QFactor 01513 HSamples [1 1 1 1] VSamples [1 1 1 1]13 gtgt13 ColorImageDict ltlt13 QFactor 01513 HSamples [1 1 1 1] VSamples [1 1 1 1]13 gtgt13 JPEG2000ColorACSImageDict ltlt13 TileWidth 25613 TileHeight 25613 Quality 3013 gtgt13 JPEG2000ColorImageDict ltlt13 TileWidth 25613 TileHeight 25613 Quality 3013 gtgt13 AntiAliasGrayImages false13 CropGrayImages true13 GrayImageMinResolution 30013 GrayImageMinResolutionPolicy OK13 DownsampleGrayImages true13 GrayImageDownsampleType Bicubic13 GrayImageResolution 30013 GrayImageDepth -113 GrayImageMinDownsampleDepth 213 GrayImageDownsampleThreshold 15000013 EncodeGrayImages true13 GrayImageFilter DCTEncode13 AutoFilterGrayImages true13 GrayImageAutoFilterStrategy JPEG13 GrayACSImageDict ltlt13 QFactor 01513 HSamples [1 1 1 1] VSamples [1 1 1 1]13 gtgt13 GrayImageDict ltlt13 QFactor 01513 HSamples [1 1 1 1] VSamples [1 1 1 1]13 gtgt13 JPEG2000GrayACSImageDict ltlt13 TileWidth 25613 TileHeight 25613 Quality 3013 gtgt13 JPEG2000GrayImageDict ltlt13 TileWidth 25613 TileHeight 25613 Quality 3013 gtgt13 AntiAliasMonoImages false13 CropMonoImages true13 MonoImageMinResolution 120013 MonoImageMinResolutionPolicy OK13 DownsampleMonoImages true13 MonoImageDownsampleType Bicubic13 MonoImageResolution 120013 MonoImageDepth -113 MonoImageDownsampleThreshold 15000013 EncodeMonoImages true13 MonoImageFilter CCITTFaxEncode13 MonoImageDict ltlt13 K -113 gtgt13 AllowPSXObjects false13 CheckCompliance [13 None13 ]13 PDFX1aCheck false13 PDFX3Check false13 PDFXCompliantPDFOnly false13 PDFXNoTrimBoxError true13 PDFXTrimBoxToMediaBoxOffset [13 00000013 00000013 00000013 00000013 ]13 PDFXSetBleedBoxToMediaBox true13 PDFXBleedBoxToTrimBoxOffset [13 00000013 00000013 00000013 00000013 ]13 PDFXOutputIntentProfile ()13 PDFXOutputConditionIdentifier ()13 PDFXOutputCondition ()13 PDFXRegistryName ()13 PDFXTrapped False1313 CreateJDFFile false13 Description ltlt13 ARA 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 BGR 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 CHS ltFEFF4f7f75288fd94e9b8bbe5b9a521b5efa7684002000410064006f006200650020005000440046002065876863900275284e8e9ad88d2891cf76845370524d53705237300260a853ef4ee54f7f75280020004100630072006f0062006100740020548c002000410064006f00620065002000520065006100640065007200200035002e003000204ee553ca66f49ad87248672c676562535f00521b5efa768400200050004400460020658768633002gt13 CHT ltFEFF4f7f752890194e9b8a2d7f6e5efa7acb7684002000410064006f006200650020005000440046002065874ef69069752865bc9ad854c18cea76845370524d5370523786557406300260a853ef4ee54f7f75280020004100630072006f0062006100740020548c002000410064006f00620065002000520065006100640065007200200035002e003000204ee553ca66f49ad87248672c4f86958b555f5df25efa7acb76840020005000440046002065874ef63002gt13 CZE ltFEFF005400610074006f0020006e006100730074006100760065006e00ed00200070006f0075017e0069006a007400650020006b0020007600790074007600e101590065006e00ed00200064006f006b0075006d0065006e0074016f002000410064006f006200650020005000440046002c0020006b00740065007200e90020007300650020006e0065006a006c00e90070006500200068006f006400ed002000700072006f0020006b00760061006c00690074006e00ed0020007400690073006b00200061002000700072006500700072006500730073002e002000200056007900740076006f01590065006e00e900200064006f006b0075006d0065006e007400790020005000440046002000620075006400650020006d006f017e006e00e90020006f007400650076015900ed007400200076002000700072006f006700720061006d0065006300680020004100630072006f00620061007400200061002000410064006f00620065002000520065006100640065007200200035002e0030002000610020006e006f0076011b006a016100ed00630068002egt13 DAN ltFEFF004200720075006700200069006e0064007300740069006c006c0069006e006700650072006e0065002000740069006c0020006100740020006f007000720065007400740065002000410064006f006200650020005000440046002d0064006f006b0075006d0065006e007400650072002c0020006400650072002000620065006400730074002000650067006e006500720020007300690067002000740069006c002000700072006500700072006500730073002d007500640073006b007200690076006e0069006e00670020006100660020006800f8006a0020006b00760061006c0069007400650074002e0020004400650020006f007000720065007400740065006400650020005000440046002d0064006f006b0075006d0065006e0074006500720020006b0061006e002000e50062006e00650073002000690020004100630072006f00620061007400200065006c006c006500720020004100630072006f006200610074002000520065006100640065007200200035002e00300020006f00670020006e0079006500720065002egt13 DEU 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 ESP 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 ETI 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 FRA ltFEFF005500740069006c006900730065007a00200063006500730020006f007000740069006f006e00730020006100660069006e00200064006500200063007200e900650072002000640065007300200064006f00630075006d0065006e00740073002000410064006f00620065002000500044004600200070006f0075007200200075006e00650020007100750061006c0069007400e90020006400270069006d007000720065007300730069006f006e00200070007200e9007000720065007300730065002e0020004c0065007300200064006f00630075006d0065006e00740073002000500044004600200063007200e900e90073002000700065007500760065006e0074002000ea0074007200650020006f007500760065007200740073002000640061006e00730020004100630072006f006200610074002c002000610069006e00730069002000710075002700410064006f00620065002000520065006100640065007200200035002e0030002000650074002000760065007200730069006f006e007300200075006c007400e90072006900650075007200650073002egt13 GRE ltFEFF03a703c103b703c303b903bc03bf03c003bf03b903ae03c303c403b5002003b103c503c403ad03c2002003c403b903c2002003c103c503b803bc03af03c303b503b903c2002003b303b903b1002003bd03b1002003b403b703bc03b903bf03c503c103b303ae03c303b503c403b5002003ad03b303b303c103b103c603b1002000410064006f006200650020005000440046002003c003bf03c5002003b503af03bd03b103b9002003ba03b103c42019002003b503be03bf03c703ae03bd002003ba03b103c403ac03bb03bb03b703bb03b1002003b303b903b1002003c003c103bf002d03b503ba03c403c503c003c903c403b903ba03ad03c2002003b503c103b303b103c303af03b503c2002003c503c803b703bb03ae03c2002003c003bf03b903cc03c403b703c403b103c2002e0020002003a403b10020005000440046002003ad03b303b303c103b103c603b1002003c003bf03c5002003ad03c703b503c403b5002003b403b703bc03b903bf03c503c103b303ae03c303b503b9002003bc03c003bf03c103bf03cd03bd002003bd03b1002003b103bd03bf03b903c703c403bf03cd03bd002003bc03b5002003c403bf0020004100630072006f006200610074002c002003c403bf002000410064006f00620065002000520065006100640065007200200035002e0030002003ba03b103b9002003bc03b503c403b103b303b503bd03ad03c303c403b503c103b503c2002003b503ba03b403cc03c303b503b903c2002egt13 HEB 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 HRV (Za stvaranje Adobe PDF dokumenata najpogodnijih za visokokvalitetni ispis prije tiskanja koristite ove postavke Stvoreni PDF dokumenti mogu se otvoriti Acrobat i Adobe Reader 50 i kasnijim verzijama)13 HUN 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 ITA 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 JPN ltFEFF9ad854c18cea306a30d730ea30d730ec30b951fa529b7528002000410064006f0062006500200050004400460020658766f8306e4f5c6210306b4f7f75283057307e305930023053306e8a2d5b9a30674f5c62103055308c305f0020005000440046002030d530a130a430eb306f3001004100630072006f0062006100740020304a30883073002000410064006f00620065002000520065006100640065007200200035002e003000204ee5964d3067958b304f30533068304c3067304d307e305930023053306e8a2d5b9a306b306f30d530a930f330c8306e57cb30818fbc307f304c5fc59808306730593002gt13 KOR ltFEFFc7740020c124c815c7440020c0acc6a9d558c5ec0020ace0d488c9c80020c2dcd5d80020c778c1c4c5d00020ac00c7a50020c801d569d55c002000410064006f0062006500200050004400460020bb38c11cb97c0020c791c131d569b2c8b2e4002e0020c774b807ac8c0020c791c131b41c00200050004400460020bb38c11cb2940020004100630072006f0062006100740020bc0f002000410064006f00620065002000520065006100640065007200200035002e00300020c774c0c1c5d0c11c0020c5f40020c2180020c788c2b5b2c8b2e4002egt13 LTH 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 LVI 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 NLD (Gebruik deze instellingen om Adobe PDF-documenten te maken die zijn geoptimaliseerd voor prepress-afdrukken van hoge kwaliteit De gemaakte PDF-documenten kunnen worden geopend met Acrobat en Adobe Reader 50 en hoger)13 NOR ltFEFF004200720075006b00200064006900730073006500200069006e006e007300740069006c006c0069006e00670065006e0065002000740069006c002000e50020006f0070007000720065007400740065002000410064006f006200650020005000440046002d0064006f006b0075006d0065006e00740065007200200073006f006d00200065007200200062006500730074002000650067006e0065007400200066006f00720020006600f80072007400720079006b006b0073007500740073006b00720069006600740020006100760020006800f800790020006b00760061006c0069007400650074002e0020005000440046002d0064006f006b0075006d0065006e00740065006e00650020006b0061006e002000e50070006e00650073002000690020004100630072006f00620061007400200065006c006c00650072002000410064006f00620065002000520065006100640065007200200035002e003000200065006c006c00650072002000730065006e006500720065002egt13 POL 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 PTB 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 RUM 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 RUS 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 SKY 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 SLV 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 SUO 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 SVE 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 TUR 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 UKR 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 ENU (Use these settings to create Adobe PDF documents best suited for high-quality prepress printing Created PDF documents can be opened with Acrobat and Adobe Reader 50 and later)13 gtgt13 Namespace [13 (Adobe)13 (Common)13 (10)13 ]13 OtherNamespaces [13 ltlt13 AsReaderSpreads false13 CropImagesToFrames true13 ErrorControl WarnAndContinue13 FlattenerIgnoreSpreadOverrides false13 IncludeGuidesGrids false13 IncludeNonPrinting false13 IncludeSlug false13 Namespace [13 (Adobe)13 (InDesign)13 (40)13 ]13 OmitPlacedBitmaps false13 OmitPlacedEPS false13 OmitPlacedPDF false13 SimulateOverprint Legacy13 gtgt13 ltlt13 AddBleedMarks false13 AddColorBars false13 AddCropMarks false13 AddPageInfo false13 AddRegMarks false13 ConvertColors ConvertToCMYK13 DestinationProfileName ()13 DestinationProfileSelector DocumentCMYK13 Downsample16BitImages true13 FlattenerPreset ltlt13 PresetSelector MediumResolution13 gtgt13 FormElements false13 GenerateStructure false13 IncludeBookmarks false13 IncludeHyperlinks false13 IncludeInteractive false13 IncludeLayers false13 IncludeProfiles false13 MultimediaHandling UseObjectSettings13 Namespace [13 (Adobe)13 (CreativeSuite)13 (20)13 ]13 PDFXOutputIntentProfileSelector DocumentCMYK13 PreserveEditing true13 UntaggedCMYKHandling LeaveUntagged13 UntaggedRGBHandling UseDocumentProfile13 UseDocumentBleed false13 gtgt13 ]13gtgt setdistillerparams13ltlt13 HWResolution [2400 2400]13 PageSize [612000 792000]13gtgt setpagedevice13

Page 4: February 2014 MEE Questions and AnalysesPreface The Multistate Essay Examination (MEE) is developed by the National Conference of Bar Examiners (NCBE). This publication includes the

Preface

The Multistate Essay Examination (MEE) is developed by the National Conference of Bar Examiners (NCBE) This publication includes the questions and analyses from the February 2014 MEE (In the actual test the questions are simply numbered rather than being identified by area of law) The instructions for the test appear on page iii

The model analyses for the MEE are illustrative of the discussions that might appear in excellent answers to the questions They are provided to the user jurisdictions to assist graders in grading the examination They address all the legal and factual issues the drafters intended to raise in the questions

The subjects covered by each question are listed on the first page of its accompanying analysis identified by roman numerals that refer to the MEE subject matter outline for that subject For example the Federal Civil Procedure question on the February 2014 MEE tested the following area from the Federal Civil Procedure outline IVD Pretrial proceduresmdashDiscovery (including e-discovery)

For more information about the MEE including subject matter outlines visit the NBCE website at wwwncbexorg

Description of the MEE

The MEE consists of six 30-minute essay questions and is a component of the Uniform Bar Examination (UBE) It is administered by participating jurisdictions on the Tuesday before the last Wednesday in February and July of each year The areas of law that may be covered by the questions on any MEE are Business Associations (Agency and Partnership Corporations and Limited Liability Companies) Conflict of Laws Constitutional Law Contracts Criminal Law and Procedure Evidence Family Law Federal Civil Procedure Real Property Torts Trusts and Estates (Decedentsrsquo Estates Trusts and Future Interests) and Uniform Commercial Code (Negotiable Instruments and Bank Deposits and Collections Secured Transactions) Some questions may include issues in more than one area of law The particular areas covered vary from exam to exam

The purpose of the MEE is to test the examineersquos ability to (1) identify legal issues raised by a hypothetical factual situation (2) separate material which is relevant from that which is not (3) present a reasoned analysis of the relevant issues in a clear concise and well-organized composition and (4) demonstrate an understanding of the fundamental legal principles relevant to the probable solution of the issues raised by the factual situation The primary distinction between the MEE and the Multistate Bar Examination (MBE) is that the MEE requires the examinee to demonstrate an ability to communicate effectively in writing

ii

Instructions The back cover of each test booklet contains the following instructions

You will be instructed when to begin and when to stop this test Do not break the seal on this booklet until you are told to begin

You may answer the questions in any order you wish Do not answer more than one question in each answer booklet If you make a mistake or wish to revise your answer simply draw a line through the material you wish to delete

If you are using a laptop computer to answer the questions your jurisdiction will provide you with specific instructions

Read each fact situation very carefully and do not assume facts that are not given in the question Do not assume that each question covers only a single area of the law some of the questions may cover more than one of the areas you are responsible for knowing

Demonstrate your ability to reason and analyze Each of your answers should show an understanding of the facts a recognition of the issues included a knowledge of the applicable principles of law and the reasoning by which you arrive at your conclusions The value of your answer depends not as much upon your conclusions as upon the presence and quality of the elements mentioned above

Clarity and conciseness are important but make your answer complete Do not volunteer irrelevant or immaterial information

Answer all questions according to generally accepted fundamental legal principles unless your testing jurisdiction has instructed you to answer according to local case or statutory law

NOTE Examinees testing in UBE jurisdictions must answer according to generally accepted fundamental legal principles rather than local case or statutory law

iii

February 2014 MEE

QUESTIONS Constitutional Law

Trusts and Future Interests Secured Transactions

Federal Civil Procedure Criminal Law and Procedure

Agency and Partnership

CONSTITUTIONAL LAW QUESTION ___________

A city ordinance required each downtown business to install high-powered halogen floodlights that would illuminate the property owned by that business and the adjoining sidewalks A study commissioned by the city estimated that installation of the floodlights would cost a typical business about $1000 but that increased business traffic due to enhanced public safety especially after dark would likely offset this cost

A downtown restaurant applied to the city for a building permit to construct an addition that would increase its seating capacity In its permit application the restaurant accurately noted that its current facility did not have sufficient seating to accommodate all potential customers during peak hours The city approved the permit on the condition that the restaurant grant the city an easement over a narrow strip of the restaurantrsquos property to be used by the city to install video surveillance equipment that would cover nearby public streets and parking lots The city based its permit decision entirely on findings that the increased patronage that would result from the increased capacity of the restaurant might also attract additional crime to the neighborhood and that installing video surveillance equipment might alleviate that problem

The restaurant has challenged both the ordinance requiring it to install floodlights and the easement condition imposed on approval of the building permit

1 Under the Fifth Amendment as applied to the states through the Fourteenth Amendment is the city ordinance requiring the restaurant to install floodlights an unconstitutional taking Explain

2 Under the Fifth Amendment as applied to the states through the Fourteenth Amendment is the cityrsquos requirement that the restaurant grant the city an easement as a condition for obtaining the building permit an unconstitutional taking Explain

3

TRUSTS AND FUTURE INTERESTS QUESTION _______________

Ten years ago a testator died survived by his only children a son age 26 and a daughter age 18

A testamentary trust was created under the testatorrsquos duly probated will The will specified that all trust income would be paid to the son during the sonrsquos lifetime and that upon the sonrsquos death the trust would terminate and trust principal would be distributed to the testatorrsquos ldquograndchildren who shall surviverdquo the son The testator provided for his daughter in other sections of the will

Five years ago the trustee of the testamentary trust purchased an office building with $500000 from the trust principal Other than this building the trust assets consist of publicly traded securities

Last year the trustee received $30000 in rents from the office building The trustee also received with respect to the securities owned by the trust cash dividends of $20000 and a stock dividend of 400 shares of Acme Corp common stock distributed to the trust by Acme Corp

Eight months ago the trustee sold the office building for $700000

Six months ago the son delivered a letter to the trustee stating ldquoI hereby disclaim any interest I may have in the income interest of the trustrdquo On the date the son delivered this letter to the trustee the son had no living children the daughter had one living minor child

A statute in this jurisdiction provides that ldquoa disclaimer of any interest created by will is valid only if made within nine months after the testatorrsquos death and if an interest is validly disclaimed the disclaiming party is deemed to have predeceased the testatorrdquo

1 How should the rents sales proceeds cash dividends and stock dividends received prior to the trusteersquos receipt of the sonrsquos letter have been allocated between trust principal and income Explain

2 How if at all does the sonrsquos letter to the trustee affect the future distribution of trust income and principal Explain

4

SECURED TRANSACTIONS QUESTION

On March 1 the owner of a manufacturing business entered into negotiations with a bank to obtain a loan of $100000 for the business The bank loan officer informed the business owner that the interest rate for a loan would be lower if the repayment obligation were secured by all the businessrsquos present and future equipment The loan officer also informed the business owner that the bank could not commit to making the loan until its credit investigation was completed but that funds could be advanced faster following loan approval if a financing statement with respect to the transaction were filed in advance Accordingly the business owner signed a form on behalf of the business authorizing the bank to file a financing statement with respect to the proposed transaction The bank properly filed a financing statement the next day correctly providing the name of the business as the debtor and indicating ldquoequipmentrdquo as the collateral

On March 15 the business owner had heard nothing from the bank about whether the loan had been approved so the business owner approached a finance company for a loan The finance company quickly agreed to lend $100000 to the business secured by all the businessrsquos present and future equipment That same day the finance company loaned to the business $100000 and the business owner signed an agreement obligating the business to repay the loan and granting the finance company a security interest in all the businessrsquos ldquopresent and future equipmentrdquo to secure the repayment obligation Also on that day the finance company properly filed a financing statement correctly providing the businessrsquos name as the debtor and indicating ldquoequipmentrdquo as the collateral

On March 21 the bank loan officer contacted the business owner and indicated that the loan application had been approved On the next day March 22 the bank loaned the business $100000 The loan agreement signed by the owner on behalf of the business granted the bank a security interest in all the businessrsquos ldquopresent and future equipmentrdquo

On April 10 the business sold an item of manufacturing equipment to a competitor for $20000 This was the first time the business had ever sold any of its equipment The competitor paid the purchase price in cash and took possession of the equipment that day The competitor acted in good faith at all times and had no knowledge of the businessrsquos prior transactions with the bank and the finance company

The business has defaulted on its obligations with respect to the loans from the bank and the finance company Each of them has asserted a claim to all the businessrsquos equipment as well as to the item of equipment sold to the businessrsquos competitor

Assume that the business owner had the authority to enter into all these transactions on behalf of the business

1 As between the bank and the finance company which has a superior claim to the businessrsquos equipment Explain

2 Do the claims of the bank and the finance company to the businessrsquos equipment continue in the item of equipment sold to the competitor Explain

5

FEDERAL CIVIL PROCEDURE QUESTION

A builder constructed a vacation house for an out-of-state customer on the customerrsquos land The house was completed on June 1 at which point the customer still owed $200000 of the $800000 contract price which was payable in full five days later

On June 14 the basement of the house was flooded with two inches of water during a heavy rainfall When the customer complained the builder told the customer ldquoThe flooding was caused by poorly designed landscaping Our work is fine and fully up to code Have an engineer look at the foundation If therersquos a problem wersquoll fix itrdquo

The customer pleased by the builderrsquos cooperative attitude immediately hired a structural engineer to examine the foundation of the house On June 30 the engineer provided the customer with a written report on the condition of the foundation which stated that the foundation was properly constructed

Unhappy with the conclusions in the engineerrsquos report the customer then hired a home inspector to evaluate the house The home inspectorrsquos report concluded that the foundation of the house had been poorly constructed and was inadequately waterproofed

On July 10 the customer sent the builder the home inspectorrsquos report with a note that said ldquoUntil you fix this problem you wonrsquot get another penny from merdquo The builder immediately contacted an attorney and directed the attorney to prepare a draft complaint against the customer for nonpayment Hoping to avoid litigation the builder sent several more requests for payment to the customer The customer ignored all these requests

On September 10 the builder filed suit in federal district court properly invoking the courtrsquos diversity jurisdiction and seeking $200000 in damages for breach of contract The customerrsquos answer denied liability on the basis of alleged defective construction of the housersquos foundation

Several months later the case is nearly ready for trial However two discovery disputes have not yet been resolved

First despite a request from the builder the customer has refused to provide a copy of the report prepared by the structural engineer who examined the foundation of the house The customer claims that the report is ldquowork productrdquo and not discoverable because the customer does not intend to ask the engineer to testify at trial The builder has asked the court to order the customer to turn over the engineerrsquos report

Second the customer has asked the court to impose sanctions for the builderrsquos failure to comply with the customerrsquos demand for copies of all emails concerning construction of the foundation of the house The builder has truthfully informed the customer that all such emails were destroyed on August 2 This destruction was pursuant to the builderrsquos standard practice of permanently deleting all project-related emails from company records 60 days after construction of a project is complete There is no relevant state records-retention law

1 Should the court order the customer to turn over the engineerrsquos report Explain

2 Should the court sanction the builder for the destruction of emails related to the case and if so what factors should the court consider in determining those sanctions Explain

6

CRIMINAL LAW AND PROCEDURE QUESTION _____

A defendant was charged under state law with felony theft (Class D) and felony residential burglary (Class C) The indictment alleged that the defendant entered his neighborsrsquo home without their consent and stole a diamond ring worth at least $2500

Defense counsel filed a pretrial motion to dismiss the charges on the ground that prosecuting the defendant for both burglary and theft would constitute double jeopardy The trial court denied the motion and the defendant was prosecuted for both crimes The only evidence of the ringrsquos value offered at the defendantrsquos jury trial was the ownerrsquos testimony that she had purchased the ring two years earlier for $3000

At trial the judge issued the following jury instruction on the burglary charge prior to deliberations

If after consideration of all the evidence presented by the prosecution and defense you find beyond a reasonable doubt that the defendant entered the dwelling without the ownersrsquo consent you may presume that the defendant entered with the intent to commit a felony therein

The jury found the defendant guilty of both offenses

At the defendantrsquos sentencing hearing an expert witness called by the prosecutor testified that the diamond ring was worth between $7000 and $8000 Over defense objection the judge concluded by a preponderance of the evidence that the value of the stolen ring exceeded $5000 The judge sentenced the defendant to four yearsrsquo incarceration on the theft conviction On the burglary conviction the defendant received a consecutive sentence of seven yearsrsquo incarceration

In this state residential burglary is defined as ldquoentry into the dwelling of another without the consent of the lawful resident with the intent to commit a felony thereinrdquo Residential burglary is a Class C felony for which the minimum sentence is five years and the maximum sentence is ten years of incarceration

In this state theft is defined as ldquotaking and carrying away the property of another with the intent to permanently deprive the owner of possessionrdquo Theft is a Class D felony if the value of the item(s) taken is between $2500 and $10000 The sentence for a Class D felony theft is determined by the value of the items taken If the value is between $2500 and $5000 the maximum sentence is three yearsrsquo incarceration If the value of the items exceeds $5000 the maximum sentence is five yearsrsquo incarceration

This state affords a criminal defendant no greater rights than those mandated by the United States Constitution

1 Did the trial court err when it denied the defendantrsquos pretrial motion to dismiss on double jeopardy grounds Explain

2 Did the trial court err in its instruction to the jury on the burglary charge Explain

3 Did the trial court err when it sentenced the defendant to an additional year of incarceration on the theft conviction based on the expertrsquos testimony Explain

7

AGENCY AND PARTNERSHIP QUESTION _____

Five years ago Adam and Ben formed a general partnership Empire Partnership (Empire) to buy and sell antique automobiles at a showroom in State A Adam contributed $800000 to Empire and Ben contributed $200000 Their written partnership agreement allocated 80 of profits losses and control to Adam and 20 to Ben No filings of any type were made in connection with the formation of Empire

Three years ago a collector purchased one of Empirersquos antique cars for $3400000 The collector was willing to pay this price because of Benrsquos false representation (repeated in the sales contract) that a famous movie star had once owned the car Without the movie-star connection the car was worth only $100000 One month later when the collector discovered the truth he sued Adam Ben and Empire for $3300000 in damages The lawsuit is still pending

Two years ago Adam and Ben admitted a new partner Diane to Empire in return for her contribution of $250000 The three agreed to allocate profits losses and control 75 to Adam 10 to Ben and 15 to Diane Before joining the partnership Diane learned of the collectorrsquos claim and stated her concern to Adam and Ben that she might become liable if the claim were reduced to a judgment

Following Dianersquos admission to Empire the three partners sought to convert Empire into a limited liability partnership (LLP) Adamrsquos lawyer proposed to file with State A a ldquostatement of qualificationrdquo making an LLP election and declaring the name of the partnership to be ldquoEmpire LLPrdquo Benrsquos lawyer stated that this would not work and that a new LLP had to be formed with the assets of the old partnership transferred to the new one In the end the conversion was done the way Adamrsquos lawyer suggested with the approval of all three partners

One year ago a driver purchased a vintage car from Empire LLP based on the representation that the car was ldquofully roadworthy and capable of touring at 70 mph all dayrdquo The driver took the car on the highway at 50 mph whereupon the front suspension collapsed resulting in a crash in which the car was destroyed and the driver killed The driverrsquos estate sued Adam Ben Diane and Empire LLP for $10000000 The lawsuit is still pending

Although profitable Empire LLP does not have resources sufficient to pay the collectorrsquos claim or the claim of the driverrsquos estate

Assume that the Uniform Partnership Act (1997) applies

1 Before the filing of the statement of qualification (a) was Adam personally liable on the collectorrsquos claim Explain (b) was Diane personally liable on the collectorrsquos claim Explain

2 After the filing of the statement of qualification was Adam Ben or Diane personally liable as a partner on (a) the collectorrsquos claim or (b) the driverrsquos estatersquos claim Explain

8

February 2014 MEE

ANALYSES Constitutional Law

Trusts and Future Interests Secured Transactions

Federal Civil Procedure Criminal Law and Procedure

Agency and Partnership

CONSTITUTIONAL LAW ANALYSIS (Constitutional Law IVD)

ANALYSIS

Legal Problems

(1) Is the city ordinance requirement that businesses install floodlights a taking

(2) Is conditioning the approval of a building permit on the grant of an easement to install surveillance equipment a taking of property

DISCUSSION

Summary

The ordinance requiring businesses to install floodlights is not a per se taking under Loretto because it does not force a private landowner to allow a third party to enter and place a physical object on the land Here the city ordinance requires the businessmdashnot a third partymdashto install the floodlights

The ordinance is likely not a regulatory taking under the Penn Central balancing test While the ordinance will impose a cost on business owners that cost may be offset by the expected increase in business due to the ordinance and the ordinance does not appear to interfere with the ownerrsquos primary use of the property as a restaurant

The permit condition however is likely an uncompensated taking of property While the condition has an essential nexus with the cityrsquos legitimate interest in promoting public safety the city has not made an individualized determination that the easement condition is roughly proportional to the possibility of increased crime due to the restaurantrsquos proposed addition Thus the permit condition likely violates the Fifth Amendment as applied to the states through the Fourteenth Amendment

Point One (50) The ordinance requiring that businesses install floodlights is not a per se taking under Loretto It is not a regulatory taking under the Penn Central balancing test because the cost of compliance with the ordinance may be offset by an expected increase in business and compliance does not interfere with the businessrsquos primary use of its property as a restaurant

The city ordinance requiring a business to install floodlights does not effect a per se taking of the sort described in Loretto v Teleprompter Manhattan CATV Corp 458 US 419 (1982) because no property is physically taken by the government and the ordinance does not involve a physical invasion of private property by a third party

Even though the ordinance does not constitute an occupation of the property by either the government or a third party it is still subject to the three-factor balancing test under Penn Central Transportation Co v City of New York 438 US 104 (1978) to determine whether it is a ldquoregulatory takingrdquo Under Penn Central a court must balance (1) ldquo[t]he economic impact of the regulation on the claimantrdquo (2) ldquothe extent to which the regulation has interfered with distinct investment-backed expectationsrdquo and (3) ldquothe character of the governmental actionrdquo Id at 124 Here each factor weighs against finding that the ordinance is a taking

11

Constitutional Law Analysis

First the ordinance requirement likely has a minimal economic impact on the restaurant Compliance with the ordinance is estimated to cost $1000 and the city has found that businesses will likely recoup that cost in increased sales Also because the ordinance does not interfere with the operation of the restaurant the owner may still earn a reasonable return on its investment in the property

Second the ordinance does not interfere with the businessrsquos investment-backed expectations As in Penn Central the challenged law does not interfere with the ownerrsquos ldquoprimary expectationrdquo for use of the propertymdashin Penn Central as a railroad terminal and here as a restaurant Further the ordinance does not prevent the restaurant from expanding to meet the changing business environment

Third the character of the government action does not weigh in favor of a taking While Penn Central does say that a ldquophysical invasionrdquo is more likely to pose a taking Loretto suggests that the Courtrsquos main concern is with physical invasions by third parties Also like the landmark law challenged in Penn Central the ordinance here ldquoadjust[s] the benefits and burdens of economic life to promote the common goodrdquo Id In Penn Central the landmark law restricted development of the railroad terminal to promote the common interest in preserving historic landmarks Here the ordinance requires the businesses to install floodlights to promote the common interest in crime prevention and public safety

Because the ordinance is clearly a valid exercise of the police power it satisfies the takings clausersquos public-use requirement Kelo v City of New London 545 US 469 (2005)

In sum all three factors weigh against finding a taking under the Penn Central balancing test

Point Two (50) The permit condition may be unconstitutional as an uncompensated taking of property because the city has not made an individualized determination that the easement condition is roughly proportional to the impact of the restaurantrsquos proposed addition

In Dolan v City of Tigard 512 US 374 (1994) the Supreme Court set forth the test for determining whether an exaction imposed by a government in exchange for a discretionary benefit conferred by the government such as a condition on the approval of a building permit in this case constitutes an uncompensated taking under the Fifth Amendment The exaction is not a taking if (1) there is an ldquoessential nexusrdquo between the ldquopublic need or burdenrdquo to which the proposed development contributes and ldquothe permit condition exacted by the cityrdquo id at 386 and (2) the government makes ldquosome sort of individualized determination that the required dedication is [roughly proportional] both in nature and extent to the impact of the proposed developmentrdquo Id at 391 see also Nollan v California Coastal Commission 483 US 825 (1987)

Here the city likely can meet the nexus requirement In Dolan the landowner sought to double the size of its business which would have increased traffic on nearby roadways In exchange for approving the development the city sought an easement for a bike and pedestrian path The Court found the required nexus between the easement and the cityrsquos ldquoattempt to reduce traffic congestion by providing for alternative means of transportationrdquo 512 US at 387 Here a similar nexus likely exists between the requested easement and the cityrsquos interest in crime prevention and public safety Increased patronage and economic activity at the restaurant might attract additional crime to the area and the requested easement to install surveillance equipment would attempt to address that increased crime

12

Constitutional Law Analysis

The exaction here however may fail the second prong of the Dolan testmdashthat the exaction be roughly proportional to the anticipated impact of the requested development As noted the city in Dolan claimed that a bike and pedestrian path was needed to offset the increase in traffic due to the proposed doubling of the business The Court explained that the government must demonstrate that the additional traffic reasonably was related to the requested exaction and that the government must ldquomake some effort to quantify its findings in support of the dedication for the pedestrianbicycle pathway beyond the conclusory statement that it could offset some of the traffic demand generatedrdquo Id at 395 Here the city did not carry its burden The city simply speculates that increased patronage of the restaurant ldquomightrdquo increase crime and that the surveillance equipment ldquomightrdquo alleviate this increased crime Because the city has not made ldquosome effort to quantify its findingsrdquo in support of the easement it has not shown that the burden of the easement is roughly proportional to the benefits thought to flow from it

Thus the exaction appears to be an uncompensated taking of property in violation of the Fifth Amendment as applied to the states through the Fourteenth Amendment

13

TRUSTS AND FUTURE INTERESTS ANALYSIS ____ (Trusts and Future Interests IE3 I5 IIIA amp B)

ANALYSIS

Legal Problems

(1) How should rents dividends and sales proceeds received by the trustee prior to receipt of the sonrsquos letter have been allocated between trust income and principal

(2)(a) Did the remainder interest in the trust accelerate and become immediately payable to the daughterrsquos minor child upon the trusteersquos receipt of the sonrsquos letter and if not how should the trustee handle the distribution of the principal in the future

(2)(b) Following the trusteersquos receipt of the sonrsquos letter how should the trustee distribute future receipts of income prior to the distribution of the principal

DISCUSSION

Summary

Prior to the trusteersquos receipt of the sonrsquos letter cash dividends and rents should have been allocated to trust income and were distributable to the son the income beneficiary of the trust sales proceeds and stock dividends should have been allocated to principal

Because the sonrsquos letter to the trustee did not result in a valid disclaimer under state law (having been made more than nine months after the testatorrsquos death) the son is not deemed to have predeceased the testator Because the son is still living the class gift to the testatorrsquos grandchildren who survive the son has not closed and is not possessory it will not become possessory until the son dies The daughterrsquos minor child being the testatorrsquos only living grandchild is not currently entitled to a distribution of trust principal Trust principal will instead be distributable upon the sonrsquos death to the testatorrsquos then-living grandchildren or if there are none to the testatorrsquos then-living heirs

As for future income the trustee should either distribute the trust income to the son and the daughter as the testatorrsquos heirs accumulate the income for future distribution to those individuals ultimately entitled to the trust principal or distribute it to those presumptively entitled to the principal upon the sonrsquos death ie the daughterrsquos minor child

Point One (45) Cash dividends and rents are allocable to income sales proceeds and stock dividends are allocable to principal Items allocable to income for the period prior to the sonrsquos attempted disclaimer were distributable to the son

Receipts earned during the administration of a trust are allocable either to income or to principal Almost all states have adopted the most recent or an earlier version of the Uniform Principal and Income Act (the Act) which specifies how such receipts should be allocated

Under the Act rents (UNIF PRIN amp INC ACT (2000) sect 405 UNIF PRIN amp INC ACT (1962) sect 3(a)(1)) and cash dividends received from a corporation (UNIF PRIN amp INC ACT (2000) sect 401(b) UNIF PRIN amp INC ACT (1962) sect 6(d)) are allocable to income and are distributable to the income beneficiary of the trust

14

Trusts and Future Interests Analysis Sales proceeds (UNIF PRIN amp INC ACT (2000) sect 404(2) UNIF PRIN amp INC ACT (1962)

sect 3(b)(1)) and dividends paid in the stock of the distributing corporation (UNIF PRIN amp INC ACT (2000) sect 401(c)(1) UNIF PRIN amp INC ACT (1962) sect 3(b)(4)) are allocable to principal and added to the principal of the trust

Here the cash dividends and office building rents should have been allocated to income and until the trustee received the sonrsquos letter should have been distributed to him as the sole income beneficiary of the trust The stock dividend and proceeds from the sale of the office building should have been allocated to principal and held by the trustee for future distribution to the ultimate remaindermen of the trust

[NOTE The 2000 Uniform Principal and Income Act has been adopted in Alabama Arkansas Colorado Connecticut the District of Columbia Hawaii Idaho Iowa Kentucky Missouri Montana Nebraska New Mexico North Dakota Oregon South Dakota Utah and West Virginia]

Point Two(a) (45) Because the son did not disclaim within nine months of the testatorrsquos death there is no valid disclaimer under state law Therefore the son is not deemed to have predeceased the testator Furthermore because of the express survivorship contingency in the will the remainder in the trust does not accelerate and become distributable until the son in fact dies When the son dies the trust principal will be distributable to the testatorrsquos then-living grandchildren or if none then to the testatorrsquos then-living heirs

When a trust remainder is given to a class the class closes (ie no new persons can join the class) when there is no outstanding income interest and at least one member of the class is then entitled to demand possession of his or her share of the remainder This principle is called the rule of convenience See generally HERBERT HOVENKAMP amp SHELDON F KURTZ PRINCIPLES OF PROPERTY LAW 199ndash200 (6th ed 2005) A class member may demand possession of his or her share of the remainder upon termination of the income interest only when the class memberrsquos interest is not otherwise subject to a condition precedent See id

When a beneficiary timely disclaims an interest in a trust that beneficiary is treated as if he had predeceased the testator Here had the son disclaimed within nine months of the testatorrsquos death as required by the state statute he would have been deemed to have predeceased the testator This would have closed the class of remaindermen and the testatorrsquos then-living grandchildren (ie the daughterrsquos child) would have been entitled to the trust principal However under the state statute the sonrsquos disclaimer was not timely because he did not disclaim within nine months of the testatorrsquos death Thus because the statute is inapplicable and the son is still alive the class of grandchildren entitled to share in trust principal did not close

Because here the statute is inapplicable due to the sonrsquos failure to comply with the statutory time requirements then presumably the common-law rule allowing disclaimers (aka renunciations) at any time should apply Under the common law if a life estate is renounced the remainder interest accelerates and becomes immediately distributable to the remaindermen of the trust if the remainder is vested but not if the remainder is contingent JESSE DUKEMINIER amp ROBERT H SITKOFF WILLS TRUSTS AND ESTATES 844ndash845 (9th ed 2013) Here because the remainder is contingent upon there being grandchildren who survive the son the remainder will not accelerate It will remain open until the son dies leaving open the possibility that additional grandchildren will be included in the class or the daughterrsquos child could fall out of the class because that child fails to survive the son

And if none of the testatorrsquos grandchildren survive the son the trust principal will be distributed to the testatorrsquos heirs living at the sonrsquos death

15

Trusts and Future Interests Analysis

Point Two(b) (10) Until the trust terminates the trustee must continue to hold the trust assets The distribution of income in the meantime is unclear There are at least three possibilities Income earned on the undistributed assets could be distributed to the son and daughter as the testatorrsquos heirs accumulated and added to principal for distribution to the ultimate remaindermen or distributed from time to time to those persons who are presumptively remaindermen

When trust principal is not immediately distributable the trustee must continue to hold trust assets until the ultimate remaindermen are ascertained During this period trust income will be distributed or retained according to any instructions contained in the trust instrument See WILLIAM M MCGOVERN JR SHELDON F KURTZ amp DAVID M ENGLISH WILLS TRUSTS amp ESTATES sect 102 (4th ed 2010)

Here the testator did not specify what the trustee should do with trust income in the event the sonrsquos disclaimer did not comply with the state statute There are at least three approaches One approach would have the trustee distribute the trust income to the testatorrsquos heirs on the theory that the income represents property that was not disposed of by the testatorrsquos will and which thus passes by partial intestacy to the testatorrsquos heirs A second approach would have the trustee accumulate trust income for distribution to the ultimate remaindermen Under this approach only those individuals ultimately entitled to the principal would share in the income A third approach would have the trustee distribute trust income to those individuals who would be the remaindermen if the trust were to terminate when the income is received by the trustee under this approach trust income would be distributed to the daughterrsquos minor child until another presumptive remainderman is born This approach could result in individuals not ultimately entitled to principal say because they do not survive the son receiving income It could also result in a disproportionate distribution of income among the individuals ultimately entitled to income

[NOTE Examinees should demonstrate a recognition and understanding of the income-allocation problem and the alternatives available to address that issue There is no widely accepted solution to the problem Examinees who cite any of these possible problem-solving approaches may receive credit]

16

SECURED TRANSACTIONS ANALYSIS (Secured Transactions IB IID E amp F IIIB IVA B amp F)

ANALYSIS

Legal Problems

(1)(a) What is the nature of the bankrsquos claim to the businessrsquos equipment

(1)(b) What is the nature of the finance companyrsquos claim to the businessrsquos equipment

(1)(c) As between the bank and the finance company whose claim to the businessrsquos equipment has priority

(2) Do the claims of the bank and the finance company continue in the item of equipment sold by the business to the competitor

DISCUSSION

Summary

The bank and the finance company both have perfected security interests in the businessrsquos equipment Even though the finance companyrsquos perfected security interest was created first the bankrsquos perfected security interest has priority because the bankrsquos financing statement was filed before the finance companyrsquos financing statement The security interests of the bank and the finance company continue in the item of equipment sold by the business to the competitor because their security interests were perfected and the competitor was not a buyer in ordinary course of business

Point One(a) (25) The bank has a perfected security interest in the businessrsquos equipment

The bank has met all criteria necessary for it to have an attached and enforceable security interest in the businessrsquos equipment First value must be given UCC sect 9-203(b)(1) This criterion is fulfilled by the loan made by the bank to the business Second the debtor must have rights in the collateral UCC sect 9-203(b)(2) Clearly the business has rights in its equipment Third either the secured party must take possession of the collateral or the debtor must authenticate a security agreement containing a description of the collateral UCC sect 9-203(b)(3) The agreement that the business owner signed is a ldquosecurity agreementrdquo because it is an agreement that creates or provides for a security interest UCC sect 9-102(a)(74) By signing the security agreement the business owner authenticated it UCC sect 9-102(a)(7) Therefore all three criteria are fulfilled and the bank has an enforceable and attached security interest

A security interest is perfected when it has attached and when any additional steps required for perfection have occurred UCC sect 9-308(a) Generally speaking the additional steps will either be possession of the collateral by the secured party or the filing of a financing statement with respect to the collateral See UCC sectsect 9-310 9-313 In this case the bank filed a financing statement naming the debtor and sufficiently indicating the collateral The collateral indication is sufficient because it identifies the collateral by type of property See UCC sectsect 9-504 9-108 The fact that the financing statement was filed before the security interest was created is

17

Secured Transactions Analysis

not a problem Even though the security agreement had not yet been signed the business had authorized the filing of the financing statement in an authenticated record UCC sect 9-509(a)(1) Moreover the financing statement may be filed before the security agreement is created UCC sect 9-502(d)

Point One(b) (10) The finance company also has a perfected security interest in the businessrsquos equipment

The finance companyrsquos security interest is enforceable and attached for the same reasons as the bankrsquos security interest The loan from the finance company to the business constitutes value the business has rights in the collateral and the business owner has authenticated a security agreement containing a description of the collateral The finance companyrsquos security interest is perfected because the finance company filed a financing statement with respect to it that provides that the business is the debtor and indicates that the collateral is equipment

Point One(c) (30) The bankrsquos security interest has priority over the finance companyrsquos security interest because the bankrsquos financing statement was filed first

As between two perfected security interests the general rule is that the security interest that was the earlier to be either perfected or the subject of a filed financing statement has priority UCC sect 9-322(a)(1) While the finance companyrsquos security interest was perfected before the bankrsquos (March 15 vs March 22) the bankrsquos financing statement was filed even earlier on March 2 Thus under the first-to-file-or-perfect rule of UCC sect 9-322(a)(1) the bankrsquos security interest has priority No exceptions to the general rule apply here

Point Two (35) A security interest in collateral continues notwithstanding its sale unless an exception applies Because the security interests of the bank and the finance company were perfected and the competitor was not a buyer in ordinary course of business no exception applies and the security interests of both creditors continue in the equipment sold to the competitor

As a general rule a security interest in collateral continues notwithstanding the fact that the debtor has sold the collateral to another person UCC sect 9-315(a)(1) Thus unless an exception applies the security interests of the bank and the finance company will continue in the item of equipment sold to the competitor

A buyer of goods will take free of an unperfected security interest in those goods See UCC sect 9-317(a)(2) However when the competitor bought the businessrsquos equipment both the bank and the finance company had perfected security interests in the equipment

A buyer can take free even of a perfected security interest in goods if the buyer is a ldquobuyer in ordinary course of businessrdquo See UCC sect 9-320(a) However the competitor was not a buyer in ordinary course of business To be a ldquobuyer in ordinary course of businessrdquo a buyer must buy goods from a seller that is in the business of selling goods of that kind See UCC sect 1-201(b)(9) The competitor bought this equipment from a seller that is not in the business of selling goods of this kind so the competitor was not a buyer in ordinary course of business with respect to these goods

Because no exception applies the security interests of the bank and the finance company continue even after the item of equipment was sold to the competitor

18

FEDERAL CIVIL PROCEDURE ANALYSIS (Federal Civil Procedure IVD)

ANALYSIS

Legal Problems

(1) Is a document prepared in the course of a contract dispute protected from discovery as ldquowork productrdquo when there is no evidence that the document was prepared in anticipation of litigation

(2)(a) Is a partyrsquos failure to provide relevant electronically stored information excused when the information was destroyed pursuant to a routine document retention scheme at a time when litigation was contemplated by the destroying party

(2)(b) What sanctions should be imposed on a party for allowing the destruction of evidence that is relevant to potential future litigation

DISCUSSION

Summary

The report prepared by the structural engineer is probably not work product and is thus discoverable The engineer examined the foundation of the house at the customerrsquos request and the engineerrsquos findings are potentially relevant to the customerrsquos claim that the foundation is defective The report was not prepared in anticipation of litigation The customer appears to have sought the engineerrsquos opinion in response to the builderrsquos offer to fix any problems with the foundation that an engineer might identify Because the report was not prepared in anticipation of litigation it is not protected by the work-product doctrine

The builder should have taken appropriate steps to preserve evidence including suspending its document retention program as soon as it began planning for litigationmdashie on July 10 Its destruction of potentially relevant material after that date was wrongful However a court is unlikely to impose severe sanctions on the builder because there are no facts indicating that the builder acted in bad faith and the customer can prove that the foundation is defective without the destroyed emails

Point One (40) The customer must turn over the engineerrsquos report because it was not prepared in anticipation of litigation

In general a party to a lawsuit in federal court ldquomay obtain discovery regarding any nonprivileged matter that is relevant to any partyrsquos claim or defenserdquo FED R CIV P 26(b)(1) (2009) This includes the right to inspect and copy documents in the other partyrsquos possession FED R CIV P 34(a)(1) Here the customer hired a structural engineer to examine the foundation of the house The engineerrsquos report on the foundation is likely to include information that would be relevant to the customerrsquos claim that the foundation was defectively constructed

The so-called ldquowork productrdquo rule allows a party to refuse to turn over ldquodocuments that are prepared in anticipation of litigation or for trialrdquo by that partyrsquos representative including

19

Federal Civil Procedure Analysis

a consultant Thus if the customer had hired the structural engineer to prepare a report ldquoin anticipation of litigationrdquo that report might not be discoverable See FED R CIV P 26(b)(3)

In this case however the customer hired the engineer to evaluate the foundation of the house as part of the customerrsquos negotiation with the builder concerning the housersquos flooding problem The builder told the customer that the housersquos landscaping was the reason for the flooding and the builder told the customer ldquoHave an engineer look at the foundation If therersquos a problem wersquoll fix itrdquo The customer appears to have acted in response to that statement There is no indication that the customer anticipated any kind of legal action at the time that the structural engineer was hired Accordingly the structural engineerrsquos report is discoverable and the court should order the customer to turn it over

[NOTE If an examinee concludes that the structural engineerrsquos report was prepared in anticipation of litigation then the examinee should also conclude that the report is not discoverable Documents prepared in anticipation of litigation do not need to be disclosed to an adverse party unless that party can demonstrate a ldquosubstantial needrdquo for the documents and an inability to obtain substantially equivalent information without ldquoundue hardshiprdquo FED R CIV P 26(b)(3)(A)(ii) Furthermore a report prepared by an expert who is not expected to testify is not discoverable in the absence of ldquoexceptional circumstancesrdquo making it ldquoimpracticablerdquo to obtain the information in another way FED R CIV P 26(b)(4)(D)(ii) The builder probably cannot make these showings here unless the engineerrsquos report deals with circumstances that have since changed There is no evidence that the structural engineer would have had access to any information or facts that the builder would not already know as a result of its construction and subsequent inspection of the house In addition if necessary the builder could ask the court for permission to arrange for a further inspection of the house by a structural engineer hired by the builder See FED R CIV P 34(a)(2) Accordingly if an examinee concludes that the report was prepared in anticipation of litigation the examinee should also conclude that the builder is not entitled to see the report]

Point Two(a) (30) Because the builder anticipated that it might be involved in litigation concerning its contract with the customer the builder acted wrongfully in destroying emails that were relevant to the housersquos construction even though the emails were destroyed pursuant to a routine document retention plan

As noted above a party to a lawsuit in federal court ldquomay obtain discovery regarding any nonprivileged matter that is relevant to any partyrsquos claim or defenserdquo FED R CIV P 26(b)(1) This includes emails and other electronically stored information FED R CIV P 34(a)(1)(A) Here the customer has requested all the builderrsquos emails pertaining to work done on the foundation of the house Ordinarily the builder would be obliged to turn over this information which is relevant to the customerrsquos defense that the housersquos foundation was poorly constructed

Unfortunately the emails in question no longer exist because the builder destroyed them on August 2

In general spoliation of evidence (destruction or alteration of evidence) is improper if the party who destroyed or altered the evidence ldquohas notice that the evidence is relevant to litigation or should have known that the evidence may be relevant to future litigationrdquo Fujitsu Ltd v Federal Express Corp 247 F3d 423 436 (2d Cir 2001) It is improper for a party to destroy electronic information relevant to pending litigation even if the destruction occurs before there is any request or order seeking the information See eg Leon v IDX Sys Corp 464 F3d 951 (9th Cir 2006) (plaintiffrsquos intentional destruction of computer files warranted dismissal even

20

In this case the builderrsquos destruction of the emails was pursuant to a routine document retention plan The Federal Rules provide expressly that in the absence of ldquoexceptional circumstancesrdquo parties should not be sanctioned for the loss of electronically stored information when the loss occurs pursuant to ldquoroutine good-faith operation of an electronic information systemrdquo FED R CIV P 37(e) However when a party anticipates litigation ldquoit must suspend its routine document retentiondestruction policy and put in place a lsquolitigation holdrsquo to ensure the preservation of relevant documentsrdquo Zubulake v UBS Warburg LLC 220 FRD 212 218 (SDNY 2003)

Federal Civil Procedure Analysis

though spoliation occurred before order compelling discovery) Similarly the duty to preserve evidence applies to a party who anticipates litigation even if litigation has not yet been commenced See THE SEDONA PRINCIPLES BEST PRACTICES RECOMMENDATIONS amp PRINCIPLES FOR ADDRESSING ELECTRONIC DOCUMENT PRODUCTION 70 cmt 14a (2d ed 2007)

The builder destroyed the emails on August 2 At that time the builder knew that litigation was a possibility because the builder had already directed its attorney to prepare a draft complaint for possible filing Knowing that litigation was a possibility the builder had a duty to take steps to preserve evidence including the emails in question See generally Fujitsu Ltd

Thus the builderrsquos destruction of potentially relevant emails at a time when it knew that litigation was a possibility was improper It had a duty to preserve evidence and it breached that duty

[NOTE Because courts have used different words to describe the test for when evidence must be preserved an examineersquos precise formulation of the test is not critical]

Point Two(b) (30) In determining appropriate sanctions for spoliation courts consider both the level of culpability of the spoliating party and the degree of prejudice the loss of evidence has caused the other party Here the builderrsquos destruction of evidence does not appear to have been willful nor is it likely to pose a significant obstacle to the customerrsquos defense Any sanctions imposed by the court should be modest

Federal courts have inherent power to control the litigation process and can sanction misbehavior including spoliation even when there has been no specific violation of the Federal Rules of Civil Procedure See generally Chambers v NASCO Inc 501 US 32 (1991) (discussing courtrsquos inherent power to control the litigation process) The range of available sanctions is broad It can include such sanctions as the payment of expenses incurred by the other party as a result of the destruction of the evidence an instruction to the jury authorizing it to draw an adverse inference from the destruction of the evidence a shifting of the burden of proof on the relevant issue or even judgment against the responsible party See eg Residential Funding Corp v DeGeorge Financial Corp 306 F3d 99 108 (2d Cir 2002) (adverse inference) Silvestri v General Motors Corp 271 F3d 583 593 (4th Cir 2001) (possibility of dismissal) Cf FED R CIV P 37(b)(2)(A) (listing remedies for failure to comply with discovery obligations)

In determining appropriate sanctions for spoliation courts consider both the level of culpability of the spoliating party and the degree of prejudice the loss of evidence has caused the other party Many courts impose severe sanctions (such as an adverse-inference instruction or the entry of judgment against the spoliating party) only when there is evidence of bad faith in the form of an intentional effort to hide information Eg Greyhound Lines Inc v Wade 485 F3d 1032 1035 (8th Cir 2007) (spoliation sanction requires intentional destruction out of desire ldquoto suppress the truthrdquo) However other courts have said that negligence in preserving evidence can

21

Federal Civil Procedure Analysis

support an adverse-inference instruction See Residential Funding 306 F3d at 108 (negligence enough under some circumstances)

Although a court might well order an evidentiary hearing on the issue of sanctions the facts presented do not seem appropriate for severe sanctions First the evidence was destroyed pursuant to the builderrsquos standard document retention plan and there is no evidence that the builder deliberately failed to suspend its usual procedures with the purpose of allowing the destruction of evidence Second the loss of this evidence will not severely hinder the customerrsquos presentation of his case The central issue is whether the foundation of the house was properly constructed If the construction job was poorly done the customer can present evidence derived from inspection of the premises to prove that point The customer can also depose witnesses about any issues that arose during construction

Under the circumstances a court is not likely to impose particularly severe sanctions although it might shift the burden to the builder to show that the foundation was properly constructed or it might require the builder to reimburse any expenses the customer incurs to discover and prove the facts about issues or disputes that arose during construction of the foundation

[NOTE The result reached by the examinee is less important than the examineersquos recognition that (a) a range of sanctions is available to the court and (b) the appropriate sanction depends both on the culpability of the builder and the prejudice suffered by the customer]

22

CRIMINAL LAW AND PROCEDURE ANALYSIS (Criminal Law and Procedure IIA amp D VE amp F)

ANALYSIS

Legal Problems

(1) Did charging the defendant with both theft and burglary constitute double jeopardy

(2) Did the jury instruction violate the due process clause either by relieving the prosecution of the burden of proving the element of intent or by shifting the burden to the defendant to disprove that element

(3) Did the sentence imposed in this case for the theft conviction unconstitutionally deprive the defendant of his right to a jury trial on the issue of the value of the stolen item

DISCUSSION

Summary

The trial court properly denied the defendantrsquos pretrial motion to dismiss the charges on double jeopardy grounds The defendant may be charged with and convicted of both theft and burglary Each of the charges has an element that the other does not Neither charge is a lesser-included offense nor are they multiplicitous Thus charging both theft and burglary does not violate double jeopardy

The jury instruction on the burglary charge was constitutionally flawed It could have been reasonably understood by the jury as either (1) an irrebuttable conclusive presumption (which relieved the prosecution of proving the element of intent and removed the issue from the jury) or (2) a rebuttable mandatory presumption (which unconstitutionally shifted the burden of proof on an element of a charged offense from the prosecution to the defendant)

Because the four-year sentence imposed by the judge was based on the judgersquos finding by a preponderance of the evidence that the value of the stolen ring exceeded $5000 the sentence violates the defendantrsquos right to a jury determination beyond a reasonable doubt of the value of the ring

Point One (30) Charging the defendant with theft and burglary did not constitute double jeopardy

The Double Jeopardy Clause of the Fifth Amendment provides that a person shall not be twice put in jeopardy for the ldquosame offenserdquo Thus the question is whether the elements of the theft charge are wholly contained in the burglary charge or vice versa If the elements of the lesser charge (theft) are not wholly contained in the greater charge (burglary)mdashie if each charge requires proof of a fact that the other does notmdashthen convicting the defendant of both crimes would not violate double jeopardy even when the two offenses occurred at the same time and are thus arguably part of the ldquosame transactionrdquo Blockburger v United States 284 US 299 304 (1932) See also Albernaz v United States 450 US 333 344 n3 (1981) United States v Dixon 509 US 688 704 (1993)

23

Criminal Law and Procedure Analysis

Here theft and burglary each require proof of an element not required for the other crime Burglary may be defined differently in different jurisdictions However it almost invariably requires entry into a building or dwelling of another with the specific intent to commit a felony therein and the crime of burglary is complete upon the entry into the building or dwelling with such intent See eg Cannon v Oklahoma 827 P2d 1339 1342 (Okla Crim App 1992) In contrast theft which also may be defined differently in different states almost invariably requires the taking and carrying away of an item of personal property belonging to another with the intent to steal or permanently deprive the owner of possession

Here the ldquotakingrdquo or ldquostealingrdquo element is not contained in the definition of burglary and the ldquoentryrdquo element of burglary is not contained in the definition of theft Because theft is not a lesser-included offense of burglary and burglary is not a lesser-included offense of theft charging the defendant for both burglary and theft did not violate double jeopardy and the court properly denied the defense motion on those grounds Yparrea v Dorsey 64 F3d 577 579ndash80 (10th Cir 1995) citing Blockburger 284 US at 304

Finally the defendantrsquos motion to dismiss all the charges on double jeopardy grounds was improper because if both charges were for the same offense the motion should have requested dismissal of one charge not both

Point Two (35) The jury instruction on the burglary charge violated the Due Process Clause because it created either (1) an irrebuttable conclusive presumption (which relieved the prosecution of proving the element of intent and removed that issue from the jury) or (2) a rebuttable mandatory presumption (which unconstitutionally shifted the burden of proof on an element of a charged offense to the defendant)

The Supreme Court has interpreted the Due Process Clause of the US Constitution to require that the prosecution prove all elements of an offense beyond a reasonable doubt See In re Winship 397 US 358 364 (1970) The burden of proof cannot be shifted to the defendant by presuming an essential element upon proof of other elements of the offense because shifting the burden of persuasion with respect to any element of a criminal offense is contrary to the Due Process Clause See Mullaney v Wilbur 421 US 684 (1975)

The crime of burglary includes entry into a building or dwelling with the specific intent to commit a felony therein The requirement that the prosecutor prove beyond a reasonable doubt that the defendant had this specific intent distinguishes burglary from general-intent crimes like trespass See Sandstrom v Montana 442 US 510 523 (1979)

Here the jury was instructed that if ldquoafter consideration of all the evidence presented by the prosecution and defense you find beyond a reasonable doubt that the defendant entered the dwelling without the ownersrsquo consent you may presume that the defendant entered with the intent to commit a felony thereinrdquo This instruction was unconstitutional because it created either an irrebuttable conclusive presumption or a rebuttable mandatory presumption

A conclusive presumption is ldquoan irrebuttable direction by the court to find intent once convinced of the facts triggering the presumptionrdquo Id at 517 Here the jurors were instructed that once the prosecutor established that the defendant entered the neighborsrsquo house without consent they ldquomay presumerdquo that he intended to commit a felony therein The jurors may have reasonably concluded from this instruction that if they found that the defendant intended to enter his neighborsrsquo home without permission they must further find that he entered with the specific intent to commit a felony therein Because this instruction could operate as a conclusive

24

Criminal Law and Procedure Analysis

irrebuttable presumption by eliminating intent ldquoas an ingredient of the offenserdquo it violated due process by relieving the prosecution of the burden of proof for this element Id at 522

In the alternative the jury instruction could have been reasonably understood to create a rebuttable mandatory presumption which ldquotells [the jury] they must find the elemental fact upon proof of the basic fact at least unless the defendant has come forward with some evidence to rebut the presumed connection between the two factsrdquo County Court of Ulster County New York v Allen 442 US 140 157 (1979) The due process problem created by rebuttable mandatory presumptions is that ldquo[t]o the extent that the trier of fact is forced to abide by the presumption and may not reject it based on an independent evaluation of the particular facts presented by the State the analysis of the presumptionrsquos constitutional validity is logically divorced from those facts and based on the presumptionrsquos accuracy in the run of casesrdquo Id at 159

Unlike irrebuttable conclusive presumptions rebuttable mandatory presumptions are not always per se violations of the Due Process Clause However the Supreme Court of the United States has held that jury instructions that could reasonably be understood as shifting the burden of proof to the defendant on an element of the offense are unconstitutional Francis v Franklin 471 US 307 (1985) Here the argument that the jury instruction operated as a rebuttable mandatory presumption is supported by the fact that the judge also instructed the jury to ldquoconsider[ ] all the evidence presented by the prosecution and defenserdquo However even if the instruction created a rebuttable mandatory presumption it would be unconstitutional because it shifted the burden to the defense on an element of the offense Sandstrom 442 US at 524 Mullaney 421 US at 686

[NOTE Whether an examinee identifies the jury instruction as containing a ldquoconclusiverdquo or ldquomandatoryrdquo presumption is less important than the examineersquos analysis of the constitutional infirmities]

Point Three (35) The trial court violated the defendantrsquos Sixth Amendment right to a jury trial on an essential element of the offense when it found by a preponderance of the evidence that the ring was worth over $5000 and increased the defendantrsquos sentence based on this finding

In the statutory scheme under which the defendant was tried and convicted a Class D felony theft is defined as theft of item(s) with a value between $2500 and $10000 The jury found that the value of the diamond ring was at least $2500 and convicted the defendant of felony theft However at sentencing the trial court made a separate finding by a preponderance of the evidence that the value of the ring was greater than $5000 Following the statutersquos two-tiered sentencing scheme the judge then imposed on the defendant a sentence that was one year longer than the maximum that would otherwise have been allowed

The judgersquos sentence was unconstitutional because it violated the defendantrsquos Sixth Amendment right to a jury trial on this question The Supreme Court held in Apprendi v New Jersey 530 US 466 (2000) that ldquo[o]ther than the fact of a prior conviction any fact that increases the penalty for a crime beyond the prescribed statutory maximum must be submitted to a jury and proved beyond a reasonable doubtrdquo because ldquo[i]t is unconstitutional for a legislature to remove from the jury the assessment of facts that increase the prescribed range of penalties to which a criminal defendant is exposed [because] such facts must be established by proof beyond a reasonable doubtrdquo Id The Court reaffirmed Apprendi in Blakely v Washington 542 US 296 (2004) holding that the ldquolsquostatutory maximumrsquo for Apprendi purposes is the maximum sentence a judge may impose solely on the basis of the facts reflected in the jury verdict or admitted by the defendantrdquo Id at 303 (emphasis in original) In United States v Booker 543 US 220 (2005)

25

Criminal Law and Procedure Analysis

the Court relied on Blakely and Apprendi to conclude that protecting a defendantrsquos Sixth Amendment right to a jury trial required that ldquo[a]ny fact which is necessary to support a sentence exceeding the maximum authorized by the facts established by a plea of guilty or a jury verdict must be admitted by the defendant or proved to a jury beyond a reasonable doubtrdquo Id at 244

Thus in order to constitutionally increase a sentence above the statutory maximum of three years the jury must have found beyond a reasonable doubt that the value of the ring exceeded $5000 Here the court made the finding based on an appraisal proffered by the prosecutor only at sentencing and the judgersquos finding was by a preponderance of the evidence rather than beyond a reasonable doubt

26

AGENCY AND PARTNERSHIP ANALYSIS __________ (Agency and Partnership VA amp C VI)

ANALYSIS

Legal Problems

(1) Is a partner in a general partnership personally liable on a claim arising from misrepresentations by another partner made in the course of the partnership business

(2) Does a newly admitted partner in a general partnership become personally liable on existing claims against the partnership

(3) After the filing by a general partnership of a statement of qualification as a limited liability partnership are the partners personally liable as partners on (a) an existing claim against the general partnership and (b) a claim against the partnership that arose after the filing

DISCUSSION

Summary

Adam and Ben formed a general partnership under which they were jointly and severally liable for obligations of the partnership Thus Adam was personally liable for misrepresentations by Ben made in the ordinary course of the partnership business

Upon joining the general partnership Diane became personally liable for the obligations of the partnership arising after her admission but not for obligations pre-existing her admission such as the collectorrsquos claim

By filing a statement of qualification the three partners properly elected limited liability partnership status As partners in an LLP none of the three partners is personally liable as a partner for partnership obligations arising after the election such as the claim by the driverrsquos estate The election however does not change their personal liability on pre-existing claims that arose before the election such as the collectorrsquos claim

Point One (30) As a general partner of Empire a general partnership Adam became personally liable on the collectorrsquos claim a valid claim against the partnership that arose because of Benrsquos wrongful act in the ordinary course of the partnership business

When the collectorrsquos claim arose Empire was a general partnership composed of Adam and Ben Under UPA (1997) sect 306(a) partners of a general partnership are liable jointly and severally for all obligations of the partnership Under UPA (1997) sect 305(a) the partnership could become obligated for the loss caused to the collector as a result of the misrepresentation by Ben provided he was acting in the ordinary course of the partnership business Because there was no statement that limited his partnership authority Ben as partner was ldquoan agent of the partnership for the purpose of its businessrdquo See UPA (1997) sect 301(1) Benrsquos misrepresentation to the collector even if intentional appears to be in the ordinary course of the partnershiprsquos business of dealing

27

Agency and Partnership Analysis

in antique cars Thus Benrsquos wrongful act created a partnership obligation for which Adam was jointly and severally liable

[NOTE Generally a partnership creditor must ldquoexhaust the partnershiprsquos assets before levying on a judgment debtor partnerrsquos individual property where the partner is personally liable for the partnership obligationrdquo as a result of his status as a partner UPA (1997) sect 307 cmt 4 As the UPA comments explain this places Adam more in the position of guarantor than principal debtor on the partnership obligation Id cmt 4 Although an examinee might discuss this point the call focuses on whether Adam is personally liable not how the liability might be enforced]

Point Two (30) Because the collectorrsquos claim arose before Diane joined Empire Diane did not become personally liable on the claim

Diane was admitted to Empire when it was a general partnership and after the collectorrsquos claim arose While the general rule under UPA (1997) sect 306(a) is that the partners of a general partnership are liable jointly and severally for all obligations of the partnership there is a special rule for partners who are admitted during the duration of the partnership Under UPA (1997) sect 306(b) a person admitted to an existing partnership is not personally liable for any partnership obligations incurred before the personrsquos admission Because Diane was admitted to Empire after the collectorrsquos claim arose Diane is not personally liable on the claim

Dianersquos knowledge of the pre-existing claim and her stated concern about becoming liable on the collectorrsquos claim do not change her personal nonliability to the collector Although partners who have a liability shield can assume liability to third parties through private contractual guarantees or modifications to the partnership agreement Dianersquos stated concern constituted neither a guaranty to the collector nor ldquoan intentional waiver of liability protectionsrdquo See UPA (1997) sect 306 cmt 3 (describing methods for waiver of liability protections under sect 306(c) applicable in limited liability partnerships)

At most Diane will lose her investment in the partnership as a result of the collectorrsquos claim Although Diane did not become personally liable on the collectorrsquos claim when she joined the partnership the $250000 she contributed to the partnership is ldquoat risk for the satisfaction of existing partnership debtsrdquo UPA (1997) sect 306 cmt 2

Point Three (40) Filing the statement of qualification was effective to elect limited liability partnership status Despite this new status Adam and Ben remain personally liable on the collectorrsquos claim which arose before the election But as partners in an LLP neither Adam Ben nor Diane is personally liable as a partner on the driverrsquos estatersquos claim which arose after the election

Under UPA (1997) sect 1001 a general partnership can make an election and become a limited liability partnershipmdashif the partners approve the conversion by a vote equivalent to that necessary to amend the partnership agreement and the partnership then files a statement of qualification that specifies the name of the partnership its principal office and its election to be an LLP Here the partners agreed unanimouslymdashsufficient to amend their agreement under UPA (1997) sect 401(j)mdashand the statement of qualification was filed In addition the name of Empire LLP properly included an appropriate ending ldquoLLPrdquo See UPA (1997) sect 1002

Although another way to effectuate a ldquoconversionrdquo (as suggested by Benrsquos lawyer) is to form a new LLP and transfer the assets of the old general partnership to the new LLP the

28

Agency and Partnership Analysis

method used here (approval by the partners and the filing of a statement of qualification) is also sufficient to create LLP status

Thus Empire became Empire LLP as of the date of filing of the statement of qualification See UPA (1997) sect 1001 What effect did this have on the collectorrsquos claim which predated the filing According to UPA (1997) sect 306(c) an obligation incurred while a partnership is an LLP is solely a partnership obligation As the collectorrsquos claim predated the LLP Adam and Ben remain personally liable on the collectorrsquos claim Diane on the other hand was not personally liable on the collectorrsquos claim either before or after the filing of the statement of qualification See Point Two above

The driverrsquos estatersquos claim arose after Empire became Empire LLP Under UPA (1997) sect 306(c) an obligation incurred while a partnership is an LLP is solely a partnership obligationThus Adam Ben and Diane as partners are all protected from personal liability on the driverrsquos estatersquos claim But there may be personal liability if any of them was negligent or otherwise acted wrongfully by not informing the buyer of the bad suspension that caused the accident

29

National Conference of Bar Examiners 302 South Bedford Street | Madison WI 53703-3622 Phone 608-280-8550 | Fax 608-280-8552 | TDD 608-661-1275

wwwncbexorg e-mail contactncbexorg

  • Preface
  • Description of the MEE
  • Instructions
  • February 2014 Questions
    • Constitutinal Law Question
    • Trusts and Future Interests Question
    • Secured Transactions Question
    • Federal Civil Procedure Question
    • Criminal Law and Procedure Question
    • Agency and Partnership Question
      • February 2014 Analyses
        • Constitutional Law Analysis
        • Trust and Future Interests Analysis
        • Secured Transactions Analysis
        • Federal Civil Procedure Analysis
        • Criminal Law and Procedure Analysis
        • Agency and Partnership Analysis
            • ltlt13 ASCII85EncodePages false13 AllowTransparency false13 AutoPositionEPSFiles true13 AutoRotatePages None13 Binding Left13 CalGrayProfile (Dot Gain 20)13 CalRGBProfile (sRGB IEC61966-21)13 CalCMYKProfile (US Web Coated 050SWOP051 v2)13 sRGBProfile (sRGB IEC61966-21)13 CannotEmbedFontPolicy Error13 CompatibilityLevel 1413 CompressObjects Tags13 CompressPages true13 ConvertImagesToIndexed true13 PassThroughJPEGImages true13 CreateJobTicket false13 DefaultRenderingIntent Default13 DetectBlends true13 DetectCurves 0000013 ColorConversionStrategy CMYK13 DoThumbnails false13 EmbedAllFonts true13 EmbedOpenType false13 ParseICCProfilesInComments true13 EmbedJobOptions true13 DSCReportingLevel 013 EmitDSCWarnings false13 EndPage -113 ImageMemory 104857613 LockDistillerParams false13 MaxSubsetPct 10013 Optimize true13 OPM 113 ParseDSCComments true13 ParseDSCCommentsForDocInfo true13 PreserveCopyPage true13 PreserveDICMYKValues true13 PreserveEPSInfo true13 PreserveFlatness true13 PreserveHalftoneInfo false13 PreserveOPIComments true13 PreserveOverprintSettings true13 StartPage 113 SubsetFonts true13 TransferFunctionInfo Apply13 UCRandBGInfo Preserve13 UsePrologue false13 ColorSettingsFile ()13 AlwaysEmbed [ true13 ]13 NeverEmbed [ true13 ]13 AntiAliasColorImages false13 CropColorImages true13 ColorImageMinResolution 30013 ColorImageMinResolutionPolicy OK13 DownsampleColorImages true13 ColorImageDownsampleType Bicubic13 ColorImageResolution 30013 ColorImageDepth -113 ColorImageMinDownsampleDepth 113 ColorImageDownsampleThreshold 15000013 EncodeColorImages true13 ColorImageFilter DCTEncode13 AutoFilterColorImages true13 ColorImageAutoFilterStrategy JPEG13 ColorACSImageDict ltlt13 QFactor 01513 HSamples [1 1 1 1] VSamples [1 1 1 1]13 gtgt13 ColorImageDict ltlt13 QFactor 01513 HSamples [1 1 1 1] VSamples [1 1 1 1]13 gtgt13 JPEG2000ColorACSImageDict ltlt13 TileWidth 25613 TileHeight 25613 Quality 3013 gtgt13 JPEG2000ColorImageDict ltlt13 TileWidth 25613 TileHeight 25613 Quality 3013 gtgt13 AntiAliasGrayImages false13 CropGrayImages true13 GrayImageMinResolution 30013 GrayImageMinResolutionPolicy OK13 DownsampleGrayImages true13 GrayImageDownsampleType Bicubic13 GrayImageResolution 30013 GrayImageDepth -113 GrayImageMinDownsampleDepth 213 GrayImageDownsampleThreshold 15000013 EncodeGrayImages true13 GrayImageFilter DCTEncode13 AutoFilterGrayImages true13 GrayImageAutoFilterStrategy JPEG13 GrayACSImageDict ltlt13 QFactor 01513 HSamples [1 1 1 1] VSamples [1 1 1 1]13 gtgt13 GrayImageDict ltlt13 QFactor 01513 HSamples [1 1 1 1] VSamples [1 1 1 1]13 gtgt13 JPEG2000GrayACSImageDict ltlt13 TileWidth 25613 TileHeight 25613 Quality 3013 gtgt13 JPEG2000GrayImageDict ltlt13 TileWidth 25613 TileHeight 25613 Quality 3013 gtgt13 AntiAliasMonoImages false13 CropMonoImages true13 MonoImageMinResolution 120013 MonoImageMinResolutionPolicy OK13 DownsampleMonoImages true13 MonoImageDownsampleType Bicubic13 MonoImageResolution 120013 MonoImageDepth -113 MonoImageDownsampleThreshold 15000013 EncodeMonoImages true13 MonoImageFilter CCITTFaxEncode13 MonoImageDict ltlt13 K -113 gtgt13 AllowPSXObjects false13 CheckCompliance [13 None13 ]13 PDFX1aCheck false13 PDFX3Check false13 PDFXCompliantPDFOnly false13 PDFXNoTrimBoxError true13 PDFXTrimBoxToMediaBoxOffset [13 00000013 00000013 00000013 00000013 ]13 PDFXSetBleedBoxToMediaBox true13 PDFXBleedBoxToTrimBoxOffset [13 00000013 00000013 00000013 00000013 ]13 PDFXOutputIntentProfile ()13 PDFXOutputConditionIdentifier ()13 PDFXOutputCondition ()13 PDFXRegistryName ()13 PDFXTrapped False1313 CreateJDFFile false13 Description ltlt13 ARA 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 BGR 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 CHS ltFEFF4f7f75288fd94e9b8bbe5b9a521b5efa7684002000410064006f006200650020005000440046002065876863900275284e8e9ad88d2891cf76845370524d53705237300260a853ef4ee54f7f75280020004100630072006f0062006100740020548c002000410064006f00620065002000520065006100640065007200200035002e003000204ee553ca66f49ad87248672c676562535f00521b5efa768400200050004400460020658768633002gt13 CHT ltFEFF4f7f752890194e9b8a2d7f6e5efa7acb7684002000410064006f006200650020005000440046002065874ef69069752865bc9ad854c18cea76845370524d5370523786557406300260a853ef4ee54f7f75280020004100630072006f0062006100740020548c002000410064006f00620065002000520065006100640065007200200035002e003000204ee553ca66f49ad87248672c4f86958b555f5df25efa7acb76840020005000440046002065874ef63002gt13 CZE 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 DAN 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 DEU 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 ESP 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 ETI 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 FRA 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 GRE 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 HEB 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 HRV (Za stvaranje Adobe PDF dokumenata najpogodnijih za visokokvalitetni ispis prije tiskanja koristite ove postavke Stvoreni PDF dokumenti mogu se otvoriti Acrobat i Adobe Reader 50 i kasnijim verzijama)13 HUN 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 ITA ltFEFF005500740069006c0069007a007a006100720065002000710075006500730074006500200069006d0070006f007300740061007a0069006f006e00690020007000650072002000630072006500610072006500200064006f00630075006d0065006e00740069002000410064006f00620065002000500044004600200070006900f900200061006400610074007400690020006100200075006e00610020007000720065007300740061006d0070006100200064006900200061006c007400610020007100750061006c0069007400e0002e0020004900200064006f00630075006d0065006e007400690020005000440046002000630072006500610074006900200070006f00730073006f006e006f0020006500730073006500720065002000610070006500720074006900200063006f006e0020004100630072006f00620061007400200065002000410064006f00620065002000520065006100640065007200200035002e003000200065002000760065007200730069006f006e006900200073007500630063006500730073006900760065002egt13 JPN ltFEFF9ad854c18cea306a30d730ea30d730ec30b951fa529b7528002000410064006f0062006500200050004400460020658766f8306e4f5c6210306b4f7f75283057307e305930023053306e8a2d5b9a30674f5c62103055308c305f0020005000440046002030d530a130a430eb306f3001004100630072006f0062006100740020304a30883073002000410064006f00620065002000520065006100640065007200200035002e003000204ee5964d3067958b304f30533068304c3067304d307e305930023053306e8a2d5b9a306b306f30d530a930f330c8306e57cb30818fbc307f304c5fc59808306730593002gt13 KOR ltFEFFc7740020c124c815c7440020c0acc6a9d558c5ec0020ace0d488c9c80020c2dcd5d80020c778c1c4c5d00020ac00c7a50020c801d569d55c002000410064006f0062006500200050004400460020bb38c11cb97c0020c791c131d569b2c8b2e4002e0020c774b807ac8c0020c791c131b41c00200050004400460020bb38c11cb2940020004100630072006f0062006100740020bc0f002000410064006f00620065002000520065006100640065007200200035002e00300020c774c0c1c5d0c11c0020c5f40020c2180020c788c2b5b2c8b2e4002egt13 LTH 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 LVI 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 NLD (Gebruik deze instellingen om Adobe PDF-documenten te maken die zijn geoptimaliseerd voor prepress-afdrukken van hoge kwaliteit De gemaakte PDF-documenten kunnen worden geopend met Acrobat en Adobe Reader 50 en hoger)13 NOR ltFEFF004200720075006b00200064006900730073006500200069006e006e007300740069006c006c0069006e00670065006e0065002000740069006c002000e50020006f0070007000720065007400740065002000410064006f006200650020005000440046002d0064006f006b0075006d0065006e00740065007200200073006f006d00200065007200200062006500730074002000650067006e0065007400200066006f00720020006600f80072007400720079006b006b0073007500740073006b00720069006600740020006100760020006800f800790020006b00760061006c0069007400650074002e0020005000440046002d0064006f006b0075006d0065006e00740065006e00650020006b0061006e002000e50070006e00650073002000690020004100630072006f00620061007400200065006c006c00650072002000410064006f00620065002000520065006100640065007200200035002e003000200065006c006c00650072002000730065006e006500720065002egt13 POL 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 PTB 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 RUM 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 RUS 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 SKY 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 SLV 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 SUO 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 SVE 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 TUR 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 UKR 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 ENU (Use these settings to create Adobe PDF documents best suited for high-quality prepress printing Created PDF documents can be opened with Acrobat and Adobe Reader 50 and later)13 gtgt13 Namespace [13 (Adobe)13 (Common)13 (10)13 ]13 OtherNamespaces [13 ltlt13 AsReaderSpreads false13 CropImagesToFrames true13 ErrorControl WarnAndContinue13 FlattenerIgnoreSpreadOverrides false13 IncludeGuidesGrids false13 IncludeNonPrinting false13 IncludeSlug false13 Namespace [13 (Adobe)13 (InDesign)13 (40)13 ]13 OmitPlacedBitmaps false13 OmitPlacedEPS false13 OmitPlacedPDF false13 SimulateOverprint Legacy13 gtgt13 ltlt13 AddBleedMarks false13 AddColorBars false13 AddCropMarks false13 AddPageInfo false13 AddRegMarks false13 ConvertColors ConvertToCMYK13 DestinationProfileName ()13 DestinationProfileSelector DocumentCMYK13 Downsample16BitImages true13 FlattenerPreset ltlt13 PresetSelector MediumResolution13 gtgt13 FormElements false13 GenerateStructure false13 IncludeBookmarks false13 IncludeHyperlinks false13 IncludeInteractive false13 IncludeLayers false13 IncludeProfiles false13 MultimediaHandling UseObjectSettings13 Namespace [13 (Adobe)13 (CreativeSuite)13 (20)13 ]13 PDFXOutputIntentProfileSelector DocumentCMYK13 PreserveEditing true13 UntaggedCMYKHandling LeaveUntagged13 UntaggedRGBHandling UseDocumentProfile13 UseDocumentBleed false13 gtgt13 ]13gtgt setdistillerparams13ltlt13 HWResolution [2400 2400]13 PageSize [612000 792000]13gtgt setpagedevice13

Page 5: February 2014 MEE Questions and AnalysesPreface The Multistate Essay Examination (MEE) is developed by the National Conference of Bar Examiners (NCBE). This publication includes the

Instructions The back cover of each test booklet contains the following instructions

You will be instructed when to begin and when to stop this test Do not break the seal on this booklet until you are told to begin

You may answer the questions in any order you wish Do not answer more than one question in each answer booklet If you make a mistake or wish to revise your answer simply draw a line through the material you wish to delete

If you are using a laptop computer to answer the questions your jurisdiction will provide you with specific instructions

Read each fact situation very carefully and do not assume facts that are not given in the question Do not assume that each question covers only a single area of the law some of the questions may cover more than one of the areas you are responsible for knowing

Demonstrate your ability to reason and analyze Each of your answers should show an understanding of the facts a recognition of the issues included a knowledge of the applicable principles of law and the reasoning by which you arrive at your conclusions The value of your answer depends not as much upon your conclusions as upon the presence and quality of the elements mentioned above

Clarity and conciseness are important but make your answer complete Do not volunteer irrelevant or immaterial information

Answer all questions according to generally accepted fundamental legal principles unless your testing jurisdiction has instructed you to answer according to local case or statutory law

NOTE Examinees testing in UBE jurisdictions must answer according to generally accepted fundamental legal principles rather than local case or statutory law

iii

February 2014 MEE

QUESTIONS Constitutional Law

Trusts and Future Interests Secured Transactions

Federal Civil Procedure Criminal Law and Procedure

Agency and Partnership

CONSTITUTIONAL LAW QUESTION ___________

A city ordinance required each downtown business to install high-powered halogen floodlights that would illuminate the property owned by that business and the adjoining sidewalks A study commissioned by the city estimated that installation of the floodlights would cost a typical business about $1000 but that increased business traffic due to enhanced public safety especially after dark would likely offset this cost

A downtown restaurant applied to the city for a building permit to construct an addition that would increase its seating capacity In its permit application the restaurant accurately noted that its current facility did not have sufficient seating to accommodate all potential customers during peak hours The city approved the permit on the condition that the restaurant grant the city an easement over a narrow strip of the restaurantrsquos property to be used by the city to install video surveillance equipment that would cover nearby public streets and parking lots The city based its permit decision entirely on findings that the increased patronage that would result from the increased capacity of the restaurant might also attract additional crime to the neighborhood and that installing video surveillance equipment might alleviate that problem

The restaurant has challenged both the ordinance requiring it to install floodlights and the easement condition imposed on approval of the building permit

1 Under the Fifth Amendment as applied to the states through the Fourteenth Amendment is the city ordinance requiring the restaurant to install floodlights an unconstitutional taking Explain

2 Under the Fifth Amendment as applied to the states through the Fourteenth Amendment is the cityrsquos requirement that the restaurant grant the city an easement as a condition for obtaining the building permit an unconstitutional taking Explain

3

TRUSTS AND FUTURE INTERESTS QUESTION _______________

Ten years ago a testator died survived by his only children a son age 26 and a daughter age 18

A testamentary trust was created under the testatorrsquos duly probated will The will specified that all trust income would be paid to the son during the sonrsquos lifetime and that upon the sonrsquos death the trust would terminate and trust principal would be distributed to the testatorrsquos ldquograndchildren who shall surviverdquo the son The testator provided for his daughter in other sections of the will

Five years ago the trustee of the testamentary trust purchased an office building with $500000 from the trust principal Other than this building the trust assets consist of publicly traded securities

Last year the trustee received $30000 in rents from the office building The trustee also received with respect to the securities owned by the trust cash dividends of $20000 and a stock dividend of 400 shares of Acme Corp common stock distributed to the trust by Acme Corp

Eight months ago the trustee sold the office building for $700000

Six months ago the son delivered a letter to the trustee stating ldquoI hereby disclaim any interest I may have in the income interest of the trustrdquo On the date the son delivered this letter to the trustee the son had no living children the daughter had one living minor child

A statute in this jurisdiction provides that ldquoa disclaimer of any interest created by will is valid only if made within nine months after the testatorrsquos death and if an interest is validly disclaimed the disclaiming party is deemed to have predeceased the testatorrdquo

1 How should the rents sales proceeds cash dividends and stock dividends received prior to the trusteersquos receipt of the sonrsquos letter have been allocated between trust principal and income Explain

2 How if at all does the sonrsquos letter to the trustee affect the future distribution of trust income and principal Explain

4

SECURED TRANSACTIONS QUESTION

On March 1 the owner of a manufacturing business entered into negotiations with a bank to obtain a loan of $100000 for the business The bank loan officer informed the business owner that the interest rate for a loan would be lower if the repayment obligation were secured by all the businessrsquos present and future equipment The loan officer also informed the business owner that the bank could not commit to making the loan until its credit investigation was completed but that funds could be advanced faster following loan approval if a financing statement with respect to the transaction were filed in advance Accordingly the business owner signed a form on behalf of the business authorizing the bank to file a financing statement with respect to the proposed transaction The bank properly filed a financing statement the next day correctly providing the name of the business as the debtor and indicating ldquoequipmentrdquo as the collateral

On March 15 the business owner had heard nothing from the bank about whether the loan had been approved so the business owner approached a finance company for a loan The finance company quickly agreed to lend $100000 to the business secured by all the businessrsquos present and future equipment That same day the finance company loaned to the business $100000 and the business owner signed an agreement obligating the business to repay the loan and granting the finance company a security interest in all the businessrsquos ldquopresent and future equipmentrdquo to secure the repayment obligation Also on that day the finance company properly filed a financing statement correctly providing the businessrsquos name as the debtor and indicating ldquoequipmentrdquo as the collateral

On March 21 the bank loan officer contacted the business owner and indicated that the loan application had been approved On the next day March 22 the bank loaned the business $100000 The loan agreement signed by the owner on behalf of the business granted the bank a security interest in all the businessrsquos ldquopresent and future equipmentrdquo

On April 10 the business sold an item of manufacturing equipment to a competitor for $20000 This was the first time the business had ever sold any of its equipment The competitor paid the purchase price in cash and took possession of the equipment that day The competitor acted in good faith at all times and had no knowledge of the businessrsquos prior transactions with the bank and the finance company

The business has defaulted on its obligations with respect to the loans from the bank and the finance company Each of them has asserted a claim to all the businessrsquos equipment as well as to the item of equipment sold to the businessrsquos competitor

Assume that the business owner had the authority to enter into all these transactions on behalf of the business

1 As between the bank and the finance company which has a superior claim to the businessrsquos equipment Explain

2 Do the claims of the bank and the finance company to the businessrsquos equipment continue in the item of equipment sold to the competitor Explain

5

FEDERAL CIVIL PROCEDURE QUESTION

A builder constructed a vacation house for an out-of-state customer on the customerrsquos land The house was completed on June 1 at which point the customer still owed $200000 of the $800000 contract price which was payable in full five days later

On June 14 the basement of the house was flooded with two inches of water during a heavy rainfall When the customer complained the builder told the customer ldquoThe flooding was caused by poorly designed landscaping Our work is fine and fully up to code Have an engineer look at the foundation If therersquos a problem wersquoll fix itrdquo

The customer pleased by the builderrsquos cooperative attitude immediately hired a structural engineer to examine the foundation of the house On June 30 the engineer provided the customer with a written report on the condition of the foundation which stated that the foundation was properly constructed

Unhappy with the conclusions in the engineerrsquos report the customer then hired a home inspector to evaluate the house The home inspectorrsquos report concluded that the foundation of the house had been poorly constructed and was inadequately waterproofed

On July 10 the customer sent the builder the home inspectorrsquos report with a note that said ldquoUntil you fix this problem you wonrsquot get another penny from merdquo The builder immediately contacted an attorney and directed the attorney to prepare a draft complaint against the customer for nonpayment Hoping to avoid litigation the builder sent several more requests for payment to the customer The customer ignored all these requests

On September 10 the builder filed suit in federal district court properly invoking the courtrsquos diversity jurisdiction and seeking $200000 in damages for breach of contract The customerrsquos answer denied liability on the basis of alleged defective construction of the housersquos foundation

Several months later the case is nearly ready for trial However two discovery disputes have not yet been resolved

First despite a request from the builder the customer has refused to provide a copy of the report prepared by the structural engineer who examined the foundation of the house The customer claims that the report is ldquowork productrdquo and not discoverable because the customer does not intend to ask the engineer to testify at trial The builder has asked the court to order the customer to turn over the engineerrsquos report

Second the customer has asked the court to impose sanctions for the builderrsquos failure to comply with the customerrsquos demand for copies of all emails concerning construction of the foundation of the house The builder has truthfully informed the customer that all such emails were destroyed on August 2 This destruction was pursuant to the builderrsquos standard practice of permanently deleting all project-related emails from company records 60 days after construction of a project is complete There is no relevant state records-retention law

1 Should the court order the customer to turn over the engineerrsquos report Explain

2 Should the court sanction the builder for the destruction of emails related to the case and if so what factors should the court consider in determining those sanctions Explain

6

CRIMINAL LAW AND PROCEDURE QUESTION _____

A defendant was charged under state law with felony theft (Class D) and felony residential burglary (Class C) The indictment alleged that the defendant entered his neighborsrsquo home without their consent and stole a diamond ring worth at least $2500

Defense counsel filed a pretrial motion to dismiss the charges on the ground that prosecuting the defendant for both burglary and theft would constitute double jeopardy The trial court denied the motion and the defendant was prosecuted for both crimes The only evidence of the ringrsquos value offered at the defendantrsquos jury trial was the ownerrsquos testimony that she had purchased the ring two years earlier for $3000

At trial the judge issued the following jury instruction on the burglary charge prior to deliberations

If after consideration of all the evidence presented by the prosecution and defense you find beyond a reasonable doubt that the defendant entered the dwelling without the ownersrsquo consent you may presume that the defendant entered with the intent to commit a felony therein

The jury found the defendant guilty of both offenses

At the defendantrsquos sentencing hearing an expert witness called by the prosecutor testified that the diamond ring was worth between $7000 and $8000 Over defense objection the judge concluded by a preponderance of the evidence that the value of the stolen ring exceeded $5000 The judge sentenced the defendant to four yearsrsquo incarceration on the theft conviction On the burglary conviction the defendant received a consecutive sentence of seven yearsrsquo incarceration

In this state residential burglary is defined as ldquoentry into the dwelling of another without the consent of the lawful resident with the intent to commit a felony thereinrdquo Residential burglary is a Class C felony for which the minimum sentence is five years and the maximum sentence is ten years of incarceration

In this state theft is defined as ldquotaking and carrying away the property of another with the intent to permanently deprive the owner of possessionrdquo Theft is a Class D felony if the value of the item(s) taken is between $2500 and $10000 The sentence for a Class D felony theft is determined by the value of the items taken If the value is between $2500 and $5000 the maximum sentence is three yearsrsquo incarceration If the value of the items exceeds $5000 the maximum sentence is five yearsrsquo incarceration

This state affords a criminal defendant no greater rights than those mandated by the United States Constitution

1 Did the trial court err when it denied the defendantrsquos pretrial motion to dismiss on double jeopardy grounds Explain

2 Did the trial court err in its instruction to the jury on the burglary charge Explain

3 Did the trial court err when it sentenced the defendant to an additional year of incarceration on the theft conviction based on the expertrsquos testimony Explain

7

AGENCY AND PARTNERSHIP QUESTION _____

Five years ago Adam and Ben formed a general partnership Empire Partnership (Empire) to buy and sell antique automobiles at a showroom in State A Adam contributed $800000 to Empire and Ben contributed $200000 Their written partnership agreement allocated 80 of profits losses and control to Adam and 20 to Ben No filings of any type were made in connection with the formation of Empire

Three years ago a collector purchased one of Empirersquos antique cars for $3400000 The collector was willing to pay this price because of Benrsquos false representation (repeated in the sales contract) that a famous movie star had once owned the car Without the movie-star connection the car was worth only $100000 One month later when the collector discovered the truth he sued Adam Ben and Empire for $3300000 in damages The lawsuit is still pending

Two years ago Adam and Ben admitted a new partner Diane to Empire in return for her contribution of $250000 The three agreed to allocate profits losses and control 75 to Adam 10 to Ben and 15 to Diane Before joining the partnership Diane learned of the collectorrsquos claim and stated her concern to Adam and Ben that she might become liable if the claim were reduced to a judgment

Following Dianersquos admission to Empire the three partners sought to convert Empire into a limited liability partnership (LLP) Adamrsquos lawyer proposed to file with State A a ldquostatement of qualificationrdquo making an LLP election and declaring the name of the partnership to be ldquoEmpire LLPrdquo Benrsquos lawyer stated that this would not work and that a new LLP had to be formed with the assets of the old partnership transferred to the new one In the end the conversion was done the way Adamrsquos lawyer suggested with the approval of all three partners

One year ago a driver purchased a vintage car from Empire LLP based on the representation that the car was ldquofully roadworthy and capable of touring at 70 mph all dayrdquo The driver took the car on the highway at 50 mph whereupon the front suspension collapsed resulting in a crash in which the car was destroyed and the driver killed The driverrsquos estate sued Adam Ben Diane and Empire LLP for $10000000 The lawsuit is still pending

Although profitable Empire LLP does not have resources sufficient to pay the collectorrsquos claim or the claim of the driverrsquos estate

Assume that the Uniform Partnership Act (1997) applies

1 Before the filing of the statement of qualification (a) was Adam personally liable on the collectorrsquos claim Explain (b) was Diane personally liable on the collectorrsquos claim Explain

2 After the filing of the statement of qualification was Adam Ben or Diane personally liable as a partner on (a) the collectorrsquos claim or (b) the driverrsquos estatersquos claim Explain

8

February 2014 MEE

ANALYSES Constitutional Law

Trusts and Future Interests Secured Transactions

Federal Civil Procedure Criminal Law and Procedure

Agency and Partnership

CONSTITUTIONAL LAW ANALYSIS (Constitutional Law IVD)

ANALYSIS

Legal Problems

(1) Is the city ordinance requirement that businesses install floodlights a taking

(2) Is conditioning the approval of a building permit on the grant of an easement to install surveillance equipment a taking of property

DISCUSSION

Summary

The ordinance requiring businesses to install floodlights is not a per se taking under Loretto because it does not force a private landowner to allow a third party to enter and place a physical object on the land Here the city ordinance requires the businessmdashnot a third partymdashto install the floodlights

The ordinance is likely not a regulatory taking under the Penn Central balancing test While the ordinance will impose a cost on business owners that cost may be offset by the expected increase in business due to the ordinance and the ordinance does not appear to interfere with the ownerrsquos primary use of the property as a restaurant

The permit condition however is likely an uncompensated taking of property While the condition has an essential nexus with the cityrsquos legitimate interest in promoting public safety the city has not made an individualized determination that the easement condition is roughly proportional to the possibility of increased crime due to the restaurantrsquos proposed addition Thus the permit condition likely violates the Fifth Amendment as applied to the states through the Fourteenth Amendment

Point One (50) The ordinance requiring that businesses install floodlights is not a per se taking under Loretto It is not a regulatory taking under the Penn Central balancing test because the cost of compliance with the ordinance may be offset by an expected increase in business and compliance does not interfere with the businessrsquos primary use of its property as a restaurant

The city ordinance requiring a business to install floodlights does not effect a per se taking of the sort described in Loretto v Teleprompter Manhattan CATV Corp 458 US 419 (1982) because no property is physically taken by the government and the ordinance does not involve a physical invasion of private property by a third party

Even though the ordinance does not constitute an occupation of the property by either the government or a third party it is still subject to the three-factor balancing test under Penn Central Transportation Co v City of New York 438 US 104 (1978) to determine whether it is a ldquoregulatory takingrdquo Under Penn Central a court must balance (1) ldquo[t]he economic impact of the regulation on the claimantrdquo (2) ldquothe extent to which the regulation has interfered with distinct investment-backed expectationsrdquo and (3) ldquothe character of the governmental actionrdquo Id at 124 Here each factor weighs against finding that the ordinance is a taking

11

Constitutional Law Analysis

First the ordinance requirement likely has a minimal economic impact on the restaurant Compliance with the ordinance is estimated to cost $1000 and the city has found that businesses will likely recoup that cost in increased sales Also because the ordinance does not interfere with the operation of the restaurant the owner may still earn a reasonable return on its investment in the property

Second the ordinance does not interfere with the businessrsquos investment-backed expectations As in Penn Central the challenged law does not interfere with the ownerrsquos ldquoprimary expectationrdquo for use of the propertymdashin Penn Central as a railroad terminal and here as a restaurant Further the ordinance does not prevent the restaurant from expanding to meet the changing business environment

Third the character of the government action does not weigh in favor of a taking While Penn Central does say that a ldquophysical invasionrdquo is more likely to pose a taking Loretto suggests that the Courtrsquos main concern is with physical invasions by third parties Also like the landmark law challenged in Penn Central the ordinance here ldquoadjust[s] the benefits and burdens of economic life to promote the common goodrdquo Id In Penn Central the landmark law restricted development of the railroad terminal to promote the common interest in preserving historic landmarks Here the ordinance requires the businesses to install floodlights to promote the common interest in crime prevention and public safety

Because the ordinance is clearly a valid exercise of the police power it satisfies the takings clausersquos public-use requirement Kelo v City of New London 545 US 469 (2005)

In sum all three factors weigh against finding a taking under the Penn Central balancing test

Point Two (50) The permit condition may be unconstitutional as an uncompensated taking of property because the city has not made an individualized determination that the easement condition is roughly proportional to the impact of the restaurantrsquos proposed addition

In Dolan v City of Tigard 512 US 374 (1994) the Supreme Court set forth the test for determining whether an exaction imposed by a government in exchange for a discretionary benefit conferred by the government such as a condition on the approval of a building permit in this case constitutes an uncompensated taking under the Fifth Amendment The exaction is not a taking if (1) there is an ldquoessential nexusrdquo between the ldquopublic need or burdenrdquo to which the proposed development contributes and ldquothe permit condition exacted by the cityrdquo id at 386 and (2) the government makes ldquosome sort of individualized determination that the required dedication is [roughly proportional] both in nature and extent to the impact of the proposed developmentrdquo Id at 391 see also Nollan v California Coastal Commission 483 US 825 (1987)

Here the city likely can meet the nexus requirement In Dolan the landowner sought to double the size of its business which would have increased traffic on nearby roadways In exchange for approving the development the city sought an easement for a bike and pedestrian path The Court found the required nexus between the easement and the cityrsquos ldquoattempt to reduce traffic congestion by providing for alternative means of transportationrdquo 512 US at 387 Here a similar nexus likely exists between the requested easement and the cityrsquos interest in crime prevention and public safety Increased patronage and economic activity at the restaurant might attract additional crime to the area and the requested easement to install surveillance equipment would attempt to address that increased crime

12

Constitutional Law Analysis

The exaction here however may fail the second prong of the Dolan testmdashthat the exaction be roughly proportional to the anticipated impact of the requested development As noted the city in Dolan claimed that a bike and pedestrian path was needed to offset the increase in traffic due to the proposed doubling of the business The Court explained that the government must demonstrate that the additional traffic reasonably was related to the requested exaction and that the government must ldquomake some effort to quantify its findings in support of the dedication for the pedestrianbicycle pathway beyond the conclusory statement that it could offset some of the traffic demand generatedrdquo Id at 395 Here the city did not carry its burden The city simply speculates that increased patronage of the restaurant ldquomightrdquo increase crime and that the surveillance equipment ldquomightrdquo alleviate this increased crime Because the city has not made ldquosome effort to quantify its findingsrdquo in support of the easement it has not shown that the burden of the easement is roughly proportional to the benefits thought to flow from it

Thus the exaction appears to be an uncompensated taking of property in violation of the Fifth Amendment as applied to the states through the Fourteenth Amendment

13

TRUSTS AND FUTURE INTERESTS ANALYSIS ____ (Trusts and Future Interests IE3 I5 IIIA amp B)

ANALYSIS

Legal Problems

(1) How should rents dividends and sales proceeds received by the trustee prior to receipt of the sonrsquos letter have been allocated between trust income and principal

(2)(a) Did the remainder interest in the trust accelerate and become immediately payable to the daughterrsquos minor child upon the trusteersquos receipt of the sonrsquos letter and if not how should the trustee handle the distribution of the principal in the future

(2)(b) Following the trusteersquos receipt of the sonrsquos letter how should the trustee distribute future receipts of income prior to the distribution of the principal

DISCUSSION

Summary

Prior to the trusteersquos receipt of the sonrsquos letter cash dividends and rents should have been allocated to trust income and were distributable to the son the income beneficiary of the trust sales proceeds and stock dividends should have been allocated to principal

Because the sonrsquos letter to the trustee did not result in a valid disclaimer under state law (having been made more than nine months after the testatorrsquos death) the son is not deemed to have predeceased the testator Because the son is still living the class gift to the testatorrsquos grandchildren who survive the son has not closed and is not possessory it will not become possessory until the son dies The daughterrsquos minor child being the testatorrsquos only living grandchild is not currently entitled to a distribution of trust principal Trust principal will instead be distributable upon the sonrsquos death to the testatorrsquos then-living grandchildren or if there are none to the testatorrsquos then-living heirs

As for future income the trustee should either distribute the trust income to the son and the daughter as the testatorrsquos heirs accumulate the income for future distribution to those individuals ultimately entitled to the trust principal or distribute it to those presumptively entitled to the principal upon the sonrsquos death ie the daughterrsquos minor child

Point One (45) Cash dividends and rents are allocable to income sales proceeds and stock dividends are allocable to principal Items allocable to income for the period prior to the sonrsquos attempted disclaimer were distributable to the son

Receipts earned during the administration of a trust are allocable either to income or to principal Almost all states have adopted the most recent or an earlier version of the Uniform Principal and Income Act (the Act) which specifies how such receipts should be allocated

Under the Act rents (UNIF PRIN amp INC ACT (2000) sect 405 UNIF PRIN amp INC ACT (1962) sect 3(a)(1)) and cash dividends received from a corporation (UNIF PRIN amp INC ACT (2000) sect 401(b) UNIF PRIN amp INC ACT (1962) sect 6(d)) are allocable to income and are distributable to the income beneficiary of the trust

14

Trusts and Future Interests Analysis Sales proceeds (UNIF PRIN amp INC ACT (2000) sect 404(2) UNIF PRIN amp INC ACT (1962)

sect 3(b)(1)) and dividends paid in the stock of the distributing corporation (UNIF PRIN amp INC ACT (2000) sect 401(c)(1) UNIF PRIN amp INC ACT (1962) sect 3(b)(4)) are allocable to principal and added to the principal of the trust

Here the cash dividends and office building rents should have been allocated to income and until the trustee received the sonrsquos letter should have been distributed to him as the sole income beneficiary of the trust The stock dividend and proceeds from the sale of the office building should have been allocated to principal and held by the trustee for future distribution to the ultimate remaindermen of the trust

[NOTE The 2000 Uniform Principal and Income Act has been adopted in Alabama Arkansas Colorado Connecticut the District of Columbia Hawaii Idaho Iowa Kentucky Missouri Montana Nebraska New Mexico North Dakota Oregon South Dakota Utah and West Virginia]

Point Two(a) (45) Because the son did not disclaim within nine months of the testatorrsquos death there is no valid disclaimer under state law Therefore the son is not deemed to have predeceased the testator Furthermore because of the express survivorship contingency in the will the remainder in the trust does not accelerate and become distributable until the son in fact dies When the son dies the trust principal will be distributable to the testatorrsquos then-living grandchildren or if none then to the testatorrsquos then-living heirs

When a trust remainder is given to a class the class closes (ie no new persons can join the class) when there is no outstanding income interest and at least one member of the class is then entitled to demand possession of his or her share of the remainder This principle is called the rule of convenience See generally HERBERT HOVENKAMP amp SHELDON F KURTZ PRINCIPLES OF PROPERTY LAW 199ndash200 (6th ed 2005) A class member may demand possession of his or her share of the remainder upon termination of the income interest only when the class memberrsquos interest is not otherwise subject to a condition precedent See id

When a beneficiary timely disclaims an interest in a trust that beneficiary is treated as if he had predeceased the testator Here had the son disclaimed within nine months of the testatorrsquos death as required by the state statute he would have been deemed to have predeceased the testator This would have closed the class of remaindermen and the testatorrsquos then-living grandchildren (ie the daughterrsquos child) would have been entitled to the trust principal However under the state statute the sonrsquos disclaimer was not timely because he did not disclaim within nine months of the testatorrsquos death Thus because the statute is inapplicable and the son is still alive the class of grandchildren entitled to share in trust principal did not close

Because here the statute is inapplicable due to the sonrsquos failure to comply with the statutory time requirements then presumably the common-law rule allowing disclaimers (aka renunciations) at any time should apply Under the common law if a life estate is renounced the remainder interest accelerates and becomes immediately distributable to the remaindermen of the trust if the remainder is vested but not if the remainder is contingent JESSE DUKEMINIER amp ROBERT H SITKOFF WILLS TRUSTS AND ESTATES 844ndash845 (9th ed 2013) Here because the remainder is contingent upon there being grandchildren who survive the son the remainder will not accelerate It will remain open until the son dies leaving open the possibility that additional grandchildren will be included in the class or the daughterrsquos child could fall out of the class because that child fails to survive the son

And if none of the testatorrsquos grandchildren survive the son the trust principal will be distributed to the testatorrsquos heirs living at the sonrsquos death

15

Trusts and Future Interests Analysis

Point Two(b) (10) Until the trust terminates the trustee must continue to hold the trust assets The distribution of income in the meantime is unclear There are at least three possibilities Income earned on the undistributed assets could be distributed to the son and daughter as the testatorrsquos heirs accumulated and added to principal for distribution to the ultimate remaindermen or distributed from time to time to those persons who are presumptively remaindermen

When trust principal is not immediately distributable the trustee must continue to hold trust assets until the ultimate remaindermen are ascertained During this period trust income will be distributed or retained according to any instructions contained in the trust instrument See WILLIAM M MCGOVERN JR SHELDON F KURTZ amp DAVID M ENGLISH WILLS TRUSTS amp ESTATES sect 102 (4th ed 2010)

Here the testator did not specify what the trustee should do with trust income in the event the sonrsquos disclaimer did not comply with the state statute There are at least three approaches One approach would have the trustee distribute the trust income to the testatorrsquos heirs on the theory that the income represents property that was not disposed of by the testatorrsquos will and which thus passes by partial intestacy to the testatorrsquos heirs A second approach would have the trustee accumulate trust income for distribution to the ultimate remaindermen Under this approach only those individuals ultimately entitled to the principal would share in the income A third approach would have the trustee distribute trust income to those individuals who would be the remaindermen if the trust were to terminate when the income is received by the trustee under this approach trust income would be distributed to the daughterrsquos minor child until another presumptive remainderman is born This approach could result in individuals not ultimately entitled to principal say because they do not survive the son receiving income It could also result in a disproportionate distribution of income among the individuals ultimately entitled to income

[NOTE Examinees should demonstrate a recognition and understanding of the income-allocation problem and the alternatives available to address that issue There is no widely accepted solution to the problem Examinees who cite any of these possible problem-solving approaches may receive credit]

16

SECURED TRANSACTIONS ANALYSIS (Secured Transactions IB IID E amp F IIIB IVA B amp F)

ANALYSIS

Legal Problems

(1)(a) What is the nature of the bankrsquos claim to the businessrsquos equipment

(1)(b) What is the nature of the finance companyrsquos claim to the businessrsquos equipment

(1)(c) As between the bank and the finance company whose claim to the businessrsquos equipment has priority

(2) Do the claims of the bank and the finance company continue in the item of equipment sold by the business to the competitor

DISCUSSION

Summary

The bank and the finance company both have perfected security interests in the businessrsquos equipment Even though the finance companyrsquos perfected security interest was created first the bankrsquos perfected security interest has priority because the bankrsquos financing statement was filed before the finance companyrsquos financing statement The security interests of the bank and the finance company continue in the item of equipment sold by the business to the competitor because their security interests were perfected and the competitor was not a buyer in ordinary course of business

Point One(a) (25) The bank has a perfected security interest in the businessrsquos equipment

The bank has met all criteria necessary for it to have an attached and enforceable security interest in the businessrsquos equipment First value must be given UCC sect 9-203(b)(1) This criterion is fulfilled by the loan made by the bank to the business Second the debtor must have rights in the collateral UCC sect 9-203(b)(2) Clearly the business has rights in its equipment Third either the secured party must take possession of the collateral or the debtor must authenticate a security agreement containing a description of the collateral UCC sect 9-203(b)(3) The agreement that the business owner signed is a ldquosecurity agreementrdquo because it is an agreement that creates or provides for a security interest UCC sect 9-102(a)(74) By signing the security agreement the business owner authenticated it UCC sect 9-102(a)(7) Therefore all three criteria are fulfilled and the bank has an enforceable and attached security interest

A security interest is perfected when it has attached and when any additional steps required for perfection have occurred UCC sect 9-308(a) Generally speaking the additional steps will either be possession of the collateral by the secured party or the filing of a financing statement with respect to the collateral See UCC sectsect 9-310 9-313 In this case the bank filed a financing statement naming the debtor and sufficiently indicating the collateral The collateral indication is sufficient because it identifies the collateral by type of property See UCC sectsect 9-504 9-108 The fact that the financing statement was filed before the security interest was created is

17

Secured Transactions Analysis

not a problem Even though the security agreement had not yet been signed the business had authorized the filing of the financing statement in an authenticated record UCC sect 9-509(a)(1) Moreover the financing statement may be filed before the security agreement is created UCC sect 9-502(d)

Point One(b) (10) The finance company also has a perfected security interest in the businessrsquos equipment

The finance companyrsquos security interest is enforceable and attached for the same reasons as the bankrsquos security interest The loan from the finance company to the business constitutes value the business has rights in the collateral and the business owner has authenticated a security agreement containing a description of the collateral The finance companyrsquos security interest is perfected because the finance company filed a financing statement with respect to it that provides that the business is the debtor and indicates that the collateral is equipment

Point One(c) (30) The bankrsquos security interest has priority over the finance companyrsquos security interest because the bankrsquos financing statement was filed first

As between two perfected security interests the general rule is that the security interest that was the earlier to be either perfected or the subject of a filed financing statement has priority UCC sect 9-322(a)(1) While the finance companyrsquos security interest was perfected before the bankrsquos (March 15 vs March 22) the bankrsquos financing statement was filed even earlier on March 2 Thus under the first-to-file-or-perfect rule of UCC sect 9-322(a)(1) the bankrsquos security interest has priority No exceptions to the general rule apply here

Point Two (35) A security interest in collateral continues notwithstanding its sale unless an exception applies Because the security interests of the bank and the finance company were perfected and the competitor was not a buyer in ordinary course of business no exception applies and the security interests of both creditors continue in the equipment sold to the competitor

As a general rule a security interest in collateral continues notwithstanding the fact that the debtor has sold the collateral to another person UCC sect 9-315(a)(1) Thus unless an exception applies the security interests of the bank and the finance company will continue in the item of equipment sold to the competitor

A buyer of goods will take free of an unperfected security interest in those goods See UCC sect 9-317(a)(2) However when the competitor bought the businessrsquos equipment both the bank and the finance company had perfected security interests in the equipment

A buyer can take free even of a perfected security interest in goods if the buyer is a ldquobuyer in ordinary course of businessrdquo See UCC sect 9-320(a) However the competitor was not a buyer in ordinary course of business To be a ldquobuyer in ordinary course of businessrdquo a buyer must buy goods from a seller that is in the business of selling goods of that kind See UCC sect 1-201(b)(9) The competitor bought this equipment from a seller that is not in the business of selling goods of this kind so the competitor was not a buyer in ordinary course of business with respect to these goods

Because no exception applies the security interests of the bank and the finance company continue even after the item of equipment was sold to the competitor

18

FEDERAL CIVIL PROCEDURE ANALYSIS (Federal Civil Procedure IVD)

ANALYSIS

Legal Problems

(1) Is a document prepared in the course of a contract dispute protected from discovery as ldquowork productrdquo when there is no evidence that the document was prepared in anticipation of litigation

(2)(a) Is a partyrsquos failure to provide relevant electronically stored information excused when the information was destroyed pursuant to a routine document retention scheme at a time when litigation was contemplated by the destroying party

(2)(b) What sanctions should be imposed on a party for allowing the destruction of evidence that is relevant to potential future litigation

DISCUSSION

Summary

The report prepared by the structural engineer is probably not work product and is thus discoverable The engineer examined the foundation of the house at the customerrsquos request and the engineerrsquos findings are potentially relevant to the customerrsquos claim that the foundation is defective The report was not prepared in anticipation of litigation The customer appears to have sought the engineerrsquos opinion in response to the builderrsquos offer to fix any problems with the foundation that an engineer might identify Because the report was not prepared in anticipation of litigation it is not protected by the work-product doctrine

The builder should have taken appropriate steps to preserve evidence including suspending its document retention program as soon as it began planning for litigationmdashie on July 10 Its destruction of potentially relevant material after that date was wrongful However a court is unlikely to impose severe sanctions on the builder because there are no facts indicating that the builder acted in bad faith and the customer can prove that the foundation is defective without the destroyed emails

Point One (40) The customer must turn over the engineerrsquos report because it was not prepared in anticipation of litigation

In general a party to a lawsuit in federal court ldquomay obtain discovery regarding any nonprivileged matter that is relevant to any partyrsquos claim or defenserdquo FED R CIV P 26(b)(1) (2009) This includes the right to inspect and copy documents in the other partyrsquos possession FED R CIV P 34(a)(1) Here the customer hired a structural engineer to examine the foundation of the house The engineerrsquos report on the foundation is likely to include information that would be relevant to the customerrsquos claim that the foundation was defectively constructed

The so-called ldquowork productrdquo rule allows a party to refuse to turn over ldquodocuments that are prepared in anticipation of litigation or for trialrdquo by that partyrsquos representative including

19

Federal Civil Procedure Analysis

a consultant Thus if the customer had hired the structural engineer to prepare a report ldquoin anticipation of litigationrdquo that report might not be discoverable See FED R CIV P 26(b)(3)

In this case however the customer hired the engineer to evaluate the foundation of the house as part of the customerrsquos negotiation with the builder concerning the housersquos flooding problem The builder told the customer that the housersquos landscaping was the reason for the flooding and the builder told the customer ldquoHave an engineer look at the foundation If therersquos a problem wersquoll fix itrdquo The customer appears to have acted in response to that statement There is no indication that the customer anticipated any kind of legal action at the time that the structural engineer was hired Accordingly the structural engineerrsquos report is discoverable and the court should order the customer to turn it over

[NOTE If an examinee concludes that the structural engineerrsquos report was prepared in anticipation of litigation then the examinee should also conclude that the report is not discoverable Documents prepared in anticipation of litigation do not need to be disclosed to an adverse party unless that party can demonstrate a ldquosubstantial needrdquo for the documents and an inability to obtain substantially equivalent information without ldquoundue hardshiprdquo FED R CIV P 26(b)(3)(A)(ii) Furthermore a report prepared by an expert who is not expected to testify is not discoverable in the absence of ldquoexceptional circumstancesrdquo making it ldquoimpracticablerdquo to obtain the information in another way FED R CIV P 26(b)(4)(D)(ii) The builder probably cannot make these showings here unless the engineerrsquos report deals with circumstances that have since changed There is no evidence that the structural engineer would have had access to any information or facts that the builder would not already know as a result of its construction and subsequent inspection of the house In addition if necessary the builder could ask the court for permission to arrange for a further inspection of the house by a structural engineer hired by the builder See FED R CIV P 34(a)(2) Accordingly if an examinee concludes that the report was prepared in anticipation of litigation the examinee should also conclude that the builder is not entitled to see the report]

Point Two(a) (30) Because the builder anticipated that it might be involved in litigation concerning its contract with the customer the builder acted wrongfully in destroying emails that were relevant to the housersquos construction even though the emails were destroyed pursuant to a routine document retention plan

As noted above a party to a lawsuit in federal court ldquomay obtain discovery regarding any nonprivileged matter that is relevant to any partyrsquos claim or defenserdquo FED R CIV P 26(b)(1) This includes emails and other electronically stored information FED R CIV P 34(a)(1)(A) Here the customer has requested all the builderrsquos emails pertaining to work done on the foundation of the house Ordinarily the builder would be obliged to turn over this information which is relevant to the customerrsquos defense that the housersquos foundation was poorly constructed

Unfortunately the emails in question no longer exist because the builder destroyed them on August 2

In general spoliation of evidence (destruction or alteration of evidence) is improper if the party who destroyed or altered the evidence ldquohas notice that the evidence is relevant to litigation or should have known that the evidence may be relevant to future litigationrdquo Fujitsu Ltd v Federal Express Corp 247 F3d 423 436 (2d Cir 2001) It is improper for a party to destroy electronic information relevant to pending litigation even if the destruction occurs before there is any request or order seeking the information See eg Leon v IDX Sys Corp 464 F3d 951 (9th Cir 2006) (plaintiffrsquos intentional destruction of computer files warranted dismissal even

20

In this case the builderrsquos destruction of the emails was pursuant to a routine document retention plan The Federal Rules provide expressly that in the absence of ldquoexceptional circumstancesrdquo parties should not be sanctioned for the loss of electronically stored information when the loss occurs pursuant to ldquoroutine good-faith operation of an electronic information systemrdquo FED R CIV P 37(e) However when a party anticipates litigation ldquoit must suspend its routine document retentiondestruction policy and put in place a lsquolitigation holdrsquo to ensure the preservation of relevant documentsrdquo Zubulake v UBS Warburg LLC 220 FRD 212 218 (SDNY 2003)

Federal Civil Procedure Analysis

though spoliation occurred before order compelling discovery) Similarly the duty to preserve evidence applies to a party who anticipates litigation even if litigation has not yet been commenced See THE SEDONA PRINCIPLES BEST PRACTICES RECOMMENDATIONS amp PRINCIPLES FOR ADDRESSING ELECTRONIC DOCUMENT PRODUCTION 70 cmt 14a (2d ed 2007)

The builder destroyed the emails on August 2 At that time the builder knew that litigation was a possibility because the builder had already directed its attorney to prepare a draft complaint for possible filing Knowing that litigation was a possibility the builder had a duty to take steps to preserve evidence including the emails in question See generally Fujitsu Ltd

Thus the builderrsquos destruction of potentially relevant emails at a time when it knew that litigation was a possibility was improper It had a duty to preserve evidence and it breached that duty

[NOTE Because courts have used different words to describe the test for when evidence must be preserved an examineersquos precise formulation of the test is not critical]

Point Two(b) (30) In determining appropriate sanctions for spoliation courts consider both the level of culpability of the spoliating party and the degree of prejudice the loss of evidence has caused the other party Here the builderrsquos destruction of evidence does not appear to have been willful nor is it likely to pose a significant obstacle to the customerrsquos defense Any sanctions imposed by the court should be modest

Federal courts have inherent power to control the litigation process and can sanction misbehavior including spoliation even when there has been no specific violation of the Federal Rules of Civil Procedure See generally Chambers v NASCO Inc 501 US 32 (1991) (discussing courtrsquos inherent power to control the litigation process) The range of available sanctions is broad It can include such sanctions as the payment of expenses incurred by the other party as a result of the destruction of the evidence an instruction to the jury authorizing it to draw an adverse inference from the destruction of the evidence a shifting of the burden of proof on the relevant issue or even judgment against the responsible party See eg Residential Funding Corp v DeGeorge Financial Corp 306 F3d 99 108 (2d Cir 2002) (adverse inference) Silvestri v General Motors Corp 271 F3d 583 593 (4th Cir 2001) (possibility of dismissal) Cf FED R CIV P 37(b)(2)(A) (listing remedies for failure to comply with discovery obligations)

In determining appropriate sanctions for spoliation courts consider both the level of culpability of the spoliating party and the degree of prejudice the loss of evidence has caused the other party Many courts impose severe sanctions (such as an adverse-inference instruction or the entry of judgment against the spoliating party) only when there is evidence of bad faith in the form of an intentional effort to hide information Eg Greyhound Lines Inc v Wade 485 F3d 1032 1035 (8th Cir 2007) (spoliation sanction requires intentional destruction out of desire ldquoto suppress the truthrdquo) However other courts have said that negligence in preserving evidence can

21

Federal Civil Procedure Analysis

support an adverse-inference instruction See Residential Funding 306 F3d at 108 (negligence enough under some circumstances)

Although a court might well order an evidentiary hearing on the issue of sanctions the facts presented do not seem appropriate for severe sanctions First the evidence was destroyed pursuant to the builderrsquos standard document retention plan and there is no evidence that the builder deliberately failed to suspend its usual procedures with the purpose of allowing the destruction of evidence Second the loss of this evidence will not severely hinder the customerrsquos presentation of his case The central issue is whether the foundation of the house was properly constructed If the construction job was poorly done the customer can present evidence derived from inspection of the premises to prove that point The customer can also depose witnesses about any issues that arose during construction

Under the circumstances a court is not likely to impose particularly severe sanctions although it might shift the burden to the builder to show that the foundation was properly constructed or it might require the builder to reimburse any expenses the customer incurs to discover and prove the facts about issues or disputes that arose during construction of the foundation

[NOTE The result reached by the examinee is less important than the examineersquos recognition that (a) a range of sanctions is available to the court and (b) the appropriate sanction depends both on the culpability of the builder and the prejudice suffered by the customer]

22

CRIMINAL LAW AND PROCEDURE ANALYSIS (Criminal Law and Procedure IIA amp D VE amp F)

ANALYSIS

Legal Problems

(1) Did charging the defendant with both theft and burglary constitute double jeopardy

(2) Did the jury instruction violate the due process clause either by relieving the prosecution of the burden of proving the element of intent or by shifting the burden to the defendant to disprove that element

(3) Did the sentence imposed in this case for the theft conviction unconstitutionally deprive the defendant of his right to a jury trial on the issue of the value of the stolen item

DISCUSSION

Summary

The trial court properly denied the defendantrsquos pretrial motion to dismiss the charges on double jeopardy grounds The defendant may be charged with and convicted of both theft and burglary Each of the charges has an element that the other does not Neither charge is a lesser-included offense nor are they multiplicitous Thus charging both theft and burglary does not violate double jeopardy

The jury instruction on the burglary charge was constitutionally flawed It could have been reasonably understood by the jury as either (1) an irrebuttable conclusive presumption (which relieved the prosecution of proving the element of intent and removed the issue from the jury) or (2) a rebuttable mandatory presumption (which unconstitutionally shifted the burden of proof on an element of a charged offense from the prosecution to the defendant)

Because the four-year sentence imposed by the judge was based on the judgersquos finding by a preponderance of the evidence that the value of the stolen ring exceeded $5000 the sentence violates the defendantrsquos right to a jury determination beyond a reasonable doubt of the value of the ring

Point One (30) Charging the defendant with theft and burglary did not constitute double jeopardy

The Double Jeopardy Clause of the Fifth Amendment provides that a person shall not be twice put in jeopardy for the ldquosame offenserdquo Thus the question is whether the elements of the theft charge are wholly contained in the burglary charge or vice versa If the elements of the lesser charge (theft) are not wholly contained in the greater charge (burglary)mdashie if each charge requires proof of a fact that the other does notmdashthen convicting the defendant of both crimes would not violate double jeopardy even when the two offenses occurred at the same time and are thus arguably part of the ldquosame transactionrdquo Blockburger v United States 284 US 299 304 (1932) See also Albernaz v United States 450 US 333 344 n3 (1981) United States v Dixon 509 US 688 704 (1993)

23

Criminal Law and Procedure Analysis

Here theft and burglary each require proof of an element not required for the other crime Burglary may be defined differently in different jurisdictions However it almost invariably requires entry into a building or dwelling of another with the specific intent to commit a felony therein and the crime of burglary is complete upon the entry into the building or dwelling with such intent See eg Cannon v Oklahoma 827 P2d 1339 1342 (Okla Crim App 1992) In contrast theft which also may be defined differently in different states almost invariably requires the taking and carrying away of an item of personal property belonging to another with the intent to steal or permanently deprive the owner of possession

Here the ldquotakingrdquo or ldquostealingrdquo element is not contained in the definition of burglary and the ldquoentryrdquo element of burglary is not contained in the definition of theft Because theft is not a lesser-included offense of burglary and burglary is not a lesser-included offense of theft charging the defendant for both burglary and theft did not violate double jeopardy and the court properly denied the defense motion on those grounds Yparrea v Dorsey 64 F3d 577 579ndash80 (10th Cir 1995) citing Blockburger 284 US at 304

Finally the defendantrsquos motion to dismiss all the charges on double jeopardy grounds was improper because if both charges were for the same offense the motion should have requested dismissal of one charge not both

Point Two (35) The jury instruction on the burglary charge violated the Due Process Clause because it created either (1) an irrebuttable conclusive presumption (which relieved the prosecution of proving the element of intent and removed that issue from the jury) or (2) a rebuttable mandatory presumption (which unconstitutionally shifted the burden of proof on an element of a charged offense to the defendant)

The Supreme Court has interpreted the Due Process Clause of the US Constitution to require that the prosecution prove all elements of an offense beyond a reasonable doubt See In re Winship 397 US 358 364 (1970) The burden of proof cannot be shifted to the defendant by presuming an essential element upon proof of other elements of the offense because shifting the burden of persuasion with respect to any element of a criminal offense is contrary to the Due Process Clause See Mullaney v Wilbur 421 US 684 (1975)

The crime of burglary includes entry into a building or dwelling with the specific intent to commit a felony therein The requirement that the prosecutor prove beyond a reasonable doubt that the defendant had this specific intent distinguishes burglary from general-intent crimes like trespass See Sandstrom v Montana 442 US 510 523 (1979)

Here the jury was instructed that if ldquoafter consideration of all the evidence presented by the prosecution and defense you find beyond a reasonable doubt that the defendant entered the dwelling without the ownersrsquo consent you may presume that the defendant entered with the intent to commit a felony thereinrdquo This instruction was unconstitutional because it created either an irrebuttable conclusive presumption or a rebuttable mandatory presumption

A conclusive presumption is ldquoan irrebuttable direction by the court to find intent once convinced of the facts triggering the presumptionrdquo Id at 517 Here the jurors were instructed that once the prosecutor established that the defendant entered the neighborsrsquo house without consent they ldquomay presumerdquo that he intended to commit a felony therein The jurors may have reasonably concluded from this instruction that if they found that the defendant intended to enter his neighborsrsquo home without permission they must further find that he entered with the specific intent to commit a felony therein Because this instruction could operate as a conclusive

24

Criminal Law and Procedure Analysis

irrebuttable presumption by eliminating intent ldquoas an ingredient of the offenserdquo it violated due process by relieving the prosecution of the burden of proof for this element Id at 522

In the alternative the jury instruction could have been reasonably understood to create a rebuttable mandatory presumption which ldquotells [the jury] they must find the elemental fact upon proof of the basic fact at least unless the defendant has come forward with some evidence to rebut the presumed connection between the two factsrdquo County Court of Ulster County New York v Allen 442 US 140 157 (1979) The due process problem created by rebuttable mandatory presumptions is that ldquo[t]o the extent that the trier of fact is forced to abide by the presumption and may not reject it based on an independent evaluation of the particular facts presented by the State the analysis of the presumptionrsquos constitutional validity is logically divorced from those facts and based on the presumptionrsquos accuracy in the run of casesrdquo Id at 159

Unlike irrebuttable conclusive presumptions rebuttable mandatory presumptions are not always per se violations of the Due Process Clause However the Supreme Court of the United States has held that jury instructions that could reasonably be understood as shifting the burden of proof to the defendant on an element of the offense are unconstitutional Francis v Franklin 471 US 307 (1985) Here the argument that the jury instruction operated as a rebuttable mandatory presumption is supported by the fact that the judge also instructed the jury to ldquoconsider[ ] all the evidence presented by the prosecution and defenserdquo However even if the instruction created a rebuttable mandatory presumption it would be unconstitutional because it shifted the burden to the defense on an element of the offense Sandstrom 442 US at 524 Mullaney 421 US at 686

[NOTE Whether an examinee identifies the jury instruction as containing a ldquoconclusiverdquo or ldquomandatoryrdquo presumption is less important than the examineersquos analysis of the constitutional infirmities]

Point Three (35) The trial court violated the defendantrsquos Sixth Amendment right to a jury trial on an essential element of the offense when it found by a preponderance of the evidence that the ring was worth over $5000 and increased the defendantrsquos sentence based on this finding

In the statutory scheme under which the defendant was tried and convicted a Class D felony theft is defined as theft of item(s) with a value between $2500 and $10000 The jury found that the value of the diamond ring was at least $2500 and convicted the defendant of felony theft However at sentencing the trial court made a separate finding by a preponderance of the evidence that the value of the ring was greater than $5000 Following the statutersquos two-tiered sentencing scheme the judge then imposed on the defendant a sentence that was one year longer than the maximum that would otherwise have been allowed

The judgersquos sentence was unconstitutional because it violated the defendantrsquos Sixth Amendment right to a jury trial on this question The Supreme Court held in Apprendi v New Jersey 530 US 466 (2000) that ldquo[o]ther than the fact of a prior conviction any fact that increases the penalty for a crime beyond the prescribed statutory maximum must be submitted to a jury and proved beyond a reasonable doubtrdquo because ldquo[i]t is unconstitutional for a legislature to remove from the jury the assessment of facts that increase the prescribed range of penalties to which a criminal defendant is exposed [because] such facts must be established by proof beyond a reasonable doubtrdquo Id The Court reaffirmed Apprendi in Blakely v Washington 542 US 296 (2004) holding that the ldquolsquostatutory maximumrsquo for Apprendi purposes is the maximum sentence a judge may impose solely on the basis of the facts reflected in the jury verdict or admitted by the defendantrdquo Id at 303 (emphasis in original) In United States v Booker 543 US 220 (2005)

25

Criminal Law and Procedure Analysis

the Court relied on Blakely and Apprendi to conclude that protecting a defendantrsquos Sixth Amendment right to a jury trial required that ldquo[a]ny fact which is necessary to support a sentence exceeding the maximum authorized by the facts established by a plea of guilty or a jury verdict must be admitted by the defendant or proved to a jury beyond a reasonable doubtrdquo Id at 244

Thus in order to constitutionally increase a sentence above the statutory maximum of three years the jury must have found beyond a reasonable doubt that the value of the ring exceeded $5000 Here the court made the finding based on an appraisal proffered by the prosecutor only at sentencing and the judgersquos finding was by a preponderance of the evidence rather than beyond a reasonable doubt

26

AGENCY AND PARTNERSHIP ANALYSIS __________ (Agency and Partnership VA amp C VI)

ANALYSIS

Legal Problems

(1) Is a partner in a general partnership personally liable on a claim arising from misrepresentations by another partner made in the course of the partnership business

(2) Does a newly admitted partner in a general partnership become personally liable on existing claims against the partnership

(3) After the filing by a general partnership of a statement of qualification as a limited liability partnership are the partners personally liable as partners on (a) an existing claim against the general partnership and (b) a claim against the partnership that arose after the filing

DISCUSSION

Summary

Adam and Ben formed a general partnership under which they were jointly and severally liable for obligations of the partnership Thus Adam was personally liable for misrepresentations by Ben made in the ordinary course of the partnership business

Upon joining the general partnership Diane became personally liable for the obligations of the partnership arising after her admission but not for obligations pre-existing her admission such as the collectorrsquos claim

By filing a statement of qualification the three partners properly elected limited liability partnership status As partners in an LLP none of the three partners is personally liable as a partner for partnership obligations arising after the election such as the claim by the driverrsquos estate The election however does not change their personal liability on pre-existing claims that arose before the election such as the collectorrsquos claim

Point One (30) As a general partner of Empire a general partnership Adam became personally liable on the collectorrsquos claim a valid claim against the partnership that arose because of Benrsquos wrongful act in the ordinary course of the partnership business

When the collectorrsquos claim arose Empire was a general partnership composed of Adam and Ben Under UPA (1997) sect 306(a) partners of a general partnership are liable jointly and severally for all obligations of the partnership Under UPA (1997) sect 305(a) the partnership could become obligated for the loss caused to the collector as a result of the misrepresentation by Ben provided he was acting in the ordinary course of the partnership business Because there was no statement that limited his partnership authority Ben as partner was ldquoan agent of the partnership for the purpose of its businessrdquo See UPA (1997) sect 301(1) Benrsquos misrepresentation to the collector even if intentional appears to be in the ordinary course of the partnershiprsquos business of dealing

27

Agency and Partnership Analysis

in antique cars Thus Benrsquos wrongful act created a partnership obligation for which Adam was jointly and severally liable

[NOTE Generally a partnership creditor must ldquoexhaust the partnershiprsquos assets before levying on a judgment debtor partnerrsquos individual property where the partner is personally liable for the partnership obligationrdquo as a result of his status as a partner UPA (1997) sect 307 cmt 4 As the UPA comments explain this places Adam more in the position of guarantor than principal debtor on the partnership obligation Id cmt 4 Although an examinee might discuss this point the call focuses on whether Adam is personally liable not how the liability might be enforced]

Point Two (30) Because the collectorrsquos claim arose before Diane joined Empire Diane did not become personally liable on the claim

Diane was admitted to Empire when it was a general partnership and after the collectorrsquos claim arose While the general rule under UPA (1997) sect 306(a) is that the partners of a general partnership are liable jointly and severally for all obligations of the partnership there is a special rule for partners who are admitted during the duration of the partnership Under UPA (1997) sect 306(b) a person admitted to an existing partnership is not personally liable for any partnership obligations incurred before the personrsquos admission Because Diane was admitted to Empire after the collectorrsquos claim arose Diane is not personally liable on the claim

Dianersquos knowledge of the pre-existing claim and her stated concern about becoming liable on the collectorrsquos claim do not change her personal nonliability to the collector Although partners who have a liability shield can assume liability to third parties through private contractual guarantees or modifications to the partnership agreement Dianersquos stated concern constituted neither a guaranty to the collector nor ldquoan intentional waiver of liability protectionsrdquo See UPA (1997) sect 306 cmt 3 (describing methods for waiver of liability protections under sect 306(c) applicable in limited liability partnerships)

At most Diane will lose her investment in the partnership as a result of the collectorrsquos claim Although Diane did not become personally liable on the collectorrsquos claim when she joined the partnership the $250000 she contributed to the partnership is ldquoat risk for the satisfaction of existing partnership debtsrdquo UPA (1997) sect 306 cmt 2

Point Three (40) Filing the statement of qualification was effective to elect limited liability partnership status Despite this new status Adam and Ben remain personally liable on the collectorrsquos claim which arose before the election But as partners in an LLP neither Adam Ben nor Diane is personally liable as a partner on the driverrsquos estatersquos claim which arose after the election

Under UPA (1997) sect 1001 a general partnership can make an election and become a limited liability partnershipmdashif the partners approve the conversion by a vote equivalent to that necessary to amend the partnership agreement and the partnership then files a statement of qualification that specifies the name of the partnership its principal office and its election to be an LLP Here the partners agreed unanimouslymdashsufficient to amend their agreement under UPA (1997) sect 401(j)mdashand the statement of qualification was filed In addition the name of Empire LLP properly included an appropriate ending ldquoLLPrdquo See UPA (1997) sect 1002

Although another way to effectuate a ldquoconversionrdquo (as suggested by Benrsquos lawyer) is to form a new LLP and transfer the assets of the old general partnership to the new LLP the

28

Agency and Partnership Analysis

method used here (approval by the partners and the filing of a statement of qualification) is also sufficient to create LLP status

Thus Empire became Empire LLP as of the date of filing of the statement of qualification See UPA (1997) sect 1001 What effect did this have on the collectorrsquos claim which predated the filing According to UPA (1997) sect 306(c) an obligation incurred while a partnership is an LLP is solely a partnership obligation As the collectorrsquos claim predated the LLP Adam and Ben remain personally liable on the collectorrsquos claim Diane on the other hand was not personally liable on the collectorrsquos claim either before or after the filing of the statement of qualification See Point Two above

The driverrsquos estatersquos claim arose after Empire became Empire LLP Under UPA (1997) sect 306(c) an obligation incurred while a partnership is an LLP is solely a partnership obligationThus Adam Ben and Diane as partners are all protected from personal liability on the driverrsquos estatersquos claim But there may be personal liability if any of them was negligent or otherwise acted wrongfully by not informing the buyer of the bad suspension that caused the accident

29

National Conference of Bar Examiners 302 South Bedford Street | Madison WI 53703-3622 Phone 608-280-8550 | Fax 608-280-8552 | TDD 608-661-1275

wwwncbexorg e-mail contactncbexorg

  • Preface
  • Description of the MEE
  • Instructions
  • February 2014 Questions
    • Constitutinal Law Question
    • Trusts and Future Interests Question
    • Secured Transactions Question
    • Federal Civil Procedure Question
    • Criminal Law and Procedure Question
    • Agency and Partnership Question
      • February 2014 Analyses
        • Constitutional Law Analysis
        • Trust and Future Interests Analysis
        • Secured Transactions Analysis
        • Federal Civil Procedure Analysis
        • Criminal Law and Procedure Analysis
        • Agency and Partnership Analysis
            • ltlt13 ASCII85EncodePages false13 AllowTransparency false13 AutoPositionEPSFiles true13 AutoRotatePages None13 Binding Left13 CalGrayProfile (Dot Gain 20)13 CalRGBProfile (sRGB IEC61966-21)13 CalCMYKProfile (US Web Coated 050SWOP051 v2)13 sRGBProfile (sRGB IEC61966-21)13 CannotEmbedFontPolicy Error13 CompatibilityLevel 1413 CompressObjects Tags13 CompressPages true13 ConvertImagesToIndexed true13 PassThroughJPEGImages true13 CreateJobTicket false13 DefaultRenderingIntent Default13 DetectBlends true13 DetectCurves 0000013 ColorConversionStrategy CMYK13 DoThumbnails false13 EmbedAllFonts true13 EmbedOpenType false13 ParseICCProfilesInComments true13 EmbedJobOptions true13 DSCReportingLevel 013 EmitDSCWarnings false13 EndPage -113 ImageMemory 104857613 LockDistillerParams false13 MaxSubsetPct 10013 Optimize true13 OPM 113 ParseDSCComments true13 ParseDSCCommentsForDocInfo true13 PreserveCopyPage true13 PreserveDICMYKValues true13 PreserveEPSInfo true13 PreserveFlatness true13 PreserveHalftoneInfo false13 PreserveOPIComments true13 PreserveOverprintSettings true13 StartPage 113 SubsetFonts true13 TransferFunctionInfo Apply13 UCRandBGInfo Preserve13 UsePrologue false13 ColorSettingsFile ()13 AlwaysEmbed [ true13 ]13 NeverEmbed [ true13 ]13 AntiAliasColorImages false13 CropColorImages true13 ColorImageMinResolution 30013 ColorImageMinResolutionPolicy OK13 DownsampleColorImages true13 ColorImageDownsampleType Bicubic13 ColorImageResolution 30013 ColorImageDepth -113 ColorImageMinDownsampleDepth 113 ColorImageDownsampleThreshold 15000013 EncodeColorImages true13 ColorImageFilter DCTEncode13 AutoFilterColorImages true13 ColorImageAutoFilterStrategy JPEG13 ColorACSImageDict ltlt13 QFactor 01513 HSamples [1 1 1 1] VSamples [1 1 1 1]13 gtgt13 ColorImageDict ltlt13 QFactor 01513 HSamples [1 1 1 1] VSamples [1 1 1 1]13 gtgt13 JPEG2000ColorACSImageDict ltlt13 TileWidth 25613 TileHeight 25613 Quality 3013 gtgt13 JPEG2000ColorImageDict ltlt13 TileWidth 25613 TileHeight 25613 Quality 3013 gtgt13 AntiAliasGrayImages false13 CropGrayImages true13 GrayImageMinResolution 30013 GrayImageMinResolutionPolicy OK13 DownsampleGrayImages true13 GrayImageDownsampleType Bicubic13 GrayImageResolution 30013 GrayImageDepth -113 GrayImageMinDownsampleDepth 213 GrayImageDownsampleThreshold 15000013 EncodeGrayImages true13 GrayImageFilter DCTEncode13 AutoFilterGrayImages true13 GrayImageAutoFilterStrategy JPEG13 GrayACSImageDict ltlt13 QFactor 01513 HSamples [1 1 1 1] VSamples [1 1 1 1]13 gtgt13 GrayImageDict ltlt13 QFactor 01513 HSamples [1 1 1 1] VSamples [1 1 1 1]13 gtgt13 JPEG2000GrayACSImageDict ltlt13 TileWidth 25613 TileHeight 25613 Quality 3013 gtgt13 JPEG2000GrayImageDict ltlt13 TileWidth 25613 TileHeight 25613 Quality 3013 gtgt13 AntiAliasMonoImages false13 CropMonoImages true13 MonoImageMinResolution 120013 MonoImageMinResolutionPolicy OK13 DownsampleMonoImages true13 MonoImageDownsampleType Bicubic13 MonoImageResolution 120013 MonoImageDepth -113 MonoImageDownsampleThreshold 15000013 EncodeMonoImages true13 MonoImageFilter CCITTFaxEncode13 MonoImageDict ltlt13 K -113 gtgt13 AllowPSXObjects false13 CheckCompliance [13 None13 ]13 PDFX1aCheck false13 PDFX3Check false13 PDFXCompliantPDFOnly false13 PDFXNoTrimBoxError true13 PDFXTrimBoxToMediaBoxOffset [13 00000013 00000013 00000013 00000013 ]13 PDFXSetBleedBoxToMediaBox true13 PDFXBleedBoxToTrimBoxOffset [13 00000013 00000013 00000013 00000013 ]13 PDFXOutputIntentProfile ()13 PDFXOutputConditionIdentifier ()13 PDFXOutputCondition ()13 PDFXRegistryName ()13 PDFXTrapped False1313 CreateJDFFile false13 Description ltlt13 ARA 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 BGR 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 CHS ltFEFF4f7f75288fd94e9b8bbe5b9a521b5efa7684002000410064006f006200650020005000440046002065876863900275284e8e9ad88d2891cf76845370524d53705237300260a853ef4ee54f7f75280020004100630072006f0062006100740020548c002000410064006f00620065002000520065006100640065007200200035002e003000204ee553ca66f49ad87248672c676562535f00521b5efa768400200050004400460020658768633002gt13 CHT ltFEFF4f7f752890194e9b8a2d7f6e5efa7acb7684002000410064006f006200650020005000440046002065874ef69069752865bc9ad854c18cea76845370524d5370523786557406300260a853ef4ee54f7f75280020004100630072006f0062006100740020548c002000410064006f00620065002000520065006100640065007200200035002e003000204ee553ca66f49ad87248672c4f86958b555f5df25efa7acb76840020005000440046002065874ef63002gt13 CZE 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 DAN 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 DEU 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 ESP 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 ETI 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 FRA 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 GRE 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 HEB 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 HRV (Za stvaranje Adobe PDF dokumenata najpogodnijih za visokokvalitetni ispis prije tiskanja koristite ove postavke Stvoreni PDF dokumenti mogu se otvoriti Acrobat i Adobe Reader 50 i kasnijim verzijama)13 HUN 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 ITA 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 JPN ltFEFF9ad854c18cea306a30d730ea30d730ec30b951fa529b7528002000410064006f0062006500200050004400460020658766f8306e4f5c6210306b4f7f75283057307e305930023053306e8a2d5b9a30674f5c62103055308c305f0020005000440046002030d530a130a430eb306f3001004100630072006f0062006100740020304a30883073002000410064006f00620065002000520065006100640065007200200035002e003000204ee5964d3067958b304f30533068304c3067304d307e305930023053306e8a2d5b9a306b306f30d530a930f330c8306e57cb30818fbc307f304c5fc59808306730593002gt13 KOR ltFEFFc7740020c124c815c7440020c0acc6a9d558c5ec0020ace0d488c9c80020c2dcd5d80020c778c1c4c5d00020ac00c7a50020c801d569d55c002000410064006f0062006500200050004400460020bb38c11cb97c0020c791c131d569b2c8b2e4002e0020c774b807ac8c0020c791c131b41c00200050004400460020bb38c11cb2940020004100630072006f0062006100740020bc0f002000410064006f00620065002000520065006100640065007200200035002e00300020c774c0c1c5d0c11c0020c5f40020c2180020c788c2b5b2c8b2e4002egt13 LTH ltFEFF004e006100750064006f006b0069007400650020016100690075006f007300200070006100720061006d006500740072007500730020006e006f0072011700640061006d00690020006b0075007200740069002000410064006f00620065002000500044004600200064006f006b0075006d0065006e007400750073002c0020006b00750072006900650020006c0061006200690061007500730069006100690020007000720069007400610069006b007900740069002000610075006b01610074006f00730020006b006f006b007900620117007300200070006100720065006e006700740069006e00690061006d00200073007000610075007300640069006e0069006d00750069002e0020002000530075006b0075007200740069002000500044004600200064006f006b0075006d0065006e007400610069002000670061006c006900200062016b007400690020006100740069006400610072006f006d00690020004100630072006f006200610074002000690072002000410064006f00620065002000520065006100640065007200200035002e0030002000610072002000760117006c00650073006e0117006d00690073002000760065007200730069006a006f006d00690073002egt13 LVI 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 NLD (Gebruik deze instellingen om Adobe PDF-documenten te maken die zijn geoptimaliseerd voor prepress-afdrukken van hoge kwaliteit De gemaakte PDF-documenten kunnen worden geopend met Acrobat en Adobe Reader 50 en hoger)13 NOR 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 POL ltFEFF0055007300740061007700690065006e0069006100200064006f002000740077006f0072007a0065006e0069006100200064006f006b0075006d0065006e007400f300770020005000440046002000700072007a0065007a006e00610063007a006f006e00790063006800200064006f002000770079006400720075006b00f30077002000770020007700790073006f006b00690065006a0020006a0061006b006f015b00630069002e002000200044006f006b0075006d0065006e0074007900200050004400460020006d006f017c006e00610020006f007400770069006500720061010700200077002000700072006f006700720061006d006900650020004100630072006f00620061007400200069002000410064006f00620065002000520065006100640065007200200035002e0030002000690020006e006f00770073007a0079006d002egt13 PTB 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 RUM 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 RUS ltFEFF04180441043f043e043b044c04370443043904420435002004340430043d043d044b04350020043d0430044104420440043e0439043a043800200434043b044f00200441043e043704340430043d0438044f00200434043e043a0443043c0435043d0442043e0432002000410064006f006200650020005000440046002c0020043c0430043a04410438043c0430043b044c043d043e0020043f043e04340445043e0434044f04490438044500200434043b044f00200432044b0441043e043a043e043a0430044704350441044204320435043d043d043e0433043e00200434043e043f0435044704300442043d043e0433043e00200432044b0432043e04340430002e002000200421043e043704340430043d043d044b04350020005000440046002d0434043e043a0443043c0435043d0442044b0020043c043e0436043d043e0020043e0442043a0440044b043204300442044c002004410020043f043e043c043e0449044c044e0020004100630072006f00620061007400200438002000410064006f00620065002000520065006100640065007200200035002e00300020043800200431043e043b043504350020043f043e04370434043d043804450020043204350440044104380439002egt13 SKY 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 SLV 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 SUO 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 SVE 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 TUR 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 UKR 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 ENU (Use these settings to create Adobe PDF documents best suited for high-quality prepress printing Created PDF documents can be opened with Acrobat and Adobe Reader 50 and later)13 gtgt13 Namespace [13 (Adobe)13 (Common)13 (10)13 ]13 OtherNamespaces [13 ltlt13 AsReaderSpreads false13 CropImagesToFrames true13 ErrorControl WarnAndContinue13 FlattenerIgnoreSpreadOverrides false13 IncludeGuidesGrids false13 IncludeNonPrinting false13 IncludeSlug false13 Namespace [13 (Adobe)13 (InDesign)13 (40)13 ]13 OmitPlacedBitmaps false13 OmitPlacedEPS false13 OmitPlacedPDF false13 SimulateOverprint Legacy13 gtgt13 ltlt13 AddBleedMarks false13 AddColorBars false13 AddCropMarks false13 AddPageInfo false13 AddRegMarks false13 ConvertColors ConvertToCMYK13 DestinationProfileName ()13 DestinationProfileSelector DocumentCMYK13 Downsample16BitImages true13 FlattenerPreset ltlt13 PresetSelector MediumResolution13 gtgt13 FormElements false13 GenerateStructure false13 IncludeBookmarks false13 IncludeHyperlinks false13 IncludeInteractive false13 IncludeLayers false13 IncludeProfiles false13 MultimediaHandling UseObjectSettings13 Namespace [13 (Adobe)13 (CreativeSuite)13 (20)13 ]13 PDFXOutputIntentProfileSelector DocumentCMYK13 PreserveEditing true13 UntaggedCMYKHandling LeaveUntagged13 UntaggedRGBHandling UseDocumentProfile13 UseDocumentBleed false13 gtgt13 ]13gtgt setdistillerparams13ltlt13 HWResolution [2400 2400]13 PageSize [612000 792000]13gtgt setpagedevice13

Page 6: February 2014 MEE Questions and AnalysesPreface The Multistate Essay Examination (MEE) is developed by the National Conference of Bar Examiners (NCBE). This publication includes the

February 2014 MEE

QUESTIONS Constitutional Law

Trusts and Future Interests Secured Transactions

Federal Civil Procedure Criminal Law and Procedure

Agency and Partnership

CONSTITUTIONAL LAW QUESTION ___________

A city ordinance required each downtown business to install high-powered halogen floodlights that would illuminate the property owned by that business and the adjoining sidewalks A study commissioned by the city estimated that installation of the floodlights would cost a typical business about $1000 but that increased business traffic due to enhanced public safety especially after dark would likely offset this cost

A downtown restaurant applied to the city for a building permit to construct an addition that would increase its seating capacity In its permit application the restaurant accurately noted that its current facility did not have sufficient seating to accommodate all potential customers during peak hours The city approved the permit on the condition that the restaurant grant the city an easement over a narrow strip of the restaurantrsquos property to be used by the city to install video surveillance equipment that would cover nearby public streets and parking lots The city based its permit decision entirely on findings that the increased patronage that would result from the increased capacity of the restaurant might also attract additional crime to the neighborhood and that installing video surveillance equipment might alleviate that problem

The restaurant has challenged both the ordinance requiring it to install floodlights and the easement condition imposed on approval of the building permit

1 Under the Fifth Amendment as applied to the states through the Fourteenth Amendment is the city ordinance requiring the restaurant to install floodlights an unconstitutional taking Explain

2 Under the Fifth Amendment as applied to the states through the Fourteenth Amendment is the cityrsquos requirement that the restaurant grant the city an easement as a condition for obtaining the building permit an unconstitutional taking Explain

3

TRUSTS AND FUTURE INTERESTS QUESTION _______________

Ten years ago a testator died survived by his only children a son age 26 and a daughter age 18

A testamentary trust was created under the testatorrsquos duly probated will The will specified that all trust income would be paid to the son during the sonrsquos lifetime and that upon the sonrsquos death the trust would terminate and trust principal would be distributed to the testatorrsquos ldquograndchildren who shall surviverdquo the son The testator provided for his daughter in other sections of the will

Five years ago the trustee of the testamentary trust purchased an office building with $500000 from the trust principal Other than this building the trust assets consist of publicly traded securities

Last year the trustee received $30000 in rents from the office building The trustee also received with respect to the securities owned by the trust cash dividends of $20000 and a stock dividend of 400 shares of Acme Corp common stock distributed to the trust by Acme Corp

Eight months ago the trustee sold the office building for $700000

Six months ago the son delivered a letter to the trustee stating ldquoI hereby disclaim any interest I may have in the income interest of the trustrdquo On the date the son delivered this letter to the trustee the son had no living children the daughter had one living minor child

A statute in this jurisdiction provides that ldquoa disclaimer of any interest created by will is valid only if made within nine months after the testatorrsquos death and if an interest is validly disclaimed the disclaiming party is deemed to have predeceased the testatorrdquo

1 How should the rents sales proceeds cash dividends and stock dividends received prior to the trusteersquos receipt of the sonrsquos letter have been allocated between trust principal and income Explain

2 How if at all does the sonrsquos letter to the trustee affect the future distribution of trust income and principal Explain

4

SECURED TRANSACTIONS QUESTION

On March 1 the owner of a manufacturing business entered into negotiations with a bank to obtain a loan of $100000 for the business The bank loan officer informed the business owner that the interest rate for a loan would be lower if the repayment obligation were secured by all the businessrsquos present and future equipment The loan officer also informed the business owner that the bank could not commit to making the loan until its credit investigation was completed but that funds could be advanced faster following loan approval if a financing statement with respect to the transaction were filed in advance Accordingly the business owner signed a form on behalf of the business authorizing the bank to file a financing statement with respect to the proposed transaction The bank properly filed a financing statement the next day correctly providing the name of the business as the debtor and indicating ldquoequipmentrdquo as the collateral

On March 15 the business owner had heard nothing from the bank about whether the loan had been approved so the business owner approached a finance company for a loan The finance company quickly agreed to lend $100000 to the business secured by all the businessrsquos present and future equipment That same day the finance company loaned to the business $100000 and the business owner signed an agreement obligating the business to repay the loan and granting the finance company a security interest in all the businessrsquos ldquopresent and future equipmentrdquo to secure the repayment obligation Also on that day the finance company properly filed a financing statement correctly providing the businessrsquos name as the debtor and indicating ldquoequipmentrdquo as the collateral

On March 21 the bank loan officer contacted the business owner and indicated that the loan application had been approved On the next day March 22 the bank loaned the business $100000 The loan agreement signed by the owner on behalf of the business granted the bank a security interest in all the businessrsquos ldquopresent and future equipmentrdquo

On April 10 the business sold an item of manufacturing equipment to a competitor for $20000 This was the first time the business had ever sold any of its equipment The competitor paid the purchase price in cash and took possession of the equipment that day The competitor acted in good faith at all times and had no knowledge of the businessrsquos prior transactions with the bank and the finance company

The business has defaulted on its obligations with respect to the loans from the bank and the finance company Each of them has asserted a claim to all the businessrsquos equipment as well as to the item of equipment sold to the businessrsquos competitor

Assume that the business owner had the authority to enter into all these transactions on behalf of the business

1 As between the bank and the finance company which has a superior claim to the businessrsquos equipment Explain

2 Do the claims of the bank and the finance company to the businessrsquos equipment continue in the item of equipment sold to the competitor Explain

5

FEDERAL CIVIL PROCEDURE QUESTION

A builder constructed a vacation house for an out-of-state customer on the customerrsquos land The house was completed on June 1 at which point the customer still owed $200000 of the $800000 contract price which was payable in full five days later

On June 14 the basement of the house was flooded with two inches of water during a heavy rainfall When the customer complained the builder told the customer ldquoThe flooding was caused by poorly designed landscaping Our work is fine and fully up to code Have an engineer look at the foundation If therersquos a problem wersquoll fix itrdquo

The customer pleased by the builderrsquos cooperative attitude immediately hired a structural engineer to examine the foundation of the house On June 30 the engineer provided the customer with a written report on the condition of the foundation which stated that the foundation was properly constructed

Unhappy with the conclusions in the engineerrsquos report the customer then hired a home inspector to evaluate the house The home inspectorrsquos report concluded that the foundation of the house had been poorly constructed and was inadequately waterproofed

On July 10 the customer sent the builder the home inspectorrsquos report with a note that said ldquoUntil you fix this problem you wonrsquot get another penny from merdquo The builder immediately contacted an attorney and directed the attorney to prepare a draft complaint against the customer for nonpayment Hoping to avoid litigation the builder sent several more requests for payment to the customer The customer ignored all these requests

On September 10 the builder filed suit in federal district court properly invoking the courtrsquos diversity jurisdiction and seeking $200000 in damages for breach of contract The customerrsquos answer denied liability on the basis of alleged defective construction of the housersquos foundation

Several months later the case is nearly ready for trial However two discovery disputes have not yet been resolved

First despite a request from the builder the customer has refused to provide a copy of the report prepared by the structural engineer who examined the foundation of the house The customer claims that the report is ldquowork productrdquo and not discoverable because the customer does not intend to ask the engineer to testify at trial The builder has asked the court to order the customer to turn over the engineerrsquos report

Second the customer has asked the court to impose sanctions for the builderrsquos failure to comply with the customerrsquos demand for copies of all emails concerning construction of the foundation of the house The builder has truthfully informed the customer that all such emails were destroyed on August 2 This destruction was pursuant to the builderrsquos standard practice of permanently deleting all project-related emails from company records 60 days after construction of a project is complete There is no relevant state records-retention law

1 Should the court order the customer to turn over the engineerrsquos report Explain

2 Should the court sanction the builder for the destruction of emails related to the case and if so what factors should the court consider in determining those sanctions Explain

6

CRIMINAL LAW AND PROCEDURE QUESTION _____

A defendant was charged under state law with felony theft (Class D) and felony residential burglary (Class C) The indictment alleged that the defendant entered his neighborsrsquo home without their consent and stole a diamond ring worth at least $2500

Defense counsel filed a pretrial motion to dismiss the charges on the ground that prosecuting the defendant for both burglary and theft would constitute double jeopardy The trial court denied the motion and the defendant was prosecuted for both crimes The only evidence of the ringrsquos value offered at the defendantrsquos jury trial was the ownerrsquos testimony that she had purchased the ring two years earlier for $3000

At trial the judge issued the following jury instruction on the burglary charge prior to deliberations

If after consideration of all the evidence presented by the prosecution and defense you find beyond a reasonable doubt that the defendant entered the dwelling without the ownersrsquo consent you may presume that the defendant entered with the intent to commit a felony therein

The jury found the defendant guilty of both offenses

At the defendantrsquos sentencing hearing an expert witness called by the prosecutor testified that the diamond ring was worth between $7000 and $8000 Over defense objection the judge concluded by a preponderance of the evidence that the value of the stolen ring exceeded $5000 The judge sentenced the defendant to four yearsrsquo incarceration on the theft conviction On the burglary conviction the defendant received a consecutive sentence of seven yearsrsquo incarceration

In this state residential burglary is defined as ldquoentry into the dwelling of another without the consent of the lawful resident with the intent to commit a felony thereinrdquo Residential burglary is a Class C felony for which the minimum sentence is five years and the maximum sentence is ten years of incarceration

In this state theft is defined as ldquotaking and carrying away the property of another with the intent to permanently deprive the owner of possessionrdquo Theft is a Class D felony if the value of the item(s) taken is between $2500 and $10000 The sentence for a Class D felony theft is determined by the value of the items taken If the value is between $2500 and $5000 the maximum sentence is three yearsrsquo incarceration If the value of the items exceeds $5000 the maximum sentence is five yearsrsquo incarceration

This state affords a criminal defendant no greater rights than those mandated by the United States Constitution

1 Did the trial court err when it denied the defendantrsquos pretrial motion to dismiss on double jeopardy grounds Explain

2 Did the trial court err in its instruction to the jury on the burglary charge Explain

3 Did the trial court err when it sentenced the defendant to an additional year of incarceration on the theft conviction based on the expertrsquos testimony Explain

7

AGENCY AND PARTNERSHIP QUESTION _____

Five years ago Adam and Ben formed a general partnership Empire Partnership (Empire) to buy and sell antique automobiles at a showroom in State A Adam contributed $800000 to Empire and Ben contributed $200000 Their written partnership agreement allocated 80 of profits losses and control to Adam and 20 to Ben No filings of any type were made in connection with the formation of Empire

Three years ago a collector purchased one of Empirersquos antique cars for $3400000 The collector was willing to pay this price because of Benrsquos false representation (repeated in the sales contract) that a famous movie star had once owned the car Without the movie-star connection the car was worth only $100000 One month later when the collector discovered the truth he sued Adam Ben and Empire for $3300000 in damages The lawsuit is still pending

Two years ago Adam and Ben admitted a new partner Diane to Empire in return for her contribution of $250000 The three agreed to allocate profits losses and control 75 to Adam 10 to Ben and 15 to Diane Before joining the partnership Diane learned of the collectorrsquos claim and stated her concern to Adam and Ben that she might become liable if the claim were reduced to a judgment

Following Dianersquos admission to Empire the three partners sought to convert Empire into a limited liability partnership (LLP) Adamrsquos lawyer proposed to file with State A a ldquostatement of qualificationrdquo making an LLP election and declaring the name of the partnership to be ldquoEmpire LLPrdquo Benrsquos lawyer stated that this would not work and that a new LLP had to be formed with the assets of the old partnership transferred to the new one In the end the conversion was done the way Adamrsquos lawyer suggested with the approval of all three partners

One year ago a driver purchased a vintage car from Empire LLP based on the representation that the car was ldquofully roadworthy and capable of touring at 70 mph all dayrdquo The driver took the car on the highway at 50 mph whereupon the front suspension collapsed resulting in a crash in which the car was destroyed and the driver killed The driverrsquos estate sued Adam Ben Diane and Empire LLP for $10000000 The lawsuit is still pending

Although profitable Empire LLP does not have resources sufficient to pay the collectorrsquos claim or the claim of the driverrsquos estate

Assume that the Uniform Partnership Act (1997) applies

1 Before the filing of the statement of qualification (a) was Adam personally liable on the collectorrsquos claim Explain (b) was Diane personally liable on the collectorrsquos claim Explain

2 After the filing of the statement of qualification was Adam Ben or Diane personally liable as a partner on (a) the collectorrsquos claim or (b) the driverrsquos estatersquos claim Explain

8

February 2014 MEE

ANALYSES Constitutional Law

Trusts and Future Interests Secured Transactions

Federal Civil Procedure Criminal Law and Procedure

Agency and Partnership

CONSTITUTIONAL LAW ANALYSIS (Constitutional Law IVD)

ANALYSIS

Legal Problems

(1) Is the city ordinance requirement that businesses install floodlights a taking

(2) Is conditioning the approval of a building permit on the grant of an easement to install surveillance equipment a taking of property

DISCUSSION

Summary

The ordinance requiring businesses to install floodlights is not a per se taking under Loretto because it does not force a private landowner to allow a third party to enter and place a physical object on the land Here the city ordinance requires the businessmdashnot a third partymdashto install the floodlights

The ordinance is likely not a regulatory taking under the Penn Central balancing test While the ordinance will impose a cost on business owners that cost may be offset by the expected increase in business due to the ordinance and the ordinance does not appear to interfere with the ownerrsquos primary use of the property as a restaurant

The permit condition however is likely an uncompensated taking of property While the condition has an essential nexus with the cityrsquos legitimate interest in promoting public safety the city has not made an individualized determination that the easement condition is roughly proportional to the possibility of increased crime due to the restaurantrsquos proposed addition Thus the permit condition likely violates the Fifth Amendment as applied to the states through the Fourteenth Amendment

Point One (50) The ordinance requiring that businesses install floodlights is not a per se taking under Loretto It is not a regulatory taking under the Penn Central balancing test because the cost of compliance with the ordinance may be offset by an expected increase in business and compliance does not interfere with the businessrsquos primary use of its property as a restaurant

The city ordinance requiring a business to install floodlights does not effect a per se taking of the sort described in Loretto v Teleprompter Manhattan CATV Corp 458 US 419 (1982) because no property is physically taken by the government and the ordinance does not involve a physical invasion of private property by a third party

Even though the ordinance does not constitute an occupation of the property by either the government or a third party it is still subject to the three-factor balancing test under Penn Central Transportation Co v City of New York 438 US 104 (1978) to determine whether it is a ldquoregulatory takingrdquo Under Penn Central a court must balance (1) ldquo[t]he economic impact of the regulation on the claimantrdquo (2) ldquothe extent to which the regulation has interfered with distinct investment-backed expectationsrdquo and (3) ldquothe character of the governmental actionrdquo Id at 124 Here each factor weighs against finding that the ordinance is a taking

11

Constitutional Law Analysis

First the ordinance requirement likely has a minimal economic impact on the restaurant Compliance with the ordinance is estimated to cost $1000 and the city has found that businesses will likely recoup that cost in increased sales Also because the ordinance does not interfere with the operation of the restaurant the owner may still earn a reasonable return on its investment in the property

Second the ordinance does not interfere with the businessrsquos investment-backed expectations As in Penn Central the challenged law does not interfere with the ownerrsquos ldquoprimary expectationrdquo for use of the propertymdashin Penn Central as a railroad terminal and here as a restaurant Further the ordinance does not prevent the restaurant from expanding to meet the changing business environment

Third the character of the government action does not weigh in favor of a taking While Penn Central does say that a ldquophysical invasionrdquo is more likely to pose a taking Loretto suggests that the Courtrsquos main concern is with physical invasions by third parties Also like the landmark law challenged in Penn Central the ordinance here ldquoadjust[s] the benefits and burdens of economic life to promote the common goodrdquo Id In Penn Central the landmark law restricted development of the railroad terminal to promote the common interest in preserving historic landmarks Here the ordinance requires the businesses to install floodlights to promote the common interest in crime prevention and public safety

Because the ordinance is clearly a valid exercise of the police power it satisfies the takings clausersquos public-use requirement Kelo v City of New London 545 US 469 (2005)

In sum all three factors weigh against finding a taking under the Penn Central balancing test

Point Two (50) The permit condition may be unconstitutional as an uncompensated taking of property because the city has not made an individualized determination that the easement condition is roughly proportional to the impact of the restaurantrsquos proposed addition

In Dolan v City of Tigard 512 US 374 (1994) the Supreme Court set forth the test for determining whether an exaction imposed by a government in exchange for a discretionary benefit conferred by the government such as a condition on the approval of a building permit in this case constitutes an uncompensated taking under the Fifth Amendment The exaction is not a taking if (1) there is an ldquoessential nexusrdquo between the ldquopublic need or burdenrdquo to which the proposed development contributes and ldquothe permit condition exacted by the cityrdquo id at 386 and (2) the government makes ldquosome sort of individualized determination that the required dedication is [roughly proportional] both in nature and extent to the impact of the proposed developmentrdquo Id at 391 see also Nollan v California Coastal Commission 483 US 825 (1987)

Here the city likely can meet the nexus requirement In Dolan the landowner sought to double the size of its business which would have increased traffic on nearby roadways In exchange for approving the development the city sought an easement for a bike and pedestrian path The Court found the required nexus between the easement and the cityrsquos ldquoattempt to reduce traffic congestion by providing for alternative means of transportationrdquo 512 US at 387 Here a similar nexus likely exists between the requested easement and the cityrsquos interest in crime prevention and public safety Increased patronage and economic activity at the restaurant might attract additional crime to the area and the requested easement to install surveillance equipment would attempt to address that increased crime

12

Constitutional Law Analysis

The exaction here however may fail the second prong of the Dolan testmdashthat the exaction be roughly proportional to the anticipated impact of the requested development As noted the city in Dolan claimed that a bike and pedestrian path was needed to offset the increase in traffic due to the proposed doubling of the business The Court explained that the government must demonstrate that the additional traffic reasonably was related to the requested exaction and that the government must ldquomake some effort to quantify its findings in support of the dedication for the pedestrianbicycle pathway beyond the conclusory statement that it could offset some of the traffic demand generatedrdquo Id at 395 Here the city did not carry its burden The city simply speculates that increased patronage of the restaurant ldquomightrdquo increase crime and that the surveillance equipment ldquomightrdquo alleviate this increased crime Because the city has not made ldquosome effort to quantify its findingsrdquo in support of the easement it has not shown that the burden of the easement is roughly proportional to the benefits thought to flow from it

Thus the exaction appears to be an uncompensated taking of property in violation of the Fifth Amendment as applied to the states through the Fourteenth Amendment

13

TRUSTS AND FUTURE INTERESTS ANALYSIS ____ (Trusts and Future Interests IE3 I5 IIIA amp B)

ANALYSIS

Legal Problems

(1) How should rents dividends and sales proceeds received by the trustee prior to receipt of the sonrsquos letter have been allocated between trust income and principal

(2)(a) Did the remainder interest in the trust accelerate and become immediately payable to the daughterrsquos minor child upon the trusteersquos receipt of the sonrsquos letter and if not how should the trustee handle the distribution of the principal in the future

(2)(b) Following the trusteersquos receipt of the sonrsquos letter how should the trustee distribute future receipts of income prior to the distribution of the principal

DISCUSSION

Summary

Prior to the trusteersquos receipt of the sonrsquos letter cash dividends and rents should have been allocated to trust income and were distributable to the son the income beneficiary of the trust sales proceeds and stock dividends should have been allocated to principal

Because the sonrsquos letter to the trustee did not result in a valid disclaimer under state law (having been made more than nine months after the testatorrsquos death) the son is not deemed to have predeceased the testator Because the son is still living the class gift to the testatorrsquos grandchildren who survive the son has not closed and is not possessory it will not become possessory until the son dies The daughterrsquos minor child being the testatorrsquos only living grandchild is not currently entitled to a distribution of trust principal Trust principal will instead be distributable upon the sonrsquos death to the testatorrsquos then-living grandchildren or if there are none to the testatorrsquos then-living heirs

As for future income the trustee should either distribute the trust income to the son and the daughter as the testatorrsquos heirs accumulate the income for future distribution to those individuals ultimately entitled to the trust principal or distribute it to those presumptively entitled to the principal upon the sonrsquos death ie the daughterrsquos minor child

Point One (45) Cash dividends and rents are allocable to income sales proceeds and stock dividends are allocable to principal Items allocable to income for the period prior to the sonrsquos attempted disclaimer were distributable to the son

Receipts earned during the administration of a trust are allocable either to income or to principal Almost all states have adopted the most recent or an earlier version of the Uniform Principal and Income Act (the Act) which specifies how such receipts should be allocated

Under the Act rents (UNIF PRIN amp INC ACT (2000) sect 405 UNIF PRIN amp INC ACT (1962) sect 3(a)(1)) and cash dividends received from a corporation (UNIF PRIN amp INC ACT (2000) sect 401(b) UNIF PRIN amp INC ACT (1962) sect 6(d)) are allocable to income and are distributable to the income beneficiary of the trust

14

Trusts and Future Interests Analysis Sales proceeds (UNIF PRIN amp INC ACT (2000) sect 404(2) UNIF PRIN amp INC ACT (1962)

sect 3(b)(1)) and dividends paid in the stock of the distributing corporation (UNIF PRIN amp INC ACT (2000) sect 401(c)(1) UNIF PRIN amp INC ACT (1962) sect 3(b)(4)) are allocable to principal and added to the principal of the trust

Here the cash dividends and office building rents should have been allocated to income and until the trustee received the sonrsquos letter should have been distributed to him as the sole income beneficiary of the trust The stock dividend and proceeds from the sale of the office building should have been allocated to principal and held by the trustee for future distribution to the ultimate remaindermen of the trust

[NOTE The 2000 Uniform Principal and Income Act has been adopted in Alabama Arkansas Colorado Connecticut the District of Columbia Hawaii Idaho Iowa Kentucky Missouri Montana Nebraska New Mexico North Dakota Oregon South Dakota Utah and West Virginia]

Point Two(a) (45) Because the son did not disclaim within nine months of the testatorrsquos death there is no valid disclaimer under state law Therefore the son is not deemed to have predeceased the testator Furthermore because of the express survivorship contingency in the will the remainder in the trust does not accelerate and become distributable until the son in fact dies When the son dies the trust principal will be distributable to the testatorrsquos then-living grandchildren or if none then to the testatorrsquos then-living heirs

When a trust remainder is given to a class the class closes (ie no new persons can join the class) when there is no outstanding income interest and at least one member of the class is then entitled to demand possession of his or her share of the remainder This principle is called the rule of convenience See generally HERBERT HOVENKAMP amp SHELDON F KURTZ PRINCIPLES OF PROPERTY LAW 199ndash200 (6th ed 2005) A class member may demand possession of his or her share of the remainder upon termination of the income interest only when the class memberrsquos interest is not otherwise subject to a condition precedent See id

When a beneficiary timely disclaims an interest in a trust that beneficiary is treated as if he had predeceased the testator Here had the son disclaimed within nine months of the testatorrsquos death as required by the state statute he would have been deemed to have predeceased the testator This would have closed the class of remaindermen and the testatorrsquos then-living grandchildren (ie the daughterrsquos child) would have been entitled to the trust principal However under the state statute the sonrsquos disclaimer was not timely because he did not disclaim within nine months of the testatorrsquos death Thus because the statute is inapplicable and the son is still alive the class of grandchildren entitled to share in trust principal did not close

Because here the statute is inapplicable due to the sonrsquos failure to comply with the statutory time requirements then presumably the common-law rule allowing disclaimers (aka renunciations) at any time should apply Under the common law if a life estate is renounced the remainder interest accelerates and becomes immediately distributable to the remaindermen of the trust if the remainder is vested but not if the remainder is contingent JESSE DUKEMINIER amp ROBERT H SITKOFF WILLS TRUSTS AND ESTATES 844ndash845 (9th ed 2013) Here because the remainder is contingent upon there being grandchildren who survive the son the remainder will not accelerate It will remain open until the son dies leaving open the possibility that additional grandchildren will be included in the class or the daughterrsquos child could fall out of the class because that child fails to survive the son

And if none of the testatorrsquos grandchildren survive the son the trust principal will be distributed to the testatorrsquos heirs living at the sonrsquos death

15

Trusts and Future Interests Analysis

Point Two(b) (10) Until the trust terminates the trustee must continue to hold the trust assets The distribution of income in the meantime is unclear There are at least three possibilities Income earned on the undistributed assets could be distributed to the son and daughter as the testatorrsquos heirs accumulated and added to principal for distribution to the ultimate remaindermen or distributed from time to time to those persons who are presumptively remaindermen

When trust principal is not immediately distributable the trustee must continue to hold trust assets until the ultimate remaindermen are ascertained During this period trust income will be distributed or retained according to any instructions contained in the trust instrument See WILLIAM M MCGOVERN JR SHELDON F KURTZ amp DAVID M ENGLISH WILLS TRUSTS amp ESTATES sect 102 (4th ed 2010)

Here the testator did not specify what the trustee should do with trust income in the event the sonrsquos disclaimer did not comply with the state statute There are at least three approaches One approach would have the trustee distribute the trust income to the testatorrsquos heirs on the theory that the income represents property that was not disposed of by the testatorrsquos will and which thus passes by partial intestacy to the testatorrsquos heirs A second approach would have the trustee accumulate trust income for distribution to the ultimate remaindermen Under this approach only those individuals ultimately entitled to the principal would share in the income A third approach would have the trustee distribute trust income to those individuals who would be the remaindermen if the trust were to terminate when the income is received by the trustee under this approach trust income would be distributed to the daughterrsquos minor child until another presumptive remainderman is born This approach could result in individuals not ultimately entitled to principal say because they do not survive the son receiving income It could also result in a disproportionate distribution of income among the individuals ultimately entitled to income

[NOTE Examinees should demonstrate a recognition and understanding of the income-allocation problem and the alternatives available to address that issue There is no widely accepted solution to the problem Examinees who cite any of these possible problem-solving approaches may receive credit]

16

SECURED TRANSACTIONS ANALYSIS (Secured Transactions IB IID E amp F IIIB IVA B amp F)

ANALYSIS

Legal Problems

(1)(a) What is the nature of the bankrsquos claim to the businessrsquos equipment

(1)(b) What is the nature of the finance companyrsquos claim to the businessrsquos equipment

(1)(c) As between the bank and the finance company whose claim to the businessrsquos equipment has priority

(2) Do the claims of the bank and the finance company continue in the item of equipment sold by the business to the competitor

DISCUSSION

Summary

The bank and the finance company both have perfected security interests in the businessrsquos equipment Even though the finance companyrsquos perfected security interest was created first the bankrsquos perfected security interest has priority because the bankrsquos financing statement was filed before the finance companyrsquos financing statement The security interests of the bank and the finance company continue in the item of equipment sold by the business to the competitor because their security interests were perfected and the competitor was not a buyer in ordinary course of business

Point One(a) (25) The bank has a perfected security interest in the businessrsquos equipment

The bank has met all criteria necessary for it to have an attached and enforceable security interest in the businessrsquos equipment First value must be given UCC sect 9-203(b)(1) This criterion is fulfilled by the loan made by the bank to the business Second the debtor must have rights in the collateral UCC sect 9-203(b)(2) Clearly the business has rights in its equipment Third either the secured party must take possession of the collateral or the debtor must authenticate a security agreement containing a description of the collateral UCC sect 9-203(b)(3) The agreement that the business owner signed is a ldquosecurity agreementrdquo because it is an agreement that creates or provides for a security interest UCC sect 9-102(a)(74) By signing the security agreement the business owner authenticated it UCC sect 9-102(a)(7) Therefore all three criteria are fulfilled and the bank has an enforceable and attached security interest

A security interest is perfected when it has attached and when any additional steps required for perfection have occurred UCC sect 9-308(a) Generally speaking the additional steps will either be possession of the collateral by the secured party or the filing of a financing statement with respect to the collateral See UCC sectsect 9-310 9-313 In this case the bank filed a financing statement naming the debtor and sufficiently indicating the collateral The collateral indication is sufficient because it identifies the collateral by type of property See UCC sectsect 9-504 9-108 The fact that the financing statement was filed before the security interest was created is

17

Secured Transactions Analysis

not a problem Even though the security agreement had not yet been signed the business had authorized the filing of the financing statement in an authenticated record UCC sect 9-509(a)(1) Moreover the financing statement may be filed before the security agreement is created UCC sect 9-502(d)

Point One(b) (10) The finance company also has a perfected security interest in the businessrsquos equipment

The finance companyrsquos security interest is enforceable and attached for the same reasons as the bankrsquos security interest The loan from the finance company to the business constitutes value the business has rights in the collateral and the business owner has authenticated a security agreement containing a description of the collateral The finance companyrsquos security interest is perfected because the finance company filed a financing statement with respect to it that provides that the business is the debtor and indicates that the collateral is equipment

Point One(c) (30) The bankrsquos security interest has priority over the finance companyrsquos security interest because the bankrsquos financing statement was filed first

As between two perfected security interests the general rule is that the security interest that was the earlier to be either perfected or the subject of a filed financing statement has priority UCC sect 9-322(a)(1) While the finance companyrsquos security interest was perfected before the bankrsquos (March 15 vs March 22) the bankrsquos financing statement was filed even earlier on March 2 Thus under the first-to-file-or-perfect rule of UCC sect 9-322(a)(1) the bankrsquos security interest has priority No exceptions to the general rule apply here

Point Two (35) A security interest in collateral continues notwithstanding its sale unless an exception applies Because the security interests of the bank and the finance company were perfected and the competitor was not a buyer in ordinary course of business no exception applies and the security interests of both creditors continue in the equipment sold to the competitor

As a general rule a security interest in collateral continues notwithstanding the fact that the debtor has sold the collateral to another person UCC sect 9-315(a)(1) Thus unless an exception applies the security interests of the bank and the finance company will continue in the item of equipment sold to the competitor

A buyer of goods will take free of an unperfected security interest in those goods See UCC sect 9-317(a)(2) However when the competitor bought the businessrsquos equipment both the bank and the finance company had perfected security interests in the equipment

A buyer can take free even of a perfected security interest in goods if the buyer is a ldquobuyer in ordinary course of businessrdquo See UCC sect 9-320(a) However the competitor was not a buyer in ordinary course of business To be a ldquobuyer in ordinary course of businessrdquo a buyer must buy goods from a seller that is in the business of selling goods of that kind See UCC sect 1-201(b)(9) The competitor bought this equipment from a seller that is not in the business of selling goods of this kind so the competitor was not a buyer in ordinary course of business with respect to these goods

Because no exception applies the security interests of the bank and the finance company continue even after the item of equipment was sold to the competitor

18

FEDERAL CIVIL PROCEDURE ANALYSIS (Federal Civil Procedure IVD)

ANALYSIS

Legal Problems

(1) Is a document prepared in the course of a contract dispute protected from discovery as ldquowork productrdquo when there is no evidence that the document was prepared in anticipation of litigation

(2)(a) Is a partyrsquos failure to provide relevant electronically stored information excused when the information was destroyed pursuant to a routine document retention scheme at a time when litigation was contemplated by the destroying party

(2)(b) What sanctions should be imposed on a party for allowing the destruction of evidence that is relevant to potential future litigation

DISCUSSION

Summary

The report prepared by the structural engineer is probably not work product and is thus discoverable The engineer examined the foundation of the house at the customerrsquos request and the engineerrsquos findings are potentially relevant to the customerrsquos claim that the foundation is defective The report was not prepared in anticipation of litigation The customer appears to have sought the engineerrsquos opinion in response to the builderrsquos offer to fix any problems with the foundation that an engineer might identify Because the report was not prepared in anticipation of litigation it is not protected by the work-product doctrine

The builder should have taken appropriate steps to preserve evidence including suspending its document retention program as soon as it began planning for litigationmdashie on July 10 Its destruction of potentially relevant material after that date was wrongful However a court is unlikely to impose severe sanctions on the builder because there are no facts indicating that the builder acted in bad faith and the customer can prove that the foundation is defective without the destroyed emails

Point One (40) The customer must turn over the engineerrsquos report because it was not prepared in anticipation of litigation

In general a party to a lawsuit in federal court ldquomay obtain discovery regarding any nonprivileged matter that is relevant to any partyrsquos claim or defenserdquo FED R CIV P 26(b)(1) (2009) This includes the right to inspect and copy documents in the other partyrsquos possession FED R CIV P 34(a)(1) Here the customer hired a structural engineer to examine the foundation of the house The engineerrsquos report on the foundation is likely to include information that would be relevant to the customerrsquos claim that the foundation was defectively constructed

The so-called ldquowork productrdquo rule allows a party to refuse to turn over ldquodocuments that are prepared in anticipation of litigation or for trialrdquo by that partyrsquos representative including

19

Federal Civil Procedure Analysis

a consultant Thus if the customer had hired the structural engineer to prepare a report ldquoin anticipation of litigationrdquo that report might not be discoverable See FED R CIV P 26(b)(3)

In this case however the customer hired the engineer to evaluate the foundation of the house as part of the customerrsquos negotiation with the builder concerning the housersquos flooding problem The builder told the customer that the housersquos landscaping was the reason for the flooding and the builder told the customer ldquoHave an engineer look at the foundation If therersquos a problem wersquoll fix itrdquo The customer appears to have acted in response to that statement There is no indication that the customer anticipated any kind of legal action at the time that the structural engineer was hired Accordingly the structural engineerrsquos report is discoverable and the court should order the customer to turn it over

[NOTE If an examinee concludes that the structural engineerrsquos report was prepared in anticipation of litigation then the examinee should also conclude that the report is not discoverable Documents prepared in anticipation of litigation do not need to be disclosed to an adverse party unless that party can demonstrate a ldquosubstantial needrdquo for the documents and an inability to obtain substantially equivalent information without ldquoundue hardshiprdquo FED R CIV P 26(b)(3)(A)(ii) Furthermore a report prepared by an expert who is not expected to testify is not discoverable in the absence of ldquoexceptional circumstancesrdquo making it ldquoimpracticablerdquo to obtain the information in another way FED R CIV P 26(b)(4)(D)(ii) The builder probably cannot make these showings here unless the engineerrsquos report deals with circumstances that have since changed There is no evidence that the structural engineer would have had access to any information or facts that the builder would not already know as a result of its construction and subsequent inspection of the house In addition if necessary the builder could ask the court for permission to arrange for a further inspection of the house by a structural engineer hired by the builder See FED R CIV P 34(a)(2) Accordingly if an examinee concludes that the report was prepared in anticipation of litigation the examinee should also conclude that the builder is not entitled to see the report]

Point Two(a) (30) Because the builder anticipated that it might be involved in litigation concerning its contract with the customer the builder acted wrongfully in destroying emails that were relevant to the housersquos construction even though the emails were destroyed pursuant to a routine document retention plan

As noted above a party to a lawsuit in federal court ldquomay obtain discovery regarding any nonprivileged matter that is relevant to any partyrsquos claim or defenserdquo FED R CIV P 26(b)(1) This includes emails and other electronically stored information FED R CIV P 34(a)(1)(A) Here the customer has requested all the builderrsquos emails pertaining to work done on the foundation of the house Ordinarily the builder would be obliged to turn over this information which is relevant to the customerrsquos defense that the housersquos foundation was poorly constructed

Unfortunately the emails in question no longer exist because the builder destroyed them on August 2

In general spoliation of evidence (destruction or alteration of evidence) is improper if the party who destroyed or altered the evidence ldquohas notice that the evidence is relevant to litigation or should have known that the evidence may be relevant to future litigationrdquo Fujitsu Ltd v Federal Express Corp 247 F3d 423 436 (2d Cir 2001) It is improper for a party to destroy electronic information relevant to pending litigation even if the destruction occurs before there is any request or order seeking the information See eg Leon v IDX Sys Corp 464 F3d 951 (9th Cir 2006) (plaintiffrsquos intentional destruction of computer files warranted dismissal even

20

In this case the builderrsquos destruction of the emails was pursuant to a routine document retention plan The Federal Rules provide expressly that in the absence of ldquoexceptional circumstancesrdquo parties should not be sanctioned for the loss of electronically stored information when the loss occurs pursuant to ldquoroutine good-faith operation of an electronic information systemrdquo FED R CIV P 37(e) However when a party anticipates litigation ldquoit must suspend its routine document retentiondestruction policy and put in place a lsquolitigation holdrsquo to ensure the preservation of relevant documentsrdquo Zubulake v UBS Warburg LLC 220 FRD 212 218 (SDNY 2003)

Federal Civil Procedure Analysis

though spoliation occurred before order compelling discovery) Similarly the duty to preserve evidence applies to a party who anticipates litigation even if litigation has not yet been commenced See THE SEDONA PRINCIPLES BEST PRACTICES RECOMMENDATIONS amp PRINCIPLES FOR ADDRESSING ELECTRONIC DOCUMENT PRODUCTION 70 cmt 14a (2d ed 2007)

The builder destroyed the emails on August 2 At that time the builder knew that litigation was a possibility because the builder had already directed its attorney to prepare a draft complaint for possible filing Knowing that litigation was a possibility the builder had a duty to take steps to preserve evidence including the emails in question See generally Fujitsu Ltd

Thus the builderrsquos destruction of potentially relevant emails at a time when it knew that litigation was a possibility was improper It had a duty to preserve evidence and it breached that duty

[NOTE Because courts have used different words to describe the test for when evidence must be preserved an examineersquos precise formulation of the test is not critical]

Point Two(b) (30) In determining appropriate sanctions for spoliation courts consider both the level of culpability of the spoliating party and the degree of prejudice the loss of evidence has caused the other party Here the builderrsquos destruction of evidence does not appear to have been willful nor is it likely to pose a significant obstacle to the customerrsquos defense Any sanctions imposed by the court should be modest

Federal courts have inherent power to control the litigation process and can sanction misbehavior including spoliation even when there has been no specific violation of the Federal Rules of Civil Procedure See generally Chambers v NASCO Inc 501 US 32 (1991) (discussing courtrsquos inherent power to control the litigation process) The range of available sanctions is broad It can include such sanctions as the payment of expenses incurred by the other party as a result of the destruction of the evidence an instruction to the jury authorizing it to draw an adverse inference from the destruction of the evidence a shifting of the burden of proof on the relevant issue or even judgment against the responsible party See eg Residential Funding Corp v DeGeorge Financial Corp 306 F3d 99 108 (2d Cir 2002) (adverse inference) Silvestri v General Motors Corp 271 F3d 583 593 (4th Cir 2001) (possibility of dismissal) Cf FED R CIV P 37(b)(2)(A) (listing remedies for failure to comply with discovery obligations)

In determining appropriate sanctions for spoliation courts consider both the level of culpability of the spoliating party and the degree of prejudice the loss of evidence has caused the other party Many courts impose severe sanctions (such as an adverse-inference instruction or the entry of judgment against the spoliating party) only when there is evidence of bad faith in the form of an intentional effort to hide information Eg Greyhound Lines Inc v Wade 485 F3d 1032 1035 (8th Cir 2007) (spoliation sanction requires intentional destruction out of desire ldquoto suppress the truthrdquo) However other courts have said that negligence in preserving evidence can

21

Federal Civil Procedure Analysis

support an adverse-inference instruction See Residential Funding 306 F3d at 108 (negligence enough under some circumstances)

Although a court might well order an evidentiary hearing on the issue of sanctions the facts presented do not seem appropriate for severe sanctions First the evidence was destroyed pursuant to the builderrsquos standard document retention plan and there is no evidence that the builder deliberately failed to suspend its usual procedures with the purpose of allowing the destruction of evidence Second the loss of this evidence will not severely hinder the customerrsquos presentation of his case The central issue is whether the foundation of the house was properly constructed If the construction job was poorly done the customer can present evidence derived from inspection of the premises to prove that point The customer can also depose witnesses about any issues that arose during construction

Under the circumstances a court is not likely to impose particularly severe sanctions although it might shift the burden to the builder to show that the foundation was properly constructed or it might require the builder to reimburse any expenses the customer incurs to discover and prove the facts about issues or disputes that arose during construction of the foundation

[NOTE The result reached by the examinee is less important than the examineersquos recognition that (a) a range of sanctions is available to the court and (b) the appropriate sanction depends both on the culpability of the builder and the prejudice suffered by the customer]

22

CRIMINAL LAW AND PROCEDURE ANALYSIS (Criminal Law and Procedure IIA amp D VE amp F)

ANALYSIS

Legal Problems

(1) Did charging the defendant with both theft and burglary constitute double jeopardy

(2) Did the jury instruction violate the due process clause either by relieving the prosecution of the burden of proving the element of intent or by shifting the burden to the defendant to disprove that element

(3) Did the sentence imposed in this case for the theft conviction unconstitutionally deprive the defendant of his right to a jury trial on the issue of the value of the stolen item

DISCUSSION

Summary

The trial court properly denied the defendantrsquos pretrial motion to dismiss the charges on double jeopardy grounds The defendant may be charged with and convicted of both theft and burglary Each of the charges has an element that the other does not Neither charge is a lesser-included offense nor are they multiplicitous Thus charging both theft and burglary does not violate double jeopardy

The jury instruction on the burglary charge was constitutionally flawed It could have been reasonably understood by the jury as either (1) an irrebuttable conclusive presumption (which relieved the prosecution of proving the element of intent and removed the issue from the jury) or (2) a rebuttable mandatory presumption (which unconstitutionally shifted the burden of proof on an element of a charged offense from the prosecution to the defendant)

Because the four-year sentence imposed by the judge was based on the judgersquos finding by a preponderance of the evidence that the value of the stolen ring exceeded $5000 the sentence violates the defendantrsquos right to a jury determination beyond a reasonable doubt of the value of the ring

Point One (30) Charging the defendant with theft and burglary did not constitute double jeopardy

The Double Jeopardy Clause of the Fifth Amendment provides that a person shall not be twice put in jeopardy for the ldquosame offenserdquo Thus the question is whether the elements of the theft charge are wholly contained in the burglary charge or vice versa If the elements of the lesser charge (theft) are not wholly contained in the greater charge (burglary)mdashie if each charge requires proof of a fact that the other does notmdashthen convicting the defendant of both crimes would not violate double jeopardy even when the two offenses occurred at the same time and are thus arguably part of the ldquosame transactionrdquo Blockburger v United States 284 US 299 304 (1932) See also Albernaz v United States 450 US 333 344 n3 (1981) United States v Dixon 509 US 688 704 (1993)

23

Criminal Law and Procedure Analysis

Here theft and burglary each require proof of an element not required for the other crime Burglary may be defined differently in different jurisdictions However it almost invariably requires entry into a building or dwelling of another with the specific intent to commit a felony therein and the crime of burglary is complete upon the entry into the building or dwelling with such intent See eg Cannon v Oklahoma 827 P2d 1339 1342 (Okla Crim App 1992) In contrast theft which also may be defined differently in different states almost invariably requires the taking and carrying away of an item of personal property belonging to another with the intent to steal or permanently deprive the owner of possession

Here the ldquotakingrdquo or ldquostealingrdquo element is not contained in the definition of burglary and the ldquoentryrdquo element of burglary is not contained in the definition of theft Because theft is not a lesser-included offense of burglary and burglary is not a lesser-included offense of theft charging the defendant for both burglary and theft did not violate double jeopardy and the court properly denied the defense motion on those grounds Yparrea v Dorsey 64 F3d 577 579ndash80 (10th Cir 1995) citing Blockburger 284 US at 304

Finally the defendantrsquos motion to dismiss all the charges on double jeopardy grounds was improper because if both charges were for the same offense the motion should have requested dismissal of one charge not both

Point Two (35) The jury instruction on the burglary charge violated the Due Process Clause because it created either (1) an irrebuttable conclusive presumption (which relieved the prosecution of proving the element of intent and removed that issue from the jury) or (2) a rebuttable mandatory presumption (which unconstitutionally shifted the burden of proof on an element of a charged offense to the defendant)

The Supreme Court has interpreted the Due Process Clause of the US Constitution to require that the prosecution prove all elements of an offense beyond a reasonable doubt See In re Winship 397 US 358 364 (1970) The burden of proof cannot be shifted to the defendant by presuming an essential element upon proof of other elements of the offense because shifting the burden of persuasion with respect to any element of a criminal offense is contrary to the Due Process Clause See Mullaney v Wilbur 421 US 684 (1975)

The crime of burglary includes entry into a building or dwelling with the specific intent to commit a felony therein The requirement that the prosecutor prove beyond a reasonable doubt that the defendant had this specific intent distinguishes burglary from general-intent crimes like trespass See Sandstrom v Montana 442 US 510 523 (1979)

Here the jury was instructed that if ldquoafter consideration of all the evidence presented by the prosecution and defense you find beyond a reasonable doubt that the defendant entered the dwelling without the ownersrsquo consent you may presume that the defendant entered with the intent to commit a felony thereinrdquo This instruction was unconstitutional because it created either an irrebuttable conclusive presumption or a rebuttable mandatory presumption

A conclusive presumption is ldquoan irrebuttable direction by the court to find intent once convinced of the facts triggering the presumptionrdquo Id at 517 Here the jurors were instructed that once the prosecutor established that the defendant entered the neighborsrsquo house without consent they ldquomay presumerdquo that he intended to commit a felony therein The jurors may have reasonably concluded from this instruction that if they found that the defendant intended to enter his neighborsrsquo home without permission they must further find that he entered with the specific intent to commit a felony therein Because this instruction could operate as a conclusive

24

Criminal Law and Procedure Analysis

irrebuttable presumption by eliminating intent ldquoas an ingredient of the offenserdquo it violated due process by relieving the prosecution of the burden of proof for this element Id at 522

In the alternative the jury instruction could have been reasonably understood to create a rebuttable mandatory presumption which ldquotells [the jury] they must find the elemental fact upon proof of the basic fact at least unless the defendant has come forward with some evidence to rebut the presumed connection between the two factsrdquo County Court of Ulster County New York v Allen 442 US 140 157 (1979) The due process problem created by rebuttable mandatory presumptions is that ldquo[t]o the extent that the trier of fact is forced to abide by the presumption and may not reject it based on an independent evaluation of the particular facts presented by the State the analysis of the presumptionrsquos constitutional validity is logically divorced from those facts and based on the presumptionrsquos accuracy in the run of casesrdquo Id at 159

Unlike irrebuttable conclusive presumptions rebuttable mandatory presumptions are not always per se violations of the Due Process Clause However the Supreme Court of the United States has held that jury instructions that could reasonably be understood as shifting the burden of proof to the defendant on an element of the offense are unconstitutional Francis v Franklin 471 US 307 (1985) Here the argument that the jury instruction operated as a rebuttable mandatory presumption is supported by the fact that the judge also instructed the jury to ldquoconsider[ ] all the evidence presented by the prosecution and defenserdquo However even if the instruction created a rebuttable mandatory presumption it would be unconstitutional because it shifted the burden to the defense on an element of the offense Sandstrom 442 US at 524 Mullaney 421 US at 686

[NOTE Whether an examinee identifies the jury instruction as containing a ldquoconclusiverdquo or ldquomandatoryrdquo presumption is less important than the examineersquos analysis of the constitutional infirmities]

Point Three (35) The trial court violated the defendantrsquos Sixth Amendment right to a jury trial on an essential element of the offense when it found by a preponderance of the evidence that the ring was worth over $5000 and increased the defendantrsquos sentence based on this finding

In the statutory scheme under which the defendant was tried and convicted a Class D felony theft is defined as theft of item(s) with a value between $2500 and $10000 The jury found that the value of the diamond ring was at least $2500 and convicted the defendant of felony theft However at sentencing the trial court made a separate finding by a preponderance of the evidence that the value of the ring was greater than $5000 Following the statutersquos two-tiered sentencing scheme the judge then imposed on the defendant a sentence that was one year longer than the maximum that would otherwise have been allowed

The judgersquos sentence was unconstitutional because it violated the defendantrsquos Sixth Amendment right to a jury trial on this question The Supreme Court held in Apprendi v New Jersey 530 US 466 (2000) that ldquo[o]ther than the fact of a prior conviction any fact that increases the penalty for a crime beyond the prescribed statutory maximum must be submitted to a jury and proved beyond a reasonable doubtrdquo because ldquo[i]t is unconstitutional for a legislature to remove from the jury the assessment of facts that increase the prescribed range of penalties to which a criminal defendant is exposed [because] such facts must be established by proof beyond a reasonable doubtrdquo Id The Court reaffirmed Apprendi in Blakely v Washington 542 US 296 (2004) holding that the ldquolsquostatutory maximumrsquo for Apprendi purposes is the maximum sentence a judge may impose solely on the basis of the facts reflected in the jury verdict or admitted by the defendantrdquo Id at 303 (emphasis in original) In United States v Booker 543 US 220 (2005)

25

Criminal Law and Procedure Analysis

the Court relied on Blakely and Apprendi to conclude that protecting a defendantrsquos Sixth Amendment right to a jury trial required that ldquo[a]ny fact which is necessary to support a sentence exceeding the maximum authorized by the facts established by a plea of guilty or a jury verdict must be admitted by the defendant or proved to a jury beyond a reasonable doubtrdquo Id at 244

Thus in order to constitutionally increase a sentence above the statutory maximum of three years the jury must have found beyond a reasonable doubt that the value of the ring exceeded $5000 Here the court made the finding based on an appraisal proffered by the prosecutor only at sentencing and the judgersquos finding was by a preponderance of the evidence rather than beyond a reasonable doubt

26

AGENCY AND PARTNERSHIP ANALYSIS __________ (Agency and Partnership VA amp C VI)

ANALYSIS

Legal Problems

(1) Is a partner in a general partnership personally liable on a claim arising from misrepresentations by another partner made in the course of the partnership business

(2) Does a newly admitted partner in a general partnership become personally liable on existing claims against the partnership

(3) After the filing by a general partnership of a statement of qualification as a limited liability partnership are the partners personally liable as partners on (a) an existing claim against the general partnership and (b) a claim against the partnership that arose after the filing

DISCUSSION

Summary

Adam and Ben formed a general partnership under which they were jointly and severally liable for obligations of the partnership Thus Adam was personally liable for misrepresentations by Ben made in the ordinary course of the partnership business

Upon joining the general partnership Diane became personally liable for the obligations of the partnership arising after her admission but not for obligations pre-existing her admission such as the collectorrsquos claim

By filing a statement of qualification the three partners properly elected limited liability partnership status As partners in an LLP none of the three partners is personally liable as a partner for partnership obligations arising after the election such as the claim by the driverrsquos estate The election however does not change their personal liability on pre-existing claims that arose before the election such as the collectorrsquos claim

Point One (30) As a general partner of Empire a general partnership Adam became personally liable on the collectorrsquos claim a valid claim against the partnership that arose because of Benrsquos wrongful act in the ordinary course of the partnership business

When the collectorrsquos claim arose Empire was a general partnership composed of Adam and Ben Under UPA (1997) sect 306(a) partners of a general partnership are liable jointly and severally for all obligations of the partnership Under UPA (1997) sect 305(a) the partnership could become obligated for the loss caused to the collector as a result of the misrepresentation by Ben provided he was acting in the ordinary course of the partnership business Because there was no statement that limited his partnership authority Ben as partner was ldquoan agent of the partnership for the purpose of its businessrdquo See UPA (1997) sect 301(1) Benrsquos misrepresentation to the collector even if intentional appears to be in the ordinary course of the partnershiprsquos business of dealing

27

Agency and Partnership Analysis

in antique cars Thus Benrsquos wrongful act created a partnership obligation for which Adam was jointly and severally liable

[NOTE Generally a partnership creditor must ldquoexhaust the partnershiprsquos assets before levying on a judgment debtor partnerrsquos individual property where the partner is personally liable for the partnership obligationrdquo as a result of his status as a partner UPA (1997) sect 307 cmt 4 As the UPA comments explain this places Adam more in the position of guarantor than principal debtor on the partnership obligation Id cmt 4 Although an examinee might discuss this point the call focuses on whether Adam is personally liable not how the liability might be enforced]

Point Two (30) Because the collectorrsquos claim arose before Diane joined Empire Diane did not become personally liable on the claim

Diane was admitted to Empire when it was a general partnership and after the collectorrsquos claim arose While the general rule under UPA (1997) sect 306(a) is that the partners of a general partnership are liable jointly and severally for all obligations of the partnership there is a special rule for partners who are admitted during the duration of the partnership Under UPA (1997) sect 306(b) a person admitted to an existing partnership is not personally liable for any partnership obligations incurred before the personrsquos admission Because Diane was admitted to Empire after the collectorrsquos claim arose Diane is not personally liable on the claim

Dianersquos knowledge of the pre-existing claim and her stated concern about becoming liable on the collectorrsquos claim do not change her personal nonliability to the collector Although partners who have a liability shield can assume liability to third parties through private contractual guarantees or modifications to the partnership agreement Dianersquos stated concern constituted neither a guaranty to the collector nor ldquoan intentional waiver of liability protectionsrdquo See UPA (1997) sect 306 cmt 3 (describing methods for waiver of liability protections under sect 306(c) applicable in limited liability partnerships)

At most Diane will lose her investment in the partnership as a result of the collectorrsquos claim Although Diane did not become personally liable on the collectorrsquos claim when she joined the partnership the $250000 she contributed to the partnership is ldquoat risk for the satisfaction of existing partnership debtsrdquo UPA (1997) sect 306 cmt 2

Point Three (40) Filing the statement of qualification was effective to elect limited liability partnership status Despite this new status Adam and Ben remain personally liable on the collectorrsquos claim which arose before the election But as partners in an LLP neither Adam Ben nor Diane is personally liable as a partner on the driverrsquos estatersquos claim which arose after the election

Under UPA (1997) sect 1001 a general partnership can make an election and become a limited liability partnershipmdashif the partners approve the conversion by a vote equivalent to that necessary to amend the partnership agreement and the partnership then files a statement of qualification that specifies the name of the partnership its principal office and its election to be an LLP Here the partners agreed unanimouslymdashsufficient to amend their agreement under UPA (1997) sect 401(j)mdashand the statement of qualification was filed In addition the name of Empire LLP properly included an appropriate ending ldquoLLPrdquo See UPA (1997) sect 1002

Although another way to effectuate a ldquoconversionrdquo (as suggested by Benrsquos lawyer) is to form a new LLP and transfer the assets of the old general partnership to the new LLP the

28

Agency and Partnership Analysis

method used here (approval by the partners and the filing of a statement of qualification) is also sufficient to create LLP status

Thus Empire became Empire LLP as of the date of filing of the statement of qualification See UPA (1997) sect 1001 What effect did this have on the collectorrsquos claim which predated the filing According to UPA (1997) sect 306(c) an obligation incurred while a partnership is an LLP is solely a partnership obligation As the collectorrsquos claim predated the LLP Adam and Ben remain personally liable on the collectorrsquos claim Diane on the other hand was not personally liable on the collectorrsquos claim either before or after the filing of the statement of qualification See Point Two above

The driverrsquos estatersquos claim arose after Empire became Empire LLP Under UPA (1997) sect 306(c) an obligation incurred while a partnership is an LLP is solely a partnership obligationThus Adam Ben and Diane as partners are all protected from personal liability on the driverrsquos estatersquos claim But there may be personal liability if any of them was negligent or otherwise acted wrongfully by not informing the buyer of the bad suspension that caused the accident

29

National Conference of Bar Examiners 302 South Bedford Street | Madison WI 53703-3622 Phone 608-280-8550 | Fax 608-280-8552 | TDD 608-661-1275

wwwncbexorg e-mail contactncbexorg

  • Preface
  • Description of the MEE
  • Instructions
  • February 2014 Questions
    • Constitutinal Law Question
    • Trusts and Future Interests Question
    • Secured Transactions Question
    • Federal Civil Procedure Question
    • Criminal Law and Procedure Question
    • Agency and Partnership Question
      • February 2014 Analyses
        • Constitutional Law Analysis
        • Trust and Future Interests Analysis
        • Secured Transactions Analysis
        • Federal Civil Procedure Analysis
        • Criminal Law and Procedure Analysis
        • Agency and Partnership Analysis
            • ltlt13 ASCII85EncodePages false13 AllowTransparency false13 AutoPositionEPSFiles true13 AutoRotatePages None13 Binding Left13 CalGrayProfile (Dot Gain 20)13 CalRGBProfile (sRGB IEC61966-21)13 CalCMYKProfile (US Web Coated 050SWOP051 v2)13 sRGBProfile (sRGB IEC61966-21)13 CannotEmbedFontPolicy Error13 CompatibilityLevel 1413 CompressObjects Tags13 CompressPages true13 ConvertImagesToIndexed true13 PassThroughJPEGImages true13 CreateJobTicket false13 DefaultRenderingIntent Default13 DetectBlends true13 DetectCurves 0000013 ColorConversionStrategy CMYK13 DoThumbnails false13 EmbedAllFonts true13 EmbedOpenType false13 ParseICCProfilesInComments true13 EmbedJobOptions true13 DSCReportingLevel 013 EmitDSCWarnings false13 EndPage -113 ImageMemory 104857613 LockDistillerParams false13 MaxSubsetPct 10013 Optimize true13 OPM 113 ParseDSCComments true13 ParseDSCCommentsForDocInfo true13 PreserveCopyPage true13 PreserveDICMYKValues true13 PreserveEPSInfo true13 PreserveFlatness true13 PreserveHalftoneInfo false13 PreserveOPIComments true13 PreserveOverprintSettings true13 StartPage 113 SubsetFonts true13 TransferFunctionInfo Apply13 UCRandBGInfo Preserve13 UsePrologue false13 ColorSettingsFile ()13 AlwaysEmbed [ true13 ]13 NeverEmbed [ true13 ]13 AntiAliasColorImages false13 CropColorImages true13 ColorImageMinResolution 30013 ColorImageMinResolutionPolicy OK13 DownsampleColorImages true13 ColorImageDownsampleType Bicubic13 ColorImageResolution 30013 ColorImageDepth -113 ColorImageMinDownsampleDepth 113 ColorImageDownsampleThreshold 15000013 EncodeColorImages true13 ColorImageFilter DCTEncode13 AutoFilterColorImages true13 ColorImageAutoFilterStrategy JPEG13 ColorACSImageDict ltlt13 QFactor 01513 HSamples [1 1 1 1] VSamples [1 1 1 1]13 gtgt13 ColorImageDict ltlt13 QFactor 01513 HSamples [1 1 1 1] VSamples [1 1 1 1]13 gtgt13 JPEG2000ColorACSImageDict ltlt13 TileWidth 25613 TileHeight 25613 Quality 3013 gtgt13 JPEG2000ColorImageDict ltlt13 TileWidth 25613 TileHeight 25613 Quality 3013 gtgt13 AntiAliasGrayImages false13 CropGrayImages true13 GrayImageMinResolution 30013 GrayImageMinResolutionPolicy OK13 DownsampleGrayImages true13 GrayImageDownsampleType Bicubic13 GrayImageResolution 30013 GrayImageDepth -113 GrayImageMinDownsampleDepth 213 GrayImageDownsampleThreshold 15000013 EncodeGrayImages true13 GrayImageFilter DCTEncode13 AutoFilterGrayImages true13 GrayImageAutoFilterStrategy JPEG13 GrayACSImageDict ltlt13 QFactor 01513 HSamples [1 1 1 1] VSamples [1 1 1 1]13 gtgt13 GrayImageDict ltlt13 QFactor 01513 HSamples [1 1 1 1] VSamples [1 1 1 1]13 gtgt13 JPEG2000GrayACSImageDict ltlt13 TileWidth 25613 TileHeight 25613 Quality 3013 gtgt13 JPEG2000GrayImageDict ltlt13 TileWidth 25613 TileHeight 25613 Quality 3013 gtgt13 AntiAliasMonoImages false13 CropMonoImages true13 MonoImageMinResolution 120013 MonoImageMinResolutionPolicy OK13 DownsampleMonoImages true13 MonoImageDownsampleType Bicubic13 MonoImageResolution 120013 MonoImageDepth -113 MonoImageDownsampleThreshold 15000013 EncodeMonoImages true13 MonoImageFilter CCITTFaxEncode13 MonoImageDict ltlt13 K -113 gtgt13 AllowPSXObjects false13 CheckCompliance [13 None13 ]13 PDFX1aCheck false13 PDFX3Check false13 PDFXCompliantPDFOnly false13 PDFXNoTrimBoxError true13 PDFXTrimBoxToMediaBoxOffset [13 00000013 00000013 00000013 00000013 ]13 PDFXSetBleedBoxToMediaBox true13 PDFXBleedBoxToTrimBoxOffset [13 00000013 00000013 00000013 00000013 ]13 PDFXOutputIntentProfile ()13 PDFXOutputConditionIdentifier ()13 PDFXOutputCondition ()13 PDFXRegistryName ()13 PDFXTrapped False1313 CreateJDFFile false13 Description ltlt13 ARA 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 BGR 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 CHS ltFEFF4f7f75288fd94e9b8bbe5b9a521b5efa7684002000410064006f006200650020005000440046002065876863900275284e8e9ad88d2891cf76845370524d53705237300260a853ef4ee54f7f75280020004100630072006f0062006100740020548c002000410064006f00620065002000520065006100640065007200200035002e003000204ee553ca66f49ad87248672c676562535f00521b5efa768400200050004400460020658768633002gt13 CHT ltFEFF4f7f752890194e9b8a2d7f6e5efa7acb7684002000410064006f006200650020005000440046002065874ef69069752865bc9ad854c18cea76845370524d5370523786557406300260a853ef4ee54f7f75280020004100630072006f0062006100740020548c002000410064006f00620065002000520065006100640065007200200035002e003000204ee553ca66f49ad87248672c4f86958b555f5df25efa7acb76840020005000440046002065874ef63002gt13 CZE 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 DAN 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 DEU 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 ESP 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 ETI 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 FRA 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 GRE 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 HEB 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 HRV (Za stvaranje Adobe PDF dokumenata najpogodnijih za visokokvalitetni ispis prije tiskanja koristite ove postavke Stvoreni PDF dokumenti mogu se otvoriti Acrobat i Adobe Reader 50 i kasnijim verzijama)13 HUN 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 ITA 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 JPN ltFEFF9ad854c18cea306a30d730ea30d730ec30b951fa529b7528002000410064006f0062006500200050004400460020658766f8306e4f5c6210306b4f7f75283057307e305930023053306e8a2d5b9a30674f5c62103055308c305f0020005000440046002030d530a130a430eb306f3001004100630072006f0062006100740020304a30883073002000410064006f00620065002000520065006100640065007200200035002e003000204ee5964d3067958b304f30533068304c3067304d307e305930023053306e8a2d5b9a306b306f30d530a930f330c8306e57cb30818fbc307f304c5fc59808306730593002gt13 KOR ltFEFFc7740020c124c815c7440020c0acc6a9d558c5ec0020ace0d488c9c80020c2dcd5d80020c778c1c4c5d00020ac00c7a50020c801d569d55c002000410064006f0062006500200050004400460020bb38c11cb97c0020c791c131d569b2c8b2e4002e0020c774b807ac8c0020c791c131b41c00200050004400460020bb38c11cb2940020004100630072006f0062006100740020bc0f002000410064006f00620065002000520065006100640065007200200035002e00300020c774c0c1c5d0c11c0020c5f40020c2180020c788c2b5b2c8b2e4002egt13 LTH ltFEFF004e006100750064006f006b0069007400650020016100690075006f007300200070006100720061006d006500740072007500730020006e006f0072011700640061006d00690020006b0075007200740069002000410064006f00620065002000500044004600200064006f006b0075006d0065006e007400750073002c0020006b00750072006900650020006c0061006200690061007500730069006100690020007000720069007400610069006b007900740069002000610075006b01610074006f00730020006b006f006b007900620117007300200070006100720065006e006700740069006e00690061006d00200073007000610075007300640069006e0069006d00750069002e0020002000530075006b0075007200740069002000500044004600200064006f006b0075006d0065006e007400610069002000670061006c006900200062016b007400690020006100740069006400610072006f006d00690020004100630072006f006200610074002000690072002000410064006f00620065002000520065006100640065007200200035002e0030002000610072002000760117006c00650073006e0117006d00690073002000760065007200730069006a006f006d00690073002egt13 LVI 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 NLD (Gebruik deze instellingen om Adobe PDF-documenten te maken die zijn geoptimaliseerd voor prepress-afdrukken van hoge kwaliteit De gemaakte PDF-documenten kunnen worden geopend met Acrobat en Adobe Reader 50 en hoger)13 NOR 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 POL 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 PTB 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 RUM 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 RUS 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 SKY ltFEFF0054006900650074006f0020006e006100730074006100760065006e0069006100200070006f0075017e0069007400650020006e00610020007600790074007600e100720061006e0069006500200064006f006b0075006d0065006e0074006f0076002000410064006f006200650020005000440046002c0020006b0074006f007200e90020007300610020006e0061006a006c0065007001610069006500200068006f0064006900610020006e00610020006b00760061006c00690074006e00fa00200074006c0061010d00200061002000700072006500700072006500730073002e00200056007900740076006f00720065006e00e900200064006f006b0075006d0065006e007400790020005000440046002000620075006400650020006d006f017e006e00e90020006f00740076006f00720069016500200076002000700072006f006700720061006d006f006300680020004100630072006f00620061007400200061002000410064006f00620065002000520065006100640065007200200035002e0030002000610020006e006f0076016100ed00630068002egt13 SLV 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 SUO 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 SVE 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 TUR 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 UKR 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 ENU (Use these settings to create Adobe PDF documents best suited for high-quality prepress printing Created PDF documents can be opened with Acrobat and Adobe Reader 50 and later)13 gtgt13 Namespace [13 (Adobe)13 (Common)13 (10)13 ]13 OtherNamespaces [13 ltlt13 AsReaderSpreads false13 CropImagesToFrames true13 ErrorControl WarnAndContinue13 FlattenerIgnoreSpreadOverrides false13 IncludeGuidesGrids false13 IncludeNonPrinting false13 IncludeSlug false13 Namespace [13 (Adobe)13 (InDesign)13 (40)13 ]13 OmitPlacedBitmaps false13 OmitPlacedEPS false13 OmitPlacedPDF false13 SimulateOverprint Legacy13 gtgt13 ltlt13 AddBleedMarks false13 AddColorBars false13 AddCropMarks false13 AddPageInfo false13 AddRegMarks false13 ConvertColors ConvertToCMYK13 DestinationProfileName ()13 DestinationProfileSelector DocumentCMYK13 Downsample16BitImages true13 FlattenerPreset ltlt13 PresetSelector MediumResolution13 gtgt13 FormElements false13 GenerateStructure false13 IncludeBookmarks false13 IncludeHyperlinks false13 IncludeInteractive false13 IncludeLayers false13 IncludeProfiles false13 MultimediaHandling UseObjectSettings13 Namespace [13 (Adobe)13 (CreativeSuite)13 (20)13 ]13 PDFXOutputIntentProfileSelector DocumentCMYK13 PreserveEditing true13 UntaggedCMYKHandling LeaveUntagged13 UntaggedRGBHandling UseDocumentProfile13 UseDocumentBleed false13 gtgt13 ]13gtgt setdistillerparams13ltlt13 HWResolution [2400 2400]13 PageSize [612000 792000]13gtgt setpagedevice13

Page 7: February 2014 MEE Questions and AnalysesPreface The Multistate Essay Examination (MEE) is developed by the National Conference of Bar Examiners (NCBE). This publication includes the

CONSTITUTIONAL LAW QUESTION ___________

A city ordinance required each downtown business to install high-powered halogen floodlights that would illuminate the property owned by that business and the adjoining sidewalks A study commissioned by the city estimated that installation of the floodlights would cost a typical business about $1000 but that increased business traffic due to enhanced public safety especially after dark would likely offset this cost

A downtown restaurant applied to the city for a building permit to construct an addition that would increase its seating capacity In its permit application the restaurant accurately noted that its current facility did not have sufficient seating to accommodate all potential customers during peak hours The city approved the permit on the condition that the restaurant grant the city an easement over a narrow strip of the restaurantrsquos property to be used by the city to install video surveillance equipment that would cover nearby public streets and parking lots The city based its permit decision entirely on findings that the increased patronage that would result from the increased capacity of the restaurant might also attract additional crime to the neighborhood and that installing video surveillance equipment might alleviate that problem

The restaurant has challenged both the ordinance requiring it to install floodlights and the easement condition imposed on approval of the building permit

1 Under the Fifth Amendment as applied to the states through the Fourteenth Amendment is the city ordinance requiring the restaurant to install floodlights an unconstitutional taking Explain

2 Under the Fifth Amendment as applied to the states through the Fourteenth Amendment is the cityrsquos requirement that the restaurant grant the city an easement as a condition for obtaining the building permit an unconstitutional taking Explain

3

TRUSTS AND FUTURE INTERESTS QUESTION _______________

Ten years ago a testator died survived by his only children a son age 26 and a daughter age 18

A testamentary trust was created under the testatorrsquos duly probated will The will specified that all trust income would be paid to the son during the sonrsquos lifetime and that upon the sonrsquos death the trust would terminate and trust principal would be distributed to the testatorrsquos ldquograndchildren who shall surviverdquo the son The testator provided for his daughter in other sections of the will

Five years ago the trustee of the testamentary trust purchased an office building with $500000 from the trust principal Other than this building the trust assets consist of publicly traded securities

Last year the trustee received $30000 in rents from the office building The trustee also received with respect to the securities owned by the trust cash dividends of $20000 and a stock dividend of 400 shares of Acme Corp common stock distributed to the trust by Acme Corp

Eight months ago the trustee sold the office building for $700000

Six months ago the son delivered a letter to the trustee stating ldquoI hereby disclaim any interest I may have in the income interest of the trustrdquo On the date the son delivered this letter to the trustee the son had no living children the daughter had one living minor child

A statute in this jurisdiction provides that ldquoa disclaimer of any interest created by will is valid only if made within nine months after the testatorrsquos death and if an interest is validly disclaimed the disclaiming party is deemed to have predeceased the testatorrdquo

1 How should the rents sales proceeds cash dividends and stock dividends received prior to the trusteersquos receipt of the sonrsquos letter have been allocated between trust principal and income Explain

2 How if at all does the sonrsquos letter to the trustee affect the future distribution of trust income and principal Explain

4

SECURED TRANSACTIONS QUESTION

On March 1 the owner of a manufacturing business entered into negotiations with a bank to obtain a loan of $100000 for the business The bank loan officer informed the business owner that the interest rate for a loan would be lower if the repayment obligation were secured by all the businessrsquos present and future equipment The loan officer also informed the business owner that the bank could not commit to making the loan until its credit investigation was completed but that funds could be advanced faster following loan approval if a financing statement with respect to the transaction were filed in advance Accordingly the business owner signed a form on behalf of the business authorizing the bank to file a financing statement with respect to the proposed transaction The bank properly filed a financing statement the next day correctly providing the name of the business as the debtor and indicating ldquoequipmentrdquo as the collateral

On March 15 the business owner had heard nothing from the bank about whether the loan had been approved so the business owner approached a finance company for a loan The finance company quickly agreed to lend $100000 to the business secured by all the businessrsquos present and future equipment That same day the finance company loaned to the business $100000 and the business owner signed an agreement obligating the business to repay the loan and granting the finance company a security interest in all the businessrsquos ldquopresent and future equipmentrdquo to secure the repayment obligation Also on that day the finance company properly filed a financing statement correctly providing the businessrsquos name as the debtor and indicating ldquoequipmentrdquo as the collateral

On March 21 the bank loan officer contacted the business owner and indicated that the loan application had been approved On the next day March 22 the bank loaned the business $100000 The loan agreement signed by the owner on behalf of the business granted the bank a security interest in all the businessrsquos ldquopresent and future equipmentrdquo

On April 10 the business sold an item of manufacturing equipment to a competitor for $20000 This was the first time the business had ever sold any of its equipment The competitor paid the purchase price in cash and took possession of the equipment that day The competitor acted in good faith at all times and had no knowledge of the businessrsquos prior transactions with the bank and the finance company

The business has defaulted on its obligations with respect to the loans from the bank and the finance company Each of them has asserted a claim to all the businessrsquos equipment as well as to the item of equipment sold to the businessrsquos competitor

Assume that the business owner had the authority to enter into all these transactions on behalf of the business

1 As between the bank and the finance company which has a superior claim to the businessrsquos equipment Explain

2 Do the claims of the bank and the finance company to the businessrsquos equipment continue in the item of equipment sold to the competitor Explain

5

FEDERAL CIVIL PROCEDURE QUESTION

A builder constructed a vacation house for an out-of-state customer on the customerrsquos land The house was completed on June 1 at which point the customer still owed $200000 of the $800000 contract price which was payable in full five days later

On June 14 the basement of the house was flooded with two inches of water during a heavy rainfall When the customer complained the builder told the customer ldquoThe flooding was caused by poorly designed landscaping Our work is fine and fully up to code Have an engineer look at the foundation If therersquos a problem wersquoll fix itrdquo

The customer pleased by the builderrsquos cooperative attitude immediately hired a structural engineer to examine the foundation of the house On June 30 the engineer provided the customer with a written report on the condition of the foundation which stated that the foundation was properly constructed

Unhappy with the conclusions in the engineerrsquos report the customer then hired a home inspector to evaluate the house The home inspectorrsquos report concluded that the foundation of the house had been poorly constructed and was inadequately waterproofed

On July 10 the customer sent the builder the home inspectorrsquos report with a note that said ldquoUntil you fix this problem you wonrsquot get another penny from merdquo The builder immediately contacted an attorney and directed the attorney to prepare a draft complaint against the customer for nonpayment Hoping to avoid litigation the builder sent several more requests for payment to the customer The customer ignored all these requests

On September 10 the builder filed suit in federal district court properly invoking the courtrsquos diversity jurisdiction and seeking $200000 in damages for breach of contract The customerrsquos answer denied liability on the basis of alleged defective construction of the housersquos foundation

Several months later the case is nearly ready for trial However two discovery disputes have not yet been resolved

First despite a request from the builder the customer has refused to provide a copy of the report prepared by the structural engineer who examined the foundation of the house The customer claims that the report is ldquowork productrdquo and not discoverable because the customer does not intend to ask the engineer to testify at trial The builder has asked the court to order the customer to turn over the engineerrsquos report

Second the customer has asked the court to impose sanctions for the builderrsquos failure to comply with the customerrsquos demand for copies of all emails concerning construction of the foundation of the house The builder has truthfully informed the customer that all such emails were destroyed on August 2 This destruction was pursuant to the builderrsquos standard practice of permanently deleting all project-related emails from company records 60 days after construction of a project is complete There is no relevant state records-retention law

1 Should the court order the customer to turn over the engineerrsquos report Explain

2 Should the court sanction the builder for the destruction of emails related to the case and if so what factors should the court consider in determining those sanctions Explain

6

CRIMINAL LAW AND PROCEDURE QUESTION _____

A defendant was charged under state law with felony theft (Class D) and felony residential burglary (Class C) The indictment alleged that the defendant entered his neighborsrsquo home without their consent and stole a diamond ring worth at least $2500

Defense counsel filed a pretrial motion to dismiss the charges on the ground that prosecuting the defendant for both burglary and theft would constitute double jeopardy The trial court denied the motion and the defendant was prosecuted for both crimes The only evidence of the ringrsquos value offered at the defendantrsquos jury trial was the ownerrsquos testimony that she had purchased the ring two years earlier for $3000

At trial the judge issued the following jury instruction on the burglary charge prior to deliberations

If after consideration of all the evidence presented by the prosecution and defense you find beyond a reasonable doubt that the defendant entered the dwelling without the ownersrsquo consent you may presume that the defendant entered with the intent to commit a felony therein

The jury found the defendant guilty of both offenses

At the defendantrsquos sentencing hearing an expert witness called by the prosecutor testified that the diamond ring was worth between $7000 and $8000 Over defense objection the judge concluded by a preponderance of the evidence that the value of the stolen ring exceeded $5000 The judge sentenced the defendant to four yearsrsquo incarceration on the theft conviction On the burglary conviction the defendant received a consecutive sentence of seven yearsrsquo incarceration

In this state residential burglary is defined as ldquoentry into the dwelling of another without the consent of the lawful resident with the intent to commit a felony thereinrdquo Residential burglary is a Class C felony for which the minimum sentence is five years and the maximum sentence is ten years of incarceration

In this state theft is defined as ldquotaking and carrying away the property of another with the intent to permanently deprive the owner of possessionrdquo Theft is a Class D felony if the value of the item(s) taken is between $2500 and $10000 The sentence for a Class D felony theft is determined by the value of the items taken If the value is between $2500 and $5000 the maximum sentence is three yearsrsquo incarceration If the value of the items exceeds $5000 the maximum sentence is five yearsrsquo incarceration

This state affords a criminal defendant no greater rights than those mandated by the United States Constitution

1 Did the trial court err when it denied the defendantrsquos pretrial motion to dismiss on double jeopardy grounds Explain

2 Did the trial court err in its instruction to the jury on the burglary charge Explain

3 Did the trial court err when it sentenced the defendant to an additional year of incarceration on the theft conviction based on the expertrsquos testimony Explain

7

AGENCY AND PARTNERSHIP QUESTION _____

Five years ago Adam and Ben formed a general partnership Empire Partnership (Empire) to buy and sell antique automobiles at a showroom in State A Adam contributed $800000 to Empire and Ben contributed $200000 Their written partnership agreement allocated 80 of profits losses and control to Adam and 20 to Ben No filings of any type were made in connection with the formation of Empire

Three years ago a collector purchased one of Empirersquos antique cars for $3400000 The collector was willing to pay this price because of Benrsquos false representation (repeated in the sales contract) that a famous movie star had once owned the car Without the movie-star connection the car was worth only $100000 One month later when the collector discovered the truth he sued Adam Ben and Empire for $3300000 in damages The lawsuit is still pending

Two years ago Adam and Ben admitted a new partner Diane to Empire in return for her contribution of $250000 The three agreed to allocate profits losses and control 75 to Adam 10 to Ben and 15 to Diane Before joining the partnership Diane learned of the collectorrsquos claim and stated her concern to Adam and Ben that she might become liable if the claim were reduced to a judgment

Following Dianersquos admission to Empire the three partners sought to convert Empire into a limited liability partnership (LLP) Adamrsquos lawyer proposed to file with State A a ldquostatement of qualificationrdquo making an LLP election and declaring the name of the partnership to be ldquoEmpire LLPrdquo Benrsquos lawyer stated that this would not work and that a new LLP had to be formed with the assets of the old partnership transferred to the new one In the end the conversion was done the way Adamrsquos lawyer suggested with the approval of all three partners

One year ago a driver purchased a vintage car from Empire LLP based on the representation that the car was ldquofully roadworthy and capable of touring at 70 mph all dayrdquo The driver took the car on the highway at 50 mph whereupon the front suspension collapsed resulting in a crash in which the car was destroyed and the driver killed The driverrsquos estate sued Adam Ben Diane and Empire LLP for $10000000 The lawsuit is still pending

Although profitable Empire LLP does not have resources sufficient to pay the collectorrsquos claim or the claim of the driverrsquos estate

Assume that the Uniform Partnership Act (1997) applies

1 Before the filing of the statement of qualification (a) was Adam personally liable on the collectorrsquos claim Explain (b) was Diane personally liable on the collectorrsquos claim Explain

2 After the filing of the statement of qualification was Adam Ben or Diane personally liable as a partner on (a) the collectorrsquos claim or (b) the driverrsquos estatersquos claim Explain

8

February 2014 MEE

ANALYSES Constitutional Law

Trusts and Future Interests Secured Transactions

Federal Civil Procedure Criminal Law and Procedure

Agency and Partnership

CONSTITUTIONAL LAW ANALYSIS (Constitutional Law IVD)

ANALYSIS

Legal Problems

(1) Is the city ordinance requirement that businesses install floodlights a taking

(2) Is conditioning the approval of a building permit on the grant of an easement to install surveillance equipment a taking of property

DISCUSSION

Summary

The ordinance requiring businesses to install floodlights is not a per se taking under Loretto because it does not force a private landowner to allow a third party to enter and place a physical object on the land Here the city ordinance requires the businessmdashnot a third partymdashto install the floodlights

The ordinance is likely not a regulatory taking under the Penn Central balancing test While the ordinance will impose a cost on business owners that cost may be offset by the expected increase in business due to the ordinance and the ordinance does not appear to interfere with the ownerrsquos primary use of the property as a restaurant

The permit condition however is likely an uncompensated taking of property While the condition has an essential nexus with the cityrsquos legitimate interest in promoting public safety the city has not made an individualized determination that the easement condition is roughly proportional to the possibility of increased crime due to the restaurantrsquos proposed addition Thus the permit condition likely violates the Fifth Amendment as applied to the states through the Fourteenth Amendment

Point One (50) The ordinance requiring that businesses install floodlights is not a per se taking under Loretto It is not a regulatory taking under the Penn Central balancing test because the cost of compliance with the ordinance may be offset by an expected increase in business and compliance does not interfere with the businessrsquos primary use of its property as a restaurant

The city ordinance requiring a business to install floodlights does not effect a per se taking of the sort described in Loretto v Teleprompter Manhattan CATV Corp 458 US 419 (1982) because no property is physically taken by the government and the ordinance does not involve a physical invasion of private property by a third party

Even though the ordinance does not constitute an occupation of the property by either the government or a third party it is still subject to the three-factor balancing test under Penn Central Transportation Co v City of New York 438 US 104 (1978) to determine whether it is a ldquoregulatory takingrdquo Under Penn Central a court must balance (1) ldquo[t]he economic impact of the regulation on the claimantrdquo (2) ldquothe extent to which the regulation has interfered with distinct investment-backed expectationsrdquo and (3) ldquothe character of the governmental actionrdquo Id at 124 Here each factor weighs against finding that the ordinance is a taking

11

Constitutional Law Analysis

First the ordinance requirement likely has a minimal economic impact on the restaurant Compliance with the ordinance is estimated to cost $1000 and the city has found that businesses will likely recoup that cost in increased sales Also because the ordinance does not interfere with the operation of the restaurant the owner may still earn a reasonable return on its investment in the property

Second the ordinance does not interfere with the businessrsquos investment-backed expectations As in Penn Central the challenged law does not interfere with the ownerrsquos ldquoprimary expectationrdquo for use of the propertymdashin Penn Central as a railroad terminal and here as a restaurant Further the ordinance does not prevent the restaurant from expanding to meet the changing business environment

Third the character of the government action does not weigh in favor of a taking While Penn Central does say that a ldquophysical invasionrdquo is more likely to pose a taking Loretto suggests that the Courtrsquos main concern is with physical invasions by third parties Also like the landmark law challenged in Penn Central the ordinance here ldquoadjust[s] the benefits and burdens of economic life to promote the common goodrdquo Id In Penn Central the landmark law restricted development of the railroad terminal to promote the common interest in preserving historic landmarks Here the ordinance requires the businesses to install floodlights to promote the common interest in crime prevention and public safety

Because the ordinance is clearly a valid exercise of the police power it satisfies the takings clausersquos public-use requirement Kelo v City of New London 545 US 469 (2005)

In sum all three factors weigh against finding a taking under the Penn Central balancing test

Point Two (50) The permit condition may be unconstitutional as an uncompensated taking of property because the city has not made an individualized determination that the easement condition is roughly proportional to the impact of the restaurantrsquos proposed addition

In Dolan v City of Tigard 512 US 374 (1994) the Supreme Court set forth the test for determining whether an exaction imposed by a government in exchange for a discretionary benefit conferred by the government such as a condition on the approval of a building permit in this case constitutes an uncompensated taking under the Fifth Amendment The exaction is not a taking if (1) there is an ldquoessential nexusrdquo between the ldquopublic need or burdenrdquo to which the proposed development contributes and ldquothe permit condition exacted by the cityrdquo id at 386 and (2) the government makes ldquosome sort of individualized determination that the required dedication is [roughly proportional] both in nature and extent to the impact of the proposed developmentrdquo Id at 391 see also Nollan v California Coastal Commission 483 US 825 (1987)

Here the city likely can meet the nexus requirement In Dolan the landowner sought to double the size of its business which would have increased traffic on nearby roadways In exchange for approving the development the city sought an easement for a bike and pedestrian path The Court found the required nexus between the easement and the cityrsquos ldquoattempt to reduce traffic congestion by providing for alternative means of transportationrdquo 512 US at 387 Here a similar nexus likely exists between the requested easement and the cityrsquos interest in crime prevention and public safety Increased patronage and economic activity at the restaurant might attract additional crime to the area and the requested easement to install surveillance equipment would attempt to address that increased crime

12

Constitutional Law Analysis

The exaction here however may fail the second prong of the Dolan testmdashthat the exaction be roughly proportional to the anticipated impact of the requested development As noted the city in Dolan claimed that a bike and pedestrian path was needed to offset the increase in traffic due to the proposed doubling of the business The Court explained that the government must demonstrate that the additional traffic reasonably was related to the requested exaction and that the government must ldquomake some effort to quantify its findings in support of the dedication for the pedestrianbicycle pathway beyond the conclusory statement that it could offset some of the traffic demand generatedrdquo Id at 395 Here the city did not carry its burden The city simply speculates that increased patronage of the restaurant ldquomightrdquo increase crime and that the surveillance equipment ldquomightrdquo alleviate this increased crime Because the city has not made ldquosome effort to quantify its findingsrdquo in support of the easement it has not shown that the burden of the easement is roughly proportional to the benefits thought to flow from it

Thus the exaction appears to be an uncompensated taking of property in violation of the Fifth Amendment as applied to the states through the Fourteenth Amendment

13

TRUSTS AND FUTURE INTERESTS ANALYSIS ____ (Trusts and Future Interests IE3 I5 IIIA amp B)

ANALYSIS

Legal Problems

(1) How should rents dividends and sales proceeds received by the trustee prior to receipt of the sonrsquos letter have been allocated between trust income and principal

(2)(a) Did the remainder interest in the trust accelerate and become immediately payable to the daughterrsquos minor child upon the trusteersquos receipt of the sonrsquos letter and if not how should the trustee handle the distribution of the principal in the future

(2)(b) Following the trusteersquos receipt of the sonrsquos letter how should the trustee distribute future receipts of income prior to the distribution of the principal

DISCUSSION

Summary

Prior to the trusteersquos receipt of the sonrsquos letter cash dividends and rents should have been allocated to trust income and were distributable to the son the income beneficiary of the trust sales proceeds and stock dividends should have been allocated to principal

Because the sonrsquos letter to the trustee did not result in a valid disclaimer under state law (having been made more than nine months after the testatorrsquos death) the son is not deemed to have predeceased the testator Because the son is still living the class gift to the testatorrsquos grandchildren who survive the son has not closed and is not possessory it will not become possessory until the son dies The daughterrsquos minor child being the testatorrsquos only living grandchild is not currently entitled to a distribution of trust principal Trust principal will instead be distributable upon the sonrsquos death to the testatorrsquos then-living grandchildren or if there are none to the testatorrsquos then-living heirs

As for future income the trustee should either distribute the trust income to the son and the daughter as the testatorrsquos heirs accumulate the income for future distribution to those individuals ultimately entitled to the trust principal or distribute it to those presumptively entitled to the principal upon the sonrsquos death ie the daughterrsquos minor child

Point One (45) Cash dividends and rents are allocable to income sales proceeds and stock dividends are allocable to principal Items allocable to income for the period prior to the sonrsquos attempted disclaimer were distributable to the son

Receipts earned during the administration of a trust are allocable either to income or to principal Almost all states have adopted the most recent or an earlier version of the Uniform Principal and Income Act (the Act) which specifies how such receipts should be allocated

Under the Act rents (UNIF PRIN amp INC ACT (2000) sect 405 UNIF PRIN amp INC ACT (1962) sect 3(a)(1)) and cash dividends received from a corporation (UNIF PRIN amp INC ACT (2000) sect 401(b) UNIF PRIN amp INC ACT (1962) sect 6(d)) are allocable to income and are distributable to the income beneficiary of the trust

14

Trusts and Future Interests Analysis Sales proceeds (UNIF PRIN amp INC ACT (2000) sect 404(2) UNIF PRIN amp INC ACT (1962)

sect 3(b)(1)) and dividends paid in the stock of the distributing corporation (UNIF PRIN amp INC ACT (2000) sect 401(c)(1) UNIF PRIN amp INC ACT (1962) sect 3(b)(4)) are allocable to principal and added to the principal of the trust

Here the cash dividends and office building rents should have been allocated to income and until the trustee received the sonrsquos letter should have been distributed to him as the sole income beneficiary of the trust The stock dividend and proceeds from the sale of the office building should have been allocated to principal and held by the trustee for future distribution to the ultimate remaindermen of the trust

[NOTE The 2000 Uniform Principal and Income Act has been adopted in Alabama Arkansas Colorado Connecticut the District of Columbia Hawaii Idaho Iowa Kentucky Missouri Montana Nebraska New Mexico North Dakota Oregon South Dakota Utah and West Virginia]

Point Two(a) (45) Because the son did not disclaim within nine months of the testatorrsquos death there is no valid disclaimer under state law Therefore the son is not deemed to have predeceased the testator Furthermore because of the express survivorship contingency in the will the remainder in the trust does not accelerate and become distributable until the son in fact dies When the son dies the trust principal will be distributable to the testatorrsquos then-living grandchildren or if none then to the testatorrsquos then-living heirs

When a trust remainder is given to a class the class closes (ie no new persons can join the class) when there is no outstanding income interest and at least one member of the class is then entitled to demand possession of his or her share of the remainder This principle is called the rule of convenience See generally HERBERT HOVENKAMP amp SHELDON F KURTZ PRINCIPLES OF PROPERTY LAW 199ndash200 (6th ed 2005) A class member may demand possession of his or her share of the remainder upon termination of the income interest only when the class memberrsquos interest is not otherwise subject to a condition precedent See id

When a beneficiary timely disclaims an interest in a trust that beneficiary is treated as if he had predeceased the testator Here had the son disclaimed within nine months of the testatorrsquos death as required by the state statute he would have been deemed to have predeceased the testator This would have closed the class of remaindermen and the testatorrsquos then-living grandchildren (ie the daughterrsquos child) would have been entitled to the trust principal However under the state statute the sonrsquos disclaimer was not timely because he did not disclaim within nine months of the testatorrsquos death Thus because the statute is inapplicable and the son is still alive the class of grandchildren entitled to share in trust principal did not close

Because here the statute is inapplicable due to the sonrsquos failure to comply with the statutory time requirements then presumably the common-law rule allowing disclaimers (aka renunciations) at any time should apply Under the common law if a life estate is renounced the remainder interest accelerates and becomes immediately distributable to the remaindermen of the trust if the remainder is vested but not if the remainder is contingent JESSE DUKEMINIER amp ROBERT H SITKOFF WILLS TRUSTS AND ESTATES 844ndash845 (9th ed 2013) Here because the remainder is contingent upon there being grandchildren who survive the son the remainder will not accelerate It will remain open until the son dies leaving open the possibility that additional grandchildren will be included in the class or the daughterrsquos child could fall out of the class because that child fails to survive the son

And if none of the testatorrsquos grandchildren survive the son the trust principal will be distributed to the testatorrsquos heirs living at the sonrsquos death

15

Trusts and Future Interests Analysis

Point Two(b) (10) Until the trust terminates the trustee must continue to hold the trust assets The distribution of income in the meantime is unclear There are at least three possibilities Income earned on the undistributed assets could be distributed to the son and daughter as the testatorrsquos heirs accumulated and added to principal for distribution to the ultimate remaindermen or distributed from time to time to those persons who are presumptively remaindermen

When trust principal is not immediately distributable the trustee must continue to hold trust assets until the ultimate remaindermen are ascertained During this period trust income will be distributed or retained according to any instructions contained in the trust instrument See WILLIAM M MCGOVERN JR SHELDON F KURTZ amp DAVID M ENGLISH WILLS TRUSTS amp ESTATES sect 102 (4th ed 2010)

Here the testator did not specify what the trustee should do with trust income in the event the sonrsquos disclaimer did not comply with the state statute There are at least three approaches One approach would have the trustee distribute the trust income to the testatorrsquos heirs on the theory that the income represents property that was not disposed of by the testatorrsquos will and which thus passes by partial intestacy to the testatorrsquos heirs A second approach would have the trustee accumulate trust income for distribution to the ultimate remaindermen Under this approach only those individuals ultimately entitled to the principal would share in the income A third approach would have the trustee distribute trust income to those individuals who would be the remaindermen if the trust were to terminate when the income is received by the trustee under this approach trust income would be distributed to the daughterrsquos minor child until another presumptive remainderman is born This approach could result in individuals not ultimately entitled to principal say because they do not survive the son receiving income It could also result in a disproportionate distribution of income among the individuals ultimately entitled to income

[NOTE Examinees should demonstrate a recognition and understanding of the income-allocation problem and the alternatives available to address that issue There is no widely accepted solution to the problem Examinees who cite any of these possible problem-solving approaches may receive credit]

16

SECURED TRANSACTIONS ANALYSIS (Secured Transactions IB IID E amp F IIIB IVA B amp F)

ANALYSIS

Legal Problems

(1)(a) What is the nature of the bankrsquos claim to the businessrsquos equipment

(1)(b) What is the nature of the finance companyrsquos claim to the businessrsquos equipment

(1)(c) As between the bank and the finance company whose claim to the businessrsquos equipment has priority

(2) Do the claims of the bank and the finance company continue in the item of equipment sold by the business to the competitor

DISCUSSION

Summary

The bank and the finance company both have perfected security interests in the businessrsquos equipment Even though the finance companyrsquos perfected security interest was created first the bankrsquos perfected security interest has priority because the bankrsquos financing statement was filed before the finance companyrsquos financing statement The security interests of the bank and the finance company continue in the item of equipment sold by the business to the competitor because their security interests were perfected and the competitor was not a buyer in ordinary course of business

Point One(a) (25) The bank has a perfected security interest in the businessrsquos equipment

The bank has met all criteria necessary for it to have an attached and enforceable security interest in the businessrsquos equipment First value must be given UCC sect 9-203(b)(1) This criterion is fulfilled by the loan made by the bank to the business Second the debtor must have rights in the collateral UCC sect 9-203(b)(2) Clearly the business has rights in its equipment Third either the secured party must take possession of the collateral or the debtor must authenticate a security agreement containing a description of the collateral UCC sect 9-203(b)(3) The agreement that the business owner signed is a ldquosecurity agreementrdquo because it is an agreement that creates or provides for a security interest UCC sect 9-102(a)(74) By signing the security agreement the business owner authenticated it UCC sect 9-102(a)(7) Therefore all three criteria are fulfilled and the bank has an enforceable and attached security interest

A security interest is perfected when it has attached and when any additional steps required for perfection have occurred UCC sect 9-308(a) Generally speaking the additional steps will either be possession of the collateral by the secured party or the filing of a financing statement with respect to the collateral See UCC sectsect 9-310 9-313 In this case the bank filed a financing statement naming the debtor and sufficiently indicating the collateral The collateral indication is sufficient because it identifies the collateral by type of property See UCC sectsect 9-504 9-108 The fact that the financing statement was filed before the security interest was created is

17

Secured Transactions Analysis

not a problem Even though the security agreement had not yet been signed the business had authorized the filing of the financing statement in an authenticated record UCC sect 9-509(a)(1) Moreover the financing statement may be filed before the security agreement is created UCC sect 9-502(d)

Point One(b) (10) The finance company also has a perfected security interest in the businessrsquos equipment

The finance companyrsquos security interest is enforceable and attached for the same reasons as the bankrsquos security interest The loan from the finance company to the business constitutes value the business has rights in the collateral and the business owner has authenticated a security agreement containing a description of the collateral The finance companyrsquos security interest is perfected because the finance company filed a financing statement with respect to it that provides that the business is the debtor and indicates that the collateral is equipment

Point One(c) (30) The bankrsquos security interest has priority over the finance companyrsquos security interest because the bankrsquos financing statement was filed first

As between two perfected security interests the general rule is that the security interest that was the earlier to be either perfected or the subject of a filed financing statement has priority UCC sect 9-322(a)(1) While the finance companyrsquos security interest was perfected before the bankrsquos (March 15 vs March 22) the bankrsquos financing statement was filed even earlier on March 2 Thus under the first-to-file-or-perfect rule of UCC sect 9-322(a)(1) the bankrsquos security interest has priority No exceptions to the general rule apply here

Point Two (35) A security interest in collateral continues notwithstanding its sale unless an exception applies Because the security interests of the bank and the finance company were perfected and the competitor was not a buyer in ordinary course of business no exception applies and the security interests of both creditors continue in the equipment sold to the competitor

As a general rule a security interest in collateral continues notwithstanding the fact that the debtor has sold the collateral to another person UCC sect 9-315(a)(1) Thus unless an exception applies the security interests of the bank and the finance company will continue in the item of equipment sold to the competitor

A buyer of goods will take free of an unperfected security interest in those goods See UCC sect 9-317(a)(2) However when the competitor bought the businessrsquos equipment both the bank and the finance company had perfected security interests in the equipment

A buyer can take free even of a perfected security interest in goods if the buyer is a ldquobuyer in ordinary course of businessrdquo See UCC sect 9-320(a) However the competitor was not a buyer in ordinary course of business To be a ldquobuyer in ordinary course of businessrdquo a buyer must buy goods from a seller that is in the business of selling goods of that kind See UCC sect 1-201(b)(9) The competitor bought this equipment from a seller that is not in the business of selling goods of this kind so the competitor was not a buyer in ordinary course of business with respect to these goods

Because no exception applies the security interests of the bank and the finance company continue even after the item of equipment was sold to the competitor

18

FEDERAL CIVIL PROCEDURE ANALYSIS (Federal Civil Procedure IVD)

ANALYSIS

Legal Problems

(1) Is a document prepared in the course of a contract dispute protected from discovery as ldquowork productrdquo when there is no evidence that the document was prepared in anticipation of litigation

(2)(a) Is a partyrsquos failure to provide relevant electronically stored information excused when the information was destroyed pursuant to a routine document retention scheme at a time when litigation was contemplated by the destroying party

(2)(b) What sanctions should be imposed on a party for allowing the destruction of evidence that is relevant to potential future litigation

DISCUSSION

Summary

The report prepared by the structural engineer is probably not work product and is thus discoverable The engineer examined the foundation of the house at the customerrsquos request and the engineerrsquos findings are potentially relevant to the customerrsquos claim that the foundation is defective The report was not prepared in anticipation of litigation The customer appears to have sought the engineerrsquos opinion in response to the builderrsquos offer to fix any problems with the foundation that an engineer might identify Because the report was not prepared in anticipation of litigation it is not protected by the work-product doctrine

The builder should have taken appropriate steps to preserve evidence including suspending its document retention program as soon as it began planning for litigationmdashie on July 10 Its destruction of potentially relevant material after that date was wrongful However a court is unlikely to impose severe sanctions on the builder because there are no facts indicating that the builder acted in bad faith and the customer can prove that the foundation is defective without the destroyed emails

Point One (40) The customer must turn over the engineerrsquos report because it was not prepared in anticipation of litigation

In general a party to a lawsuit in federal court ldquomay obtain discovery regarding any nonprivileged matter that is relevant to any partyrsquos claim or defenserdquo FED R CIV P 26(b)(1) (2009) This includes the right to inspect and copy documents in the other partyrsquos possession FED R CIV P 34(a)(1) Here the customer hired a structural engineer to examine the foundation of the house The engineerrsquos report on the foundation is likely to include information that would be relevant to the customerrsquos claim that the foundation was defectively constructed

The so-called ldquowork productrdquo rule allows a party to refuse to turn over ldquodocuments that are prepared in anticipation of litigation or for trialrdquo by that partyrsquos representative including

19

Federal Civil Procedure Analysis

a consultant Thus if the customer had hired the structural engineer to prepare a report ldquoin anticipation of litigationrdquo that report might not be discoverable See FED R CIV P 26(b)(3)

In this case however the customer hired the engineer to evaluate the foundation of the house as part of the customerrsquos negotiation with the builder concerning the housersquos flooding problem The builder told the customer that the housersquos landscaping was the reason for the flooding and the builder told the customer ldquoHave an engineer look at the foundation If therersquos a problem wersquoll fix itrdquo The customer appears to have acted in response to that statement There is no indication that the customer anticipated any kind of legal action at the time that the structural engineer was hired Accordingly the structural engineerrsquos report is discoverable and the court should order the customer to turn it over

[NOTE If an examinee concludes that the structural engineerrsquos report was prepared in anticipation of litigation then the examinee should also conclude that the report is not discoverable Documents prepared in anticipation of litigation do not need to be disclosed to an adverse party unless that party can demonstrate a ldquosubstantial needrdquo for the documents and an inability to obtain substantially equivalent information without ldquoundue hardshiprdquo FED R CIV P 26(b)(3)(A)(ii) Furthermore a report prepared by an expert who is not expected to testify is not discoverable in the absence of ldquoexceptional circumstancesrdquo making it ldquoimpracticablerdquo to obtain the information in another way FED R CIV P 26(b)(4)(D)(ii) The builder probably cannot make these showings here unless the engineerrsquos report deals with circumstances that have since changed There is no evidence that the structural engineer would have had access to any information or facts that the builder would not already know as a result of its construction and subsequent inspection of the house In addition if necessary the builder could ask the court for permission to arrange for a further inspection of the house by a structural engineer hired by the builder See FED R CIV P 34(a)(2) Accordingly if an examinee concludes that the report was prepared in anticipation of litigation the examinee should also conclude that the builder is not entitled to see the report]

Point Two(a) (30) Because the builder anticipated that it might be involved in litigation concerning its contract with the customer the builder acted wrongfully in destroying emails that were relevant to the housersquos construction even though the emails were destroyed pursuant to a routine document retention plan

As noted above a party to a lawsuit in federal court ldquomay obtain discovery regarding any nonprivileged matter that is relevant to any partyrsquos claim or defenserdquo FED R CIV P 26(b)(1) This includes emails and other electronically stored information FED R CIV P 34(a)(1)(A) Here the customer has requested all the builderrsquos emails pertaining to work done on the foundation of the house Ordinarily the builder would be obliged to turn over this information which is relevant to the customerrsquos defense that the housersquos foundation was poorly constructed

Unfortunately the emails in question no longer exist because the builder destroyed them on August 2

In general spoliation of evidence (destruction or alteration of evidence) is improper if the party who destroyed or altered the evidence ldquohas notice that the evidence is relevant to litigation or should have known that the evidence may be relevant to future litigationrdquo Fujitsu Ltd v Federal Express Corp 247 F3d 423 436 (2d Cir 2001) It is improper for a party to destroy electronic information relevant to pending litigation even if the destruction occurs before there is any request or order seeking the information See eg Leon v IDX Sys Corp 464 F3d 951 (9th Cir 2006) (plaintiffrsquos intentional destruction of computer files warranted dismissal even

20

In this case the builderrsquos destruction of the emails was pursuant to a routine document retention plan The Federal Rules provide expressly that in the absence of ldquoexceptional circumstancesrdquo parties should not be sanctioned for the loss of electronically stored information when the loss occurs pursuant to ldquoroutine good-faith operation of an electronic information systemrdquo FED R CIV P 37(e) However when a party anticipates litigation ldquoit must suspend its routine document retentiondestruction policy and put in place a lsquolitigation holdrsquo to ensure the preservation of relevant documentsrdquo Zubulake v UBS Warburg LLC 220 FRD 212 218 (SDNY 2003)

Federal Civil Procedure Analysis

though spoliation occurred before order compelling discovery) Similarly the duty to preserve evidence applies to a party who anticipates litigation even if litigation has not yet been commenced See THE SEDONA PRINCIPLES BEST PRACTICES RECOMMENDATIONS amp PRINCIPLES FOR ADDRESSING ELECTRONIC DOCUMENT PRODUCTION 70 cmt 14a (2d ed 2007)

The builder destroyed the emails on August 2 At that time the builder knew that litigation was a possibility because the builder had already directed its attorney to prepare a draft complaint for possible filing Knowing that litigation was a possibility the builder had a duty to take steps to preserve evidence including the emails in question See generally Fujitsu Ltd

Thus the builderrsquos destruction of potentially relevant emails at a time when it knew that litigation was a possibility was improper It had a duty to preserve evidence and it breached that duty

[NOTE Because courts have used different words to describe the test for when evidence must be preserved an examineersquos precise formulation of the test is not critical]

Point Two(b) (30) In determining appropriate sanctions for spoliation courts consider both the level of culpability of the spoliating party and the degree of prejudice the loss of evidence has caused the other party Here the builderrsquos destruction of evidence does not appear to have been willful nor is it likely to pose a significant obstacle to the customerrsquos defense Any sanctions imposed by the court should be modest

Federal courts have inherent power to control the litigation process and can sanction misbehavior including spoliation even when there has been no specific violation of the Federal Rules of Civil Procedure See generally Chambers v NASCO Inc 501 US 32 (1991) (discussing courtrsquos inherent power to control the litigation process) The range of available sanctions is broad It can include such sanctions as the payment of expenses incurred by the other party as a result of the destruction of the evidence an instruction to the jury authorizing it to draw an adverse inference from the destruction of the evidence a shifting of the burden of proof on the relevant issue or even judgment against the responsible party See eg Residential Funding Corp v DeGeorge Financial Corp 306 F3d 99 108 (2d Cir 2002) (adverse inference) Silvestri v General Motors Corp 271 F3d 583 593 (4th Cir 2001) (possibility of dismissal) Cf FED R CIV P 37(b)(2)(A) (listing remedies for failure to comply with discovery obligations)

In determining appropriate sanctions for spoliation courts consider both the level of culpability of the spoliating party and the degree of prejudice the loss of evidence has caused the other party Many courts impose severe sanctions (such as an adverse-inference instruction or the entry of judgment against the spoliating party) only when there is evidence of bad faith in the form of an intentional effort to hide information Eg Greyhound Lines Inc v Wade 485 F3d 1032 1035 (8th Cir 2007) (spoliation sanction requires intentional destruction out of desire ldquoto suppress the truthrdquo) However other courts have said that negligence in preserving evidence can

21

Federal Civil Procedure Analysis

support an adverse-inference instruction See Residential Funding 306 F3d at 108 (negligence enough under some circumstances)

Although a court might well order an evidentiary hearing on the issue of sanctions the facts presented do not seem appropriate for severe sanctions First the evidence was destroyed pursuant to the builderrsquos standard document retention plan and there is no evidence that the builder deliberately failed to suspend its usual procedures with the purpose of allowing the destruction of evidence Second the loss of this evidence will not severely hinder the customerrsquos presentation of his case The central issue is whether the foundation of the house was properly constructed If the construction job was poorly done the customer can present evidence derived from inspection of the premises to prove that point The customer can also depose witnesses about any issues that arose during construction

Under the circumstances a court is not likely to impose particularly severe sanctions although it might shift the burden to the builder to show that the foundation was properly constructed or it might require the builder to reimburse any expenses the customer incurs to discover and prove the facts about issues or disputes that arose during construction of the foundation

[NOTE The result reached by the examinee is less important than the examineersquos recognition that (a) a range of sanctions is available to the court and (b) the appropriate sanction depends both on the culpability of the builder and the prejudice suffered by the customer]

22

CRIMINAL LAW AND PROCEDURE ANALYSIS (Criminal Law and Procedure IIA amp D VE amp F)

ANALYSIS

Legal Problems

(1) Did charging the defendant with both theft and burglary constitute double jeopardy

(2) Did the jury instruction violate the due process clause either by relieving the prosecution of the burden of proving the element of intent or by shifting the burden to the defendant to disprove that element

(3) Did the sentence imposed in this case for the theft conviction unconstitutionally deprive the defendant of his right to a jury trial on the issue of the value of the stolen item

DISCUSSION

Summary

The trial court properly denied the defendantrsquos pretrial motion to dismiss the charges on double jeopardy grounds The defendant may be charged with and convicted of both theft and burglary Each of the charges has an element that the other does not Neither charge is a lesser-included offense nor are they multiplicitous Thus charging both theft and burglary does not violate double jeopardy

The jury instruction on the burglary charge was constitutionally flawed It could have been reasonably understood by the jury as either (1) an irrebuttable conclusive presumption (which relieved the prosecution of proving the element of intent and removed the issue from the jury) or (2) a rebuttable mandatory presumption (which unconstitutionally shifted the burden of proof on an element of a charged offense from the prosecution to the defendant)

Because the four-year sentence imposed by the judge was based on the judgersquos finding by a preponderance of the evidence that the value of the stolen ring exceeded $5000 the sentence violates the defendantrsquos right to a jury determination beyond a reasonable doubt of the value of the ring

Point One (30) Charging the defendant with theft and burglary did not constitute double jeopardy

The Double Jeopardy Clause of the Fifth Amendment provides that a person shall not be twice put in jeopardy for the ldquosame offenserdquo Thus the question is whether the elements of the theft charge are wholly contained in the burglary charge or vice versa If the elements of the lesser charge (theft) are not wholly contained in the greater charge (burglary)mdashie if each charge requires proof of a fact that the other does notmdashthen convicting the defendant of both crimes would not violate double jeopardy even when the two offenses occurred at the same time and are thus arguably part of the ldquosame transactionrdquo Blockburger v United States 284 US 299 304 (1932) See also Albernaz v United States 450 US 333 344 n3 (1981) United States v Dixon 509 US 688 704 (1993)

23

Criminal Law and Procedure Analysis

Here theft and burglary each require proof of an element not required for the other crime Burglary may be defined differently in different jurisdictions However it almost invariably requires entry into a building or dwelling of another with the specific intent to commit a felony therein and the crime of burglary is complete upon the entry into the building or dwelling with such intent See eg Cannon v Oklahoma 827 P2d 1339 1342 (Okla Crim App 1992) In contrast theft which also may be defined differently in different states almost invariably requires the taking and carrying away of an item of personal property belonging to another with the intent to steal or permanently deprive the owner of possession

Here the ldquotakingrdquo or ldquostealingrdquo element is not contained in the definition of burglary and the ldquoentryrdquo element of burglary is not contained in the definition of theft Because theft is not a lesser-included offense of burglary and burglary is not a lesser-included offense of theft charging the defendant for both burglary and theft did not violate double jeopardy and the court properly denied the defense motion on those grounds Yparrea v Dorsey 64 F3d 577 579ndash80 (10th Cir 1995) citing Blockburger 284 US at 304

Finally the defendantrsquos motion to dismiss all the charges on double jeopardy grounds was improper because if both charges were for the same offense the motion should have requested dismissal of one charge not both

Point Two (35) The jury instruction on the burglary charge violated the Due Process Clause because it created either (1) an irrebuttable conclusive presumption (which relieved the prosecution of proving the element of intent and removed that issue from the jury) or (2) a rebuttable mandatory presumption (which unconstitutionally shifted the burden of proof on an element of a charged offense to the defendant)

The Supreme Court has interpreted the Due Process Clause of the US Constitution to require that the prosecution prove all elements of an offense beyond a reasonable doubt See In re Winship 397 US 358 364 (1970) The burden of proof cannot be shifted to the defendant by presuming an essential element upon proof of other elements of the offense because shifting the burden of persuasion with respect to any element of a criminal offense is contrary to the Due Process Clause See Mullaney v Wilbur 421 US 684 (1975)

The crime of burglary includes entry into a building or dwelling with the specific intent to commit a felony therein The requirement that the prosecutor prove beyond a reasonable doubt that the defendant had this specific intent distinguishes burglary from general-intent crimes like trespass See Sandstrom v Montana 442 US 510 523 (1979)

Here the jury was instructed that if ldquoafter consideration of all the evidence presented by the prosecution and defense you find beyond a reasonable doubt that the defendant entered the dwelling without the ownersrsquo consent you may presume that the defendant entered with the intent to commit a felony thereinrdquo This instruction was unconstitutional because it created either an irrebuttable conclusive presumption or a rebuttable mandatory presumption

A conclusive presumption is ldquoan irrebuttable direction by the court to find intent once convinced of the facts triggering the presumptionrdquo Id at 517 Here the jurors were instructed that once the prosecutor established that the defendant entered the neighborsrsquo house without consent they ldquomay presumerdquo that he intended to commit a felony therein The jurors may have reasonably concluded from this instruction that if they found that the defendant intended to enter his neighborsrsquo home without permission they must further find that he entered with the specific intent to commit a felony therein Because this instruction could operate as a conclusive

24

Criminal Law and Procedure Analysis

irrebuttable presumption by eliminating intent ldquoas an ingredient of the offenserdquo it violated due process by relieving the prosecution of the burden of proof for this element Id at 522

In the alternative the jury instruction could have been reasonably understood to create a rebuttable mandatory presumption which ldquotells [the jury] they must find the elemental fact upon proof of the basic fact at least unless the defendant has come forward with some evidence to rebut the presumed connection between the two factsrdquo County Court of Ulster County New York v Allen 442 US 140 157 (1979) The due process problem created by rebuttable mandatory presumptions is that ldquo[t]o the extent that the trier of fact is forced to abide by the presumption and may not reject it based on an independent evaluation of the particular facts presented by the State the analysis of the presumptionrsquos constitutional validity is logically divorced from those facts and based on the presumptionrsquos accuracy in the run of casesrdquo Id at 159

Unlike irrebuttable conclusive presumptions rebuttable mandatory presumptions are not always per se violations of the Due Process Clause However the Supreme Court of the United States has held that jury instructions that could reasonably be understood as shifting the burden of proof to the defendant on an element of the offense are unconstitutional Francis v Franklin 471 US 307 (1985) Here the argument that the jury instruction operated as a rebuttable mandatory presumption is supported by the fact that the judge also instructed the jury to ldquoconsider[ ] all the evidence presented by the prosecution and defenserdquo However even if the instruction created a rebuttable mandatory presumption it would be unconstitutional because it shifted the burden to the defense on an element of the offense Sandstrom 442 US at 524 Mullaney 421 US at 686

[NOTE Whether an examinee identifies the jury instruction as containing a ldquoconclusiverdquo or ldquomandatoryrdquo presumption is less important than the examineersquos analysis of the constitutional infirmities]

Point Three (35) The trial court violated the defendantrsquos Sixth Amendment right to a jury trial on an essential element of the offense when it found by a preponderance of the evidence that the ring was worth over $5000 and increased the defendantrsquos sentence based on this finding

In the statutory scheme under which the defendant was tried and convicted a Class D felony theft is defined as theft of item(s) with a value between $2500 and $10000 The jury found that the value of the diamond ring was at least $2500 and convicted the defendant of felony theft However at sentencing the trial court made a separate finding by a preponderance of the evidence that the value of the ring was greater than $5000 Following the statutersquos two-tiered sentencing scheme the judge then imposed on the defendant a sentence that was one year longer than the maximum that would otherwise have been allowed

The judgersquos sentence was unconstitutional because it violated the defendantrsquos Sixth Amendment right to a jury trial on this question The Supreme Court held in Apprendi v New Jersey 530 US 466 (2000) that ldquo[o]ther than the fact of a prior conviction any fact that increases the penalty for a crime beyond the prescribed statutory maximum must be submitted to a jury and proved beyond a reasonable doubtrdquo because ldquo[i]t is unconstitutional for a legislature to remove from the jury the assessment of facts that increase the prescribed range of penalties to which a criminal defendant is exposed [because] such facts must be established by proof beyond a reasonable doubtrdquo Id The Court reaffirmed Apprendi in Blakely v Washington 542 US 296 (2004) holding that the ldquolsquostatutory maximumrsquo for Apprendi purposes is the maximum sentence a judge may impose solely on the basis of the facts reflected in the jury verdict or admitted by the defendantrdquo Id at 303 (emphasis in original) In United States v Booker 543 US 220 (2005)

25

Criminal Law and Procedure Analysis

the Court relied on Blakely and Apprendi to conclude that protecting a defendantrsquos Sixth Amendment right to a jury trial required that ldquo[a]ny fact which is necessary to support a sentence exceeding the maximum authorized by the facts established by a plea of guilty or a jury verdict must be admitted by the defendant or proved to a jury beyond a reasonable doubtrdquo Id at 244

Thus in order to constitutionally increase a sentence above the statutory maximum of three years the jury must have found beyond a reasonable doubt that the value of the ring exceeded $5000 Here the court made the finding based on an appraisal proffered by the prosecutor only at sentencing and the judgersquos finding was by a preponderance of the evidence rather than beyond a reasonable doubt

26

AGENCY AND PARTNERSHIP ANALYSIS __________ (Agency and Partnership VA amp C VI)

ANALYSIS

Legal Problems

(1) Is a partner in a general partnership personally liable on a claim arising from misrepresentations by another partner made in the course of the partnership business

(2) Does a newly admitted partner in a general partnership become personally liable on existing claims against the partnership

(3) After the filing by a general partnership of a statement of qualification as a limited liability partnership are the partners personally liable as partners on (a) an existing claim against the general partnership and (b) a claim against the partnership that arose after the filing

DISCUSSION

Summary

Adam and Ben formed a general partnership under which they were jointly and severally liable for obligations of the partnership Thus Adam was personally liable for misrepresentations by Ben made in the ordinary course of the partnership business

Upon joining the general partnership Diane became personally liable for the obligations of the partnership arising after her admission but not for obligations pre-existing her admission such as the collectorrsquos claim

By filing a statement of qualification the three partners properly elected limited liability partnership status As partners in an LLP none of the three partners is personally liable as a partner for partnership obligations arising after the election such as the claim by the driverrsquos estate The election however does not change their personal liability on pre-existing claims that arose before the election such as the collectorrsquos claim

Point One (30) As a general partner of Empire a general partnership Adam became personally liable on the collectorrsquos claim a valid claim against the partnership that arose because of Benrsquos wrongful act in the ordinary course of the partnership business

When the collectorrsquos claim arose Empire was a general partnership composed of Adam and Ben Under UPA (1997) sect 306(a) partners of a general partnership are liable jointly and severally for all obligations of the partnership Under UPA (1997) sect 305(a) the partnership could become obligated for the loss caused to the collector as a result of the misrepresentation by Ben provided he was acting in the ordinary course of the partnership business Because there was no statement that limited his partnership authority Ben as partner was ldquoan agent of the partnership for the purpose of its businessrdquo See UPA (1997) sect 301(1) Benrsquos misrepresentation to the collector even if intentional appears to be in the ordinary course of the partnershiprsquos business of dealing

27

Agency and Partnership Analysis

in antique cars Thus Benrsquos wrongful act created a partnership obligation for which Adam was jointly and severally liable

[NOTE Generally a partnership creditor must ldquoexhaust the partnershiprsquos assets before levying on a judgment debtor partnerrsquos individual property where the partner is personally liable for the partnership obligationrdquo as a result of his status as a partner UPA (1997) sect 307 cmt 4 As the UPA comments explain this places Adam more in the position of guarantor than principal debtor on the partnership obligation Id cmt 4 Although an examinee might discuss this point the call focuses on whether Adam is personally liable not how the liability might be enforced]

Point Two (30) Because the collectorrsquos claim arose before Diane joined Empire Diane did not become personally liable on the claim

Diane was admitted to Empire when it was a general partnership and after the collectorrsquos claim arose While the general rule under UPA (1997) sect 306(a) is that the partners of a general partnership are liable jointly and severally for all obligations of the partnership there is a special rule for partners who are admitted during the duration of the partnership Under UPA (1997) sect 306(b) a person admitted to an existing partnership is not personally liable for any partnership obligations incurred before the personrsquos admission Because Diane was admitted to Empire after the collectorrsquos claim arose Diane is not personally liable on the claim

Dianersquos knowledge of the pre-existing claim and her stated concern about becoming liable on the collectorrsquos claim do not change her personal nonliability to the collector Although partners who have a liability shield can assume liability to third parties through private contractual guarantees or modifications to the partnership agreement Dianersquos stated concern constituted neither a guaranty to the collector nor ldquoan intentional waiver of liability protectionsrdquo See UPA (1997) sect 306 cmt 3 (describing methods for waiver of liability protections under sect 306(c) applicable in limited liability partnerships)

At most Diane will lose her investment in the partnership as a result of the collectorrsquos claim Although Diane did not become personally liable on the collectorrsquos claim when she joined the partnership the $250000 she contributed to the partnership is ldquoat risk for the satisfaction of existing partnership debtsrdquo UPA (1997) sect 306 cmt 2

Point Three (40) Filing the statement of qualification was effective to elect limited liability partnership status Despite this new status Adam and Ben remain personally liable on the collectorrsquos claim which arose before the election But as partners in an LLP neither Adam Ben nor Diane is personally liable as a partner on the driverrsquos estatersquos claim which arose after the election

Under UPA (1997) sect 1001 a general partnership can make an election and become a limited liability partnershipmdashif the partners approve the conversion by a vote equivalent to that necessary to amend the partnership agreement and the partnership then files a statement of qualification that specifies the name of the partnership its principal office and its election to be an LLP Here the partners agreed unanimouslymdashsufficient to amend their agreement under UPA (1997) sect 401(j)mdashand the statement of qualification was filed In addition the name of Empire LLP properly included an appropriate ending ldquoLLPrdquo See UPA (1997) sect 1002

Although another way to effectuate a ldquoconversionrdquo (as suggested by Benrsquos lawyer) is to form a new LLP and transfer the assets of the old general partnership to the new LLP the

28

Agency and Partnership Analysis

method used here (approval by the partners and the filing of a statement of qualification) is also sufficient to create LLP status

Thus Empire became Empire LLP as of the date of filing of the statement of qualification See UPA (1997) sect 1001 What effect did this have on the collectorrsquos claim which predated the filing According to UPA (1997) sect 306(c) an obligation incurred while a partnership is an LLP is solely a partnership obligation As the collectorrsquos claim predated the LLP Adam and Ben remain personally liable on the collectorrsquos claim Diane on the other hand was not personally liable on the collectorrsquos claim either before or after the filing of the statement of qualification See Point Two above

The driverrsquos estatersquos claim arose after Empire became Empire LLP Under UPA (1997) sect 306(c) an obligation incurred while a partnership is an LLP is solely a partnership obligationThus Adam Ben and Diane as partners are all protected from personal liability on the driverrsquos estatersquos claim But there may be personal liability if any of them was negligent or otherwise acted wrongfully by not informing the buyer of the bad suspension that caused the accident

29

National Conference of Bar Examiners 302 South Bedford Street | Madison WI 53703-3622 Phone 608-280-8550 | Fax 608-280-8552 | TDD 608-661-1275

wwwncbexorg e-mail contactncbexorg

  • Preface
  • Description of the MEE
  • Instructions
  • February 2014 Questions
    • Constitutinal Law Question
    • Trusts and Future Interests Question
    • Secured Transactions Question
    • Federal Civil Procedure Question
    • Criminal Law and Procedure Question
    • Agency and Partnership Question
      • February 2014 Analyses
        • Constitutional Law Analysis
        • Trust and Future Interests Analysis
        • Secured Transactions Analysis
        • Federal Civil Procedure Analysis
        • Criminal Law and Procedure Analysis
        • Agency and Partnership Analysis
            • ltlt13 ASCII85EncodePages false13 AllowTransparency false13 AutoPositionEPSFiles true13 AutoRotatePages None13 Binding Left13 CalGrayProfile (Dot Gain 20)13 CalRGBProfile (sRGB IEC61966-21)13 CalCMYKProfile (US Web Coated 050SWOP051 v2)13 sRGBProfile (sRGB IEC61966-21)13 CannotEmbedFontPolicy Error13 CompatibilityLevel 1413 CompressObjects Tags13 CompressPages true13 ConvertImagesToIndexed true13 PassThroughJPEGImages true13 CreateJobTicket false13 DefaultRenderingIntent Default13 DetectBlends true13 DetectCurves 0000013 ColorConversionStrategy CMYK13 DoThumbnails false13 EmbedAllFonts true13 EmbedOpenType false13 ParseICCProfilesInComments true13 EmbedJobOptions true13 DSCReportingLevel 013 EmitDSCWarnings false13 EndPage -113 ImageMemory 104857613 LockDistillerParams false13 MaxSubsetPct 10013 Optimize true13 OPM 113 ParseDSCComments true13 ParseDSCCommentsForDocInfo true13 PreserveCopyPage true13 PreserveDICMYKValues true13 PreserveEPSInfo true13 PreserveFlatness true13 PreserveHalftoneInfo false13 PreserveOPIComments true13 PreserveOverprintSettings true13 StartPage 113 SubsetFonts true13 TransferFunctionInfo Apply13 UCRandBGInfo Preserve13 UsePrologue false13 ColorSettingsFile ()13 AlwaysEmbed [ true13 ]13 NeverEmbed [ true13 ]13 AntiAliasColorImages false13 CropColorImages true13 ColorImageMinResolution 30013 ColorImageMinResolutionPolicy OK13 DownsampleColorImages true13 ColorImageDownsampleType Bicubic13 ColorImageResolution 30013 ColorImageDepth -113 ColorImageMinDownsampleDepth 113 ColorImageDownsampleThreshold 15000013 EncodeColorImages true13 ColorImageFilter DCTEncode13 AutoFilterColorImages true13 ColorImageAutoFilterStrategy JPEG13 ColorACSImageDict ltlt13 QFactor 01513 HSamples [1 1 1 1] VSamples [1 1 1 1]13 gtgt13 ColorImageDict ltlt13 QFactor 01513 HSamples [1 1 1 1] VSamples [1 1 1 1]13 gtgt13 JPEG2000ColorACSImageDict ltlt13 TileWidth 25613 TileHeight 25613 Quality 3013 gtgt13 JPEG2000ColorImageDict ltlt13 TileWidth 25613 TileHeight 25613 Quality 3013 gtgt13 AntiAliasGrayImages false13 CropGrayImages true13 GrayImageMinResolution 30013 GrayImageMinResolutionPolicy OK13 DownsampleGrayImages true13 GrayImageDownsampleType Bicubic13 GrayImageResolution 30013 GrayImageDepth -113 GrayImageMinDownsampleDepth 213 GrayImageDownsampleThreshold 15000013 EncodeGrayImages true13 GrayImageFilter DCTEncode13 AutoFilterGrayImages true13 GrayImageAutoFilterStrategy JPEG13 GrayACSImageDict ltlt13 QFactor 01513 HSamples [1 1 1 1] VSamples [1 1 1 1]13 gtgt13 GrayImageDict ltlt13 QFactor 01513 HSamples [1 1 1 1] VSamples [1 1 1 1]13 gtgt13 JPEG2000GrayACSImageDict ltlt13 TileWidth 25613 TileHeight 25613 Quality 3013 gtgt13 JPEG2000GrayImageDict ltlt13 TileWidth 25613 TileHeight 25613 Quality 3013 gtgt13 AntiAliasMonoImages false13 CropMonoImages true13 MonoImageMinResolution 120013 MonoImageMinResolutionPolicy OK13 DownsampleMonoImages true13 MonoImageDownsampleType Bicubic13 MonoImageResolution 120013 MonoImageDepth -113 MonoImageDownsampleThreshold 15000013 EncodeMonoImages true13 MonoImageFilter CCITTFaxEncode13 MonoImageDict ltlt13 K -113 gtgt13 AllowPSXObjects false13 CheckCompliance [13 None13 ]13 PDFX1aCheck false13 PDFX3Check false13 PDFXCompliantPDFOnly false13 PDFXNoTrimBoxError true13 PDFXTrimBoxToMediaBoxOffset [13 00000013 00000013 00000013 00000013 ]13 PDFXSetBleedBoxToMediaBox true13 PDFXBleedBoxToTrimBoxOffset [13 00000013 00000013 00000013 00000013 ]13 PDFXOutputIntentProfile ()13 PDFXOutputConditionIdentifier ()13 PDFXOutputCondition ()13 PDFXRegistryName ()13 PDFXTrapped False1313 CreateJDFFile false13 Description ltlt13 ARA 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 BGR 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 CHS ltFEFF4f7f75288fd94e9b8bbe5b9a521b5efa7684002000410064006f006200650020005000440046002065876863900275284e8e9ad88d2891cf76845370524d53705237300260a853ef4ee54f7f75280020004100630072006f0062006100740020548c002000410064006f00620065002000520065006100640065007200200035002e003000204ee553ca66f49ad87248672c676562535f00521b5efa768400200050004400460020658768633002gt13 CHT ltFEFF4f7f752890194e9b8a2d7f6e5efa7acb7684002000410064006f006200650020005000440046002065874ef69069752865bc9ad854c18cea76845370524d5370523786557406300260a853ef4ee54f7f75280020004100630072006f0062006100740020548c002000410064006f00620065002000520065006100640065007200200035002e003000204ee553ca66f49ad87248672c4f86958b555f5df25efa7acb76840020005000440046002065874ef63002gt13 CZE 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 DAN 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 DEU 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 ESP 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 ETI 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 FRA 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 GRE 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 HEB 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 HRV (Za stvaranje Adobe PDF dokumenata najpogodnijih za visokokvalitetni ispis prije tiskanja koristite ove postavke Stvoreni PDF dokumenti mogu se otvoriti Acrobat i Adobe Reader 50 i kasnijim verzijama)13 HUN 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 ITA 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 JPN ltFEFF9ad854c18cea306a30d730ea30d730ec30b951fa529b7528002000410064006f0062006500200050004400460020658766f8306e4f5c6210306b4f7f75283057307e305930023053306e8a2d5b9a30674f5c62103055308c305f0020005000440046002030d530a130a430eb306f3001004100630072006f0062006100740020304a30883073002000410064006f00620065002000520065006100640065007200200035002e003000204ee5964d3067958b304f30533068304c3067304d307e305930023053306e8a2d5b9a306b306f30d530a930f330c8306e57cb30818fbc307f304c5fc59808306730593002gt13 KOR ltFEFFc7740020c124c815c7440020c0acc6a9d558c5ec0020ace0d488c9c80020c2dcd5d80020c778c1c4c5d00020ac00c7a50020c801d569d55c002000410064006f0062006500200050004400460020bb38c11cb97c0020c791c131d569b2c8b2e4002e0020c774b807ac8c0020c791c131b41c00200050004400460020bb38c11cb2940020004100630072006f0062006100740020bc0f002000410064006f00620065002000520065006100640065007200200035002e00300020c774c0c1c5d0c11c0020c5f40020c2180020c788c2b5b2c8b2e4002egt13 LTH 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 LVI ltFEFF0049007a006d0061006e0074006f006a00690065007400200161006f00730020006900650073007400610074012b006a0075006d00750073002c0020006c0061006900200076006500690064006f00740075002000410064006f00620065002000500044004600200064006f006b0075006d0065006e007400750073002c0020006b006100730020006900720020012b00700061016100690020007000690065006d01130072006f00740069002000610075006700730074006100730020006b00760061006c0069007401010074006500730020007000690072006d007300690065007300700069006501610061006e006100730020006400720075006b00610069002e00200049007a0076006500690064006f006a006900650074002000500044004600200064006f006b0075006d0065006e007400750073002c0020006b006f002000760061007200200061007400760113007200740020006100720020004100630072006f00620061007400200075006e002000410064006f00620065002000520065006100640065007200200035002e0030002c0020006b0101002000610072012b00200074006f0020006a00610075006e0101006b0101006d002000760065007200730069006a0101006d002egt13 NLD (Gebruik deze instellingen om Adobe PDF-documenten te maken die zijn geoptimaliseerd voor prepress-afdrukken van hoge kwaliteit De gemaakte PDF-documenten kunnen worden geopend met Acrobat en Adobe Reader 50 en hoger)13 NOR 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 POL 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 PTB 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 RUM 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 RUS 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 SKY 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 SLV 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 SUO ltFEFF004b00e40079007400e40020006e00e40069007400e4002000610073006500740075006b007300690061002c0020006b0075006e0020006c0075006f00740020006c00e400680069006e006e00e4002000760061006100740069007600610061006e0020007000610069006e006100740075006b00730065006e002000760061006c006d0069007300740065006c00750074007900f6006800f6006e00200073006f00700069007600690061002000410064006f0062006500200050004400460020002d0064006f006b0075006d0065006e007400740065006a0061002e0020004c0075006f0064007500740020005000440046002d0064006f006b0075006d0065006e00740069007400200076006f0069006400610061006e0020006100760061007400610020004100630072006f0062006100740069006c006c00610020006a0061002000410064006f00620065002000520065006100640065007200200035002e0030003a006c006c00610020006a006100200075007500640065006d006d0069006c006c0061002egt13 SVE 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 TUR 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 UKR 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 ENU (Use these settings to create Adobe PDF documents best suited for high-quality prepress printing Created PDF documents can be opened with Acrobat and Adobe Reader 50 and later)13 gtgt13 Namespace [13 (Adobe)13 (Common)13 (10)13 ]13 OtherNamespaces [13 ltlt13 AsReaderSpreads false13 CropImagesToFrames true13 ErrorControl WarnAndContinue13 FlattenerIgnoreSpreadOverrides false13 IncludeGuidesGrids false13 IncludeNonPrinting false13 IncludeSlug false13 Namespace [13 (Adobe)13 (InDesign)13 (40)13 ]13 OmitPlacedBitmaps false13 OmitPlacedEPS false13 OmitPlacedPDF false13 SimulateOverprint Legacy13 gtgt13 ltlt13 AddBleedMarks false13 AddColorBars false13 AddCropMarks false13 AddPageInfo false13 AddRegMarks false13 ConvertColors ConvertToCMYK13 DestinationProfileName ()13 DestinationProfileSelector DocumentCMYK13 Downsample16BitImages true13 FlattenerPreset ltlt13 PresetSelector MediumResolution13 gtgt13 FormElements false13 GenerateStructure false13 IncludeBookmarks false13 IncludeHyperlinks false13 IncludeInteractive false13 IncludeLayers false13 IncludeProfiles false13 MultimediaHandling UseObjectSettings13 Namespace [13 (Adobe)13 (CreativeSuite)13 (20)13 ]13 PDFXOutputIntentProfileSelector DocumentCMYK13 PreserveEditing true13 UntaggedCMYKHandling LeaveUntagged13 UntaggedRGBHandling UseDocumentProfile13 UseDocumentBleed false13 gtgt13 ]13gtgt setdistillerparams13ltlt13 HWResolution [2400 2400]13 PageSize [612000 792000]13gtgt setpagedevice13

Page 8: February 2014 MEE Questions and AnalysesPreface The Multistate Essay Examination (MEE) is developed by the National Conference of Bar Examiners (NCBE). This publication includes the

TRUSTS AND FUTURE INTERESTS QUESTION _______________

Ten years ago a testator died survived by his only children a son age 26 and a daughter age 18

A testamentary trust was created under the testatorrsquos duly probated will The will specified that all trust income would be paid to the son during the sonrsquos lifetime and that upon the sonrsquos death the trust would terminate and trust principal would be distributed to the testatorrsquos ldquograndchildren who shall surviverdquo the son The testator provided for his daughter in other sections of the will

Five years ago the trustee of the testamentary trust purchased an office building with $500000 from the trust principal Other than this building the trust assets consist of publicly traded securities

Last year the trustee received $30000 in rents from the office building The trustee also received with respect to the securities owned by the trust cash dividends of $20000 and a stock dividend of 400 shares of Acme Corp common stock distributed to the trust by Acme Corp

Eight months ago the trustee sold the office building for $700000

Six months ago the son delivered a letter to the trustee stating ldquoI hereby disclaim any interest I may have in the income interest of the trustrdquo On the date the son delivered this letter to the trustee the son had no living children the daughter had one living minor child

A statute in this jurisdiction provides that ldquoa disclaimer of any interest created by will is valid only if made within nine months after the testatorrsquos death and if an interest is validly disclaimed the disclaiming party is deemed to have predeceased the testatorrdquo

1 How should the rents sales proceeds cash dividends and stock dividends received prior to the trusteersquos receipt of the sonrsquos letter have been allocated between trust principal and income Explain

2 How if at all does the sonrsquos letter to the trustee affect the future distribution of trust income and principal Explain

4

SECURED TRANSACTIONS QUESTION

On March 1 the owner of a manufacturing business entered into negotiations with a bank to obtain a loan of $100000 for the business The bank loan officer informed the business owner that the interest rate for a loan would be lower if the repayment obligation were secured by all the businessrsquos present and future equipment The loan officer also informed the business owner that the bank could not commit to making the loan until its credit investigation was completed but that funds could be advanced faster following loan approval if a financing statement with respect to the transaction were filed in advance Accordingly the business owner signed a form on behalf of the business authorizing the bank to file a financing statement with respect to the proposed transaction The bank properly filed a financing statement the next day correctly providing the name of the business as the debtor and indicating ldquoequipmentrdquo as the collateral

On March 15 the business owner had heard nothing from the bank about whether the loan had been approved so the business owner approached a finance company for a loan The finance company quickly agreed to lend $100000 to the business secured by all the businessrsquos present and future equipment That same day the finance company loaned to the business $100000 and the business owner signed an agreement obligating the business to repay the loan and granting the finance company a security interest in all the businessrsquos ldquopresent and future equipmentrdquo to secure the repayment obligation Also on that day the finance company properly filed a financing statement correctly providing the businessrsquos name as the debtor and indicating ldquoequipmentrdquo as the collateral

On March 21 the bank loan officer contacted the business owner and indicated that the loan application had been approved On the next day March 22 the bank loaned the business $100000 The loan agreement signed by the owner on behalf of the business granted the bank a security interest in all the businessrsquos ldquopresent and future equipmentrdquo

On April 10 the business sold an item of manufacturing equipment to a competitor for $20000 This was the first time the business had ever sold any of its equipment The competitor paid the purchase price in cash and took possession of the equipment that day The competitor acted in good faith at all times and had no knowledge of the businessrsquos prior transactions with the bank and the finance company

The business has defaulted on its obligations with respect to the loans from the bank and the finance company Each of them has asserted a claim to all the businessrsquos equipment as well as to the item of equipment sold to the businessrsquos competitor

Assume that the business owner had the authority to enter into all these transactions on behalf of the business

1 As between the bank and the finance company which has a superior claim to the businessrsquos equipment Explain

2 Do the claims of the bank and the finance company to the businessrsquos equipment continue in the item of equipment sold to the competitor Explain

5

FEDERAL CIVIL PROCEDURE QUESTION

A builder constructed a vacation house for an out-of-state customer on the customerrsquos land The house was completed on June 1 at which point the customer still owed $200000 of the $800000 contract price which was payable in full five days later

On June 14 the basement of the house was flooded with two inches of water during a heavy rainfall When the customer complained the builder told the customer ldquoThe flooding was caused by poorly designed landscaping Our work is fine and fully up to code Have an engineer look at the foundation If therersquos a problem wersquoll fix itrdquo

The customer pleased by the builderrsquos cooperative attitude immediately hired a structural engineer to examine the foundation of the house On June 30 the engineer provided the customer with a written report on the condition of the foundation which stated that the foundation was properly constructed

Unhappy with the conclusions in the engineerrsquos report the customer then hired a home inspector to evaluate the house The home inspectorrsquos report concluded that the foundation of the house had been poorly constructed and was inadequately waterproofed

On July 10 the customer sent the builder the home inspectorrsquos report with a note that said ldquoUntil you fix this problem you wonrsquot get another penny from merdquo The builder immediately contacted an attorney and directed the attorney to prepare a draft complaint against the customer for nonpayment Hoping to avoid litigation the builder sent several more requests for payment to the customer The customer ignored all these requests

On September 10 the builder filed suit in federal district court properly invoking the courtrsquos diversity jurisdiction and seeking $200000 in damages for breach of contract The customerrsquos answer denied liability on the basis of alleged defective construction of the housersquos foundation

Several months later the case is nearly ready for trial However two discovery disputes have not yet been resolved

First despite a request from the builder the customer has refused to provide a copy of the report prepared by the structural engineer who examined the foundation of the house The customer claims that the report is ldquowork productrdquo and not discoverable because the customer does not intend to ask the engineer to testify at trial The builder has asked the court to order the customer to turn over the engineerrsquos report

Second the customer has asked the court to impose sanctions for the builderrsquos failure to comply with the customerrsquos demand for copies of all emails concerning construction of the foundation of the house The builder has truthfully informed the customer that all such emails were destroyed on August 2 This destruction was pursuant to the builderrsquos standard practice of permanently deleting all project-related emails from company records 60 days after construction of a project is complete There is no relevant state records-retention law

1 Should the court order the customer to turn over the engineerrsquos report Explain

2 Should the court sanction the builder for the destruction of emails related to the case and if so what factors should the court consider in determining those sanctions Explain

6

CRIMINAL LAW AND PROCEDURE QUESTION _____

A defendant was charged under state law with felony theft (Class D) and felony residential burglary (Class C) The indictment alleged that the defendant entered his neighborsrsquo home without their consent and stole a diamond ring worth at least $2500

Defense counsel filed a pretrial motion to dismiss the charges on the ground that prosecuting the defendant for both burglary and theft would constitute double jeopardy The trial court denied the motion and the defendant was prosecuted for both crimes The only evidence of the ringrsquos value offered at the defendantrsquos jury trial was the ownerrsquos testimony that she had purchased the ring two years earlier for $3000

At trial the judge issued the following jury instruction on the burglary charge prior to deliberations

If after consideration of all the evidence presented by the prosecution and defense you find beyond a reasonable doubt that the defendant entered the dwelling without the ownersrsquo consent you may presume that the defendant entered with the intent to commit a felony therein

The jury found the defendant guilty of both offenses

At the defendantrsquos sentencing hearing an expert witness called by the prosecutor testified that the diamond ring was worth between $7000 and $8000 Over defense objection the judge concluded by a preponderance of the evidence that the value of the stolen ring exceeded $5000 The judge sentenced the defendant to four yearsrsquo incarceration on the theft conviction On the burglary conviction the defendant received a consecutive sentence of seven yearsrsquo incarceration

In this state residential burglary is defined as ldquoentry into the dwelling of another without the consent of the lawful resident with the intent to commit a felony thereinrdquo Residential burglary is a Class C felony for which the minimum sentence is five years and the maximum sentence is ten years of incarceration

In this state theft is defined as ldquotaking and carrying away the property of another with the intent to permanently deprive the owner of possessionrdquo Theft is a Class D felony if the value of the item(s) taken is between $2500 and $10000 The sentence for a Class D felony theft is determined by the value of the items taken If the value is between $2500 and $5000 the maximum sentence is three yearsrsquo incarceration If the value of the items exceeds $5000 the maximum sentence is five yearsrsquo incarceration

This state affords a criminal defendant no greater rights than those mandated by the United States Constitution

1 Did the trial court err when it denied the defendantrsquos pretrial motion to dismiss on double jeopardy grounds Explain

2 Did the trial court err in its instruction to the jury on the burglary charge Explain

3 Did the trial court err when it sentenced the defendant to an additional year of incarceration on the theft conviction based on the expertrsquos testimony Explain

7

AGENCY AND PARTNERSHIP QUESTION _____

Five years ago Adam and Ben formed a general partnership Empire Partnership (Empire) to buy and sell antique automobiles at a showroom in State A Adam contributed $800000 to Empire and Ben contributed $200000 Their written partnership agreement allocated 80 of profits losses and control to Adam and 20 to Ben No filings of any type were made in connection with the formation of Empire

Three years ago a collector purchased one of Empirersquos antique cars for $3400000 The collector was willing to pay this price because of Benrsquos false representation (repeated in the sales contract) that a famous movie star had once owned the car Without the movie-star connection the car was worth only $100000 One month later when the collector discovered the truth he sued Adam Ben and Empire for $3300000 in damages The lawsuit is still pending

Two years ago Adam and Ben admitted a new partner Diane to Empire in return for her contribution of $250000 The three agreed to allocate profits losses and control 75 to Adam 10 to Ben and 15 to Diane Before joining the partnership Diane learned of the collectorrsquos claim and stated her concern to Adam and Ben that she might become liable if the claim were reduced to a judgment

Following Dianersquos admission to Empire the three partners sought to convert Empire into a limited liability partnership (LLP) Adamrsquos lawyer proposed to file with State A a ldquostatement of qualificationrdquo making an LLP election and declaring the name of the partnership to be ldquoEmpire LLPrdquo Benrsquos lawyer stated that this would not work and that a new LLP had to be formed with the assets of the old partnership transferred to the new one In the end the conversion was done the way Adamrsquos lawyer suggested with the approval of all three partners

One year ago a driver purchased a vintage car from Empire LLP based on the representation that the car was ldquofully roadworthy and capable of touring at 70 mph all dayrdquo The driver took the car on the highway at 50 mph whereupon the front suspension collapsed resulting in a crash in which the car was destroyed and the driver killed The driverrsquos estate sued Adam Ben Diane and Empire LLP for $10000000 The lawsuit is still pending

Although profitable Empire LLP does not have resources sufficient to pay the collectorrsquos claim or the claim of the driverrsquos estate

Assume that the Uniform Partnership Act (1997) applies

1 Before the filing of the statement of qualification (a) was Adam personally liable on the collectorrsquos claim Explain (b) was Diane personally liable on the collectorrsquos claim Explain

2 After the filing of the statement of qualification was Adam Ben or Diane personally liable as a partner on (a) the collectorrsquos claim or (b) the driverrsquos estatersquos claim Explain

8

February 2014 MEE

ANALYSES Constitutional Law

Trusts and Future Interests Secured Transactions

Federal Civil Procedure Criminal Law and Procedure

Agency and Partnership

CONSTITUTIONAL LAW ANALYSIS (Constitutional Law IVD)

ANALYSIS

Legal Problems

(1) Is the city ordinance requirement that businesses install floodlights a taking

(2) Is conditioning the approval of a building permit on the grant of an easement to install surveillance equipment a taking of property

DISCUSSION

Summary

The ordinance requiring businesses to install floodlights is not a per se taking under Loretto because it does not force a private landowner to allow a third party to enter and place a physical object on the land Here the city ordinance requires the businessmdashnot a third partymdashto install the floodlights

The ordinance is likely not a regulatory taking under the Penn Central balancing test While the ordinance will impose a cost on business owners that cost may be offset by the expected increase in business due to the ordinance and the ordinance does not appear to interfere with the ownerrsquos primary use of the property as a restaurant

The permit condition however is likely an uncompensated taking of property While the condition has an essential nexus with the cityrsquos legitimate interest in promoting public safety the city has not made an individualized determination that the easement condition is roughly proportional to the possibility of increased crime due to the restaurantrsquos proposed addition Thus the permit condition likely violates the Fifth Amendment as applied to the states through the Fourteenth Amendment

Point One (50) The ordinance requiring that businesses install floodlights is not a per se taking under Loretto It is not a regulatory taking under the Penn Central balancing test because the cost of compliance with the ordinance may be offset by an expected increase in business and compliance does not interfere with the businessrsquos primary use of its property as a restaurant

The city ordinance requiring a business to install floodlights does not effect a per se taking of the sort described in Loretto v Teleprompter Manhattan CATV Corp 458 US 419 (1982) because no property is physically taken by the government and the ordinance does not involve a physical invasion of private property by a third party

Even though the ordinance does not constitute an occupation of the property by either the government or a third party it is still subject to the three-factor balancing test under Penn Central Transportation Co v City of New York 438 US 104 (1978) to determine whether it is a ldquoregulatory takingrdquo Under Penn Central a court must balance (1) ldquo[t]he economic impact of the regulation on the claimantrdquo (2) ldquothe extent to which the regulation has interfered with distinct investment-backed expectationsrdquo and (3) ldquothe character of the governmental actionrdquo Id at 124 Here each factor weighs against finding that the ordinance is a taking

11

Constitutional Law Analysis

First the ordinance requirement likely has a minimal economic impact on the restaurant Compliance with the ordinance is estimated to cost $1000 and the city has found that businesses will likely recoup that cost in increased sales Also because the ordinance does not interfere with the operation of the restaurant the owner may still earn a reasonable return on its investment in the property

Second the ordinance does not interfere with the businessrsquos investment-backed expectations As in Penn Central the challenged law does not interfere with the ownerrsquos ldquoprimary expectationrdquo for use of the propertymdashin Penn Central as a railroad terminal and here as a restaurant Further the ordinance does not prevent the restaurant from expanding to meet the changing business environment

Third the character of the government action does not weigh in favor of a taking While Penn Central does say that a ldquophysical invasionrdquo is more likely to pose a taking Loretto suggests that the Courtrsquos main concern is with physical invasions by third parties Also like the landmark law challenged in Penn Central the ordinance here ldquoadjust[s] the benefits and burdens of economic life to promote the common goodrdquo Id In Penn Central the landmark law restricted development of the railroad terminal to promote the common interest in preserving historic landmarks Here the ordinance requires the businesses to install floodlights to promote the common interest in crime prevention and public safety

Because the ordinance is clearly a valid exercise of the police power it satisfies the takings clausersquos public-use requirement Kelo v City of New London 545 US 469 (2005)

In sum all three factors weigh against finding a taking under the Penn Central balancing test

Point Two (50) The permit condition may be unconstitutional as an uncompensated taking of property because the city has not made an individualized determination that the easement condition is roughly proportional to the impact of the restaurantrsquos proposed addition

In Dolan v City of Tigard 512 US 374 (1994) the Supreme Court set forth the test for determining whether an exaction imposed by a government in exchange for a discretionary benefit conferred by the government such as a condition on the approval of a building permit in this case constitutes an uncompensated taking under the Fifth Amendment The exaction is not a taking if (1) there is an ldquoessential nexusrdquo between the ldquopublic need or burdenrdquo to which the proposed development contributes and ldquothe permit condition exacted by the cityrdquo id at 386 and (2) the government makes ldquosome sort of individualized determination that the required dedication is [roughly proportional] both in nature and extent to the impact of the proposed developmentrdquo Id at 391 see also Nollan v California Coastal Commission 483 US 825 (1987)

Here the city likely can meet the nexus requirement In Dolan the landowner sought to double the size of its business which would have increased traffic on nearby roadways In exchange for approving the development the city sought an easement for a bike and pedestrian path The Court found the required nexus between the easement and the cityrsquos ldquoattempt to reduce traffic congestion by providing for alternative means of transportationrdquo 512 US at 387 Here a similar nexus likely exists between the requested easement and the cityrsquos interest in crime prevention and public safety Increased patronage and economic activity at the restaurant might attract additional crime to the area and the requested easement to install surveillance equipment would attempt to address that increased crime

12

Constitutional Law Analysis

The exaction here however may fail the second prong of the Dolan testmdashthat the exaction be roughly proportional to the anticipated impact of the requested development As noted the city in Dolan claimed that a bike and pedestrian path was needed to offset the increase in traffic due to the proposed doubling of the business The Court explained that the government must demonstrate that the additional traffic reasonably was related to the requested exaction and that the government must ldquomake some effort to quantify its findings in support of the dedication for the pedestrianbicycle pathway beyond the conclusory statement that it could offset some of the traffic demand generatedrdquo Id at 395 Here the city did not carry its burden The city simply speculates that increased patronage of the restaurant ldquomightrdquo increase crime and that the surveillance equipment ldquomightrdquo alleviate this increased crime Because the city has not made ldquosome effort to quantify its findingsrdquo in support of the easement it has not shown that the burden of the easement is roughly proportional to the benefits thought to flow from it

Thus the exaction appears to be an uncompensated taking of property in violation of the Fifth Amendment as applied to the states through the Fourteenth Amendment

13

TRUSTS AND FUTURE INTERESTS ANALYSIS ____ (Trusts and Future Interests IE3 I5 IIIA amp B)

ANALYSIS

Legal Problems

(1) How should rents dividends and sales proceeds received by the trustee prior to receipt of the sonrsquos letter have been allocated between trust income and principal

(2)(a) Did the remainder interest in the trust accelerate and become immediately payable to the daughterrsquos minor child upon the trusteersquos receipt of the sonrsquos letter and if not how should the trustee handle the distribution of the principal in the future

(2)(b) Following the trusteersquos receipt of the sonrsquos letter how should the trustee distribute future receipts of income prior to the distribution of the principal

DISCUSSION

Summary

Prior to the trusteersquos receipt of the sonrsquos letter cash dividends and rents should have been allocated to trust income and were distributable to the son the income beneficiary of the trust sales proceeds and stock dividends should have been allocated to principal

Because the sonrsquos letter to the trustee did not result in a valid disclaimer under state law (having been made more than nine months after the testatorrsquos death) the son is not deemed to have predeceased the testator Because the son is still living the class gift to the testatorrsquos grandchildren who survive the son has not closed and is not possessory it will not become possessory until the son dies The daughterrsquos minor child being the testatorrsquos only living grandchild is not currently entitled to a distribution of trust principal Trust principal will instead be distributable upon the sonrsquos death to the testatorrsquos then-living grandchildren or if there are none to the testatorrsquos then-living heirs

As for future income the trustee should either distribute the trust income to the son and the daughter as the testatorrsquos heirs accumulate the income for future distribution to those individuals ultimately entitled to the trust principal or distribute it to those presumptively entitled to the principal upon the sonrsquos death ie the daughterrsquos minor child

Point One (45) Cash dividends and rents are allocable to income sales proceeds and stock dividends are allocable to principal Items allocable to income for the period prior to the sonrsquos attempted disclaimer were distributable to the son

Receipts earned during the administration of a trust are allocable either to income or to principal Almost all states have adopted the most recent or an earlier version of the Uniform Principal and Income Act (the Act) which specifies how such receipts should be allocated

Under the Act rents (UNIF PRIN amp INC ACT (2000) sect 405 UNIF PRIN amp INC ACT (1962) sect 3(a)(1)) and cash dividends received from a corporation (UNIF PRIN amp INC ACT (2000) sect 401(b) UNIF PRIN amp INC ACT (1962) sect 6(d)) are allocable to income and are distributable to the income beneficiary of the trust

14

Trusts and Future Interests Analysis Sales proceeds (UNIF PRIN amp INC ACT (2000) sect 404(2) UNIF PRIN amp INC ACT (1962)

sect 3(b)(1)) and dividends paid in the stock of the distributing corporation (UNIF PRIN amp INC ACT (2000) sect 401(c)(1) UNIF PRIN amp INC ACT (1962) sect 3(b)(4)) are allocable to principal and added to the principal of the trust

Here the cash dividends and office building rents should have been allocated to income and until the trustee received the sonrsquos letter should have been distributed to him as the sole income beneficiary of the trust The stock dividend and proceeds from the sale of the office building should have been allocated to principal and held by the trustee for future distribution to the ultimate remaindermen of the trust

[NOTE The 2000 Uniform Principal and Income Act has been adopted in Alabama Arkansas Colorado Connecticut the District of Columbia Hawaii Idaho Iowa Kentucky Missouri Montana Nebraska New Mexico North Dakota Oregon South Dakota Utah and West Virginia]

Point Two(a) (45) Because the son did not disclaim within nine months of the testatorrsquos death there is no valid disclaimer under state law Therefore the son is not deemed to have predeceased the testator Furthermore because of the express survivorship contingency in the will the remainder in the trust does not accelerate and become distributable until the son in fact dies When the son dies the trust principal will be distributable to the testatorrsquos then-living grandchildren or if none then to the testatorrsquos then-living heirs

When a trust remainder is given to a class the class closes (ie no new persons can join the class) when there is no outstanding income interest and at least one member of the class is then entitled to demand possession of his or her share of the remainder This principle is called the rule of convenience See generally HERBERT HOVENKAMP amp SHELDON F KURTZ PRINCIPLES OF PROPERTY LAW 199ndash200 (6th ed 2005) A class member may demand possession of his or her share of the remainder upon termination of the income interest only when the class memberrsquos interest is not otherwise subject to a condition precedent See id

When a beneficiary timely disclaims an interest in a trust that beneficiary is treated as if he had predeceased the testator Here had the son disclaimed within nine months of the testatorrsquos death as required by the state statute he would have been deemed to have predeceased the testator This would have closed the class of remaindermen and the testatorrsquos then-living grandchildren (ie the daughterrsquos child) would have been entitled to the trust principal However under the state statute the sonrsquos disclaimer was not timely because he did not disclaim within nine months of the testatorrsquos death Thus because the statute is inapplicable and the son is still alive the class of grandchildren entitled to share in trust principal did not close

Because here the statute is inapplicable due to the sonrsquos failure to comply with the statutory time requirements then presumably the common-law rule allowing disclaimers (aka renunciations) at any time should apply Under the common law if a life estate is renounced the remainder interest accelerates and becomes immediately distributable to the remaindermen of the trust if the remainder is vested but not if the remainder is contingent JESSE DUKEMINIER amp ROBERT H SITKOFF WILLS TRUSTS AND ESTATES 844ndash845 (9th ed 2013) Here because the remainder is contingent upon there being grandchildren who survive the son the remainder will not accelerate It will remain open until the son dies leaving open the possibility that additional grandchildren will be included in the class or the daughterrsquos child could fall out of the class because that child fails to survive the son

And if none of the testatorrsquos grandchildren survive the son the trust principal will be distributed to the testatorrsquos heirs living at the sonrsquos death

15

Trusts and Future Interests Analysis

Point Two(b) (10) Until the trust terminates the trustee must continue to hold the trust assets The distribution of income in the meantime is unclear There are at least three possibilities Income earned on the undistributed assets could be distributed to the son and daughter as the testatorrsquos heirs accumulated and added to principal for distribution to the ultimate remaindermen or distributed from time to time to those persons who are presumptively remaindermen

When trust principal is not immediately distributable the trustee must continue to hold trust assets until the ultimate remaindermen are ascertained During this period trust income will be distributed or retained according to any instructions contained in the trust instrument See WILLIAM M MCGOVERN JR SHELDON F KURTZ amp DAVID M ENGLISH WILLS TRUSTS amp ESTATES sect 102 (4th ed 2010)

Here the testator did not specify what the trustee should do with trust income in the event the sonrsquos disclaimer did not comply with the state statute There are at least three approaches One approach would have the trustee distribute the trust income to the testatorrsquos heirs on the theory that the income represents property that was not disposed of by the testatorrsquos will and which thus passes by partial intestacy to the testatorrsquos heirs A second approach would have the trustee accumulate trust income for distribution to the ultimate remaindermen Under this approach only those individuals ultimately entitled to the principal would share in the income A third approach would have the trustee distribute trust income to those individuals who would be the remaindermen if the trust were to terminate when the income is received by the trustee under this approach trust income would be distributed to the daughterrsquos minor child until another presumptive remainderman is born This approach could result in individuals not ultimately entitled to principal say because they do not survive the son receiving income It could also result in a disproportionate distribution of income among the individuals ultimately entitled to income

[NOTE Examinees should demonstrate a recognition and understanding of the income-allocation problem and the alternatives available to address that issue There is no widely accepted solution to the problem Examinees who cite any of these possible problem-solving approaches may receive credit]

16

SECURED TRANSACTIONS ANALYSIS (Secured Transactions IB IID E amp F IIIB IVA B amp F)

ANALYSIS

Legal Problems

(1)(a) What is the nature of the bankrsquos claim to the businessrsquos equipment

(1)(b) What is the nature of the finance companyrsquos claim to the businessrsquos equipment

(1)(c) As between the bank and the finance company whose claim to the businessrsquos equipment has priority

(2) Do the claims of the bank and the finance company continue in the item of equipment sold by the business to the competitor

DISCUSSION

Summary

The bank and the finance company both have perfected security interests in the businessrsquos equipment Even though the finance companyrsquos perfected security interest was created first the bankrsquos perfected security interest has priority because the bankrsquos financing statement was filed before the finance companyrsquos financing statement The security interests of the bank and the finance company continue in the item of equipment sold by the business to the competitor because their security interests were perfected and the competitor was not a buyer in ordinary course of business

Point One(a) (25) The bank has a perfected security interest in the businessrsquos equipment

The bank has met all criteria necessary for it to have an attached and enforceable security interest in the businessrsquos equipment First value must be given UCC sect 9-203(b)(1) This criterion is fulfilled by the loan made by the bank to the business Second the debtor must have rights in the collateral UCC sect 9-203(b)(2) Clearly the business has rights in its equipment Third either the secured party must take possession of the collateral or the debtor must authenticate a security agreement containing a description of the collateral UCC sect 9-203(b)(3) The agreement that the business owner signed is a ldquosecurity agreementrdquo because it is an agreement that creates or provides for a security interest UCC sect 9-102(a)(74) By signing the security agreement the business owner authenticated it UCC sect 9-102(a)(7) Therefore all three criteria are fulfilled and the bank has an enforceable and attached security interest

A security interest is perfected when it has attached and when any additional steps required for perfection have occurred UCC sect 9-308(a) Generally speaking the additional steps will either be possession of the collateral by the secured party or the filing of a financing statement with respect to the collateral See UCC sectsect 9-310 9-313 In this case the bank filed a financing statement naming the debtor and sufficiently indicating the collateral The collateral indication is sufficient because it identifies the collateral by type of property See UCC sectsect 9-504 9-108 The fact that the financing statement was filed before the security interest was created is

17

Secured Transactions Analysis

not a problem Even though the security agreement had not yet been signed the business had authorized the filing of the financing statement in an authenticated record UCC sect 9-509(a)(1) Moreover the financing statement may be filed before the security agreement is created UCC sect 9-502(d)

Point One(b) (10) The finance company also has a perfected security interest in the businessrsquos equipment

The finance companyrsquos security interest is enforceable and attached for the same reasons as the bankrsquos security interest The loan from the finance company to the business constitutes value the business has rights in the collateral and the business owner has authenticated a security agreement containing a description of the collateral The finance companyrsquos security interest is perfected because the finance company filed a financing statement with respect to it that provides that the business is the debtor and indicates that the collateral is equipment

Point One(c) (30) The bankrsquos security interest has priority over the finance companyrsquos security interest because the bankrsquos financing statement was filed first

As between two perfected security interests the general rule is that the security interest that was the earlier to be either perfected or the subject of a filed financing statement has priority UCC sect 9-322(a)(1) While the finance companyrsquos security interest was perfected before the bankrsquos (March 15 vs March 22) the bankrsquos financing statement was filed even earlier on March 2 Thus under the first-to-file-or-perfect rule of UCC sect 9-322(a)(1) the bankrsquos security interest has priority No exceptions to the general rule apply here

Point Two (35) A security interest in collateral continues notwithstanding its sale unless an exception applies Because the security interests of the bank and the finance company were perfected and the competitor was not a buyer in ordinary course of business no exception applies and the security interests of both creditors continue in the equipment sold to the competitor

As a general rule a security interest in collateral continues notwithstanding the fact that the debtor has sold the collateral to another person UCC sect 9-315(a)(1) Thus unless an exception applies the security interests of the bank and the finance company will continue in the item of equipment sold to the competitor

A buyer of goods will take free of an unperfected security interest in those goods See UCC sect 9-317(a)(2) However when the competitor bought the businessrsquos equipment both the bank and the finance company had perfected security interests in the equipment

A buyer can take free even of a perfected security interest in goods if the buyer is a ldquobuyer in ordinary course of businessrdquo See UCC sect 9-320(a) However the competitor was not a buyer in ordinary course of business To be a ldquobuyer in ordinary course of businessrdquo a buyer must buy goods from a seller that is in the business of selling goods of that kind See UCC sect 1-201(b)(9) The competitor bought this equipment from a seller that is not in the business of selling goods of this kind so the competitor was not a buyer in ordinary course of business with respect to these goods

Because no exception applies the security interests of the bank and the finance company continue even after the item of equipment was sold to the competitor

18

FEDERAL CIVIL PROCEDURE ANALYSIS (Federal Civil Procedure IVD)

ANALYSIS

Legal Problems

(1) Is a document prepared in the course of a contract dispute protected from discovery as ldquowork productrdquo when there is no evidence that the document was prepared in anticipation of litigation

(2)(a) Is a partyrsquos failure to provide relevant electronically stored information excused when the information was destroyed pursuant to a routine document retention scheme at a time when litigation was contemplated by the destroying party

(2)(b) What sanctions should be imposed on a party for allowing the destruction of evidence that is relevant to potential future litigation

DISCUSSION

Summary

The report prepared by the structural engineer is probably not work product and is thus discoverable The engineer examined the foundation of the house at the customerrsquos request and the engineerrsquos findings are potentially relevant to the customerrsquos claim that the foundation is defective The report was not prepared in anticipation of litigation The customer appears to have sought the engineerrsquos opinion in response to the builderrsquos offer to fix any problems with the foundation that an engineer might identify Because the report was not prepared in anticipation of litigation it is not protected by the work-product doctrine

The builder should have taken appropriate steps to preserve evidence including suspending its document retention program as soon as it began planning for litigationmdashie on July 10 Its destruction of potentially relevant material after that date was wrongful However a court is unlikely to impose severe sanctions on the builder because there are no facts indicating that the builder acted in bad faith and the customer can prove that the foundation is defective without the destroyed emails

Point One (40) The customer must turn over the engineerrsquos report because it was not prepared in anticipation of litigation

In general a party to a lawsuit in federal court ldquomay obtain discovery regarding any nonprivileged matter that is relevant to any partyrsquos claim or defenserdquo FED R CIV P 26(b)(1) (2009) This includes the right to inspect and copy documents in the other partyrsquos possession FED R CIV P 34(a)(1) Here the customer hired a structural engineer to examine the foundation of the house The engineerrsquos report on the foundation is likely to include information that would be relevant to the customerrsquos claim that the foundation was defectively constructed

The so-called ldquowork productrdquo rule allows a party to refuse to turn over ldquodocuments that are prepared in anticipation of litigation or for trialrdquo by that partyrsquos representative including

19

Federal Civil Procedure Analysis

a consultant Thus if the customer had hired the structural engineer to prepare a report ldquoin anticipation of litigationrdquo that report might not be discoverable See FED R CIV P 26(b)(3)

In this case however the customer hired the engineer to evaluate the foundation of the house as part of the customerrsquos negotiation with the builder concerning the housersquos flooding problem The builder told the customer that the housersquos landscaping was the reason for the flooding and the builder told the customer ldquoHave an engineer look at the foundation If therersquos a problem wersquoll fix itrdquo The customer appears to have acted in response to that statement There is no indication that the customer anticipated any kind of legal action at the time that the structural engineer was hired Accordingly the structural engineerrsquos report is discoverable and the court should order the customer to turn it over

[NOTE If an examinee concludes that the structural engineerrsquos report was prepared in anticipation of litigation then the examinee should also conclude that the report is not discoverable Documents prepared in anticipation of litigation do not need to be disclosed to an adverse party unless that party can demonstrate a ldquosubstantial needrdquo for the documents and an inability to obtain substantially equivalent information without ldquoundue hardshiprdquo FED R CIV P 26(b)(3)(A)(ii) Furthermore a report prepared by an expert who is not expected to testify is not discoverable in the absence of ldquoexceptional circumstancesrdquo making it ldquoimpracticablerdquo to obtain the information in another way FED R CIV P 26(b)(4)(D)(ii) The builder probably cannot make these showings here unless the engineerrsquos report deals with circumstances that have since changed There is no evidence that the structural engineer would have had access to any information or facts that the builder would not already know as a result of its construction and subsequent inspection of the house In addition if necessary the builder could ask the court for permission to arrange for a further inspection of the house by a structural engineer hired by the builder See FED R CIV P 34(a)(2) Accordingly if an examinee concludes that the report was prepared in anticipation of litigation the examinee should also conclude that the builder is not entitled to see the report]

Point Two(a) (30) Because the builder anticipated that it might be involved in litigation concerning its contract with the customer the builder acted wrongfully in destroying emails that were relevant to the housersquos construction even though the emails were destroyed pursuant to a routine document retention plan

As noted above a party to a lawsuit in federal court ldquomay obtain discovery regarding any nonprivileged matter that is relevant to any partyrsquos claim or defenserdquo FED R CIV P 26(b)(1) This includes emails and other electronically stored information FED R CIV P 34(a)(1)(A) Here the customer has requested all the builderrsquos emails pertaining to work done on the foundation of the house Ordinarily the builder would be obliged to turn over this information which is relevant to the customerrsquos defense that the housersquos foundation was poorly constructed

Unfortunately the emails in question no longer exist because the builder destroyed them on August 2

In general spoliation of evidence (destruction or alteration of evidence) is improper if the party who destroyed or altered the evidence ldquohas notice that the evidence is relevant to litigation or should have known that the evidence may be relevant to future litigationrdquo Fujitsu Ltd v Federal Express Corp 247 F3d 423 436 (2d Cir 2001) It is improper for a party to destroy electronic information relevant to pending litigation even if the destruction occurs before there is any request or order seeking the information See eg Leon v IDX Sys Corp 464 F3d 951 (9th Cir 2006) (plaintiffrsquos intentional destruction of computer files warranted dismissal even

20

In this case the builderrsquos destruction of the emails was pursuant to a routine document retention plan The Federal Rules provide expressly that in the absence of ldquoexceptional circumstancesrdquo parties should not be sanctioned for the loss of electronically stored information when the loss occurs pursuant to ldquoroutine good-faith operation of an electronic information systemrdquo FED R CIV P 37(e) However when a party anticipates litigation ldquoit must suspend its routine document retentiondestruction policy and put in place a lsquolitigation holdrsquo to ensure the preservation of relevant documentsrdquo Zubulake v UBS Warburg LLC 220 FRD 212 218 (SDNY 2003)

Federal Civil Procedure Analysis

though spoliation occurred before order compelling discovery) Similarly the duty to preserve evidence applies to a party who anticipates litigation even if litigation has not yet been commenced See THE SEDONA PRINCIPLES BEST PRACTICES RECOMMENDATIONS amp PRINCIPLES FOR ADDRESSING ELECTRONIC DOCUMENT PRODUCTION 70 cmt 14a (2d ed 2007)

The builder destroyed the emails on August 2 At that time the builder knew that litigation was a possibility because the builder had already directed its attorney to prepare a draft complaint for possible filing Knowing that litigation was a possibility the builder had a duty to take steps to preserve evidence including the emails in question See generally Fujitsu Ltd

Thus the builderrsquos destruction of potentially relevant emails at a time when it knew that litigation was a possibility was improper It had a duty to preserve evidence and it breached that duty

[NOTE Because courts have used different words to describe the test for when evidence must be preserved an examineersquos precise formulation of the test is not critical]

Point Two(b) (30) In determining appropriate sanctions for spoliation courts consider both the level of culpability of the spoliating party and the degree of prejudice the loss of evidence has caused the other party Here the builderrsquos destruction of evidence does not appear to have been willful nor is it likely to pose a significant obstacle to the customerrsquos defense Any sanctions imposed by the court should be modest

Federal courts have inherent power to control the litigation process and can sanction misbehavior including spoliation even when there has been no specific violation of the Federal Rules of Civil Procedure See generally Chambers v NASCO Inc 501 US 32 (1991) (discussing courtrsquos inherent power to control the litigation process) The range of available sanctions is broad It can include such sanctions as the payment of expenses incurred by the other party as a result of the destruction of the evidence an instruction to the jury authorizing it to draw an adverse inference from the destruction of the evidence a shifting of the burden of proof on the relevant issue or even judgment against the responsible party See eg Residential Funding Corp v DeGeorge Financial Corp 306 F3d 99 108 (2d Cir 2002) (adverse inference) Silvestri v General Motors Corp 271 F3d 583 593 (4th Cir 2001) (possibility of dismissal) Cf FED R CIV P 37(b)(2)(A) (listing remedies for failure to comply with discovery obligations)

In determining appropriate sanctions for spoliation courts consider both the level of culpability of the spoliating party and the degree of prejudice the loss of evidence has caused the other party Many courts impose severe sanctions (such as an adverse-inference instruction or the entry of judgment against the spoliating party) only when there is evidence of bad faith in the form of an intentional effort to hide information Eg Greyhound Lines Inc v Wade 485 F3d 1032 1035 (8th Cir 2007) (spoliation sanction requires intentional destruction out of desire ldquoto suppress the truthrdquo) However other courts have said that negligence in preserving evidence can

21

Federal Civil Procedure Analysis

support an adverse-inference instruction See Residential Funding 306 F3d at 108 (negligence enough under some circumstances)

Although a court might well order an evidentiary hearing on the issue of sanctions the facts presented do not seem appropriate for severe sanctions First the evidence was destroyed pursuant to the builderrsquos standard document retention plan and there is no evidence that the builder deliberately failed to suspend its usual procedures with the purpose of allowing the destruction of evidence Second the loss of this evidence will not severely hinder the customerrsquos presentation of his case The central issue is whether the foundation of the house was properly constructed If the construction job was poorly done the customer can present evidence derived from inspection of the premises to prove that point The customer can also depose witnesses about any issues that arose during construction

Under the circumstances a court is not likely to impose particularly severe sanctions although it might shift the burden to the builder to show that the foundation was properly constructed or it might require the builder to reimburse any expenses the customer incurs to discover and prove the facts about issues or disputes that arose during construction of the foundation

[NOTE The result reached by the examinee is less important than the examineersquos recognition that (a) a range of sanctions is available to the court and (b) the appropriate sanction depends both on the culpability of the builder and the prejudice suffered by the customer]

22

CRIMINAL LAW AND PROCEDURE ANALYSIS (Criminal Law and Procedure IIA amp D VE amp F)

ANALYSIS

Legal Problems

(1) Did charging the defendant with both theft and burglary constitute double jeopardy

(2) Did the jury instruction violate the due process clause either by relieving the prosecution of the burden of proving the element of intent or by shifting the burden to the defendant to disprove that element

(3) Did the sentence imposed in this case for the theft conviction unconstitutionally deprive the defendant of his right to a jury trial on the issue of the value of the stolen item

DISCUSSION

Summary

The trial court properly denied the defendantrsquos pretrial motion to dismiss the charges on double jeopardy grounds The defendant may be charged with and convicted of both theft and burglary Each of the charges has an element that the other does not Neither charge is a lesser-included offense nor are they multiplicitous Thus charging both theft and burglary does not violate double jeopardy

The jury instruction on the burglary charge was constitutionally flawed It could have been reasonably understood by the jury as either (1) an irrebuttable conclusive presumption (which relieved the prosecution of proving the element of intent and removed the issue from the jury) or (2) a rebuttable mandatory presumption (which unconstitutionally shifted the burden of proof on an element of a charged offense from the prosecution to the defendant)

Because the four-year sentence imposed by the judge was based on the judgersquos finding by a preponderance of the evidence that the value of the stolen ring exceeded $5000 the sentence violates the defendantrsquos right to a jury determination beyond a reasonable doubt of the value of the ring

Point One (30) Charging the defendant with theft and burglary did not constitute double jeopardy

The Double Jeopardy Clause of the Fifth Amendment provides that a person shall not be twice put in jeopardy for the ldquosame offenserdquo Thus the question is whether the elements of the theft charge are wholly contained in the burglary charge or vice versa If the elements of the lesser charge (theft) are not wholly contained in the greater charge (burglary)mdashie if each charge requires proof of a fact that the other does notmdashthen convicting the defendant of both crimes would not violate double jeopardy even when the two offenses occurred at the same time and are thus arguably part of the ldquosame transactionrdquo Blockburger v United States 284 US 299 304 (1932) See also Albernaz v United States 450 US 333 344 n3 (1981) United States v Dixon 509 US 688 704 (1993)

23

Criminal Law and Procedure Analysis

Here theft and burglary each require proof of an element not required for the other crime Burglary may be defined differently in different jurisdictions However it almost invariably requires entry into a building or dwelling of another with the specific intent to commit a felony therein and the crime of burglary is complete upon the entry into the building or dwelling with such intent See eg Cannon v Oklahoma 827 P2d 1339 1342 (Okla Crim App 1992) In contrast theft which also may be defined differently in different states almost invariably requires the taking and carrying away of an item of personal property belonging to another with the intent to steal or permanently deprive the owner of possession

Here the ldquotakingrdquo or ldquostealingrdquo element is not contained in the definition of burglary and the ldquoentryrdquo element of burglary is not contained in the definition of theft Because theft is not a lesser-included offense of burglary and burglary is not a lesser-included offense of theft charging the defendant for both burglary and theft did not violate double jeopardy and the court properly denied the defense motion on those grounds Yparrea v Dorsey 64 F3d 577 579ndash80 (10th Cir 1995) citing Blockburger 284 US at 304

Finally the defendantrsquos motion to dismiss all the charges on double jeopardy grounds was improper because if both charges were for the same offense the motion should have requested dismissal of one charge not both

Point Two (35) The jury instruction on the burglary charge violated the Due Process Clause because it created either (1) an irrebuttable conclusive presumption (which relieved the prosecution of proving the element of intent and removed that issue from the jury) or (2) a rebuttable mandatory presumption (which unconstitutionally shifted the burden of proof on an element of a charged offense to the defendant)

The Supreme Court has interpreted the Due Process Clause of the US Constitution to require that the prosecution prove all elements of an offense beyond a reasonable doubt See In re Winship 397 US 358 364 (1970) The burden of proof cannot be shifted to the defendant by presuming an essential element upon proof of other elements of the offense because shifting the burden of persuasion with respect to any element of a criminal offense is contrary to the Due Process Clause See Mullaney v Wilbur 421 US 684 (1975)

The crime of burglary includes entry into a building or dwelling with the specific intent to commit a felony therein The requirement that the prosecutor prove beyond a reasonable doubt that the defendant had this specific intent distinguishes burglary from general-intent crimes like trespass See Sandstrom v Montana 442 US 510 523 (1979)

Here the jury was instructed that if ldquoafter consideration of all the evidence presented by the prosecution and defense you find beyond a reasonable doubt that the defendant entered the dwelling without the ownersrsquo consent you may presume that the defendant entered with the intent to commit a felony thereinrdquo This instruction was unconstitutional because it created either an irrebuttable conclusive presumption or a rebuttable mandatory presumption

A conclusive presumption is ldquoan irrebuttable direction by the court to find intent once convinced of the facts triggering the presumptionrdquo Id at 517 Here the jurors were instructed that once the prosecutor established that the defendant entered the neighborsrsquo house without consent they ldquomay presumerdquo that he intended to commit a felony therein The jurors may have reasonably concluded from this instruction that if they found that the defendant intended to enter his neighborsrsquo home without permission they must further find that he entered with the specific intent to commit a felony therein Because this instruction could operate as a conclusive

24

Criminal Law and Procedure Analysis

irrebuttable presumption by eliminating intent ldquoas an ingredient of the offenserdquo it violated due process by relieving the prosecution of the burden of proof for this element Id at 522

In the alternative the jury instruction could have been reasonably understood to create a rebuttable mandatory presumption which ldquotells [the jury] they must find the elemental fact upon proof of the basic fact at least unless the defendant has come forward with some evidence to rebut the presumed connection between the two factsrdquo County Court of Ulster County New York v Allen 442 US 140 157 (1979) The due process problem created by rebuttable mandatory presumptions is that ldquo[t]o the extent that the trier of fact is forced to abide by the presumption and may not reject it based on an independent evaluation of the particular facts presented by the State the analysis of the presumptionrsquos constitutional validity is logically divorced from those facts and based on the presumptionrsquos accuracy in the run of casesrdquo Id at 159

Unlike irrebuttable conclusive presumptions rebuttable mandatory presumptions are not always per se violations of the Due Process Clause However the Supreme Court of the United States has held that jury instructions that could reasonably be understood as shifting the burden of proof to the defendant on an element of the offense are unconstitutional Francis v Franklin 471 US 307 (1985) Here the argument that the jury instruction operated as a rebuttable mandatory presumption is supported by the fact that the judge also instructed the jury to ldquoconsider[ ] all the evidence presented by the prosecution and defenserdquo However even if the instruction created a rebuttable mandatory presumption it would be unconstitutional because it shifted the burden to the defense on an element of the offense Sandstrom 442 US at 524 Mullaney 421 US at 686

[NOTE Whether an examinee identifies the jury instruction as containing a ldquoconclusiverdquo or ldquomandatoryrdquo presumption is less important than the examineersquos analysis of the constitutional infirmities]

Point Three (35) The trial court violated the defendantrsquos Sixth Amendment right to a jury trial on an essential element of the offense when it found by a preponderance of the evidence that the ring was worth over $5000 and increased the defendantrsquos sentence based on this finding

In the statutory scheme under which the defendant was tried and convicted a Class D felony theft is defined as theft of item(s) with a value between $2500 and $10000 The jury found that the value of the diamond ring was at least $2500 and convicted the defendant of felony theft However at sentencing the trial court made a separate finding by a preponderance of the evidence that the value of the ring was greater than $5000 Following the statutersquos two-tiered sentencing scheme the judge then imposed on the defendant a sentence that was one year longer than the maximum that would otherwise have been allowed

The judgersquos sentence was unconstitutional because it violated the defendantrsquos Sixth Amendment right to a jury trial on this question The Supreme Court held in Apprendi v New Jersey 530 US 466 (2000) that ldquo[o]ther than the fact of a prior conviction any fact that increases the penalty for a crime beyond the prescribed statutory maximum must be submitted to a jury and proved beyond a reasonable doubtrdquo because ldquo[i]t is unconstitutional for a legislature to remove from the jury the assessment of facts that increase the prescribed range of penalties to which a criminal defendant is exposed [because] such facts must be established by proof beyond a reasonable doubtrdquo Id The Court reaffirmed Apprendi in Blakely v Washington 542 US 296 (2004) holding that the ldquolsquostatutory maximumrsquo for Apprendi purposes is the maximum sentence a judge may impose solely on the basis of the facts reflected in the jury verdict or admitted by the defendantrdquo Id at 303 (emphasis in original) In United States v Booker 543 US 220 (2005)

25

Criminal Law and Procedure Analysis

the Court relied on Blakely and Apprendi to conclude that protecting a defendantrsquos Sixth Amendment right to a jury trial required that ldquo[a]ny fact which is necessary to support a sentence exceeding the maximum authorized by the facts established by a plea of guilty or a jury verdict must be admitted by the defendant or proved to a jury beyond a reasonable doubtrdquo Id at 244

Thus in order to constitutionally increase a sentence above the statutory maximum of three years the jury must have found beyond a reasonable doubt that the value of the ring exceeded $5000 Here the court made the finding based on an appraisal proffered by the prosecutor only at sentencing and the judgersquos finding was by a preponderance of the evidence rather than beyond a reasonable doubt

26

AGENCY AND PARTNERSHIP ANALYSIS __________ (Agency and Partnership VA amp C VI)

ANALYSIS

Legal Problems

(1) Is a partner in a general partnership personally liable on a claim arising from misrepresentations by another partner made in the course of the partnership business

(2) Does a newly admitted partner in a general partnership become personally liable on existing claims against the partnership

(3) After the filing by a general partnership of a statement of qualification as a limited liability partnership are the partners personally liable as partners on (a) an existing claim against the general partnership and (b) a claim against the partnership that arose after the filing

DISCUSSION

Summary

Adam and Ben formed a general partnership under which they were jointly and severally liable for obligations of the partnership Thus Adam was personally liable for misrepresentations by Ben made in the ordinary course of the partnership business

Upon joining the general partnership Diane became personally liable for the obligations of the partnership arising after her admission but not for obligations pre-existing her admission such as the collectorrsquos claim

By filing a statement of qualification the three partners properly elected limited liability partnership status As partners in an LLP none of the three partners is personally liable as a partner for partnership obligations arising after the election such as the claim by the driverrsquos estate The election however does not change their personal liability on pre-existing claims that arose before the election such as the collectorrsquos claim

Point One (30) As a general partner of Empire a general partnership Adam became personally liable on the collectorrsquos claim a valid claim against the partnership that arose because of Benrsquos wrongful act in the ordinary course of the partnership business

When the collectorrsquos claim arose Empire was a general partnership composed of Adam and Ben Under UPA (1997) sect 306(a) partners of a general partnership are liable jointly and severally for all obligations of the partnership Under UPA (1997) sect 305(a) the partnership could become obligated for the loss caused to the collector as a result of the misrepresentation by Ben provided he was acting in the ordinary course of the partnership business Because there was no statement that limited his partnership authority Ben as partner was ldquoan agent of the partnership for the purpose of its businessrdquo See UPA (1997) sect 301(1) Benrsquos misrepresentation to the collector even if intentional appears to be in the ordinary course of the partnershiprsquos business of dealing

27

Agency and Partnership Analysis

in antique cars Thus Benrsquos wrongful act created a partnership obligation for which Adam was jointly and severally liable

[NOTE Generally a partnership creditor must ldquoexhaust the partnershiprsquos assets before levying on a judgment debtor partnerrsquos individual property where the partner is personally liable for the partnership obligationrdquo as a result of his status as a partner UPA (1997) sect 307 cmt 4 As the UPA comments explain this places Adam more in the position of guarantor than principal debtor on the partnership obligation Id cmt 4 Although an examinee might discuss this point the call focuses on whether Adam is personally liable not how the liability might be enforced]

Point Two (30) Because the collectorrsquos claim arose before Diane joined Empire Diane did not become personally liable on the claim

Diane was admitted to Empire when it was a general partnership and after the collectorrsquos claim arose While the general rule under UPA (1997) sect 306(a) is that the partners of a general partnership are liable jointly and severally for all obligations of the partnership there is a special rule for partners who are admitted during the duration of the partnership Under UPA (1997) sect 306(b) a person admitted to an existing partnership is not personally liable for any partnership obligations incurred before the personrsquos admission Because Diane was admitted to Empire after the collectorrsquos claim arose Diane is not personally liable on the claim

Dianersquos knowledge of the pre-existing claim and her stated concern about becoming liable on the collectorrsquos claim do not change her personal nonliability to the collector Although partners who have a liability shield can assume liability to third parties through private contractual guarantees or modifications to the partnership agreement Dianersquos stated concern constituted neither a guaranty to the collector nor ldquoan intentional waiver of liability protectionsrdquo See UPA (1997) sect 306 cmt 3 (describing methods for waiver of liability protections under sect 306(c) applicable in limited liability partnerships)

At most Diane will lose her investment in the partnership as a result of the collectorrsquos claim Although Diane did not become personally liable on the collectorrsquos claim when she joined the partnership the $250000 she contributed to the partnership is ldquoat risk for the satisfaction of existing partnership debtsrdquo UPA (1997) sect 306 cmt 2

Point Three (40) Filing the statement of qualification was effective to elect limited liability partnership status Despite this new status Adam and Ben remain personally liable on the collectorrsquos claim which arose before the election But as partners in an LLP neither Adam Ben nor Diane is personally liable as a partner on the driverrsquos estatersquos claim which arose after the election

Under UPA (1997) sect 1001 a general partnership can make an election and become a limited liability partnershipmdashif the partners approve the conversion by a vote equivalent to that necessary to amend the partnership agreement and the partnership then files a statement of qualification that specifies the name of the partnership its principal office and its election to be an LLP Here the partners agreed unanimouslymdashsufficient to amend their agreement under UPA (1997) sect 401(j)mdashand the statement of qualification was filed In addition the name of Empire LLP properly included an appropriate ending ldquoLLPrdquo See UPA (1997) sect 1002

Although another way to effectuate a ldquoconversionrdquo (as suggested by Benrsquos lawyer) is to form a new LLP and transfer the assets of the old general partnership to the new LLP the

28

Agency and Partnership Analysis

method used here (approval by the partners and the filing of a statement of qualification) is also sufficient to create LLP status

Thus Empire became Empire LLP as of the date of filing of the statement of qualification See UPA (1997) sect 1001 What effect did this have on the collectorrsquos claim which predated the filing According to UPA (1997) sect 306(c) an obligation incurred while a partnership is an LLP is solely a partnership obligation As the collectorrsquos claim predated the LLP Adam and Ben remain personally liable on the collectorrsquos claim Diane on the other hand was not personally liable on the collectorrsquos claim either before or after the filing of the statement of qualification See Point Two above

The driverrsquos estatersquos claim arose after Empire became Empire LLP Under UPA (1997) sect 306(c) an obligation incurred while a partnership is an LLP is solely a partnership obligationThus Adam Ben and Diane as partners are all protected from personal liability on the driverrsquos estatersquos claim But there may be personal liability if any of them was negligent or otherwise acted wrongfully by not informing the buyer of the bad suspension that caused the accident

29

National Conference of Bar Examiners 302 South Bedford Street | Madison WI 53703-3622 Phone 608-280-8550 | Fax 608-280-8552 | TDD 608-661-1275

wwwncbexorg e-mail contactncbexorg

  • Preface
  • Description of the MEE
  • Instructions
  • February 2014 Questions
    • Constitutinal Law Question
    • Trusts and Future Interests Question
    • Secured Transactions Question
    • Federal Civil Procedure Question
    • Criminal Law and Procedure Question
    • Agency and Partnership Question
      • February 2014 Analyses
        • Constitutional Law Analysis
        • Trust and Future Interests Analysis
        • Secured Transactions Analysis
        • Federal Civil Procedure Analysis
        • Criminal Law and Procedure Analysis
        • Agency and Partnership Analysis
            • ltlt13 ASCII85EncodePages false13 AllowTransparency false13 AutoPositionEPSFiles true13 AutoRotatePages None13 Binding Left13 CalGrayProfile (Dot Gain 20)13 CalRGBProfile (sRGB IEC61966-21)13 CalCMYKProfile (US Web Coated 050SWOP051 v2)13 sRGBProfile (sRGB IEC61966-21)13 CannotEmbedFontPolicy Error13 CompatibilityLevel 1413 CompressObjects Tags13 CompressPages true13 ConvertImagesToIndexed true13 PassThroughJPEGImages true13 CreateJobTicket false13 DefaultRenderingIntent Default13 DetectBlends true13 DetectCurves 0000013 ColorConversionStrategy CMYK13 DoThumbnails false13 EmbedAllFonts true13 EmbedOpenType false13 ParseICCProfilesInComments true13 EmbedJobOptions true13 DSCReportingLevel 013 EmitDSCWarnings false13 EndPage -113 ImageMemory 104857613 LockDistillerParams false13 MaxSubsetPct 10013 Optimize true13 OPM 113 ParseDSCComments true13 ParseDSCCommentsForDocInfo true13 PreserveCopyPage true13 PreserveDICMYKValues true13 PreserveEPSInfo true13 PreserveFlatness true13 PreserveHalftoneInfo false13 PreserveOPIComments true13 PreserveOverprintSettings true13 StartPage 113 SubsetFonts true13 TransferFunctionInfo Apply13 UCRandBGInfo Preserve13 UsePrologue false13 ColorSettingsFile ()13 AlwaysEmbed [ true13 ]13 NeverEmbed [ true13 ]13 AntiAliasColorImages false13 CropColorImages true13 ColorImageMinResolution 30013 ColorImageMinResolutionPolicy OK13 DownsampleColorImages true13 ColorImageDownsampleType Bicubic13 ColorImageResolution 30013 ColorImageDepth -113 ColorImageMinDownsampleDepth 113 ColorImageDownsampleThreshold 15000013 EncodeColorImages true13 ColorImageFilter DCTEncode13 AutoFilterColorImages true13 ColorImageAutoFilterStrategy JPEG13 ColorACSImageDict ltlt13 QFactor 01513 HSamples [1 1 1 1] VSamples [1 1 1 1]13 gtgt13 ColorImageDict ltlt13 QFactor 01513 HSamples [1 1 1 1] VSamples [1 1 1 1]13 gtgt13 JPEG2000ColorACSImageDict ltlt13 TileWidth 25613 TileHeight 25613 Quality 3013 gtgt13 JPEG2000ColorImageDict ltlt13 TileWidth 25613 TileHeight 25613 Quality 3013 gtgt13 AntiAliasGrayImages false13 CropGrayImages true13 GrayImageMinResolution 30013 GrayImageMinResolutionPolicy OK13 DownsampleGrayImages true13 GrayImageDownsampleType Bicubic13 GrayImageResolution 30013 GrayImageDepth -113 GrayImageMinDownsampleDepth 213 GrayImageDownsampleThreshold 15000013 EncodeGrayImages true13 GrayImageFilter DCTEncode13 AutoFilterGrayImages true13 GrayImageAutoFilterStrategy JPEG13 GrayACSImageDict ltlt13 QFactor 01513 HSamples [1 1 1 1] VSamples [1 1 1 1]13 gtgt13 GrayImageDict ltlt13 QFactor 01513 HSamples [1 1 1 1] VSamples [1 1 1 1]13 gtgt13 JPEG2000GrayACSImageDict ltlt13 TileWidth 25613 TileHeight 25613 Quality 3013 gtgt13 JPEG2000GrayImageDict ltlt13 TileWidth 25613 TileHeight 25613 Quality 3013 gtgt13 AntiAliasMonoImages false13 CropMonoImages true13 MonoImageMinResolution 120013 MonoImageMinResolutionPolicy OK13 DownsampleMonoImages true13 MonoImageDownsampleType Bicubic13 MonoImageResolution 120013 MonoImageDepth -113 MonoImageDownsampleThreshold 15000013 EncodeMonoImages true13 MonoImageFilter CCITTFaxEncode13 MonoImageDict ltlt13 K -113 gtgt13 AllowPSXObjects false13 CheckCompliance [13 None13 ]13 PDFX1aCheck false13 PDFX3Check false13 PDFXCompliantPDFOnly false13 PDFXNoTrimBoxError true13 PDFXTrimBoxToMediaBoxOffset [13 00000013 00000013 00000013 00000013 ]13 PDFXSetBleedBoxToMediaBox true13 PDFXBleedBoxToTrimBoxOffset [13 00000013 00000013 00000013 00000013 ]13 PDFXOutputIntentProfile ()13 PDFXOutputConditionIdentifier ()13 PDFXOutputCondition ()13 PDFXRegistryName ()13 PDFXTrapped False1313 CreateJDFFile false13 Description ltlt13 ARA 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 BGR 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 CHS ltFEFF4f7f75288fd94e9b8bbe5b9a521b5efa7684002000410064006f006200650020005000440046002065876863900275284e8e9ad88d2891cf76845370524d53705237300260a853ef4ee54f7f75280020004100630072006f0062006100740020548c002000410064006f00620065002000520065006100640065007200200035002e003000204ee553ca66f49ad87248672c676562535f00521b5efa768400200050004400460020658768633002gt13 CHT ltFEFF4f7f752890194e9b8a2d7f6e5efa7acb7684002000410064006f006200650020005000440046002065874ef69069752865bc9ad854c18cea76845370524d5370523786557406300260a853ef4ee54f7f75280020004100630072006f0062006100740020548c002000410064006f00620065002000520065006100640065007200200035002e003000204ee553ca66f49ad87248672c4f86958b555f5df25efa7acb76840020005000440046002065874ef63002gt13 CZE 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 DAN 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 DEU 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 ESP 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 ETI 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 FRA 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 GRE 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 HEB ltFEFF05D405E905EA05DE05E905D5002005D105D405D205D305E805D505EA002005D005DC05D4002005DB05D305D9002005DC05D905E605D505E8002005DE05E105DE05DB05D9002000410064006F006200650020005000440046002005D405DE05D505EA05D005DE05D905DD002005DC05D405D305E405E105EA002005E705D305DD002D05D305E405D505E1002005D005D905DB05D505EA05D905EA002E002005DE05E105DE05DB05D90020005000440046002005E905E005D505E605E805D5002005E005D905EA05E005D905DD002005DC05E405EA05D905D705D4002005D105D005DE05E605E205D505EA0020004100630072006F006200610074002005D5002D00410064006F00620065002000520065006100640065007200200035002E0030002005D505D205E805E105D005D505EA002005DE05EA05E705D305DE05D505EA002005D905D505EA05E8002E05D005DE05D905DD002005DC002D005000440046002F0058002D0033002C002005E205D905D905E005D5002005D105DE05D305E805D905DA002005DC05DE05E905EA05DE05E9002005E905DC0020004100630072006F006200610074002E002005DE05E105DE05DB05D90020005000440046002005E905E005D505E605E805D5002005E005D905EA05E005D905DD002005DC05E405EA05D905D705D4002005D105D005DE05E605E205D505EA0020004100630072006F006200610074002005D5002D00410064006F00620065002000520065006100640065007200200035002E0030002005D505D205E805E105D005D505EA002005DE05EA05E705D305DE05D505EA002005D905D505EA05E8002Egt13 HRV (Za stvaranje Adobe PDF dokumenata najpogodnijih za visokokvalitetni ispis prije tiskanja koristite ove postavke Stvoreni PDF dokumenti mogu se otvoriti Acrobat i Adobe Reader 50 i kasnijim verzijama)13 HUN 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 ITA 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 JPN ltFEFF9ad854c18cea306a30d730ea30d730ec30b951fa529b7528002000410064006f0062006500200050004400460020658766f8306e4f5c6210306b4f7f75283057307e305930023053306e8a2d5b9a30674f5c62103055308c305f0020005000440046002030d530a130a430eb306f3001004100630072006f0062006100740020304a30883073002000410064006f00620065002000520065006100640065007200200035002e003000204ee5964d3067958b304f30533068304c3067304d307e305930023053306e8a2d5b9a306b306f30d530a930f330c8306e57cb30818fbc307f304c5fc59808306730593002gt13 KOR ltFEFFc7740020c124c815c7440020c0acc6a9d558c5ec0020ace0d488c9c80020c2dcd5d80020c778c1c4c5d00020ac00c7a50020c801d569d55c002000410064006f0062006500200050004400460020bb38c11cb97c0020c791c131d569b2c8b2e4002e0020c774b807ac8c0020c791c131b41c00200050004400460020bb38c11cb2940020004100630072006f0062006100740020bc0f002000410064006f00620065002000520065006100640065007200200035002e00300020c774c0c1c5d0c11c0020c5f40020c2180020c788c2b5b2c8b2e4002egt13 LTH 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 LVI 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 NLD (Gebruik deze instellingen om Adobe PDF-documenten te maken die zijn geoptimaliseerd voor prepress-afdrukken van hoge kwaliteit De gemaakte PDF-documenten kunnen worden geopend met Acrobat en Adobe Reader 50 en hoger)13 NOR 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 POL ltFEFF0055007300740061007700690065006e0069006100200064006f002000740077006f0072007a0065006e0069006100200064006f006b0075006d0065006e007400f300770020005000440046002000700072007a0065007a006e00610063007a006f006e00790063006800200064006f002000770079006400720075006b00f30077002000770020007700790073006f006b00690065006a0020006a0061006b006f015b00630069002e002000200044006f006b0075006d0065006e0074007900200050004400460020006d006f017c006e00610020006f007400770069006500720061010700200077002000700072006f006700720061006d006900650020004100630072006f00620061007400200069002000410064006f00620065002000520065006100640065007200200035002e0030002000690020006e006f00770073007a0079006d002egt13 PTB 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 RUM ltFEFF005500740069006c0069007a00610163006900200061006300650073007400650020007300650074010300720069002000700065006e007400720075002000610020006300720065006100200064006f00630075006d0065006e00740065002000410064006f006200650020005000440046002000610064006500630076006100740065002000700065006e0074007200750020007400690070010300720069007200650061002000700072006500700072006500730073002000640065002000630061006c006900740061007400650020007300750070006500720069006f006100720103002e002000200044006f00630075006d0065006e00740065006c00650020005000440046002000630072006500610074006500200070006f00740020006600690020006400650073006300680069007300650020006300750020004100630072006f006200610074002c002000410064006f00620065002000520065006100640065007200200035002e00300020015f00690020007600650072007300690075006e0069006c006500200075006c0074006500720069006f006100720065002egt13 RUS 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 SKY ltFEFF0054006900650074006f0020006e006100730074006100760065006e0069006100200070006f0075017e0069007400650020006e00610020007600790074007600e100720061006e0069006500200064006f006b0075006d0065006e0074006f0076002000410064006f006200650020005000440046002c0020006b0074006f007200e90020007300610020006e0061006a006c0065007001610069006500200068006f0064006900610020006e00610020006b00760061006c00690074006e00fa00200074006c0061010d00200061002000700072006500700072006500730073002e00200056007900740076006f00720065006e00e900200064006f006b0075006d0065006e007400790020005000440046002000620075006400650020006d006f017e006e00e90020006f00740076006f00720069016500200076002000700072006f006700720061006d006f006300680020004100630072006f00620061007400200061002000410064006f00620065002000520065006100640065007200200035002e0030002000610020006e006f0076016100ed00630068002egt13 SLV 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 SUO 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 SVE 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 TUR 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 UKR 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 ENU (Use these settings to create Adobe PDF documents best suited for high-quality prepress printing Created PDF documents can be opened with Acrobat and Adobe Reader 50 and later)13 gtgt13 Namespace [13 (Adobe)13 (Common)13 (10)13 ]13 OtherNamespaces [13 ltlt13 AsReaderSpreads false13 CropImagesToFrames true13 ErrorControl WarnAndContinue13 FlattenerIgnoreSpreadOverrides false13 IncludeGuidesGrids false13 IncludeNonPrinting false13 IncludeSlug false13 Namespace [13 (Adobe)13 (InDesign)13 (40)13 ]13 OmitPlacedBitmaps false13 OmitPlacedEPS false13 OmitPlacedPDF false13 SimulateOverprint Legacy13 gtgt13 ltlt13 AddBleedMarks false13 AddColorBars false13 AddCropMarks false13 AddPageInfo false13 AddRegMarks false13 ConvertColors ConvertToCMYK13 DestinationProfileName ()13 DestinationProfileSelector DocumentCMYK13 Downsample16BitImages true13 FlattenerPreset ltlt13 PresetSelector MediumResolution13 gtgt13 FormElements false13 GenerateStructure false13 IncludeBookmarks false13 IncludeHyperlinks false13 IncludeInteractive false13 IncludeLayers false13 IncludeProfiles false13 MultimediaHandling UseObjectSettings13 Namespace [13 (Adobe)13 (CreativeSuite)13 (20)13 ]13 PDFXOutputIntentProfileSelector DocumentCMYK13 PreserveEditing true13 UntaggedCMYKHandling LeaveUntagged13 UntaggedRGBHandling UseDocumentProfile13 UseDocumentBleed false13 gtgt13 ]13gtgt setdistillerparams13ltlt13 HWResolution [2400 2400]13 PageSize [612000 792000]13gtgt setpagedevice13

Page 9: February 2014 MEE Questions and AnalysesPreface The Multistate Essay Examination (MEE) is developed by the National Conference of Bar Examiners (NCBE). This publication includes the

SECURED TRANSACTIONS QUESTION

On March 1 the owner of a manufacturing business entered into negotiations with a bank to obtain a loan of $100000 for the business The bank loan officer informed the business owner that the interest rate for a loan would be lower if the repayment obligation were secured by all the businessrsquos present and future equipment The loan officer also informed the business owner that the bank could not commit to making the loan until its credit investigation was completed but that funds could be advanced faster following loan approval if a financing statement with respect to the transaction were filed in advance Accordingly the business owner signed a form on behalf of the business authorizing the bank to file a financing statement with respect to the proposed transaction The bank properly filed a financing statement the next day correctly providing the name of the business as the debtor and indicating ldquoequipmentrdquo as the collateral

On March 15 the business owner had heard nothing from the bank about whether the loan had been approved so the business owner approached a finance company for a loan The finance company quickly agreed to lend $100000 to the business secured by all the businessrsquos present and future equipment That same day the finance company loaned to the business $100000 and the business owner signed an agreement obligating the business to repay the loan and granting the finance company a security interest in all the businessrsquos ldquopresent and future equipmentrdquo to secure the repayment obligation Also on that day the finance company properly filed a financing statement correctly providing the businessrsquos name as the debtor and indicating ldquoequipmentrdquo as the collateral

On March 21 the bank loan officer contacted the business owner and indicated that the loan application had been approved On the next day March 22 the bank loaned the business $100000 The loan agreement signed by the owner on behalf of the business granted the bank a security interest in all the businessrsquos ldquopresent and future equipmentrdquo

On April 10 the business sold an item of manufacturing equipment to a competitor for $20000 This was the first time the business had ever sold any of its equipment The competitor paid the purchase price in cash and took possession of the equipment that day The competitor acted in good faith at all times and had no knowledge of the businessrsquos prior transactions with the bank and the finance company

The business has defaulted on its obligations with respect to the loans from the bank and the finance company Each of them has asserted a claim to all the businessrsquos equipment as well as to the item of equipment sold to the businessrsquos competitor

Assume that the business owner had the authority to enter into all these transactions on behalf of the business

1 As between the bank and the finance company which has a superior claim to the businessrsquos equipment Explain

2 Do the claims of the bank and the finance company to the businessrsquos equipment continue in the item of equipment sold to the competitor Explain

5

FEDERAL CIVIL PROCEDURE QUESTION

A builder constructed a vacation house for an out-of-state customer on the customerrsquos land The house was completed on June 1 at which point the customer still owed $200000 of the $800000 contract price which was payable in full five days later

On June 14 the basement of the house was flooded with two inches of water during a heavy rainfall When the customer complained the builder told the customer ldquoThe flooding was caused by poorly designed landscaping Our work is fine and fully up to code Have an engineer look at the foundation If therersquos a problem wersquoll fix itrdquo

The customer pleased by the builderrsquos cooperative attitude immediately hired a structural engineer to examine the foundation of the house On June 30 the engineer provided the customer with a written report on the condition of the foundation which stated that the foundation was properly constructed

Unhappy with the conclusions in the engineerrsquos report the customer then hired a home inspector to evaluate the house The home inspectorrsquos report concluded that the foundation of the house had been poorly constructed and was inadequately waterproofed

On July 10 the customer sent the builder the home inspectorrsquos report with a note that said ldquoUntil you fix this problem you wonrsquot get another penny from merdquo The builder immediately contacted an attorney and directed the attorney to prepare a draft complaint against the customer for nonpayment Hoping to avoid litigation the builder sent several more requests for payment to the customer The customer ignored all these requests

On September 10 the builder filed suit in federal district court properly invoking the courtrsquos diversity jurisdiction and seeking $200000 in damages for breach of contract The customerrsquos answer denied liability on the basis of alleged defective construction of the housersquos foundation

Several months later the case is nearly ready for trial However two discovery disputes have not yet been resolved

First despite a request from the builder the customer has refused to provide a copy of the report prepared by the structural engineer who examined the foundation of the house The customer claims that the report is ldquowork productrdquo and not discoverable because the customer does not intend to ask the engineer to testify at trial The builder has asked the court to order the customer to turn over the engineerrsquos report

Second the customer has asked the court to impose sanctions for the builderrsquos failure to comply with the customerrsquos demand for copies of all emails concerning construction of the foundation of the house The builder has truthfully informed the customer that all such emails were destroyed on August 2 This destruction was pursuant to the builderrsquos standard practice of permanently deleting all project-related emails from company records 60 days after construction of a project is complete There is no relevant state records-retention law

1 Should the court order the customer to turn over the engineerrsquos report Explain

2 Should the court sanction the builder for the destruction of emails related to the case and if so what factors should the court consider in determining those sanctions Explain

6

CRIMINAL LAW AND PROCEDURE QUESTION _____

A defendant was charged under state law with felony theft (Class D) and felony residential burglary (Class C) The indictment alleged that the defendant entered his neighborsrsquo home without their consent and stole a diamond ring worth at least $2500

Defense counsel filed a pretrial motion to dismiss the charges on the ground that prosecuting the defendant for both burglary and theft would constitute double jeopardy The trial court denied the motion and the defendant was prosecuted for both crimes The only evidence of the ringrsquos value offered at the defendantrsquos jury trial was the ownerrsquos testimony that she had purchased the ring two years earlier for $3000

At trial the judge issued the following jury instruction on the burglary charge prior to deliberations

If after consideration of all the evidence presented by the prosecution and defense you find beyond a reasonable doubt that the defendant entered the dwelling without the ownersrsquo consent you may presume that the defendant entered with the intent to commit a felony therein

The jury found the defendant guilty of both offenses

At the defendantrsquos sentencing hearing an expert witness called by the prosecutor testified that the diamond ring was worth between $7000 and $8000 Over defense objection the judge concluded by a preponderance of the evidence that the value of the stolen ring exceeded $5000 The judge sentenced the defendant to four yearsrsquo incarceration on the theft conviction On the burglary conviction the defendant received a consecutive sentence of seven yearsrsquo incarceration

In this state residential burglary is defined as ldquoentry into the dwelling of another without the consent of the lawful resident with the intent to commit a felony thereinrdquo Residential burglary is a Class C felony for which the minimum sentence is five years and the maximum sentence is ten years of incarceration

In this state theft is defined as ldquotaking and carrying away the property of another with the intent to permanently deprive the owner of possessionrdquo Theft is a Class D felony if the value of the item(s) taken is between $2500 and $10000 The sentence for a Class D felony theft is determined by the value of the items taken If the value is between $2500 and $5000 the maximum sentence is three yearsrsquo incarceration If the value of the items exceeds $5000 the maximum sentence is five yearsrsquo incarceration

This state affords a criminal defendant no greater rights than those mandated by the United States Constitution

1 Did the trial court err when it denied the defendantrsquos pretrial motion to dismiss on double jeopardy grounds Explain

2 Did the trial court err in its instruction to the jury on the burglary charge Explain

3 Did the trial court err when it sentenced the defendant to an additional year of incarceration on the theft conviction based on the expertrsquos testimony Explain

7

AGENCY AND PARTNERSHIP QUESTION _____

Five years ago Adam and Ben formed a general partnership Empire Partnership (Empire) to buy and sell antique automobiles at a showroom in State A Adam contributed $800000 to Empire and Ben contributed $200000 Their written partnership agreement allocated 80 of profits losses and control to Adam and 20 to Ben No filings of any type were made in connection with the formation of Empire

Three years ago a collector purchased one of Empirersquos antique cars for $3400000 The collector was willing to pay this price because of Benrsquos false representation (repeated in the sales contract) that a famous movie star had once owned the car Without the movie-star connection the car was worth only $100000 One month later when the collector discovered the truth he sued Adam Ben and Empire for $3300000 in damages The lawsuit is still pending

Two years ago Adam and Ben admitted a new partner Diane to Empire in return for her contribution of $250000 The three agreed to allocate profits losses and control 75 to Adam 10 to Ben and 15 to Diane Before joining the partnership Diane learned of the collectorrsquos claim and stated her concern to Adam and Ben that she might become liable if the claim were reduced to a judgment

Following Dianersquos admission to Empire the three partners sought to convert Empire into a limited liability partnership (LLP) Adamrsquos lawyer proposed to file with State A a ldquostatement of qualificationrdquo making an LLP election and declaring the name of the partnership to be ldquoEmpire LLPrdquo Benrsquos lawyer stated that this would not work and that a new LLP had to be formed with the assets of the old partnership transferred to the new one In the end the conversion was done the way Adamrsquos lawyer suggested with the approval of all three partners

One year ago a driver purchased a vintage car from Empire LLP based on the representation that the car was ldquofully roadworthy and capable of touring at 70 mph all dayrdquo The driver took the car on the highway at 50 mph whereupon the front suspension collapsed resulting in a crash in which the car was destroyed and the driver killed The driverrsquos estate sued Adam Ben Diane and Empire LLP for $10000000 The lawsuit is still pending

Although profitable Empire LLP does not have resources sufficient to pay the collectorrsquos claim or the claim of the driverrsquos estate

Assume that the Uniform Partnership Act (1997) applies

1 Before the filing of the statement of qualification (a) was Adam personally liable on the collectorrsquos claim Explain (b) was Diane personally liable on the collectorrsquos claim Explain

2 After the filing of the statement of qualification was Adam Ben or Diane personally liable as a partner on (a) the collectorrsquos claim or (b) the driverrsquos estatersquos claim Explain

8

February 2014 MEE

ANALYSES Constitutional Law

Trusts and Future Interests Secured Transactions

Federal Civil Procedure Criminal Law and Procedure

Agency and Partnership

CONSTITUTIONAL LAW ANALYSIS (Constitutional Law IVD)

ANALYSIS

Legal Problems

(1) Is the city ordinance requirement that businesses install floodlights a taking

(2) Is conditioning the approval of a building permit on the grant of an easement to install surveillance equipment a taking of property

DISCUSSION

Summary

The ordinance requiring businesses to install floodlights is not a per se taking under Loretto because it does not force a private landowner to allow a third party to enter and place a physical object on the land Here the city ordinance requires the businessmdashnot a third partymdashto install the floodlights

The ordinance is likely not a regulatory taking under the Penn Central balancing test While the ordinance will impose a cost on business owners that cost may be offset by the expected increase in business due to the ordinance and the ordinance does not appear to interfere with the ownerrsquos primary use of the property as a restaurant

The permit condition however is likely an uncompensated taking of property While the condition has an essential nexus with the cityrsquos legitimate interest in promoting public safety the city has not made an individualized determination that the easement condition is roughly proportional to the possibility of increased crime due to the restaurantrsquos proposed addition Thus the permit condition likely violates the Fifth Amendment as applied to the states through the Fourteenth Amendment

Point One (50) The ordinance requiring that businesses install floodlights is not a per se taking under Loretto It is not a regulatory taking under the Penn Central balancing test because the cost of compliance with the ordinance may be offset by an expected increase in business and compliance does not interfere with the businessrsquos primary use of its property as a restaurant

The city ordinance requiring a business to install floodlights does not effect a per se taking of the sort described in Loretto v Teleprompter Manhattan CATV Corp 458 US 419 (1982) because no property is physically taken by the government and the ordinance does not involve a physical invasion of private property by a third party

Even though the ordinance does not constitute an occupation of the property by either the government or a third party it is still subject to the three-factor balancing test under Penn Central Transportation Co v City of New York 438 US 104 (1978) to determine whether it is a ldquoregulatory takingrdquo Under Penn Central a court must balance (1) ldquo[t]he economic impact of the regulation on the claimantrdquo (2) ldquothe extent to which the regulation has interfered with distinct investment-backed expectationsrdquo and (3) ldquothe character of the governmental actionrdquo Id at 124 Here each factor weighs against finding that the ordinance is a taking

11

Constitutional Law Analysis

First the ordinance requirement likely has a minimal economic impact on the restaurant Compliance with the ordinance is estimated to cost $1000 and the city has found that businesses will likely recoup that cost in increased sales Also because the ordinance does not interfere with the operation of the restaurant the owner may still earn a reasonable return on its investment in the property

Second the ordinance does not interfere with the businessrsquos investment-backed expectations As in Penn Central the challenged law does not interfere with the ownerrsquos ldquoprimary expectationrdquo for use of the propertymdashin Penn Central as a railroad terminal and here as a restaurant Further the ordinance does not prevent the restaurant from expanding to meet the changing business environment

Third the character of the government action does not weigh in favor of a taking While Penn Central does say that a ldquophysical invasionrdquo is more likely to pose a taking Loretto suggests that the Courtrsquos main concern is with physical invasions by third parties Also like the landmark law challenged in Penn Central the ordinance here ldquoadjust[s] the benefits and burdens of economic life to promote the common goodrdquo Id In Penn Central the landmark law restricted development of the railroad terminal to promote the common interest in preserving historic landmarks Here the ordinance requires the businesses to install floodlights to promote the common interest in crime prevention and public safety

Because the ordinance is clearly a valid exercise of the police power it satisfies the takings clausersquos public-use requirement Kelo v City of New London 545 US 469 (2005)

In sum all three factors weigh against finding a taking under the Penn Central balancing test

Point Two (50) The permit condition may be unconstitutional as an uncompensated taking of property because the city has not made an individualized determination that the easement condition is roughly proportional to the impact of the restaurantrsquos proposed addition

In Dolan v City of Tigard 512 US 374 (1994) the Supreme Court set forth the test for determining whether an exaction imposed by a government in exchange for a discretionary benefit conferred by the government such as a condition on the approval of a building permit in this case constitutes an uncompensated taking under the Fifth Amendment The exaction is not a taking if (1) there is an ldquoessential nexusrdquo between the ldquopublic need or burdenrdquo to which the proposed development contributes and ldquothe permit condition exacted by the cityrdquo id at 386 and (2) the government makes ldquosome sort of individualized determination that the required dedication is [roughly proportional] both in nature and extent to the impact of the proposed developmentrdquo Id at 391 see also Nollan v California Coastal Commission 483 US 825 (1987)

Here the city likely can meet the nexus requirement In Dolan the landowner sought to double the size of its business which would have increased traffic on nearby roadways In exchange for approving the development the city sought an easement for a bike and pedestrian path The Court found the required nexus between the easement and the cityrsquos ldquoattempt to reduce traffic congestion by providing for alternative means of transportationrdquo 512 US at 387 Here a similar nexus likely exists between the requested easement and the cityrsquos interest in crime prevention and public safety Increased patronage and economic activity at the restaurant might attract additional crime to the area and the requested easement to install surveillance equipment would attempt to address that increased crime

12

Constitutional Law Analysis

The exaction here however may fail the second prong of the Dolan testmdashthat the exaction be roughly proportional to the anticipated impact of the requested development As noted the city in Dolan claimed that a bike and pedestrian path was needed to offset the increase in traffic due to the proposed doubling of the business The Court explained that the government must demonstrate that the additional traffic reasonably was related to the requested exaction and that the government must ldquomake some effort to quantify its findings in support of the dedication for the pedestrianbicycle pathway beyond the conclusory statement that it could offset some of the traffic demand generatedrdquo Id at 395 Here the city did not carry its burden The city simply speculates that increased patronage of the restaurant ldquomightrdquo increase crime and that the surveillance equipment ldquomightrdquo alleviate this increased crime Because the city has not made ldquosome effort to quantify its findingsrdquo in support of the easement it has not shown that the burden of the easement is roughly proportional to the benefits thought to flow from it

Thus the exaction appears to be an uncompensated taking of property in violation of the Fifth Amendment as applied to the states through the Fourteenth Amendment

13

TRUSTS AND FUTURE INTERESTS ANALYSIS ____ (Trusts and Future Interests IE3 I5 IIIA amp B)

ANALYSIS

Legal Problems

(1) How should rents dividends and sales proceeds received by the trustee prior to receipt of the sonrsquos letter have been allocated between trust income and principal

(2)(a) Did the remainder interest in the trust accelerate and become immediately payable to the daughterrsquos minor child upon the trusteersquos receipt of the sonrsquos letter and if not how should the trustee handle the distribution of the principal in the future

(2)(b) Following the trusteersquos receipt of the sonrsquos letter how should the trustee distribute future receipts of income prior to the distribution of the principal

DISCUSSION

Summary

Prior to the trusteersquos receipt of the sonrsquos letter cash dividends and rents should have been allocated to trust income and were distributable to the son the income beneficiary of the trust sales proceeds and stock dividends should have been allocated to principal

Because the sonrsquos letter to the trustee did not result in a valid disclaimer under state law (having been made more than nine months after the testatorrsquos death) the son is not deemed to have predeceased the testator Because the son is still living the class gift to the testatorrsquos grandchildren who survive the son has not closed and is not possessory it will not become possessory until the son dies The daughterrsquos minor child being the testatorrsquos only living grandchild is not currently entitled to a distribution of trust principal Trust principal will instead be distributable upon the sonrsquos death to the testatorrsquos then-living grandchildren or if there are none to the testatorrsquos then-living heirs

As for future income the trustee should either distribute the trust income to the son and the daughter as the testatorrsquos heirs accumulate the income for future distribution to those individuals ultimately entitled to the trust principal or distribute it to those presumptively entitled to the principal upon the sonrsquos death ie the daughterrsquos minor child

Point One (45) Cash dividends and rents are allocable to income sales proceeds and stock dividends are allocable to principal Items allocable to income for the period prior to the sonrsquos attempted disclaimer were distributable to the son

Receipts earned during the administration of a trust are allocable either to income or to principal Almost all states have adopted the most recent or an earlier version of the Uniform Principal and Income Act (the Act) which specifies how such receipts should be allocated

Under the Act rents (UNIF PRIN amp INC ACT (2000) sect 405 UNIF PRIN amp INC ACT (1962) sect 3(a)(1)) and cash dividends received from a corporation (UNIF PRIN amp INC ACT (2000) sect 401(b) UNIF PRIN amp INC ACT (1962) sect 6(d)) are allocable to income and are distributable to the income beneficiary of the trust

14

Trusts and Future Interests Analysis Sales proceeds (UNIF PRIN amp INC ACT (2000) sect 404(2) UNIF PRIN amp INC ACT (1962)

sect 3(b)(1)) and dividends paid in the stock of the distributing corporation (UNIF PRIN amp INC ACT (2000) sect 401(c)(1) UNIF PRIN amp INC ACT (1962) sect 3(b)(4)) are allocable to principal and added to the principal of the trust

Here the cash dividends and office building rents should have been allocated to income and until the trustee received the sonrsquos letter should have been distributed to him as the sole income beneficiary of the trust The stock dividend and proceeds from the sale of the office building should have been allocated to principal and held by the trustee for future distribution to the ultimate remaindermen of the trust

[NOTE The 2000 Uniform Principal and Income Act has been adopted in Alabama Arkansas Colorado Connecticut the District of Columbia Hawaii Idaho Iowa Kentucky Missouri Montana Nebraska New Mexico North Dakota Oregon South Dakota Utah and West Virginia]

Point Two(a) (45) Because the son did not disclaim within nine months of the testatorrsquos death there is no valid disclaimer under state law Therefore the son is not deemed to have predeceased the testator Furthermore because of the express survivorship contingency in the will the remainder in the trust does not accelerate and become distributable until the son in fact dies When the son dies the trust principal will be distributable to the testatorrsquos then-living grandchildren or if none then to the testatorrsquos then-living heirs

When a trust remainder is given to a class the class closes (ie no new persons can join the class) when there is no outstanding income interest and at least one member of the class is then entitled to demand possession of his or her share of the remainder This principle is called the rule of convenience See generally HERBERT HOVENKAMP amp SHELDON F KURTZ PRINCIPLES OF PROPERTY LAW 199ndash200 (6th ed 2005) A class member may demand possession of his or her share of the remainder upon termination of the income interest only when the class memberrsquos interest is not otherwise subject to a condition precedent See id

When a beneficiary timely disclaims an interest in a trust that beneficiary is treated as if he had predeceased the testator Here had the son disclaimed within nine months of the testatorrsquos death as required by the state statute he would have been deemed to have predeceased the testator This would have closed the class of remaindermen and the testatorrsquos then-living grandchildren (ie the daughterrsquos child) would have been entitled to the trust principal However under the state statute the sonrsquos disclaimer was not timely because he did not disclaim within nine months of the testatorrsquos death Thus because the statute is inapplicable and the son is still alive the class of grandchildren entitled to share in trust principal did not close

Because here the statute is inapplicable due to the sonrsquos failure to comply with the statutory time requirements then presumably the common-law rule allowing disclaimers (aka renunciations) at any time should apply Under the common law if a life estate is renounced the remainder interest accelerates and becomes immediately distributable to the remaindermen of the trust if the remainder is vested but not if the remainder is contingent JESSE DUKEMINIER amp ROBERT H SITKOFF WILLS TRUSTS AND ESTATES 844ndash845 (9th ed 2013) Here because the remainder is contingent upon there being grandchildren who survive the son the remainder will not accelerate It will remain open until the son dies leaving open the possibility that additional grandchildren will be included in the class or the daughterrsquos child could fall out of the class because that child fails to survive the son

And if none of the testatorrsquos grandchildren survive the son the trust principal will be distributed to the testatorrsquos heirs living at the sonrsquos death

15

Trusts and Future Interests Analysis

Point Two(b) (10) Until the trust terminates the trustee must continue to hold the trust assets The distribution of income in the meantime is unclear There are at least three possibilities Income earned on the undistributed assets could be distributed to the son and daughter as the testatorrsquos heirs accumulated and added to principal for distribution to the ultimate remaindermen or distributed from time to time to those persons who are presumptively remaindermen

When trust principal is not immediately distributable the trustee must continue to hold trust assets until the ultimate remaindermen are ascertained During this period trust income will be distributed or retained according to any instructions contained in the trust instrument See WILLIAM M MCGOVERN JR SHELDON F KURTZ amp DAVID M ENGLISH WILLS TRUSTS amp ESTATES sect 102 (4th ed 2010)

Here the testator did not specify what the trustee should do with trust income in the event the sonrsquos disclaimer did not comply with the state statute There are at least three approaches One approach would have the trustee distribute the trust income to the testatorrsquos heirs on the theory that the income represents property that was not disposed of by the testatorrsquos will and which thus passes by partial intestacy to the testatorrsquos heirs A second approach would have the trustee accumulate trust income for distribution to the ultimate remaindermen Under this approach only those individuals ultimately entitled to the principal would share in the income A third approach would have the trustee distribute trust income to those individuals who would be the remaindermen if the trust were to terminate when the income is received by the trustee under this approach trust income would be distributed to the daughterrsquos minor child until another presumptive remainderman is born This approach could result in individuals not ultimately entitled to principal say because they do not survive the son receiving income It could also result in a disproportionate distribution of income among the individuals ultimately entitled to income

[NOTE Examinees should demonstrate a recognition and understanding of the income-allocation problem and the alternatives available to address that issue There is no widely accepted solution to the problem Examinees who cite any of these possible problem-solving approaches may receive credit]

16

SECURED TRANSACTIONS ANALYSIS (Secured Transactions IB IID E amp F IIIB IVA B amp F)

ANALYSIS

Legal Problems

(1)(a) What is the nature of the bankrsquos claim to the businessrsquos equipment

(1)(b) What is the nature of the finance companyrsquos claim to the businessrsquos equipment

(1)(c) As between the bank and the finance company whose claim to the businessrsquos equipment has priority

(2) Do the claims of the bank and the finance company continue in the item of equipment sold by the business to the competitor

DISCUSSION

Summary

The bank and the finance company both have perfected security interests in the businessrsquos equipment Even though the finance companyrsquos perfected security interest was created first the bankrsquos perfected security interest has priority because the bankrsquos financing statement was filed before the finance companyrsquos financing statement The security interests of the bank and the finance company continue in the item of equipment sold by the business to the competitor because their security interests were perfected and the competitor was not a buyer in ordinary course of business

Point One(a) (25) The bank has a perfected security interest in the businessrsquos equipment

The bank has met all criteria necessary for it to have an attached and enforceable security interest in the businessrsquos equipment First value must be given UCC sect 9-203(b)(1) This criterion is fulfilled by the loan made by the bank to the business Second the debtor must have rights in the collateral UCC sect 9-203(b)(2) Clearly the business has rights in its equipment Third either the secured party must take possession of the collateral or the debtor must authenticate a security agreement containing a description of the collateral UCC sect 9-203(b)(3) The agreement that the business owner signed is a ldquosecurity agreementrdquo because it is an agreement that creates or provides for a security interest UCC sect 9-102(a)(74) By signing the security agreement the business owner authenticated it UCC sect 9-102(a)(7) Therefore all three criteria are fulfilled and the bank has an enforceable and attached security interest

A security interest is perfected when it has attached and when any additional steps required for perfection have occurred UCC sect 9-308(a) Generally speaking the additional steps will either be possession of the collateral by the secured party or the filing of a financing statement with respect to the collateral See UCC sectsect 9-310 9-313 In this case the bank filed a financing statement naming the debtor and sufficiently indicating the collateral The collateral indication is sufficient because it identifies the collateral by type of property See UCC sectsect 9-504 9-108 The fact that the financing statement was filed before the security interest was created is

17

Secured Transactions Analysis

not a problem Even though the security agreement had not yet been signed the business had authorized the filing of the financing statement in an authenticated record UCC sect 9-509(a)(1) Moreover the financing statement may be filed before the security agreement is created UCC sect 9-502(d)

Point One(b) (10) The finance company also has a perfected security interest in the businessrsquos equipment

The finance companyrsquos security interest is enforceable and attached for the same reasons as the bankrsquos security interest The loan from the finance company to the business constitutes value the business has rights in the collateral and the business owner has authenticated a security agreement containing a description of the collateral The finance companyrsquos security interest is perfected because the finance company filed a financing statement with respect to it that provides that the business is the debtor and indicates that the collateral is equipment

Point One(c) (30) The bankrsquos security interest has priority over the finance companyrsquos security interest because the bankrsquos financing statement was filed first

As between two perfected security interests the general rule is that the security interest that was the earlier to be either perfected or the subject of a filed financing statement has priority UCC sect 9-322(a)(1) While the finance companyrsquos security interest was perfected before the bankrsquos (March 15 vs March 22) the bankrsquos financing statement was filed even earlier on March 2 Thus under the first-to-file-or-perfect rule of UCC sect 9-322(a)(1) the bankrsquos security interest has priority No exceptions to the general rule apply here

Point Two (35) A security interest in collateral continues notwithstanding its sale unless an exception applies Because the security interests of the bank and the finance company were perfected and the competitor was not a buyer in ordinary course of business no exception applies and the security interests of both creditors continue in the equipment sold to the competitor

As a general rule a security interest in collateral continues notwithstanding the fact that the debtor has sold the collateral to another person UCC sect 9-315(a)(1) Thus unless an exception applies the security interests of the bank and the finance company will continue in the item of equipment sold to the competitor

A buyer of goods will take free of an unperfected security interest in those goods See UCC sect 9-317(a)(2) However when the competitor bought the businessrsquos equipment both the bank and the finance company had perfected security interests in the equipment

A buyer can take free even of a perfected security interest in goods if the buyer is a ldquobuyer in ordinary course of businessrdquo See UCC sect 9-320(a) However the competitor was not a buyer in ordinary course of business To be a ldquobuyer in ordinary course of businessrdquo a buyer must buy goods from a seller that is in the business of selling goods of that kind See UCC sect 1-201(b)(9) The competitor bought this equipment from a seller that is not in the business of selling goods of this kind so the competitor was not a buyer in ordinary course of business with respect to these goods

Because no exception applies the security interests of the bank and the finance company continue even after the item of equipment was sold to the competitor

18

FEDERAL CIVIL PROCEDURE ANALYSIS (Federal Civil Procedure IVD)

ANALYSIS

Legal Problems

(1) Is a document prepared in the course of a contract dispute protected from discovery as ldquowork productrdquo when there is no evidence that the document was prepared in anticipation of litigation

(2)(a) Is a partyrsquos failure to provide relevant electronically stored information excused when the information was destroyed pursuant to a routine document retention scheme at a time when litigation was contemplated by the destroying party

(2)(b) What sanctions should be imposed on a party for allowing the destruction of evidence that is relevant to potential future litigation

DISCUSSION

Summary

The report prepared by the structural engineer is probably not work product and is thus discoverable The engineer examined the foundation of the house at the customerrsquos request and the engineerrsquos findings are potentially relevant to the customerrsquos claim that the foundation is defective The report was not prepared in anticipation of litigation The customer appears to have sought the engineerrsquos opinion in response to the builderrsquos offer to fix any problems with the foundation that an engineer might identify Because the report was not prepared in anticipation of litigation it is not protected by the work-product doctrine

The builder should have taken appropriate steps to preserve evidence including suspending its document retention program as soon as it began planning for litigationmdashie on July 10 Its destruction of potentially relevant material after that date was wrongful However a court is unlikely to impose severe sanctions on the builder because there are no facts indicating that the builder acted in bad faith and the customer can prove that the foundation is defective without the destroyed emails

Point One (40) The customer must turn over the engineerrsquos report because it was not prepared in anticipation of litigation

In general a party to a lawsuit in federal court ldquomay obtain discovery regarding any nonprivileged matter that is relevant to any partyrsquos claim or defenserdquo FED R CIV P 26(b)(1) (2009) This includes the right to inspect and copy documents in the other partyrsquos possession FED R CIV P 34(a)(1) Here the customer hired a structural engineer to examine the foundation of the house The engineerrsquos report on the foundation is likely to include information that would be relevant to the customerrsquos claim that the foundation was defectively constructed

The so-called ldquowork productrdquo rule allows a party to refuse to turn over ldquodocuments that are prepared in anticipation of litigation or for trialrdquo by that partyrsquos representative including

19

Federal Civil Procedure Analysis

a consultant Thus if the customer had hired the structural engineer to prepare a report ldquoin anticipation of litigationrdquo that report might not be discoverable See FED R CIV P 26(b)(3)

In this case however the customer hired the engineer to evaluate the foundation of the house as part of the customerrsquos negotiation with the builder concerning the housersquos flooding problem The builder told the customer that the housersquos landscaping was the reason for the flooding and the builder told the customer ldquoHave an engineer look at the foundation If therersquos a problem wersquoll fix itrdquo The customer appears to have acted in response to that statement There is no indication that the customer anticipated any kind of legal action at the time that the structural engineer was hired Accordingly the structural engineerrsquos report is discoverable and the court should order the customer to turn it over

[NOTE If an examinee concludes that the structural engineerrsquos report was prepared in anticipation of litigation then the examinee should also conclude that the report is not discoverable Documents prepared in anticipation of litigation do not need to be disclosed to an adverse party unless that party can demonstrate a ldquosubstantial needrdquo for the documents and an inability to obtain substantially equivalent information without ldquoundue hardshiprdquo FED R CIV P 26(b)(3)(A)(ii) Furthermore a report prepared by an expert who is not expected to testify is not discoverable in the absence of ldquoexceptional circumstancesrdquo making it ldquoimpracticablerdquo to obtain the information in another way FED R CIV P 26(b)(4)(D)(ii) The builder probably cannot make these showings here unless the engineerrsquos report deals with circumstances that have since changed There is no evidence that the structural engineer would have had access to any information or facts that the builder would not already know as a result of its construction and subsequent inspection of the house In addition if necessary the builder could ask the court for permission to arrange for a further inspection of the house by a structural engineer hired by the builder See FED R CIV P 34(a)(2) Accordingly if an examinee concludes that the report was prepared in anticipation of litigation the examinee should also conclude that the builder is not entitled to see the report]

Point Two(a) (30) Because the builder anticipated that it might be involved in litigation concerning its contract with the customer the builder acted wrongfully in destroying emails that were relevant to the housersquos construction even though the emails were destroyed pursuant to a routine document retention plan

As noted above a party to a lawsuit in federal court ldquomay obtain discovery regarding any nonprivileged matter that is relevant to any partyrsquos claim or defenserdquo FED R CIV P 26(b)(1) This includes emails and other electronically stored information FED R CIV P 34(a)(1)(A) Here the customer has requested all the builderrsquos emails pertaining to work done on the foundation of the house Ordinarily the builder would be obliged to turn over this information which is relevant to the customerrsquos defense that the housersquos foundation was poorly constructed

Unfortunately the emails in question no longer exist because the builder destroyed them on August 2

In general spoliation of evidence (destruction or alteration of evidence) is improper if the party who destroyed or altered the evidence ldquohas notice that the evidence is relevant to litigation or should have known that the evidence may be relevant to future litigationrdquo Fujitsu Ltd v Federal Express Corp 247 F3d 423 436 (2d Cir 2001) It is improper for a party to destroy electronic information relevant to pending litigation even if the destruction occurs before there is any request or order seeking the information See eg Leon v IDX Sys Corp 464 F3d 951 (9th Cir 2006) (plaintiffrsquos intentional destruction of computer files warranted dismissal even

20

In this case the builderrsquos destruction of the emails was pursuant to a routine document retention plan The Federal Rules provide expressly that in the absence of ldquoexceptional circumstancesrdquo parties should not be sanctioned for the loss of electronically stored information when the loss occurs pursuant to ldquoroutine good-faith operation of an electronic information systemrdquo FED R CIV P 37(e) However when a party anticipates litigation ldquoit must suspend its routine document retentiondestruction policy and put in place a lsquolitigation holdrsquo to ensure the preservation of relevant documentsrdquo Zubulake v UBS Warburg LLC 220 FRD 212 218 (SDNY 2003)

Federal Civil Procedure Analysis

though spoliation occurred before order compelling discovery) Similarly the duty to preserve evidence applies to a party who anticipates litigation even if litigation has not yet been commenced See THE SEDONA PRINCIPLES BEST PRACTICES RECOMMENDATIONS amp PRINCIPLES FOR ADDRESSING ELECTRONIC DOCUMENT PRODUCTION 70 cmt 14a (2d ed 2007)

The builder destroyed the emails on August 2 At that time the builder knew that litigation was a possibility because the builder had already directed its attorney to prepare a draft complaint for possible filing Knowing that litigation was a possibility the builder had a duty to take steps to preserve evidence including the emails in question See generally Fujitsu Ltd

Thus the builderrsquos destruction of potentially relevant emails at a time when it knew that litigation was a possibility was improper It had a duty to preserve evidence and it breached that duty

[NOTE Because courts have used different words to describe the test for when evidence must be preserved an examineersquos precise formulation of the test is not critical]

Point Two(b) (30) In determining appropriate sanctions for spoliation courts consider both the level of culpability of the spoliating party and the degree of prejudice the loss of evidence has caused the other party Here the builderrsquos destruction of evidence does not appear to have been willful nor is it likely to pose a significant obstacle to the customerrsquos defense Any sanctions imposed by the court should be modest

Federal courts have inherent power to control the litigation process and can sanction misbehavior including spoliation even when there has been no specific violation of the Federal Rules of Civil Procedure See generally Chambers v NASCO Inc 501 US 32 (1991) (discussing courtrsquos inherent power to control the litigation process) The range of available sanctions is broad It can include such sanctions as the payment of expenses incurred by the other party as a result of the destruction of the evidence an instruction to the jury authorizing it to draw an adverse inference from the destruction of the evidence a shifting of the burden of proof on the relevant issue or even judgment against the responsible party See eg Residential Funding Corp v DeGeorge Financial Corp 306 F3d 99 108 (2d Cir 2002) (adverse inference) Silvestri v General Motors Corp 271 F3d 583 593 (4th Cir 2001) (possibility of dismissal) Cf FED R CIV P 37(b)(2)(A) (listing remedies for failure to comply with discovery obligations)

In determining appropriate sanctions for spoliation courts consider both the level of culpability of the spoliating party and the degree of prejudice the loss of evidence has caused the other party Many courts impose severe sanctions (such as an adverse-inference instruction or the entry of judgment against the spoliating party) only when there is evidence of bad faith in the form of an intentional effort to hide information Eg Greyhound Lines Inc v Wade 485 F3d 1032 1035 (8th Cir 2007) (spoliation sanction requires intentional destruction out of desire ldquoto suppress the truthrdquo) However other courts have said that negligence in preserving evidence can

21

Federal Civil Procedure Analysis

support an adverse-inference instruction See Residential Funding 306 F3d at 108 (negligence enough under some circumstances)

Although a court might well order an evidentiary hearing on the issue of sanctions the facts presented do not seem appropriate for severe sanctions First the evidence was destroyed pursuant to the builderrsquos standard document retention plan and there is no evidence that the builder deliberately failed to suspend its usual procedures with the purpose of allowing the destruction of evidence Second the loss of this evidence will not severely hinder the customerrsquos presentation of his case The central issue is whether the foundation of the house was properly constructed If the construction job was poorly done the customer can present evidence derived from inspection of the premises to prove that point The customer can also depose witnesses about any issues that arose during construction

Under the circumstances a court is not likely to impose particularly severe sanctions although it might shift the burden to the builder to show that the foundation was properly constructed or it might require the builder to reimburse any expenses the customer incurs to discover and prove the facts about issues or disputes that arose during construction of the foundation

[NOTE The result reached by the examinee is less important than the examineersquos recognition that (a) a range of sanctions is available to the court and (b) the appropriate sanction depends both on the culpability of the builder and the prejudice suffered by the customer]

22

CRIMINAL LAW AND PROCEDURE ANALYSIS (Criminal Law and Procedure IIA amp D VE amp F)

ANALYSIS

Legal Problems

(1) Did charging the defendant with both theft and burglary constitute double jeopardy

(2) Did the jury instruction violate the due process clause either by relieving the prosecution of the burden of proving the element of intent or by shifting the burden to the defendant to disprove that element

(3) Did the sentence imposed in this case for the theft conviction unconstitutionally deprive the defendant of his right to a jury trial on the issue of the value of the stolen item

DISCUSSION

Summary

The trial court properly denied the defendantrsquos pretrial motion to dismiss the charges on double jeopardy grounds The defendant may be charged with and convicted of both theft and burglary Each of the charges has an element that the other does not Neither charge is a lesser-included offense nor are they multiplicitous Thus charging both theft and burglary does not violate double jeopardy

The jury instruction on the burglary charge was constitutionally flawed It could have been reasonably understood by the jury as either (1) an irrebuttable conclusive presumption (which relieved the prosecution of proving the element of intent and removed the issue from the jury) or (2) a rebuttable mandatory presumption (which unconstitutionally shifted the burden of proof on an element of a charged offense from the prosecution to the defendant)

Because the four-year sentence imposed by the judge was based on the judgersquos finding by a preponderance of the evidence that the value of the stolen ring exceeded $5000 the sentence violates the defendantrsquos right to a jury determination beyond a reasonable doubt of the value of the ring

Point One (30) Charging the defendant with theft and burglary did not constitute double jeopardy

The Double Jeopardy Clause of the Fifth Amendment provides that a person shall not be twice put in jeopardy for the ldquosame offenserdquo Thus the question is whether the elements of the theft charge are wholly contained in the burglary charge or vice versa If the elements of the lesser charge (theft) are not wholly contained in the greater charge (burglary)mdashie if each charge requires proof of a fact that the other does notmdashthen convicting the defendant of both crimes would not violate double jeopardy even when the two offenses occurred at the same time and are thus arguably part of the ldquosame transactionrdquo Blockburger v United States 284 US 299 304 (1932) See also Albernaz v United States 450 US 333 344 n3 (1981) United States v Dixon 509 US 688 704 (1993)

23

Criminal Law and Procedure Analysis

Here theft and burglary each require proof of an element not required for the other crime Burglary may be defined differently in different jurisdictions However it almost invariably requires entry into a building or dwelling of another with the specific intent to commit a felony therein and the crime of burglary is complete upon the entry into the building or dwelling with such intent See eg Cannon v Oklahoma 827 P2d 1339 1342 (Okla Crim App 1992) In contrast theft which also may be defined differently in different states almost invariably requires the taking and carrying away of an item of personal property belonging to another with the intent to steal or permanently deprive the owner of possession

Here the ldquotakingrdquo or ldquostealingrdquo element is not contained in the definition of burglary and the ldquoentryrdquo element of burglary is not contained in the definition of theft Because theft is not a lesser-included offense of burglary and burglary is not a lesser-included offense of theft charging the defendant for both burglary and theft did not violate double jeopardy and the court properly denied the defense motion on those grounds Yparrea v Dorsey 64 F3d 577 579ndash80 (10th Cir 1995) citing Blockburger 284 US at 304

Finally the defendantrsquos motion to dismiss all the charges on double jeopardy grounds was improper because if both charges were for the same offense the motion should have requested dismissal of one charge not both

Point Two (35) The jury instruction on the burglary charge violated the Due Process Clause because it created either (1) an irrebuttable conclusive presumption (which relieved the prosecution of proving the element of intent and removed that issue from the jury) or (2) a rebuttable mandatory presumption (which unconstitutionally shifted the burden of proof on an element of a charged offense to the defendant)

The Supreme Court has interpreted the Due Process Clause of the US Constitution to require that the prosecution prove all elements of an offense beyond a reasonable doubt See In re Winship 397 US 358 364 (1970) The burden of proof cannot be shifted to the defendant by presuming an essential element upon proof of other elements of the offense because shifting the burden of persuasion with respect to any element of a criminal offense is contrary to the Due Process Clause See Mullaney v Wilbur 421 US 684 (1975)

The crime of burglary includes entry into a building or dwelling with the specific intent to commit a felony therein The requirement that the prosecutor prove beyond a reasonable doubt that the defendant had this specific intent distinguishes burglary from general-intent crimes like trespass See Sandstrom v Montana 442 US 510 523 (1979)

Here the jury was instructed that if ldquoafter consideration of all the evidence presented by the prosecution and defense you find beyond a reasonable doubt that the defendant entered the dwelling without the ownersrsquo consent you may presume that the defendant entered with the intent to commit a felony thereinrdquo This instruction was unconstitutional because it created either an irrebuttable conclusive presumption or a rebuttable mandatory presumption

A conclusive presumption is ldquoan irrebuttable direction by the court to find intent once convinced of the facts triggering the presumptionrdquo Id at 517 Here the jurors were instructed that once the prosecutor established that the defendant entered the neighborsrsquo house without consent they ldquomay presumerdquo that he intended to commit a felony therein The jurors may have reasonably concluded from this instruction that if they found that the defendant intended to enter his neighborsrsquo home without permission they must further find that he entered with the specific intent to commit a felony therein Because this instruction could operate as a conclusive

24

Criminal Law and Procedure Analysis

irrebuttable presumption by eliminating intent ldquoas an ingredient of the offenserdquo it violated due process by relieving the prosecution of the burden of proof for this element Id at 522

In the alternative the jury instruction could have been reasonably understood to create a rebuttable mandatory presumption which ldquotells [the jury] they must find the elemental fact upon proof of the basic fact at least unless the defendant has come forward with some evidence to rebut the presumed connection between the two factsrdquo County Court of Ulster County New York v Allen 442 US 140 157 (1979) The due process problem created by rebuttable mandatory presumptions is that ldquo[t]o the extent that the trier of fact is forced to abide by the presumption and may not reject it based on an independent evaluation of the particular facts presented by the State the analysis of the presumptionrsquos constitutional validity is logically divorced from those facts and based on the presumptionrsquos accuracy in the run of casesrdquo Id at 159

Unlike irrebuttable conclusive presumptions rebuttable mandatory presumptions are not always per se violations of the Due Process Clause However the Supreme Court of the United States has held that jury instructions that could reasonably be understood as shifting the burden of proof to the defendant on an element of the offense are unconstitutional Francis v Franklin 471 US 307 (1985) Here the argument that the jury instruction operated as a rebuttable mandatory presumption is supported by the fact that the judge also instructed the jury to ldquoconsider[ ] all the evidence presented by the prosecution and defenserdquo However even if the instruction created a rebuttable mandatory presumption it would be unconstitutional because it shifted the burden to the defense on an element of the offense Sandstrom 442 US at 524 Mullaney 421 US at 686

[NOTE Whether an examinee identifies the jury instruction as containing a ldquoconclusiverdquo or ldquomandatoryrdquo presumption is less important than the examineersquos analysis of the constitutional infirmities]

Point Three (35) The trial court violated the defendantrsquos Sixth Amendment right to a jury trial on an essential element of the offense when it found by a preponderance of the evidence that the ring was worth over $5000 and increased the defendantrsquos sentence based on this finding

In the statutory scheme under which the defendant was tried and convicted a Class D felony theft is defined as theft of item(s) with a value between $2500 and $10000 The jury found that the value of the diamond ring was at least $2500 and convicted the defendant of felony theft However at sentencing the trial court made a separate finding by a preponderance of the evidence that the value of the ring was greater than $5000 Following the statutersquos two-tiered sentencing scheme the judge then imposed on the defendant a sentence that was one year longer than the maximum that would otherwise have been allowed

The judgersquos sentence was unconstitutional because it violated the defendantrsquos Sixth Amendment right to a jury trial on this question The Supreme Court held in Apprendi v New Jersey 530 US 466 (2000) that ldquo[o]ther than the fact of a prior conviction any fact that increases the penalty for a crime beyond the prescribed statutory maximum must be submitted to a jury and proved beyond a reasonable doubtrdquo because ldquo[i]t is unconstitutional for a legislature to remove from the jury the assessment of facts that increase the prescribed range of penalties to which a criminal defendant is exposed [because] such facts must be established by proof beyond a reasonable doubtrdquo Id The Court reaffirmed Apprendi in Blakely v Washington 542 US 296 (2004) holding that the ldquolsquostatutory maximumrsquo for Apprendi purposes is the maximum sentence a judge may impose solely on the basis of the facts reflected in the jury verdict or admitted by the defendantrdquo Id at 303 (emphasis in original) In United States v Booker 543 US 220 (2005)

25

Criminal Law and Procedure Analysis

the Court relied on Blakely and Apprendi to conclude that protecting a defendantrsquos Sixth Amendment right to a jury trial required that ldquo[a]ny fact which is necessary to support a sentence exceeding the maximum authorized by the facts established by a plea of guilty or a jury verdict must be admitted by the defendant or proved to a jury beyond a reasonable doubtrdquo Id at 244

Thus in order to constitutionally increase a sentence above the statutory maximum of three years the jury must have found beyond a reasonable doubt that the value of the ring exceeded $5000 Here the court made the finding based on an appraisal proffered by the prosecutor only at sentencing and the judgersquos finding was by a preponderance of the evidence rather than beyond a reasonable doubt

26

AGENCY AND PARTNERSHIP ANALYSIS __________ (Agency and Partnership VA amp C VI)

ANALYSIS

Legal Problems

(1) Is a partner in a general partnership personally liable on a claim arising from misrepresentations by another partner made in the course of the partnership business

(2) Does a newly admitted partner in a general partnership become personally liable on existing claims against the partnership

(3) After the filing by a general partnership of a statement of qualification as a limited liability partnership are the partners personally liable as partners on (a) an existing claim against the general partnership and (b) a claim against the partnership that arose after the filing

DISCUSSION

Summary

Adam and Ben formed a general partnership under which they were jointly and severally liable for obligations of the partnership Thus Adam was personally liable for misrepresentations by Ben made in the ordinary course of the partnership business

Upon joining the general partnership Diane became personally liable for the obligations of the partnership arising after her admission but not for obligations pre-existing her admission such as the collectorrsquos claim

By filing a statement of qualification the three partners properly elected limited liability partnership status As partners in an LLP none of the three partners is personally liable as a partner for partnership obligations arising after the election such as the claim by the driverrsquos estate The election however does not change their personal liability on pre-existing claims that arose before the election such as the collectorrsquos claim

Point One (30) As a general partner of Empire a general partnership Adam became personally liable on the collectorrsquos claim a valid claim against the partnership that arose because of Benrsquos wrongful act in the ordinary course of the partnership business

When the collectorrsquos claim arose Empire was a general partnership composed of Adam and Ben Under UPA (1997) sect 306(a) partners of a general partnership are liable jointly and severally for all obligations of the partnership Under UPA (1997) sect 305(a) the partnership could become obligated for the loss caused to the collector as a result of the misrepresentation by Ben provided he was acting in the ordinary course of the partnership business Because there was no statement that limited his partnership authority Ben as partner was ldquoan agent of the partnership for the purpose of its businessrdquo See UPA (1997) sect 301(1) Benrsquos misrepresentation to the collector even if intentional appears to be in the ordinary course of the partnershiprsquos business of dealing

27

Agency and Partnership Analysis

in antique cars Thus Benrsquos wrongful act created a partnership obligation for which Adam was jointly and severally liable

[NOTE Generally a partnership creditor must ldquoexhaust the partnershiprsquos assets before levying on a judgment debtor partnerrsquos individual property where the partner is personally liable for the partnership obligationrdquo as a result of his status as a partner UPA (1997) sect 307 cmt 4 As the UPA comments explain this places Adam more in the position of guarantor than principal debtor on the partnership obligation Id cmt 4 Although an examinee might discuss this point the call focuses on whether Adam is personally liable not how the liability might be enforced]

Point Two (30) Because the collectorrsquos claim arose before Diane joined Empire Diane did not become personally liable on the claim

Diane was admitted to Empire when it was a general partnership and after the collectorrsquos claim arose While the general rule under UPA (1997) sect 306(a) is that the partners of a general partnership are liable jointly and severally for all obligations of the partnership there is a special rule for partners who are admitted during the duration of the partnership Under UPA (1997) sect 306(b) a person admitted to an existing partnership is not personally liable for any partnership obligations incurred before the personrsquos admission Because Diane was admitted to Empire after the collectorrsquos claim arose Diane is not personally liable on the claim

Dianersquos knowledge of the pre-existing claim and her stated concern about becoming liable on the collectorrsquos claim do not change her personal nonliability to the collector Although partners who have a liability shield can assume liability to third parties through private contractual guarantees or modifications to the partnership agreement Dianersquos stated concern constituted neither a guaranty to the collector nor ldquoan intentional waiver of liability protectionsrdquo See UPA (1997) sect 306 cmt 3 (describing methods for waiver of liability protections under sect 306(c) applicable in limited liability partnerships)

At most Diane will lose her investment in the partnership as a result of the collectorrsquos claim Although Diane did not become personally liable on the collectorrsquos claim when she joined the partnership the $250000 she contributed to the partnership is ldquoat risk for the satisfaction of existing partnership debtsrdquo UPA (1997) sect 306 cmt 2

Point Three (40) Filing the statement of qualification was effective to elect limited liability partnership status Despite this new status Adam and Ben remain personally liable on the collectorrsquos claim which arose before the election But as partners in an LLP neither Adam Ben nor Diane is personally liable as a partner on the driverrsquos estatersquos claim which arose after the election

Under UPA (1997) sect 1001 a general partnership can make an election and become a limited liability partnershipmdashif the partners approve the conversion by a vote equivalent to that necessary to amend the partnership agreement and the partnership then files a statement of qualification that specifies the name of the partnership its principal office and its election to be an LLP Here the partners agreed unanimouslymdashsufficient to amend their agreement under UPA (1997) sect 401(j)mdashand the statement of qualification was filed In addition the name of Empire LLP properly included an appropriate ending ldquoLLPrdquo See UPA (1997) sect 1002

Although another way to effectuate a ldquoconversionrdquo (as suggested by Benrsquos lawyer) is to form a new LLP and transfer the assets of the old general partnership to the new LLP the

28

Agency and Partnership Analysis

method used here (approval by the partners and the filing of a statement of qualification) is also sufficient to create LLP status

Thus Empire became Empire LLP as of the date of filing of the statement of qualification See UPA (1997) sect 1001 What effect did this have on the collectorrsquos claim which predated the filing According to UPA (1997) sect 306(c) an obligation incurred while a partnership is an LLP is solely a partnership obligation As the collectorrsquos claim predated the LLP Adam and Ben remain personally liable on the collectorrsquos claim Diane on the other hand was not personally liable on the collectorrsquos claim either before or after the filing of the statement of qualification See Point Two above

The driverrsquos estatersquos claim arose after Empire became Empire LLP Under UPA (1997) sect 306(c) an obligation incurred while a partnership is an LLP is solely a partnership obligationThus Adam Ben and Diane as partners are all protected from personal liability on the driverrsquos estatersquos claim But there may be personal liability if any of them was negligent or otherwise acted wrongfully by not informing the buyer of the bad suspension that caused the accident

29

National Conference of Bar Examiners 302 South Bedford Street | Madison WI 53703-3622 Phone 608-280-8550 | Fax 608-280-8552 | TDD 608-661-1275

wwwncbexorg e-mail contactncbexorg

  • Preface
  • Description of the MEE
  • Instructions
  • February 2014 Questions
    • Constitutinal Law Question
    • Trusts and Future Interests Question
    • Secured Transactions Question
    • Federal Civil Procedure Question
    • Criminal Law and Procedure Question
    • Agency and Partnership Question
      • February 2014 Analyses
        • Constitutional Law Analysis
        • Trust and Future Interests Analysis
        • Secured Transactions Analysis
        • Federal Civil Procedure Analysis
        • Criminal Law and Procedure Analysis
        • Agency and Partnership Analysis
            • ltlt13 ASCII85EncodePages false13 AllowTransparency false13 AutoPositionEPSFiles true13 AutoRotatePages None13 Binding Left13 CalGrayProfile (Dot Gain 20)13 CalRGBProfile (sRGB IEC61966-21)13 CalCMYKProfile (US Web Coated 050SWOP051 v2)13 sRGBProfile (sRGB IEC61966-21)13 CannotEmbedFontPolicy Error13 CompatibilityLevel 1413 CompressObjects Tags13 CompressPages true13 ConvertImagesToIndexed true13 PassThroughJPEGImages true13 CreateJobTicket false13 DefaultRenderingIntent Default13 DetectBlends true13 DetectCurves 0000013 ColorConversionStrategy CMYK13 DoThumbnails false13 EmbedAllFonts true13 EmbedOpenType false13 ParseICCProfilesInComments true13 EmbedJobOptions true13 DSCReportingLevel 013 EmitDSCWarnings false13 EndPage -113 ImageMemory 104857613 LockDistillerParams false13 MaxSubsetPct 10013 Optimize true13 OPM 113 ParseDSCComments true13 ParseDSCCommentsForDocInfo true13 PreserveCopyPage true13 PreserveDICMYKValues true13 PreserveEPSInfo true13 PreserveFlatness true13 PreserveHalftoneInfo false13 PreserveOPIComments true13 PreserveOverprintSettings true13 StartPage 113 SubsetFonts true13 TransferFunctionInfo Apply13 UCRandBGInfo Preserve13 UsePrologue false13 ColorSettingsFile ()13 AlwaysEmbed [ true13 ]13 NeverEmbed [ true13 ]13 AntiAliasColorImages false13 CropColorImages true13 ColorImageMinResolution 30013 ColorImageMinResolutionPolicy OK13 DownsampleColorImages true13 ColorImageDownsampleType Bicubic13 ColorImageResolution 30013 ColorImageDepth -113 ColorImageMinDownsampleDepth 113 ColorImageDownsampleThreshold 15000013 EncodeColorImages true13 ColorImageFilter DCTEncode13 AutoFilterColorImages true13 ColorImageAutoFilterStrategy JPEG13 ColorACSImageDict ltlt13 QFactor 01513 HSamples [1 1 1 1] VSamples [1 1 1 1]13 gtgt13 ColorImageDict ltlt13 QFactor 01513 HSamples [1 1 1 1] VSamples [1 1 1 1]13 gtgt13 JPEG2000ColorACSImageDict ltlt13 TileWidth 25613 TileHeight 25613 Quality 3013 gtgt13 JPEG2000ColorImageDict ltlt13 TileWidth 25613 TileHeight 25613 Quality 3013 gtgt13 AntiAliasGrayImages false13 CropGrayImages true13 GrayImageMinResolution 30013 GrayImageMinResolutionPolicy OK13 DownsampleGrayImages true13 GrayImageDownsampleType Bicubic13 GrayImageResolution 30013 GrayImageDepth -113 GrayImageMinDownsampleDepth 213 GrayImageDownsampleThreshold 15000013 EncodeGrayImages true13 GrayImageFilter DCTEncode13 AutoFilterGrayImages true13 GrayImageAutoFilterStrategy JPEG13 GrayACSImageDict ltlt13 QFactor 01513 HSamples [1 1 1 1] VSamples [1 1 1 1]13 gtgt13 GrayImageDict ltlt13 QFactor 01513 HSamples [1 1 1 1] VSamples [1 1 1 1]13 gtgt13 JPEG2000GrayACSImageDict ltlt13 TileWidth 25613 TileHeight 25613 Quality 3013 gtgt13 JPEG2000GrayImageDict ltlt13 TileWidth 25613 TileHeight 25613 Quality 3013 gtgt13 AntiAliasMonoImages false13 CropMonoImages true13 MonoImageMinResolution 120013 MonoImageMinResolutionPolicy OK13 DownsampleMonoImages true13 MonoImageDownsampleType Bicubic13 MonoImageResolution 120013 MonoImageDepth -113 MonoImageDownsampleThreshold 15000013 EncodeMonoImages true13 MonoImageFilter CCITTFaxEncode13 MonoImageDict ltlt13 K -113 gtgt13 AllowPSXObjects false13 CheckCompliance [13 None13 ]13 PDFX1aCheck false13 PDFX3Check false13 PDFXCompliantPDFOnly false13 PDFXNoTrimBoxError true13 PDFXTrimBoxToMediaBoxOffset [13 00000013 00000013 00000013 00000013 ]13 PDFXSetBleedBoxToMediaBox true13 PDFXBleedBoxToTrimBoxOffset [13 00000013 00000013 00000013 00000013 ]13 PDFXOutputIntentProfile ()13 PDFXOutputConditionIdentifier ()13 PDFXOutputCondition ()13 PDFXRegistryName ()13 PDFXTrapped False1313 CreateJDFFile false13 Description ltlt13 ARA ltFEFF06270633062A062E062F0645002006470630064700200627064406250639062F0627062F0627062A002006440625064606340627062100200648062B062706260642002000410064006F00620065002000500044004600200645062A064806270641064206290020064406440637062806270639062900200641064A00200627064406450637062706280639002006300627062A0020062F0631062C0627062A002006270644062C0648062F0629002006270644063906270644064A0629061B0020064A06450643064600200641062A062D00200648062B0627062606420020005000440046002006270644064506460634062306290020062806270633062A062E062F062706450020004100630072006F0062006100740020064800410064006F006200650020005200650061006400650072002006250635062F0627063100200035002E0030002006480627064406250635062F062706310627062A0020062706440623062D062F062B002E0635062F0627063100200035002E0030002006480627064406250635062F062706310627062A0020062706440623062D062F062B002Egt13 BGR 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 CHS ltFEFF4f7f75288fd94e9b8bbe5b9a521b5efa7684002000410064006f006200650020005000440046002065876863900275284e8e9ad88d2891cf76845370524d53705237300260a853ef4ee54f7f75280020004100630072006f0062006100740020548c002000410064006f00620065002000520065006100640065007200200035002e003000204ee553ca66f49ad87248672c676562535f00521b5efa768400200050004400460020658768633002gt13 CHT ltFEFF4f7f752890194e9b8a2d7f6e5efa7acb7684002000410064006f006200650020005000440046002065874ef69069752865bc9ad854c18cea76845370524d5370523786557406300260a853ef4ee54f7f75280020004100630072006f0062006100740020548c002000410064006f00620065002000520065006100640065007200200035002e003000204ee553ca66f49ad87248672c4f86958b555f5df25efa7acb76840020005000440046002065874ef63002gt13 CZE 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 DAN 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 DEU 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 ESP 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 ETI 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 FRA 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 GRE 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 HEB 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 HRV (Za stvaranje Adobe PDF dokumenata najpogodnijih za visokokvalitetni ispis prije tiskanja koristite ove postavke Stvoreni PDF dokumenti mogu se otvoriti Acrobat i Adobe Reader 50 i kasnijim verzijama)13 HUN ltFEFF004b0069007600e1006c00f30020006d0069006e0151007300e9006701710020006e0079006f006d00640061006900200065006c0151006b00e90073007a00ed007401510020006e0079006f006d00740061007400e100730068006f007a0020006c006500670069006e006b00e1006200620020006d0065006700660065006c0065006c0151002000410064006f00620065002000500044004600200064006f006b0075006d0065006e00740075006d006f006b0061007400200065007a0065006b006b0065006c0020006100200062006500e1006c006c00ed007400e10073006f006b006b0061006c0020006b00e90073007a00ed0074006800650074002e0020002000410020006c00e90074007200650068006f007a006f00740074002000500044004600200064006f006b0075006d0065006e00740075006d006f006b00200061007a0020004100630072006f006200610074002000e9007300200061007a002000410064006f00620065002000520065006100640065007200200035002e0030002c0020007600610067007900200061007a002000610074007400f3006c0020006b00e9007301510062006200690020007600650072007a006900f3006b006b0061006c0020006e00790069007400680061007400f3006b0020006d00650067002egt13 ITA 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 JPN ltFEFF9ad854c18cea306a30d730ea30d730ec30b951fa529b7528002000410064006f0062006500200050004400460020658766f8306e4f5c6210306b4f7f75283057307e305930023053306e8a2d5b9a30674f5c62103055308c305f0020005000440046002030d530a130a430eb306f3001004100630072006f0062006100740020304a30883073002000410064006f00620065002000520065006100640065007200200035002e003000204ee5964d3067958b304f30533068304c3067304d307e305930023053306e8a2d5b9a306b306f30d530a930f330c8306e57cb30818fbc307f304c5fc59808306730593002gt13 KOR ltFEFFc7740020c124c815c7440020c0acc6a9d558c5ec0020ace0d488c9c80020c2dcd5d80020c778c1c4c5d00020ac00c7a50020c801d569d55c002000410064006f0062006500200050004400460020bb38c11cb97c0020c791c131d569b2c8b2e4002e0020c774b807ac8c0020c791c131b41c00200050004400460020bb38c11cb2940020004100630072006f0062006100740020bc0f002000410064006f00620065002000520065006100640065007200200035002e00300020c774c0c1c5d0c11c0020c5f40020c2180020c788c2b5b2c8b2e4002egt13 LTH 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 LVI 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 NLD (Gebruik deze instellingen om Adobe PDF-documenten te maken die zijn geoptimaliseerd voor prepress-afdrukken van hoge kwaliteit De gemaakte PDF-documenten kunnen worden geopend met Acrobat en Adobe Reader 50 en hoger)13 NOR ltFEFF004200720075006b00200064006900730073006500200069006e006e007300740069006c006c0069006e00670065006e0065002000740069006c002000e50020006f0070007000720065007400740065002000410064006f006200650020005000440046002d0064006f006b0075006d0065006e00740065007200200073006f006d00200065007200200062006500730074002000650067006e0065007400200066006f00720020006600f80072007400720079006b006b0073007500740073006b00720069006600740020006100760020006800f800790020006b00760061006c0069007400650074002e0020005000440046002d0064006f006b0075006d0065006e00740065006e00650020006b0061006e002000e50070006e00650073002000690020004100630072006f00620061007400200065006c006c00650072002000410064006f00620065002000520065006100640065007200200035002e003000200065006c006c00650072002000730065006e006500720065002egt13 POL 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 PTB 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 RUM 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 RUS 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 SKY ltFEFF0054006900650074006f0020006e006100730074006100760065006e0069006100200070006f0075017e0069007400650020006e00610020007600790074007600e100720061006e0069006500200064006f006b0075006d0065006e0074006f0076002000410064006f006200650020005000440046002c0020006b0074006f007200e90020007300610020006e0061006a006c0065007001610069006500200068006f0064006900610020006e00610020006b00760061006c00690074006e00fa00200074006c0061010d00200061002000700072006500700072006500730073002e00200056007900740076006f00720065006e00e900200064006f006b0075006d0065006e007400790020005000440046002000620075006400650020006d006f017e006e00e90020006f00740076006f00720069016500200076002000700072006f006700720061006d006f006300680020004100630072006f00620061007400200061002000410064006f00620065002000520065006100640065007200200035002e0030002000610020006e006f0076016100ed00630068002egt13 SLV 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 SUO 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 SVE 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 TUR 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 UKR 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 ENU (Use these settings to create Adobe PDF documents best suited for high-quality prepress printing Created PDF documents can be opened with Acrobat and Adobe Reader 50 and later)13 gtgt13 Namespace [13 (Adobe)13 (Common)13 (10)13 ]13 OtherNamespaces [13 ltlt13 AsReaderSpreads false13 CropImagesToFrames true13 ErrorControl WarnAndContinue13 FlattenerIgnoreSpreadOverrides false13 IncludeGuidesGrids false13 IncludeNonPrinting false13 IncludeSlug false13 Namespace [13 (Adobe)13 (InDesign)13 (40)13 ]13 OmitPlacedBitmaps false13 OmitPlacedEPS false13 OmitPlacedPDF false13 SimulateOverprint Legacy13 gtgt13 ltlt13 AddBleedMarks false13 AddColorBars false13 AddCropMarks false13 AddPageInfo false13 AddRegMarks false13 ConvertColors ConvertToCMYK13 DestinationProfileName ()13 DestinationProfileSelector DocumentCMYK13 Downsample16BitImages true13 FlattenerPreset ltlt13 PresetSelector MediumResolution13 gtgt13 FormElements false13 GenerateStructure false13 IncludeBookmarks false13 IncludeHyperlinks false13 IncludeInteractive false13 IncludeLayers false13 IncludeProfiles false13 MultimediaHandling UseObjectSettings13 Namespace [13 (Adobe)13 (CreativeSuite)13 (20)13 ]13 PDFXOutputIntentProfileSelector DocumentCMYK13 PreserveEditing true13 UntaggedCMYKHandling LeaveUntagged13 UntaggedRGBHandling UseDocumentProfile13 UseDocumentBleed false13 gtgt13 ]13gtgt setdistillerparams13ltlt13 HWResolution [2400 2400]13 PageSize [612000 792000]13gtgt setpagedevice13

Page 10: February 2014 MEE Questions and AnalysesPreface The Multistate Essay Examination (MEE) is developed by the National Conference of Bar Examiners (NCBE). This publication includes the

FEDERAL CIVIL PROCEDURE QUESTION

A builder constructed a vacation house for an out-of-state customer on the customerrsquos land The house was completed on June 1 at which point the customer still owed $200000 of the $800000 contract price which was payable in full five days later

On June 14 the basement of the house was flooded with two inches of water during a heavy rainfall When the customer complained the builder told the customer ldquoThe flooding was caused by poorly designed landscaping Our work is fine and fully up to code Have an engineer look at the foundation If therersquos a problem wersquoll fix itrdquo

The customer pleased by the builderrsquos cooperative attitude immediately hired a structural engineer to examine the foundation of the house On June 30 the engineer provided the customer with a written report on the condition of the foundation which stated that the foundation was properly constructed

Unhappy with the conclusions in the engineerrsquos report the customer then hired a home inspector to evaluate the house The home inspectorrsquos report concluded that the foundation of the house had been poorly constructed and was inadequately waterproofed

On July 10 the customer sent the builder the home inspectorrsquos report with a note that said ldquoUntil you fix this problem you wonrsquot get another penny from merdquo The builder immediately contacted an attorney and directed the attorney to prepare a draft complaint against the customer for nonpayment Hoping to avoid litigation the builder sent several more requests for payment to the customer The customer ignored all these requests

On September 10 the builder filed suit in federal district court properly invoking the courtrsquos diversity jurisdiction and seeking $200000 in damages for breach of contract The customerrsquos answer denied liability on the basis of alleged defective construction of the housersquos foundation

Several months later the case is nearly ready for trial However two discovery disputes have not yet been resolved

First despite a request from the builder the customer has refused to provide a copy of the report prepared by the structural engineer who examined the foundation of the house The customer claims that the report is ldquowork productrdquo and not discoverable because the customer does not intend to ask the engineer to testify at trial The builder has asked the court to order the customer to turn over the engineerrsquos report

Second the customer has asked the court to impose sanctions for the builderrsquos failure to comply with the customerrsquos demand for copies of all emails concerning construction of the foundation of the house The builder has truthfully informed the customer that all such emails were destroyed on August 2 This destruction was pursuant to the builderrsquos standard practice of permanently deleting all project-related emails from company records 60 days after construction of a project is complete There is no relevant state records-retention law

1 Should the court order the customer to turn over the engineerrsquos report Explain

2 Should the court sanction the builder for the destruction of emails related to the case and if so what factors should the court consider in determining those sanctions Explain

6

CRIMINAL LAW AND PROCEDURE QUESTION _____

A defendant was charged under state law with felony theft (Class D) and felony residential burglary (Class C) The indictment alleged that the defendant entered his neighborsrsquo home without their consent and stole a diamond ring worth at least $2500

Defense counsel filed a pretrial motion to dismiss the charges on the ground that prosecuting the defendant for both burglary and theft would constitute double jeopardy The trial court denied the motion and the defendant was prosecuted for both crimes The only evidence of the ringrsquos value offered at the defendantrsquos jury trial was the ownerrsquos testimony that she had purchased the ring two years earlier for $3000

At trial the judge issued the following jury instruction on the burglary charge prior to deliberations

If after consideration of all the evidence presented by the prosecution and defense you find beyond a reasonable doubt that the defendant entered the dwelling without the ownersrsquo consent you may presume that the defendant entered with the intent to commit a felony therein

The jury found the defendant guilty of both offenses

At the defendantrsquos sentencing hearing an expert witness called by the prosecutor testified that the diamond ring was worth between $7000 and $8000 Over defense objection the judge concluded by a preponderance of the evidence that the value of the stolen ring exceeded $5000 The judge sentenced the defendant to four yearsrsquo incarceration on the theft conviction On the burglary conviction the defendant received a consecutive sentence of seven yearsrsquo incarceration

In this state residential burglary is defined as ldquoentry into the dwelling of another without the consent of the lawful resident with the intent to commit a felony thereinrdquo Residential burglary is a Class C felony for which the minimum sentence is five years and the maximum sentence is ten years of incarceration

In this state theft is defined as ldquotaking and carrying away the property of another with the intent to permanently deprive the owner of possessionrdquo Theft is a Class D felony if the value of the item(s) taken is between $2500 and $10000 The sentence for a Class D felony theft is determined by the value of the items taken If the value is between $2500 and $5000 the maximum sentence is three yearsrsquo incarceration If the value of the items exceeds $5000 the maximum sentence is five yearsrsquo incarceration

This state affords a criminal defendant no greater rights than those mandated by the United States Constitution

1 Did the trial court err when it denied the defendantrsquos pretrial motion to dismiss on double jeopardy grounds Explain

2 Did the trial court err in its instruction to the jury on the burglary charge Explain

3 Did the trial court err when it sentenced the defendant to an additional year of incarceration on the theft conviction based on the expertrsquos testimony Explain

7

AGENCY AND PARTNERSHIP QUESTION _____

Five years ago Adam and Ben formed a general partnership Empire Partnership (Empire) to buy and sell antique automobiles at a showroom in State A Adam contributed $800000 to Empire and Ben contributed $200000 Their written partnership agreement allocated 80 of profits losses and control to Adam and 20 to Ben No filings of any type were made in connection with the formation of Empire

Three years ago a collector purchased one of Empirersquos antique cars for $3400000 The collector was willing to pay this price because of Benrsquos false representation (repeated in the sales contract) that a famous movie star had once owned the car Without the movie-star connection the car was worth only $100000 One month later when the collector discovered the truth he sued Adam Ben and Empire for $3300000 in damages The lawsuit is still pending

Two years ago Adam and Ben admitted a new partner Diane to Empire in return for her contribution of $250000 The three agreed to allocate profits losses and control 75 to Adam 10 to Ben and 15 to Diane Before joining the partnership Diane learned of the collectorrsquos claim and stated her concern to Adam and Ben that she might become liable if the claim were reduced to a judgment

Following Dianersquos admission to Empire the three partners sought to convert Empire into a limited liability partnership (LLP) Adamrsquos lawyer proposed to file with State A a ldquostatement of qualificationrdquo making an LLP election and declaring the name of the partnership to be ldquoEmpire LLPrdquo Benrsquos lawyer stated that this would not work and that a new LLP had to be formed with the assets of the old partnership transferred to the new one In the end the conversion was done the way Adamrsquos lawyer suggested with the approval of all three partners

One year ago a driver purchased a vintage car from Empire LLP based on the representation that the car was ldquofully roadworthy and capable of touring at 70 mph all dayrdquo The driver took the car on the highway at 50 mph whereupon the front suspension collapsed resulting in a crash in which the car was destroyed and the driver killed The driverrsquos estate sued Adam Ben Diane and Empire LLP for $10000000 The lawsuit is still pending

Although profitable Empire LLP does not have resources sufficient to pay the collectorrsquos claim or the claim of the driverrsquos estate

Assume that the Uniform Partnership Act (1997) applies

1 Before the filing of the statement of qualification (a) was Adam personally liable on the collectorrsquos claim Explain (b) was Diane personally liable on the collectorrsquos claim Explain

2 After the filing of the statement of qualification was Adam Ben or Diane personally liable as a partner on (a) the collectorrsquos claim or (b) the driverrsquos estatersquos claim Explain

8

February 2014 MEE

ANALYSES Constitutional Law

Trusts and Future Interests Secured Transactions

Federal Civil Procedure Criminal Law and Procedure

Agency and Partnership

CONSTITUTIONAL LAW ANALYSIS (Constitutional Law IVD)

ANALYSIS

Legal Problems

(1) Is the city ordinance requirement that businesses install floodlights a taking

(2) Is conditioning the approval of a building permit on the grant of an easement to install surveillance equipment a taking of property

DISCUSSION

Summary

The ordinance requiring businesses to install floodlights is not a per se taking under Loretto because it does not force a private landowner to allow a third party to enter and place a physical object on the land Here the city ordinance requires the businessmdashnot a third partymdashto install the floodlights

The ordinance is likely not a regulatory taking under the Penn Central balancing test While the ordinance will impose a cost on business owners that cost may be offset by the expected increase in business due to the ordinance and the ordinance does not appear to interfere with the ownerrsquos primary use of the property as a restaurant

The permit condition however is likely an uncompensated taking of property While the condition has an essential nexus with the cityrsquos legitimate interest in promoting public safety the city has not made an individualized determination that the easement condition is roughly proportional to the possibility of increased crime due to the restaurantrsquos proposed addition Thus the permit condition likely violates the Fifth Amendment as applied to the states through the Fourteenth Amendment

Point One (50) The ordinance requiring that businesses install floodlights is not a per se taking under Loretto It is not a regulatory taking under the Penn Central balancing test because the cost of compliance with the ordinance may be offset by an expected increase in business and compliance does not interfere with the businessrsquos primary use of its property as a restaurant

The city ordinance requiring a business to install floodlights does not effect a per se taking of the sort described in Loretto v Teleprompter Manhattan CATV Corp 458 US 419 (1982) because no property is physically taken by the government and the ordinance does not involve a physical invasion of private property by a third party

Even though the ordinance does not constitute an occupation of the property by either the government or a third party it is still subject to the three-factor balancing test under Penn Central Transportation Co v City of New York 438 US 104 (1978) to determine whether it is a ldquoregulatory takingrdquo Under Penn Central a court must balance (1) ldquo[t]he economic impact of the regulation on the claimantrdquo (2) ldquothe extent to which the regulation has interfered with distinct investment-backed expectationsrdquo and (3) ldquothe character of the governmental actionrdquo Id at 124 Here each factor weighs against finding that the ordinance is a taking

11

Constitutional Law Analysis

First the ordinance requirement likely has a minimal economic impact on the restaurant Compliance with the ordinance is estimated to cost $1000 and the city has found that businesses will likely recoup that cost in increased sales Also because the ordinance does not interfere with the operation of the restaurant the owner may still earn a reasonable return on its investment in the property

Second the ordinance does not interfere with the businessrsquos investment-backed expectations As in Penn Central the challenged law does not interfere with the ownerrsquos ldquoprimary expectationrdquo for use of the propertymdashin Penn Central as a railroad terminal and here as a restaurant Further the ordinance does not prevent the restaurant from expanding to meet the changing business environment

Third the character of the government action does not weigh in favor of a taking While Penn Central does say that a ldquophysical invasionrdquo is more likely to pose a taking Loretto suggests that the Courtrsquos main concern is with physical invasions by third parties Also like the landmark law challenged in Penn Central the ordinance here ldquoadjust[s] the benefits and burdens of economic life to promote the common goodrdquo Id In Penn Central the landmark law restricted development of the railroad terminal to promote the common interest in preserving historic landmarks Here the ordinance requires the businesses to install floodlights to promote the common interest in crime prevention and public safety

Because the ordinance is clearly a valid exercise of the police power it satisfies the takings clausersquos public-use requirement Kelo v City of New London 545 US 469 (2005)

In sum all three factors weigh against finding a taking under the Penn Central balancing test

Point Two (50) The permit condition may be unconstitutional as an uncompensated taking of property because the city has not made an individualized determination that the easement condition is roughly proportional to the impact of the restaurantrsquos proposed addition

In Dolan v City of Tigard 512 US 374 (1994) the Supreme Court set forth the test for determining whether an exaction imposed by a government in exchange for a discretionary benefit conferred by the government such as a condition on the approval of a building permit in this case constitutes an uncompensated taking under the Fifth Amendment The exaction is not a taking if (1) there is an ldquoessential nexusrdquo between the ldquopublic need or burdenrdquo to which the proposed development contributes and ldquothe permit condition exacted by the cityrdquo id at 386 and (2) the government makes ldquosome sort of individualized determination that the required dedication is [roughly proportional] both in nature and extent to the impact of the proposed developmentrdquo Id at 391 see also Nollan v California Coastal Commission 483 US 825 (1987)

Here the city likely can meet the nexus requirement In Dolan the landowner sought to double the size of its business which would have increased traffic on nearby roadways In exchange for approving the development the city sought an easement for a bike and pedestrian path The Court found the required nexus between the easement and the cityrsquos ldquoattempt to reduce traffic congestion by providing for alternative means of transportationrdquo 512 US at 387 Here a similar nexus likely exists between the requested easement and the cityrsquos interest in crime prevention and public safety Increased patronage and economic activity at the restaurant might attract additional crime to the area and the requested easement to install surveillance equipment would attempt to address that increased crime

12

Constitutional Law Analysis

The exaction here however may fail the second prong of the Dolan testmdashthat the exaction be roughly proportional to the anticipated impact of the requested development As noted the city in Dolan claimed that a bike and pedestrian path was needed to offset the increase in traffic due to the proposed doubling of the business The Court explained that the government must demonstrate that the additional traffic reasonably was related to the requested exaction and that the government must ldquomake some effort to quantify its findings in support of the dedication for the pedestrianbicycle pathway beyond the conclusory statement that it could offset some of the traffic demand generatedrdquo Id at 395 Here the city did not carry its burden The city simply speculates that increased patronage of the restaurant ldquomightrdquo increase crime and that the surveillance equipment ldquomightrdquo alleviate this increased crime Because the city has not made ldquosome effort to quantify its findingsrdquo in support of the easement it has not shown that the burden of the easement is roughly proportional to the benefits thought to flow from it

Thus the exaction appears to be an uncompensated taking of property in violation of the Fifth Amendment as applied to the states through the Fourteenth Amendment

13

TRUSTS AND FUTURE INTERESTS ANALYSIS ____ (Trusts and Future Interests IE3 I5 IIIA amp B)

ANALYSIS

Legal Problems

(1) How should rents dividends and sales proceeds received by the trustee prior to receipt of the sonrsquos letter have been allocated between trust income and principal

(2)(a) Did the remainder interest in the trust accelerate and become immediately payable to the daughterrsquos minor child upon the trusteersquos receipt of the sonrsquos letter and if not how should the trustee handle the distribution of the principal in the future

(2)(b) Following the trusteersquos receipt of the sonrsquos letter how should the trustee distribute future receipts of income prior to the distribution of the principal

DISCUSSION

Summary

Prior to the trusteersquos receipt of the sonrsquos letter cash dividends and rents should have been allocated to trust income and were distributable to the son the income beneficiary of the trust sales proceeds and stock dividends should have been allocated to principal

Because the sonrsquos letter to the trustee did not result in a valid disclaimer under state law (having been made more than nine months after the testatorrsquos death) the son is not deemed to have predeceased the testator Because the son is still living the class gift to the testatorrsquos grandchildren who survive the son has not closed and is not possessory it will not become possessory until the son dies The daughterrsquos minor child being the testatorrsquos only living grandchild is not currently entitled to a distribution of trust principal Trust principal will instead be distributable upon the sonrsquos death to the testatorrsquos then-living grandchildren or if there are none to the testatorrsquos then-living heirs

As for future income the trustee should either distribute the trust income to the son and the daughter as the testatorrsquos heirs accumulate the income for future distribution to those individuals ultimately entitled to the trust principal or distribute it to those presumptively entitled to the principal upon the sonrsquos death ie the daughterrsquos minor child

Point One (45) Cash dividends and rents are allocable to income sales proceeds and stock dividends are allocable to principal Items allocable to income for the period prior to the sonrsquos attempted disclaimer were distributable to the son

Receipts earned during the administration of a trust are allocable either to income or to principal Almost all states have adopted the most recent or an earlier version of the Uniform Principal and Income Act (the Act) which specifies how such receipts should be allocated

Under the Act rents (UNIF PRIN amp INC ACT (2000) sect 405 UNIF PRIN amp INC ACT (1962) sect 3(a)(1)) and cash dividends received from a corporation (UNIF PRIN amp INC ACT (2000) sect 401(b) UNIF PRIN amp INC ACT (1962) sect 6(d)) are allocable to income and are distributable to the income beneficiary of the trust

14

Trusts and Future Interests Analysis Sales proceeds (UNIF PRIN amp INC ACT (2000) sect 404(2) UNIF PRIN amp INC ACT (1962)

sect 3(b)(1)) and dividends paid in the stock of the distributing corporation (UNIF PRIN amp INC ACT (2000) sect 401(c)(1) UNIF PRIN amp INC ACT (1962) sect 3(b)(4)) are allocable to principal and added to the principal of the trust

Here the cash dividends and office building rents should have been allocated to income and until the trustee received the sonrsquos letter should have been distributed to him as the sole income beneficiary of the trust The stock dividend and proceeds from the sale of the office building should have been allocated to principal and held by the trustee for future distribution to the ultimate remaindermen of the trust

[NOTE The 2000 Uniform Principal and Income Act has been adopted in Alabama Arkansas Colorado Connecticut the District of Columbia Hawaii Idaho Iowa Kentucky Missouri Montana Nebraska New Mexico North Dakota Oregon South Dakota Utah and West Virginia]

Point Two(a) (45) Because the son did not disclaim within nine months of the testatorrsquos death there is no valid disclaimer under state law Therefore the son is not deemed to have predeceased the testator Furthermore because of the express survivorship contingency in the will the remainder in the trust does not accelerate and become distributable until the son in fact dies When the son dies the trust principal will be distributable to the testatorrsquos then-living grandchildren or if none then to the testatorrsquos then-living heirs

When a trust remainder is given to a class the class closes (ie no new persons can join the class) when there is no outstanding income interest and at least one member of the class is then entitled to demand possession of his or her share of the remainder This principle is called the rule of convenience See generally HERBERT HOVENKAMP amp SHELDON F KURTZ PRINCIPLES OF PROPERTY LAW 199ndash200 (6th ed 2005) A class member may demand possession of his or her share of the remainder upon termination of the income interest only when the class memberrsquos interest is not otherwise subject to a condition precedent See id

When a beneficiary timely disclaims an interest in a trust that beneficiary is treated as if he had predeceased the testator Here had the son disclaimed within nine months of the testatorrsquos death as required by the state statute he would have been deemed to have predeceased the testator This would have closed the class of remaindermen and the testatorrsquos then-living grandchildren (ie the daughterrsquos child) would have been entitled to the trust principal However under the state statute the sonrsquos disclaimer was not timely because he did not disclaim within nine months of the testatorrsquos death Thus because the statute is inapplicable and the son is still alive the class of grandchildren entitled to share in trust principal did not close

Because here the statute is inapplicable due to the sonrsquos failure to comply with the statutory time requirements then presumably the common-law rule allowing disclaimers (aka renunciations) at any time should apply Under the common law if a life estate is renounced the remainder interest accelerates and becomes immediately distributable to the remaindermen of the trust if the remainder is vested but not if the remainder is contingent JESSE DUKEMINIER amp ROBERT H SITKOFF WILLS TRUSTS AND ESTATES 844ndash845 (9th ed 2013) Here because the remainder is contingent upon there being grandchildren who survive the son the remainder will not accelerate It will remain open until the son dies leaving open the possibility that additional grandchildren will be included in the class or the daughterrsquos child could fall out of the class because that child fails to survive the son

And if none of the testatorrsquos grandchildren survive the son the trust principal will be distributed to the testatorrsquos heirs living at the sonrsquos death

15

Trusts and Future Interests Analysis

Point Two(b) (10) Until the trust terminates the trustee must continue to hold the trust assets The distribution of income in the meantime is unclear There are at least three possibilities Income earned on the undistributed assets could be distributed to the son and daughter as the testatorrsquos heirs accumulated and added to principal for distribution to the ultimate remaindermen or distributed from time to time to those persons who are presumptively remaindermen

When trust principal is not immediately distributable the trustee must continue to hold trust assets until the ultimate remaindermen are ascertained During this period trust income will be distributed or retained according to any instructions contained in the trust instrument See WILLIAM M MCGOVERN JR SHELDON F KURTZ amp DAVID M ENGLISH WILLS TRUSTS amp ESTATES sect 102 (4th ed 2010)

Here the testator did not specify what the trustee should do with trust income in the event the sonrsquos disclaimer did not comply with the state statute There are at least three approaches One approach would have the trustee distribute the trust income to the testatorrsquos heirs on the theory that the income represents property that was not disposed of by the testatorrsquos will and which thus passes by partial intestacy to the testatorrsquos heirs A second approach would have the trustee accumulate trust income for distribution to the ultimate remaindermen Under this approach only those individuals ultimately entitled to the principal would share in the income A third approach would have the trustee distribute trust income to those individuals who would be the remaindermen if the trust were to terminate when the income is received by the trustee under this approach trust income would be distributed to the daughterrsquos minor child until another presumptive remainderman is born This approach could result in individuals not ultimately entitled to principal say because they do not survive the son receiving income It could also result in a disproportionate distribution of income among the individuals ultimately entitled to income

[NOTE Examinees should demonstrate a recognition and understanding of the income-allocation problem and the alternatives available to address that issue There is no widely accepted solution to the problem Examinees who cite any of these possible problem-solving approaches may receive credit]

16

SECURED TRANSACTIONS ANALYSIS (Secured Transactions IB IID E amp F IIIB IVA B amp F)

ANALYSIS

Legal Problems

(1)(a) What is the nature of the bankrsquos claim to the businessrsquos equipment

(1)(b) What is the nature of the finance companyrsquos claim to the businessrsquos equipment

(1)(c) As between the bank and the finance company whose claim to the businessrsquos equipment has priority

(2) Do the claims of the bank and the finance company continue in the item of equipment sold by the business to the competitor

DISCUSSION

Summary

The bank and the finance company both have perfected security interests in the businessrsquos equipment Even though the finance companyrsquos perfected security interest was created first the bankrsquos perfected security interest has priority because the bankrsquos financing statement was filed before the finance companyrsquos financing statement The security interests of the bank and the finance company continue in the item of equipment sold by the business to the competitor because their security interests were perfected and the competitor was not a buyer in ordinary course of business

Point One(a) (25) The bank has a perfected security interest in the businessrsquos equipment

The bank has met all criteria necessary for it to have an attached and enforceable security interest in the businessrsquos equipment First value must be given UCC sect 9-203(b)(1) This criterion is fulfilled by the loan made by the bank to the business Second the debtor must have rights in the collateral UCC sect 9-203(b)(2) Clearly the business has rights in its equipment Third either the secured party must take possession of the collateral or the debtor must authenticate a security agreement containing a description of the collateral UCC sect 9-203(b)(3) The agreement that the business owner signed is a ldquosecurity agreementrdquo because it is an agreement that creates or provides for a security interest UCC sect 9-102(a)(74) By signing the security agreement the business owner authenticated it UCC sect 9-102(a)(7) Therefore all three criteria are fulfilled and the bank has an enforceable and attached security interest

A security interest is perfected when it has attached and when any additional steps required for perfection have occurred UCC sect 9-308(a) Generally speaking the additional steps will either be possession of the collateral by the secured party or the filing of a financing statement with respect to the collateral See UCC sectsect 9-310 9-313 In this case the bank filed a financing statement naming the debtor and sufficiently indicating the collateral The collateral indication is sufficient because it identifies the collateral by type of property See UCC sectsect 9-504 9-108 The fact that the financing statement was filed before the security interest was created is

17

Secured Transactions Analysis

not a problem Even though the security agreement had not yet been signed the business had authorized the filing of the financing statement in an authenticated record UCC sect 9-509(a)(1) Moreover the financing statement may be filed before the security agreement is created UCC sect 9-502(d)

Point One(b) (10) The finance company also has a perfected security interest in the businessrsquos equipment

The finance companyrsquos security interest is enforceable and attached for the same reasons as the bankrsquos security interest The loan from the finance company to the business constitutes value the business has rights in the collateral and the business owner has authenticated a security agreement containing a description of the collateral The finance companyrsquos security interest is perfected because the finance company filed a financing statement with respect to it that provides that the business is the debtor and indicates that the collateral is equipment

Point One(c) (30) The bankrsquos security interest has priority over the finance companyrsquos security interest because the bankrsquos financing statement was filed first

As between two perfected security interests the general rule is that the security interest that was the earlier to be either perfected or the subject of a filed financing statement has priority UCC sect 9-322(a)(1) While the finance companyrsquos security interest was perfected before the bankrsquos (March 15 vs March 22) the bankrsquos financing statement was filed even earlier on March 2 Thus under the first-to-file-or-perfect rule of UCC sect 9-322(a)(1) the bankrsquos security interest has priority No exceptions to the general rule apply here

Point Two (35) A security interest in collateral continues notwithstanding its sale unless an exception applies Because the security interests of the bank and the finance company were perfected and the competitor was not a buyer in ordinary course of business no exception applies and the security interests of both creditors continue in the equipment sold to the competitor

As a general rule a security interest in collateral continues notwithstanding the fact that the debtor has sold the collateral to another person UCC sect 9-315(a)(1) Thus unless an exception applies the security interests of the bank and the finance company will continue in the item of equipment sold to the competitor

A buyer of goods will take free of an unperfected security interest in those goods See UCC sect 9-317(a)(2) However when the competitor bought the businessrsquos equipment both the bank and the finance company had perfected security interests in the equipment

A buyer can take free even of a perfected security interest in goods if the buyer is a ldquobuyer in ordinary course of businessrdquo See UCC sect 9-320(a) However the competitor was not a buyer in ordinary course of business To be a ldquobuyer in ordinary course of businessrdquo a buyer must buy goods from a seller that is in the business of selling goods of that kind See UCC sect 1-201(b)(9) The competitor bought this equipment from a seller that is not in the business of selling goods of this kind so the competitor was not a buyer in ordinary course of business with respect to these goods

Because no exception applies the security interests of the bank and the finance company continue even after the item of equipment was sold to the competitor

18

FEDERAL CIVIL PROCEDURE ANALYSIS (Federal Civil Procedure IVD)

ANALYSIS

Legal Problems

(1) Is a document prepared in the course of a contract dispute protected from discovery as ldquowork productrdquo when there is no evidence that the document was prepared in anticipation of litigation

(2)(a) Is a partyrsquos failure to provide relevant electronically stored information excused when the information was destroyed pursuant to a routine document retention scheme at a time when litigation was contemplated by the destroying party

(2)(b) What sanctions should be imposed on a party for allowing the destruction of evidence that is relevant to potential future litigation

DISCUSSION

Summary

The report prepared by the structural engineer is probably not work product and is thus discoverable The engineer examined the foundation of the house at the customerrsquos request and the engineerrsquos findings are potentially relevant to the customerrsquos claim that the foundation is defective The report was not prepared in anticipation of litigation The customer appears to have sought the engineerrsquos opinion in response to the builderrsquos offer to fix any problems with the foundation that an engineer might identify Because the report was not prepared in anticipation of litigation it is not protected by the work-product doctrine

The builder should have taken appropriate steps to preserve evidence including suspending its document retention program as soon as it began planning for litigationmdashie on July 10 Its destruction of potentially relevant material after that date was wrongful However a court is unlikely to impose severe sanctions on the builder because there are no facts indicating that the builder acted in bad faith and the customer can prove that the foundation is defective without the destroyed emails

Point One (40) The customer must turn over the engineerrsquos report because it was not prepared in anticipation of litigation

In general a party to a lawsuit in federal court ldquomay obtain discovery regarding any nonprivileged matter that is relevant to any partyrsquos claim or defenserdquo FED R CIV P 26(b)(1) (2009) This includes the right to inspect and copy documents in the other partyrsquos possession FED R CIV P 34(a)(1) Here the customer hired a structural engineer to examine the foundation of the house The engineerrsquos report on the foundation is likely to include information that would be relevant to the customerrsquos claim that the foundation was defectively constructed

The so-called ldquowork productrdquo rule allows a party to refuse to turn over ldquodocuments that are prepared in anticipation of litigation or for trialrdquo by that partyrsquos representative including

19

Federal Civil Procedure Analysis

a consultant Thus if the customer had hired the structural engineer to prepare a report ldquoin anticipation of litigationrdquo that report might not be discoverable See FED R CIV P 26(b)(3)

In this case however the customer hired the engineer to evaluate the foundation of the house as part of the customerrsquos negotiation with the builder concerning the housersquos flooding problem The builder told the customer that the housersquos landscaping was the reason for the flooding and the builder told the customer ldquoHave an engineer look at the foundation If therersquos a problem wersquoll fix itrdquo The customer appears to have acted in response to that statement There is no indication that the customer anticipated any kind of legal action at the time that the structural engineer was hired Accordingly the structural engineerrsquos report is discoverable and the court should order the customer to turn it over

[NOTE If an examinee concludes that the structural engineerrsquos report was prepared in anticipation of litigation then the examinee should also conclude that the report is not discoverable Documents prepared in anticipation of litigation do not need to be disclosed to an adverse party unless that party can demonstrate a ldquosubstantial needrdquo for the documents and an inability to obtain substantially equivalent information without ldquoundue hardshiprdquo FED R CIV P 26(b)(3)(A)(ii) Furthermore a report prepared by an expert who is not expected to testify is not discoverable in the absence of ldquoexceptional circumstancesrdquo making it ldquoimpracticablerdquo to obtain the information in another way FED R CIV P 26(b)(4)(D)(ii) The builder probably cannot make these showings here unless the engineerrsquos report deals with circumstances that have since changed There is no evidence that the structural engineer would have had access to any information or facts that the builder would not already know as a result of its construction and subsequent inspection of the house In addition if necessary the builder could ask the court for permission to arrange for a further inspection of the house by a structural engineer hired by the builder See FED R CIV P 34(a)(2) Accordingly if an examinee concludes that the report was prepared in anticipation of litigation the examinee should also conclude that the builder is not entitled to see the report]

Point Two(a) (30) Because the builder anticipated that it might be involved in litigation concerning its contract with the customer the builder acted wrongfully in destroying emails that were relevant to the housersquos construction even though the emails were destroyed pursuant to a routine document retention plan

As noted above a party to a lawsuit in federal court ldquomay obtain discovery regarding any nonprivileged matter that is relevant to any partyrsquos claim or defenserdquo FED R CIV P 26(b)(1) This includes emails and other electronically stored information FED R CIV P 34(a)(1)(A) Here the customer has requested all the builderrsquos emails pertaining to work done on the foundation of the house Ordinarily the builder would be obliged to turn over this information which is relevant to the customerrsquos defense that the housersquos foundation was poorly constructed

Unfortunately the emails in question no longer exist because the builder destroyed them on August 2

In general spoliation of evidence (destruction or alteration of evidence) is improper if the party who destroyed or altered the evidence ldquohas notice that the evidence is relevant to litigation or should have known that the evidence may be relevant to future litigationrdquo Fujitsu Ltd v Federal Express Corp 247 F3d 423 436 (2d Cir 2001) It is improper for a party to destroy electronic information relevant to pending litigation even if the destruction occurs before there is any request or order seeking the information See eg Leon v IDX Sys Corp 464 F3d 951 (9th Cir 2006) (plaintiffrsquos intentional destruction of computer files warranted dismissal even

20

In this case the builderrsquos destruction of the emails was pursuant to a routine document retention plan The Federal Rules provide expressly that in the absence of ldquoexceptional circumstancesrdquo parties should not be sanctioned for the loss of electronically stored information when the loss occurs pursuant to ldquoroutine good-faith operation of an electronic information systemrdquo FED R CIV P 37(e) However when a party anticipates litigation ldquoit must suspend its routine document retentiondestruction policy and put in place a lsquolitigation holdrsquo to ensure the preservation of relevant documentsrdquo Zubulake v UBS Warburg LLC 220 FRD 212 218 (SDNY 2003)

Federal Civil Procedure Analysis

though spoliation occurred before order compelling discovery) Similarly the duty to preserve evidence applies to a party who anticipates litigation even if litigation has not yet been commenced See THE SEDONA PRINCIPLES BEST PRACTICES RECOMMENDATIONS amp PRINCIPLES FOR ADDRESSING ELECTRONIC DOCUMENT PRODUCTION 70 cmt 14a (2d ed 2007)

The builder destroyed the emails on August 2 At that time the builder knew that litigation was a possibility because the builder had already directed its attorney to prepare a draft complaint for possible filing Knowing that litigation was a possibility the builder had a duty to take steps to preserve evidence including the emails in question See generally Fujitsu Ltd

Thus the builderrsquos destruction of potentially relevant emails at a time when it knew that litigation was a possibility was improper It had a duty to preserve evidence and it breached that duty

[NOTE Because courts have used different words to describe the test for when evidence must be preserved an examineersquos precise formulation of the test is not critical]

Point Two(b) (30) In determining appropriate sanctions for spoliation courts consider both the level of culpability of the spoliating party and the degree of prejudice the loss of evidence has caused the other party Here the builderrsquos destruction of evidence does not appear to have been willful nor is it likely to pose a significant obstacle to the customerrsquos defense Any sanctions imposed by the court should be modest

Federal courts have inherent power to control the litigation process and can sanction misbehavior including spoliation even when there has been no specific violation of the Federal Rules of Civil Procedure See generally Chambers v NASCO Inc 501 US 32 (1991) (discussing courtrsquos inherent power to control the litigation process) The range of available sanctions is broad It can include such sanctions as the payment of expenses incurred by the other party as a result of the destruction of the evidence an instruction to the jury authorizing it to draw an adverse inference from the destruction of the evidence a shifting of the burden of proof on the relevant issue or even judgment against the responsible party See eg Residential Funding Corp v DeGeorge Financial Corp 306 F3d 99 108 (2d Cir 2002) (adverse inference) Silvestri v General Motors Corp 271 F3d 583 593 (4th Cir 2001) (possibility of dismissal) Cf FED R CIV P 37(b)(2)(A) (listing remedies for failure to comply with discovery obligations)

In determining appropriate sanctions for spoliation courts consider both the level of culpability of the spoliating party and the degree of prejudice the loss of evidence has caused the other party Many courts impose severe sanctions (such as an adverse-inference instruction or the entry of judgment against the spoliating party) only when there is evidence of bad faith in the form of an intentional effort to hide information Eg Greyhound Lines Inc v Wade 485 F3d 1032 1035 (8th Cir 2007) (spoliation sanction requires intentional destruction out of desire ldquoto suppress the truthrdquo) However other courts have said that negligence in preserving evidence can

21

Federal Civil Procedure Analysis

support an adverse-inference instruction See Residential Funding 306 F3d at 108 (negligence enough under some circumstances)

Although a court might well order an evidentiary hearing on the issue of sanctions the facts presented do not seem appropriate for severe sanctions First the evidence was destroyed pursuant to the builderrsquos standard document retention plan and there is no evidence that the builder deliberately failed to suspend its usual procedures with the purpose of allowing the destruction of evidence Second the loss of this evidence will not severely hinder the customerrsquos presentation of his case The central issue is whether the foundation of the house was properly constructed If the construction job was poorly done the customer can present evidence derived from inspection of the premises to prove that point The customer can also depose witnesses about any issues that arose during construction

Under the circumstances a court is not likely to impose particularly severe sanctions although it might shift the burden to the builder to show that the foundation was properly constructed or it might require the builder to reimburse any expenses the customer incurs to discover and prove the facts about issues or disputes that arose during construction of the foundation

[NOTE The result reached by the examinee is less important than the examineersquos recognition that (a) a range of sanctions is available to the court and (b) the appropriate sanction depends both on the culpability of the builder and the prejudice suffered by the customer]

22

CRIMINAL LAW AND PROCEDURE ANALYSIS (Criminal Law and Procedure IIA amp D VE amp F)

ANALYSIS

Legal Problems

(1) Did charging the defendant with both theft and burglary constitute double jeopardy

(2) Did the jury instruction violate the due process clause either by relieving the prosecution of the burden of proving the element of intent or by shifting the burden to the defendant to disprove that element

(3) Did the sentence imposed in this case for the theft conviction unconstitutionally deprive the defendant of his right to a jury trial on the issue of the value of the stolen item

DISCUSSION

Summary

The trial court properly denied the defendantrsquos pretrial motion to dismiss the charges on double jeopardy grounds The defendant may be charged with and convicted of both theft and burglary Each of the charges has an element that the other does not Neither charge is a lesser-included offense nor are they multiplicitous Thus charging both theft and burglary does not violate double jeopardy

The jury instruction on the burglary charge was constitutionally flawed It could have been reasonably understood by the jury as either (1) an irrebuttable conclusive presumption (which relieved the prosecution of proving the element of intent and removed the issue from the jury) or (2) a rebuttable mandatory presumption (which unconstitutionally shifted the burden of proof on an element of a charged offense from the prosecution to the defendant)

Because the four-year sentence imposed by the judge was based on the judgersquos finding by a preponderance of the evidence that the value of the stolen ring exceeded $5000 the sentence violates the defendantrsquos right to a jury determination beyond a reasonable doubt of the value of the ring

Point One (30) Charging the defendant with theft and burglary did not constitute double jeopardy

The Double Jeopardy Clause of the Fifth Amendment provides that a person shall not be twice put in jeopardy for the ldquosame offenserdquo Thus the question is whether the elements of the theft charge are wholly contained in the burglary charge or vice versa If the elements of the lesser charge (theft) are not wholly contained in the greater charge (burglary)mdashie if each charge requires proof of a fact that the other does notmdashthen convicting the defendant of both crimes would not violate double jeopardy even when the two offenses occurred at the same time and are thus arguably part of the ldquosame transactionrdquo Blockburger v United States 284 US 299 304 (1932) See also Albernaz v United States 450 US 333 344 n3 (1981) United States v Dixon 509 US 688 704 (1993)

23

Criminal Law and Procedure Analysis

Here theft and burglary each require proof of an element not required for the other crime Burglary may be defined differently in different jurisdictions However it almost invariably requires entry into a building or dwelling of another with the specific intent to commit a felony therein and the crime of burglary is complete upon the entry into the building or dwelling with such intent See eg Cannon v Oklahoma 827 P2d 1339 1342 (Okla Crim App 1992) In contrast theft which also may be defined differently in different states almost invariably requires the taking and carrying away of an item of personal property belonging to another with the intent to steal or permanently deprive the owner of possession

Here the ldquotakingrdquo or ldquostealingrdquo element is not contained in the definition of burglary and the ldquoentryrdquo element of burglary is not contained in the definition of theft Because theft is not a lesser-included offense of burglary and burglary is not a lesser-included offense of theft charging the defendant for both burglary and theft did not violate double jeopardy and the court properly denied the defense motion on those grounds Yparrea v Dorsey 64 F3d 577 579ndash80 (10th Cir 1995) citing Blockburger 284 US at 304

Finally the defendantrsquos motion to dismiss all the charges on double jeopardy grounds was improper because if both charges were for the same offense the motion should have requested dismissal of one charge not both

Point Two (35) The jury instruction on the burglary charge violated the Due Process Clause because it created either (1) an irrebuttable conclusive presumption (which relieved the prosecution of proving the element of intent and removed that issue from the jury) or (2) a rebuttable mandatory presumption (which unconstitutionally shifted the burden of proof on an element of a charged offense to the defendant)

The Supreme Court has interpreted the Due Process Clause of the US Constitution to require that the prosecution prove all elements of an offense beyond a reasonable doubt See In re Winship 397 US 358 364 (1970) The burden of proof cannot be shifted to the defendant by presuming an essential element upon proof of other elements of the offense because shifting the burden of persuasion with respect to any element of a criminal offense is contrary to the Due Process Clause See Mullaney v Wilbur 421 US 684 (1975)

The crime of burglary includes entry into a building or dwelling with the specific intent to commit a felony therein The requirement that the prosecutor prove beyond a reasonable doubt that the defendant had this specific intent distinguishes burglary from general-intent crimes like trespass See Sandstrom v Montana 442 US 510 523 (1979)

Here the jury was instructed that if ldquoafter consideration of all the evidence presented by the prosecution and defense you find beyond a reasonable doubt that the defendant entered the dwelling without the ownersrsquo consent you may presume that the defendant entered with the intent to commit a felony thereinrdquo This instruction was unconstitutional because it created either an irrebuttable conclusive presumption or a rebuttable mandatory presumption

A conclusive presumption is ldquoan irrebuttable direction by the court to find intent once convinced of the facts triggering the presumptionrdquo Id at 517 Here the jurors were instructed that once the prosecutor established that the defendant entered the neighborsrsquo house without consent they ldquomay presumerdquo that he intended to commit a felony therein The jurors may have reasonably concluded from this instruction that if they found that the defendant intended to enter his neighborsrsquo home without permission they must further find that he entered with the specific intent to commit a felony therein Because this instruction could operate as a conclusive

24

Criminal Law and Procedure Analysis

irrebuttable presumption by eliminating intent ldquoas an ingredient of the offenserdquo it violated due process by relieving the prosecution of the burden of proof for this element Id at 522

In the alternative the jury instruction could have been reasonably understood to create a rebuttable mandatory presumption which ldquotells [the jury] they must find the elemental fact upon proof of the basic fact at least unless the defendant has come forward with some evidence to rebut the presumed connection between the two factsrdquo County Court of Ulster County New York v Allen 442 US 140 157 (1979) The due process problem created by rebuttable mandatory presumptions is that ldquo[t]o the extent that the trier of fact is forced to abide by the presumption and may not reject it based on an independent evaluation of the particular facts presented by the State the analysis of the presumptionrsquos constitutional validity is logically divorced from those facts and based on the presumptionrsquos accuracy in the run of casesrdquo Id at 159

Unlike irrebuttable conclusive presumptions rebuttable mandatory presumptions are not always per se violations of the Due Process Clause However the Supreme Court of the United States has held that jury instructions that could reasonably be understood as shifting the burden of proof to the defendant on an element of the offense are unconstitutional Francis v Franklin 471 US 307 (1985) Here the argument that the jury instruction operated as a rebuttable mandatory presumption is supported by the fact that the judge also instructed the jury to ldquoconsider[ ] all the evidence presented by the prosecution and defenserdquo However even if the instruction created a rebuttable mandatory presumption it would be unconstitutional because it shifted the burden to the defense on an element of the offense Sandstrom 442 US at 524 Mullaney 421 US at 686

[NOTE Whether an examinee identifies the jury instruction as containing a ldquoconclusiverdquo or ldquomandatoryrdquo presumption is less important than the examineersquos analysis of the constitutional infirmities]

Point Three (35) The trial court violated the defendantrsquos Sixth Amendment right to a jury trial on an essential element of the offense when it found by a preponderance of the evidence that the ring was worth over $5000 and increased the defendantrsquos sentence based on this finding

In the statutory scheme under which the defendant was tried and convicted a Class D felony theft is defined as theft of item(s) with a value between $2500 and $10000 The jury found that the value of the diamond ring was at least $2500 and convicted the defendant of felony theft However at sentencing the trial court made a separate finding by a preponderance of the evidence that the value of the ring was greater than $5000 Following the statutersquos two-tiered sentencing scheme the judge then imposed on the defendant a sentence that was one year longer than the maximum that would otherwise have been allowed

The judgersquos sentence was unconstitutional because it violated the defendantrsquos Sixth Amendment right to a jury trial on this question The Supreme Court held in Apprendi v New Jersey 530 US 466 (2000) that ldquo[o]ther than the fact of a prior conviction any fact that increases the penalty for a crime beyond the prescribed statutory maximum must be submitted to a jury and proved beyond a reasonable doubtrdquo because ldquo[i]t is unconstitutional for a legislature to remove from the jury the assessment of facts that increase the prescribed range of penalties to which a criminal defendant is exposed [because] such facts must be established by proof beyond a reasonable doubtrdquo Id The Court reaffirmed Apprendi in Blakely v Washington 542 US 296 (2004) holding that the ldquolsquostatutory maximumrsquo for Apprendi purposes is the maximum sentence a judge may impose solely on the basis of the facts reflected in the jury verdict or admitted by the defendantrdquo Id at 303 (emphasis in original) In United States v Booker 543 US 220 (2005)

25

Criminal Law and Procedure Analysis

the Court relied on Blakely and Apprendi to conclude that protecting a defendantrsquos Sixth Amendment right to a jury trial required that ldquo[a]ny fact which is necessary to support a sentence exceeding the maximum authorized by the facts established by a plea of guilty or a jury verdict must be admitted by the defendant or proved to a jury beyond a reasonable doubtrdquo Id at 244

Thus in order to constitutionally increase a sentence above the statutory maximum of three years the jury must have found beyond a reasonable doubt that the value of the ring exceeded $5000 Here the court made the finding based on an appraisal proffered by the prosecutor only at sentencing and the judgersquos finding was by a preponderance of the evidence rather than beyond a reasonable doubt

26

AGENCY AND PARTNERSHIP ANALYSIS __________ (Agency and Partnership VA amp C VI)

ANALYSIS

Legal Problems

(1) Is a partner in a general partnership personally liable on a claim arising from misrepresentations by another partner made in the course of the partnership business

(2) Does a newly admitted partner in a general partnership become personally liable on existing claims against the partnership

(3) After the filing by a general partnership of a statement of qualification as a limited liability partnership are the partners personally liable as partners on (a) an existing claim against the general partnership and (b) a claim against the partnership that arose after the filing

DISCUSSION

Summary

Adam and Ben formed a general partnership under which they were jointly and severally liable for obligations of the partnership Thus Adam was personally liable for misrepresentations by Ben made in the ordinary course of the partnership business

Upon joining the general partnership Diane became personally liable for the obligations of the partnership arising after her admission but not for obligations pre-existing her admission such as the collectorrsquos claim

By filing a statement of qualification the three partners properly elected limited liability partnership status As partners in an LLP none of the three partners is personally liable as a partner for partnership obligations arising after the election such as the claim by the driverrsquos estate The election however does not change their personal liability on pre-existing claims that arose before the election such as the collectorrsquos claim

Point One (30) As a general partner of Empire a general partnership Adam became personally liable on the collectorrsquos claim a valid claim against the partnership that arose because of Benrsquos wrongful act in the ordinary course of the partnership business

When the collectorrsquos claim arose Empire was a general partnership composed of Adam and Ben Under UPA (1997) sect 306(a) partners of a general partnership are liable jointly and severally for all obligations of the partnership Under UPA (1997) sect 305(a) the partnership could become obligated for the loss caused to the collector as a result of the misrepresentation by Ben provided he was acting in the ordinary course of the partnership business Because there was no statement that limited his partnership authority Ben as partner was ldquoan agent of the partnership for the purpose of its businessrdquo See UPA (1997) sect 301(1) Benrsquos misrepresentation to the collector even if intentional appears to be in the ordinary course of the partnershiprsquos business of dealing

27

Agency and Partnership Analysis

in antique cars Thus Benrsquos wrongful act created a partnership obligation for which Adam was jointly and severally liable

[NOTE Generally a partnership creditor must ldquoexhaust the partnershiprsquos assets before levying on a judgment debtor partnerrsquos individual property where the partner is personally liable for the partnership obligationrdquo as a result of his status as a partner UPA (1997) sect 307 cmt 4 As the UPA comments explain this places Adam more in the position of guarantor than principal debtor on the partnership obligation Id cmt 4 Although an examinee might discuss this point the call focuses on whether Adam is personally liable not how the liability might be enforced]

Point Two (30) Because the collectorrsquos claim arose before Diane joined Empire Diane did not become personally liable on the claim

Diane was admitted to Empire when it was a general partnership and after the collectorrsquos claim arose While the general rule under UPA (1997) sect 306(a) is that the partners of a general partnership are liable jointly and severally for all obligations of the partnership there is a special rule for partners who are admitted during the duration of the partnership Under UPA (1997) sect 306(b) a person admitted to an existing partnership is not personally liable for any partnership obligations incurred before the personrsquos admission Because Diane was admitted to Empire after the collectorrsquos claim arose Diane is not personally liable on the claim

Dianersquos knowledge of the pre-existing claim and her stated concern about becoming liable on the collectorrsquos claim do not change her personal nonliability to the collector Although partners who have a liability shield can assume liability to third parties through private contractual guarantees or modifications to the partnership agreement Dianersquos stated concern constituted neither a guaranty to the collector nor ldquoan intentional waiver of liability protectionsrdquo See UPA (1997) sect 306 cmt 3 (describing methods for waiver of liability protections under sect 306(c) applicable in limited liability partnerships)

At most Diane will lose her investment in the partnership as a result of the collectorrsquos claim Although Diane did not become personally liable on the collectorrsquos claim when she joined the partnership the $250000 she contributed to the partnership is ldquoat risk for the satisfaction of existing partnership debtsrdquo UPA (1997) sect 306 cmt 2

Point Three (40) Filing the statement of qualification was effective to elect limited liability partnership status Despite this new status Adam and Ben remain personally liable on the collectorrsquos claim which arose before the election But as partners in an LLP neither Adam Ben nor Diane is personally liable as a partner on the driverrsquos estatersquos claim which arose after the election

Under UPA (1997) sect 1001 a general partnership can make an election and become a limited liability partnershipmdashif the partners approve the conversion by a vote equivalent to that necessary to amend the partnership agreement and the partnership then files a statement of qualification that specifies the name of the partnership its principal office and its election to be an LLP Here the partners agreed unanimouslymdashsufficient to amend their agreement under UPA (1997) sect 401(j)mdashand the statement of qualification was filed In addition the name of Empire LLP properly included an appropriate ending ldquoLLPrdquo See UPA (1997) sect 1002

Although another way to effectuate a ldquoconversionrdquo (as suggested by Benrsquos lawyer) is to form a new LLP and transfer the assets of the old general partnership to the new LLP the

28

Agency and Partnership Analysis

method used here (approval by the partners and the filing of a statement of qualification) is also sufficient to create LLP status

Thus Empire became Empire LLP as of the date of filing of the statement of qualification See UPA (1997) sect 1001 What effect did this have on the collectorrsquos claim which predated the filing According to UPA (1997) sect 306(c) an obligation incurred while a partnership is an LLP is solely a partnership obligation As the collectorrsquos claim predated the LLP Adam and Ben remain personally liable on the collectorrsquos claim Diane on the other hand was not personally liable on the collectorrsquos claim either before or after the filing of the statement of qualification See Point Two above

The driverrsquos estatersquos claim arose after Empire became Empire LLP Under UPA (1997) sect 306(c) an obligation incurred while a partnership is an LLP is solely a partnership obligationThus Adam Ben and Diane as partners are all protected from personal liability on the driverrsquos estatersquos claim But there may be personal liability if any of them was negligent or otherwise acted wrongfully by not informing the buyer of the bad suspension that caused the accident

29

National Conference of Bar Examiners 302 South Bedford Street | Madison WI 53703-3622 Phone 608-280-8550 | Fax 608-280-8552 | TDD 608-661-1275

wwwncbexorg e-mail contactncbexorg

  • Preface
  • Description of the MEE
  • Instructions
  • February 2014 Questions
    • Constitutinal Law Question
    • Trusts and Future Interests Question
    • Secured Transactions Question
    • Federal Civil Procedure Question
    • Criminal Law and Procedure Question
    • Agency and Partnership Question
      • February 2014 Analyses
        • Constitutional Law Analysis
        • Trust and Future Interests Analysis
        • Secured Transactions Analysis
        • Federal Civil Procedure Analysis
        • Criminal Law and Procedure Analysis
        • Agency and Partnership Analysis
            • ltlt13 ASCII85EncodePages false13 AllowTransparency false13 AutoPositionEPSFiles true13 AutoRotatePages None13 Binding Left13 CalGrayProfile (Dot Gain 20)13 CalRGBProfile (sRGB IEC61966-21)13 CalCMYKProfile (US Web Coated 050SWOP051 v2)13 sRGBProfile (sRGB IEC61966-21)13 CannotEmbedFontPolicy Error13 CompatibilityLevel 1413 CompressObjects Tags13 CompressPages true13 ConvertImagesToIndexed true13 PassThroughJPEGImages true13 CreateJobTicket false13 DefaultRenderingIntent Default13 DetectBlends true13 DetectCurves 0000013 ColorConversionStrategy CMYK13 DoThumbnails false13 EmbedAllFonts true13 EmbedOpenType false13 ParseICCProfilesInComments true13 EmbedJobOptions true13 DSCReportingLevel 013 EmitDSCWarnings false13 EndPage -113 ImageMemory 104857613 LockDistillerParams false13 MaxSubsetPct 10013 Optimize true13 OPM 113 ParseDSCComments true13 ParseDSCCommentsForDocInfo true13 PreserveCopyPage true13 PreserveDICMYKValues true13 PreserveEPSInfo true13 PreserveFlatness true13 PreserveHalftoneInfo false13 PreserveOPIComments true13 PreserveOverprintSettings true13 StartPage 113 SubsetFonts true13 TransferFunctionInfo Apply13 UCRandBGInfo Preserve13 UsePrologue false13 ColorSettingsFile ()13 AlwaysEmbed [ true13 ]13 NeverEmbed [ true13 ]13 AntiAliasColorImages false13 CropColorImages true13 ColorImageMinResolution 30013 ColorImageMinResolutionPolicy OK13 DownsampleColorImages true13 ColorImageDownsampleType Bicubic13 ColorImageResolution 30013 ColorImageDepth -113 ColorImageMinDownsampleDepth 113 ColorImageDownsampleThreshold 15000013 EncodeColorImages true13 ColorImageFilter DCTEncode13 AutoFilterColorImages true13 ColorImageAutoFilterStrategy JPEG13 ColorACSImageDict ltlt13 QFactor 01513 HSamples [1 1 1 1] VSamples [1 1 1 1]13 gtgt13 ColorImageDict ltlt13 QFactor 01513 HSamples [1 1 1 1] VSamples [1 1 1 1]13 gtgt13 JPEG2000ColorACSImageDict ltlt13 TileWidth 25613 TileHeight 25613 Quality 3013 gtgt13 JPEG2000ColorImageDict ltlt13 TileWidth 25613 TileHeight 25613 Quality 3013 gtgt13 AntiAliasGrayImages false13 CropGrayImages true13 GrayImageMinResolution 30013 GrayImageMinResolutionPolicy OK13 DownsampleGrayImages true13 GrayImageDownsampleType Bicubic13 GrayImageResolution 30013 GrayImageDepth -113 GrayImageMinDownsampleDepth 213 GrayImageDownsampleThreshold 15000013 EncodeGrayImages true13 GrayImageFilter DCTEncode13 AutoFilterGrayImages true13 GrayImageAutoFilterStrategy JPEG13 GrayACSImageDict ltlt13 QFactor 01513 HSamples [1 1 1 1] VSamples [1 1 1 1]13 gtgt13 GrayImageDict ltlt13 QFactor 01513 HSamples [1 1 1 1] VSamples [1 1 1 1]13 gtgt13 JPEG2000GrayACSImageDict ltlt13 TileWidth 25613 TileHeight 25613 Quality 3013 gtgt13 JPEG2000GrayImageDict ltlt13 TileWidth 25613 TileHeight 25613 Quality 3013 gtgt13 AntiAliasMonoImages false13 CropMonoImages true13 MonoImageMinResolution 120013 MonoImageMinResolutionPolicy OK13 DownsampleMonoImages true13 MonoImageDownsampleType Bicubic13 MonoImageResolution 120013 MonoImageDepth -113 MonoImageDownsampleThreshold 15000013 EncodeMonoImages true13 MonoImageFilter CCITTFaxEncode13 MonoImageDict ltlt13 K -113 gtgt13 AllowPSXObjects false13 CheckCompliance [13 None13 ]13 PDFX1aCheck false13 PDFX3Check false13 PDFXCompliantPDFOnly false13 PDFXNoTrimBoxError true13 PDFXTrimBoxToMediaBoxOffset [13 00000013 00000013 00000013 00000013 ]13 PDFXSetBleedBoxToMediaBox true13 PDFXBleedBoxToTrimBoxOffset [13 00000013 00000013 00000013 00000013 ]13 PDFXOutputIntentProfile ()13 PDFXOutputConditionIdentifier ()13 PDFXOutputCondition ()13 PDFXRegistryName ()13 PDFXTrapped False1313 CreateJDFFile false13 Description ltlt13 ARA 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 BGR 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 CHS ltFEFF4f7f75288fd94e9b8bbe5b9a521b5efa7684002000410064006f006200650020005000440046002065876863900275284e8e9ad88d2891cf76845370524d53705237300260a853ef4ee54f7f75280020004100630072006f0062006100740020548c002000410064006f00620065002000520065006100640065007200200035002e003000204ee553ca66f49ad87248672c676562535f00521b5efa768400200050004400460020658768633002gt13 CHT ltFEFF4f7f752890194e9b8a2d7f6e5efa7acb7684002000410064006f006200650020005000440046002065874ef69069752865bc9ad854c18cea76845370524d5370523786557406300260a853ef4ee54f7f75280020004100630072006f0062006100740020548c002000410064006f00620065002000520065006100640065007200200035002e003000204ee553ca66f49ad87248672c4f86958b555f5df25efa7acb76840020005000440046002065874ef63002gt13 CZE 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 DAN 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 DEU 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 ESP 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 ETI 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 FRA 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 GRE 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 HEB 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 HRV (Za stvaranje Adobe PDF dokumenata najpogodnijih za visokokvalitetni ispis prije tiskanja koristite ove postavke Stvoreni PDF dokumenti mogu se otvoriti Acrobat i Adobe Reader 50 i kasnijim verzijama)13 HUN 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 ITA 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 JPN ltFEFF9ad854c18cea306a30d730ea30d730ec30b951fa529b7528002000410064006f0062006500200050004400460020658766f8306e4f5c6210306b4f7f75283057307e305930023053306e8a2d5b9a30674f5c62103055308c305f0020005000440046002030d530a130a430eb306f3001004100630072006f0062006100740020304a30883073002000410064006f00620065002000520065006100640065007200200035002e003000204ee5964d3067958b304f30533068304c3067304d307e305930023053306e8a2d5b9a306b306f30d530a930f330c8306e57cb30818fbc307f304c5fc59808306730593002gt13 KOR ltFEFFc7740020c124c815c7440020c0acc6a9d558c5ec0020ace0d488c9c80020c2dcd5d80020c778c1c4c5d00020ac00c7a50020c801d569d55c002000410064006f0062006500200050004400460020bb38c11cb97c0020c791c131d569b2c8b2e4002e0020c774b807ac8c0020c791c131b41c00200050004400460020bb38c11cb2940020004100630072006f0062006100740020bc0f002000410064006f00620065002000520065006100640065007200200035002e00300020c774c0c1c5d0c11c0020c5f40020c2180020c788c2b5b2c8b2e4002egt13 LTH 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 LVI 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 NLD (Gebruik deze instellingen om Adobe PDF-documenten te maken die zijn geoptimaliseerd voor prepress-afdrukken van hoge kwaliteit De gemaakte PDF-documenten kunnen worden geopend met Acrobat en Adobe Reader 50 en hoger)13 NOR 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 POL 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 PTB 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 RUM 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 RUS ltFEFF04180441043f043e043b044c04370443043904420435002004340430043d043d044b04350020043d0430044104420440043e0439043a043800200434043b044f00200441043e043704340430043d0438044f00200434043e043a0443043c0435043d0442043e0432002000410064006f006200650020005000440046002c0020043c0430043a04410438043c0430043b044c043d043e0020043f043e04340445043e0434044f04490438044500200434043b044f00200432044b0441043e043a043e043a0430044704350441044204320435043d043d043e0433043e00200434043e043f0435044704300442043d043e0433043e00200432044b0432043e04340430002e002000200421043e043704340430043d043d044b04350020005000440046002d0434043e043a0443043c0435043d0442044b0020043c043e0436043d043e0020043e0442043a0440044b043204300442044c002004410020043f043e043c043e0449044c044e0020004100630072006f00620061007400200438002000410064006f00620065002000520065006100640065007200200035002e00300020043800200431043e043b043504350020043f043e04370434043d043804450020043204350440044104380439002egt13 SKY 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 SLV 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 SUO 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 SVE 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 TUR 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 UKR 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 ENU (Use these settings to create Adobe PDF documents best suited for high-quality prepress printing Created PDF documents can be opened with Acrobat and Adobe Reader 50 and later)13 gtgt13 Namespace [13 (Adobe)13 (Common)13 (10)13 ]13 OtherNamespaces [13 ltlt13 AsReaderSpreads false13 CropImagesToFrames true13 ErrorControl WarnAndContinue13 FlattenerIgnoreSpreadOverrides false13 IncludeGuidesGrids false13 IncludeNonPrinting false13 IncludeSlug false13 Namespace [13 (Adobe)13 (InDesign)13 (40)13 ]13 OmitPlacedBitmaps false13 OmitPlacedEPS false13 OmitPlacedPDF false13 SimulateOverprint Legacy13 gtgt13 ltlt13 AddBleedMarks false13 AddColorBars false13 AddCropMarks false13 AddPageInfo false13 AddRegMarks false13 ConvertColors ConvertToCMYK13 DestinationProfileName ()13 DestinationProfileSelector DocumentCMYK13 Downsample16BitImages true13 FlattenerPreset ltlt13 PresetSelector MediumResolution13 gtgt13 FormElements false13 GenerateStructure false13 IncludeBookmarks false13 IncludeHyperlinks false13 IncludeInteractive false13 IncludeLayers false13 IncludeProfiles false13 MultimediaHandling UseObjectSettings13 Namespace [13 (Adobe)13 (CreativeSuite)13 (20)13 ]13 PDFXOutputIntentProfileSelector DocumentCMYK13 PreserveEditing true13 UntaggedCMYKHandling LeaveUntagged13 UntaggedRGBHandling UseDocumentProfile13 UseDocumentBleed false13 gtgt13 ]13gtgt setdistillerparams13ltlt13 HWResolution [2400 2400]13 PageSize [612000 792000]13gtgt setpagedevice13

Page 11: February 2014 MEE Questions and AnalysesPreface The Multistate Essay Examination (MEE) is developed by the National Conference of Bar Examiners (NCBE). This publication includes the

CRIMINAL LAW AND PROCEDURE QUESTION _____

A defendant was charged under state law with felony theft (Class D) and felony residential burglary (Class C) The indictment alleged that the defendant entered his neighborsrsquo home without their consent and stole a diamond ring worth at least $2500

Defense counsel filed a pretrial motion to dismiss the charges on the ground that prosecuting the defendant for both burglary and theft would constitute double jeopardy The trial court denied the motion and the defendant was prosecuted for both crimes The only evidence of the ringrsquos value offered at the defendantrsquos jury trial was the ownerrsquos testimony that she had purchased the ring two years earlier for $3000

At trial the judge issued the following jury instruction on the burglary charge prior to deliberations

If after consideration of all the evidence presented by the prosecution and defense you find beyond a reasonable doubt that the defendant entered the dwelling without the ownersrsquo consent you may presume that the defendant entered with the intent to commit a felony therein

The jury found the defendant guilty of both offenses

At the defendantrsquos sentencing hearing an expert witness called by the prosecutor testified that the diamond ring was worth between $7000 and $8000 Over defense objection the judge concluded by a preponderance of the evidence that the value of the stolen ring exceeded $5000 The judge sentenced the defendant to four yearsrsquo incarceration on the theft conviction On the burglary conviction the defendant received a consecutive sentence of seven yearsrsquo incarceration

In this state residential burglary is defined as ldquoentry into the dwelling of another without the consent of the lawful resident with the intent to commit a felony thereinrdquo Residential burglary is a Class C felony for which the minimum sentence is five years and the maximum sentence is ten years of incarceration

In this state theft is defined as ldquotaking and carrying away the property of another with the intent to permanently deprive the owner of possessionrdquo Theft is a Class D felony if the value of the item(s) taken is between $2500 and $10000 The sentence for a Class D felony theft is determined by the value of the items taken If the value is between $2500 and $5000 the maximum sentence is three yearsrsquo incarceration If the value of the items exceeds $5000 the maximum sentence is five yearsrsquo incarceration

This state affords a criminal defendant no greater rights than those mandated by the United States Constitution

1 Did the trial court err when it denied the defendantrsquos pretrial motion to dismiss on double jeopardy grounds Explain

2 Did the trial court err in its instruction to the jury on the burglary charge Explain

3 Did the trial court err when it sentenced the defendant to an additional year of incarceration on the theft conviction based on the expertrsquos testimony Explain

7

AGENCY AND PARTNERSHIP QUESTION _____

Five years ago Adam and Ben formed a general partnership Empire Partnership (Empire) to buy and sell antique automobiles at a showroom in State A Adam contributed $800000 to Empire and Ben contributed $200000 Their written partnership agreement allocated 80 of profits losses and control to Adam and 20 to Ben No filings of any type were made in connection with the formation of Empire

Three years ago a collector purchased one of Empirersquos antique cars for $3400000 The collector was willing to pay this price because of Benrsquos false representation (repeated in the sales contract) that a famous movie star had once owned the car Without the movie-star connection the car was worth only $100000 One month later when the collector discovered the truth he sued Adam Ben and Empire for $3300000 in damages The lawsuit is still pending

Two years ago Adam and Ben admitted a new partner Diane to Empire in return for her contribution of $250000 The three agreed to allocate profits losses and control 75 to Adam 10 to Ben and 15 to Diane Before joining the partnership Diane learned of the collectorrsquos claim and stated her concern to Adam and Ben that she might become liable if the claim were reduced to a judgment

Following Dianersquos admission to Empire the three partners sought to convert Empire into a limited liability partnership (LLP) Adamrsquos lawyer proposed to file with State A a ldquostatement of qualificationrdquo making an LLP election and declaring the name of the partnership to be ldquoEmpire LLPrdquo Benrsquos lawyer stated that this would not work and that a new LLP had to be formed with the assets of the old partnership transferred to the new one In the end the conversion was done the way Adamrsquos lawyer suggested with the approval of all three partners

One year ago a driver purchased a vintage car from Empire LLP based on the representation that the car was ldquofully roadworthy and capable of touring at 70 mph all dayrdquo The driver took the car on the highway at 50 mph whereupon the front suspension collapsed resulting in a crash in which the car was destroyed and the driver killed The driverrsquos estate sued Adam Ben Diane and Empire LLP for $10000000 The lawsuit is still pending

Although profitable Empire LLP does not have resources sufficient to pay the collectorrsquos claim or the claim of the driverrsquos estate

Assume that the Uniform Partnership Act (1997) applies

1 Before the filing of the statement of qualification (a) was Adam personally liable on the collectorrsquos claim Explain (b) was Diane personally liable on the collectorrsquos claim Explain

2 After the filing of the statement of qualification was Adam Ben or Diane personally liable as a partner on (a) the collectorrsquos claim or (b) the driverrsquos estatersquos claim Explain

8

February 2014 MEE

ANALYSES Constitutional Law

Trusts and Future Interests Secured Transactions

Federal Civil Procedure Criminal Law and Procedure

Agency and Partnership

CONSTITUTIONAL LAW ANALYSIS (Constitutional Law IVD)

ANALYSIS

Legal Problems

(1) Is the city ordinance requirement that businesses install floodlights a taking

(2) Is conditioning the approval of a building permit on the grant of an easement to install surveillance equipment a taking of property

DISCUSSION

Summary

The ordinance requiring businesses to install floodlights is not a per se taking under Loretto because it does not force a private landowner to allow a third party to enter and place a physical object on the land Here the city ordinance requires the businessmdashnot a third partymdashto install the floodlights

The ordinance is likely not a regulatory taking under the Penn Central balancing test While the ordinance will impose a cost on business owners that cost may be offset by the expected increase in business due to the ordinance and the ordinance does not appear to interfere with the ownerrsquos primary use of the property as a restaurant

The permit condition however is likely an uncompensated taking of property While the condition has an essential nexus with the cityrsquos legitimate interest in promoting public safety the city has not made an individualized determination that the easement condition is roughly proportional to the possibility of increased crime due to the restaurantrsquos proposed addition Thus the permit condition likely violates the Fifth Amendment as applied to the states through the Fourteenth Amendment

Point One (50) The ordinance requiring that businesses install floodlights is not a per se taking under Loretto It is not a regulatory taking under the Penn Central balancing test because the cost of compliance with the ordinance may be offset by an expected increase in business and compliance does not interfere with the businessrsquos primary use of its property as a restaurant

The city ordinance requiring a business to install floodlights does not effect a per se taking of the sort described in Loretto v Teleprompter Manhattan CATV Corp 458 US 419 (1982) because no property is physically taken by the government and the ordinance does not involve a physical invasion of private property by a third party

Even though the ordinance does not constitute an occupation of the property by either the government or a third party it is still subject to the three-factor balancing test under Penn Central Transportation Co v City of New York 438 US 104 (1978) to determine whether it is a ldquoregulatory takingrdquo Under Penn Central a court must balance (1) ldquo[t]he economic impact of the regulation on the claimantrdquo (2) ldquothe extent to which the regulation has interfered with distinct investment-backed expectationsrdquo and (3) ldquothe character of the governmental actionrdquo Id at 124 Here each factor weighs against finding that the ordinance is a taking

11

Constitutional Law Analysis

First the ordinance requirement likely has a minimal economic impact on the restaurant Compliance with the ordinance is estimated to cost $1000 and the city has found that businesses will likely recoup that cost in increased sales Also because the ordinance does not interfere with the operation of the restaurant the owner may still earn a reasonable return on its investment in the property

Second the ordinance does not interfere with the businessrsquos investment-backed expectations As in Penn Central the challenged law does not interfere with the ownerrsquos ldquoprimary expectationrdquo for use of the propertymdashin Penn Central as a railroad terminal and here as a restaurant Further the ordinance does not prevent the restaurant from expanding to meet the changing business environment

Third the character of the government action does not weigh in favor of a taking While Penn Central does say that a ldquophysical invasionrdquo is more likely to pose a taking Loretto suggests that the Courtrsquos main concern is with physical invasions by third parties Also like the landmark law challenged in Penn Central the ordinance here ldquoadjust[s] the benefits and burdens of economic life to promote the common goodrdquo Id In Penn Central the landmark law restricted development of the railroad terminal to promote the common interest in preserving historic landmarks Here the ordinance requires the businesses to install floodlights to promote the common interest in crime prevention and public safety

Because the ordinance is clearly a valid exercise of the police power it satisfies the takings clausersquos public-use requirement Kelo v City of New London 545 US 469 (2005)

In sum all three factors weigh against finding a taking under the Penn Central balancing test

Point Two (50) The permit condition may be unconstitutional as an uncompensated taking of property because the city has not made an individualized determination that the easement condition is roughly proportional to the impact of the restaurantrsquos proposed addition

In Dolan v City of Tigard 512 US 374 (1994) the Supreme Court set forth the test for determining whether an exaction imposed by a government in exchange for a discretionary benefit conferred by the government such as a condition on the approval of a building permit in this case constitutes an uncompensated taking under the Fifth Amendment The exaction is not a taking if (1) there is an ldquoessential nexusrdquo between the ldquopublic need or burdenrdquo to which the proposed development contributes and ldquothe permit condition exacted by the cityrdquo id at 386 and (2) the government makes ldquosome sort of individualized determination that the required dedication is [roughly proportional] both in nature and extent to the impact of the proposed developmentrdquo Id at 391 see also Nollan v California Coastal Commission 483 US 825 (1987)

Here the city likely can meet the nexus requirement In Dolan the landowner sought to double the size of its business which would have increased traffic on nearby roadways In exchange for approving the development the city sought an easement for a bike and pedestrian path The Court found the required nexus between the easement and the cityrsquos ldquoattempt to reduce traffic congestion by providing for alternative means of transportationrdquo 512 US at 387 Here a similar nexus likely exists between the requested easement and the cityrsquos interest in crime prevention and public safety Increased patronage and economic activity at the restaurant might attract additional crime to the area and the requested easement to install surveillance equipment would attempt to address that increased crime

12

Constitutional Law Analysis

The exaction here however may fail the second prong of the Dolan testmdashthat the exaction be roughly proportional to the anticipated impact of the requested development As noted the city in Dolan claimed that a bike and pedestrian path was needed to offset the increase in traffic due to the proposed doubling of the business The Court explained that the government must demonstrate that the additional traffic reasonably was related to the requested exaction and that the government must ldquomake some effort to quantify its findings in support of the dedication for the pedestrianbicycle pathway beyond the conclusory statement that it could offset some of the traffic demand generatedrdquo Id at 395 Here the city did not carry its burden The city simply speculates that increased patronage of the restaurant ldquomightrdquo increase crime and that the surveillance equipment ldquomightrdquo alleviate this increased crime Because the city has not made ldquosome effort to quantify its findingsrdquo in support of the easement it has not shown that the burden of the easement is roughly proportional to the benefits thought to flow from it

Thus the exaction appears to be an uncompensated taking of property in violation of the Fifth Amendment as applied to the states through the Fourteenth Amendment

13

TRUSTS AND FUTURE INTERESTS ANALYSIS ____ (Trusts and Future Interests IE3 I5 IIIA amp B)

ANALYSIS

Legal Problems

(1) How should rents dividends and sales proceeds received by the trustee prior to receipt of the sonrsquos letter have been allocated between trust income and principal

(2)(a) Did the remainder interest in the trust accelerate and become immediately payable to the daughterrsquos minor child upon the trusteersquos receipt of the sonrsquos letter and if not how should the trustee handle the distribution of the principal in the future

(2)(b) Following the trusteersquos receipt of the sonrsquos letter how should the trustee distribute future receipts of income prior to the distribution of the principal

DISCUSSION

Summary

Prior to the trusteersquos receipt of the sonrsquos letter cash dividends and rents should have been allocated to trust income and were distributable to the son the income beneficiary of the trust sales proceeds and stock dividends should have been allocated to principal

Because the sonrsquos letter to the trustee did not result in a valid disclaimer under state law (having been made more than nine months after the testatorrsquos death) the son is not deemed to have predeceased the testator Because the son is still living the class gift to the testatorrsquos grandchildren who survive the son has not closed and is not possessory it will not become possessory until the son dies The daughterrsquos minor child being the testatorrsquos only living grandchild is not currently entitled to a distribution of trust principal Trust principal will instead be distributable upon the sonrsquos death to the testatorrsquos then-living grandchildren or if there are none to the testatorrsquos then-living heirs

As for future income the trustee should either distribute the trust income to the son and the daughter as the testatorrsquos heirs accumulate the income for future distribution to those individuals ultimately entitled to the trust principal or distribute it to those presumptively entitled to the principal upon the sonrsquos death ie the daughterrsquos minor child

Point One (45) Cash dividends and rents are allocable to income sales proceeds and stock dividends are allocable to principal Items allocable to income for the period prior to the sonrsquos attempted disclaimer were distributable to the son

Receipts earned during the administration of a trust are allocable either to income or to principal Almost all states have adopted the most recent or an earlier version of the Uniform Principal and Income Act (the Act) which specifies how such receipts should be allocated

Under the Act rents (UNIF PRIN amp INC ACT (2000) sect 405 UNIF PRIN amp INC ACT (1962) sect 3(a)(1)) and cash dividends received from a corporation (UNIF PRIN amp INC ACT (2000) sect 401(b) UNIF PRIN amp INC ACT (1962) sect 6(d)) are allocable to income and are distributable to the income beneficiary of the trust

14

Trusts and Future Interests Analysis Sales proceeds (UNIF PRIN amp INC ACT (2000) sect 404(2) UNIF PRIN amp INC ACT (1962)

sect 3(b)(1)) and dividends paid in the stock of the distributing corporation (UNIF PRIN amp INC ACT (2000) sect 401(c)(1) UNIF PRIN amp INC ACT (1962) sect 3(b)(4)) are allocable to principal and added to the principal of the trust

Here the cash dividends and office building rents should have been allocated to income and until the trustee received the sonrsquos letter should have been distributed to him as the sole income beneficiary of the trust The stock dividend and proceeds from the sale of the office building should have been allocated to principal and held by the trustee for future distribution to the ultimate remaindermen of the trust

[NOTE The 2000 Uniform Principal and Income Act has been adopted in Alabama Arkansas Colorado Connecticut the District of Columbia Hawaii Idaho Iowa Kentucky Missouri Montana Nebraska New Mexico North Dakota Oregon South Dakota Utah and West Virginia]

Point Two(a) (45) Because the son did not disclaim within nine months of the testatorrsquos death there is no valid disclaimer under state law Therefore the son is not deemed to have predeceased the testator Furthermore because of the express survivorship contingency in the will the remainder in the trust does not accelerate and become distributable until the son in fact dies When the son dies the trust principal will be distributable to the testatorrsquos then-living grandchildren or if none then to the testatorrsquos then-living heirs

When a trust remainder is given to a class the class closes (ie no new persons can join the class) when there is no outstanding income interest and at least one member of the class is then entitled to demand possession of his or her share of the remainder This principle is called the rule of convenience See generally HERBERT HOVENKAMP amp SHELDON F KURTZ PRINCIPLES OF PROPERTY LAW 199ndash200 (6th ed 2005) A class member may demand possession of his or her share of the remainder upon termination of the income interest only when the class memberrsquos interest is not otherwise subject to a condition precedent See id

When a beneficiary timely disclaims an interest in a trust that beneficiary is treated as if he had predeceased the testator Here had the son disclaimed within nine months of the testatorrsquos death as required by the state statute he would have been deemed to have predeceased the testator This would have closed the class of remaindermen and the testatorrsquos then-living grandchildren (ie the daughterrsquos child) would have been entitled to the trust principal However under the state statute the sonrsquos disclaimer was not timely because he did not disclaim within nine months of the testatorrsquos death Thus because the statute is inapplicable and the son is still alive the class of grandchildren entitled to share in trust principal did not close

Because here the statute is inapplicable due to the sonrsquos failure to comply with the statutory time requirements then presumably the common-law rule allowing disclaimers (aka renunciations) at any time should apply Under the common law if a life estate is renounced the remainder interest accelerates and becomes immediately distributable to the remaindermen of the trust if the remainder is vested but not if the remainder is contingent JESSE DUKEMINIER amp ROBERT H SITKOFF WILLS TRUSTS AND ESTATES 844ndash845 (9th ed 2013) Here because the remainder is contingent upon there being grandchildren who survive the son the remainder will not accelerate It will remain open until the son dies leaving open the possibility that additional grandchildren will be included in the class or the daughterrsquos child could fall out of the class because that child fails to survive the son

And if none of the testatorrsquos grandchildren survive the son the trust principal will be distributed to the testatorrsquos heirs living at the sonrsquos death

15

Trusts and Future Interests Analysis

Point Two(b) (10) Until the trust terminates the trustee must continue to hold the trust assets The distribution of income in the meantime is unclear There are at least three possibilities Income earned on the undistributed assets could be distributed to the son and daughter as the testatorrsquos heirs accumulated and added to principal for distribution to the ultimate remaindermen or distributed from time to time to those persons who are presumptively remaindermen

When trust principal is not immediately distributable the trustee must continue to hold trust assets until the ultimate remaindermen are ascertained During this period trust income will be distributed or retained according to any instructions contained in the trust instrument See WILLIAM M MCGOVERN JR SHELDON F KURTZ amp DAVID M ENGLISH WILLS TRUSTS amp ESTATES sect 102 (4th ed 2010)

Here the testator did not specify what the trustee should do with trust income in the event the sonrsquos disclaimer did not comply with the state statute There are at least three approaches One approach would have the trustee distribute the trust income to the testatorrsquos heirs on the theory that the income represents property that was not disposed of by the testatorrsquos will and which thus passes by partial intestacy to the testatorrsquos heirs A second approach would have the trustee accumulate trust income for distribution to the ultimate remaindermen Under this approach only those individuals ultimately entitled to the principal would share in the income A third approach would have the trustee distribute trust income to those individuals who would be the remaindermen if the trust were to terminate when the income is received by the trustee under this approach trust income would be distributed to the daughterrsquos minor child until another presumptive remainderman is born This approach could result in individuals not ultimately entitled to principal say because they do not survive the son receiving income It could also result in a disproportionate distribution of income among the individuals ultimately entitled to income

[NOTE Examinees should demonstrate a recognition and understanding of the income-allocation problem and the alternatives available to address that issue There is no widely accepted solution to the problem Examinees who cite any of these possible problem-solving approaches may receive credit]

16

SECURED TRANSACTIONS ANALYSIS (Secured Transactions IB IID E amp F IIIB IVA B amp F)

ANALYSIS

Legal Problems

(1)(a) What is the nature of the bankrsquos claim to the businessrsquos equipment

(1)(b) What is the nature of the finance companyrsquos claim to the businessrsquos equipment

(1)(c) As between the bank and the finance company whose claim to the businessrsquos equipment has priority

(2) Do the claims of the bank and the finance company continue in the item of equipment sold by the business to the competitor

DISCUSSION

Summary

The bank and the finance company both have perfected security interests in the businessrsquos equipment Even though the finance companyrsquos perfected security interest was created first the bankrsquos perfected security interest has priority because the bankrsquos financing statement was filed before the finance companyrsquos financing statement The security interests of the bank and the finance company continue in the item of equipment sold by the business to the competitor because their security interests were perfected and the competitor was not a buyer in ordinary course of business

Point One(a) (25) The bank has a perfected security interest in the businessrsquos equipment

The bank has met all criteria necessary for it to have an attached and enforceable security interest in the businessrsquos equipment First value must be given UCC sect 9-203(b)(1) This criterion is fulfilled by the loan made by the bank to the business Second the debtor must have rights in the collateral UCC sect 9-203(b)(2) Clearly the business has rights in its equipment Third either the secured party must take possession of the collateral or the debtor must authenticate a security agreement containing a description of the collateral UCC sect 9-203(b)(3) The agreement that the business owner signed is a ldquosecurity agreementrdquo because it is an agreement that creates or provides for a security interest UCC sect 9-102(a)(74) By signing the security agreement the business owner authenticated it UCC sect 9-102(a)(7) Therefore all three criteria are fulfilled and the bank has an enforceable and attached security interest

A security interest is perfected when it has attached and when any additional steps required for perfection have occurred UCC sect 9-308(a) Generally speaking the additional steps will either be possession of the collateral by the secured party or the filing of a financing statement with respect to the collateral See UCC sectsect 9-310 9-313 In this case the bank filed a financing statement naming the debtor and sufficiently indicating the collateral The collateral indication is sufficient because it identifies the collateral by type of property See UCC sectsect 9-504 9-108 The fact that the financing statement was filed before the security interest was created is

17

Secured Transactions Analysis

not a problem Even though the security agreement had not yet been signed the business had authorized the filing of the financing statement in an authenticated record UCC sect 9-509(a)(1) Moreover the financing statement may be filed before the security agreement is created UCC sect 9-502(d)

Point One(b) (10) The finance company also has a perfected security interest in the businessrsquos equipment

The finance companyrsquos security interest is enforceable and attached for the same reasons as the bankrsquos security interest The loan from the finance company to the business constitutes value the business has rights in the collateral and the business owner has authenticated a security agreement containing a description of the collateral The finance companyrsquos security interest is perfected because the finance company filed a financing statement with respect to it that provides that the business is the debtor and indicates that the collateral is equipment

Point One(c) (30) The bankrsquos security interest has priority over the finance companyrsquos security interest because the bankrsquos financing statement was filed first

As between two perfected security interests the general rule is that the security interest that was the earlier to be either perfected or the subject of a filed financing statement has priority UCC sect 9-322(a)(1) While the finance companyrsquos security interest was perfected before the bankrsquos (March 15 vs March 22) the bankrsquos financing statement was filed even earlier on March 2 Thus under the first-to-file-or-perfect rule of UCC sect 9-322(a)(1) the bankrsquos security interest has priority No exceptions to the general rule apply here

Point Two (35) A security interest in collateral continues notwithstanding its sale unless an exception applies Because the security interests of the bank and the finance company were perfected and the competitor was not a buyer in ordinary course of business no exception applies and the security interests of both creditors continue in the equipment sold to the competitor

As a general rule a security interest in collateral continues notwithstanding the fact that the debtor has sold the collateral to another person UCC sect 9-315(a)(1) Thus unless an exception applies the security interests of the bank and the finance company will continue in the item of equipment sold to the competitor

A buyer of goods will take free of an unperfected security interest in those goods See UCC sect 9-317(a)(2) However when the competitor bought the businessrsquos equipment both the bank and the finance company had perfected security interests in the equipment

A buyer can take free even of a perfected security interest in goods if the buyer is a ldquobuyer in ordinary course of businessrdquo See UCC sect 9-320(a) However the competitor was not a buyer in ordinary course of business To be a ldquobuyer in ordinary course of businessrdquo a buyer must buy goods from a seller that is in the business of selling goods of that kind See UCC sect 1-201(b)(9) The competitor bought this equipment from a seller that is not in the business of selling goods of this kind so the competitor was not a buyer in ordinary course of business with respect to these goods

Because no exception applies the security interests of the bank and the finance company continue even after the item of equipment was sold to the competitor

18

FEDERAL CIVIL PROCEDURE ANALYSIS (Federal Civil Procedure IVD)

ANALYSIS

Legal Problems

(1) Is a document prepared in the course of a contract dispute protected from discovery as ldquowork productrdquo when there is no evidence that the document was prepared in anticipation of litigation

(2)(a) Is a partyrsquos failure to provide relevant electronically stored information excused when the information was destroyed pursuant to a routine document retention scheme at a time when litigation was contemplated by the destroying party

(2)(b) What sanctions should be imposed on a party for allowing the destruction of evidence that is relevant to potential future litigation

DISCUSSION

Summary

The report prepared by the structural engineer is probably not work product and is thus discoverable The engineer examined the foundation of the house at the customerrsquos request and the engineerrsquos findings are potentially relevant to the customerrsquos claim that the foundation is defective The report was not prepared in anticipation of litigation The customer appears to have sought the engineerrsquos opinion in response to the builderrsquos offer to fix any problems with the foundation that an engineer might identify Because the report was not prepared in anticipation of litigation it is not protected by the work-product doctrine

The builder should have taken appropriate steps to preserve evidence including suspending its document retention program as soon as it began planning for litigationmdashie on July 10 Its destruction of potentially relevant material after that date was wrongful However a court is unlikely to impose severe sanctions on the builder because there are no facts indicating that the builder acted in bad faith and the customer can prove that the foundation is defective without the destroyed emails

Point One (40) The customer must turn over the engineerrsquos report because it was not prepared in anticipation of litigation

In general a party to a lawsuit in federal court ldquomay obtain discovery regarding any nonprivileged matter that is relevant to any partyrsquos claim or defenserdquo FED R CIV P 26(b)(1) (2009) This includes the right to inspect and copy documents in the other partyrsquos possession FED R CIV P 34(a)(1) Here the customer hired a structural engineer to examine the foundation of the house The engineerrsquos report on the foundation is likely to include information that would be relevant to the customerrsquos claim that the foundation was defectively constructed

The so-called ldquowork productrdquo rule allows a party to refuse to turn over ldquodocuments that are prepared in anticipation of litigation or for trialrdquo by that partyrsquos representative including

19

Federal Civil Procedure Analysis

a consultant Thus if the customer had hired the structural engineer to prepare a report ldquoin anticipation of litigationrdquo that report might not be discoverable See FED R CIV P 26(b)(3)

In this case however the customer hired the engineer to evaluate the foundation of the house as part of the customerrsquos negotiation with the builder concerning the housersquos flooding problem The builder told the customer that the housersquos landscaping was the reason for the flooding and the builder told the customer ldquoHave an engineer look at the foundation If therersquos a problem wersquoll fix itrdquo The customer appears to have acted in response to that statement There is no indication that the customer anticipated any kind of legal action at the time that the structural engineer was hired Accordingly the structural engineerrsquos report is discoverable and the court should order the customer to turn it over

[NOTE If an examinee concludes that the structural engineerrsquos report was prepared in anticipation of litigation then the examinee should also conclude that the report is not discoverable Documents prepared in anticipation of litigation do not need to be disclosed to an adverse party unless that party can demonstrate a ldquosubstantial needrdquo for the documents and an inability to obtain substantially equivalent information without ldquoundue hardshiprdquo FED R CIV P 26(b)(3)(A)(ii) Furthermore a report prepared by an expert who is not expected to testify is not discoverable in the absence of ldquoexceptional circumstancesrdquo making it ldquoimpracticablerdquo to obtain the information in another way FED R CIV P 26(b)(4)(D)(ii) The builder probably cannot make these showings here unless the engineerrsquos report deals with circumstances that have since changed There is no evidence that the structural engineer would have had access to any information or facts that the builder would not already know as a result of its construction and subsequent inspection of the house In addition if necessary the builder could ask the court for permission to arrange for a further inspection of the house by a structural engineer hired by the builder See FED R CIV P 34(a)(2) Accordingly if an examinee concludes that the report was prepared in anticipation of litigation the examinee should also conclude that the builder is not entitled to see the report]

Point Two(a) (30) Because the builder anticipated that it might be involved in litigation concerning its contract with the customer the builder acted wrongfully in destroying emails that were relevant to the housersquos construction even though the emails were destroyed pursuant to a routine document retention plan

As noted above a party to a lawsuit in federal court ldquomay obtain discovery regarding any nonprivileged matter that is relevant to any partyrsquos claim or defenserdquo FED R CIV P 26(b)(1) This includes emails and other electronically stored information FED R CIV P 34(a)(1)(A) Here the customer has requested all the builderrsquos emails pertaining to work done on the foundation of the house Ordinarily the builder would be obliged to turn over this information which is relevant to the customerrsquos defense that the housersquos foundation was poorly constructed

Unfortunately the emails in question no longer exist because the builder destroyed them on August 2

In general spoliation of evidence (destruction or alteration of evidence) is improper if the party who destroyed or altered the evidence ldquohas notice that the evidence is relevant to litigation or should have known that the evidence may be relevant to future litigationrdquo Fujitsu Ltd v Federal Express Corp 247 F3d 423 436 (2d Cir 2001) It is improper for a party to destroy electronic information relevant to pending litigation even if the destruction occurs before there is any request or order seeking the information See eg Leon v IDX Sys Corp 464 F3d 951 (9th Cir 2006) (plaintiffrsquos intentional destruction of computer files warranted dismissal even

20

In this case the builderrsquos destruction of the emails was pursuant to a routine document retention plan The Federal Rules provide expressly that in the absence of ldquoexceptional circumstancesrdquo parties should not be sanctioned for the loss of electronically stored information when the loss occurs pursuant to ldquoroutine good-faith operation of an electronic information systemrdquo FED R CIV P 37(e) However when a party anticipates litigation ldquoit must suspend its routine document retentiondestruction policy and put in place a lsquolitigation holdrsquo to ensure the preservation of relevant documentsrdquo Zubulake v UBS Warburg LLC 220 FRD 212 218 (SDNY 2003)

Federal Civil Procedure Analysis

though spoliation occurred before order compelling discovery) Similarly the duty to preserve evidence applies to a party who anticipates litigation even if litigation has not yet been commenced See THE SEDONA PRINCIPLES BEST PRACTICES RECOMMENDATIONS amp PRINCIPLES FOR ADDRESSING ELECTRONIC DOCUMENT PRODUCTION 70 cmt 14a (2d ed 2007)

The builder destroyed the emails on August 2 At that time the builder knew that litigation was a possibility because the builder had already directed its attorney to prepare a draft complaint for possible filing Knowing that litigation was a possibility the builder had a duty to take steps to preserve evidence including the emails in question See generally Fujitsu Ltd

Thus the builderrsquos destruction of potentially relevant emails at a time when it knew that litigation was a possibility was improper It had a duty to preserve evidence and it breached that duty

[NOTE Because courts have used different words to describe the test for when evidence must be preserved an examineersquos precise formulation of the test is not critical]

Point Two(b) (30) In determining appropriate sanctions for spoliation courts consider both the level of culpability of the spoliating party and the degree of prejudice the loss of evidence has caused the other party Here the builderrsquos destruction of evidence does not appear to have been willful nor is it likely to pose a significant obstacle to the customerrsquos defense Any sanctions imposed by the court should be modest

Federal courts have inherent power to control the litigation process and can sanction misbehavior including spoliation even when there has been no specific violation of the Federal Rules of Civil Procedure See generally Chambers v NASCO Inc 501 US 32 (1991) (discussing courtrsquos inherent power to control the litigation process) The range of available sanctions is broad It can include such sanctions as the payment of expenses incurred by the other party as a result of the destruction of the evidence an instruction to the jury authorizing it to draw an adverse inference from the destruction of the evidence a shifting of the burden of proof on the relevant issue or even judgment against the responsible party See eg Residential Funding Corp v DeGeorge Financial Corp 306 F3d 99 108 (2d Cir 2002) (adverse inference) Silvestri v General Motors Corp 271 F3d 583 593 (4th Cir 2001) (possibility of dismissal) Cf FED R CIV P 37(b)(2)(A) (listing remedies for failure to comply with discovery obligations)

In determining appropriate sanctions for spoliation courts consider both the level of culpability of the spoliating party and the degree of prejudice the loss of evidence has caused the other party Many courts impose severe sanctions (such as an adverse-inference instruction or the entry of judgment against the spoliating party) only when there is evidence of bad faith in the form of an intentional effort to hide information Eg Greyhound Lines Inc v Wade 485 F3d 1032 1035 (8th Cir 2007) (spoliation sanction requires intentional destruction out of desire ldquoto suppress the truthrdquo) However other courts have said that negligence in preserving evidence can

21

Federal Civil Procedure Analysis

support an adverse-inference instruction See Residential Funding 306 F3d at 108 (negligence enough under some circumstances)

Although a court might well order an evidentiary hearing on the issue of sanctions the facts presented do not seem appropriate for severe sanctions First the evidence was destroyed pursuant to the builderrsquos standard document retention plan and there is no evidence that the builder deliberately failed to suspend its usual procedures with the purpose of allowing the destruction of evidence Second the loss of this evidence will not severely hinder the customerrsquos presentation of his case The central issue is whether the foundation of the house was properly constructed If the construction job was poorly done the customer can present evidence derived from inspection of the premises to prove that point The customer can also depose witnesses about any issues that arose during construction

Under the circumstances a court is not likely to impose particularly severe sanctions although it might shift the burden to the builder to show that the foundation was properly constructed or it might require the builder to reimburse any expenses the customer incurs to discover and prove the facts about issues or disputes that arose during construction of the foundation

[NOTE The result reached by the examinee is less important than the examineersquos recognition that (a) a range of sanctions is available to the court and (b) the appropriate sanction depends both on the culpability of the builder and the prejudice suffered by the customer]

22

CRIMINAL LAW AND PROCEDURE ANALYSIS (Criminal Law and Procedure IIA amp D VE amp F)

ANALYSIS

Legal Problems

(1) Did charging the defendant with both theft and burglary constitute double jeopardy

(2) Did the jury instruction violate the due process clause either by relieving the prosecution of the burden of proving the element of intent or by shifting the burden to the defendant to disprove that element

(3) Did the sentence imposed in this case for the theft conviction unconstitutionally deprive the defendant of his right to a jury trial on the issue of the value of the stolen item

DISCUSSION

Summary

The trial court properly denied the defendantrsquos pretrial motion to dismiss the charges on double jeopardy grounds The defendant may be charged with and convicted of both theft and burglary Each of the charges has an element that the other does not Neither charge is a lesser-included offense nor are they multiplicitous Thus charging both theft and burglary does not violate double jeopardy

The jury instruction on the burglary charge was constitutionally flawed It could have been reasonably understood by the jury as either (1) an irrebuttable conclusive presumption (which relieved the prosecution of proving the element of intent and removed the issue from the jury) or (2) a rebuttable mandatory presumption (which unconstitutionally shifted the burden of proof on an element of a charged offense from the prosecution to the defendant)

Because the four-year sentence imposed by the judge was based on the judgersquos finding by a preponderance of the evidence that the value of the stolen ring exceeded $5000 the sentence violates the defendantrsquos right to a jury determination beyond a reasonable doubt of the value of the ring

Point One (30) Charging the defendant with theft and burglary did not constitute double jeopardy

The Double Jeopardy Clause of the Fifth Amendment provides that a person shall not be twice put in jeopardy for the ldquosame offenserdquo Thus the question is whether the elements of the theft charge are wholly contained in the burglary charge or vice versa If the elements of the lesser charge (theft) are not wholly contained in the greater charge (burglary)mdashie if each charge requires proof of a fact that the other does notmdashthen convicting the defendant of both crimes would not violate double jeopardy even when the two offenses occurred at the same time and are thus arguably part of the ldquosame transactionrdquo Blockburger v United States 284 US 299 304 (1932) See also Albernaz v United States 450 US 333 344 n3 (1981) United States v Dixon 509 US 688 704 (1993)

23

Criminal Law and Procedure Analysis

Here theft and burglary each require proof of an element not required for the other crime Burglary may be defined differently in different jurisdictions However it almost invariably requires entry into a building or dwelling of another with the specific intent to commit a felony therein and the crime of burglary is complete upon the entry into the building or dwelling with such intent See eg Cannon v Oklahoma 827 P2d 1339 1342 (Okla Crim App 1992) In contrast theft which also may be defined differently in different states almost invariably requires the taking and carrying away of an item of personal property belonging to another with the intent to steal or permanently deprive the owner of possession

Here the ldquotakingrdquo or ldquostealingrdquo element is not contained in the definition of burglary and the ldquoentryrdquo element of burglary is not contained in the definition of theft Because theft is not a lesser-included offense of burglary and burglary is not a lesser-included offense of theft charging the defendant for both burglary and theft did not violate double jeopardy and the court properly denied the defense motion on those grounds Yparrea v Dorsey 64 F3d 577 579ndash80 (10th Cir 1995) citing Blockburger 284 US at 304

Finally the defendantrsquos motion to dismiss all the charges on double jeopardy grounds was improper because if both charges were for the same offense the motion should have requested dismissal of one charge not both

Point Two (35) The jury instruction on the burglary charge violated the Due Process Clause because it created either (1) an irrebuttable conclusive presumption (which relieved the prosecution of proving the element of intent and removed that issue from the jury) or (2) a rebuttable mandatory presumption (which unconstitutionally shifted the burden of proof on an element of a charged offense to the defendant)

The Supreme Court has interpreted the Due Process Clause of the US Constitution to require that the prosecution prove all elements of an offense beyond a reasonable doubt See In re Winship 397 US 358 364 (1970) The burden of proof cannot be shifted to the defendant by presuming an essential element upon proof of other elements of the offense because shifting the burden of persuasion with respect to any element of a criminal offense is contrary to the Due Process Clause See Mullaney v Wilbur 421 US 684 (1975)

The crime of burglary includes entry into a building or dwelling with the specific intent to commit a felony therein The requirement that the prosecutor prove beyond a reasonable doubt that the defendant had this specific intent distinguishes burglary from general-intent crimes like trespass See Sandstrom v Montana 442 US 510 523 (1979)

Here the jury was instructed that if ldquoafter consideration of all the evidence presented by the prosecution and defense you find beyond a reasonable doubt that the defendant entered the dwelling without the ownersrsquo consent you may presume that the defendant entered with the intent to commit a felony thereinrdquo This instruction was unconstitutional because it created either an irrebuttable conclusive presumption or a rebuttable mandatory presumption

A conclusive presumption is ldquoan irrebuttable direction by the court to find intent once convinced of the facts triggering the presumptionrdquo Id at 517 Here the jurors were instructed that once the prosecutor established that the defendant entered the neighborsrsquo house without consent they ldquomay presumerdquo that he intended to commit a felony therein The jurors may have reasonably concluded from this instruction that if they found that the defendant intended to enter his neighborsrsquo home without permission they must further find that he entered with the specific intent to commit a felony therein Because this instruction could operate as a conclusive

24

Criminal Law and Procedure Analysis

irrebuttable presumption by eliminating intent ldquoas an ingredient of the offenserdquo it violated due process by relieving the prosecution of the burden of proof for this element Id at 522

In the alternative the jury instruction could have been reasonably understood to create a rebuttable mandatory presumption which ldquotells [the jury] they must find the elemental fact upon proof of the basic fact at least unless the defendant has come forward with some evidence to rebut the presumed connection between the two factsrdquo County Court of Ulster County New York v Allen 442 US 140 157 (1979) The due process problem created by rebuttable mandatory presumptions is that ldquo[t]o the extent that the trier of fact is forced to abide by the presumption and may not reject it based on an independent evaluation of the particular facts presented by the State the analysis of the presumptionrsquos constitutional validity is logically divorced from those facts and based on the presumptionrsquos accuracy in the run of casesrdquo Id at 159

Unlike irrebuttable conclusive presumptions rebuttable mandatory presumptions are not always per se violations of the Due Process Clause However the Supreme Court of the United States has held that jury instructions that could reasonably be understood as shifting the burden of proof to the defendant on an element of the offense are unconstitutional Francis v Franklin 471 US 307 (1985) Here the argument that the jury instruction operated as a rebuttable mandatory presumption is supported by the fact that the judge also instructed the jury to ldquoconsider[ ] all the evidence presented by the prosecution and defenserdquo However even if the instruction created a rebuttable mandatory presumption it would be unconstitutional because it shifted the burden to the defense on an element of the offense Sandstrom 442 US at 524 Mullaney 421 US at 686

[NOTE Whether an examinee identifies the jury instruction as containing a ldquoconclusiverdquo or ldquomandatoryrdquo presumption is less important than the examineersquos analysis of the constitutional infirmities]

Point Three (35) The trial court violated the defendantrsquos Sixth Amendment right to a jury trial on an essential element of the offense when it found by a preponderance of the evidence that the ring was worth over $5000 and increased the defendantrsquos sentence based on this finding

In the statutory scheme under which the defendant was tried and convicted a Class D felony theft is defined as theft of item(s) with a value between $2500 and $10000 The jury found that the value of the diamond ring was at least $2500 and convicted the defendant of felony theft However at sentencing the trial court made a separate finding by a preponderance of the evidence that the value of the ring was greater than $5000 Following the statutersquos two-tiered sentencing scheme the judge then imposed on the defendant a sentence that was one year longer than the maximum that would otherwise have been allowed

The judgersquos sentence was unconstitutional because it violated the defendantrsquos Sixth Amendment right to a jury trial on this question The Supreme Court held in Apprendi v New Jersey 530 US 466 (2000) that ldquo[o]ther than the fact of a prior conviction any fact that increases the penalty for a crime beyond the prescribed statutory maximum must be submitted to a jury and proved beyond a reasonable doubtrdquo because ldquo[i]t is unconstitutional for a legislature to remove from the jury the assessment of facts that increase the prescribed range of penalties to which a criminal defendant is exposed [because] such facts must be established by proof beyond a reasonable doubtrdquo Id The Court reaffirmed Apprendi in Blakely v Washington 542 US 296 (2004) holding that the ldquolsquostatutory maximumrsquo for Apprendi purposes is the maximum sentence a judge may impose solely on the basis of the facts reflected in the jury verdict or admitted by the defendantrdquo Id at 303 (emphasis in original) In United States v Booker 543 US 220 (2005)

25

Criminal Law and Procedure Analysis

the Court relied on Blakely and Apprendi to conclude that protecting a defendantrsquos Sixth Amendment right to a jury trial required that ldquo[a]ny fact which is necessary to support a sentence exceeding the maximum authorized by the facts established by a plea of guilty or a jury verdict must be admitted by the defendant or proved to a jury beyond a reasonable doubtrdquo Id at 244

Thus in order to constitutionally increase a sentence above the statutory maximum of three years the jury must have found beyond a reasonable doubt that the value of the ring exceeded $5000 Here the court made the finding based on an appraisal proffered by the prosecutor only at sentencing and the judgersquos finding was by a preponderance of the evidence rather than beyond a reasonable doubt

26

AGENCY AND PARTNERSHIP ANALYSIS __________ (Agency and Partnership VA amp C VI)

ANALYSIS

Legal Problems

(1) Is a partner in a general partnership personally liable on a claim arising from misrepresentations by another partner made in the course of the partnership business

(2) Does a newly admitted partner in a general partnership become personally liable on existing claims against the partnership

(3) After the filing by a general partnership of a statement of qualification as a limited liability partnership are the partners personally liable as partners on (a) an existing claim against the general partnership and (b) a claim against the partnership that arose after the filing

DISCUSSION

Summary

Adam and Ben formed a general partnership under which they were jointly and severally liable for obligations of the partnership Thus Adam was personally liable for misrepresentations by Ben made in the ordinary course of the partnership business

Upon joining the general partnership Diane became personally liable for the obligations of the partnership arising after her admission but not for obligations pre-existing her admission such as the collectorrsquos claim

By filing a statement of qualification the three partners properly elected limited liability partnership status As partners in an LLP none of the three partners is personally liable as a partner for partnership obligations arising after the election such as the claim by the driverrsquos estate The election however does not change their personal liability on pre-existing claims that arose before the election such as the collectorrsquos claim

Point One (30) As a general partner of Empire a general partnership Adam became personally liable on the collectorrsquos claim a valid claim against the partnership that arose because of Benrsquos wrongful act in the ordinary course of the partnership business

When the collectorrsquos claim arose Empire was a general partnership composed of Adam and Ben Under UPA (1997) sect 306(a) partners of a general partnership are liable jointly and severally for all obligations of the partnership Under UPA (1997) sect 305(a) the partnership could become obligated for the loss caused to the collector as a result of the misrepresentation by Ben provided he was acting in the ordinary course of the partnership business Because there was no statement that limited his partnership authority Ben as partner was ldquoan agent of the partnership for the purpose of its businessrdquo See UPA (1997) sect 301(1) Benrsquos misrepresentation to the collector even if intentional appears to be in the ordinary course of the partnershiprsquos business of dealing

27

Agency and Partnership Analysis

in antique cars Thus Benrsquos wrongful act created a partnership obligation for which Adam was jointly and severally liable

[NOTE Generally a partnership creditor must ldquoexhaust the partnershiprsquos assets before levying on a judgment debtor partnerrsquos individual property where the partner is personally liable for the partnership obligationrdquo as a result of his status as a partner UPA (1997) sect 307 cmt 4 As the UPA comments explain this places Adam more in the position of guarantor than principal debtor on the partnership obligation Id cmt 4 Although an examinee might discuss this point the call focuses on whether Adam is personally liable not how the liability might be enforced]

Point Two (30) Because the collectorrsquos claim arose before Diane joined Empire Diane did not become personally liable on the claim

Diane was admitted to Empire when it was a general partnership and after the collectorrsquos claim arose While the general rule under UPA (1997) sect 306(a) is that the partners of a general partnership are liable jointly and severally for all obligations of the partnership there is a special rule for partners who are admitted during the duration of the partnership Under UPA (1997) sect 306(b) a person admitted to an existing partnership is not personally liable for any partnership obligations incurred before the personrsquos admission Because Diane was admitted to Empire after the collectorrsquos claim arose Diane is not personally liable on the claim

Dianersquos knowledge of the pre-existing claim and her stated concern about becoming liable on the collectorrsquos claim do not change her personal nonliability to the collector Although partners who have a liability shield can assume liability to third parties through private contractual guarantees or modifications to the partnership agreement Dianersquos stated concern constituted neither a guaranty to the collector nor ldquoan intentional waiver of liability protectionsrdquo See UPA (1997) sect 306 cmt 3 (describing methods for waiver of liability protections under sect 306(c) applicable in limited liability partnerships)

At most Diane will lose her investment in the partnership as a result of the collectorrsquos claim Although Diane did not become personally liable on the collectorrsquos claim when she joined the partnership the $250000 she contributed to the partnership is ldquoat risk for the satisfaction of existing partnership debtsrdquo UPA (1997) sect 306 cmt 2

Point Three (40) Filing the statement of qualification was effective to elect limited liability partnership status Despite this new status Adam and Ben remain personally liable on the collectorrsquos claim which arose before the election But as partners in an LLP neither Adam Ben nor Diane is personally liable as a partner on the driverrsquos estatersquos claim which arose after the election

Under UPA (1997) sect 1001 a general partnership can make an election and become a limited liability partnershipmdashif the partners approve the conversion by a vote equivalent to that necessary to amend the partnership agreement and the partnership then files a statement of qualification that specifies the name of the partnership its principal office and its election to be an LLP Here the partners agreed unanimouslymdashsufficient to amend their agreement under UPA (1997) sect 401(j)mdashand the statement of qualification was filed In addition the name of Empire LLP properly included an appropriate ending ldquoLLPrdquo See UPA (1997) sect 1002

Although another way to effectuate a ldquoconversionrdquo (as suggested by Benrsquos lawyer) is to form a new LLP and transfer the assets of the old general partnership to the new LLP the

28

Agency and Partnership Analysis

method used here (approval by the partners and the filing of a statement of qualification) is also sufficient to create LLP status

Thus Empire became Empire LLP as of the date of filing of the statement of qualification See UPA (1997) sect 1001 What effect did this have on the collectorrsquos claim which predated the filing According to UPA (1997) sect 306(c) an obligation incurred while a partnership is an LLP is solely a partnership obligation As the collectorrsquos claim predated the LLP Adam and Ben remain personally liable on the collectorrsquos claim Diane on the other hand was not personally liable on the collectorrsquos claim either before or after the filing of the statement of qualification See Point Two above

The driverrsquos estatersquos claim arose after Empire became Empire LLP Under UPA (1997) sect 306(c) an obligation incurred while a partnership is an LLP is solely a partnership obligationThus Adam Ben and Diane as partners are all protected from personal liability on the driverrsquos estatersquos claim But there may be personal liability if any of them was negligent or otherwise acted wrongfully by not informing the buyer of the bad suspension that caused the accident

29

National Conference of Bar Examiners 302 South Bedford Street | Madison WI 53703-3622 Phone 608-280-8550 | Fax 608-280-8552 | TDD 608-661-1275

wwwncbexorg e-mail contactncbexorg

  • Preface
  • Description of the MEE
  • Instructions
  • February 2014 Questions
    • Constitutinal Law Question
    • Trusts and Future Interests Question
    • Secured Transactions Question
    • Federal Civil Procedure Question
    • Criminal Law and Procedure Question
    • Agency and Partnership Question
      • February 2014 Analyses
        • Constitutional Law Analysis
        • Trust and Future Interests Analysis
        • Secured Transactions Analysis
        • Federal Civil Procedure Analysis
        • Criminal Law and Procedure Analysis
        • Agency and Partnership Analysis
            • ltlt13 ASCII85EncodePages false13 AllowTransparency false13 AutoPositionEPSFiles true13 AutoRotatePages None13 Binding Left13 CalGrayProfile (Dot Gain 20)13 CalRGBProfile (sRGB IEC61966-21)13 CalCMYKProfile (US Web Coated 050SWOP051 v2)13 sRGBProfile (sRGB IEC61966-21)13 CannotEmbedFontPolicy Error13 CompatibilityLevel 1413 CompressObjects Tags13 CompressPages true13 ConvertImagesToIndexed true13 PassThroughJPEGImages true13 CreateJobTicket false13 DefaultRenderingIntent Default13 DetectBlends true13 DetectCurves 0000013 ColorConversionStrategy CMYK13 DoThumbnails false13 EmbedAllFonts true13 EmbedOpenType false13 ParseICCProfilesInComments true13 EmbedJobOptions true13 DSCReportingLevel 013 EmitDSCWarnings false13 EndPage -113 ImageMemory 104857613 LockDistillerParams false13 MaxSubsetPct 10013 Optimize true13 OPM 113 ParseDSCComments true13 ParseDSCCommentsForDocInfo true13 PreserveCopyPage true13 PreserveDICMYKValues true13 PreserveEPSInfo true13 PreserveFlatness true13 PreserveHalftoneInfo false13 PreserveOPIComments true13 PreserveOverprintSettings true13 StartPage 113 SubsetFonts true13 TransferFunctionInfo Apply13 UCRandBGInfo Preserve13 UsePrologue false13 ColorSettingsFile ()13 AlwaysEmbed [ true13 ]13 NeverEmbed [ true13 ]13 AntiAliasColorImages false13 CropColorImages true13 ColorImageMinResolution 30013 ColorImageMinResolutionPolicy OK13 DownsampleColorImages true13 ColorImageDownsampleType Bicubic13 ColorImageResolution 30013 ColorImageDepth -113 ColorImageMinDownsampleDepth 113 ColorImageDownsampleThreshold 15000013 EncodeColorImages true13 ColorImageFilter DCTEncode13 AutoFilterColorImages true13 ColorImageAutoFilterStrategy JPEG13 ColorACSImageDict ltlt13 QFactor 01513 HSamples [1 1 1 1] VSamples [1 1 1 1]13 gtgt13 ColorImageDict ltlt13 QFactor 01513 HSamples [1 1 1 1] VSamples [1 1 1 1]13 gtgt13 JPEG2000ColorACSImageDict ltlt13 TileWidth 25613 TileHeight 25613 Quality 3013 gtgt13 JPEG2000ColorImageDict ltlt13 TileWidth 25613 TileHeight 25613 Quality 3013 gtgt13 AntiAliasGrayImages false13 CropGrayImages true13 GrayImageMinResolution 30013 GrayImageMinResolutionPolicy OK13 DownsampleGrayImages true13 GrayImageDownsampleType Bicubic13 GrayImageResolution 30013 GrayImageDepth -113 GrayImageMinDownsampleDepth 213 GrayImageDownsampleThreshold 15000013 EncodeGrayImages true13 GrayImageFilter DCTEncode13 AutoFilterGrayImages true13 GrayImageAutoFilterStrategy JPEG13 GrayACSImageDict ltlt13 QFactor 01513 HSamples [1 1 1 1] VSamples [1 1 1 1]13 gtgt13 GrayImageDict ltlt13 QFactor 01513 HSamples [1 1 1 1] VSamples [1 1 1 1]13 gtgt13 JPEG2000GrayACSImageDict ltlt13 TileWidth 25613 TileHeight 25613 Quality 3013 gtgt13 JPEG2000GrayImageDict ltlt13 TileWidth 25613 TileHeight 25613 Quality 3013 gtgt13 AntiAliasMonoImages false13 CropMonoImages true13 MonoImageMinResolution 120013 MonoImageMinResolutionPolicy OK13 DownsampleMonoImages true13 MonoImageDownsampleType Bicubic13 MonoImageResolution 120013 MonoImageDepth -113 MonoImageDownsampleThreshold 15000013 EncodeMonoImages true13 MonoImageFilter CCITTFaxEncode13 MonoImageDict ltlt13 K -113 gtgt13 AllowPSXObjects false13 CheckCompliance [13 None13 ]13 PDFX1aCheck false13 PDFX3Check false13 PDFXCompliantPDFOnly false13 PDFXNoTrimBoxError true13 PDFXTrimBoxToMediaBoxOffset [13 00000013 00000013 00000013 00000013 ]13 PDFXSetBleedBoxToMediaBox true13 PDFXBleedBoxToTrimBoxOffset [13 00000013 00000013 00000013 00000013 ]13 PDFXOutputIntentProfile ()13 PDFXOutputConditionIdentifier ()13 PDFXOutputCondition ()13 PDFXRegistryName ()13 PDFXTrapped False1313 CreateJDFFile false13 Description ltlt13 ARA 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 BGR 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 CHS ltFEFF4f7f75288fd94e9b8bbe5b9a521b5efa7684002000410064006f006200650020005000440046002065876863900275284e8e9ad88d2891cf76845370524d53705237300260a853ef4ee54f7f75280020004100630072006f0062006100740020548c002000410064006f00620065002000520065006100640065007200200035002e003000204ee553ca66f49ad87248672c676562535f00521b5efa768400200050004400460020658768633002gt13 CHT ltFEFF4f7f752890194e9b8a2d7f6e5efa7acb7684002000410064006f006200650020005000440046002065874ef69069752865bc9ad854c18cea76845370524d5370523786557406300260a853ef4ee54f7f75280020004100630072006f0062006100740020548c002000410064006f00620065002000520065006100640065007200200035002e003000204ee553ca66f49ad87248672c4f86958b555f5df25efa7acb76840020005000440046002065874ef63002gt13 CZE 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 DAN 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 DEU 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 ESP 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 ETI 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 FRA ltFEFF005500740069006c006900730065007a00200063006500730020006f007000740069006f006e00730020006100660069006e00200064006500200063007200e900650072002000640065007300200064006f00630075006d0065006e00740073002000410064006f00620065002000500044004600200070006f0075007200200075006e00650020007100750061006c0069007400e90020006400270069006d007000720065007300730069006f006e00200070007200e9007000720065007300730065002e0020004c0065007300200064006f00630075006d0065006e00740073002000500044004600200063007200e900e90073002000700065007500760065006e0074002000ea0074007200650020006f007500760065007200740073002000640061006e00730020004100630072006f006200610074002c002000610069006e00730069002000710075002700410064006f00620065002000520065006100640065007200200035002e0030002000650074002000760065007200730069006f006e007300200075006c007400e90072006900650075007200650073002egt13 GRE 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 HEB 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 HRV (Za stvaranje Adobe PDF dokumenata najpogodnijih za visokokvalitetni ispis prije tiskanja koristite ove postavke Stvoreni PDF dokumenti mogu se otvoriti Acrobat i Adobe Reader 50 i kasnijim verzijama)13 HUN 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 ITA 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 JPN ltFEFF9ad854c18cea306a30d730ea30d730ec30b951fa529b7528002000410064006f0062006500200050004400460020658766f8306e4f5c6210306b4f7f75283057307e305930023053306e8a2d5b9a30674f5c62103055308c305f0020005000440046002030d530a130a430eb306f3001004100630072006f0062006100740020304a30883073002000410064006f00620065002000520065006100640065007200200035002e003000204ee5964d3067958b304f30533068304c3067304d307e305930023053306e8a2d5b9a306b306f30d530a930f330c8306e57cb30818fbc307f304c5fc59808306730593002gt13 KOR ltFEFFc7740020c124c815c7440020c0acc6a9d558c5ec0020ace0d488c9c80020c2dcd5d80020c778c1c4c5d00020ac00c7a50020c801d569d55c002000410064006f0062006500200050004400460020bb38c11cb97c0020c791c131d569b2c8b2e4002e0020c774b807ac8c0020c791c131b41c00200050004400460020bb38c11cb2940020004100630072006f0062006100740020bc0f002000410064006f00620065002000520065006100640065007200200035002e00300020c774c0c1c5d0c11c0020c5f40020c2180020c788c2b5b2c8b2e4002egt13 LTH 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 LVI 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 NLD (Gebruik deze instellingen om Adobe PDF-documenten te maken die zijn geoptimaliseerd voor prepress-afdrukken van hoge kwaliteit De gemaakte PDF-documenten kunnen worden geopend met Acrobat en Adobe Reader 50 en hoger)13 NOR 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 POL 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 PTB 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 RUM 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 RUS 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 SKY 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 SLV ltFEFF005400650020006e006100730074006100760069007400760065002000750070006f0072006100620069007400650020007a00610020007500730074007600610072006a0061006e006a006500200064006f006b0075006d0065006e0074006f0076002000410064006f006200650020005000440046002c0020006b006900200073006f0020006e0061006a007000720069006d00650072006e0065006a016100690020007a00610020006b0061006b006f0076006f00730074006e006f0020007400690073006b0061006e006a00650020007300200070007200690070007200610076006f0020006e00610020007400690073006b002e00200020005500730074007600610072006a0065006e006500200064006f006b0075006d0065006e0074006500200050004400460020006a00650020006d006f0067006f010d00650020006f0064007000720065007400690020007a0020004100630072006f00620061007400200069006e002000410064006f00620065002000520065006100640065007200200035002e003000200069006e0020006e006f00760065006a01610069006d002egt13 SUO 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 SVE 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 TUR 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 UKR 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 ENU (Use these settings to create Adobe PDF documents best suited for high-quality prepress printing Created PDF documents can be opened with Acrobat and Adobe Reader 50 and later)13 gtgt13 Namespace [13 (Adobe)13 (Common)13 (10)13 ]13 OtherNamespaces [13 ltlt13 AsReaderSpreads false13 CropImagesToFrames true13 ErrorControl WarnAndContinue13 FlattenerIgnoreSpreadOverrides false13 IncludeGuidesGrids false13 IncludeNonPrinting false13 IncludeSlug false13 Namespace [13 (Adobe)13 (InDesign)13 (40)13 ]13 OmitPlacedBitmaps false13 OmitPlacedEPS false13 OmitPlacedPDF false13 SimulateOverprint Legacy13 gtgt13 ltlt13 AddBleedMarks false13 AddColorBars false13 AddCropMarks false13 AddPageInfo false13 AddRegMarks false13 ConvertColors ConvertToCMYK13 DestinationProfileName ()13 DestinationProfileSelector DocumentCMYK13 Downsample16BitImages true13 FlattenerPreset ltlt13 PresetSelector MediumResolution13 gtgt13 FormElements false13 GenerateStructure false13 IncludeBookmarks false13 IncludeHyperlinks false13 IncludeInteractive false13 IncludeLayers false13 IncludeProfiles false13 MultimediaHandling UseObjectSettings13 Namespace [13 (Adobe)13 (CreativeSuite)13 (20)13 ]13 PDFXOutputIntentProfileSelector DocumentCMYK13 PreserveEditing true13 UntaggedCMYKHandling LeaveUntagged13 UntaggedRGBHandling UseDocumentProfile13 UseDocumentBleed false13 gtgt13 ]13gtgt setdistillerparams13ltlt13 HWResolution [2400 2400]13 PageSize [612000 792000]13gtgt setpagedevice13

Page 12: February 2014 MEE Questions and AnalysesPreface The Multistate Essay Examination (MEE) is developed by the National Conference of Bar Examiners (NCBE). This publication includes the

AGENCY AND PARTNERSHIP QUESTION _____

Five years ago Adam and Ben formed a general partnership Empire Partnership (Empire) to buy and sell antique automobiles at a showroom in State A Adam contributed $800000 to Empire and Ben contributed $200000 Their written partnership agreement allocated 80 of profits losses and control to Adam and 20 to Ben No filings of any type were made in connection with the formation of Empire

Three years ago a collector purchased one of Empirersquos antique cars for $3400000 The collector was willing to pay this price because of Benrsquos false representation (repeated in the sales contract) that a famous movie star had once owned the car Without the movie-star connection the car was worth only $100000 One month later when the collector discovered the truth he sued Adam Ben and Empire for $3300000 in damages The lawsuit is still pending

Two years ago Adam and Ben admitted a new partner Diane to Empire in return for her contribution of $250000 The three agreed to allocate profits losses and control 75 to Adam 10 to Ben and 15 to Diane Before joining the partnership Diane learned of the collectorrsquos claim and stated her concern to Adam and Ben that she might become liable if the claim were reduced to a judgment

Following Dianersquos admission to Empire the three partners sought to convert Empire into a limited liability partnership (LLP) Adamrsquos lawyer proposed to file with State A a ldquostatement of qualificationrdquo making an LLP election and declaring the name of the partnership to be ldquoEmpire LLPrdquo Benrsquos lawyer stated that this would not work and that a new LLP had to be formed with the assets of the old partnership transferred to the new one In the end the conversion was done the way Adamrsquos lawyer suggested with the approval of all three partners

One year ago a driver purchased a vintage car from Empire LLP based on the representation that the car was ldquofully roadworthy and capable of touring at 70 mph all dayrdquo The driver took the car on the highway at 50 mph whereupon the front suspension collapsed resulting in a crash in which the car was destroyed and the driver killed The driverrsquos estate sued Adam Ben Diane and Empire LLP for $10000000 The lawsuit is still pending

Although profitable Empire LLP does not have resources sufficient to pay the collectorrsquos claim or the claim of the driverrsquos estate

Assume that the Uniform Partnership Act (1997) applies

1 Before the filing of the statement of qualification (a) was Adam personally liable on the collectorrsquos claim Explain (b) was Diane personally liable on the collectorrsquos claim Explain

2 After the filing of the statement of qualification was Adam Ben or Diane personally liable as a partner on (a) the collectorrsquos claim or (b) the driverrsquos estatersquos claim Explain

8

February 2014 MEE

ANALYSES Constitutional Law

Trusts and Future Interests Secured Transactions

Federal Civil Procedure Criminal Law and Procedure

Agency and Partnership

CONSTITUTIONAL LAW ANALYSIS (Constitutional Law IVD)

ANALYSIS

Legal Problems

(1) Is the city ordinance requirement that businesses install floodlights a taking

(2) Is conditioning the approval of a building permit on the grant of an easement to install surveillance equipment a taking of property

DISCUSSION

Summary

The ordinance requiring businesses to install floodlights is not a per se taking under Loretto because it does not force a private landowner to allow a third party to enter and place a physical object on the land Here the city ordinance requires the businessmdashnot a third partymdashto install the floodlights

The ordinance is likely not a regulatory taking under the Penn Central balancing test While the ordinance will impose a cost on business owners that cost may be offset by the expected increase in business due to the ordinance and the ordinance does not appear to interfere with the ownerrsquos primary use of the property as a restaurant

The permit condition however is likely an uncompensated taking of property While the condition has an essential nexus with the cityrsquos legitimate interest in promoting public safety the city has not made an individualized determination that the easement condition is roughly proportional to the possibility of increased crime due to the restaurantrsquos proposed addition Thus the permit condition likely violates the Fifth Amendment as applied to the states through the Fourteenth Amendment

Point One (50) The ordinance requiring that businesses install floodlights is not a per se taking under Loretto It is not a regulatory taking under the Penn Central balancing test because the cost of compliance with the ordinance may be offset by an expected increase in business and compliance does not interfere with the businessrsquos primary use of its property as a restaurant

The city ordinance requiring a business to install floodlights does not effect a per se taking of the sort described in Loretto v Teleprompter Manhattan CATV Corp 458 US 419 (1982) because no property is physically taken by the government and the ordinance does not involve a physical invasion of private property by a third party

Even though the ordinance does not constitute an occupation of the property by either the government or a third party it is still subject to the three-factor balancing test under Penn Central Transportation Co v City of New York 438 US 104 (1978) to determine whether it is a ldquoregulatory takingrdquo Under Penn Central a court must balance (1) ldquo[t]he economic impact of the regulation on the claimantrdquo (2) ldquothe extent to which the regulation has interfered with distinct investment-backed expectationsrdquo and (3) ldquothe character of the governmental actionrdquo Id at 124 Here each factor weighs against finding that the ordinance is a taking

11

Constitutional Law Analysis

First the ordinance requirement likely has a minimal economic impact on the restaurant Compliance with the ordinance is estimated to cost $1000 and the city has found that businesses will likely recoup that cost in increased sales Also because the ordinance does not interfere with the operation of the restaurant the owner may still earn a reasonable return on its investment in the property

Second the ordinance does not interfere with the businessrsquos investment-backed expectations As in Penn Central the challenged law does not interfere with the ownerrsquos ldquoprimary expectationrdquo for use of the propertymdashin Penn Central as a railroad terminal and here as a restaurant Further the ordinance does not prevent the restaurant from expanding to meet the changing business environment

Third the character of the government action does not weigh in favor of a taking While Penn Central does say that a ldquophysical invasionrdquo is more likely to pose a taking Loretto suggests that the Courtrsquos main concern is with physical invasions by third parties Also like the landmark law challenged in Penn Central the ordinance here ldquoadjust[s] the benefits and burdens of economic life to promote the common goodrdquo Id In Penn Central the landmark law restricted development of the railroad terminal to promote the common interest in preserving historic landmarks Here the ordinance requires the businesses to install floodlights to promote the common interest in crime prevention and public safety

Because the ordinance is clearly a valid exercise of the police power it satisfies the takings clausersquos public-use requirement Kelo v City of New London 545 US 469 (2005)

In sum all three factors weigh against finding a taking under the Penn Central balancing test

Point Two (50) The permit condition may be unconstitutional as an uncompensated taking of property because the city has not made an individualized determination that the easement condition is roughly proportional to the impact of the restaurantrsquos proposed addition

In Dolan v City of Tigard 512 US 374 (1994) the Supreme Court set forth the test for determining whether an exaction imposed by a government in exchange for a discretionary benefit conferred by the government such as a condition on the approval of a building permit in this case constitutes an uncompensated taking under the Fifth Amendment The exaction is not a taking if (1) there is an ldquoessential nexusrdquo between the ldquopublic need or burdenrdquo to which the proposed development contributes and ldquothe permit condition exacted by the cityrdquo id at 386 and (2) the government makes ldquosome sort of individualized determination that the required dedication is [roughly proportional] both in nature and extent to the impact of the proposed developmentrdquo Id at 391 see also Nollan v California Coastal Commission 483 US 825 (1987)

Here the city likely can meet the nexus requirement In Dolan the landowner sought to double the size of its business which would have increased traffic on nearby roadways In exchange for approving the development the city sought an easement for a bike and pedestrian path The Court found the required nexus between the easement and the cityrsquos ldquoattempt to reduce traffic congestion by providing for alternative means of transportationrdquo 512 US at 387 Here a similar nexus likely exists between the requested easement and the cityrsquos interest in crime prevention and public safety Increased patronage and economic activity at the restaurant might attract additional crime to the area and the requested easement to install surveillance equipment would attempt to address that increased crime

12

Constitutional Law Analysis

The exaction here however may fail the second prong of the Dolan testmdashthat the exaction be roughly proportional to the anticipated impact of the requested development As noted the city in Dolan claimed that a bike and pedestrian path was needed to offset the increase in traffic due to the proposed doubling of the business The Court explained that the government must demonstrate that the additional traffic reasonably was related to the requested exaction and that the government must ldquomake some effort to quantify its findings in support of the dedication for the pedestrianbicycle pathway beyond the conclusory statement that it could offset some of the traffic demand generatedrdquo Id at 395 Here the city did not carry its burden The city simply speculates that increased patronage of the restaurant ldquomightrdquo increase crime and that the surveillance equipment ldquomightrdquo alleviate this increased crime Because the city has not made ldquosome effort to quantify its findingsrdquo in support of the easement it has not shown that the burden of the easement is roughly proportional to the benefits thought to flow from it

Thus the exaction appears to be an uncompensated taking of property in violation of the Fifth Amendment as applied to the states through the Fourteenth Amendment

13

TRUSTS AND FUTURE INTERESTS ANALYSIS ____ (Trusts and Future Interests IE3 I5 IIIA amp B)

ANALYSIS

Legal Problems

(1) How should rents dividends and sales proceeds received by the trustee prior to receipt of the sonrsquos letter have been allocated between trust income and principal

(2)(a) Did the remainder interest in the trust accelerate and become immediately payable to the daughterrsquos minor child upon the trusteersquos receipt of the sonrsquos letter and if not how should the trustee handle the distribution of the principal in the future

(2)(b) Following the trusteersquos receipt of the sonrsquos letter how should the trustee distribute future receipts of income prior to the distribution of the principal

DISCUSSION

Summary

Prior to the trusteersquos receipt of the sonrsquos letter cash dividends and rents should have been allocated to trust income and were distributable to the son the income beneficiary of the trust sales proceeds and stock dividends should have been allocated to principal

Because the sonrsquos letter to the trustee did not result in a valid disclaimer under state law (having been made more than nine months after the testatorrsquos death) the son is not deemed to have predeceased the testator Because the son is still living the class gift to the testatorrsquos grandchildren who survive the son has not closed and is not possessory it will not become possessory until the son dies The daughterrsquos minor child being the testatorrsquos only living grandchild is not currently entitled to a distribution of trust principal Trust principal will instead be distributable upon the sonrsquos death to the testatorrsquos then-living grandchildren or if there are none to the testatorrsquos then-living heirs

As for future income the trustee should either distribute the trust income to the son and the daughter as the testatorrsquos heirs accumulate the income for future distribution to those individuals ultimately entitled to the trust principal or distribute it to those presumptively entitled to the principal upon the sonrsquos death ie the daughterrsquos minor child

Point One (45) Cash dividends and rents are allocable to income sales proceeds and stock dividends are allocable to principal Items allocable to income for the period prior to the sonrsquos attempted disclaimer were distributable to the son

Receipts earned during the administration of a trust are allocable either to income or to principal Almost all states have adopted the most recent or an earlier version of the Uniform Principal and Income Act (the Act) which specifies how such receipts should be allocated

Under the Act rents (UNIF PRIN amp INC ACT (2000) sect 405 UNIF PRIN amp INC ACT (1962) sect 3(a)(1)) and cash dividends received from a corporation (UNIF PRIN amp INC ACT (2000) sect 401(b) UNIF PRIN amp INC ACT (1962) sect 6(d)) are allocable to income and are distributable to the income beneficiary of the trust

14

Trusts and Future Interests Analysis Sales proceeds (UNIF PRIN amp INC ACT (2000) sect 404(2) UNIF PRIN amp INC ACT (1962)

sect 3(b)(1)) and dividends paid in the stock of the distributing corporation (UNIF PRIN amp INC ACT (2000) sect 401(c)(1) UNIF PRIN amp INC ACT (1962) sect 3(b)(4)) are allocable to principal and added to the principal of the trust

Here the cash dividends and office building rents should have been allocated to income and until the trustee received the sonrsquos letter should have been distributed to him as the sole income beneficiary of the trust The stock dividend and proceeds from the sale of the office building should have been allocated to principal and held by the trustee for future distribution to the ultimate remaindermen of the trust

[NOTE The 2000 Uniform Principal and Income Act has been adopted in Alabama Arkansas Colorado Connecticut the District of Columbia Hawaii Idaho Iowa Kentucky Missouri Montana Nebraska New Mexico North Dakota Oregon South Dakota Utah and West Virginia]

Point Two(a) (45) Because the son did not disclaim within nine months of the testatorrsquos death there is no valid disclaimer under state law Therefore the son is not deemed to have predeceased the testator Furthermore because of the express survivorship contingency in the will the remainder in the trust does not accelerate and become distributable until the son in fact dies When the son dies the trust principal will be distributable to the testatorrsquos then-living grandchildren or if none then to the testatorrsquos then-living heirs

When a trust remainder is given to a class the class closes (ie no new persons can join the class) when there is no outstanding income interest and at least one member of the class is then entitled to demand possession of his or her share of the remainder This principle is called the rule of convenience See generally HERBERT HOVENKAMP amp SHELDON F KURTZ PRINCIPLES OF PROPERTY LAW 199ndash200 (6th ed 2005) A class member may demand possession of his or her share of the remainder upon termination of the income interest only when the class memberrsquos interest is not otherwise subject to a condition precedent See id

When a beneficiary timely disclaims an interest in a trust that beneficiary is treated as if he had predeceased the testator Here had the son disclaimed within nine months of the testatorrsquos death as required by the state statute he would have been deemed to have predeceased the testator This would have closed the class of remaindermen and the testatorrsquos then-living grandchildren (ie the daughterrsquos child) would have been entitled to the trust principal However under the state statute the sonrsquos disclaimer was not timely because he did not disclaim within nine months of the testatorrsquos death Thus because the statute is inapplicable and the son is still alive the class of grandchildren entitled to share in trust principal did not close

Because here the statute is inapplicable due to the sonrsquos failure to comply with the statutory time requirements then presumably the common-law rule allowing disclaimers (aka renunciations) at any time should apply Under the common law if a life estate is renounced the remainder interest accelerates and becomes immediately distributable to the remaindermen of the trust if the remainder is vested but not if the remainder is contingent JESSE DUKEMINIER amp ROBERT H SITKOFF WILLS TRUSTS AND ESTATES 844ndash845 (9th ed 2013) Here because the remainder is contingent upon there being grandchildren who survive the son the remainder will not accelerate It will remain open until the son dies leaving open the possibility that additional grandchildren will be included in the class or the daughterrsquos child could fall out of the class because that child fails to survive the son

And if none of the testatorrsquos grandchildren survive the son the trust principal will be distributed to the testatorrsquos heirs living at the sonrsquos death

15

Trusts and Future Interests Analysis

Point Two(b) (10) Until the trust terminates the trustee must continue to hold the trust assets The distribution of income in the meantime is unclear There are at least three possibilities Income earned on the undistributed assets could be distributed to the son and daughter as the testatorrsquos heirs accumulated and added to principal for distribution to the ultimate remaindermen or distributed from time to time to those persons who are presumptively remaindermen

When trust principal is not immediately distributable the trustee must continue to hold trust assets until the ultimate remaindermen are ascertained During this period trust income will be distributed or retained according to any instructions contained in the trust instrument See WILLIAM M MCGOVERN JR SHELDON F KURTZ amp DAVID M ENGLISH WILLS TRUSTS amp ESTATES sect 102 (4th ed 2010)

Here the testator did not specify what the trustee should do with trust income in the event the sonrsquos disclaimer did not comply with the state statute There are at least three approaches One approach would have the trustee distribute the trust income to the testatorrsquos heirs on the theory that the income represents property that was not disposed of by the testatorrsquos will and which thus passes by partial intestacy to the testatorrsquos heirs A second approach would have the trustee accumulate trust income for distribution to the ultimate remaindermen Under this approach only those individuals ultimately entitled to the principal would share in the income A third approach would have the trustee distribute trust income to those individuals who would be the remaindermen if the trust were to terminate when the income is received by the trustee under this approach trust income would be distributed to the daughterrsquos minor child until another presumptive remainderman is born This approach could result in individuals not ultimately entitled to principal say because they do not survive the son receiving income It could also result in a disproportionate distribution of income among the individuals ultimately entitled to income

[NOTE Examinees should demonstrate a recognition and understanding of the income-allocation problem and the alternatives available to address that issue There is no widely accepted solution to the problem Examinees who cite any of these possible problem-solving approaches may receive credit]

16

SECURED TRANSACTIONS ANALYSIS (Secured Transactions IB IID E amp F IIIB IVA B amp F)

ANALYSIS

Legal Problems

(1)(a) What is the nature of the bankrsquos claim to the businessrsquos equipment

(1)(b) What is the nature of the finance companyrsquos claim to the businessrsquos equipment

(1)(c) As between the bank and the finance company whose claim to the businessrsquos equipment has priority

(2) Do the claims of the bank and the finance company continue in the item of equipment sold by the business to the competitor

DISCUSSION

Summary

The bank and the finance company both have perfected security interests in the businessrsquos equipment Even though the finance companyrsquos perfected security interest was created first the bankrsquos perfected security interest has priority because the bankrsquos financing statement was filed before the finance companyrsquos financing statement The security interests of the bank and the finance company continue in the item of equipment sold by the business to the competitor because their security interests were perfected and the competitor was not a buyer in ordinary course of business

Point One(a) (25) The bank has a perfected security interest in the businessrsquos equipment

The bank has met all criteria necessary for it to have an attached and enforceable security interest in the businessrsquos equipment First value must be given UCC sect 9-203(b)(1) This criterion is fulfilled by the loan made by the bank to the business Second the debtor must have rights in the collateral UCC sect 9-203(b)(2) Clearly the business has rights in its equipment Third either the secured party must take possession of the collateral or the debtor must authenticate a security agreement containing a description of the collateral UCC sect 9-203(b)(3) The agreement that the business owner signed is a ldquosecurity agreementrdquo because it is an agreement that creates or provides for a security interest UCC sect 9-102(a)(74) By signing the security agreement the business owner authenticated it UCC sect 9-102(a)(7) Therefore all three criteria are fulfilled and the bank has an enforceable and attached security interest

A security interest is perfected when it has attached and when any additional steps required for perfection have occurred UCC sect 9-308(a) Generally speaking the additional steps will either be possession of the collateral by the secured party or the filing of a financing statement with respect to the collateral See UCC sectsect 9-310 9-313 In this case the bank filed a financing statement naming the debtor and sufficiently indicating the collateral The collateral indication is sufficient because it identifies the collateral by type of property See UCC sectsect 9-504 9-108 The fact that the financing statement was filed before the security interest was created is

17

Secured Transactions Analysis

not a problem Even though the security agreement had not yet been signed the business had authorized the filing of the financing statement in an authenticated record UCC sect 9-509(a)(1) Moreover the financing statement may be filed before the security agreement is created UCC sect 9-502(d)

Point One(b) (10) The finance company also has a perfected security interest in the businessrsquos equipment

The finance companyrsquos security interest is enforceable and attached for the same reasons as the bankrsquos security interest The loan from the finance company to the business constitutes value the business has rights in the collateral and the business owner has authenticated a security agreement containing a description of the collateral The finance companyrsquos security interest is perfected because the finance company filed a financing statement with respect to it that provides that the business is the debtor and indicates that the collateral is equipment

Point One(c) (30) The bankrsquos security interest has priority over the finance companyrsquos security interest because the bankrsquos financing statement was filed first

As between two perfected security interests the general rule is that the security interest that was the earlier to be either perfected or the subject of a filed financing statement has priority UCC sect 9-322(a)(1) While the finance companyrsquos security interest was perfected before the bankrsquos (March 15 vs March 22) the bankrsquos financing statement was filed even earlier on March 2 Thus under the first-to-file-or-perfect rule of UCC sect 9-322(a)(1) the bankrsquos security interest has priority No exceptions to the general rule apply here

Point Two (35) A security interest in collateral continues notwithstanding its sale unless an exception applies Because the security interests of the bank and the finance company were perfected and the competitor was not a buyer in ordinary course of business no exception applies and the security interests of both creditors continue in the equipment sold to the competitor

As a general rule a security interest in collateral continues notwithstanding the fact that the debtor has sold the collateral to another person UCC sect 9-315(a)(1) Thus unless an exception applies the security interests of the bank and the finance company will continue in the item of equipment sold to the competitor

A buyer of goods will take free of an unperfected security interest in those goods See UCC sect 9-317(a)(2) However when the competitor bought the businessrsquos equipment both the bank and the finance company had perfected security interests in the equipment

A buyer can take free even of a perfected security interest in goods if the buyer is a ldquobuyer in ordinary course of businessrdquo See UCC sect 9-320(a) However the competitor was not a buyer in ordinary course of business To be a ldquobuyer in ordinary course of businessrdquo a buyer must buy goods from a seller that is in the business of selling goods of that kind See UCC sect 1-201(b)(9) The competitor bought this equipment from a seller that is not in the business of selling goods of this kind so the competitor was not a buyer in ordinary course of business with respect to these goods

Because no exception applies the security interests of the bank and the finance company continue even after the item of equipment was sold to the competitor

18

FEDERAL CIVIL PROCEDURE ANALYSIS (Federal Civil Procedure IVD)

ANALYSIS

Legal Problems

(1) Is a document prepared in the course of a contract dispute protected from discovery as ldquowork productrdquo when there is no evidence that the document was prepared in anticipation of litigation

(2)(a) Is a partyrsquos failure to provide relevant electronically stored information excused when the information was destroyed pursuant to a routine document retention scheme at a time when litigation was contemplated by the destroying party

(2)(b) What sanctions should be imposed on a party for allowing the destruction of evidence that is relevant to potential future litigation

DISCUSSION

Summary

The report prepared by the structural engineer is probably not work product and is thus discoverable The engineer examined the foundation of the house at the customerrsquos request and the engineerrsquos findings are potentially relevant to the customerrsquos claim that the foundation is defective The report was not prepared in anticipation of litigation The customer appears to have sought the engineerrsquos opinion in response to the builderrsquos offer to fix any problems with the foundation that an engineer might identify Because the report was not prepared in anticipation of litigation it is not protected by the work-product doctrine

The builder should have taken appropriate steps to preserve evidence including suspending its document retention program as soon as it began planning for litigationmdashie on July 10 Its destruction of potentially relevant material after that date was wrongful However a court is unlikely to impose severe sanctions on the builder because there are no facts indicating that the builder acted in bad faith and the customer can prove that the foundation is defective without the destroyed emails

Point One (40) The customer must turn over the engineerrsquos report because it was not prepared in anticipation of litigation

In general a party to a lawsuit in federal court ldquomay obtain discovery regarding any nonprivileged matter that is relevant to any partyrsquos claim or defenserdquo FED R CIV P 26(b)(1) (2009) This includes the right to inspect and copy documents in the other partyrsquos possession FED R CIV P 34(a)(1) Here the customer hired a structural engineer to examine the foundation of the house The engineerrsquos report on the foundation is likely to include information that would be relevant to the customerrsquos claim that the foundation was defectively constructed

The so-called ldquowork productrdquo rule allows a party to refuse to turn over ldquodocuments that are prepared in anticipation of litigation or for trialrdquo by that partyrsquos representative including

19

Federal Civil Procedure Analysis

a consultant Thus if the customer had hired the structural engineer to prepare a report ldquoin anticipation of litigationrdquo that report might not be discoverable See FED R CIV P 26(b)(3)

In this case however the customer hired the engineer to evaluate the foundation of the house as part of the customerrsquos negotiation with the builder concerning the housersquos flooding problem The builder told the customer that the housersquos landscaping was the reason for the flooding and the builder told the customer ldquoHave an engineer look at the foundation If therersquos a problem wersquoll fix itrdquo The customer appears to have acted in response to that statement There is no indication that the customer anticipated any kind of legal action at the time that the structural engineer was hired Accordingly the structural engineerrsquos report is discoverable and the court should order the customer to turn it over

[NOTE If an examinee concludes that the structural engineerrsquos report was prepared in anticipation of litigation then the examinee should also conclude that the report is not discoverable Documents prepared in anticipation of litigation do not need to be disclosed to an adverse party unless that party can demonstrate a ldquosubstantial needrdquo for the documents and an inability to obtain substantially equivalent information without ldquoundue hardshiprdquo FED R CIV P 26(b)(3)(A)(ii) Furthermore a report prepared by an expert who is not expected to testify is not discoverable in the absence of ldquoexceptional circumstancesrdquo making it ldquoimpracticablerdquo to obtain the information in another way FED R CIV P 26(b)(4)(D)(ii) The builder probably cannot make these showings here unless the engineerrsquos report deals with circumstances that have since changed There is no evidence that the structural engineer would have had access to any information or facts that the builder would not already know as a result of its construction and subsequent inspection of the house In addition if necessary the builder could ask the court for permission to arrange for a further inspection of the house by a structural engineer hired by the builder See FED R CIV P 34(a)(2) Accordingly if an examinee concludes that the report was prepared in anticipation of litigation the examinee should also conclude that the builder is not entitled to see the report]

Point Two(a) (30) Because the builder anticipated that it might be involved in litigation concerning its contract with the customer the builder acted wrongfully in destroying emails that were relevant to the housersquos construction even though the emails were destroyed pursuant to a routine document retention plan

As noted above a party to a lawsuit in federal court ldquomay obtain discovery regarding any nonprivileged matter that is relevant to any partyrsquos claim or defenserdquo FED R CIV P 26(b)(1) This includes emails and other electronically stored information FED R CIV P 34(a)(1)(A) Here the customer has requested all the builderrsquos emails pertaining to work done on the foundation of the house Ordinarily the builder would be obliged to turn over this information which is relevant to the customerrsquos defense that the housersquos foundation was poorly constructed

Unfortunately the emails in question no longer exist because the builder destroyed them on August 2

In general spoliation of evidence (destruction or alteration of evidence) is improper if the party who destroyed or altered the evidence ldquohas notice that the evidence is relevant to litigation or should have known that the evidence may be relevant to future litigationrdquo Fujitsu Ltd v Federal Express Corp 247 F3d 423 436 (2d Cir 2001) It is improper for a party to destroy electronic information relevant to pending litigation even if the destruction occurs before there is any request or order seeking the information See eg Leon v IDX Sys Corp 464 F3d 951 (9th Cir 2006) (plaintiffrsquos intentional destruction of computer files warranted dismissal even

20

In this case the builderrsquos destruction of the emails was pursuant to a routine document retention plan The Federal Rules provide expressly that in the absence of ldquoexceptional circumstancesrdquo parties should not be sanctioned for the loss of electronically stored information when the loss occurs pursuant to ldquoroutine good-faith operation of an electronic information systemrdquo FED R CIV P 37(e) However when a party anticipates litigation ldquoit must suspend its routine document retentiondestruction policy and put in place a lsquolitigation holdrsquo to ensure the preservation of relevant documentsrdquo Zubulake v UBS Warburg LLC 220 FRD 212 218 (SDNY 2003)

Federal Civil Procedure Analysis

though spoliation occurred before order compelling discovery) Similarly the duty to preserve evidence applies to a party who anticipates litigation even if litigation has not yet been commenced See THE SEDONA PRINCIPLES BEST PRACTICES RECOMMENDATIONS amp PRINCIPLES FOR ADDRESSING ELECTRONIC DOCUMENT PRODUCTION 70 cmt 14a (2d ed 2007)

The builder destroyed the emails on August 2 At that time the builder knew that litigation was a possibility because the builder had already directed its attorney to prepare a draft complaint for possible filing Knowing that litigation was a possibility the builder had a duty to take steps to preserve evidence including the emails in question See generally Fujitsu Ltd

Thus the builderrsquos destruction of potentially relevant emails at a time when it knew that litigation was a possibility was improper It had a duty to preserve evidence and it breached that duty

[NOTE Because courts have used different words to describe the test for when evidence must be preserved an examineersquos precise formulation of the test is not critical]

Point Two(b) (30) In determining appropriate sanctions for spoliation courts consider both the level of culpability of the spoliating party and the degree of prejudice the loss of evidence has caused the other party Here the builderrsquos destruction of evidence does not appear to have been willful nor is it likely to pose a significant obstacle to the customerrsquos defense Any sanctions imposed by the court should be modest

Federal courts have inherent power to control the litigation process and can sanction misbehavior including spoliation even when there has been no specific violation of the Federal Rules of Civil Procedure See generally Chambers v NASCO Inc 501 US 32 (1991) (discussing courtrsquos inherent power to control the litigation process) The range of available sanctions is broad It can include such sanctions as the payment of expenses incurred by the other party as a result of the destruction of the evidence an instruction to the jury authorizing it to draw an adverse inference from the destruction of the evidence a shifting of the burden of proof on the relevant issue or even judgment against the responsible party See eg Residential Funding Corp v DeGeorge Financial Corp 306 F3d 99 108 (2d Cir 2002) (adverse inference) Silvestri v General Motors Corp 271 F3d 583 593 (4th Cir 2001) (possibility of dismissal) Cf FED R CIV P 37(b)(2)(A) (listing remedies for failure to comply with discovery obligations)

In determining appropriate sanctions for spoliation courts consider both the level of culpability of the spoliating party and the degree of prejudice the loss of evidence has caused the other party Many courts impose severe sanctions (such as an adverse-inference instruction or the entry of judgment against the spoliating party) only when there is evidence of bad faith in the form of an intentional effort to hide information Eg Greyhound Lines Inc v Wade 485 F3d 1032 1035 (8th Cir 2007) (spoliation sanction requires intentional destruction out of desire ldquoto suppress the truthrdquo) However other courts have said that negligence in preserving evidence can

21

Federal Civil Procedure Analysis

support an adverse-inference instruction See Residential Funding 306 F3d at 108 (negligence enough under some circumstances)

Although a court might well order an evidentiary hearing on the issue of sanctions the facts presented do not seem appropriate for severe sanctions First the evidence was destroyed pursuant to the builderrsquos standard document retention plan and there is no evidence that the builder deliberately failed to suspend its usual procedures with the purpose of allowing the destruction of evidence Second the loss of this evidence will not severely hinder the customerrsquos presentation of his case The central issue is whether the foundation of the house was properly constructed If the construction job was poorly done the customer can present evidence derived from inspection of the premises to prove that point The customer can also depose witnesses about any issues that arose during construction

Under the circumstances a court is not likely to impose particularly severe sanctions although it might shift the burden to the builder to show that the foundation was properly constructed or it might require the builder to reimburse any expenses the customer incurs to discover and prove the facts about issues or disputes that arose during construction of the foundation

[NOTE The result reached by the examinee is less important than the examineersquos recognition that (a) a range of sanctions is available to the court and (b) the appropriate sanction depends both on the culpability of the builder and the prejudice suffered by the customer]

22

CRIMINAL LAW AND PROCEDURE ANALYSIS (Criminal Law and Procedure IIA amp D VE amp F)

ANALYSIS

Legal Problems

(1) Did charging the defendant with both theft and burglary constitute double jeopardy

(2) Did the jury instruction violate the due process clause either by relieving the prosecution of the burden of proving the element of intent or by shifting the burden to the defendant to disprove that element

(3) Did the sentence imposed in this case for the theft conviction unconstitutionally deprive the defendant of his right to a jury trial on the issue of the value of the stolen item

DISCUSSION

Summary

The trial court properly denied the defendantrsquos pretrial motion to dismiss the charges on double jeopardy grounds The defendant may be charged with and convicted of both theft and burglary Each of the charges has an element that the other does not Neither charge is a lesser-included offense nor are they multiplicitous Thus charging both theft and burglary does not violate double jeopardy

The jury instruction on the burglary charge was constitutionally flawed It could have been reasonably understood by the jury as either (1) an irrebuttable conclusive presumption (which relieved the prosecution of proving the element of intent and removed the issue from the jury) or (2) a rebuttable mandatory presumption (which unconstitutionally shifted the burden of proof on an element of a charged offense from the prosecution to the defendant)

Because the four-year sentence imposed by the judge was based on the judgersquos finding by a preponderance of the evidence that the value of the stolen ring exceeded $5000 the sentence violates the defendantrsquos right to a jury determination beyond a reasonable doubt of the value of the ring

Point One (30) Charging the defendant with theft and burglary did not constitute double jeopardy

The Double Jeopardy Clause of the Fifth Amendment provides that a person shall not be twice put in jeopardy for the ldquosame offenserdquo Thus the question is whether the elements of the theft charge are wholly contained in the burglary charge or vice versa If the elements of the lesser charge (theft) are not wholly contained in the greater charge (burglary)mdashie if each charge requires proof of a fact that the other does notmdashthen convicting the defendant of both crimes would not violate double jeopardy even when the two offenses occurred at the same time and are thus arguably part of the ldquosame transactionrdquo Blockburger v United States 284 US 299 304 (1932) See also Albernaz v United States 450 US 333 344 n3 (1981) United States v Dixon 509 US 688 704 (1993)

23

Criminal Law and Procedure Analysis

Here theft and burglary each require proof of an element not required for the other crime Burglary may be defined differently in different jurisdictions However it almost invariably requires entry into a building or dwelling of another with the specific intent to commit a felony therein and the crime of burglary is complete upon the entry into the building or dwelling with such intent See eg Cannon v Oklahoma 827 P2d 1339 1342 (Okla Crim App 1992) In contrast theft which also may be defined differently in different states almost invariably requires the taking and carrying away of an item of personal property belonging to another with the intent to steal or permanently deprive the owner of possession

Here the ldquotakingrdquo or ldquostealingrdquo element is not contained in the definition of burglary and the ldquoentryrdquo element of burglary is not contained in the definition of theft Because theft is not a lesser-included offense of burglary and burglary is not a lesser-included offense of theft charging the defendant for both burglary and theft did not violate double jeopardy and the court properly denied the defense motion on those grounds Yparrea v Dorsey 64 F3d 577 579ndash80 (10th Cir 1995) citing Blockburger 284 US at 304

Finally the defendantrsquos motion to dismiss all the charges on double jeopardy grounds was improper because if both charges were for the same offense the motion should have requested dismissal of one charge not both

Point Two (35) The jury instruction on the burglary charge violated the Due Process Clause because it created either (1) an irrebuttable conclusive presumption (which relieved the prosecution of proving the element of intent and removed that issue from the jury) or (2) a rebuttable mandatory presumption (which unconstitutionally shifted the burden of proof on an element of a charged offense to the defendant)

The Supreme Court has interpreted the Due Process Clause of the US Constitution to require that the prosecution prove all elements of an offense beyond a reasonable doubt See In re Winship 397 US 358 364 (1970) The burden of proof cannot be shifted to the defendant by presuming an essential element upon proof of other elements of the offense because shifting the burden of persuasion with respect to any element of a criminal offense is contrary to the Due Process Clause See Mullaney v Wilbur 421 US 684 (1975)

The crime of burglary includes entry into a building or dwelling with the specific intent to commit a felony therein The requirement that the prosecutor prove beyond a reasonable doubt that the defendant had this specific intent distinguishes burglary from general-intent crimes like trespass See Sandstrom v Montana 442 US 510 523 (1979)

Here the jury was instructed that if ldquoafter consideration of all the evidence presented by the prosecution and defense you find beyond a reasonable doubt that the defendant entered the dwelling without the ownersrsquo consent you may presume that the defendant entered with the intent to commit a felony thereinrdquo This instruction was unconstitutional because it created either an irrebuttable conclusive presumption or a rebuttable mandatory presumption

A conclusive presumption is ldquoan irrebuttable direction by the court to find intent once convinced of the facts triggering the presumptionrdquo Id at 517 Here the jurors were instructed that once the prosecutor established that the defendant entered the neighborsrsquo house without consent they ldquomay presumerdquo that he intended to commit a felony therein The jurors may have reasonably concluded from this instruction that if they found that the defendant intended to enter his neighborsrsquo home without permission they must further find that he entered with the specific intent to commit a felony therein Because this instruction could operate as a conclusive

24

Criminal Law and Procedure Analysis

irrebuttable presumption by eliminating intent ldquoas an ingredient of the offenserdquo it violated due process by relieving the prosecution of the burden of proof for this element Id at 522

In the alternative the jury instruction could have been reasonably understood to create a rebuttable mandatory presumption which ldquotells [the jury] they must find the elemental fact upon proof of the basic fact at least unless the defendant has come forward with some evidence to rebut the presumed connection between the two factsrdquo County Court of Ulster County New York v Allen 442 US 140 157 (1979) The due process problem created by rebuttable mandatory presumptions is that ldquo[t]o the extent that the trier of fact is forced to abide by the presumption and may not reject it based on an independent evaluation of the particular facts presented by the State the analysis of the presumptionrsquos constitutional validity is logically divorced from those facts and based on the presumptionrsquos accuracy in the run of casesrdquo Id at 159

Unlike irrebuttable conclusive presumptions rebuttable mandatory presumptions are not always per se violations of the Due Process Clause However the Supreme Court of the United States has held that jury instructions that could reasonably be understood as shifting the burden of proof to the defendant on an element of the offense are unconstitutional Francis v Franklin 471 US 307 (1985) Here the argument that the jury instruction operated as a rebuttable mandatory presumption is supported by the fact that the judge also instructed the jury to ldquoconsider[ ] all the evidence presented by the prosecution and defenserdquo However even if the instruction created a rebuttable mandatory presumption it would be unconstitutional because it shifted the burden to the defense on an element of the offense Sandstrom 442 US at 524 Mullaney 421 US at 686

[NOTE Whether an examinee identifies the jury instruction as containing a ldquoconclusiverdquo or ldquomandatoryrdquo presumption is less important than the examineersquos analysis of the constitutional infirmities]

Point Three (35) The trial court violated the defendantrsquos Sixth Amendment right to a jury trial on an essential element of the offense when it found by a preponderance of the evidence that the ring was worth over $5000 and increased the defendantrsquos sentence based on this finding

In the statutory scheme under which the defendant was tried and convicted a Class D felony theft is defined as theft of item(s) with a value between $2500 and $10000 The jury found that the value of the diamond ring was at least $2500 and convicted the defendant of felony theft However at sentencing the trial court made a separate finding by a preponderance of the evidence that the value of the ring was greater than $5000 Following the statutersquos two-tiered sentencing scheme the judge then imposed on the defendant a sentence that was one year longer than the maximum that would otherwise have been allowed

The judgersquos sentence was unconstitutional because it violated the defendantrsquos Sixth Amendment right to a jury trial on this question The Supreme Court held in Apprendi v New Jersey 530 US 466 (2000) that ldquo[o]ther than the fact of a prior conviction any fact that increases the penalty for a crime beyond the prescribed statutory maximum must be submitted to a jury and proved beyond a reasonable doubtrdquo because ldquo[i]t is unconstitutional for a legislature to remove from the jury the assessment of facts that increase the prescribed range of penalties to which a criminal defendant is exposed [because] such facts must be established by proof beyond a reasonable doubtrdquo Id The Court reaffirmed Apprendi in Blakely v Washington 542 US 296 (2004) holding that the ldquolsquostatutory maximumrsquo for Apprendi purposes is the maximum sentence a judge may impose solely on the basis of the facts reflected in the jury verdict or admitted by the defendantrdquo Id at 303 (emphasis in original) In United States v Booker 543 US 220 (2005)

25

Criminal Law and Procedure Analysis

the Court relied on Blakely and Apprendi to conclude that protecting a defendantrsquos Sixth Amendment right to a jury trial required that ldquo[a]ny fact which is necessary to support a sentence exceeding the maximum authorized by the facts established by a plea of guilty or a jury verdict must be admitted by the defendant or proved to a jury beyond a reasonable doubtrdquo Id at 244

Thus in order to constitutionally increase a sentence above the statutory maximum of three years the jury must have found beyond a reasonable doubt that the value of the ring exceeded $5000 Here the court made the finding based on an appraisal proffered by the prosecutor only at sentencing and the judgersquos finding was by a preponderance of the evidence rather than beyond a reasonable doubt

26

AGENCY AND PARTNERSHIP ANALYSIS __________ (Agency and Partnership VA amp C VI)

ANALYSIS

Legal Problems

(1) Is a partner in a general partnership personally liable on a claim arising from misrepresentations by another partner made in the course of the partnership business

(2) Does a newly admitted partner in a general partnership become personally liable on existing claims against the partnership

(3) After the filing by a general partnership of a statement of qualification as a limited liability partnership are the partners personally liable as partners on (a) an existing claim against the general partnership and (b) a claim against the partnership that arose after the filing

DISCUSSION

Summary

Adam and Ben formed a general partnership under which they were jointly and severally liable for obligations of the partnership Thus Adam was personally liable for misrepresentations by Ben made in the ordinary course of the partnership business

Upon joining the general partnership Diane became personally liable for the obligations of the partnership arising after her admission but not for obligations pre-existing her admission such as the collectorrsquos claim

By filing a statement of qualification the three partners properly elected limited liability partnership status As partners in an LLP none of the three partners is personally liable as a partner for partnership obligations arising after the election such as the claim by the driverrsquos estate The election however does not change their personal liability on pre-existing claims that arose before the election such as the collectorrsquos claim

Point One (30) As a general partner of Empire a general partnership Adam became personally liable on the collectorrsquos claim a valid claim against the partnership that arose because of Benrsquos wrongful act in the ordinary course of the partnership business

When the collectorrsquos claim arose Empire was a general partnership composed of Adam and Ben Under UPA (1997) sect 306(a) partners of a general partnership are liable jointly and severally for all obligations of the partnership Under UPA (1997) sect 305(a) the partnership could become obligated for the loss caused to the collector as a result of the misrepresentation by Ben provided he was acting in the ordinary course of the partnership business Because there was no statement that limited his partnership authority Ben as partner was ldquoan agent of the partnership for the purpose of its businessrdquo See UPA (1997) sect 301(1) Benrsquos misrepresentation to the collector even if intentional appears to be in the ordinary course of the partnershiprsquos business of dealing

27

Agency and Partnership Analysis

in antique cars Thus Benrsquos wrongful act created a partnership obligation for which Adam was jointly and severally liable

[NOTE Generally a partnership creditor must ldquoexhaust the partnershiprsquos assets before levying on a judgment debtor partnerrsquos individual property where the partner is personally liable for the partnership obligationrdquo as a result of his status as a partner UPA (1997) sect 307 cmt 4 As the UPA comments explain this places Adam more in the position of guarantor than principal debtor on the partnership obligation Id cmt 4 Although an examinee might discuss this point the call focuses on whether Adam is personally liable not how the liability might be enforced]

Point Two (30) Because the collectorrsquos claim arose before Diane joined Empire Diane did not become personally liable on the claim

Diane was admitted to Empire when it was a general partnership and after the collectorrsquos claim arose While the general rule under UPA (1997) sect 306(a) is that the partners of a general partnership are liable jointly and severally for all obligations of the partnership there is a special rule for partners who are admitted during the duration of the partnership Under UPA (1997) sect 306(b) a person admitted to an existing partnership is not personally liable for any partnership obligations incurred before the personrsquos admission Because Diane was admitted to Empire after the collectorrsquos claim arose Diane is not personally liable on the claim

Dianersquos knowledge of the pre-existing claim and her stated concern about becoming liable on the collectorrsquos claim do not change her personal nonliability to the collector Although partners who have a liability shield can assume liability to third parties through private contractual guarantees or modifications to the partnership agreement Dianersquos stated concern constituted neither a guaranty to the collector nor ldquoan intentional waiver of liability protectionsrdquo See UPA (1997) sect 306 cmt 3 (describing methods for waiver of liability protections under sect 306(c) applicable in limited liability partnerships)

At most Diane will lose her investment in the partnership as a result of the collectorrsquos claim Although Diane did not become personally liable on the collectorrsquos claim when she joined the partnership the $250000 she contributed to the partnership is ldquoat risk for the satisfaction of existing partnership debtsrdquo UPA (1997) sect 306 cmt 2

Point Three (40) Filing the statement of qualification was effective to elect limited liability partnership status Despite this new status Adam and Ben remain personally liable on the collectorrsquos claim which arose before the election But as partners in an LLP neither Adam Ben nor Diane is personally liable as a partner on the driverrsquos estatersquos claim which arose after the election

Under UPA (1997) sect 1001 a general partnership can make an election and become a limited liability partnershipmdashif the partners approve the conversion by a vote equivalent to that necessary to amend the partnership agreement and the partnership then files a statement of qualification that specifies the name of the partnership its principal office and its election to be an LLP Here the partners agreed unanimouslymdashsufficient to amend their agreement under UPA (1997) sect 401(j)mdashand the statement of qualification was filed In addition the name of Empire LLP properly included an appropriate ending ldquoLLPrdquo See UPA (1997) sect 1002

Although another way to effectuate a ldquoconversionrdquo (as suggested by Benrsquos lawyer) is to form a new LLP and transfer the assets of the old general partnership to the new LLP the

28

Agency and Partnership Analysis

method used here (approval by the partners and the filing of a statement of qualification) is also sufficient to create LLP status

Thus Empire became Empire LLP as of the date of filing of the statement of qualification See UPA (1997) sect 1001 What effect did this have on the collectorrsquos claim which predated the filing According to UPA (1997) sect 306(c) an obligation incurred while a partnership is an LLP is solely a partnership obligation As the collectorrsquos claim predated the LLP Adam and Ben remain personally liable on the collectorrsquos claim Diane on the other hand was not personally liable on the collectorrsquos claim either before or after the filing of the statement of qualification See Point Two above

The driverrsquos estatersquos claim arose after Empire became Empire LLP Under UPA (1997) sect 306(c) an obligation incurred while a partnership is an LLP is solely a partnership obligationThus Adam Ben and Diane as partners are all protected from personal liability on the driverrsquos estatersquos claim But there may be personal liability if any of them was negligent or otherwise acted wrongfully by not informing the buyer of the bad suspension that caused the accident

29

National Conference of Bar Examiners 302 South Bedford Street | Madison WI 53703-3622 Phone 608-280-8550 | Fax 608-280-8552 | TDD 608-661-1275

wwwncbexorg e-mail contactncbexorg

  • Preface
  • Description of the MEE
  • Instructions
  • February 2014 Questions
    • Constitutinal Law Question
    • Trusts and Future Interests Question
    • Secured Transactions Question
    • Federal Civil Procedure Question
    • Criminal Law and Procedure Question
    • Agency and Partnership Question
      • February 2014 Analyses
        • Constitutional Law Analysis
        • Trust and Future Interests Analysis
        • Secured Transactions Analysis
        • Federal Civil Procedure Analysis
        • Criminal Law and Procedure Analysis
        • Agency and Partnership Analysis
            • ltlt13 ASCII85EncodePages false13 AllowTransparency false13 AutoPositionEPSFiles true13 AutoRotatePages None13 Binding Left13 CalGrayProfile (Dot Gain 20)13 CalRGBProfile (sRGB IEC61966-21)13 CalCMYKProfile (US Web Coated 050SWOP051 v2)13 sRGBProfile (sRGB IEC61966-21)13 CannotEmbedFontPolicy Error13 CompatibilityLevel 1413 CompressObjects Tags13 CompressPages true13 ConvertImagesToIndexed true13 PassThroughJPEGImages true13 CreateJobTicket false13 DefaultRenderingIntent Default13 DetectBlends true13 DetectCurves 0000013 ColorConversionStrategy CMYK13 DoThumbnails false13 EmbedAllFonts true13 EmbedOpenType false13 ParseICCProfilesInComments true13 EmbedJobOptions true13 DSCReportingLevel 013 EmitDSCWarnings false13 EndPage -113 ImageMemory 104857613 LockDistillerParams false13 MaxSubsetPct 10013 Optimize true13 OPM 113 ParseDSCComments true13 ParseDSCCommentsForDocInfo true13 PreserveCopyPage true13 PreserveDICMYKValues true13 PreserveEPSInfo true13 PreserveFlatness true13 PreserveHalftoneInfo false13 PreserveOPIComments true13 PreserveOverprintSettings true13 StartPage 113 SubsetFonts true13 TransferFunctionInfo Apply13 UCRandBGInfo Preserve13 UsePrologue false13 ColorSettingsFile ()13 AlwaysEmbed [ true13 ]13 NeverEmbed [ true13 ]13 AntiAliasColorImages false13 CropColorImages true13 ColorImageMinResolution 30013 ColorImageMinResolutionPolicy OK13 DownsampleColorImages true13 ColorImageDownsampleType Bicubic13 ColorImageResolution 30013 ColorImageDepth -113 ColorImageMinDownsampleDepth 113 ColorImageDownsampleThreshold 15000013 EncodeColorImages true13 ColorImageFilter DCTEncode13 AutoFilterColorImages true13 ColorImageAutoFilterStrategy JPEG13 ColorACSImageDict ltlt13 QFactor 01513 HSamples [1 1 1 1] VSamples [1 1 1 1]13 gtgt13 ColorImageDict ltlt13 QFactor 01513 HSamples [1 1 1 1] VSamples [1 1 1 1]13 gtgt13 JPEG2000ColorACSImageDict ltlt13 TileWidth 25613 TileHeight 25613 Quality 3013 gtgt13 JPEG2000ColorImageDict ltlt13 TileWidth 25613 TileHeight 25613 Quality 3013 gtgt13 AntiAliasGrayImages false13 CropGrayImages true13 GrayImageMinResolution 30013 GrayImageMinResolutionPolicy OK13 DownsampleGrayImages true13 GrayImageDownsampleType Bicubic13 GrayImageResolution 30013 GrayImageDepth -113 GrayImageMinDownsampleDepth 213 GrayImageDownsampleThreshold 15000013 EncodeGrayImages true13 GrayImageFilter DCTEncode13 AutoFilterGrayImages true13 GrayImageAutoFilterStrategy JPEG13 GrayACSImageDict ltlt13 QFactor 01513 HSamples [1 1 1 1] VSamples [1 1 1 1]13 gtgt13 GrayImageDict ltlt13 QFactor 01513 HSamples [1 1 1 1] VSamples [1 1 1 1]13 gtgt13 JPEG2000GrayACSImageDict ltlt13 TileWidth 25613 TileHeight 25613 Quality 3013 gtgt13 JPEG2000GrayImageDict ltlt13 TileWidth 25613 TileHeight 25613 Quality 3013 gtgt13 AntiAliasMonoImages false13 CropMonoImages true13 MonoImageMinResolution 120013 MonoImageMinResolutionPolicy OK13 DownsampleMonoImages true13 MonoImageDownsampleType Bicubic13 MonoImageResolution 120013 MonoImageDepth -113 MonoImageDownsampleThreshold 15000013 EncodeMonoImages true13 MonoImageFilter CCITTFaxEncode13 MonoImageDict ltlt13 K -113 gtgt13 AllowPSXObjects false13 CheckCompliance [13 None13 ]13 PDFX1aCheck false13 PDFX3Check false13 PDFXCompliantPDFOnly false13 PDFXNoTrimBoxError true13 PDFXTrimBoxToMediaBoxOffset [13 00000013 00000013 00000013 00000013 ]13 PDFXSetBleedBoxToMediaBox true13 PDFXBleedBoxToTrimBoxOffset [13 00000013 00000013 00000013 00000013 ]13 PDFXOutputIntentProfile ()13 PDFXOutputConditionIdentifier ()13 PDFXOutputCondition ()13 PDFXRegistryName ()13 PDFXTrapped False1313 CreateJDFFile false13 Description ltlt13 ARA 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 BGR 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 CHS ltFEFF4f7f75288fd94e9b8bbe5b9a521b5efa7684002000410064006f006200650020005000440046002065876863900275284e8e9ad88d2891cf76845370524d53705237300260a853ef4ee54f7f75280020004100630072006f0062006100740020548c002000410064006f00620065002000520065006100640065007200200035002e003000204ee553ca66f49ad87248672c676562535f00521b5efa768400200050004400460020658768633002gt13 CHT ltFEFF4f7f752890194e9b8a2d7f6e5efa7acb7684002000410064006f006200650020005000440046002065874ef69069752865bc9ad854c18cea76845370524d5370523786557406300260a853ef4ee54f7f75280020004100630072006f0062006100740020548c002000410064006f00620065002000520065006100640065007200200035002e003000204ee553ca66f49ad87248672c4f86958b555f5df25efa7acb76840020005000440046002065874ef63002gt13 CZE 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 DAN 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 DEU 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 ESP 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 ETI ltFEFF004b00610073007500740061006700650020006e0065006900640020007300e4007400740065006900640020006b00760061006c006900740065006500740073006500200074007200fc006b006900650065006c007300650020007000720069006e00740069006d0069007300650020006a0061006f006b007300200073006f00620069006c0069006b0065002000410064006f006200650020005000440046002d0064006f006b0075006d0065006e00740069006400650020006c006f006f006d006900730065006b0073002e00200020004c006f006f0064007500640020005000440046002d0064006f006b0075006d0065006e00740065002000730061006100740065002000610076006100640061002000700072006f006700720061006d006d006900640065006700610020004100630072006f0062006100740020006e0069006e0067002000410064006f00620065002000520065006100640065007200200035002e00300020006a00610020007500750065006d006100740065002000760065007200730069006f006f006e00690064006500670061002e000d000agt13 FRA 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 GRE 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 HEB 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 HRV (Za stvaranje Adobe PDF dokumenata najpogodnijih za visokokvalitetni ispis prije tiskanja koristite ove postavke Stvoreni PDF dokumenti mogu se otvoriti Acrobat i Adobe Reader 50 i kasnijim verzijama)13 HUN 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 ITA ltFEFF005500740069006c0069007a007a006100720065002000710075006500730074006500200069006d0070006f007300740061007a0069006f006e00690020007000650072002000630072006500610072006500200064006f00630075006d0065006e00740069002000410064006f00620065002000500044004600200070006900f900200061006400610074007400690020006100200075006e00610020007000720065007300740061006d0070006100200064006900200061006c007400610020007100750061006c0069007400e0002e0020004900200064006f00630075006d0065006e007400690020005000440046002000630072006500610074006900200070006f00730073006f006e006f0020006500730073006500720065002000610070006500720074006900200063006f006e0020004100630072006f00620061007400200065002000410064006f00620065002000520065006100640065007200200035002e003000200065002000760065007200730069006f006e006900200073007500630063006500730073006900760065002egt13 JPN ltFEFF9ad854c18cea306a30d730ea30d730ec30b951fa529b7528002000410064006f0062006500200050004400460020658766f8306e4f5c6210306b4f7f75283057307e305930023053306e8a2d5b9a30674f5c62103055308c305f0020005000440046002030d530a130a430eb306f3001004100630072006f0062006100740020304a30883073002000410064006f00620065002000520065006100640065007200200035002e003000204ee5964d3067958b304f30533068304c3067304d307e305930023053306e8a2d5b9a306b306f30d530a930f330c8306e57cb30818fbc307f304c5fc59808306730593002gt13 KOR ltFEFFc7740020c124c815c7440020c0acc6a9d558c5ec0020ace0d488c9c80020c2dcd5d80020c778c1c4c5d00020ac00c7a50020c801d569d55c002000410064006f0062006500200050004400460020bb38c11cb97c0020c791c131d569b2c8b2e4002e0020c774b807ac8c0020c791c131b41c00200050004400460020bb38c11cb2940020004100630072006f0062006100740020bc0f002000410064006f00620065002000520065006100640065007200200035002e00300020c774c0c1c5d0c11c0020c5f40020c2180020c788c2b5b2c8b2e4002egt13 LTH 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 LVI 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 NLD (Gebruik deze instellingen om Adobe PDF-documenten te maken die zijn geoptimaliseerd voor prepress-afdrukken van hoge kwaliteit De gemaakte PDF-documenten kunnen worden geopend met Acrobat en Adobe Reader 50 en hoger)13 NOR 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 POL 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 PTB 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 RUM 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 RUS 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 SKY ltFEFF0054006900650074006f0020006e006100730074006100760065006e0069006100200070006f0075017e0069007400650020006e00610020007600790074007600e100720061006e0069006500200064006f006b0075006d0065006e0074006f0076002000410064006f006200650020005000440046002c0020006b0074006f007200e90020007300610020006e0061006a006c0065007001610069006500200068006f0064006900610020006e00610020006b00760061006c00690074006e00fa00200074006c0061010d00200061002000700072006500700072006500730073002e00200056007900740076006f00720065006e00e900200064006f006b0075006d0065006e007400790020005000440046002000620075006400650020006d006f017e006e00e90020006f00740076006f00720069016500200076002000700072006f006700720061006d006f006300680020004100630072006f00620061007400200061002000410064006f00620065002000520065006100640065007200200035002e0030002000610020006e006f0076016100ed00630068002egt13 SLV 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 SUO 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 SVE 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 TUR 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 UKR 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 ENU (Use these settings to create Adobe PDF documents best suited for high-quality prepress printing Created PDF documents can be opened with Acrobat and Adobe Reader 50 and later)13 gtgt13 Namespace [13 (Adobe)13 (Common)13 (10)13 ]13 OtherNamespaces [13 ltlt13 AsReaderSpreads false13 CropImagesToFrames true13 ErrorControl WarnAndContinue13 FlattenerIgnoreSpreadOverrides false13 IncludeGuidesGrids false13 IncludeNonPrinting false13 IncludeSlug false13 Namespace [13 (Adobe)13 (InDesign)13 (40)13 ]13 OmitPlacedBitmaps false13 OmitPlacedEPS false13 OmitPlacedPDF false13 SimulateOverprint Legacy13 gtgt13 ltlt13 AddBleedMarks false13 AddColorBars false13 AddCropMarks false13 AddPageInfo false13 AddRegMarks false13 ConvertColors ConvertToCMYK13 DestinationProfileName ()13 DestinationProfileSelector DocumentCMYK13 Downsample16BitImages true13 FlattenerPreset ltlt13 PresetSelector MediumResolution13 gtgt13 FormElements false13 GenerateStructure false13 IncludeBookmarks false13 IncludeHyperlinks false13 IncludeInteractive false13 IncludeLayers false13 IncludeProfiles false13 MultimediaHandling UseObjectSettings13 Namespace [13 (Adobe)13 (CreativeSuite)13 (20)13 ]13 PDFXOutputIntentProfileSelector DocumentCMYK13 PreserveEditing true13 UntaggedCMYKHandling LeaveUntagged13 UntaggedRGBHandling UseDocumentProfile13 UseDocumentBleed false13 gtgt13 ]13gtgt setdistillerparams13ltlt13 HWResolution [2400 2400]13 PageSize [612000 792000]13gtgt setpagedevice13

Page 13: February 2014 MEE Questions and AnalysesPreface The Multistate Essay Examination (MEE) is developed by the National Conference of Bar Examiners (NCBE). This publication includes the

February 2014 MEE

ANALYSES Constitutional Law

Trusts and Future Interests Secured Transactions

Federal Civil Procedure Criminal Law and Procedure

Agency and Partnership

CONSTITUTIONAL LAW ANALYSIS (Constitutional Law IVD)

ANALYSIS

Legal Problems

(1) Is the city ordinance requirement that businesses install floodlights a taking

(2) Is conditioning the approval of a building permit on the grant of an easement to install surveillance equipment a taking of property

DISCUSSION

Summary

The ordinance requiring businesses to install floodlights is not a per se taking under Loretto because it does not force a private landowner to allow a third party to enter and place a physical object on the land Here the city ordinance requires the businessmdashnot a third partymdashto install the floodlights

The ordinance is likely not a regulatory taking under the Penn Central balancing test While the ordinance will impose a cost on business owners that cost may be offset by the expected increase in business due to the ordinance and the ordinance does not appear to interfere with the ownerrsquos primary use of the property as a restaurant

The permit condition however is likely an uncompensated taking of property While the condition has an essential nexus with the cityrsquos legitimate interest in promoting public safety the city has not made an individualized determination that the easement condition is roughly proportional to the possibility of increased crime due to the restaurantrsquos proposed addition Thus the permit condition likely violates the Fifth Amendment as applied to the states through the Fourteenth Amendment

Point One (50) The ordinance requiring that businesses install floodlights is not a per se taking under Loretto It is not a regulatory taking under the Penn Central balancing test because the cost of compliance with the ordinance may be offset by an expected increase in business and compliance does not interfere with the businessrsquos primary use of its property as a restaurant

The city ordinance requiring a business to install floodlights does not effect a per se taking of the sort described in Loretto v Teleprompter Manhattan CATV Corp 458 US 419 (1982) because no property is physically taken by the government and the ordinance does not involve a physical invasion of private property by a third party

Even though the ordinance does not constitute an occupation of the property by either the government or a third party it is still subject to the three-factor balancing test under Penn Central Transportation Co v City of New York 438 US 104 (1978) to determine whether it is a ldquoregulatory takingrdquo Under Penn Central a court must balance (1) ldquo[t]he economic impact of the regulation on the claimantrdquo (2) ldquothe extent to which the regulation has interfered with distinct investment-backed expectationsrdquo and (3) ldquothe character of the governmental actionrdquo Id at 124 Here each factor weighs against finding that the ordinance is a taking

11

Constitutional Law Analysis

First the ordinance requirement likely has a minimal economic impact on the restaurant Compliance with the ordinance is estimated to cost $1000 and the city has found that businesses will likely recoup that cost in increased sales Also because the ordinance does not interfere with the operation of the restaurant the owner may still earn a reasonable return on its investment in the property

Second the ordinance does not interfere with the businessrsquos investment-backed expectations As in Penn Central the challenged law does not interfere with the ownerrsquos ldquoprimary expectationrdquo for use of the propertymdashin Penn Central as a railroad terminal and here as a restaurant Further the ordinance does not prevent the restaurant from expanding to meet the changing business environment

Third the character of the government action does not weigh in favor of a taking While Penn Central does say that a ldquophysical invasionrdquo is more likely to pose a taking Loretto suggests that the Courtrsquos main concern is with physical invasions by third parties Also like the landmark law challenged in Penn Central the ordinance here ldquoadjust[s] the benefits and burdens of economic life to promote the common goodrdquo Id In Penn Central the landmark law restricted development of the railroad terminal to promote the common interest in preserving historic landmarks Here the ordinance requires the businesses to install floodlights to promote the common interest in crime prevention and public safety

Because the ordinance is clearly a valid exercise of the police power it satisfies the takings clausersquos public-use requirement Kelo v City of New London 545 US 469 (2005)

In sum all three factors weigh against finding a taking under the Penn Central balancing test

Point Two (50) The permit condition may be unconstitutional as an uncompensated taking of property because the city has not made an individualized determination that the easement condition is roughly proportional to the impact of the restaurantrsquos proposed addition

In Dolan v City of Tigard 512 US 374 (1994) the Supreme Court set forth the test for determining whether an exaction imposed by a government in exchange for a discretionary benefit conferred by the government such as a condition on the approval of a building permit in this case constitutes an uncompensated taking under the Fifth Amendment The exaction is not a taking if (1) there is an ldquoessential nexusrdquo between the ldquopublic need or burdenrdquo to which the proposed development contributes and ldquothe permit condition exacted by the cityrdquo id at 386 and (2) the government makes ldquosome sort of individualized determination that the required dedication is [roughly proportional] both in nature and extent to the impact of the proposed developmentrdquo Id at 391 see also Nollan v California Coastal Commission 483 US 825 (1987)

Here the city likely can meet the nexus requirement In Dolan the landowner sought to double the size of its business which would have increased traffic on nearby roadways In exchange for approving the development the city sought an easement for a bike and pedestrian path The Court found the required nexus between the easement and the cityrsquos ldquoattempt to reduce traffic congestion by providing for alternative means of transportationrdquo 512 US at 387 Here a similar nexus likely exists between the requested easement and the cityrsquos interest in crime prevention and public safety Increased patronage and economic activity at the restaurant might attract additional crime to the area and the requested easement to install surveillance equipment would attempt to address that increased crime

12

Constitutional Law Analysis

The exaction here however may fail the second prong of the Dolan testmdashthat the exaction be roughly proportional to the anticipated impact of the requested development As noted the city in Dolan claimed that a bike and pedestrian path was needed to offset the increase in traffic due to the proposed doubling of the business The Court explained that the government must demonstrate that the additional traffic reasonably was related to the requested exaction and that the government must ldquomake some effort to quantify its findings in support of the dedication for the pedestrianbicycle pathway beyond the conclusory statement that it could offset some of the traffic demand generatedrdquo Id at 395 Here the city did not carry its burden The city simply speculates that increased patronage of the restaurant ldquomightrdquo increase crime and that the surveillance equipment ldquomightrdquo alleviate this increased crime Because the city has not made ldquosome effort to quantify its findingsrdquo in support of the easement it has not shown that the burden of the easement is roughly proportional to the benefits thought to flow from it

Thus the exaction appears to be an uncompensated taking of property in violation of the Fifth Amendment as applied to the states through the Fourteenth Amendment

13

TRUSTS AND FUTURE INTERESTS ANALYSIS ____ (Trusts and Future Interests IE3 I5 IIIA amp B)

ANALYSIS

Legal Problems

(1) How should rents dividends and sales proceeds received by the trustee prior to receipt of the sonrsquos letter have been allocated between trust income and principal

(2)(a) Did the remainder interest in the trust accelerate and become immediately payable to the daughterrsquos minor child upon the trusteersquos receipt of the sonrsquos letter and if not how should the trustee handle the distribution of the principal in the future

(2)(b) Following the trusteersquos receipt of the sonrsquos letter how should the trustee distribute future receipts of income prior to the distribution of the principal

DISCUSSION

Summary

Prior to the trusteersquos receipt of the sonrsquos letter cash dividends and rents should have been allocated to trust income and were distributable to the son the income beneficiary of the trust sales proceeds and stock dividends should have been allocated to principal

Because the sonrsquos letter to the trustee did not result in a valid disclaimer under state law (having been made more than nine months after the testatorrsquos death) the son is not deemed to have predeceased the testator Because the son is still living the class gift to the testatorrsquos grandchildren who survive the son has not closed and is not possessory it will not become possessory until the son dies The daughterrsquos minor child being the testatorrsquos only living grandchild is not currently entitled to a distribution of trust principal Trust principal will instead be distributable upon the sonrsquos death to the testatorrsquos then-living grandchildren or if there are none to the testatorrsquos then-living heirs

As for future income the trustee should either distribute the trust income to the son and the daughter as the testatorrsquos heirs accumulate the income for future distribution to those individuals ultimately entitled to the trust principal or distribute it to those presumptively entitled to the principal upon the sonrsquos death ie the daughterrsquos minor child

Point One (45) Cash dividends and rents are allocable to income sales proceeds and stock dividends are allocable to principal Items allocable to income for the period prior to the sonrsquos attempted disclaimer were distributable to the son

Receipts earned during the administration of a trust are allocable either to income or to principal Almost all states have adopted the most recent or an earlier version of the Uniform Principal and Income Act (the Act) which specifies how such receipts should be allocated

Under the Act rents (UNIF PRIN amp INC ACT (2000) sect 405 UNIF PRIN amp INC ACT (1962) sect 3(a)(1)) and cash dividends received from a corporation (UNIF PRIN amp INC ACT (2000) sect 401(b) UNIF PRIN amp INC ACT (1962) sect 6(d)) are allocable to income and are distributable to the income beneficiary of the trust

14

Trusts and Future Interests Analysis Sales proceeds (UNIF PRIN amp INC ACT (2000) sect 404(2) UNIF PRIN amp INC ACT (1962)

sect 3(b)(1)) and dividends paid in the stock of the distributing corporation (UNIF PRIN amp INC ACT (2000) sect 401(c)(1) UNIF PRIN amp INC ACT (1962) sect 3(b)(4)) are allocable to principal and added to the principal of the trust

Here the cash dividends and office building rents should have been allocated to income and until the trustee received the sonrsquos letter should have been distributed to him as the sole income beneficiary of the trust The stock dividend and proceeds from the sale of the office building should have been allocated to principal and held by the trustee for future distribution to the ultimate remaindermen of the trust

[NOTE The 2000 Uniform Principal and Income Act has been adopted in Alabama Arkansas Colorado Connecticut the District of Columbia Hawaii Idaho Iowa Kentucky Missouri Montana Nebraska New Mexico North Dakota Oregon South Dakota Utah and West Virginia]

Point Two(a) (45) Because the son did not disclaim within nine months of the testatorrsquos death there is no valid disclaimer under state law Therefore the son is not deemed to have predeceased the testator Furthermore because of the express survivorship contingency in the will the remainder in the trust does not accelerate and become distributable until the son in fact dies When the son dies the trust principal will be distributable to the testatorrsquos then-living grandchildren or if none then to the testatorrsquos then-living heirs

When a trust remainder is given to a class the class closes (ie no new persons can join the class) when there is no outstanding income interest and at least one member of the class is then entitled to demand possession of his or her share of the remainder This principle is called the rule of convenience See generally HERBERT HOVENKAMP amp SHELDON F KURTZ PRINCIPLES OF PROPERTY LAW 199ndash200 (6th ed 2005) A class member may demand possession of his or her share of the remainder upon termination of the income interest only when the class memberrsquos interest is not otherwise subject to a condition precedent See id

When a beneficiary timely disclaims an interest in a trust that beneficiary is treated as if he had predeceased the testator Here had the son disclaimed within nine months of the testatorrsquos death as required by the state statute he would have been deemed to have predeceased the testator This would have closed the class of remaindermen and the testatorrsquos then-living grandchildren (ie the daughterrsquos child) would have been entitled to the trust principal However under the state statute the sonrsquos disclaimer was not timely because he did not disclaim within nine months of the testatorrsquos death Thus because the statute is inapplicable and the son is still alive the class of grandchildren entitled to share in trust principal did not close

Because here the statute is inapplicable due to the sonrsquos failure to comply with the statutory time requirements then presumably the common-law rule allowing disclaimers (aka renunciations) at any time should apply Under the common law if a life estate is renounced the remainder interest accelerates and becomes immediately distributable to the remaindermen of the trust if the remainder is vested but not if the remainder is contingent JESSE DUKEMINIER amp ROBERT H SITKOFF WILLS TRUSTS AND ESTATES 844ndash845 (9th ed 2013) Here because the remainder is contingent upon there being grandchildren who survive the son the remainder will not accelerate It will remain open until the son dies leaving open the possibility that additional grandchildren will be included in the class or the daughterrsquos child could fall out of the class because that child fails to survive the son

And if none of the testatorrsquos grandchildren survive the son the trust principal will be distributed to the testatorrsquos heirs living at the sonrsquos death

15

Trusts and Future Interests Analysis

Point Two(b) (10) Until the trust terminates the trustee must continue to hold the trust assets The distribution of income in the meantime is unclear There are at least three possibilities Income earned on the undistributed assets could be distributed to the son and daughter as the testatorrsquos heirs accumulated and added to principal for distribution to the ultimate remaindermen or distributed from time to time to those persons who are presumptively remaindermen

When trust principal is not immediately distributable the trustee must continue to hold trust assets until the ultimate remaindermen are ascertained During this period trust income will be distributed or retained according to any instructions contained in the trust instrument See WILLIAM M MCGOVERN JR SHELDON F KURTZ amp DAVID M ENGLISH WILLS TRUSTS amp ESTATES sect 102 (4th ed 2010)

Here the testator did not specify what the trustee should do with trust income in the event the sonrsquos disclaimer did not comply with the state statute There are at least three approaches One approach would have the trustee distribute the trust income to the testatorrsquos heirs on the theory that the income represents property that was not disposed of by the testatorrsquos will and which thus passes by partial intestacy to the testatorrsquos heirs A second approach would have the trustee accumulate trust income for distribution to the ultimate remaindermen Under this approach only those individuals ultimately entitled to the principal would share in the income A third approach would have the trustee distribute trust income to those individuals who would be the remaindermen if the trust were to terminate when the income is received by the trustee under this approach trust income would be distributed to the daughterrsquos minor child until another presumptive remainderman is born This approach could result in individuals not ultimately entitled to principal say because they do not survive the son receiving income It could also result in a disproportionate distribution of income among the individuals ultimately entitled to income

[NOTE Examinees should demonstrate a recognition and understanding of the income-allocation problem and the alternatives available to address that issue There is no widely accepted solution to the problem Examinees who cite any of these possible problem-solving approaches may receive credit]

16

SECURED TRANSACTIONS ANALYSIS (Secured Transactions IB IID E amp F IIIB IVA B amp F)

ANALYSIS

Legal Problems

(1)(a) What is the nature of the bankrsquos claim to the businessrsquos equipment

(1)(b) What is the nature of the finance companyrsquos claim to the businessrsquos equipment

(1)(c) As between the bank and the finance company whose claim to the businessrsquos equipment has priority

(2) Do the claims of the bank and the finance company continue in the item of equipment sold by the business to the competitor

DISCUSSION

Summary

The bank and the finance company both have perfected security interests in the businessrsquos equipment Even though the finance companyrsquos perfected security interest was created first the bankrsquos perfected security interest has priority because the bankrsquos financing statement was filed before the finance companyrsquos financing statement The security interests of the bank and the finance company continue in the item of equipment sold by the business to the competitor because their security interests were perfected and the competitor was not a buyer in ordinary course of business

Point One(a) (25) The bank has a perfected security interest in the businessrsquos equipment

The bank has met all criteria necessary for it to have an attached and enforceable security interest in the businessrsquos equipment First value must be given UCC sect 9-203(b)(1) This criterion is fulfilled by the loan made by the bank to the business Second the debtor must have rights in the collateral UCC sect 9-203(b)(2) Clearly the business has rights in its equipment Third either the secured party must take possession of the collateral or the debtor must authenticate a security agreement containing a description of the collateral UCC sect 9-203(b)(3) The agreement that the business owner signed is a ldquosecurity agreementrdquo because it is an agreement that creates or provides for a security interest UCC sect 9-102(a)(74) By signing the security agreement the business owner authenticated it UCC sect 9-102(a)(7) Therefore all three criteria are fulfilled and the bank has an enforceable and attached security interest

A security interest is perfected when it has attached and when any additional steps required for perfection have occurred UCC sect 9-308(a) Generally speaking the additional steps will either be possession of the collateral by the secured party or the filing of a financing statement with respect to the collateral See UCC sectsect 9-310 9-313 In this case the bank filed a financing statement naming the debtor and sufficiently indicating the collateral The collateral indication is sufficient because it identifies the collateral by type of property See UCC sectsect 9-504 9-108 The fact that the financing statement was filed before the security interest was created is

17

Secured Transactions Analysis

not a problem Even though the security agreement had not yet been signed the business had authorized the filing of the financing statement in an authenticated record UCC sect 9-509(a)(1) Moreover the financing statement may be filed before the security agreement is created UCC sect 9-502(d)

Point One(b) (10) The finance company also has a perfected security interest in the businessrsquos equipment

The finance companyrsquos security interest is enforceable and attached for the same reasons as the bankrsquos security interest The loan from the finance company to the business constitutes value the business has rights in the collateral and the business owner has authenticated a security agreement containing a description of the collateral The finance companyrsquos security interest is perfected because the finance company filed a financing statement with respect to it that provides that the business is the debtor and indicates that the collateral is equipment

Point One(c) (30) The bankrsquos security interest has priority over the finance companyrsquos security interest because the bankrsquos financing statement was filed first

As between two perfected security interests the general rule is that the security interest that was the earlier to be either perfected or the subject of a filed financing statement has priority UCC sect 9-322(a)(1) While the finance companyrsquos security interest was perfected before the bankrsquos (March 15 vs March 22) the bankrsquos financing statement was filed even earlier on March 2 Thus under the first-to-file-or-perfect rule of UCC sect 9-322(a)(1) the bankrsquos security interest has priority No exceptions to the general rule apply here

Point Two (35) A security interest in collateral continues notwithstanding its sale unless an exception applies Because the security interests of the bank and the finance company were perfected and the competitor was not a buyer in ordinary course of business no exception applies and the security interests of both creditors continue in the equipment sold to the competitor

As a general rule a security interest in collateral continues notwithstanding the fact that the debtor has sold the collateral to another person UCC sect 9-315(a)(1) Thus unless an exception applies the security interests of the bank and the finance company will continue in the item of equipment sold to the competitor

A buyer of goods will take free of an unperfected security interest in those goods See UCC sect 9-317(a)(2) However when the competitor bought the businessrsquos equipment both the bank and the finance company had perfected security interests in the equipment

A buyer can take free even of a perfected security interest in goods if the buyer is a ldquobuyer in ordinary course of businessrdquo See UCC sect 9-320(a) However the competitor was not a buyer in ordinary course of business To be a ldquobuyer in ordinary course of businessrdquo a buyer must buy goods from a seller that is in the business of selling goods of that kind See UCC sect 1-201(b)(9) The competitor bought this equipment from a seller that is not in the business of selling goods of this kind so the competitor was not a buyer in ordinary course of business with respect to these goods

Because no exception applies the security interests of the bank and the finance company continue even after the item of equipment was sold to the competitor

18

FEDERAL CIVIL PROCEDURE ANALYSIS (Federal Civil Procedure IVD)

ANALYSIS

Legal Problems

(1) Is a document prepared in the course of a contract dispute protected from discovery as ldquowork productrdquo when there is no evidence that the document was prepared in anticipation of litigation

(2)(a) Is a partyrsquos failure to provide relevant electronically stored information excused when the information was destroyed pursuant to a routine document retention scheme at a time when litigation was contemplated by the destroying party

(2)(b) What sanctions should be imposed on a party for allowing the destruction of evidence that is relevant to potential future litigation

DISCUSSION

Summary

The report prepared by the structural engineer is probably not work product and is thus discoverable The engineer examined the foundation of the house at the customerrsquos request and the engineerrsquos findings are potentially relevant to the customerrsquos claim that the foundation is defective The report was not prepared in anticipation of litigation The customer appears to have sought the engineerrsquos opinion in response to the builderrsquos offer to fix any problems with the foundation that an engineer might identify Because the report was not prepared in anticipation of litigation it is not protected by the work-product doctrine

The builder should have taken appropriate steps to preserve evidence including suspending its document retention program as soon as it began planning for litigationmdashie on July 10 Its destruction of potentially relevant material after that date was wrongful However a court is unlikely to impose severe sanctions on the builder because there are no facts indicating that the builder acted in bad faith and the customer can prove that the foundation is defective without the destroyed emails

Point One (40) The customer must turn over the engineerrsquos report because it was not prepared in anticipation of litigation

In general a party to a lawsuit in federal court ldquomay obtain discovery regarding any nonprivileged matter that is relevant to any partyrsquos claim or defenserdquo FED R CIV P 26(b)(1) (2009) This includes the right to inspect and copy documents in the other partyrsquos possession FED R CIV P 34(a)(1) Here the customer hired a structural engineer to examine the foundation of the house The engineerrsquos report on the foundation is likely to include information that would be relevant to the customerrsquos claim that the foundation was defectively constructed

The so-called ldquowork productrdquo rule allows a party to refuse to turn over ldquodocuments that are prepared in anticipation of litigation or for trialrdquo by that partyrsquos representative including

19

Federal Civil Procedure Analysis

a consultant Thus if the customer had hired the structural engineer to prepare a report ldquoin anticipation of litigationrdquo that report might not be discoverable See FED R CIV P 26(b)(3)

In this case however the customer hired the engineer to evaluate the foundation of the house as part of the customerrsquos negotiation with the builder concerning the housersquos flooding problem The builder told the customer that the housersquos landscaping was the reason for the flooding and the builder told the customer ldquoHave an engineer look at the foundation If therersquos a problem wersquoll fix itrdquo The customer appears to have acted in response to that statement There is no indication that the customer anticipated any kind of legal action at the time that the structural engineer was hired Accordingly the structural engineerrsquos report is discoverable and the court should order the customer to turn it over

[NOTE If an examinee concludes that the structural engineerrsquos report was prepared in anticipation of litigation then the examinee should also conclude that the report is not discoverable Documents prepared in anticipation of litigation do not need to be disclosed to an adverse party unless that party can demonstrate a ldquosubstantial needrdquo for the documents and an inability to obtain substantially equivalent information without ldquoundue hardshiprdquo FED R CIV P 26(b)(3)(A)(ii) Furthermore a report prepared by an expert who is not expected to testify is not discoverable in the absence of ldquoexceptional circumstancesrdquo making it ldquoimpracticablerdquo to obtain the information in another way FED R CIV P 26(b)(4)(D)(ii) The builder probably cannot make these showings here unless the engineerrsquos report deals with circumstances that have since changed There is no evidence that the structural engineer would have had access to any information or facts that the builder would not already know as a result of its construction and subsequent inspection of the house In addition if necessary the builder could ask the court for permission to arrange for a further inspection of the house by a structural engineer hired by the builder See FED R CIV P 34(a)(2) Accordingly if an examinee concludes that the report was prepared in anticipation of litigation the examinee should also conclude that the builder is not entitled to see the report]

Point Two(a) (30) Because the builder anticipated that it might be involved in litigation concerning its contract with the customer the builder acted wrongfully in destroying emails that were relevant to the housersquos construction even though the emails were destroyed pursuant to a routine document retention plan

As noted above a party to a lawsuit in federal court ldquomay obtain discovery regarding any nonprivileged matter that is relevant to any partyrsquos claim or defenserdquo FED R CIV P 26(b)(1) This includes emails and other electronically stored information FED R CIV P 34(a)(1)(A) Here the customer has requested all the builderrsquos emails pertaining to work done on the foundation of the house Ordinarily the builder would be obliged to turn over this information which is relevant to the customerrsquos defense that the housersquos foundation was poorly constructed

Unfortunately the emails in question no longer exist because the builder destroyed them on August 2

In general spoliation of evidence (destruction or alteration of evidence) is improper if the party who destroyed or altered the evidence ldquohas notice that the evidence is relevant to litigation or should have known that the evidence may be relevant to future litigationrdquo Fujitsu Ltd v Federal Express Corp 247 F3d 423 436 (2d Cir 2001) It is improper for a party to destroy electronic information relevant to pending litigation even if the destruction occurs before there is any request or order seeking the information See eg Leon v IDX Sys Corp 464 F3d 951 (9th Cir 2006) (plaintiffrsquos intentional destruction of computer files warranted dismissal even

20

In this case the builderrsquos destruction of the emails was pursuant to a routine document retention plan The Federal Rules provide expressly that in the absence of ldquoexceptional circumstancesrdquo parties should not be sanctioned for the loss of electronically stored information when the loss occurs pursuant to ldquoroutine good-faith operation of an electronic information systemrdquo FED R CIV P 37(e) However when a party anticipates litigation ldquoit must suspend its routine document retentiondestruction policy and put in place a lsquolitigation holdrsquo to ensure the preservation of relevant documentsrdquo Zubulake v UBS Warburg LLC 220 FRD 212 218 (SDNY 2003)

Federal Civil Procedure Analysis

though spoliation occurred before order compelling discovery) Similarly the duty to preserve evidence applies to a party who anticipates litigation even if litigation has not yet been commenced See THE SEDONA PRINCIPLES BEST PRACTICES RECOMMENDATIONS amp PRINCIPLES FOR ADDRESSING ELECTRONIC DOCUMENT PRODUCTION 70 cmt 14a (2d ed 2007)

The builder destroyed the emails on August 2 At that time the builder knew that litigation was a possibility because the builder had already directed its attorney to prepare a draft complaint for possible filing Knowing that litigation was a possibility the builder had a duty to take steps to preserve evidence including the emails in question See generally Fujitsu Ltd

Thus the builderrsquos destruction of potentially relevant emails at a time when it knew that litigation was a possibility was improper It had a duty to preserve evidence and it breached that duty

[NOTE Because courts have used different words to describe the test for when evidence must be preserved an examineersquos precise formulation of the test is not critical]

Point Two(b) (30) In determining appropriate sanctions for spoliation courts consider both the level of culpability of the spoliating party and the degree of prejudice the loss of evidence has caused the other party Here the builderrsquos destruction of evidence does not appear to have been willful nor is it likely to pose a significant obstacle to the customerrsquos defense Any sanctions imposed by the court should be modest

Federal courts have inherent power to control the litigation process and can sanction misbehavior including spoliation even when there has been no specific violation of the Federal Rules of Civil Procedure See generally Chambers v NASCO Inc 501 US 32 (1991) (discussing courtrsquos inherent power to control the litigation process) The range of available sanctions is broad It can include such sanctions as the payment of expenses incurred by the other party as a result of the destruction of the evidence an instruction to the jury authorizing it to draw an adverse inference from the destruction of the evidence a shifting of the burden of proof on the relevant issue or even judgment against the responsible party See eg Residential Funding Corp v DeGeorge Financial Corp 306 F3d 99 108 (2d Cir 2002) (adverse inference) Silvestri v General Motors Corp 271 F3d 583 593 (4th Cir 2001) (possibility of dismissal) Cf FED R CIV P 37(b)(2)(A) (listing remedies for failure to comply with discovery obligations)

In determining appropriate sanctions for spoliation courts consider both the level of culpability of the spoliating party and the degree of prejudice the loss of evidence has caused the other party Many courts impose severe sanctions (such as an adverse-inference instruction or the entry of judgment against the spoliating party) only when there is evidence of bad faith in the form of an intentional effort to hide information Eg Greyhound Lines Inc v Wade 485 F3d 1032 1035 (8th Cir 2007) (spoliation sanction requires intentional destruction out of desire ldquoto suppress the truthrdquo) However other courts have said that negligence in preserving evidence can

21

Federal Civil Procedure Analysis

support an adverse-inference instruction See Residential Funding 306 F3d at 108 (negligence enough under some circumstances)

Although a court might well order an evidentiary hearing on the issue of sanctions the facts presented do not seem appropriate for severe sanctions First the evidence was destroyed pursuant to the builderrsquos standard document retention plan and there is no evidence that the builder deliberately failed to suspend its usual procedures with the purpose of allowing the destruction of evidence Second the loss of this evidence will not severely hinder the customerrsquos presentation of his case The central issue is whether the foundation of the house was properly constructed If the construction job was poorly done the customer can present evidence derived from inspection of the premises to prove that point The customer can also depose witnesses about any issues that arose during construction

Under the circumstances a court is not likely to impose particularly severe sanctions although it might shift the burden to the builder to show that the foundation was properly constructed or it might require the builder to reimburse any expenses the customer incurs to discover and prove the facts about issues or disputes that arose during construction of the foundation

[NOTE The result reached by the examinee is less important than the examineersquos recognition that (a) a range of sanctions is available to the court and (b) the appropriate sanction depends both on the culpability of the builder and the prejudice suffered by the customer]

22

CRIMINAL LAW AND PROCEDURE ANALYSIS (Criminal Law and Procedure IIA amp D VE amp F)

ANALYSIS

Legal Problems

(1) Did charging the defendant with both theft and burglary constitute double jeopardy

(2) Did the jury instruction violate the due process clause either by relieving the prosecution of the burden of proving the element of intent or by shifting the burden to the defendant to disprove that element

(3) Did the sentence imposed in this case for the theft conviction unconstitutionally deprive the defendant of his right to a jury trial on the issue of the value of the stolen item

DISCUSSION

Summary

The trial court properly denied the defendantrsquos pretrial motion to dismiss the charges on double jeopardy grounds The defendant may be charged with and convicted of both theft and burglary Each of the charges has an element that the other does not Neither charge is a lesser-included offense nor are they multiplicitous Thus charging both theft and burglary does not violate double jeopardy

The jury instruction on the burglary charge was constitutionally flawed It could have been reasonably understood by the jury as either (1) an irrebuttable conclusive presumption (which relieved the prosecution of proving the element of intent and removed the issue from the jury) or (2) a rebuttable mandatory presumption (which unconstitutionally shifted the burden of proof on an element of a charged offense from the prosecution to the defendant)

Because the four-year sentence imposed by the judge was based on the judgersquos finding by a preponderance of the evidence that the value of the stolen ring exceeded $5000 the sentence violates the defendantrsquos right to a jury determination beyond a reasonable doubt of the value of the ring

Point One (30) Charging the defendant with theft and burglary did not constitute double jeopardy

The Double Jeopardy Clause of the Fifth Amendment provides that a person shall not be twice put in jeopardy for the ldquosame offenserdquo Thus the question is whether the elements of the theft charge are wholly contained in the burglary charge or vice versa If the elements of the lesser charge (theft) are not wholly contained in the greater charge (burglary)mdashie if each charge requires proof of a fact that the other does notmdashthen convicting the defendant of both crimes would not violate double jeopardy even when the two offenses occurred at the same time and are thus arguably part of the ldquosame transactionrdquo Blockburger v United States 284 US 299 304 (1932) See also Albernaz v United States 450 US 333 344 n3 (1981) United States v Dixon 509 US 688 704 (1993)

23

Criminal Law and Procedure Analysis

Here theft and burglary each require proof of an element not required for the other crime Burglary may be defined differently in different jurisdictions However it almost invariably requires entry into a building or dwelling of another with the specific intent to commit a felony therein and the crime of burglary is complete upon the entry into the building or dwelling with such intent See eg Cannon v Oklahoma 827 P2d 1339 1342 (Okla Crim App 1992) In contrast theft which also may be defined differently in different states almost invariably requires the taking and carrying away of an item of personal property belonging to another with the intent to steal or permanently deprive the owner of possession

Here the ldquotakingrdquo or ldquostealingrdquo element is not contained in the definition of burglary and the ldquoentryrdquo element of burglary is not contained in the definition of theft Because theft is not a lesser-included offense of burglary and burglary is not a lesser-included offense of theft charging the defendant for both burglary and theft did not violate double jeopardy and the court properly denied the defense motion on those grounds Yparrea v Dorsey 64 F3d 577 579ndash80 (10th Cir 1995) citing Blockburger 284 US at 304

Finally the defendantrsquos motion to dismiss all the charges on double jeopardy grounds was improper because if both charges were for the same offense the motion should have requested dismissal of one charge not both

Point Two (35) The jury instruction on the burglary charge violated the Due Process Clause because it created either (1) an irrebuttable conclusive presumption (which relieved the prosecution of proving the element of intent and removed that issue from the jury) or (2) a rebuttable mandatory presumption (which unconstitutionally shifted the burden of proof on an element of a charged offense to the defendant)

The Supreme Court has interpreted the Due Process Clause of the US Constitution to require that the prosecution prove all elements of an offense beyond a reasonable doubt See In re Winship 397 US 358 364 (1970) The burden of proof cannot be shifted to the defendant by presuming an essential element upon proof of other elements of the offense because shifting the burden of persuasion with respect to any element of a criminal offense is contrary to the Due Process Clause See Mullaney v Wilbur 421 US 684 (1975)

The crime of burglary includes entry into a building or dwelling with the specific intent to commit a felony therein The requirement that the prosecutor prove beyond a reasonable doubt that the defendant had this specific intent distinguishes burglary from general-intent crimes like trespass See Sandstrom v Montana 442 US 510 523 (1979)

Here the jury was instructed that if ldquoafter consideration of all the evidence presented by the prosecution and defense you find beyond a reasonable doubt that the defendant entered the dwelling without the ownersrsquo consent you may presume that the defendant entered with the intent to commit a felony thereinrdquo This instruction was unconstitutional because it created either an irrebuttable conclusive presumption or a rebuttable mandatory presumption

A conclusive presumption is ldquoan irrebuttable direction by the court to find intent once convinced of the facts triggering the presumptionrdquo Id at 517 Here the jurors were instructed that once the prosecutor established that the defendant entered the neighborsrsquo house without consent they ldquomay presumerdquo that he intended to commit a felony therein The jurors may have reasonably concluded from this instruction that if they found that the defendant intended to enter his neighborsrsquo home without permission they must further find that he entered with the specific intent to commit a felony therein Because this instruction could operate as a conclusive

24

Criminal Law and Procedure Analysis

irrebuttable presumption by eliminating intent ldquoas an ingredient of the offenserdquo it violated due process by relieving the prosecution of the burden of proof for this element Id at 522

In the alternative the jury instruction could have been reasonably understood to create a rebuttable mandatory presumption which ldquotells [the jury] they must find the elemental fact upon proof of the basic fact at least unless the defendant has come forward with some evidence to rebut the presumed connection between the two factsrdquo County Court of Ulster County New York v Allen 442 US 140 157 (1979) The due process problem created by rebuttable mandatory presumptions is that ldquo[t]o the extent that the trier of fact is forced to abide by the presumption and may not reject it based on an independent evaluation of the particular facts presented by the State the analysis of the presumptionrsquos constitutional validity is logically divorced from those facts and based on the presumptionrsquos accuracy in the run of casesrdquo Id at 159

Unlike irrebuttable conclusive presumptions rebuttable mandatory presumptions are not always per se violations of the Due Process Clause However the Supreme Court of the United States has held that jury instructions that could reasonably be understood as shifting the burden of proof to the defendant on an element of the offense are unconstitutional Francis v Franklin 471 US 307 (1985) Here the argument that the jury instruction operated as a rebuttable mandatory presumption is supported by the fact that the judge also instructed the jury to ldquoconsider[ ] all the evidence presented by the prosecution and defenserdquo However even if the instruction created a rebuttable mandatory presumption it would be unconstitutional because it shifted the burden to the defense on an element of the offense Sandstrom 442 US at 524 Mullaney 421 US at 686

[NOTE Whether an examinee identifies the jury instruction as containing a ldquoconclusiverdquo or ldquomandatoryrdquo presumption is less important than the examineersquos analysis of the constitutional infirmities]

Point Three (35) The trial court violated the defendantrsquos Sixth Amendment right to a jury trial on an essential element of the offense when it found by a preponderance of the evidence that the ring was worth over $5000 and increased the defendantrsquos sentence based on this finding

In the statutory scheme under which the defendant was tried and convicted a Class D felony theft is defined as theft of item(s) with a value between $2500 and $10000 The jury found that the value of the diamond ring was at least $2500 and convicted the defendant of felony theft However at sentencing the trial court made a separate finding by a preponderance of the evidence that the value of the ring was greater than $5000 Following the statutersquos two-tiered sentencing scheme the judge then imposed on the defendant a sentence that was one year longer than the maximum that would otherwise have been allowed

The judgersquos sentence was unconstitutional because it violated the defendantrsquos Sixth Amendment right to a jury trial on this question The Supreme Court held in Apprendi v New Jersey 530 US 466 (2000) that ldquo[o]ther than the fact of a prior conviction any fact that increases the penalty for a crime beyond the prescribed statutory maximum must be submitted to a jury and proved beyond a reasonable doubtrdquo because ldquo[i]t is unconstitutional for a legislature to remove from the jury the assessment of facts that increase the prescribed range of penalties to which a criminal defendant is exposed [because] such facts must be established by proof beyond a reasonable doubtrdquo Id The Court reaffirmed Apprendi in Blakely v Washington 542 US 296 (2004) holding that the ldquolsquostatutory maximumrsquo for Apprendi purposes is the maximum sentence a judge may impose solely on the basis of the facts reflected in the jury verdict or admitted by the defendantrdquo Id at 303 (emphasis in original) In United States v Booker 543 US 220 (2005)

25

Criminal Law and Procedure Analysis

the Court relied on Blakely and Apprendi to conclude that protecting a defendantrsquos Sixth Amendment right to a jury trial required that ldquo[a]ny fact which is necessary to support a sentence exceeding the maximum authorized by the facts established by a plea of guilty or a jury verdict must be admitted by the defendant or proved to a jury beyond a reasonable doubtrdquo Id at 244

Thus in order to constitutionally increase a sentence above the statutory maximum of three years the jury must have found beyond a reasonable doubt that the value of the ring exceeded $5000 Here the court made the finding based on an appraisal proffered by the prosecutor only at sentencing and the judgersquos finding was by a preponderance of the evidence rather than beyond a reasonable doubt

26

AGENCY AND PARTNERSHIP ANALYSIS __________ (Agency and Partnership VA amp C VI)

ANALYSIS

Legal Problems

(1) Is a partner in a general partnership personally liable on a claim arising from misrepresentations by another partner made in the course of the partnership business

(2) Does a newly admitted partner in a general partnership become personally liable on existing claims against the partnership

(3) After the filing by a general partnership of a statement of qualification as a limited liability partnership are the partners personally liable as partners on (a) an existing claim against the general partnership and (b) a claim against the partnership that arose after the filing

DISCUSSION

Summary

Adam and Ben formed a general partnership under which they were jointly and severally liable for obligations of the partnership Thus Adam was personally liable for misrepresentations by Ben made in the ordinary course of the partnership business

Upon joining the general partnership Diane became personally liable for the obligations of the partnership arising after her admission but not for obligations pre-existing her admission such as the collectorrsquos claim

By filing a statement of qualification the three partners properly elected limited liability partnership status As partners in an LLP none of the three partners is personally liable as a partner for partnership obligations arising after the election such as the claim by the driverrsquos estate The election however does not change their personal liability on pre-existing claims that arose before the election such as the collectorrsquos claim

Point One (30) As a general partner of Empire a general partnership Adam became personally liable on the collectorrsquos claim a valid claim against the partnership that arose because of Benrsquos wrongful act in the ordinary course of the partnership business

When the collectorrsquos claim arose Empire was a general partnership composed of Adam and Ben Under UPA (1997) sect 306(a) partners of a general partnership are liable jointly and severally for all obligations of the partnership Under UPA (1997) sect 305(a) the partnership could become obligated for the loss caused to the collector as a result of the misrepresentation by Ben provided he was acting in the ordinary course of the partnership business Because there was no statement that limited his partnership authority Ben as partner was ldquoan agent of the partnership for the purpose of its businessrdquo See UPA (1997) sect 301(1) Benrsquos misrepresentation to the collector even if intentional appears to be in the ordinary course of the partnershiprsquos business of dealing

27

Agency and Partnership Analysis

in antique cars Thus Benrsquos wrongful act created a partnership obligation for which Adam was jointly and severally liable

[NOTE Generally a partnership creditor must ldquoexhaust the partnershiprsquos assets before levying on a judgment debtor partnerrsquos individual property where the partner is personally liable for the partnership obligationrdquo as a result of his status as a partner UPA (1997) sect 307 cmt 4 As the UPA comments explain this places Adam more in the position of guarantor than principal debtor on the partnership obligation Id cmt 4 Although an examinee might discuss this point the call focuses on whether Adam is personally liable not how the liability might be enforced]

Point Two (30) Because the collectorrsquos claim arose before Diane joined Empire Diane did not become personally liable on the claim

Diane was admitted to Empire when it was a general partnership and after the collectorrsquos claim arose While the general rule under UPA (1997) sect 306(a) is that the partners of a general partnership are liable jointly and severally for all obligations of the partnership there is a special rule for partners who are admitted during the duration of the partnership Under UPA (1997) sect 306(b) a person admitted to an existing partnership is not personally liable for any partnership obligations incurred before the personrsquos admission Because Diane was admitted to Empire after the collectorrsquos claim arose Diane is not personally liable on the claim

Dianersquos knowledge of the pre-existing claim and her stated concern about becoming liable on the collectorrsquos claim do not change her personal nonliability to the collector Although partners who have a liability shield can assume liability to third parties through private contractual guarantees or modifications to the partnership agreement Dianersquos stated concern constituted neither a guaranty to the collector nor ldquoan intentional waiver of liability protectionsrdquo See UPA (1997) sect 306 cmt 3 (describing methods for waiver of liability protections under sect 306(c) applicable in limited liability partnerships)

At most Diane will lose her investment in the partnership as a result of the collectorrsquos claim Although Diane did not become personally liable on the collectorrsquos claim when she joined the partnership the $250000 she contributed to the partnership is ldquoat risk for the satisfaction of existing partnership debtsrdquo UPA (1997) sect 306 cmt 2

Point Three (40) Filing the statement of qualification was effective to elect limited liability partnership status Despite this new status Adam and Ben remain personally liable on the collectorrsquos claim which arose before the election But as partners in an LLP neither Adam Ben nor Diane is personally liable as a partner on the driverrsquos estatersquos claim which arose after the election

Under UPA (1997) sect 1001 a general partnership can make an election and become a limited liability partnershipmdashif the partners approve the conversion by a vote equivalent to that necessary to amend the partnership agreement and the partnership then files a statement of qualification that specifies the name of the partnership its principal office and its election to be an LLP Here the partners agreed unanimouslymdashsufficient to amend their agreement under UPA (1997) sect 401(j)mdashand the statement of qualification was filed In addition the name of Empire LLP properly included an appropriate ending ldquoLLPrdquo See UPA (1997) sect 1002

Although another way to effectuate a ldquoconversionrdquo (as suggested by Benrsquos lawyer) is to form a new LLP and transfer the assets of the old general partnership to the new LLP the

28

Agency and Partnership Analysis

method used here (approval by the partners and the filing of a statement of qualification) is also sufficient to create LLP status

Thus Empire became Empire LLP as of the date of filing of the statement of qualification See UPA (1997) sect 1001 What effect did this have on the collectorrsquos claim which predated the filing According to UPA (1997) sect 306(c) an obligation incurred while a partnership is an LLP is solely a partnership obligation As the collectorrsquos claim predated the LLP Adam and Ben remain personally liable on the collectorrsquos claim Diane on the other hand was not personally liable on the collectorrsquos claim either before or after the filing of the statement of qualification See Point Two above

The driverrsquos estatersquos claim arose after Empire became Empire LLP Under UPA (1997) sect 306(c) an obligation incurred while a partnership is an LLP is solely a partnership obligationThus Adam Ben and Diane as partners are all protected from personal liability on the driverrsquos estatersquos claim But there may be personal liability if any of them was negligent or otherwise acted wrongfully by not informing the buyer of the bad suspension that caused the accident

29

National Conference of Bar Examiners 302 South Bedford Street | Madison WI 53703-3622 Phone 608-280-8550 | Fax 608-280-8552 | TDD 608-661-1275

wwwncbexorg e-mail contactncbexorg

  • Preface
  • Description of the MEE
  • Instructions
  • February 2014 Questions
    • Constitutinal Law Question
    • Trusts and Future Interests Question
    • Secured Transactions Question
    • Federal Civil Procedure Question
    • Criminal Law and Procedure Question
    • Agency and Partnership Question
      • February 2014 Analyses
        • Constitutional Law Analysis
        • Trust and Future Interests Analysis
        • Secured Transactions Analysis
        • Federal Civil Procedure Analysis
        • Criminal Law and Procedure Analysis
        • Agency and Partnership Analysis
            • ltlt13 ASCII85EncodePages false13 AllowTransparency false13 AutoPositionEPSFiles true13 AutoRotatePages None13 Binding Left13 CalGrayProfile (Dot Gain 20)13 CalRGBProfile (sRGB IEC61966-21)13 CalCMYKProfile (US Web Coated 050SWOP051 v2)13 sRGBProfile (sRGB IEC61966-21)13 CannotEmbedFontPolicy Error13 CompatibilityLevel 1413 CompressObjects Tags13 CompressPages true13 ConvertImagesToIndexed true13 PassThroughJPEGImages true13 CreateJobTicket false13 DefaultRenderingIntent Default13 DetectBlends true13 DetectCurves 0000013 ColorConversionStrategy CMYK13 DoThumbnails false13 EmbedAllFonts true13 EmbedOpenType false13 ParseICCProfilesInComments true13 EmbedJobOptions true13 DSCReportingLevel 013 EmitDSCWarnings false13 EndPage -113 ImageMemory 104857613 LockDistillerParams false13 MaxSubsetPct 10013 Optimize true13 OPM 113 ParseDSCComments true13 ParseDSCCommentsForDocInfo true13 PreserveCopyPage true13 PreserveDICMYKValues true13 PreserveEPSInfo true13 PreserveFlatness true13 PreserveHalftoneInfo false13 PreserveOPIComments true13 PreserveOverprintSettings true13 StartPage 113 SubsetFonts true13 TransferFunctionInfo Apply13 UCRandBGInfo Preserve13 UsePrologue false13 ColorSettingsFile ()13 AlwaysEmbed [ true13 ]13 NeverEmbed [ true13 ]13 AntiAliasColorImages false13 CropColorImages true13 ColorImageMinResolution 30013 ColorImageMinResolutionPolicy OK13 DownsampleColorImages true13 ColorImageDownsampleType Bicubic13 ColorImageResolution 30013 ColorImageDepth -113 ColorImageMinDownsampleDepth 113 ColorImageDownsampleThreshold 15000013 EncodeColorImages true13 ColorImageFilter DCTEncode13 AutoFilterColorImages true13 ColorImageAutoFilterStrategy JPEG13 ColorACSImageDict ltlt13 QFactor 01513 HSamples [1 1 1 1] VSamples [1 1 1 1]13 gtgt13 ColorImageDict ltlt13 QFactor 01513 HSamples [1 1 1 1] VSamples [1 1 1 1]13 gtgt13 JPEG2000ColorACSImageDict ltlt13 TileWidth 25613 TileHeight 25613 Quality 3013 gtgt13 JPEG2000ColorImageDict ltlt13 TileWidth 25613 TileHeight 25613 Quality 3013 gtgt13 AntiAliasGrayImages false13 CropGrayImages true13 GrayImageMinResolution 30013 GrayImageMinResolutionPolicy OK13 DownsampleGrayImages true13 GrayImageDownsampleType Bicubic13 GrayImageResolution 30013 GrayImageDepth -113 GrayImageMinDownsampleDepth 213 GrayImageDownsampleThreshold 15000013 EncodeGrayImages true13 GrayImageFilter DCTEncode13 AutoFilterGrayImages true13 GrayImageAutoFilterStrategy JPEG13 GrayACSImageDict ltlt13 QFactor 01513 HSamples [1 1 1 1] VSamples [1 1 1 1]13 gtgt13 GrayImageDict ltlt13 QFactor 01513 HSamples [1 1 1 1] VSamples [1 1 1 1]13 gtgt13 JPEG2000GrayACSImageDict ltlt13 TileWidth 25613 TileHeight 25613 Quality 3013 gtgt13 JPEG2000GrayImageDict ltlt13 TileWidth 25613 TileHeight 25613 Quality 3013 gtgt13 AntiAliasMonoImages false13 CropMonoImages true13 MonoImageMinResolution 120013 MonoImageMinResolutionPolicy OK13 DownsampleMonoImages true13 MonoImageDownsampleType Bicubic13 MonoImageResolution 120013 MonoImageDepth -113 MonoImageDownsampleThreshold 15000013 EncodeMonoImages true13 MonoImageFilter CCITTFaxEncode13 MonoImageDict ltlt13 K -113 gtgt13 AllowPSXObjects false13 CheckCompliance [13 None13 ]13 PDFX1aCheck false13 PDFX3Check false13 PDFXCompliantPDFOnly false13 PDFXNoTrimBoxError true13 PDFXTrimBoxToMediaBoxOffset [13 00000013 00000013 00000013 00000013 ]13 PDFXSetBleedBoxToMediaBox true13 PDFXBleedBoxToTrimBoxOffset [13 00000013 00000013 00000013 00000013 ]13 PDFXOutputIntentProfile ()13 PDFXOutputConditionIdentifier ()13 PDFXOutputCondition ()13 PDFXRegistryName ()13 PDFXTrapped False1313 CreateJDFFile false13 Description ltlt13 ARA 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 BGR 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 CHS ltFEFF4f7f75288fd94e9b8bbe5b9a521b5efa7684002000410064006f006200650020005000440046002065876863900275284e8e9ad88d2891cf76845370524d53705237300260a853ef4ee54f7f75280020004100630072006f0062006100740020548c002000410064006f00620065002000520065006100640065007200200035002e003000204ee553ca66f49ad87248672c676562535f00521b5efa768400200050004400460020658768633002gt13 CHT ltFEFF4f7f752890194e9b8a2d7f6e5efa7acb7684002000410064006f006200650020005000440046002065874ef69069752865bc9ad854c18cea76845370524d5370523786557406300260a853ef4ee54f7f75280020004100630072006f0062006100740020548c002000410064006f00620065002000520065006100640065007200200035002e003000204ee553ca66f49ad87248672c4f86958b555f5df25efa7acb76840020005000440046002065874ef63002gt13 CZE 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 DAN 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 DEU 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 ESP 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 ETI 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 FRA 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 GRE ltFEFF03a703c103b703c303b903bc03bf03c003bf03b903ae03c303c403b5002003b103c503c403ad03c2002003c403b903c2002003c103c503b803bc03af03c303b503b903c2002003b303b903b1002003bd03b1002003b403b703bc03b903bf03c503c103b303ae03c303b503c403b5002003ad03b303b303c103b103c603b1002000410064006f006200650020005000440046002003c003bf03c5002003b503af03bd03b103b9002003ba03b103c42019002003b503be03bf03c703ae03bd002003ba03b103c403ac03bb03bb03b703bb03b1002003b303b903b1002003c003c103bf002d03b503ba03c403c503c003c903c403b903ba03ad03c2002003b503c103b303b103c303af03b503c2002003c503c803b703bb03ae03c2002003c003bf03b903cc03c403b703c403b103c2002e0020002003a403b10020005000440046002003ad03b303b303c103b103c603b1002003c003bf03c5002003ad03c703b503c403b5002003b403b703bc03b903bf03c503c103b303ae03c303b503b9002003bc03c003bf03c103bf03cd03bd002003bd03b1002003b103bd03bf03b903c703c403bf03cd03bd002003bc03b5002003c403bf0020004100630072006f006200610074002c002003c403bf002000410064006f00620065002000520065006100640065007200200035002e0030002003ba03b103b9002003bc03b503c403b103b303b503bd03ad03c303c403b503c103b503c2002003b503ba03b403cc03c303b503b903c2002egt13 HEB 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 HRV (Za stvaranje Adobe PDF dokumenata najpogodnijih za visokokvalitetni ispis prije tiskanja koristite ove postavke Stvoreni PDF dokumenti mogu se otvoriti Acrobat i Adobe Reader 50 i kasnijim verzijama)13 HUN 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 ITA 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 JPN ltFEFF9ad854c18cea306a30d730ea30d730ec30b951fa529b7528002000410064006f0062006500200050004400460020658766f8306e4f5c6210306b4f7f75283057307e305930023053306e8a2d5b9a30674f5c62103055308c305f0020005000440046002030d530a130a430eb306f3001004100630072006f0062006100740020304a30883073002000410064006f00620065002000520065006100640065007200200035002e003000204ee5964d3067958b304f30533068304c3067304d307e305930023053306e8a2d5b9a306b306f30d530a930f330c8306e57cb30818fbc307f304c5fc59808306730593002gt13 KOR ltFEFFc7740020c124c815c7440020c0acc6a9d558c5ec0020ace0d488c9c80020c2dcd5d80020c778c1c4c5d00020ac00c7a50020c801d569d55c002000410064006f0062006500200050004400460020bb38c11cb97c0020c791c131d569b2c8b2e4002e0020c774b807ac8c0020c791c131b41c00200050004400460020bb38c11cb2940020004100630072006f0062006100740020bc0f002000410064006f00620065002000520065006100640065007200200035002e00300020c774c0c1c5d0c11c0020c5f40020c2180020c788c2b5b2c8b2e4002egt13 LTH 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 LVI 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 NLD (Gebruik deze instellingen om Adobe PDF-documenten te maken die zijn geoptimaliseerd voor prepress-afdrukken van hoge kwaliteit De gemaakte PDF-documenten kunnen worden geopend met Acrobat en Adobe Reader 50 en hoger)13 NOR ltFEFF004200720075006b00200064006900730073006500200069006e006e007300740069006c006c0069006e00670065006e0065002000740069006c002000e50020006f0070007000720065007400740065002000410064006f006200650020005000440046002d0064006f006b0075006d0065006e00740065007200200073006f006d00200065007200200062006500730074002000650067006e0065007400200066006f00720020006600f80072007400720079006b006b0073007500740073006b00720069006600740020006100760020006800f800790020006b00760061006c0069007400650074002e0020005000440046002d0064006f006b0075006d0065006e00740065006e00650020006b0061006e002000e50070006e00650073002000690020004100630072006f00620061007400200065006c006c00650072002000410064006f00620065002000520065006100640065007200200035002e003000200065006c006c00650072002000730065006e006500720065002egt13 POL ltFEFF0055007300740061007700690065006e0069006100200064006f002000740077006f0072007a0065006e0069006100200064006f006b0075006d0065006e007400f300770020005000440046002000700072007a0065007a006e00610063007a006f006e00790063006800200064006f002000770079006400720075006b00f30077002000770020007700790073006f006b00690065006a0020006a0061006b006f015b00630069002e002000200044006f006b0075006d0065006e0074007900200050004400460020006d006f017c006e00610020006f007400770069006500720061010700200077002000700072006f006700720061006d006900650020004100630072006f00620061007400200069002000410064006f00620065002000520065006100640065007200200035002e0030002000690020006e006f00770073007a0079006d002egt13 PTB ltFEFF005500740069006c0069007a006500200065007300730061007300200063006f006e00660069006700750072006100e700f50065007300200064006500200066006f0072006d00610020006100200063007200690061007200200064006f00630075006d0065006e0074006f0073002000410064006f0062006500200050004400460020006d00610069007300200061006400650071007500610064006f00730020007000610072006100200070007200e9002d0069006d0070007200650073007300f50065007300200064006500200061006c007400610020007100750061006c00690064006100640065002e0020004f007300200064006f00630075006d0065006e0074006f00730020005000440046002000630072006900610064006f007300200070006f00640065006d0020007300650072002000610062006500720074006f007300200063006f006d0020006f0020004100630072006f006200610074002000650020006f002000410064006f00620065002000520065006100640065007200200035002e0030002000650020007600650072007300f50065007300200070006f00730074006500720069006f007200650073002egt13 RUM 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 RUS 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 SKY ltFEFF0054006900650074006f0020006e006100730074006100760065006e0069006100200070006f0075017e0069007400650020006e00610020007600790074007600e100720061006e0069006500200064006f006b0075006d0065006e0074006f0076002000410064006f006200650020005000440046002c0020006b0074006f007200e90020007300610020006e0061006a006c0065007001610069006500200068006f0064006900610020006e00610020006b00760061006c00690074006e00fa00200074006c0061010d00200061002000700072006500700072006500730073002e00200056007900740076006f00720065006e00e900200064006f006b0075006d0065006e007400790020005000440046002000620075006400650020006d006f017e006e00e90020006f00740076006f00720069016500200076002000700072006f006700720061006d006f006300680020004100630072006f00620061007400200061002000410064006f00620065002000520065006100640065007200200035002e0030002000610020006e006f0076016100ed00630068002egt13 SLV 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 SUO 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 SVE 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 TUR ltFEFF005900fc006b00730065006b0020006b0061006c006900740065006c0069002000f6006e002000790061007a006401310072006d00610020006200610073006b013100730131006e006100200065006e0020006900790069002000750079006100620069006c006500630065006b002000410064006f006200650020005000440046002000620065006c00670065006c0065007200690020006f006c0075015f007400750072006d0061006b0020006900e70069006e00200062007500200061007900610072006c0061007201310020006b0075006c006c0061006e0131006e002e00200020004f006c0075015f0074007500720075006c0061006e0020005000440046002000620065006c00670065006c0065007200690020004100630072006f006200610074002000760065002000410064006f00620065002000520065006100640065007200200035002e003000200076006500200073006f006e0072006100730131006e00640061006b00690020007300fc007200fc006d006c00650072006c00650020006100e70131006c006100620069006c00690072002egt13 UKR 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 ENU (Use these settings to create Adobe PDF documents best suited for high-quality prepress printing Created PDF documents can be opened with Acrobat and Adobe Reader 50 and later)13 gtgt13 Namespace [13 (Adobe)13 (Common)13 (10)13 ]13 OtherNamespaces [13 ltlt13 AsReaderSpreads false13 CropImagesToFrames true13 ErrorControl WarnAndContinue13 FlattenerIgnoreSpreadOverrides false13 IncludeGuidesGrids false13 IncludeNonPrinting false13 IncludeSlug false13 Namespace [13 (Adobe)13 (InDesign)13 (40)13 ]13 OmitPlacedBitmaps false13 OmitPlacedEPS false13 OmitPlacedPDF false13 SimulateOverprint Legacy13 gtgt13 ltlt13 AddBleedMarks false13 AddColorBars false13 AddCropMarks false13 AddPageInfo false13 AddRegMarks false13 ConvertColors ConvertToCMYK13 DestinationProfileName ()13 DestinationProfileSelector DocumentCMYK13 Downsample16BitImages true13 FlattenerPreset ltlt13 PresetSelector MediumResolution13 gtgt13 FormElements false13 GenerateStructure false13 IncludeBookmarks false13 IncludeHyperlinks false13 IncludeInteractive false13 IncludeLayers false13 IncludeProfiles false13 MultimediaHandling UseObjectSettings13 Namespace [13 (Adobe)13 (CreativeSuite)13 (20)13 ]13 PDFXOutputIntentProfileSelector DocumentCMYK13 PreserveEditing true13 UntaggedCMYKHandling LeaveUntagged13 UntaggedRGBHandling UseDocumentProfile13 UseDocumentBleed false13 gtgt13 ]13gtgt setdistillerparams13ltlt13 HWResolution [2400 2400]13 PageSize [612000 792000]13gtgt setpagedevice13

Page 14: February 2014 MEE Questions and AnalysesPreface The Multistate Essay Examination (MEE) is developed by the National Conference of Bar Examiners (NCBE). This publication includes the

CONSTITUTIONAL LAW ANALYSIS (Constitutional Law IVD)

ANALYSIS

Legal Problems

(1) Is the city ordinance requirement that businesses install floodlights a taking

(2) Is conditioning the approval of a building permit on the grant of an easement to install surveillance equipment a taking of property

DISCUSSION

Summary

The ordinance requiring businesses to install floodlights is not a per se taking under Loretto because it does not force a private landowner to allow a third party to enter and place a physical object on the land Here the city ordinance requires the businessmdashnot a third partymdashto install the floodlights

The ordinance is likely not a regulatory taking under the Penn Central balancing test While the ordinance will impose a cost on business owners that cost may be offset by the expected increase in business due to the ordinance and the ordinance does not appear to interfere with the ownerrsquos primary use of the property as a restaurant

The permit condition however is likely an uncompensated taking of property While the condition has an essential nexus with the cityrsquos legitimate interest in promoting public safety the city has not made an individualized determination that the easement condition is roughly proportional to the possibility of increased crime due to the restaurantrsquos proposed addition Thus the permit condition likely violates the Fifth Amendment as applied to the states through the Fourteenth Amendment

Point One (50) The ordinance requiring that businesses install floodlights is not a per se taking under Loretto It is not a regulatory taking under the Penn Central balancing test because the cost of compliance with the ordinance may be offset by an expected increase in business and compliance does not interfere with the businessrsquos primary use of its property as a restaurant

The city ordinance requiring a business to install floodlights does not effect a per se taking of the sort described in Loretto v Teleprompter Manhattan CATV Corp 458 US 419 (1982) because no property is physically taken by the government and the ordinance does not involve a physical invasion of private property by a third party

Even though the ordinance does not constitute an occupation of the property by either the government or a third party it is still subject to the three-factor balancing test under Penn Central Transportation Co v City of New York 438 US 104 (1978) to determine whether it is a ldquoregulatory takingrdquo Under Penn Central a court must balance (1) ldquo[t]he economic impact of the regulation on the claimantrdquo (2) ldquothe extent to which the regulation has interfered with distinct investment-backed expectationsrdquo and (3) ldquothe character of the governmental actionrdquo Id at 124 Here each factor weighs against finding that the ordinance is a taking

11

Constitutional Law Analysis

First the ordinance requirement likely has a minimal economic impact on the restaurant Compliance with the ordinance is estimated to cost $1000 and the city has found that businesses will likely recoup that cost in increased sales Also because the ordinance does not interfere with the operation of the restaurant the owner may still earn a reasonable return on its investment in the property

Second the ordinance does not interfere with the businessrsquos investment-backed expectations As in Penn Central the challenged law does not interfere with the ownerrsquos ldquoprimary expectationrdquo for use of the propertymdashin Penn Central as a railroad terminal and here as a restaurant Further the ordinance does not prevent the restaurant from expanding to meet the changing business environment

Third the character of the government action does not weigh in favor of a taking While Penn Central does say that a ldquophysical invasionrdquo is more likely to pose a taking Loretto suggests that the Courtrsquos main concern is with physical invasions by third parties Also like the landmark law challenged in Penn Central the ordinance here ldquoadjust[s] the benefits and burdens of economic life to promote the common goodrdquo Id In Penn Central the landmark law restricted development of the railroad terminal to promote the common interest in preserving historic landmarks Here the ordinance requires the businesses to install floodlights to promote the common interest in crime prevention and public safety

Because the ordinance is clearly a valid exercise of the police power it satisfies the takings clausersquos public-use requirement Kelo v City of New London 545 US 469 (2005)

In sum all three factors weigh against finding a taking under the Penn Central balancing test

Point Two (50) The permit condition may be unconstitutional as an uncompensated taking of property because the city has not made an individualized determination that the easement condition is roughly proportional to the impact of the restaurantrsquos proposed addition

In Dolan v City of Tigard 512 US 374 (1994) the Supreme Court set forth the test for determining whether an exaction imposed by a government in exchange for a discretionary benefit conferred by the government such as a condition on the approval of a building permit in this case constitutes an uncompensated taking under the Fifth Amendment The exaction is not a taking if (1) there is an ldquoessential nexusrdquo between the ldquopublic need or burdenrdquo to which the proposed development contributes and ldquothe permit condition exacted by the cityrdquo id at 386 and (2) the government makes ldquosome sort of individualized determination that the required dedication is [roughly proportional] both in nature and extent to the impact of the proposed developmentrdquo Id at 391 see also Nollan v California Coastal Commission 483 US 825 (1987)

Here the city likely can meet the nexus requirement In Dolan the landowner sought to double the size of its business which would have increased traffic on nearby roadways In exchange for approving the development the city sought an easement for a bike and pedestrian path The Court found the required nexus between the easement and the cityrsquos ldquoattempt to reduce traffic congestion by providing for alternative means of transportationrdquo 512 US at 387 Here a similar nexus likely exists between the requested easement and the cityrsquos interest in crime prevention and public safety Increased patronage and economic activity at the restaurant might attract additional crime to the area and the requested easement to install surveillance equipment would attempt to address that increased crime

12

Constitutional Law Analysis

The exaction here however may fail the second prong of the Dolan testmdashthat the exaction be roughly proportional to the anticipated impact of the requested development As noted the city in Dolan claimed that a bike and pedestrian path was needed to offset the increase in traffic due to the proposed doubling of the business The Court explained that the government must demonstrate that the additional traffic reasonably was related to the requested exaction and that the government must ldquomake some effort to quantify its findings in support of the dedication for the pedestrianbicycle pathway beyond the conclusory statement that it could offset some of the traffic demand generatedrdquo Id at 395 Here the city did not carry its burden The city simply speculates that increased patronage of the restaurant ldquomightrdquo increase crime and that the surveillance equipment ldquomightrdquo alleviate this increased crime Because the city has not made ldquosome effort to quantify its findingsrdquo in support of the easement it has not shown that the burden of the easement is roughly proportional to the benefits thought to flow from it

Thus the exaction appears to be an uncompensated taking of property in violation of the Fifth Amendment as applied to the states through the Fourteenth Amendment

13

TRUSTS AND FUTURE INTERESTS ANALYSIS ____ (Trusts and Future Interests IE3 I5 IIIA amp B)

ANALYSIS

Legal Problems

(1) How should rents dividends and sales proceeds received by the trustee prior to receipt of the sonrsquos letter have been allocated between trust income and principal

(2)(a) Did the remainder interest in the trust accelerate and become immediately payable to the daughterrsquos minor child upon the trusteersquos receipt of the sonrsquos letter and if not how should the trustee handle the distribution of the principal in the future

(2)(b) Following the trusteersquos receipt of the sonrsquos letter how should the trustee distribute future receipts of income prior to the distribution of the principal

DISCUSSION

Summary

Prior to the trusteersquos receipt of the sonrsquos letter cash dividends and rents should have been allocated to trust income and were distributable to the son the income beneficiary of the trust sales proceeds and stock dividends should have been allocated to principal

Because the sonrsquos letter to the trustee did not result in a valid disclaimer under state law (having been made more than nine months after the testatorrsquos death) the son is not deemed to have predeceased the testator Because the son is still living the class gift to the testatorrsquos grandchildren who survive the son has not closed and is not possessory it will not become possessory until the son dies The daughterrsquos minor child being the testatorrsquos only living grandchild is not currently entitled to a distribution of trust principal Trust principal will instead be distributable upon the sonrsquos death to the testatorrsquos then-living grandchildren or if there are none to the testatorrsquos then-living heirs

As for future income the trustee should either distribute the trust income to the son and the daughter as the testatorrsquos heirs accumulate the income for future distribution to those individuals ultimately entitled to the trust principal or distribute it to those presumptively entitled to the principal upon the sonrsquos death ie the daughterrsquos minor child

Point One (45) Cash dividends and rents are allocable to income sales proceeds and stock dividends are allocable to principal Items allocable to income for the period prior to the sonrsquos attempted disclaimer were distributable to the son

Receipts earned during the administration of a trust are allocable either to income or to principal Almost all states have adopted the most recent or an earlier version of the Uniform Principal and Income Act (the Act) which specifies how such receipts should be allocated

Under the Act rents (UNIF PRIN amp INC ACT (2000) sect 405 UNIF PRIN amp INC ACT (1962) sect 3(a)(1)) and cash dividends received from a corporation (UNIF PRIN amp INC ACT (2000) sect 401(b) UNIF PRIN amp INC ACT (1962) sect 6(d)) are allocable to income and are distributable to the income beneficiary of the trust

14

Trusts and Future Interests Analysis Sales proceeds (UNIF PRIN amp INC ACT (2000) sect 404(2) UNIF PRIN amp INC ACT (1962)

sect 3(b)(1)) and dividends paid in the stock of the distributing corporation (UNIF PRIN amp INC ACT (2000) sect 401(c)(1) UNIF PRIN amp INC ACT (1962) sect 3(b)(4)) are allocable to principal and added to the principal of the trust

Here the cash dividends and office building rents should have been allocated to income and until the trustee received the sonrsquos letter should have been distributed to him as the sole income beneficiary of the trust The stock dividend and proceeds from the sale of the office building should have been allocated to principal and held by the trustee for future distribution to the ultimate remaindermen of the trust

[NOTE The 2000 Uniform Principal and Income Act has been adopted in Alabama Arkansas Colorado Connecticut the District of Columbia Hawaii Idaho Iowa Kentucky Missouri Montana Nebraska New Mexico North Dakota Oregon South Dakota Utah and West Virginia]

Point Two(a) (45) Because the son did not disclaim within nine months of the testatorrsquos death there is no valid disclaimer under state law Therefore the son is not deemed to have predeceased the testator Furthermore because of the express survivorship contingency in the will the remainder in the trust does not accelerate and become distributable until the son in fact dies When the son dies the trust principal will be distributable to the testatorrsquos then-living grandchildren or if none then to the testatorrsquos then-living heirs

When a trust remainder is given to a class the class closes (ie no new persons can join the class) when there is no outstanding income interest and at least one member of the class is then entitled to demand possession of his or her share of the remainder This principle is called the rule of convenience See generally HERBERT HOVENKAMP amp SHELDON F KURTZ PRINCIPLES OF PROPERTY LAW 199ndash200 (6th ed 2005) A class member may demand possession of his or her share of the remainder upon termination of the income interest only when the class memberrsquos interest is not otherwise subject to a condition precedent See id

When a beneficiary timely disclaims an interest in a trust that beneficiary is treated as if he had predeceased the testator Here had the son disclaimed within nine months of the testatorrsquos death as required by the state statute he would have been deemed to have predeceased the testator This would have closed the class of remaindermen and the testatorrsquos then-living grandchildren (ie the daughterrsquos child) would have been entitled to the trust principal However under the state statute the sonrsquos disclaimer was not timely because he did not disclaim within nine months of the testatorrsquos death Thus because the statute is inapplicable and the son is still alive the class of grandchildren entitled to share in trust principal did not close

Because here the statute is inapplicable due to the sonrsquos failure to comply with the statutory time requirements then presumably the common-law rule allowing disclaimers (aka renunciations) at any time should apply Under the common law if a life estate is renounced the remainder interest accelerates and becomes immediately distributable to the remaindermen of the trust if the remainder is vested but not if the remainder is contingent JESSE DUKEMINIER amp ROBERT H SITKOFF WILLS TRUSTS AND ESTATES 844ndash845 (9th ed 2013) Here because the remainder is contingent upon there being grandchildren who survive the son the remainder will not accelerate It will remain open until the son dies leaving open the possibility that additional grandchildren will be included in the class or the daughterrsquos child could fall out of the class because that child fails to survive the son

And if none of the testatorrsquos grandchildren survive the son the trust principal will be distributed to the testatorrsquos heirs living at the sonrsquos death

15

Trusts and Future Interests Analysis

Point Two(b) (10) Until the trust terminates the trustee must continue to hold the trust assets The distribution of income in the meantime is unclear There are at least three possibilities Income earned on the undistributed assets could be distributed to the son and daughter as the testatorrsquos heirs accumulated and added to principal for distribution to the ultimate remaindermen or distributed from time to time to those persons who are presumptively remaindermen

When trust principal is not immediately distributable the trustee must continue to hold trust assets until the ultimate remaindermen are ascertained During this period trust income will be distributed or retained according to any instructions contained in the trust instrument See WILLIAM M MCGOVERN JR SHELDON F KURTZ amp DAVID M ENGLISH WILLS TRUSTS amp ESTATES sect 102 (4th ed 2010)

Here the testator did not specify what the trustee should do with trust income in the event the sonrsquos disclaimer did not comply with the state statute There are at least three approaches One approach would have the trustee distribute the trust income to the testatorrsquos heirs on the theory that the income represents property that was not disposed of by the testatorrsquos will and which thus passes by partial intestacy to the testatorrsquos heirs A second approach would have the trustee accumulate trust income for distribution to the ultimate remaindermen Under this approach only those individuals ultimately entitled to the principal would share in the income A third approach would have the trustee distribute trust income to those individuals who would be the remaindermen if the trust were to terminate when the income is received by the trustee under this approach trust income would be distributed to the daughterrsquos minor child until another presumptive remainderman is born This approach could result in individuals not ultimately entitled to principal say because they do not survive the son receiving income It could also result in a disproportionate distribution of income among the individuals ultimately entitled to income

[NOTE Examinees should demonstrate a recognition and understanding of the income-allocation problem and the alternatives available to address that issue There is no widely accepted solution to the problem Examinees who cite any of these possible problem-solving approaches may receive credit]

16

SECURED TRANSACTIONS ANALYSIS (Secured Transactions IB IID E amp F IIIB IVA B amp F)

ANALYSIS

Legal Problems

(1)(a) What is the nature of the bankrsquos claim to the businessrsquos equipment

(1)(b) What is the nature of the finance companyrsquos claim to the businessrsquos equipment

(1)(c) As between the bank and the finance company whose claim to the businessrsquos equipment has priority

(2) Do the claims of the bank and the finance company continue in the item of equipment sold by the business to the competitor

DISCUSSION

Summary

The bank and the finance company both have perfected security interests in the businessrsquos equipment Even though the finance companyrsquos perfected security interest was created first the bankrsquos perfected security interest has priority because the bankrsquos financing statement was filed before the finance companyrsquos financing statement The security interests of the bank and the finance company continue in the item of equipment sold by the business to the competitor because their security interests were perfected and the competitor was not a buyer in ordinary course of business

Point One(a) (25) The bank has a perfected security interest in the businessrsquos equipment

The bank has met all criteria necessary for it to have an attached and enforceable security interest in the businessrsquos equipment First value must be given UCC sect 9-203(b)(1) This criterion is fulfilled by the loan made by the bank to the business Second the debtor must have rights in the collateral UCC sect 9-203(b)(2) Clearly the business has rights in its equipment Third either the secured party must take possession of the collateral or the debtor must authenticate a security agreement containing a description of the collateral UCC sect 9-203(b)(3) The agreement that the business owner signed is a ldquosecurity agreementrdquo because it is an agreement that creates or provides for a security interest UCC sect 9-102(a)(74) By signing the security agreement the business owner authenticated it UCC sect 9-102(a)(7) Therefore all three criteria are fulfilled and the bank has an enforceable and attached security interest

A security interest is perfected when it has attached and when any additional steps required for perfection have occurred UCC sect 9-308(a) Generally speaking the additional steps will either be possession of the collateral by the secured party or the filing of a financing statement with respect to the collateral See UCC sectsect 9-310 9-313 In this case the bank filed a financing statement naming the debtor and sufficiently indicating the collateral The collateral indication is sufficient because it identifies the collateral by type of property See UCC sectsect 9-504 9-108 The fact that the financing statement was filed before the security interest was created is

17

Secured Transactions Analysis

not a problem Even though the security agreement had not yet been signed the business had authorized the filing of the financing statement in an authenticated record UCC sect 9-509(a)(1) Moreover the financing statement may be filed before the security agreement is created UCC sect 9-502(d)

Point One(b) (10) The finance company also has a perfected security interest in the businessrsquos equipment

The finance companyrsquos security interest is enforceable and attached for the same reasons as the bankrsquos security interest The loan from the finance company to the business constitutes value the business has rights in the collateral and the business owner has authenticated a security agreement containing a description of the collateral The finance companyrsquos security interest is perfected because the finance company filed a financing statement with respect to it that provides that the business is the debtor and indicates that the collateral is equipment

Point One(c) (30) The bankrsquos security interest has priority over the finance companyrsquos security interest because the bankrsquos financing statement was filed first

As between two perfected security interests the general rule is that the security interest that was the earlier to be either perfected or the subject of a filed financing statement has priority UCC sect 9-322(a)(1) While the finance companyrsquos security interest was perfected before the bankrsquos (March 15 vs March 22) the bankrsquos financing statement was filed even earlier on March 2 Thus under the first-to-file-or-perfect rule of UCC sect 9-322(a)(1) the bankrsquos security interest has priority No exceptions to the general rule apply here

Point Two (35) A security interest in collateral continues notwithstanding its sale unless an exception applies Because the security interests of the bank and the finance company were perfected and the competitor was not a buyer in ordinary course of business no exception applies and the security interests of both creditors continue in the equipment sold to the competitor

As a general rule a security interest in collateral continues notwithstanding the fact that the debtor has sold the collateral to another person UCC sect 9-315(a)(1) Thus unless an exception applies the security interests of the bank and the finance company will continue in the item of equipment sold to the competitor

A buyer of goods will take free of an unperfected security interest in those goods See UCC sect 9-317(a)(2) However when the competitor bought the businessrsquos equipment both the bank and the finance company had perfected security interests in the equipment

A buyer can take free even of a perfected security interest in goods if the buyer is a ldquobuyer in ordinary course of businessrdquo See UCC sect 9-320(a) However the competitor was not a buyer in ordinary course of business To be a ldquobuyer in ordinary course of businessrdquo a buyer must buy goods from a seller that is in the business of selling goods of that kind See UCC sect 1-201(b)(9) The competitor bought this equipment from a seller that is not in the business of selling goods of this kind so the competitor was not a buyer in ordinary course of business with respect to these goods

Because no exception applies the security interests of the bank and the finance company continue even after the item of equipment was sold to the competitor

18

FEDERAL CIVIL PROCEDURE ANALYSIS (Federal Civil Procedure IVD)

ANALYSIS

Legal Problems

(1) Is a document prepared in the course of a contract dispute protected from discovery as ldquowork productrdquo when there is no evidence that the document was prepared in anticipation of litigation

(2)(a) Is a partyrsquos failure to provide relevant electronically stored information excused when the information was destroyed pursuant to a routine document retention scheme at a time when litigation was contemplated by the destroying party

(2)(b) What sanctions should be imposed on a party for allowing the destruction of evidence that is relevant to potential future litigation

DISCUSSION

Summary

The report prepared by the structural engineer is probably not work product and is thus discoverable The engineer examined the foundation of the house at the customerrsquos request and the engineerrsquos findings are potentially relevant to the customerrsquos claim that the foundation is defective The report was not prepared in anticipation of litigation The customer appears to have sought the engineerrsquos opinion in response to the builderrsquos offer to fix any problems with the foundation that an engineer might identify Because the report was not prepared in anticipation of litigation it is not protected by the work-product doctrine

The builder should have taken appropriate steps to preserve evidence including suspending its document retention program as soon as it began planning for litigationmdashie on July 10 Its destruction of potentially relevant material after that date was wrongful However a court is unlikely to impose severe sanctions on the builder because there are no facts indicating that the builder acted in bad faith and the customer can prove that the foundation is defective without the destroyed emails

Point One (40) The customer must turn over the engineerrsquos report because it was not prepared in anticipation of litigation

In general a party to a lawsuit in federal court ldquomay obtain discovery regarding any nonprivileged matter that is relevant to any partyrsquos claim or defenserdquo FED R CIV P 26(b)(1) (2009) This includes the right to inspect and copy documents in the other partyrsquos possession FED R CIV P 34(a)(1) Here the customer hired a structural engineer to examine the foundation of the house The engineerrsquos report on the foundation is likely to include information that would be relevant to the customerrsquos claim that the foundation was defectively constructed

The so-called ldquowork productrdquo rule allows a party to refuse to turn over ldquodocuments that are prepared in anticipation of litigation or for trialrdquo by that partyrsquos representative including

19

Federal Civil Procedure Analysis

a consultant Thus if the customer had hired the structural engineer to prepare a report ldquoin anticipation of litigationrdquo that report might not be discoverable See FED R CIV P 26(b)(3)

In this case however the customer hired the engineer to evaluate the foundation of the house as part of the customerrsquos negotiation with the builder concerning the housersquos flooding problem The builder told the customer that the housersquos landscaping was the reason for the flooding and the builder told the customer ldquoHave an engineer look at the foundation If therersquos a problem wersquoll fix itrdquo The customer appears to have acted in response to that statement There is no indication that the customer anticipated any kind of legal action at the time that the structural engineer was hired Accordingly the structural engineerrsquos report is discoverable and the court should order the customer to turn it over

[NOTE If an examinee concludes that the structural engineerrsquos report was prepared in anticipation of litigation then the examinee should also conclude that the report is not discoverable Documents prepared in anticipation of litigation do not need to be disclosed to an adverse party unless that party can demonstrate a ldquosubstantial needrdquo for the documents and an inability to obtain substantially equivalent information without ldquoundue hardshiprdquo FED R CIV P 26(b)(3)(A)(ii) Furthermore a report prepared by an expert who is not expected to testify is not discoverable in the absence of ldquoexceptional circumstancesrdquo making it ldquoimpracticablerdquo to obtain the information in another way FED R CIV P 26(b)(4)(D)(ii) The builder probably cannot make these showings here unless the engineerrsquos report deals with circumstances that have since changed There is no evidence that the structural engineer would have had access to any information or facts that the builder would not already know as a result of its construction and subsequent inspection of the house In addition if necessary the builder could ask the court for permission to arrange for a further inspection of the house by a structural engineer hired by the builder See FED R CIV P 34(a)(2) Accordingly if an examinee concludes that the report was prepared in anticipation of litigation the examinee should also conclude that the builder is not entitled to see the report]

Point Two(a) (30) Because the builder anticipated that it might be involved in litigation concerning its contract with the customer the builder acted wrongfully in destroying emails that were relevant to the housersquos construction even though the emails were destroyed pursuant to a routine document retention plan

As noted above a party to a lawsuit in federal court ldquomay obtain discovery regarding any nonprivileged matter that is relevant to any partyrsquos claim or defenserdquo FED R CIV P 26(b)(1) This includes emails and other electronically stored information FED R CIV P 34(a)(1)(A) Here the customer has requested all the builderrsquos emails pertaining to work done on the foundation of the house Ordinarily the builder would be obliged to turn over this information which is relevant to the customerrsquos defense that the housersquos foundation was poorly constructed

Unfortunately the emails in question no longer exist because the builder destroyed them on August 2

In general spoliation of evidence (destruction or alteration of evidence) is improper if the party who destroyed or altered the evidence ldquohas notice that the evidence is relevant to litigation or should have known that the evidence may be relevant to future litigationrdquo Fujitsu Ltd v Federal Express Corp 247 F3d 423 436 (2d Cir 2001) It is improper for a party to destroy electronic information relevant to pending litigation even if the destruction occurs before there is any request or order seeking the information See eg Leon v IDX Sys Corp 464 F3d 951 (9th Cir 2006) (plaintiffrsquos intentional destruction of computer files warranted dismissal even

20

In this case the builderrsquos destruction of the emails was pursuant to a routine document retention plan The Federal Rules provide expressly that in the absence of ldquoexceptional circumstancesrdquo parties should not be sanctioned for the loss of electronically stored information when the loss occurs pursuant to ldquoroutine good-faith operation of an electronic information systemrdquo FED R CIV P 37(e) However when a party anticipates litigation ldquoit must suspend its routine document retentiondestruction policy and put in place a lsquolitigation holdrsquo to ensure the preservation of relevant documentsrdquo Zubulake v UBS Warburg LLC 220 FRD 212 218 (SDNY 2003)

Federal Civil Procedure Analysis

though spoliation occurred before order compelling discovery) Similarly the duty to preserve evidence applies to a party who anticipates litigation even if litigation has not yet been commenced See THE SEDONA PRINCIPLES BEST PRACTICES RECOMMENDATIONS amp PRINCIPLES FOR ADDRESSING ELECTRONIC DOCUMENT PRODUCTION 70 cmt 14a (2d ed 2007)

The builder destroyed the emails on August 2 At that time the builder knew that litigation was a possibility because the builder had already directed its attorney to prepare a draft complaint for possible filing Knowing that litigation was a possibility the builder had a duty to take steps to preserve evidence including the emails in question See generally Fujitsu Ltd

Thus the builderrsquos destruction of potentially relevant emails at a time when it knew that litigation was a possibility was improper It had a duty to preserve evidence and it breached that duty

[NOTE Because courts have used different words to describe the test for when evidence must be preserved an examineersquos precise formulation of the test is not critical]

Point Two(b) (30) In determining appropriate sanctions for spoliation courts consider both the level of culpability of the spoliating party and the degree of prejudice the loss of evidence has caused the other party Here the builderrsquos destruction of evidence does not appear to have been willful nor is it likely to pose a significant obstacle to the customerrsquos defense Any sanctions imposed by the court should be modest

Federal courts have inherent power to control the litigation process and can sanction misbehavior including spoliation even when there has been no specific violation of the Federal Rules of Civil Procedure See generally Chambers v NASCO Inc 501 US 32 (1991) (discussing courtrsquos inherent power to control the litigation process) The range of available sanctions is broad It can include such sanctions as the payment of expenses incurred by the other party as a result of the destruction of the evidence an instruction to the jury authorizing it to draw an adverse inference from the destruction of the evidence a shifting of the burden of proof on the relevant issue or even judgment against the responsible party See eg Residential Funding Corp v DeGeorge Financial Corp 306 F3d 99 108 (2d Cir 2002) (adverse inference) Silvestri v General Motors Corp 271 F3d 583 593 (4th Cir 2001) (possibility of dismissal) Cf FED R CIV P 37(b)(2)(A) (listing remedies for failure to comply with discovery obligations)

In determining appropriate sanctions for spoliation courts consider both the level of culpability of the spoliating party and the degree of prejudice the loss of evidence has caused the other party Many courts impose severe sanctions (such as an adverse-inference instruction or the entry of judgment against the spoliating party) only when there is evidence of bad faith in the form of an intentional effort to hide information Eg Greyhound Lines Inc v Wade 485 F3d 1032 1035 (8th Cir 2007) (spoliation sanction requires intentional destruction out of desire ldquoto suppress the truthrdquo) However other courts have said that negligence in preserving evidence can

21

Federal Civil Procedure Analysis

support an adverse-inference instruction See Residential Funding 306 F3d at 108 (negligence enough under some circumstances)

Although a court might well order an evidentiary hearing on the issue of sanctions the facts presented do not seem appropriate for severe sanctions First the evidence was destroyed pursuant to the builderrsquos standard document retention plan and there is no evidence that the builder deliberately failed to suspend its usual procedures with the purpose of allowing the destruction of evidence Second the loss of this evidence will not severely hinder the customerrsquos presentation of his case The central issue is whether the foundation of the house was properly constructed If the construction job was poorly done the customer can present evidence derived from inspection of the premises to prove that point The customer can also depose witnesses about any issues that arose during construction

Under the circumstances a court is not likely to impose particularly severe sanctions although it might shift the burden to the builder to show that the foundation was properly constructed or it might require the builder to reimburse any expenses the customer incurs to discover and prove the facts about issues or disputes that arose during construction of the foundation

[NOTE The result reached by the examinee is less important than the examineersquos recognition that (a) a range of sanctions is available to the court and (b) the appropriate sanction depends both on the culpability of the builder and the prejudice suffered by the customer]

22

CRIMINAL LAW AND PROCEDURE ANALYSIS (Criminal Law and Procedure IIA amp D VE amp F)

ANALYSIS

Legal Problems

(1) Did charging the defendant with both theft and burglary constitute double jeopardy

(2) Did the jury instruction violate the due process clause either by relieving the prosecution of the burden of proving the element of intent or by shifting the burden to the defendant to disprove that element

(3) Did the sentence imposed in this case for the theft conviction unconstitutionally deprive the defendant of his right to a jury trial on the issue of the value of the stolen item

DISCUSSION

Summary

The trial court properly denied the defendantrsquos pretrial motion to dismiss the charges on double jeopardy grounds The defendant may be charged with and convicted of both theft and burglary Each of the charges has an element that the other does not Neither charge is a lesser-included offense nor are they multiplicitous Thus charging both theft and burglary does not violate double jeopardy

The jury instruction on the burglary charge was constitutionally flawed It could have been reasonably understood by the jury as either (1) an irrebuttable conclusive presumption (which relieved the prosecution of proving the element of intent and removed the issue from the jury) or (2) a rebuttable mandatory presumption (which unconstitutionally shifted the burden of proof on an element of a charged offense from the prosecution to the defendant)

Because the four-year sentence imposed by the judge was based on the judgersquos finding by a preponderance of the evidence that the value of the stolen ring exceeded $5000 the sentence violates the defendantrsquos right to a jury determination beyond a reasonable doubt of the value of the ring

Point One (30) Charging the defendant with theft and burglary did not constitute double jeopardy

The Double Jeopardy Clause of the Fifth Amendment provides that a person shall not be twice put in jeopardy for the ldquosame offenserdquo Thus the question is whether the elements of the theft charge are wholly contained in the burglary charge or vice versa If the elements of the lesser charge (theft) are not wholly contained in the greater charge (burglary)mdashie if each charge requires proof of a fact that the other does notmdashthen convicting the defendant of both crimes would not violate double jeopardy even when the two offenses occurred at the same time and are thus arguably part of the ldquosame transactionrdquo Blockburger v United States 284 US 299 304 (1932) See also Albernaz v United States 450 US 333 344 n3 (1981) United States v Dixon 509 US 688 704 (1993)

23

Criminal Law and Procedure Analysis

Here theft and burglary each require proof of an element not required for the other crime Burglary may be defined differently in different jurisdictions However it almost invariably requires entry into a building or dwelling of another with the specific intent to commit a felony therein and the crime of burglary is complete upon the entry into the building or dwelling with such intent See eg Cannon v Oklahoma 827 P2d 1339 1342 (Okla Crim App 1992) In contrast theft which also may be defined differently in different states almost invariably requires the taking and carrying away of an item of personal property belonging to another with the intent to steal or permanently deprive the owner of possession

Here the ldquotakingrdquo or ldquostealingrdquo element is not contained in the definition of burglary and the ldquoentryrdquo element of burglary is not contained in the definition of theft Because theft is not a lesser-included offense of burglary and burglary is not a lesser-included offense of theft charging the defendant for both burglary and theft did not violate double jeopardy and the court properly denied the defense motion on those grounds Yparrea v Dorsey 64 F3d 577 579ndash80 (10th Cir 1995) citing Blockburger 284 US at 304

Finally the defendantrsquos motion to dismiss all the charges on double jeopardy grounds was improper because if both charges were for the same offense the motion should have requested dismissal of one charge not both

Point Two (35) The jury instruction on the burglary charge violated the Due Process Clause because it created either (1) an irrebuttable conclusive presumption (which relieved the prosecution of proving the element of intent and removed that issue from the jury) or (2) a rebuttable mandatory presumption (which unconstitutionally shifted the burden of proof on an element of a charged offense to the defendant)

The Supreme Court has interpreted the Due Process Clause of the US Constitution to require that the prosecution prove all elements of an offense beyond a reasonable doubt See In re Winship 397 US 358 364 (1970) The burden of proof cannot be shifted to the defendant by presuming an essential element upon proof of other elements of the offense because shifting the burden of persuasion with respect to any element of a criminal offense is contrary to the Due Process Clause See Mullaney v Wilbur 421 US 684 (1975)

The crime of burglary includes entry into a building or dwelling with the specific intent to commit a felony therein The requirement that the prosecutor prove beyond a reasonable doubt that the defendant had this specific intent distinguishes burglary from general-intent crimes like trespass See Sandstrom v Montana 442 US 510 523 (1979)

Here the jury was instructed that if ldquoafter consideration of all the evidence presented by the prosecution and defense you find beyond a reasonable doubt that the defendant entered the dwelling without the ownersrsquo consent you may presume that the defendant entered with the intent to commit a felony thereinrdquo This instruction was unconstitutional because it created either an irrebuttable conclusive presumption or a rebuttable mandatory presumption

A conclusive presumption is ldquoan irrebuttable direction by the court to find intent once convinced of the facts triggering the presumptionrdquo Id at 517 Here the jurors were instructed that once the prosecutor established that the defendant entered the neighborsrsquo house without consent they ldquomay presumerdquo that he intended to commit a felony therein The jurors may have reasonably concluded from this instruction that if they found that the defendant intended to enter his neighborsrsquo home without permission they must further find that he entered with the specific intent to commit a felony therein Because this instruction could operate as a conclusive

24

Criminal Law and Procedure Analysis

irrebuttable presumption by eliminating intent ldquoas an ingredient of the offenserdquo it violated due process by relieving the prosecution of the burden of proof for this element Id at 522

In the alternative the jury instruction could have been reasonably understood to create a rebuttable mandatory presumption which ldquotells [the jury] they must find the elemental fact upon proof of the basic fact at least unless the defendant has come forward with some evidence to rebut the presumed connection between the two factsrdquo County Court of Ulster County New York v Allen 442 US 140 157 (1979) The due process problem created by rebuttable mandatory presumptions is that ldquo[t]o the extent that the trier of fact is forced to abide by the presumption and may not reject it based on an independent evaluation of the particular facts presented by the State the analysis of the presumptionrsquos constitutional validity is logically divorced from those facts and based on the presumptionrsquos accuracy in the run of casesrdquo Id at 159

Unlike irrebuttable conclusive presumptions rebuttable mandatory presumptions are not always per se violations of the Due Process Clause However the Supreme Court of the United States has held that jury instructions that could reasonably be understood as shifting the burden of proof to the defendant on an element of the offense are unconstitutional Francis v Franklin 471 US 307 (1985) Here the argument that the jury instruction operated as a rebuttable mandatory presumption is supported by the fact that the judge also instructed the jury to ldquoconsider[ ] all the evidence presented by the prosecution and defenserdquo However even if the instruction created a rebuttable mandatory presumption it would be unconstitutional because it shifted the burden to the defense on an element of the offense Sandstrom 442 US at 524 Mullaney 421 US at 686

[NOTE Whether an examinee identifies the jury instruction as containing a ldquoconclusiverdquo or ldquomandatoryrdquo presumption is less important than the examineersquos analysis of the constitutional infirmities]

Point Three (35) The trial court violated the defendantrsquos Sixth Amendment right to a jury trial on an essential element of the offense when it found by a preponderance of the evidence that the ring was worth over $5000 and increased the defendantrsquos sentence based on this finding

In the statutory scheme under which the defendant was tried and convicted a Class D felony theft is defined as theft of item(s) with a value between $2500 and $10000 The jury found that the value of the diamond ring was at least $2500 and convicted the defendant of felony theft However at sentencing the trial court made a separate finding by a preponderance of the evidence that the value of the ring was greater than $5000 Following the statutersquos two-tiered sentencing scheme the judge then imposed on the defendant a sentence that was one year longer than the maximum that would otherwise have been allowed

The judgersquos sentence was unconstitutional because it violated the defendantrsquos Sixth Amendment right to a jury trial on this question The Supreme Court held in Apprendi v New Jersey 530 US 466 (2000) that ldquo[o]ther than the fact of a prior conviction any fact that increases the penalty for a crime beyond the prescribed statutory maximum must be submitted to a jury and proved beyond a reasonable doubtrdquo because ldquo[i]t is unconstitutional for a legislature to remove from the jury the assessment of facts that increase the prescribed range of penalties to which a criminal defendant is exposed [because] such facts must be established by proof beyond a reasonable doubtrdquo Id The Court reaffirmed Apprendi in Blakely v Washington 542 US 296 (2004) holding that the ldquolsquostatutory maximumrsquo for Apprendi purposes is the maximum sentence a judge may impose solely on the basis of the facts reflected in the jury verdict or admitted by the defendantrdquo Id at 303 (emphasis in original) In United States v Booker 543 US 220 (2005)

25

Criminal Law and Procedure Analysis

the Court relied on Blakely and Apprendi to conclude that protecting a defendantrsquos Sixth Amendment right to a jury trial required that ldquo[a]ny fact which is necessary to support a sentence exceeding the maximum authorized by the facts established by a plea of guilty or a jury verdict must be admitted by the defendant or proved to a jury beyond a reasonable doubtrdquo Id at 244

Thus in order to constitutionally increase a sentence above the statutory maximum of three years the jury must have found beyond a reasonable doubt that the value of the ring exceeded $5000 Here the court made the finding based on an appraisal proffered by the prosecutor only at sentencing and the judgersquos finding was by a preponderance of the evidence rather than beyond a reasonable doubt

26

AGENCY AND PARTNERSHIP ANALYSIS __________ (Agency and Partnership VA amp C VI)

ANALYSIS

Legal Problems

(1) Is a partner in a general partnership personally liable on a claim arising from misrepresentations by another partner made in the course of the partnership business

(2) Does a newly admitted partner in a general partnership become personally liable on existing claims against the partnership

(3) After the filing by a general partnership of a statement of qualification as a limited liability partnership are the partners personally liable as partners on (a) an existing claim against the general partnership and (b) a claim against the partnership that arose after the filing

DISCUSSION

Summary

Adam and Ben formed a general partnership under which they were jointly and severally liable for obligations of the partnership Thus Adam was personally liable for misrepresentations by Ben made in the ordinary course of the partnership business

Upon joining the general partnership Diane became personally liable for the obligations of the partnership arising after her admission but not for obligations pre-existing her admission such as the collectorrsquos claim

By filing a statement of qualification the three partners properly elected limited liability partnership status As partners in an LLP none of the three partners is personally liable as a partner for partnership obligations arising after the election such as the claim by the driverrsquos estate The election however does not change their personal liability on pre-existing claims that arose before the election such as the collectorrsquos claim

Point One (30) As a general partner of Empire a general partnership Adam became personally liable on the collectorrsquos claim a valid claim against the partnership that arose because of Benrsquos wrongful act in the ordinary course of the partnership business

When the collectorrsquos claim arose Empire was a general partnership composed of Adam and Ben Under UPA (1997) sect 306(a) partners of a general partnership are liable jointly and severally for all obligations of the partnership Under UPA (1997) sect 305(a) the partnership could become obligated for the loss caused to the collector as a result of the misrepresentation by Ben provided he was acting in the ordinary course of the partnership business Because there was no statement that limited his partnership authority Ben as partner was ldquoan agent of the partnership for the purpose of its businessrdquo See UPA (1997) sect 301(1) Benrsquos misrepresentation to the collector even if intentional appears to be in the ordinary course of the partnershiprsquos business of dealing

27

Agency and Partnership Analysis

in antique cars Thus Benrsquos wrongful act created a partnership obligation for which Adam was jointly and severally liable

[NOTE Generally a partnership creditor must ldquoexhaust the partnershiprsquos assets before levying on a judgment debtor partnerrsquos individual property where the partner is personally liable for the partnership obligationrdquo as a result of his status as a partner UPA (1997) sect 307 cmt 4 As the UPA comments explain this places Adam more in the position of guarantor than principal debtor on the partnership obligation Id cmt 4 Although an examinee might discuss this point the call focuses on whether Adam is personally liable not how the liability might be enforced]

Point Two (30) Because the collectorrsquos claim arose before Diane joined Empire Diane did not become personally liable on the claim

Diane was admitted to Empire when it was a general partnership and after the collectorrsquos claim arose While the general rule under UPA (1997) sect 306(a) is that the partners of a general partnership are liable jointly and severally for all obligations of the partnership there is a special rule for partners who are admitted during the duration of the partnership Under UPA (1997) sect 306(b) a person admitted to an existing partnership is not personally liable for any partnership obligations incurred before the personrsquos admission Because Diane was admitted to Empire after the collectorrsquos claim arose Diane is not personally liable on the claim

Dianersquos knowledge of the pre-existing claim and her stated concern about becoming liable on the collectorrsquos claim do not change her personal nonliability to the collector Although partners who have a liability shield can assume liability to third parties through private contractual guarantees or modifications to the partnership agreement Dianersquos stated concern constituted neither a guaranty to the collector nor ldquoan intentional waiver of liability protectionsrdquo See UPA (1997) sect 306 cmt 3 (describing methods for waiver of liability protections under sect 306(c) applicable in limited liability partnerships)

At most Diane will lose her investment in the partnership as a result of the collectorrsquos claim Although Diane did not become personally liable on the collectorrsquos claim when she joined the partnership the $250000 she contributed to the partnership is ldquoat risk for the satisfaction of existing partnership debtsrdquo UPA (1997) sect 306 cmt 2

Point Three (40) Filing the statement of qualification was effective to elect limited liability partnership status Despite this new status Adam and Ben remain personally liable on the collectorrsquos claim which arose before the election But as partners in an LLP neither Adam Ben nor Diane is personally liable as a partner on the driverrsquos estatersquos claim which arose after the election

Under UPA (1997) sect 1001 a general partnership can make an election and become a limited liability partnershipmdashif the partners approve the conversion by a vote equivalent to that necessary to amend the partnership agreement and the partnership then files a statement of qualification that specifies the name of the partnership its principal office and its election to be an LLP Here the partners agreed unanimouslymdashsufficient to amend their agreement under UPA (1997) sect 401(j)mdashand the statement of qualification was filed In addition the name of Empire LLP properly included an appropriate ending ldquoLLPrdquo See UPA (1997) sect 1002

Although another way to effectuate a ldquoconversionrdquo (as suggested by Benrsquos lawyer) is to form a new LLP and transfer the assets of the old general partnership to the new LLP the

28

Agency and Partnership Analysis

method used here (approval by the partners and the filing of a statement of qualification) is also sufficient to create LLP status

Thus Empire became Empire LLP as of the date of filing of the statement of qualification See UPA (1997) sect 1001 What effect did this have on the collectorrsquos claim which predated the filing According to UPA (1997) sect 306(c) an obligation incurred while a partnership is an LLP is solely a partnership obligation As the collectorrsquos claim predated the LLP Adam and Ben remain personally liable on the collectorrsquos claim Diane on the other hand was not personally liable on the collectorrsquos claim either before or after the filing of the statement of qualification See Point Two above

The driverrsquos estatersquos claim arose after Empire became Empire LLP Under UPA (1997) sect 306(c) an obligation incurred while a partnership is an LLP is solely a partnership obligationThus Adam Ben and Diane as partners are all protected from personal liability on the driverrsquos estatersquos claim But there may be personal liability if any of them was negligent or otherwise acted wrongfully by not informing the buyer of the bad suspension that caused the accident

29

National Conference of Bar Examiners 302 South Bedford Street | Madison WI 53703-3622 Phone 608-280-8550 | Fax 608-280-8552 | TDD 608-661-1275

wwwncbexorg e-mail contactncbexorg

  • Preface
  • Description of the MEE
  • Instructions
  • February 2014 Questions
    • Constitutinal Law Question
    • Trusts and Future Interests Question
    • Secured Transactions Question
    • Federal Civil Procedure Question
    • Criminal Law and Procedure Question
    • Agency and Partnership Question
      • February 2014 Analyses
        • Constitutional Law Analysis
        • Trust and Future Interests Analysis
        • Secured Transactions Analysis
        • Federal Civil Procedure Analysis
        • Criminal Law and Procedure Analysis
        • Agency and Partnership Analysis
            • ltlt13 ASCII85EncodePages false13 AllowTransparency false13 AutoPositionEPSFiles true13 AutoRotatePages None13 Binding Left13 CalGrayProfile (Dot Gain 20)13 CalRGBProfile (sRGB IEC61966-21)13 CalCMYKProfile (US Web Coated 050SWOP051 v2)13 sRGBProfile (sRGB IEC61966-21)13 CannotEmbedFontPolicy Error13 CompatibilityLevel 1413 CompressObjects Tags13 CompressPages true13 ConvertImagesToIndexed true13 PassThroughJPEGImages true13 CreateJobTicket false13 DefaultRenderingIntent Default13 DetectBlends true13 DetectCurves 0000013 ColorConversionStrategy CMYK13 DoThumbnails false13 EmbedAllFonts true13 EmbedOpenType false13 ParseICCProfilesInComments true13 EmbedJobOptions true13 DSCReportingLevel 013 EmitDSCWarnings false13 EndPage -113 ImageMemory 104857613 LockDistillerParams false13 MaxSubsetPct 10013 Optimize true13 OPM 113 ParseDSCComments true13 ParseDSCCommentsForDocInfo true13 PreserveCopyPage true13 PreserveDICMYKValues true13 PreserveEPSInfo true13 PreserveFlatness true13 PreserveHalftoneInfo false13 PreserveOPIComments true13 PreserveOverprintSettings true13 StartPage 113 SubsetFonts true13 TransferFunctionInfo Apply13 UCRandBGInfo Preserve13 UsePrologue false13 ColorSettingsFile ()13 AlwaysEmbed [ true13 ]13 NeverEmbed [ true13 ]13 AntiAliasColorImages false13 CropColorImages true13 ColorImageMinResolution 30013 ColorImageMinResolutionPolicy OK13 DownsampleColorImages true13 ColorImageDownsampleType Bicubic13 ColorImageResolution 30013 ColorImageDepth -113 ColorImageMinDownsampleDepth 113 ColorImageDownsampleThreshold 15000013 EncodeColorImages true13 ColorImageFilter DCTEncode13 AutoFilterColorImages true13 ColorImageAutoFilterStrategy JPEG13 ColorACSImageDict ltlt13 QFactor 01513 HSamples [1 1 1 1] VSamples [1 1 1 1]13 gtgt13 ColorImageDict ltlt13 QFactor 01513 HSamples [1 1 1 1] VSamples [1 1 1 1]13 gtgt13 JPEG2000ColorACSImageDict ltlt13 TileWidth 25613 TileHeight 25613 Quality 3013 gtgt13 JPEG2000ColorImageDict ltlt13 TileWidth 25613 TileHeight 25613 Quality 3013 gtgt13 AntiAliasGrayImages false13 CropGrayImages true13 GrayImageMinResolution 30013 GrayImageMinResolutionPolicy OK13 DownsampleGrayImages true13 GrayImageDownsampleType Bicubic13 GrayImageResolution 30013 GrayImageDepth -113 GrayImageMinDownsampleDepth 213 GrayImageDownsampleThreshold 15000013 EncodeGrayImages true13 GrayImageFilter DCTEncode13 AutoFilterGrayImages true13 GrayImageAutoFilterStrategy JPEG13 GrayACSImageDict ltlt13 QFactor 01513 HSamples [1 1 1 1] VSamples [1 1 1 1]13 gtgt13 GrayImageDict ltlt13 QFactor 01513 HSamples [1 1 1 1] VSamples [1 1 1 1]13 gtgt13 JPEG2000GrayACSImageDict ltlt13 TileWidth 25613 TileHeight 25613 Quality 3013 gtgt13 JPEG2000GrayImageDict ltlt13 TileWidth 25613 TileHeight 25613 Quality 3013 gtgt13 AntiAliasMonoImages false13 CropMonoImages true13 MonoImageMinResolution 120013 MonoImageMinResolutionPolicy OK13 DownsampleMonoImages true13 MonoImageDownsampleType Bicubic13 MonoImageResolution 120013 MonoImageDepth -113 MonoImageDownsampleThreshold 15000013 EncodeMonoImages true13 MonoImageFilter CCITTFaxEncode13 MonoImageDict ltlt13 K -113 gtgt13 AllowPSXObjects false13 CheckCompliance [13 None13 ]13 PDFX1aCheck false13 PDFX3Check false13 PDFXCompliantPDFOnly false13 PDFXNoTrimBoxError true13 PDFXTrimBoxToMediaBoxOffset [13 00000013 00000013 00000013 00000013 ]13 PDFXSetBleedBoxToMediaBox true13 PDFXBleedBoxToTrimBoxOffset [13 00000013 00000013 00000013 00000013 ]13 PDFXOutputIntentProfile ()13 PDFXOutputConditionIdentifier ()13 PDFXOutputCondition ()13 PDFXRegistryName ()13 PDFXTrapped False1313 CreateJDFFile false13 Description ltlt13 ARA 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 BGR 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 CHS ltFEFF4f7f75288fd94e9b8bbe5b9a521b5efa7684002000410064006f006200650020005000440046002065876863900275284e8e9ad88d2891cf76845370524d53705237300260a853ef4ee54f7f75280020004100630072006f0062006100740020548c002000410064006f00620065002000520065006100640065007200200035002e003000204ee553ca66f49ad87248672c676562535f00521b5efa768400200050004400460020658768633002gt13 CHT ltFEFF4f7f752890194e9b8a2d7f6e5efa7acb7684002000410064006f006200650020005000440046002065874ef69069752865bc9ad854c18cea76845370524d5370523786557406300260a853ef4ee54f7f75280020004100630072006f0062006100740020548c002000410064006f00620065002000520065006100640065007200200035002e003000204ee553ca66f49ad87248672c4f86958b555f5df25efa7acb76840020005000440046002065874ef63002gt13 CZE 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 DAN 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 DEU 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 ESP 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 ETI 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 FRA ltFEFF005500740069006c006900730065007a00200063006500730020006f007000740069006f006e00730020006100660069006e00200064006500200063007200e900650072002000640065007300200064006f00630075006d0065006e00740073002000410064006f00620065002000500044004600200070006f0075007200200075006e00650020007100750061006c0069007400e90020006400270069006d007000720065007300730069006f006e00200070007200e9007000720065007300730065002e0020004c0065007300200064006f00630075006d0065006e00740073002000500044004600200063007200e900e90073002000700065007500760065006e0074002000ea0074007200650020006f007500760065007200740073002000640061006e00730020004100630072006f006200610074002c002000610069006e00730069002000710075002700410064006f00620065002000520065006100640065007200200035002e0030002000650074002000760065007200730069006f006e007300200075006c007400e90072006900650075007200650073002egt13 GRE 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 HEB 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 HRV (Za stvaranje Adobe PDF dokumenata najpogodnijih za visokokvalitetni ispis prije tiskanja koristite ove postavke Stvoreni PDF dokumenti mogu se otvoriti Acrobat i Adobe Reader 50 i kasnijim verzijama)13 HUN 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 ITA 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 JPN ltFEFF9ad854c18cea306a30d730ea30d730ec30b951fa529b7528002000410064006f0062006500200050004400460020658766f8306e4f5c6210306b4f7f75283057307e305930023053306e8a2d5b9a30674f5c62103055308c305f0020005000440046002030d530a130a430eb306f3001004100630072006f0062006100740020304a30883073002000410064006f00620065002000520065006100640065007200200035002e003000204ee5964d3067958b304f30533068304c3067304d307e305930023053306e8a2d5b9a306b306f30d530a930f330c8306e57cb30818fbc307f304c5fc59808306730593002gt13 KOR ltFEFFc7740020c124c815c7440020c0acc6a9d558c5ec0020ace0d488c9c80020c2dcd5d80020c778c1c4c5d00020ac00c7a50020c801d569d55c002000410064006f0062006500200050004400460020bb38c11cb97c0020c791c131d569b2c8b2e4002e0020c774b807ac8c0020c791c131b41c00200050004400460020bb38c11cb2940020004100630072006f0062006100740020bc0f002000410064006f00620065002000520065006100640065007200200035002e00300020c774c0c1c5d0c11c0020c5f40020c2180020c788c2b5b2c8b2e4002egt13 LTH ltFEFF004e006100750064006f006b0069007400650020016100690075006f007300200070006100720061006d006500740072007500730020006e006f0072011700640061006d00690020006b0075007200740069002000410064006f00620065002000500044004600200064006f006b0075006d0065006e007400750073002c0020006b00750072006900650020006c0061006200690061007500730069006100690020007000720069007400610069006b007900740069002000610075006b01610074006f00730020006b006f006b007900620117007300200070006100720065006e006700740069006e00690061006d00200073007000610075007300640069006e0069006d00750069002e0020002000530075006b0075007200740069002000500044004600200064006f006b0075006d0065006e007400610069002000670061006c006900200062016b007400690020006100740069006400610072006f006d00690020004100630072006f006200610074002000690072002000410064006f00620065002000520065006100640065007200200035002e0030002000610072002000760117006c00650073006e0117006d00690073002000760065007200730069006a006f006d00690073002egt13 LVI 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 NLD (Gebruik deze instellingen om Adobe PDF-documenten te maken die zijn geoptimaliseerd voor prepress-afdrukken van hoge kwaliteit De gemaakte PDF-documenten kunnen worden geopend met Acrobat en Adobe Reader 50 en hoger)13 NOR 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 POL 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 PTB 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 RUM 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 RUS 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 SKY ltFEFF0054006900650074006f0020006e006100730074006100760065006e0069006100200070006f0075017e0069007400650020006e00610020007600790074007600e100720061006e0069006500200064006f006b0075006d0065006e0074006f0076002000410064006f006200650020005000440046002c0020006b0074006f007200e90020007300610020006e0061006a006c0065007001610069006500200068006f0064006900610020006e00610020006b00760061006c00690074006e00fa00200074006c0061010d00200061002000700072006500700072006500730073002e00200056007900740076006f00720065006e00e900200064006f006b0075006d0065006e007400790020005000440046002000620075006400650020006d006f017e006e00e90020006f00740076006f00720069016500200076002000700072006f006700720061006d006f006300680020004100630072006f00620061007400200061002000410064006f00620065002000520065006100640065007200200035002e0030002000610020006e006f0076016100ed00630068002egt13 SLV 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 SUO 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 SVE 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 TUR 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 UKR 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 ENU (Use these settings to create Adobe PDF documents best suited for high-quality prepress printing Created PDF documents can be opened with Acrobat and Adobe Reader 50 and later)13 gtgt13 Namespace [13 (Adobe)13 (Common)13 (10)13 ]13 OtherNamespaces [13 ltlt13 AsReaderSpreads false13 CropImagesToFrames true13 ErrorControl WarnAndContinue13 FlattenerIgnoreSpreadOverrides false13 IncludeGuidesGrids false13 IncludeNonPrinting false13 IncludeSlug false13 Namespace [13 (Adobe)13 (InDesign)13 (40)13 ]13 OmitPlacedBitmaps false13 OmitPlacedEPS false13 OmitPlacedPDF false13 SimulateOverprint Legacy13 gtgt13 ltlt13 AddBleedMarks false13 AddColorBars false13 AddCropMarks false13 AddPageInfo false13 AddRegMarks false13 ConvertColors ConvertToCMYK13 DestinationProfileName ()13 DestinationProfileSelector DocumentCMYK13 Downsample16BitImages true13 FlattenerPreset ltlt13 PresetSelector MediumResolution13 gtgt13 FormElements false13 GenerateStructure false13 IncludeBookmarks false13 IncludeHyperlinks false13 IncludeInteractive false13 IncludeLayers false13 IncludeProfiles false13 MultimediaHandling UseObjectSettings13 Namespace [13 (Adobe)13 (CreativeSuite)13 (20)13 ]13 PDFXOutputIntentProfileSelector DocumentCMYK13 PreserveEditing true13 UntaggedCMYKHandling LeaveUntagged13 UntaggedRGBHandling UseDocumentProfile13 UseDocumentBleed false13 gtgt13 ]13gtgt setdistillerparams13ltlt13 HWResolution [2400 2400]13 PageSize [612000 792000]13gtgt setpagedevice13

Page 15: February 2014 MEE Questions and AnalysesPreface The Multistate Essay Examination (MEE) is developed by the National Conference of Bar Examiners (NCBE). This publication includes the

Constitutional Law Analysis

First the ordinance requirement likely has a minimal economic impact on the restaurant Compliance with the ordinance is estimated to cost $1000 and the city has found that businesses will likely recoup that cost in increased sales Also because the ordinance does not interfere with the operation of the restaurant the owner may still earn a reasonable return on its investment in the property

Second the ordinance does not interfere with the businessrsquos investment-backed expectations As in Penn Central the challenged law does not interfere with the ownerrsquos ldquoprimary expectationrdquo for use of the propertymdashin Penn Central as a railroad terminal and here as a restaurant Further the ordinance does not prevent the restaurant from expanding to meet the changing business environment

Third the character of the government action does not weigh in favor of a taking While Penn Central does say that a ldquophysical invasionrdquo is more likely to pose a taking Loretto suggests that the Courtrsquos main concern is with physical invasions by third parties Also like the landmark law challenged in Penn Central the ordinance here ldquoadjust[s] the benefits and burdens of economic life to promote the common goodrdquo Id In Penn Central the landmark law restricted development of the railroad terminal to promote the common interest in preserving historic landmarks Here the ordinance requires the businesses to install floodlights to promote the common interest in crime prevention and public safety

Because the ordinance is clearly a valid exercise of the police power it satisfies the takings clausersquos public-use requirement Kelo v City of New London 545 US 469 (2005)

In sum all three factors weigh against finding a taking under the Penn Central balancing test

Point Two (50) The permit condition may be unconstitutional as an uncompensated taking of property because the city has not made an individualized determination that the easement condition is roughly proportional to the impact of the restaurantrsquos proposed addition

In Dolan v City of Tigard 512 US 374 (1994) the Supreme Court set forth the test for determining whether an exaction imposed by a government in exchange for a discretionary benefit conferred by the government such as a condition on the approval of a building permit in this case constitutes an uncompensated taking under the Fifth Amendment The exaction is not a taking if (1) there is an ldquoessential nexusrdquo between the ldquopublic need or burdenrdquo to which the proposed development contributes and ldquothe permit condition exacted by the cityrdquo id at 386 and (2) the government makes ldquosome sort of individualized determination that the required dedication is [roughly proportional] both in nature and extent to the impact of the proposed developmentrdquo Id at 391 see also Nollan v California Coastal Commission 483 US 825 (1987)

Here the city likely can meet the nexus requirement In Dolan the landowner sought to double the size of its business which would have increased traffic on nearby roadways In exchange for approving the development the city sought an easement for a bike and pedestrian path The Court found the required nexus between the easement and the cityrsquos ldquoattempt to reduce traffic congestion by providing for alternative means of transportationrdquo 512 US at 387 Here a similar nexus likely exists between the requested easement and the cityrsquos interest in crime prevention and public safety Increased patronage and economic activity at the restaurant might attract additional crime to the area and the requested easement to install surveillance equipment would attempt to address that increased crime

12

Constitutional Law Analysis

The exaction here however may fail the second prong of the Dolan testmdashthat the exaction be roughly proportional to the anticipated impact of the requested development As noted the city in Dolan claimed that a bike and pedestrian path was needed to offset the increase in traffic due to the proposed doubling of the business The Court explained that the government must demonstrate that the additional traffic reasonably was related to the requested exaction and that the government must ldquomake some effort to quantify its findings in support of the dedication for the pedestrianbicycle pathway beyond the conclusory statement that it could offset some of the traffic demand generatedrdquo Id at 395 Here the city did not carry its burden The city simply speculates that increased patronage of the restaurant ldquomightrdquo increase crime and that the surveillance equipment ldquomightrdquo alleviate this increased crime Because the city has not made ldquosome effort to quantify its findingsrdquo in support of the easement it has not shown that the burden of the easement is roughly proportional to the benefits thought to flow from it

Thus the exaction appears to be an uncompensated taking of property in violation of the Fifth Amendment as applied to the states through the Fourteenth Amendment

13

TRUSTS AND FUTURE INTERESTS ANALYSIS ____ (Trusts and Future Interests IE3 I5 IIIA amp B)

ANALYSIS

Legal Problems

(1) How should rents dividends and sales proceeds received by the trustee prior to receipt of the sonrsquos letter have been allocated between trust income and principal

(2)(a) Did the remainder interest in the trust accelerate and become immediately payable to the daughterrsquos minor child upon the trusteersquos receipt of the sonrsquos letter and if not how should the trustee handle the distribution of the principal in the future

(2)(b) Following the trusteersquos receipt of the sonrsquos letter how should the trustee distribute future receipts of income prior to the distribution of the principal

DISCUSSION

Summary

Prior to the trusteersquos receipt of the sonrsquos letter cash dividends and rents should have been allocated to trust income and were distributable to the son the income beneficiary of the trust sales proceeds and stock dividends should have been allocated to principal

Because the sonrsquos letter to the trustee did not result in a valid disclaimer under state law (having been made more than nine months after the testatorrsquos death) the son is not deemed to have predeceased the testator Because the son is still living the class gift to the testatorrsquos grandchildren who survive the son has not closed and is not possessory it will not become possessory until the son dies The daughterrsquos minor child being the testatorrsquos only living grandchild is not currently entitled to a distribution of trust principal Trust principal will instead be distributable upon the sonrsquos death to the testatorrsquos then-living grandchildren or if there are none to the testatorrsquos then-living heirs

As for future income the trustee should either distribute the trust income to the son and the daughter as the testatorrsquos heirs accumulate the income for future distribution to those individuals ultimately entitled to the trust principal or distribute it to those presumptively entitled to the principal upon the sonrsquos death ie the daughterrsquos minor child

Point One (45) Cash dividends and rents are allocable to income sales proceeds and stock dividends are allocable to principal Items allocable to income for the period prior to the sonrsquos attempted disclaimer were distributable to the son

Receipts earned during the administration of a trust are allocable either to income or to principal Almost all states have adopted the most recent or an earlier version of the Uniform Principal and Income Act (the Act) which specifies how such receipts should be allocated

Under the Act rents (UNIF PRIN amp INC ACT (2000) sect 405 UNIF PRIN amp INC ACT (1962) sect 3(a)(1)) and cash dividends received from a corporation (UNIF PRIN amp INC ACT (2000) sect 401(b) UNIF PRIN amp INC ACT (1962) sect 6(d)) are allocable to income and are distributable to the income beneficiary of the trust

14

Trusts and Future Interests Analysis Sales proceeds (UNIF PRIN amp INC ACT (2000) sect 404(2) UNIF PRIN amp INC ACT (1962)

sect 3(b)(1)) and dividends paid in the stock of the distributing corporation (UNIF PRIN amp INC ACT (2000) sect 401(c)(1) UNIF PRIN amp INC ACT (1962) sect 3(b)(4)) are allocable to principal and added to the principal of the trust

Here the cash dividends and office building rents should have been allocated to income and until the trustee received the sonrsquos letter should have been distributed to him as the sole income beneficiary of the trust The stock dividend and proceeds from the sale of the office building should have been allocated to principal and held by the trustee for future distribution to the ultimate remaindermen of the trust

[NOTE The 2000 Uniform Principal and Income Act has been adopted in Alabama Arkansas Colorado Connecticut the District of Columbia Hawaii Idaho Iowa Kentucky Missouri Montana Nebraska New Mexico North Dakota Oregon South Dakota Utah and West Virginia]

Point Two(a) (45) Because the son did not disclaim within nine months of the testatorrsquos death there is no valid disclaimer under state law Therefore the son is not deemed to have predeceased the testator Furthermore because of the express survivorship contingency in the will the remainder in the trust does not accelerate and become distributable until the son in fact dies When the son dies the trust principal will be distributable to the testatorrsquos then-living grandchildren or if none then to the testatorrsquos then-living heirs

When a trust remainder is given to a class the class closes (ie no new persons can join the class) when there is no outstanding income interest and at least one member of the class is then entitled to demand possession of his or her share of the remainder This principle is called the rule of convenience See generally HERBERT HOVENKAMP amp SHELDON F KURTZ PRINCIPLES OF PROPERTY LAW 199ndash200 (6th ed 2005) A class member may demand possession of his or her share of the remainder upon termination of the income interest only when the class memberrsquos interest is not otherwise subject to a condition precedent See id

When a beneficiary timely disclaims an interest in a trust that beneficiary is treated as if he had predeceased the testator Here had the son disclaimed within nine months of the testatorrsquos death as required by the state statute he would have been deemed to have predeceased the testator This would have closed the class of remaindermen and the testatorrsquos then-living grandchildren (ie the daughterrsquos child) would have been entitled to the trust principal However under the state statute the sonrsquos disclaimer was not timely because he did not disclaim within nine months of the testatorrsquos death Thus because the statute is inapplicable and the son is still alive the class of grandchildren entitled to share in trust principal did not close

Because here the statute is inapplicable due to the sonrsquos failure to comply with the statutory time requirements then presumably the common-law rule allowing disclaimers (aka renunciations) at any time should apply Under the common law if a life estate is renounced the remainder interest accelerates and becomes immediately distributable to the remaindermen of the trust if the remainder is vested but not if the remainder is contingent JESSE DUKEMINIER amp ROBERT H SITKOFF WILLS TRUSTS AND ESTATES 844ndash845 (9th ed 2013) Here because the remainder is contingent upon there being grandchildren who survive the son the remainder will not accelerate It will remain open until the son dies leaving open the possibility that additional grandchildren will be included in the class or the daughterrsquos child could fall out of the class because that child fails to survive the son

And if none of the testatorrsquos grandchildren survive the son the trust principal will be distributed to the testatorrsquos heirs living at the sonrsquos death

15

Trusts and Future Interests Analysis

Point Two(b) (10) Until the trust terminates the trustee must continue to hold the trust assets The distribution of income in the meantime is unclear There are at least three possibilities Income earned on the undistributed assets could be distributed to the son and daughter as the testatorrsquos heirs accumulated and added to principal for distribution to the ultimate remaindermen or distributed from time to time to those persons who are presumptively remaindermen

When trust principal is not immediately distributable the trustee must continue to hold trust assets until the ultimate remaindermen are ascertained During this period trust income will be distributed or retained according to any instructions contained in the trust instrument See WILLIAM M MCGOVERN JR SHELDON F KURTZ amp DAVID M ENGLISH WILLS TRUSTS amp ESTATES sect 102 (4th ed 2010)

Here the testator did not specify what the trustee should do with trust income in the event the sonrsquos disclaimer did not comply with the state statute There are at least three approaches One approach would have the trustee distribute the trust income to the testatorrsquos heirs on the theory that the income represents property that was not disposed of by the testatorrsquos will and which thus passes by partial intestacy to the testatorrsquos heirs A second approach would have the trustee accumulate trust income for distribution to the ultimate remaindermen Under this approach only those individuals ultimately entitled to the principal would share in the income A third approach would have the trustee distribute trust income to those individuals who would be the remaindermen if the trust were to terminate when the income is received by the trustee under this approach trust income would be distributed to the daughterrsquos minor child until another presumptive remainderman is born This approach could result in individuals not ultimately entitled to principal say because they do not survive the son receiving income It could also result in a disproportionate distribution of income among the individuals ultimately entitled to income

[NOTE Examinees should demonstrate a recognition and understanding of the income-allocation problem and the alternatives available to address that issue There is no widely accepted solution to the problem Examinees who cite any of these possible problem-solving approaches may receive credit]

16

SECURED TRANSACTIONS ANALYSIS (Secured Transactions IB IID E amp F IIIB IVA B amp F)

ANALYSIS

Legal Problems

(1)(a) What is the nature of the bankrsquos claim to the businessrsquos equipment

(1)(b) What is the nature of the finance companyrsquos claim to the businessrsquos equipment

(1)(c) As between the bank and the finance company whose claim to the businessrsquos equipment has priority

(2) Do the claims of the bank and the finance company continue in the item of equipment sold by the business to the competitor

DISCUSSION

Summary

The bank and the finance company both have perfected security interests in the businessrsquos equipment Even though the finance companyrsquos perfected security interest was created first the bankrsquos perfected security interest has priority because the bankrsquos financing statement was filed before the finance companyrsquos financing statement The security interests of the bank and the finance company continue in the item of equipment sold by the business to the competitor because their security interests were perfected and the competitor was not a buyer in ordinary course of business

Point One(a) (25) The bank has a perfected security interest in the businessrsquos equipment

The bank has met all criteria necessary for it to have an attached and enforceable security interest in the businessrsquos equipment First value must be given UCC sect 9-203(b)(1) This criterion is fulfilled by the loan made by the bank to the business Second the debtor must have rights in the collateral UCC sect 9-203(b)(2) Clearly the business has rights in its equipment Third either the secured party must take possession of the collateral or the debtor must authenticate a security agreement containing a description of the collateral UCC sect 9-203(b)(3) The agreement that the business owner signed is a ldquosecurity agreementrdquo because it is an agreement that creates or provides for a security interest UCC sect 9-102(a)(74) By signing the security agreement the business owner authenticated it UCC sect 9-102(a)(7) Therefore all three criteria are fulfilled and the bank has an enforceable and attached security interest

A security interest is perfected when it has attached and when any additional steps required for perfection have occurred UCC sect 9-308(a) Generally speaking the additional steps will either be possession of the collateral by the secured party or the filing of a financing statement with respect to the collateral See UCC sectsect 9-310 9-313 In this case the bank filed a financing statement naming the debtor and sufficiently indicating the collateral The collateral indication is sufficient because it identifies the collateral by type of property See UCC sectsect 9-504 9-108 The fact that the financing statement was filed before the security interest was created is

17

Secured Transactions Analysis

not a problem Even though the security agreement had not yet been signed the business had authorized the filing of the financing statement in an authenticated record UCC sect 9-509(a)(1) Moreover the financing statement may be filed before the security agreement is created UCC sect 9-502(d)

Point One(b) (10) The finance company also has a perfected security interest in the businessrsquos equipment

The finance companyrsquos security interest is enforceable and attached for the same reasons as the bankrsquos security interest The loan from the finance company to the business constitutes value the business has rights in the collateral and the business owner has authenticated a security agreement containing a description of the collateral The finance companyrsquos security interest is perfected because the finance company filed a financing statement with respect to it that provides that the business is the debtor and indicates that the collateral is equipment

Point One(c) (30) The bankrsquos security interest has priority over the finance companyrsquos security interest because the bankrsquos financing statement was filed first

As between two perfected security interests the general rule is that the security interest that was the earlier to be either perfected or the subject of a filed financing statement has priority UCC sect 9-322(a)(1) While the finance companyrsquos security interest was perfected before the bankrsquos (March 15 vs March 22) the bankrsquos financing statement was filed even earlier on March 2 Thus under the first-to-file-or-perfect rule of UCC sect 9-322(a)(1) the bankrsquos security interest has priority No exceptions to the general rule apply here

Point Two (35) A security interest in collateral continues notwithstanding its sale unless an exception applies Because the security interests of the bank and the finance company were perfected and the competitor was not a buyer in ordinary course of business no exception applies and the security interests of both creditors continue in the equipment sold to the competitor

As a general rule a security interest in collateral continues notwithstanding the fact that the debtor has sold the collateral to another person UCC sect 9-315(a)(1) Thus unless an exception applies the security interests of the bank and the finance company will continue in the item of equipment sold to the competitor

A buyer of goods will take free of an unperfected security interest in those goods See UCC sect 9-317(a)(2) However when the competitor bought the businessrsquos equipment both the bank and the finance company had perfected security interests in the equipment

A buyer can take free even of a perfected security interest in goods if the buyer is a ldquobuyer in ordinary course of businessrdquo See UCC sect 9-320(a) However the competitor was not a buyer in ordinary course of business To be a ldquobuyer in ordinary course of businessrdquo a buyer must buy goods from a seller that is in the business of selling goods of that kind See UCC sect 1-201(b)(9) The competitor bought this equipment from a seller that is not in the business of selling goods of this kind so the competitor was not a buyer in ordinary course of business with respect to these goods

Because no exception applies the security interests of the bank and the finance company continue even after the item of equipment was sold to the competitor

18

FEDERAL CIVIL PROCEDURE ANALYSIS (Federal Civil Procedure IVD)

ANALYSIS

Legal Problems

(1) Is a document prepared in the course of a contract dispute protected from discovery as ldquowork productrdquo when there is no evidence that the document was prepared in anticipation of litigation

(2)(a) Is a partyrsquos failure to provide relevant electronically stored information excused when the information was destroyed pursuant to a routine document retention scheme at a time when litigation was contemplated by the destroying party

(2)(b) What sanctions should be imposed on a party for allowing the destruction of evidence that is relevant to potential future litigation

DISCUSSION

Summary

The report prepared by the structural engineer is probably not work product and is thus discoverable The engineer examined the foundation of the house at the customerrsquos request and the engineerrsquos findings are potentially relevant to the customerrsquos claim that the foundation is defective The report was not prepared in anticipation of litigation The customer appears to have sought the engineerrsquos opinion in response to the builderrsquos offer to fix any problems with the foundation that an engineer might identify Because the report was not prepared in anticipation of litigation it is not protected by the work-product doctrine

The builder should have taken appropriate steps to preserve evidence including suspending its document retention program as soon as it began planning for litigationmdashie on July 10 Its destruction of potentially relevant material after that date was wrongful However a court is unlikely to impose severe sanctions on the builder because there are no facts indicating that the builder acted in bad faith and the customer can prove that the foundation is defective without the destroyed emails

Point One (40) The customer must turn over the engineerrsquos report because it was not prepared in anticipation of litigation

In general a party to a lawsuit in federal court ldquomay obtain discovery regarding any nonprivileged matter that is relevant to any partyrsquos claim or defenserdquo FED R CIV P 26(b)(1) (2009) This includes the right to inspect and copy documents in the other partyrsquos possession FED R CIV P 34(a)(1) Here the customer hired a structural engineer to examine the foundation of the house The engineerrsquos report on the foundation is likely to include information that would be relevant to the customerrsquos claim that the foundation was defectively constructed

The so-called ldquowork productrdquo rule allows a party to refuse to turn over ldquodocuments that are prepared in anticipation of litigation or for trialrdquo by that partyrsquos representative including

19

Federal Civil Procedure Analysis

a consultant Thus if the customer had hired the structural engineer to prepare a report ldquoin anticipation of litigationrdquo that report might not be discoverable See FED R CIV P 26(b)(3)

In this case however the customer hired the engineer to evaluate the foundation of the house as part of the customerrsquos negotiation with the builder concerning the housersquos flooding problem The builder told the customer that the housersquos landscaping was the reason for the flooding and the builder told the customer ldquoHave an engineer look at the foundation If therersquos a problem wersquoll fix itrdquo The customer appears to have acted in response to that statement There is no indication that the customer anticipated any kind of legal action at the time that the structural engineer was hired Accordingly the structural engineerrsquos report is discoverable and the court should order the customer to turn it over

[NOTE If an examinee concludes that the structural engineerrsquos report was prepared in anticipation of litigation then the examinee should also conclude that the report is not discoverable Documents prepared in anticipation of litigation do not need to be disclosed to an adverse party unless that party can demonstrate a ldquosubstantial needrdquo for the documents and an inability to obtain substantially equivalent information without ldquoundue hardshiprdquo FED R CIV P 26(b)(3)(A)(ii) Furthermore a report prepared by an expert who is not expected to testify is not discoverable in the absence of ldquoexceptional circumstancesrdquo making it ldquoimpracticablerdquo to obtain the information in another way FED R CIV P 26(b)(4)(D)(ii) The builder probably cannot make these showings here unless the engineerrsquos report deals with circumstances that have since changed There is no evidence that the structural engineer would have had access to any information or facts that the builder would not already know as a result of its construction and subsequent inspection of the house In addition if necessary the builder could ask the court for permission to arrange for a further inspection of the house by a structural engineer hired by the builder See FED R CIV P 34(a)(2) Accordingly if an examinee concludes that the report was prepared in anticipation of litigation the examinee should also conclude that the builder is not entitled to see the report]

Point Two(a) (30) Because the builder anticipated that it might be involved in litigation concerning its contract with the customer the builder acted wrongfully in destroying emails that were relevant to the housersquos construction even though the emails were destroyed pursuant to a routine document retention plan

As noted above a party to a lawsuit in federal court ldquomay obtain discovery regarding any nonprivileged matter that is relevant to any partyrsquos claim or defenserdquo FED R CIV P 26(b)(1) This includes emails and other electronically stored information FED R CIV P 34(a)(1)(A) Here the customer has requested all the builderrsquos emails pertaining to work done on the foundation of the house Ordinarily the builder would be obliged to turn over this information which is relevant to the customerrsquos defense that the housersquos foundation was poorly constructed

Unfortunately the emails in question no longer exist because the builder destroyed them on August 2

In general spoliation of evidence (destruction or alteration of evidence) is improper if the party who destroyed or altered the evidence ldquohas notice that the evidence is relevant to litigation or should have known that the evidence may be relevant to future litigationrdquo Fujitsu Ltd v Federal Express Corp 247 F3d 423 436 (2d Cir 2001) It is improper for a party to destroy electronic information relevant to pending litigation even if the destruction occurs before there is any request or order seeking the information See eg Leon v IDX Sys Corp 464 F3d 951 (9th Cir 2006) (plaintiffrsquos intentional destruction of computer files warranted dismissal even

20

In this case the builderrsquos destruction of the emails was pursuant to a routine document retention plan The Federal Rules provide expressly that in the absence of ldquoexceptional circumstancesrdquo parties should not be sanctioned for the loss of electronically stored information when the loss occurs pursuant to ldquoroutine good-faith operation of an electronic information systemrdquo FED R CIV P 37(e) However when a party anticipates litigation ldquoit must suspend its routine document retentiondestruction policy and put in place a lsquolitigation holdrsquo to ensure the preservation of relevant documentsrdquo Zubulake v UBS Warburg LLC 220 FRD 212 218 (SDNY 2003)

Federal Civil Procedure Analysis

though spoliation occurred before order compelling discovery) Similarly the duty to preserve evidence applies to a party who anticipates litigation even if litigation has not yet been commenced See THE SEDONA PRINCIPLES BEST PRACTICES RECOMMENDATIONS amp PRINCIPLES FOR ADDRESSING ELECTRONIC DOCUMENT PRODUCTION 70 cmt 14a (2d ed 2007)

The builder destroyed the emails on August 2 At that time the builder knew that litigation was a possibility because the builder had already directed its attorney to prepare a draft complaint for possible filing Knowing that litigation was a possibility the builder had a duty to take steps to preserve evidence including the emails in question See generally Fujitsu Ltd

Thus the builderrsquos destruction of potentially relevant emails at a time when it knew that litigation was a possibility was improper It had a duty to preserve evidence and it breached that duty

[NOTE Because courts have used different words to describe the test for when evidence must be preserved an examineersquos precise formulation of the test is not critical]

Point Two(b) (30) In determining appropriate sanctions for spoliation courts consider both the level of culpability of the spoliating party and the degree of prejudice the loss of evidence has caused the other party Here the builderrsquos destruction of evidence does not appear to have been willful nor is it likely to pose a significant obstacle to the customerrsquos defense Any sanctions imposed by the court should be modest

Federal courts have inherent power to control the litigation process and can sanction misbehavior including spoliation even when there has been no specific violation of the Federal Rules of Civil Procedure See generally Chambers v NASCO Inc 501 US 32 (1991) (discussing courtrsquos inherent power to control the litigation process) The range of available sanctions is broad It can include such sanctions as the payment of expenses incurred by the other party as a result of the destruction of the evidence an instruction to the jury authorizing it to draw an adverse inference from the destruction of the evidence a shifting of the burden of proof on the relevant issue or even judgment against the responsible party See eg Residential Funding Corp v DeGeorge Financial Corp 306 F3d 99 108 (2d Cir 2002) (adverse inference) Silvestri v General Motors Corp 271 F3d 583 593 (4th Cir 2001) (possibility of dismissal) Cf FED R CIV P 37(b)(2)(A) (listing remedies for failure to comply with discovery obligations)

In determining appropriate sanctions for spoliation courts consider both the level of culpability of the spoliating party and the degree of prejudice the loss of evidence has caused the other party Many courts impose severe sanctions (such as an adverse-inference instruction or the entry of judgment against the spoliating party) only when there is evidence of bad faith in the form of an intentional effort to hide information Eg Greyhound Lines Inc v Wade 485 F3d 1032 1035 (8th Cir 2007) (spoliation sanction requires intentional destruction out of desire ldquoto suppress the truthrdquo) However other courts have said that negligence in preserving evidence can

21

Federal Civil Procedure Analysis

support an adverse-inference instruction See Residential Funding 306 F3d at 108 (negligence enough under some circumstances)

Although a court might well order an evidentiary hearing on the issue of sanctions the facts presented do not seem appropriate for severe sanctions First the evidence was destroyed pursuant to the builderrsquos standard document retention plan and there is no evidence that the builder deliberately failed to suspend its usual procedures with the purpose of allowing the destruction of evidence Second the loss of this evidence will not severely hinder the customerrsquos presentation of his case The central issue is whether the foundation of the house was properly constructed If the construction job was poorly done the customer can present evidence derived from inspection of the premises to prove that point The customer can also depose witnesses about any issues that arose during construction

Under the circumstances a court is not likely to impose particularly severe sanctions although it might shift the burden to the builder to show that the foundation was properly constructed or it might require the builder to reimburse any expenses the customer incurs to discover and prove the facts about issues or disputes that arose during construction of the foundation

[NOTE The result reached by the examinee is less important than the examineersquos recognition that (a) a range of sanctions is available to the court and (b) the appropriate sanction depends both on the culpability of the builder and the prejudice suffered by the customer]

22

CRIMINAL LAW AND PROCEDURE ANALYSIS (Criminal Law and Procedure IIA amp D VE amp F)

ANALYSIS

Legal Problems

(1) Did charging the defendant with both theft and burglary constitute double jeopardy

(2) Did the jury instruction violate the due process clause either by relieving the prosecution of the burden of proving the element of intent or by shifting the burden to the defendant to disprove that element

(3) Did the sentence imposed in this case for the theft conviction unconstitutionally deprive the defendant of his right to a jury trial on the issue of the value of the stolen item

DISCUSSION

Summary

The trial court properly denied the defendantrsquos pretrial motion to dismiss the charges on double jeopardy grounds The defendant may be charged with and convicted of both theft and burglary Each of the charges has an element that the other does not Neither charge is a lesser-included offense nor are they multiplicitous Thus charging both theft and burglary does not violate double jeopardy

The jury instruction on the burglary charge was constitutionally flawed It could have been reasonably understood by the jury as either (1) an irrebuttable conclusive presumption (which relieved the prosecution of proving the element of intent and removed the issue from the jury) or (2) a rebuttable mandatory presumption (which unconstitutionally shifted the burden of proof on an element of a charged offense from the prosecution to the defendant)

Because the four-year sentence imposed by the judge was based on the judgersquos finding by a preponderance of the evidence that the value of the stolen ring exceeded $5000 the sentence violates the defendantrsquos right to a jury determination beyond a reasonable doubt of the value of the ring

Point One (30) Charging the defendant with theft and burglary did not constitute double jeopardy

The Double Jeopardy Clause of the Fifth Amendment provides that a person shall not be twice put in jeopardy for the ldquosame offenserdquo Thus the question is whether the elements of the theft charge are wholly contained in the burglary charge or vice versa If the elements of the lesser charge (theft) are not wholly contained in the greater charge (burglary)mdashie if each charge requires proof of a fact that the other does notmdashthen convicting the defendant of both crimes would not violate double jeopardy even when the two offenses occurred at the same time and are thus arguably part of the ldquosame transactionrdquo Blockburger v United States 284 US 299 304 (1932) See also Albernaz v United States 450 US 333 344 n3 (1981) United States v Dixon 509 US 688 704 (1993)

23

Criminal Law and Procedure Analysis

Here theft and burglary each require proof of an element not required for the other crime Burglary may be defined differently in different jurisdictions However it almost invariably requires entry into a building or dwelling of another with the specific intent to commit a felony therein and the crime of burglary is complete upon the entry into the building or dwelling with such intent See eg Cannon v Oklahoma 827 P2d 1339 1342 (Okla Crim App 1992) In contrast theft which also may be defined differently in different states almost invariably requires the taking and carrying away of an item of personal property belonging to another with the intent to steal or permanently deprive the owner of possession

Here the ldquotakingrdquo or ldquostealingrdquo element is not contained in the definition of burglary and the ldquoentryrdquo element of burglary is not contained in the definition of theft Because theft is not a lesser-included offense of burglary and burglary is not a lesser-included offense of theft charging the defendant for both burglary and theft did not violate double jeopardy and the court properly denied the defense motion on those grounds Yparrea v Dorsey 64 F3d 577 579ndash80 (10th Cir 1995) citing Blockburger 284 US at 304

Finally the defendantrsquos motion to dismiss all the charges on double jeopardy grounds was improper because if both charges were for the same offense the motion should have requested dismissal of one charge not both

Point Two (35) The jury instruction on the burglary charge violated the Due Process Clause because it created either (1) an irrebuttable conclusive presumption (which relieved the prosecution of proving the element of intent and removed that issue from the jury) or (2) a rebuttable mandatory presumption (which unconstitutionally shifted the burden of proof on an element of a charged offense to the defendant)

The Supreme Court has interpreted the Due Process Clause of the US Constitution to require that the prosecution prove all elements of an offense beyond a reasonable doubt See In re Winship 397 US 358 364 (1970) The burden of proof cannot be shifted to the defendant by presuming an essential element upon proof of other elements of the offense because shifting the burden of persuasion with respect to any element of a criminal offense is contrary to the Due Process Clause See Mullaney v Wilbur 421 US 684 (1975)

The crime of burglary includes entry into a building or dwelling with the specific intent to commit a felony therein The requirement that the prosecutor prove beyond a reasonable doubt that the defendant had this specific intent distinguishes burglary from general-intent crimes like trespass See Sandstrom v Montana 442 US 510 523 (1979)

Here the jury was instructed that if ldquoafter consideration of all the evidence presented by the prosecution and defense you find beyond a reasonable doubt that the defendant entered the dwelling without the ownersrsquo consent you may presume that the defendant entered with the intent to commit a felony thereinrdquo This instruction was unconstitutional because it created either an irrebuttable conclusive presumption or a rebuttable mandatory presumption

A conclusive presumption is ldquoan irrebuttable direction by the court to find intent once convinced of the facts triggering the presumptionrdquo Id at 517 Here the jurors were instructed that once the prosecutor established that the defendant entered the neighborsrsquo house without consent they ldquomay presumerdquo that he intended to commit a felony therein The jurors may have reasonably concluded from this instruction that if they found that the defendant intended to enter his neighborsrsquo home without permission they must further find that he entered with the specific intent to commit a felony therein Because this instruction could operate as a conclusive

24

Criminal Law and Procedure Analysis

irrebuttable presumption by eliminating intent ldquoas an ingredient of the offenserdquo it violated due process by relieving the prosecution of the burden of proof for this element Id at 522

In the alternative the jury instruction could have been reasonably understood to create a rebuttable mandatory presumption which ldquotells [the jury] they must find the elemental fact upon proof of the basic fact at least unless the defendant has come forward with some evidence to rebut the presumed connection between the two factsrdquo County Court of Ulster County New York v Allen 442 US 140 157 (1979) The due process problem created by rebuttable mandatory presumptions is that ldquo[t]o the extent that the trier of fact is forced to abide by the presumption and may not reject it based on an independent evaluation of the particular facts presented by the State the analysis of the presumptionrsquos constitutional validity is logically divorced from those facts and based on the presumptionrsquos accuracy in the run of casesrdquo Id at 159

Unlike irrebuttable conclusive presumptions rebuttable mandatory presumptions are not always per se violations of the Due Process Clause However the Supreme Court of the United States has held that jury instructions that could reasonably be understood as shifting the burden of proof to the defendant on an element of the offense are unconstitutional Francis v Franklin 471 US 307 (1985) Here the argument that the jury instruction operated as a rebuttable mandatory presumption is supported by the fact that the judge also instructed the jury to ldquoconsider[ ] all the evidence presented by the prosecution and defenserdquo However even if the instruction created a rebuttable mandatory presumption it would be unconstitutional because it shifted the burden to the defense on an element of the offense Sandstrom 442 US at 524 Mullaney 421 US at 686

[NOTE Whether an examinee identifies the jury instruction as containing a ldquoconclusiverdquo or ldquomandatoryrdquo presumption is less important than the examineersquos analysis of the constitutional infirmities]

Point Three (35) The trial court violated the defendantrsquos Sixth Amendment right to a jury trial on an essential element of the offense when it found by a preponderance of the evidence that the ring was worth over $5000 and increased the defendantrsquos sentence based on this finding

In the statutory scheme under which the defendant was tried and convicted a Class D felony theft is defined as theft of item(s) with a value between $2500 and $10000 The jury found that the value of the diamond ring was at least $2500 and convicted the defendant of felony theft However at sentencing the trial court made a separate finding by a preponderance of the evidence that the value of the ring was greater than $5000 Following the statutersquos two-tiered sentencing scheme the judge then imposed on the defendant a sentence that was one year longer than the maximum that would otherwise have been allowed

The judgersquos sentence was unconstitutional because it violated the defendantrsquos Sixth Amendment right to a jury trial on this question The Supreme Court held in Apprendi v New Jersey 530 US 466 (2000) that ldquo[o]ther than the fact of a prior conviction any fact that increases the penalty for a crime beyond the prescribed statutory maximum must be submitted to a jury and proved beyond a reasonable doubtrdquo because ldquo[i]t is unconstitutional for a legislature to remove from the jury the assessment of facts that increase the prescribed range of penalties to which a criminal defendant is exposed [because] such facts must be established by proof beyond a reasonable doubtrdquo Id The Court reaffirmed Apprendi in Blakely v Washington 542 US 296 (2004) holding that the ldquolsquostatutory maximumrsquo for Apprendi purposes is the maximum sentence a judge may impose solely on the basis of the facts reflected in the jury verdict or admitted by the defendantrdquo Id at 303 (emphasis in original) In United States v Booker 543 US 220 (2005)

25

Criminal Law and Procedure Analysis

the Court relied on Blakely and Apprendi to conclude that protecting a defendantrsquos Sixth Amendment right to a jury trial required that ldquo[a]ny fact which is necessary to support a sentence exceeding the maximum authorized by the facts established by a plea of guilty or a jury verdict must be admitted by the defendant or proved to a jury beyond a reasonable doubtrdquo Id at 244

Thus in order to constitutionally increase a sentence above the statutory maximum of three years the jury must have found beyond a reasonable doubt that the value of the ring exceeded $5000 Here the court made the finding based on an appraisal proffered by the prosecutor only at sentencing and the judgersquos finding was by a preponderance of the evidence rather than beyond a reasonable doubt

26

AGENCY AND PARTNERSHIP ANALYSIS __________ (Agency and Partnership VA amp C VI)

ANALYSIS

Legal Problems

(1) Is a partner in a general partnership personally liable on a claim arising from misrepresentations by another partner made in the course of the partnership business

(2) Does a newly admitted partner in a general partnership become personally liable on existing claims against the partnership

(3) After the filing by a general partnership of a statement of qualification as a limited liability partnership are the partners personally liable as partners on (a) an existing claim against the general partnership and (b) a claim against the partnership that arose after the filing

DISCUSSION

Summary

Adam and Ben formed a general partnership under which they were jointly and severally liable for obligations of the partnership Thus Adam was personally liable for misrepresentations by Ben made in the ordinary course of the partnership business

Upon joining the general partnership Diane became personally liable for the obligations of the partnership arising after her admission but not for obligations pre-existing her admission such as the collectorrsquos claim

By filing a statement of qualification the three partners properly elected limited liability partnership status As partners in an LLP none of the three partners is personally liable as a partner for partnership obligations arising after the election such as the claim by the driverrsquos estate The election however does not change their personal liability on pre-existing claims that arose before the election such as the collectorrsquos claim

Point One (30) As a general partner of Empire a general partnership Adam became personally liable on the collectorrsquos claim a valid claim against the partnership that arose because of Benrsquos wrongful act in the ordinary course of the partnership business

When the collectorrsquos claim arose Empire was a general partnership composed of Adam and Ben Under UPA (1997) sect 306(a) partners of a general partnership are liable jointly and severally for all obligations of the partnership Under UPA (1997) sect 305(a) the partnership could become obligated for the loss caused to the collector as a result of the misrepresentation by Ben provided he was acting in the ordinary course of the partnership business Because there was no statement that limited his partnership authority Ben as partner was ldquoan agent of the partnership for the purpose of its businessrdquo See UPA (1997) sect 301(1) Benrsquos misrepresentation to the collector even if intentional appears to be in the ordinary course of the partnershiprsquos business of dealing

27

Agency and Partnership Analysis

in antique cars Thus Benrsquos wrongful act created a partnership obligation for which Adam was jointly and severally liable

[NOTE Generally a partnership creditor must ldquoexhaust the partnershiprsquos assets before levying on a judgment debtor partnerrsquos individual property where the partner is personally liable for the partnership obligationrdquo as a result of his status as a partner UPA (1997) sect 307 cmt 4 As the UPA comments explain this places Adam more in the position of guarantor than principal debtor on the partnership obligation Id cmt 4 Although an examinee might discuss this point the call focuses on whether Adam is personally liable not how the liability might be enforced]

Point Two (30) Because the collectorrsquos claim arose before Diane joined Empire Diane did not become personally liable on the claim

Diane was admitted to Empire when it was a general partnership and after the collectorrsquos claim arose While the general rule under UPA (1997) sect 306(a) is that the partners of a general partnership are liable jointly and severally for all obligations of the partnership there is a special rule for partners who are admitted during the duration of the partnership Under UPA (1997) sect 306(b) a person admitted to an existing partnership is not personally liable for any partnership obligations incurred before the personrsquos admission Because Diane was admitted to Empire after the collectorrsquos claim arose Diane is not personally liable on the claim

Dianersquos knowledge of the pre-existing claim and her stated concern about becoming liable on the collectorrsquos claim do not change her personal nonliability to the collector Although partners who have a liability shield can assume liability to third parties through private contractual guarantees or modifications to the partnership agreement Dianersquos stated concern constituted neither a guaranty to the collector nor ldquoan intentional waiver of liability protectionsrdquo See UPA (1997) sect 306 cmt 3 (describing methods for waiver of liability protections under sect 306(c) applicable in limited liability partnerships)

At most Diane will lose her investment in the partnership as a result of the collectorrsquos claim Although Diane did not become personally liable on the collectorrsquos claim when she joined the partnership the $250000 she contributed to the partnership is ldquoat risk for the satisfaction of existing partnership debtsrdquo UPA (1997) sect 306 cmt 2

Point Three (40) Filing the statement of qualification was effective to elect limited liability partnership status Despite this new status Adam and Ben remain personally liable on the collectorrsquos claim which arose before the election But as partners in an LLP neither Adam Ben nor Diane is personally liable as a partner on the driverrsquos estatersquos claim which arose after the election

Under UPA (1997) sect 1001 a general partnership can make an election and become a limited liability partnershipmdashif the partners approve the conversion by a vote equivalent to that necessary to amend the partnership agreement and the partnership then files a statement of qualification that specifies the name of the partnership its principal office and its election to be an LLP Here the partners agreed unanimouslymdashsufficient to amend their agreement under UPA (1997) sect 401(j)mdashand the statement of qualification was filed In addition the name of Empire LLP properly included an appropriate ending ldquoLLPrdquo See UPA (1997) sect 1002

Although another way to effectuate a ldquoconversionrdquo (as suggested by Benrsquos lawyer) is to form a new LLP and transfer the assets of the old general partnership to the new LLP the

28

Agency and Partnership Analysis

method used here (approval by the partners and the filing of a statement of qualification) is also sufficient to create LLP status

Thus Empire became Empire LLP as of the date of filing of the statement of qualification See UPA (1997) sect 1001 What effect did this have on the collectorrsquos claim which predated the filing According to UPA (1997) sect 306(c) an obligation incurred while a partnership is an LLP is solely a partnership obligation As the collectorrsquos claim predated the LLP Adam and Ben remain personally liable on the collectorrsquos claim Diane on the other hand was not personally liable on the collectorrsquos claim either before or after the filing of the statement of qualification See Point Two above

The driverrsquos estatersquos claim arose after Empire became Empire LLP Under UPA (1997) sect 306(c) an obligation incurred while a partnership is an LLP is solely a partnership obligationThus Adam Ben and Diane as partners are all protected from personal liability on the driverrsquos estatersquos claim But there may be personal liability if any of them was negligent or otherwise acted wrongfully by not informing the buyer of the bad suspension that caused the accident

29

National Conference of Bar Examiners 302 South Bedford Street | Madison WI 53703-3622 Phone 608-280-8550 | Fax 608-280-8552 | TDD 608-661-1275

wwwncbexorg e-mail contactncbexorg

  • Preface
  • Description of the MEE
  • Instructions
  • February 2014 Questions
    • Constitutinal Law Question
    • Trusts and Future Interests Question
    • Secured Transactions Question
    • Federal Civil Procedure Question
    • Criminal Law and Procedure Question
    • Agency and Partnership Question
      • February 2014 Analyses
        • Constitutional Law Analysis
        • Trust and Future Interests Analysis
        • Secured Transactions Analysis
        • Federal Civil Procedure Analysis
        • Criminal Law and Procedure Analysis
        • Agency and Partnership Analysis
            • ltlt13 ASCII85EncodePages false13 AllowTransparency false13 AutoPositionEPSFiles true13 AutoRotatePages None13 Binding Left13 CalGrayProfile (Dot Gain 20)13 CalRGBProfile (sRGB IEC61966-21)13 CalCMYKProfile (US Web Coated 050SWOP051 v2)13 sRGBProfile (sRGB IEC61966-21)13 CannotEmbedFontPolicy Error13 CompatibilityLevel 1413 CompressObjects Tags13 CompressPages true13 ConvertImagesToIndexed true13 PassThroughJPEGImages true13 CreateJobTicket false13 DefaultRenderingIntent Default13 DetectBlends true13 DetectCurves 0000013 ColorConversionStrategy CMYK13 DoThumbnails false13 EmbedAllFonts true13 EmbedOpenType false13 ParseICCProfilesInComments true13 EmbedJobOptions true13 DSCReportingLevel 013 EmitDSCWarnings false13 EndPage -113 ImageMemory 104857613 LockDistillerParams false13 MaxSubsetPct 10013 Optimize true13 OPM 113 ParseDSCComments true13 ParseDSCCommentsForDocInfo true13 PreserveCopyPage true13 PreserveDICMYKValues true13 PreserveEPSInfo true13 PreserveFlatness true13 PreserveHalftoneInfo false13 PreserveOPIComments true13 PreserveOverprintSettings true13 StartPage 113 SubsetFonts true13 TransferFunctionInfo Apply13 UCRandBGInfo Preserve13 UsePrologue false13 ColorSettingsFile ()13 AlwaysEmbed [ true13 ]13 NeverEmbed [ true13 ]13 AntiAliasColorImages false13 CropColorImages true13 ColorImageMinResolution 30013 ColorImageMinResolutionPolicy OK13 DownsampleColorImages true13 ColorImageDownsampleType Bicubic13 ColorImageResolution 30013 ColorImageDepth -113 ColorImageMinDownsampleDepth 113 ColorImageDownsampleThreshold 15000013 EncodeColorImages true13 ColorImageFilter DCTEncode13 AutoFilterColorImages true13 ColorImageAutoFilterStrategy JPEG13 ColorACSImageDict ltlt13 QFactor 01513 HSamples [1 1 1 1] VSamples [1 1 1 1]13 gtgt13 ColorImageDict ltlt13 QFactor 01513 HSamples [1 1 1 1] VSamples [1 1 1 1]13 gtgt13 JPEG2000ColorACSImageDict ltlt13 TileWidth 25613 TileHeight 25613 Quality 3013 gtgt13 JPEG2000ColorImageDict ltlt13 TileWidth 25613 TileHeight 25613 Quality 3013 gtgt13 AntiAliasGrayImages false13 CropGrayImages true13 GrayImageMinResolution 30013 GrayImageMinResolutionPolicy OK13 DownsampleGrayImages true13 GrayImageDownsampleType Bicubic13 GrayImageResolution 30013 GrayImageDepth -113 GrayImageMinDownsampleDepth 213 GrayImageDownsampleThreshold 15000013 EncodeGrayImages true13 GrayImageFilter DCTEncode13 AutoFilterGrayImages true13 GrayImageAutoFilterStrategy JPEG13 GrayACSImageDict ltlt13 QFactor 01513 HSamples [1 1 1 1] VSamples [1 1 1 1]13 gtgt13 GrayImageDict ltlt13 QFactor 01513 HSamples [1 1 1 1] VSamples [1 1 1 1]13 gtgt13 JPEG2000GrayACSImageDict ltlt13 TileWidth 25613 TileHeight 25613 Quality 3013 gtgt13 JPEG2000GrayImageDict ltlt13 TileWidth 25613 TileHeight 25613 Quality 3013 gtgt13 AntiAliasMonoImages false13 CropMonoImages true13 MonoImageMinResolution 120013 MonoImageMinResolutionPolicy OK13 DownsampleMonoImages true13 MonoImageDownsampleType Bicubic13 MonoImageResolution 120013 MonoImageDepth -113 MonoImageDownsampleThreshold 15000013 EncodeMonoImages true13 MonoImageFilter CCITTFaxEncode13 MonoImageDict ltlt13 K -113 gtgt13 AllowPSXObjects false13 CheckCompliance [13 None13 ]13 PDFX1aCheck false13 PDFX3Check false13 PDFXCompliantPDFOnly false13 PDFXNoTrimBoxError true13 PDFXTrimBoxToMediaBoxOffset [13 00000013 00000013 00000013 00000013 ]13 PDFXSetBleedBoxToMediaBox true13 PDFXBleedBoxToTrimBoxOffset [13 00000013 00000013 00000013 00000013 ]13 PDFXOutputIntentProfile ()13 PDFXOutputConditionIdentifier ()13 PDFXOutputCondition ()13 PDFXRegistryName ()13 PDFXTrapped False1313 CreateJDFFile false13 Description ltlt13 ARA 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 BGR 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 CHS ltFEFF4f7f75288fd94e9b8bbe5b9a521b5efa7684002000410064006f006200650020005000440046002065876863900275284e8e9ad88d2891cf76845370524d53705237300260a853ef4ee54f7f75280020004100630072006f0062006100740020548c002000410064006f00620065002000520065006100640065007200200035002e003000204ee553ca66f49ad87248672c676562535f00521b5efa768400200050004400460020658768633002gt13 CHT ltFEFF4f7f752890194e9b8a2d7f6e5efa7acb7684002000410064006f006200650020005000440046002065874ef69069752865bc9ad854c18cea76845370524d5370523786557406300260a853ef4ee54f7f75280020004100630072006f0062006100740020548c002000410064006f00620065002000520065006100640065007200200035002e003000204ee553ca66f49ad87248672c4f86958b555f5df25efa7acb76840020005000440046002065874ef63002gt13 CZE 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 DAN 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 DEU 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 ESP ltFEFF005500740069006c0069006300650020006500730074006100200063006f006e0066006900670075007200610063006900f3006e0020007000610072006100200063007200650061007200200064006f00630075006d0065006e0074006f00730020005000440046002000640065002000410064006f0062006500200061006400650063007500610064006f00730020007000610072006100200069006d0070007200650073006900f3006e0020007000720065002d0065006400690074006f007200690061006c00200064006500200061006c00740061002000630061006c0069006400610064002e002000530065002000700075006500640065006e00200061006200720069007200200064006f00630075006d0065006e0074006f00730020005000440046002000630072006500610064006f007300200063006f006e0020004100630072006f006200610074002c002000410064006f00620065002000520065006100640065007200200035002e003000200079002000760065007200730069006f006e0065007300200070006f00730074006500720069006f007200650073002egt13 ETI ltFEFF004b00610073007500740061006700650020006e0065006900640020007300e4007400740065006900640020006b00760061006c006900740065006500740073006500200074007200fc006b006900650065006c007300650020007000720069006e00740069006d0069007300650020006a0061006f006b007300200073006f00620069006c0069006b0065002000410064006f006200650020005000440046002d0064006f006b0075006d0065006e00740069006400650020006c006f006f006d006900730065006b0073002e00200020004c006f006f0064007500640020005000440046002d0064006f006b0075006d0065006e00740065002000730061006100740065002000610076006100640061002000700072006f006700720061006d006d006900640065006700610020004100630072006f0062006100740020006e0069006e0067002000410064006f00620065002000520065006100640065007200200035002e00300020006a00610020007500750065006d006100740065002000760065007200730069006f006f006e00690064006500670061002e000d000agt13 FRA 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 GRE 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 HEB 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 HRV (Za stvaranje Adobe PDF dokumenata najpogodnijih za visokokvalitetni ispis prije tiskanja koristite ove postavke Stvoreni PDF dokumenti mogu se otvoriti Acrobat i Adobe Reader 50 i kasnijim verzijama)13 HUN 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 ITA 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 JPN ltFEFF9ad854c18cea306a30d730ea30d730ec30b951fa529b7528002000410064006f0062006500200050004400460020658766f8306e4f5c6210306b4f7f75283057307e305930023053306e8a2d5b9a30674f5c62103055308c305f0020005000440046002030d530a130a430eb306f3001004100630072006f0062006100740020304a30883073002000410064006f00620065002000520065006100640065007200200035002e003000204ee5964d3067958b304f30533068304c3067304d307e305930023053306e8a2d5b9a306b306f30d530a930f330c8306e57cb30818fbc307f304c5fc59808306730593002gt13 KOR ltFEFFc7740020c124c815c7440020c0acc6a9d558c5ec0020ace0d488c9c80020c2dcd5d80020c778c1c4c5d00020ac00c7a50020c801d569d55c002000410064006f0062006500200050004400460020bb38c11cb97c0020c791c131d569b2c8b2e4002e0020c774b807ac8c0020c791c131b41c00200050004400460020bb38c11cb2940020004100630072006f0062006100740020bc0f002000410064006f00620065002000520065006100640065007200200035002e00300020c774c0c1c5d0c11c0020c5f40020c2180020c788c2b5b2c8b2e4002egt13 LTH 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 LVI 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 NLD (Gebruik deze instellingen om Adobe PDF-documenten te maken die zijn geoptimaliseerd voor prepress-afdrukken van hoge kwaliteit De gemaakte PDF-documenten kunnen worden geopend met Acrobat en Adobe Reader 50 en hoger)13 NOR 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 POL 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 PTB ltFEFF005500740069006c0069007a006500200065007300730061007300200063006f006e00660069006700750072006100e700f50065007300200064006500200066006f0072006d00610020006100200063007200690061007200200064006f00630075006d0065006e0074006f0073002000410064006f0062006500200050004400460020006d00610069007300200061006400650071007500610064006f00730020007000610072006100200070007200e9002d0069006d0070007200650073007300f50065007300200064006500200061006c007400610020007100750061006c00690064006100640065002e0020004f007300200064006f00630075006d0065006e0074006f00730020005000440046002000630072006900610064006f007300200070006f00640065006d0020007300650072002000610062006500720074006f007300200063006f006d0020006f0020004100630072006f006200610074002000650020006f002000410064006f00620065002000520065006100640065007200200035002e0030002000650020007600650072007300f50065007300200070006f00730074006500720069006f007200650073002egt13 RUM 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 RUS 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 SKY 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 SLV ltFEFF005400650020006e006100730074006100760069007400760065002000750070006f0072006100620069007400650020007a00610020007500730074007600610072006a0061006e006a006500200064006f006b0075006d0065006e0074006f0076002000410064006f006200650020005000440046002c0020006b006900200073006f0020006e0061006a007000720069006d00650072006e0065006a016100690020007a00610020006b0061006b006f0076006f00730074006e006f0020007400690073006b0061006e006a00650020007300200070007200690070007200610076006f0020006e00610020007400690073006b002e00200020005500730074007600610072006a0065006e006500200064006f006b0075006d0065006e0074006500200050004400460020006a00650020006d006f0067006f010d00650020006f0064007000720065007400690020007a0020004100630072006f00620061007400200069006e002000410064006f00620065002000520065006100640065007200200035002e003000200069006e0020006e006f00760065006a01610069006d002egt13 SUO 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 SVE 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 TUR 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 UKR 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 ENU (Use these settings to create Adobe PDF documents best suited for high-quality prepress printing Created PDF documents can be opened with Acrobat and Adobe Reader 50 and later)13 gtgt13 Namespace [13 (Adobe)13 (Common)13 (10)13 ]13 OtherNamespaces [13 ltlt13 AsReaderSpreads false13 CropImagesToFrames true13 ErrorControl WarnAndContinue13 FlattenerIgnoreSpreadOverrides false13 IncludeGuidesGrids false13 IncludeNonPrinting false13 IncludeSlug false13 Namespace [13 (Adobe)13 (InDesign)13 (40)13 ]13 OmitPlacedBitmaps false13 OmitPlacedEPS false13 OmitPlacedPDF false13 SimulateOverprint Legacy13 gtgt13 ltlt13 AddBleedMarks false13 AddColorBars false13 AddCropMarks false13 AddPageInfo false13 AddRegMarks false13 ConvertColors ConvertToCMYK13 DestinationProfileName ()13 DestinationProfileSelector DocumentCMYK13 Downsample16BitImages true13 FlattenerPreset ltlt13 PresetSelector MediumResolution13 gtgt13 FormElements false13 GenerateStructure false13 IncludeBookmarks false13 IncludeHyperlinks false13 IncludeInteractive false13 IncludeLayers false13 IncludeProfiles false13 MultimediaHandling UseObjectSettings13 Namespace [13 (Adobe)13 (CreativeSuite)13 (20)13 ]13 PDFXOutputIntentProfileSelector DocumentCMYK13 PreserveEditing true13 UntaggedCMYKHandling LeaveUntagged13 UntaggedRGBHandling UseDocumentProfile13 UseDocumentBleed false13 gtgt13 ]13gtgt setdistillerparams13ltlt13 HWResolution [2400 2400]13 PageSize [612000 792000]13gtgt setpagedevice13

Page 16: February 2014 MEE Questions and AnalysesPreface The Multistate Essay Examination (MEE) is developed by the National Conference of Bar Examiners (NCBE). This publication includes the

Constitutional Law Analysis

The exaction here however may fail the second prong of the Dolan testmdashthat the exaction be roughly proportional to the anticipated impact of the requested development As noted the city in Dolan claimed that a bike and pedestrian path was needed to offset the increase in traffic due to the proposed doubling of the business The Court explained that the government must demonstrate that the additional traffic reasonably was related to the requested exaction and that the government must ldquomake some effort to quantify its findings in support of the dedication for the pedestrianbicycle pathway beyond the conclusory statement that it could offset some of the traffic demand generatedrdquo Id at 395 Here the city did not carry its burden The city simply speculates that increased patronage of the restaurant ldquomightrdquo increase crime and that the surveillance equipment ldquomightrdquo alleviate this increased crime Because the city has not made ldquosome effort to quantify its findingsrdquo in support of the easement it has not shown that the burden of the easement is roughly proportional to the benefits thought to flow from it

Thus the exaction appears to be an uncompensated taking of property in violation of the Fifth Amendment as applied to the states through the Fourteenth Amendment

13

TRUSTS AND FUTURE INTERESTS ANALYSIS ____ (Trusts and Future Interests IE3 I5 IIIA amp B)

ANALYSIS

Legal Problems

(1) How should rents dividends and sales proceeds received by the trustee prior to receipt of the sonrsquos letter have been allocated between trust income and principal

(2)(a) Did the remainder interest in the trust accelerate and become immediately payable to the daughterrsquos minor child upon the trusteersquos receipt of the sonrsquos letter and if not how should the trustee handle the distribution of the principal in the future

(2)(b) Following the trusteersquos receipt of the sonrsquos letter how should the trustee distribute future receipts of income prior to the distribution of the principal

DISCUSSION

Summary

Prior to the trusteersquos receipt of the sonrsquos letter cash dividends and rents should have been allocated to trust income and were distributable to the son the income beneficiary of the trust sales proceeds and stock dividends should have been allocated to principal

Because the sonrsquos letter to the trustee did not result in a valid disclaimer under state law (having been made more than nine months after the testatorrsquos death) the son is not deemed to have predeceased the testator Because the son is still living the class gift to the testatorrsquos grandchildren who survive the son has not closed and is not possessory it will not become possessory until the son dies The daughterrsquos minor child being the testatorrsquos only living grandchild is not currently entitled to a distribution of trust principal Trust principal will instead be distributable upon the sonrsquos death to the testatorrsquos then-living grandchildren or if there are none to the testatorrsquos then-living heirs

As for future income the trustee should either distribute the trust income to the son and the daughter as the testatorrsquos heirs accumulate the income for future distribution to those individuals ultimately entitled to the trust principal or distribute it to those presumptively entitled to the principal upon the sonrsquos death ie the daughterrsquos minor child

Point One (45) Cash dividends and rents are allocable to income sales proceeds and stock dividends are allocable to principal Items allocable to income for the period prior to the sonrsquos attempted disclaimer were distributable to the son

Receipts earned during the administration of a trust are allocable either to income or to principal Almost all states have adopted the most recent or an earlier version of the Uniform Principal and Income Act (the Act) which specifies how such receipts should be allocated

Under the Act rents (UNIF PRIN amp INC ACT (2000) sect 405 UNIF PRIN amp INC ACT (1962) sect 3(a)(1)) and cash dividends received from a corporation (UNIF PRIN amp INC ACT (2000) sect 401(b) UNIF PRIN amp INC ACT (1962) sect 6(d)) are allocable to income and are distributable to the income beneficiary of the trust

14

Trusts and Future Interests Analysis Sales proceeds (UNIF PRIN amp INC ACT (2000) sect 404(2) UNIF PRIN amp INC ACT (1962)

sect 3(b)(1)) and dividends paid in the stock of the distributing corporation (UNIF PRIN amp INC ACT (2000) sect 401(c)(1) UNIF PRIN amp INC ACT (1962) sect 3(b)(4)) are allocable to principal and added to the principal of the trust

Here the cash dividends and office building rents should have been allocated to income and until the trustee received the sonrsquos letter should have been distributed to him as the sole income beneficiary of the trust The stock dividend and proceeds from the sale of the office building should have been allocated to principal and held by the trustee for future distribution to the ultimate remaindermen of the trust

[NOTE The 2000 Uniform Principal and Income Act has been adopted in Alabama Arkansas Colorado Connecticut the District of Columbia Hawaii Idaho Iowa Kentucky Missouri Montana Nebraska New Mexico North Dakota Oregon South Dakota Utah and West Virginia]

Point Two(a) (45) Because the son did not disclaim within nine months of the testatorrsquos death there is no valid disclaimer under state law Therefore the son is not deemed to have predeceased the testator Furthermore because of the express survivorship contingency in the will the remainder in the trust does not accelerate and become distributable until the son in fact dies When the son dies the trust principal will be distributable to the testatorrsquos then-living grandchildren or if none then to the testatorrsquos then-living heirs

When a trust remainder is given to a class the class closes (ie no new persons can join the class) when there is no outstanding income interest and at least one member of the class is then entitled to demand possession of his or her share of the remainder This principle is called the rule of convenience See generally HERBERT HOVENKAMP amp SHELDON F KURTZ PRINCIPLES OF PROPERTY LAW 199ndash200 (6th ed 2005) A class member may demand possession of his or her share of the remainder upon termination of the income interest only when the class memberrsquos interest is not otherwise subject to a condition precedent See id

When a beneficiary timely disclaims an interest in a trust that beneficiary is treated as if he had predeceased the testator Here had the son disclaimed within nine months of the testatorrsquos death as required by the state statute he would have been deemed to have predeceased the testator This would have closed the class of remaindermen and the testatorrsquos then-living grandchildren (ie the daughterrsquos child) would have been entitled to the trust principal However under the state statute the sonrsquos disclaimer was not timely because he did not disclaim within nine months of the testatorrsquos death Thus because the statute is inapplicable and the son is still alive the class of grandchildren entitled to share in trust principal did not close

Because here the statute is inapplicable due to the sonrsquos failure to comply with the statutory time requirements then presumably the common-law rule allowing disclaimers (aka renunciations) at any time should apply Under the common law if a life estate is renounced the remainder interest accelerates and becomes immediately distributable to the remaindermen of the trust if the remainder is vested but not if the remainder is contingent JESSE DUKEMINIER amp ROBERT H SITKOFF WILLS TRUSTS AND ESTATES 844ndash845 (9th ed 2013) Here because the remainder is contingent upon there being grandchildren who survive the son the remainder will not accelerate It will remain open until the son dies leaving open the possibility that additional grandchildren will be included in the class or the daughterrsquos child could fall out of the class because that child fails to survive the son

And if none of the testatorrsquos grandchildren survive the son the trust principal will be distributed to the testatorrsquos heirs living at the sonrsquos death

15

Trusts and Future Interests Analysis

Point Two(b) (10) Until the trust terminates the trustee must continue to hold the trust assets The distribution of income in the meantime is unclear There are at least three possibilities Income earned on the undistributed assets could be distributed to the son and daughter as the testatorrsquos heirs accumulated and added to principal for distribution to the ultimate remaindermen or distributed from time to time to those persons who are presumptively remaindermen

When trust principal is not immediately distributable the trustee must continue to hold trust assets until the ultimate remaindermen are ascertained During this period trust income will be distributed or retained according to any instructions contained in the trust instrument See WILLIAM M MCGOVERN JR SHELDON F KURTZ amp DAVID M ENGLISH WILLS TRUSTS amp ESTATES sect 102 (4th ed 2010)

Here the testator did not specify what the trustee should do with trust income in the event the sonrsquos disclaimer did not comply with the state statute There are at least three approaches One approach would have the trustee distribute the trust income to the testatorrsquos heirs on the theory that the income represents property that was not disposed of by the testatorrsquos will and which thus passes by partial intestacy to the testatorrsquos heirs A second approach would have the trustee accumulate trust income for distribution to the ultimate remaindermen Under this approach only those individuals ultimately entitled to the principal would share in the income A third approach would have the trustee distribute trust income to those individuals who would be the remaindermen if the trust were to terminate when the income is received by the trustee under this approach trust income would be distributed to the daughterrsquos minor child until another presumptive remainderman is born This approach could result in individuals not ultimately entitled to principal say because they do not survive the son receiving income It could also result in a disproportionate distribution of income among the individuals ultimately entitled to income

[NOTE Examinees should demonstrate a recognition and understanding of the income-allocation problem and the alternatives available to address that issue There is no widely accepted solution to the problem Examinees who cite any of these possible problem-solving approaches may receive credit]

16

SECURED TRANSACTIONS ANALYSIS (Secured Transactions IB IID E amp F IIIB IVA B amp F)

ANALYSIS

Legal Problems

(1)(a) What is the nature of the bankrsquos claim to the businessrsquos equipment

(1)(b) What is the nature of the finance companyrsquos claim to the businessrsquos equipment

(1)(c) As between the bank and the finance company whose claim to the businessrsquos equipment has priority

(2) Do the claims of the bank and the finance company continue in the item of equipment sold by the business to the competitor

DISCUSSION

Summary

The bank and the finance company both have perfected security interests in the businessrsquos equipment Even though the finance companyrsquos perfected security interest was created first the bankrsquos perfected security interest has priority because the bankrsquos financing statement was filed before the finance companyrsquos financing statement The security interests of the bank and the finance company continue in the item of equipment sold by the business to the competitor because their security interests were perfected and the competitor was not a buyer in ordinary course of business

Point One(a) (25) The bank has a perfected security interest in the businessrsquos equipment

The bank has met all criteria necessary for it to have an attached and enforceable security interest in the businessrsquos equipment First value must be given UCC sect 9-203(b)(1) This criterion is fulfilled by the loan made by the bank to the business Second the debtor must have rights in the collateral UCC sect 9-203(b)(2) Clearly the business has rights in its equipment Third either the secured party must take possession of the collateral or the debtor must authenticate a security agreement containing a description of the collateral UCC sect 9-203(b)(3) The agreement that the business owner signed is a ldquosecurity agreementrdquo because it is an agreement that creates or provides for a security interest UCC sect 9-102(a)(74) By signing the security agreement the business owner authenticated it UCC sect 9-102(a)(7) Therefore all three criteria are fulfilled and the bank has an enforceable and attached security interest

A security interest is perfected when it has attached and when any additional steps required for perfection have occurred UCC sect 9-308(a) Generally speaking the additional steps will either be possession of the collateral by the secured party or the filing of a financing statement with respect to the collateral See UCC sectsect 9-310 9-313 In this case the bank filed a financing statement naming the debtor and sufficiently indicating the collateral The collateral indication is sufficient because it identifies the collateral by type of property See UCC sectsect 9-504 9-108 The fact that the financing statement was filed before the security interest was created is

17

Secured Transactions Analysis

not a problem Even though the security agreement had not yet been signed the business had authorized the filing of the financing statement in an authenticated record UCC sect 9-509(a)(1) Moreover the financing statement may be filed before the security agreement is created UCC sect 9-502(d)

Point One(b) (10) The finance company also has a perfected security interest in the businessrsquos equipment

The finance companyrsquos security interest is enforceable and attached for the same reasons as the bankrsquos security interest The loan from the finance company to the business constitutes value the business has rights in the collateral and the business owner has authenticated a security agreement containing a description of the collateral The finance companyrsquos security interest is perfected because the finance company filed a financing statement with respect to it that provides that the business is the debtor and indicates that the collateral is equipment

Point One(c) (30) The bankrsquos security interest has priority over the finance companyrsquos security interest because the bankrsquos financing statement was filed first

As between two perfected security interests the general rule is that the security interest that was the earlier to be either perfected or the subject of a filed financing statement has priority UCC sect 9-322(a)(1) While the finance companyrsquos security interest was perfected before the bankrsquos (March 15 vs March 22) the bankrsquos financing statement was filed even earlier on March 2 Thus under the first-to-file-or-perfect rule of UCC sect 9-322(a)(1) the bankrsquos security interest has priority No exceptions to the general rule apply here

Point Two (35) A security interest in collateral continues notwithstanding its sale unless an exception applies Because the security interests of the bank and the finance company were perfected and the competitor was not a buyer in ordinary course of business no exception applies and the security interests of both creditors continue in the equipment sold to the competitor

As a general rule a security interest in collateral continues notwithstanding the fact that the debtor has sold the collateral to another person UCC sect 9-315(a)(1) Thus unless an exception applies the security interests of the bank and the finance company will continue in the item of equipment sold to the competitor

A buyer of goods will take free of an unperfected security interest in those goods See UCC sect 9-317(a)(2) However when the competitor bought the businessrsquos equipment both the bank and the finance company had perfected security interests in the equipment

A buyer can take free even of a perfected security interest in goods if the buyer is a ldquobuyer in ordinary course of businessrdquo See UCC sect 9-320(a) However the competitor was not a buyer in ordinary course of business To be a ldquobuyer in ordinary course of businessrdquo a buyer must buy goods from a seller that is in the business of selling goods of that kind See UCC sect 1-201(b)(9) The competitor bought this equipment from a seller that is not in the business of selling goods of this kind so the competitor was not a buyer in ordinary course of business with respect to these goods

Because no exception applies the security interests of the bank and the finance company continue even after the item of equipment was sold to the competitor

18

FEDERAL CIVIL PROCEDURE ANALYSIS (Federal Civil Procedure IVD)

ANALYSIS

Legal Problems

(1) Is a document prepared in the course of a contract dispute protected from discovery as ldquowork productrdquo when there is no evidence that the document was prepared in anticipation of litigation

(2)(a) Is a partyrsquos failure to provide relevant electronically stored information excused when the information was destroyed pursuant to a routine document retention scheme at a time when litigation was contemplated by the destroying party

(2)(b) What sanctions should be imposed on a party for allowing the destruction of evidence that is relevant to potential future litigation

DISCUSSION

Summary

The report prepared by the structural engineer is probably not work product and is thus discoverable The engineer examined the foundation of the house at the customerrsquos request and the engineerrsquos findings are potentially relevant to the customerrsquos claim that the foundation is defective The report was not prepared in anticipation of litigation The customer appears to have sought the engineerrsquos opinion in response to the builderrsquos offer to fix any problems with the foundation that an engineer might identify Because the report was not prepared in anticipation of litigation it is not protected by the work-product doctrine

The builder should have taken appropriate steps to preserve evidence including suspending its document retention program as soon as it began planning for litigationmdashie on July 10 Its destruction of potentially relevant material after that date was wrongful However a court is unlikely to impose severe sanctions on the builder because there are no facts indicating that the builder acted in bad faith and the customer can prove that the foundation is defective without the destroyed emails

Point One (40) The customer must turn over the engineerrsquos report because it was not prepared in anticipation of litigation

In general a party to a lawsuit in federal court ldquomay obtain discovery regarding any nonprivileged matter that is relevant to any partyrsquos claim or defenserdquo FED R CIV P 26(b)(1) (2009) This includes the right to inspect and copy documents in the other partyrsquos possession FED R CIV P 34(a)(1) Here the customer hired a structural engineer to examine the foundation of the house The engineerrsquos report on the foundation is likely to include information that would be relevant to the customerrsquos claim that the foundation was defectively constructed

The so-called ldquowork productrdquo rule allows a party to refuse to turn over ldquodocuments that are prepared in anticipation of litigation or for trialrdquo by that partyrsquos representative including

19

Federal Civil Procedure Analysis

a consultant Thus if the customer had hired the structural engineer to prepare a report ldquoin anticipation of litigationrdquo that report might not be discoverable See FED R CIV P 26(b)(3)

In this case however the customer hired the engineer to evaluate the foundation of the house as part of the customerrsquos negotiation with the builder concerning the housersquos flooding problem The builder told the customer that the housersquos landscaping was the reason for the flooding and the builder told the customer ldquoHave an engineer look at the foundation If therersquos a problem wersquoll fix itrdquo The customer appears to have acted in response to that statement There is no indication that the customer anticipated any kind of legal action at the time that the structural engineer was hired Accordingly the structural engineerrsquos report is discoverable and the court should order the customer to turn it over

[NOTE If an examinee concludes that the structural engineerrsquos report was prepared in anticipation of litigation then the examinee should also conclude that the report is not discoverable Documents prepared in anticipation of litigation do not need to be disclosed to an adverse party unless that party can demonstrate a ldquosubstantial needrdquo for the documents and an inability to obtain substantially equivalent information without ldquoundue hardshiprdquo FED R CIV P 26(b)(3)(A)(ii) Furthermore a report prepared by an expert who is not expected to testify is not discoverable in the absence of ldquoexceptional circumstancesrdquo making it ldquoimpracticablerdquo to obtain the information in another way FED R CIV P 26(b)(4)(D)(ii) The builder probably cannot make these showings here unless the engineerrsquos report deals with circumstances that have since changed There is no evidence that the structural engineer would have had access to any information or facts that the builder would not already know as a result of its construction and subsequent inspection of the house In addition if necessary the builder could ask the court for permission to arrange for a further inspection of the house by a structural engineer hired by the builder See FED R CIV P 34(a)(2) Accordingly if an examinee concludes that the report was prepared in anticipation of litigation the examinee should also conclude that the builder is not entitled to see the report]

Point Two(a) (30) Because the builder anticipated that it might be involved in litigation concerning its contract with the customer the builder acted wrongfully in destroying emails that were relevant to the housersquos construction even though the emails were destroyed pursuant to a routine document retention plan

As noted above a party to a lawsuit in federal court ldquomay obtain discovery regarding any nonprivileged matter that is relevant to any partyrsquos claim or defenserdquo FED R CIV P 26(b)(1) This includes emails and other electronically stored information FED R CIV P 34(a)(1)(A) Here the customer has requested all the builderrsquos emails pertaining to work done on the foundation of the house Ordinarily the builder would be obliged to turn over this information which is relevant to the customerrsquos defense that the housersquos foundation was poorly constructed

Unfortunately the emails in question no longer exist because the builder destroyed them on August 2

In general spoliation of evidence (destruction or alteration of evidence) is improper if the party who destroyed or altered the evidence ldquohas notice that the evidence is relevant to litigation or should have known that the evidence may be relevant to future litigationrdquo Fujitsu Ltd v Federal Express Corp 247 F3d 423 436 (2d Cir 2001) It is improper for a party to destroy electronic information relevant to pending litigation even if the destruction occurs before there is any request or order seeking the information See eg Leon v IDX Sys Corp 464 F3d 951 (9th Cir 2006) (plaintiffrsquos intentional destruction of computer files warranted dismissal even

20

In this case the builderrsquos destruction of the emails was pursuant to a routine document retention plan The Federal Rules provide expressly that in the absence of ldquoexceptional circumstancesrdquo parties should not be sanctioned for the loss of electronically stored information when the loss occurs pursuant to ldquoroutine good-faith operation of an electronic information systemrdquo FED R CIV P 37(e) However when a party anticipates litigation ldquoit must suspend its routine document retentiondestruction policy and put in place a lsquolitigation holdrsquo to ensure the preservation of relevant documentsrdquo Zubulake v UBS Warburg LLC 220 FRD 212 218 (SDNY 2003)

Federal Civil Procedure Analysis

though spoliation occurred before order compelling discovery) Similarly the duty to preserve evidence applies to a party who anticipates litigation even if litigation has not yet been commenced See THE SEDONA PRINCIPLES BEST PRACTICES RECOMMENDATIONS amp PRINCIPLES FOR ADDRESSING ELECTRONIC DOCUMENT PRODUCTION 70 cmt 14a (2d ed 2007)

The builder destroyed the emails on August 2 At that time the builder knew that litigation was a possibility because the builder had already directed its attorney to prepare a draft complaint for possible filing Knowing that litigation was a possibility the builder had a duty to take steps to preserve evidence including the emails in question See generally Fujitsu Ltd

Thus the builderrsquos destruction of potentially relevant emails at a time when it knew that litigation was a possibility was improper It had a duty to preserve evidence and it breached that duty

[NOTE Because courts have used different words to describe the test for when evidence must be preserved an examineersquos precise formulation of the test is not critical]

Point Two(b) (30) In determining appropriate sanctions for spoliation courts consider both the level of culpability of the spoliating party and the degree of prejudice the loss of evidence has caused the other party Here the builderrsquos destruction of evidence does not appear to have been willful nor is it likely to pose a significant obstacle to the customerrsquos defense Any sanctions imposed by the court should be modest

Federal courts have inherent power to control the litigation process and can sanction misbehavior including spoliation even when there has been no specific violation of the Federal Rules of Civil Procedure See generally Chambers v NASCO Inc 501 US 32 (1991) (discussing courtrsquos inherent power to control the litigation process) The range of available sanctions is broad It can include such sanctions as the payment of expenses incurred by the other party as a result of the destruction of the evidence an instruction to the jury authorizing it to draw an adverse inference from the destruction of the evidence a shifting of the burden of proof on the relevant issue or even judgment against the responsible party See eg Residential Funding Corp v DeGeorge Financial Corp 306 F3d 99 108 (2d Cir 2002) (adverse inference) Silvestri v General Motors Corp 271 F3d 583 593 (4th Cir 2001) (possibility of dismissal) Cf FED R CIV P 37(b)(2)(A) (listing remedies for failure to comply with discovery obligations)

In determining appropriate sanctions for spoliation courts consider both the level of culpability of the spoliating party and the degree of prejudice the loss of evidence has caused the other party Many courts impose severe sanctions (such as an adverse-inference instruction or the entry of judgment against the spoliating party) only when there is evidence of bad faith in the form of an intentional effort to hide information Eg Greyhound Lines Inc v Wade 485 F3d 1032 1035 (8th Cir 2007) (spoliation sanction requires intentional destruction out of desire ldquoto suppress the truthrdquo) However other courts have said that negligence in preserving evidence can

21

Federal Civil Procedure Analysis

support an adverse-inference instruction See Residential Funding 306 F3d at 108 (negligence enough under some circumstances)

Although a court might well order an evidentiary hearing on the issue of sanctions the facts presented do not seem appropriate for severe sanctions First the evidence was destroyed pursuant to the builderrsquos standard document retention plan and there is no evidence that the builder deliberately failed to suspend its usual procedures with the purpose of allowing the destruction of evidence Second the loss of this evidence will not severely hinder the customerrsquos presentation of his case The central issue is whether the foundation of the house was properly constructed If the construction job was poorly done the customer can present evidence derived from inspection of the premises to prove that point The customer can also depose witnesses about any issues that arose during construction

Under the circumstances a court is not likely to impose particularly severe sanctions although it might shift the burden to the builder to show that the foundation was properly constructed or it might require the builder to reimburse any expenses the customer incurs to discover and prove the facts about issues or disputes that arose during construction of the foundation

[NOTE The result reached by the examinee is less important than the examineersquos recognition that (a) a range of sanctions is available to the court and (b) the appropriate sanction depends both on the culpability of the builder and the prejudice suffered by the customer]

22

CRIMINAL LAW AND PROCEDURE ANALYSIS (Criminal Law and Procedure IIA amp D VE amp F)

ANALYSIS

Legal Problems

(1) Did charging the defendant with both theft and burglary constitute double jeopardy

(2) Did the jury instruction violate the due process clause either by relieving the prosecution of the burden of proving the element of intent or by shifting the burden to the defendant to disprove that element

(3) Did the sentence imposed in this case for the theft conviction unconstitutionally deprive the defendant of his right to a jury trial on the issue of the value of the stolen item

DISCUSSION

Summary

The trial court properly denied the defendantrsquos pretrial motion to dismiss the charges on double jeopardy grounds The defendant may be charged with and convicted of both theft and burglary Each of the charges has an element that the other does not Neither charge is a lesser-included offense nor are they multiplicitous Thus charging both theft and burglary does not violate double jeopardy

The jury instruction on the burglary charge was constitutionally flawed It could have been reasonably understood by the jury as either (1) an irrebuttable conclusive presumption (which relieved the prosecution of proving the element of intent and removed the issue from the jury) or (2) a rebuttable mandatory presumption (which unconstitutionally shifted the burden of proof on an element of a charged offense from the prosecution to the defendant)

Because the four-year sentence imposed by the judge was based on the judgersquos finding by a preponderance of the evidence that the value of the stolen ring exceeded $5000 the sentence violates the defendantrsquos right to a jury determination beyond a reasonable doubt of the value of the ring

Point One (30) Charging the defendant with theft and burglary did not constitute double jeopardy

The Double Jeopardy Clause of the Fifth Amendment provides that a person shall not be twice put in jeopardy for the ldquosame offenserdquo Thus the question is whether the elements of the theft charge are wholly contained in the burglary charge or vice versa If the elements of the lesser charge (theft) are not wholly contained in the greater charge (burglary)mdashie if each charge requires proof of a fact that the other does notmdashthen convicting the defendant of both crimes would not violate double jeopardy even when the two offenses occurred at the same time and are thus arguably part of the ldquosame transactionrdquo Blockburger v United States 284 US 299 304 (1932) See also Albernaz v United States 450 US 333 344 n3 (1981) United States v Dixon 509 US 688 704 (1993)

23

Criminal Law and Procedure Analysis

Here theft and burglary each require proof of an element not required for the other crime Burglary may be defined differently in different jurisdictions However it almost invariably requires entry into a building or dwelling of another with the specific intent to commit a felony therein and the crime of burglary is complete upon the entry into the building or dwelling with such intent See eg Cannon v Oklahoma 827 P2d 1339 1342 (Okla Crim App 1992) In contrast theft which also may be defined differently in different states almost invariably requires the taking and carrying away of an item of personal property belonging to another with the intent to steal or permanently deprive the owner of possession

Here the ldquotakingrdquo or ldquostealingrdquo element is not contained in the definition of burglary and the ldquoentryrdquo element of burglary is not contained in the definition of theft Because theft is not a lesser-included offense of burglary and burglary is not a lesser-included offense of theft charging the defendant for both burglary and theft did not violate double jeopardy and the court properly denied the defense motion on those grounds Yparrea v Dorsey 64 F3d 577 579ndash80 (10th Cir 1995) citing Blockburger 284 US at 304

Finally the defendantrsquos motion to dismiss all the charges on double jeopardy grounds was improper because if both charges were for the same offense the motion should have requested dismissal of one charge not both

Point Two (35) The jury instruction on the burglary charge violated the Due Process Clause because it created either (1) an irrebuttable conclusive presumption (which relieved the prosecution of proving the element of intent and removed that issue from the jury) or (2) a rebuttable mandatory presumption (which unconstitutionally shifted the burden of proof on an element of a charged offense to the defendant)

The Supreme Court has interpreted the Due Process Clause of the US Constitution to require that the prosecution prove all elements of an offense beyond a reasonable doubt See In re Winship 397 US 358 364 (1970) The burden of proof cannot be shifted to the defendant by presuming an essential element upon proof of other elements of the offense because shifting the burden of persuasion with respect to any element of a criminal offense is contrary to the Due Process Clause See Mullaney v Wilbur 421 US 684 (1975)

The crime of burglary includes entry into a building or dwelling with the specific intent to commit a felony therein The requirement that the prosecutor prove beyond a reasonable doubt that the defendant had this specific intent distinguishes burglary from general-intent crimes like trespass See Sandstrom v Montana 442 US 510 523 (1979)

Here the jury was instructed that if ldquoafter consideration of all the evidence presented by the prosecution and defense you find beyond a reasonable doubt that the defendant entered the dwelling without the ownersrsquo consent you may presume that the defendant entered with the intent to commit a felony thereinrdquo This instruction was unconstitutional because it created either an irrebuttable conclusive presumption or a rebuttable mandatory presumption

A conclusive presumption is ldquoan irrebuttable direction by the court to find intent once convinced of the facts triggering the presumptionrdquo Id at 517 Here the jurors were instructed that once the prosecutor established that the defendant entered the neighborsrsquo house without consent they ldquomay presumerdquo that he intended to commit a felony therein The jurors may have reasonably concluded from this instruction that if they found that the defendant intended to enter his neighborsrsquo home without permission they must further find that he entered with the specific intent to commit a felony therein Because this instruction could operate as a conclusive

24

Criminal Law and Procedure Analysis

irrebuttable presumption by eliminating intent ldquoas an ingredient of the offenserdquo it violated due process by relieving the prosecution of the burden of proof for this element Id at 522

In the alternative the jury instruction could have been reasonably understood to create a rebuttable mandatory presumption which ldquotells [the jury] they must find the elemental fact upon proof of the basic fact at least unless the defendant has come forward with some evidence to rebut the presumed connection between the two factsrdquo County Court of Ulster County New York v Allen 442 US 140 157 (1979) The due process problem created by rebuttable mandatory presumptions is that ldquo[t]o the extent that the trier of fact is forced to abide by the presumption and may not reject it based on an independent evaluation of the particular facts presented by the State the analysis of the presumptionrsquos constitutional validity is logically divorced from those facts and based on the presumptionrsquos accuracy in the run of casesrdquo Id at 159

Unlike irrebuttable conclusive presumptions rebuttable mandatory presumptions are not always per se violations of the Due Process Clause However the Supreme Court of the United States has held that jury instructions that could reasonably be understood as shifting the burden of proof to the defendant on an element of the offense are unconstitutional Francis v Franklin 471 US 307 (1985) Here the argument that the jury instruction operated as a rebuttable mandatory presumption is supported by the fact that the judge also instructed the jury to ldquoconsider[ ] all the evidence presented by the prosecution and defenserdquo However even if the instruction created a rebuttable mandatory presumption it would be unconstitutional because it shifted the burden to the defense on an element of the offense Sandstrom 442 US at 524 Mullaney 421 US at 686

[NOTE Whether an examinee identifies the jury instruction as containing a ldquoconclusiverdquo or ldquomandatoryrdquo presumption is less important than the examineersquos analysis of the constitutional infirmities]

Point Three (35) The trial court violated the defendantrsquos Sixth Amendment right to a jury trial on an essential element of the offense when it found by a preponderance of the evidence that the ring was worth over $5000 and increased the defendantrsquos sentence based on this finding

In the statutory scheme under which the defendant was tried and convicted a Class D felony theft is defined as theft of item(s) with a value between $2500 and $10000 The jury found that the value of the diamond ring was at least $2500 and convicted the defendant of felony theft However at sentencing the trial court made a separate finding by a preponderance of the evidence that the value of the ring was greater than $5000 Following the statutersquos two-tiered sentencing scheme the judge then imposed on the defendant a sentence that was one year longer than the maximum that would otherwise have been allowed

The judgersquos sentence was unconstitutional because it violated the defendantrsquos Sixth Amendment right to a jury trial on this question The Supreme Court held in Apprendi v New Jersey 530 US 466 (2000) that ldquo[o]ther than the fact of a prior conviction any fact that increases the penalty for a crime beyond the prescribed statutory maximum must be submitted to a jury and proved beyond a reasonable doubtrdquo because ldquo[i]t is unconstitutional for a legislature to remove from the jury the assessment of facts that increase the prescribed range of penalties to which a criminal defendant is exposed [because] such facts must be established by proof beyond a reasonable doubtrdquo Id The Court reaffirmed Apprendi in Blakely v Washington 542 US 296 (2004) holding that the ldquolsquostatutory maximumrsquo for Apprendi purposes is the maximum sentence a judge may impose solely on the basis of the facts reflected in the jury verdict or admitted by the defendantrdquo Id at 303 (emphasis in original) In United States v Booker 543 US 220 (2005)

25

Criminal Law and Procedure Analysis

the Court relied on Blakely and Apprendi to conclude that protecting a defendantrsquos Sixth Amendment right to a jury trial required that ldquo[a]ny fact which is necessary to support a sentence exceeding the maximum authorized by the facts established by a plea of guilty or a jury verdict must be admitted by the defendant or proved to a jury beyond a reasonable doubtrdquo Id at 244

Thus in order to constitutionally increase a sentence above the statutory maximum of three years the jury must have found beyond a reasonable doubt that the value of the ring exceeded $5000 Here the court made the finding based on an appraisal proffered by the prosecutor only at sentencing and the judgersquos finding was by a preponderance of the evidence rather than beyond a reasonable doubt

26

AGENCY AND PARTNERSHIP ANALYSIS __________ (Agency and Partnership VA amp C VI)

ANALYSIS

Legal Problems

(1) Is a partner in a general partnership personally liable on a claim arising from misrepresentations by another partner made in the course of the partnership business

(2) Does a newly admitted partner in a general partnership become personally liable on existing claims against the partnership

(3) After the filing by a general partnership of a statement of qualification as a limited liability partnership are the partners personally liable as partners on (a) an existing claim against the general partnership and (b) a claim against the partnership that arose after the filing

DISCUSSION

Summary

Adam and Ben formed a general partnership under which they were jointly and severally liable for obligations of the partnership Thus Adam was personally liable for misrepresentations by Ben made in the ordinary course of the partnership business

Upon joining the general partnership Diane became personally liable for the obligations of the partnership arising after her admission but not for obligations pre-existing her admission such as the collectorrsquos claim

By filing a statement of qualification the three partners properly elected limited liability partnership status As partners in an LLP none of the three partners is personally liable as a partner for partnership obligations arising after the election such as the claim by the driverrsquos estate The election however does not change their personal liability on pre-existing claims that arose before the election such as the collectorrsquos claim

Point One (30) As a general partner of Empire a general partnership Adam became personally liable on the collectorrsquos claim a valid claim against the partnership that arose because of Benrsquos wrongful act in the ordinary course of the partnership business

When the collectorrsquos claim arose Empire was a general partnership composed of Adam and Ben Under UPA (1997) sect 306(a) partners of a general partnership are liable jointly and severally for all obligations of the partnership Under UPA (1997) sect 305(a) the partnership could become obligated for the loss caused to the collector as a result of the misrepresentation by Ben provided he was acting in the ordinary course of the partnership business Because there was no statement that limited his partnership authority Ben as partner was ldquoan agent of the partnership for the purpose of its businessrdquo See UPA (1997) sect 301(1) Benrsquos misrepresentation to the collector even if intentional appears to be in the ordinary course of the partnershiprsquos business of dealing

27

Agency and Partnership Analysis

in antique cars Thus Benrsquos wrongful act created a partnership obligation for which Adam was jointly and severally liable

[NOTE Generally a partnership creditor must ldquoexhaust the partnershiprsquos assets before levying on a judgment debtor partnerrsquos individual property where the partner is personally liable for the partnership obligationrdquo as a result of his status as a partner UPA (1997) sect 307 cmt 4 As the UPA comments explain this places Adam more in the position of guarantor than principal debtor on the partnership obligation Id cmt 4 Although an examinee might discuss this point the call focuses on whether Adam is personally liable not how the liability might be enforced]

Point Two (30) Because the collectorrsquos claim arose before Diane joined Empire Diane did not become personally liable on the claim

Diane was admitted to Empire when it was a general partnership and after the collectorrsquos claim arose While the general rule under UPA (1997) sect 306(a) is that the partners of a general partnership are liable jointly and severally for all obligations of the partnership there is a special rule for partners who are admitted during the duration of the partnership Under UPA (1997) sect 306(b) a person admitted to an existing partnership is not personally liable for any partnership obligations incurred before the personrsquos admission Because Diane was admitted to Empire after the collectorrsquos claim arose Diane is not personally liable on the claim

Dianersquos knowledge of the pre-existing claim and her stated concern about becoming liable on the collectorrsquos claim do not change her personal nonliability to the collector Although partners who have a liability shield can assume liability to third parties through private contractual guarantees or modifications to the partnership agreement Dianersquos stated concern constituted neither a guaranty to the collector nor ldquoan intentional waiver of liability protectionsrdquo See UPA (1997) sect 306 cmt 3 (describing methods for waiver of liability protections under sect 306(c) applicable in limited liability partnerships)

At most Diane will lose her investment in the partnership as a result of the collectorrsquos claim Although Diane did not become personally liable on the collectorrsquos claim when she joined the partnership the $250000 she contributed to the partnership is ldquoat risk for the satisfaction of existing partnership debtsrdquo UPA (1997) sect 306 cmt 2

Point Three (40) Filing the statement of qualification was effective to elect limited liability partnership status Despite this new status Adam and Ben remain personally liable on the collectorrsquos claim which arose before the election But as partners in an LLP neither Adam Ben nor Diane is personally liable as a partner on the driverrsquos estatersquos claim which arose after the election

Under UPA (1997) sect 1001 a general partnership can make an election and become a limited liability partnershipmdashif the partners approve the conversion by a vote equivalent to that necessary to amend the partnership agreement and the partnership then files a statement of qualification that specifies the name of the partnership its principal office and its election to be an LLP Here the partners agreed unanimouslymdashsufficient to amend their agreement under UPA (1997) sect 401(j)mdashand the statement of qualification was filed In addition the name of Empire LLP properly included an appropriate ending ldquoLLPrdquo See UPA (1997) sect 1002

Although another way to effectuate a ldquoconversionrdquo (as suggested by Benrsquos lawyer) is to form a new LLP and transfer the assets of the old general partnership to the new LLP the

28

Agency and Partnership Analysis

method used here (approval by the partners and the filing of a statement of qualification) is also sufficient to create LLP status

Thus Empire became Empire LLP as of the date of filing of the statement of qualification See UPA (1997) sect 1001 What effect did this have on the collectorrsquos claim which predated the filing According to UPA (1997) sect 306(c) an obligation incurred while a partnership is an LLP is solely a partnership obligation As the collectorrsquos claim predated the LLP Adam and Ben remain personally liable on the collectorrsquos claim Diane on the other hand was not personally liable on the collectorrsquos claim either before or after the filing of the statement of qualification See Point Two above

The driverrsquos estatersquos claim arose after Empire became Empire LLP Under UPA (1997) sect 306(c) an obligation incurred while a partnership is an LLP is solely a partnership obligationThus Adam Ben and Diane as partners are all protected from personal liability on the driverrsquos estatersquos claim But there may be personal liability if any of them was negligent or otherwise acted wrongfully by not informing the buyer of the bad suspension that caused the accident

29

National Conference of Bar Examiners 302 South Bedford Street | Madison WI 53703-3622 Phone 608-280-8550 | Fax 608-280-8552 | TDD 608-661-1275

wwwncbexorg e-mail contactncbexorg

  • Preface
  • Description of the MEE
  • Instructions
  • February 2014 Questions
    • Constitutinal Law Question
    • Trusts and Future Interests Question
    • Secured Transactions Question
    • Federal Civil Procedure Question
    • Criminal Law and Procedure Question
    • Agency and Partnership Question
      • February 2014 Analyses
        • Constitutional Law Analysis
        • Trust and Future Interests Analysis
        • Secured Transactions Analysis
        • Federal Civil Procedure Analysis
        • Criminal Law and Procedure Analysis
        • Agency and Partnership Analysis
            • ltlt13 ASCII85EncodePages false13 AllowTransparency false13 AutoPositionEPSFiles true13 AutoRotatePages None13 Binding Left13 CalGrayProfile (Dot Gain 20)13 CalRGBProfile (sRGB IEC61966-21)13 CalCMYKProfile (US Web Coated 050SWOP051 v2)13 sRGBProfile (sRGB IEC61966-21)13 CannotEmbedFontPolicy Error13 CompatibilityLevel 1413 CompressObjects Tags13 CompressPages true13 ConvertImagesToIndexed true13 PassThroughJPEGImages true13 CreateJobTicket false13 DefaultRenderingIntent Default13 DetectBlends true13 DetectCurves 0000013 ColorConversionStrategy CMYK13 DoThumbnails false13 EmbedAllFonts true13 EmbedOpenType false13 ParseICCProfilesInComments true13 EmbedJobOptions true13 DSCReportingLevel 013 EmitDSCWarnings false13 EndPage -113 ImageMemory 104857613 LockDistillerParams false13 MaxSubsetPct 10013 Optimize true13 OPM 113 ParseDSCComments true13 ParseDSCCommentsForDocInfo true13 PreserveCopyPage true13 PreserveDICMYKValues true13 PreserveEPSInfo true13 PreserveFlatness true13 PreserveHalftoneInfo false13 PreserveOPIComments true13 PreserveOverprintSettings true13 StartPage 113 SubsetFonts true13 TransferFunctionInfo Apply13 UCRandBGInfo Preserve13 UsePrologue false13 ColorSettingsFile ()13 AlwaysEmbed [ true13 ]13 NeverEmbed [ true13 ]13 AntiAliasColorImages false13 CropColorImages true13 ColorImageMinResolution 30013 ColorImageMinResolutionPolicy OK13 DownsampleColorImages true13 ColorImageDownsampleType Bicubic13 ColorImageResolution 30013 ColorImageDepth -113 ColorImageMinDownsampleDepth 113 ColorImageDownsampleThreshold 15000013 EncodeColorImages true13 ColorImageFilter DCTEncode13 AutoFilterColorImages true13 ColorImageAutoFilterStrategy JPEG13 ColorACSImageDict ltlt13 QFactor 01513 HSamples [1 1 1 1] VSamples [1 1 1 1]13 gtgt13 ColorImageDict ltlt13 QFactor 01513 HSamples [1 1 1 1] VSamples [1 1 1 1]13 gtgt13 JPEG2000ColorACSImageDict ltlt13 TileWidth 25613 TileHeight 25613 Quality 3013 gtgt13 JPEG2000ColorImageDict ltlt13 TileWidth 25613 TileHeight 25613 Quality 3013 gtgt13 AntiAliasGrayImages false13 CropGrayImages true13 GrayImageMinResolution 30013 GrayImageMinResolutionPolicy OK13 DownsampleGrayImages true13 GrayImageDownsampleType Bicubic13 GrayImageResolution 30013 GrayImageDepth -113 GrayImageMinDownsampleDepth 213 GrayImageDownsampleThreshold 15000013 EncodeGrayImages true13 GrayImageFilter DCTEncode13 AutoFilterGrayImages true13 GrayImageAutoFilterStrategy JPEG13 GrayACSImageDict ltlt13 QFactor 01513 HSamples [1 1 1 1] VSamples [1 1 1 1]13 gtgt13 GrayImageDict ltlt13 QFactor 01513 HSamples [1 1 1 1] VSamples [1 1 1 1]13 gtgt13 JPEG2000GrayACSImageDict ltlt13 TileWidth 25613 TileHeight 25613 Quality 3013 gtgt13 JPEG2000GrayImageDict ltlt13 TileWidth 25613 TileHeight 25613 Quality 3013 gtgt13 AntiAliasMonoImages false13 CropMonoImages true13 MonoImageMinResolution 120013 MonoImageMinResolutionPolicy OK13 DownsampleMonoImages true13 MonoImageDownsampleType Bicubic13 MonoImageResolution 120013 MonoImageDepth -113 MonoImageDownsampleThreshold 15000013 EncodeMonoImages true13 MonoImageFilter CCITTFaxEncode13 MonoImageDict ltlt13 K -113 gtgt13 AllowPSXObjects false13 CheckCompliance [13 None13 ]13 PDFX1aCheck false13 PDFX3Check false13 PDFXCompliantPDFOnly false13 PDFXNoTrimBoxError true13 PDFXTrimBoxToMediaBoxOffset [13 00000013 00000013 00000013 00000013 ]13 PDFXSetBleedBoxToMediaBox true13 PDFXBleedBoxToTrimBoxOffset [13 00000013 00000013 00000013 00000013 ]13 PDFXOutputIntentProfile ()13 PDFXOutputConditionIdentifier ()13 PDFXOutputCondition ()13 PDFXRegistryName ()13 PDFXTrapped False1313 CreateJDFFile false13 Description ltlt13 ARA 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 BGR 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 CHS ltFEFF4f7f75288fd94e9b8bbe5b9a521b5efa7684002000410064006f006200650020005000440046002065876863900275284e8e9ad88d2891cf76845370524d53705237300260a853ef4ee54f7f75280020004100630072006f0062006100740020548c002000410064006f00620065002000520065006100640065007200200035002e003000204ee553ca66f49ad87248672c676562535f00521b5efa768400200050004400460020658768633002gt13 CHT ltFEFF4f7f752890194e9b8a2d7f6e5efa7acb7684002000410064006f006200650020005000440046002065874ef69069752865bc9ad854c18cea76845370524d5370523786557406300260a853ef4ee54f7f75280020004100630072006f0062006100740020548c002000410064006f00620065002000520065006100640065007200200035002e003000204ee553ca66f49ad87248672c4f86958b555f5df25efa7acb76840020005000440046002065874ef63002gt13 CZE 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 DAN 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 DEU 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 ESP ltFEFF005500740069006c0069006300650020006500730074006100200063006f006e0066006900670075007200610063006900f3006e0020007000610072006100200063007200650061007200200064006f00630075006d0065006e0074006f00730020005000440046002000640065002000410064006f0062006500200061006400650063007500610064006f00730020007000610072006100200069006d0070007200650073006900f3006e0020007000720065002d0065006400690074006f007200690061006c00200064006500200061006c00740061002000630061006c0069006400610064002e002000530065002000700075006500640065006e00200061006200720069007200200064006f00630075006d0065006e0074006f00730020005000440046002000630072006500610064006f007300200063006f006e0020004100630072006f006200610074002c002000410064006f00620065002000520065006100640065007200200035002e003000200079002000760065007200730069006f006e0065007300200070006f00730074006500720069006f007200650073002egt13 ETI 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 FRA 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 GRE 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 HEB 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 HRV (Za stvaranje Adobe PDF dokumenata najpogodnijih za visokokvalitetni ispis prije tiskanja koristite ove postavke Stvoreni PDF dokumenti mogu se otvoriti Acrobat i Adobe Reader 50 i kasnijim verzijama)13 HUN 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 ITA 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 JPN ltFEFF9ad854c18cea306a30d730ea30d730ec30b951fa529b7528002000410064006f0062006500200050004400460020658766f8306e4f5c6210306b4f7f75283057307e305930023053306e8a2d5b9a30674f5c62103055308c305f0020005000440046002030d530a130a430eb306f3001004100630072006f0062006100740020304a30883073002000410064006f00620065002000520065006100640065007200200035002e003000204ee5964d3067958b304f30533068304c3067304d307e305930023053306e8a2d5b9a306b306f30d530a930f330c8306e57cb30818fbc307f304c5fc59808306730593002gt13 KOR ltFEFFc7740020c124c815c7440020c0acc6a9d558c5ec0020ace0d488c9c80020c2dcd5d80020c778c1c4c5d00020ac00c7a50020c801d569d55c002000410064006f0062006500200050004400460020bb38c11cb97c0020c791c131d569b2c8b2e4002e0020c774b807ac8c0020c791c131b41c00200050004400460020bb38c11cb2940020004100630072006f0062006100740020bc0f002000410064006f00620065002000520065006100640065007200200035002e00300020c774c0c1c5d0c11c0020c5f40020c2180020c788c2b5b2c8b2e4002egt13 LTH 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 LVI 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 NLD (Gebruik deze instellingen om Adobe PDF-documenten te maken die zijn geoptimaliseerd voor prepress-afdrukken van hoge kwaliteit De gemaakte PDF-documenten kunnen worden geopend met Acrobat en Adobe Reader 50 en hoger)13 NOR 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 POL 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 PTB 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 RUM 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 RUS ltFEFF04180441043f043e043b044c04370443043904420435002004340430043d043d044b04350020043d0430044104420440043e0439043a043800200434043b044f00200441043e043704340430043d0438044f00200434043e043a0443043c0435043d0442043e0432002000410064006f006200650020005000440046002c0020043c0430043a04410438043c0430043b044c043d043e0020043f043e04340445043e0434044f04490438044500200434043b044f00200432044b0441043e043a043e043a0430044704350441044204320435043d043d043e0433043e00200434043e043f0435044704300442043d043e0433043e00200432044b0432043e04340430002e002000200421043e043704340430043d043d044b04350020005000440046002d0434043e043a0443043c0435043d0442044b0020043c043e0436043d043e0020043e0442043a0440044b043204300442044c002004410020043f043e043c043e0449044c044e0020004100630072006f00620061007400200438002000410064006f00620065002000520065006100640065007200200035002e00300020043800200431043e043b043504350020043f043e04370434043d043804450020043204350440044104380439002egt13 SKY 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 SLV 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 SUO 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 SVE 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 TUR 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 UKR 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 ENU (Use these settings to create Adobe PDF documents best suited for high-quality prepress printing Created PDF documents can be opened with Acrobat and Adobe Reader 50 and later)13 gtgt13 Namespace [13 (Adobe)13 (Common)13 (10)13 ]13 OtherNamespaces [13 ltlt13 AsReaderSpreads false13 CropImagesToFrames true13 ErrorControl WarnAndContinue13 FlattenerIgnoreSpreadOverrides false13 IncludeGuidesGrids false13 IncludeNonPrinting false13 IncludeSlug false13 Namespace [13 (Adobe)13 (InDesign)13 (40)13 ]13 OmitPlacedBitmaps false13 OmitPlacedEPS false13 OmitPlacedPDF false13 SimulateOverprint Legacy13 gtgt13 ltlt13 AddBleedMarks false13 AddColorBars false13 AddCropMarks false13 AddPageInfo false13 AddRegMarks false13 ConvertColors ConvertToCMYK13 DestinationProfileName ()13 DestinationProfileSelector DocumentCMYK13 Downsample16BitImages true13 FlattenerPreset ltlt13 PresetSelector MediumResolution13 gtgt13 FormElements false13 GenerateStructure false13 IncludeBookmarks false13 IncludeHyperlinks false13 IncludeInteractive false13 IncludeLayers false13 IncludeProfiles false13 MultimediaHandling UseObjectSettings13 Namespace [13 (Adobe)13 (CreativeSuite)13 (20)13 ]13 PDFXOutputIntentProfileSelector DocumentCMYK13 PreserveEditing true13 UntaggedCMYKHandling LeaveUntagged13 UntaggedRGBHandling UseDocumentProfile13 UseDocumentBleed false13 gtgt13 ]13gtgt setdistillerparams13ltlt13 HWResolution [2400 2400]13 PageSize [612000 792000]13gtgt setpagedevice13

Page 17: February 2014 MEE Questions and AnalysesPreface The Multistate Essay Examination (MEE) is developed by the National Conference of Bar Examiners (NCBE). This publication includes the

TRUSTS AND FUTURE INTERESTS ANALYSIS ____ (Trusts and Future Interests IE3 I5 IIIA amp B)

ANALYSIS

Legal Problems

(1) How should rents dividends and sales proceeds received by the trustee prior to receipt of the sonrsquos letter have been allocated between trust income and principal

(2)(a) Did the remainder interest in the trust accelerate and become immediately payable to the daughterrsquos minor child upon the trusteersquos receipt of the sonrsquos letter and if not how should the trustee handle the distribution of the principal in the future

(2)(b) Following the trusteersquos receipt of the sonrsquos letter how should the trustee distribute future receipts of income prior to the distribution of the principal

DISCUSSION

Summary

Prior to the trusteersquos receipt of the sonrsquos letter cash dividends and rents should have been allocated to trust income and were distributable to the son the income beneficiary of the trust sales proceeds and stock dividends should have been allocated to principal

Because the sonrsquos letter to the trustee did not result in a valid disclaimer under state law (having been made more than nine months after the testatorrsquos death) the son is not deemed to have predeceased the testator Because the son is still living the class gift to the testatorrsquos grandchildren who survive the son has not closed and is not possessory it will not become possessory until the son dies The daughterrsquos minor child being the testatorrsquos only living grandchild is not currently entitled to a distribution of trust principal Trust principal will instead be distributable upon the sonrsquos death to the testatorrsquos then-living grandchildren or if there are none to the testatorrsquos then-living heirs

As for future income the trustee should either distribute the trust income to the son and the daughter as the testatorrsquos heirs accumulate the income for future distribution to those individuals ultimately entitled to the trust principal or distribute it to those presumptively entitled to the principal upon the sonrsquos death ie the daughterrsquos minor child

Point One (45) Cash dividends and rents are allocable to income sales proceeds and stock dividends are allocable to principal Items allocable to income for the period prior to the sonrsquos attempted disclaimer were distributable to the son

Receipts earned during the administration of a trust are allocable either to income or to principal Almost all states have adopted the most recent or an earlier version of the Uniform Principal and Income Act (the Act) which specifies how such receipts should be allocated

Under the Act rents (UNIF PRIN amp INC ACT (2000) sect 405 UNIF PRIN amp INC ACT (1962) sect 3(a)(1)) and cash dividends received from a corporation (UNIF PRIN amp INC ACT (2000) sect 401(b) UNIF PRIN amp INC ACT (1962) sect 6(d)) are allocable to income and are distributable to the income beneficiary of the trust

14

Trusts and Future Interests Analysis Sales proceeds (UNIF PRIN amp INC ACT (2000) sect 404(2) UNIF PRIN amp INC ACT (1962)

sect 3(b)(1)) and dividends paid in the stock of the distributing corporation (UNIF PRIN amp INC ACT (2000) sect 401(c)(1) UNIF PRIN amp INC ACT (1962) sect 3(b)(4)) are allocable to principal and added to the principal of the trust

Here the cash dividends and office building rents should have been allocated to income and until the trustee received the sonrsquos letter should have been distributed to him as the sole income beneficiary of the trust The stock dividend and proceeds from the sale of the office building should have been allocated to principal and held by the trustee for future distribution to the ultimate remaindermen of the trust

[NOTE The 2000 Uniform Principal and Income Act has been adopted in Alabama Arkansas Colorado Connecticut the District of Columbia Hawaii Idaho Iowa Kentucky Missouri Montana Nebraska New Mexico North Dakota Oregon South Dakota Utah and West Virginia]

Point Two(a) (45) Because the son did not disclaim within nine months of the testatorrsquos death there is no valid disclaimer under state law Therefore the son is not deemed to have predeceased the testator Furthermore because of the express survivorship contingency in the will the remainder in the trust does not accelerate and become distributable until the son in fact dies When the son dies the trust principal will be distributable to the testatorrsquos then-living grandchildren or if none then to the testatorrsquos then-living heirs

When a trust remainder is given to a class the class closes (ie no new persons can join the class) when there is no outstanding income interest and at least one member of the class is then entitled to demand possession of his or her share of the remainder This principle is called the rule of convenience See generally HERBERT HOVENKAMP amp SHELDON F KURTZ PRINCIPLES OF PROPERTY LAW 199ndash200 (6th ed 2005) A class member may demand possession of his or her share of the remainder upon termination of the income interest only when the class memberrsquos interest is not otherwise subject to a condition precedent See id

When a beneficiary timely disclaims an interest in a trust that beneficiary is treated as if he had predeceased the testator Here had the son disclaimed within nine months of the testatorrsquos death as required by the state statute he would have been deemed to have predeceased the testator This would have closed the class of remaindermen and the testatorrsquos then-living grandchildren (ie the daughterrsquos child) would have been entitled to the trust principal However under the state statute the sonrsquos disclaimer was not timely because he did not disclaim within nine months of the testatorrsquos death Thus because the statute is inapplicable and the son is still alive the class of grandchildren entitled to share in trust principal did not close

Because here the statute is inapplicable due to the sonrsquos failure to comply with the statutory time requirements then presumably the common-law rule allowing disclaimers (aka renunciations) at any time should apply Under the common law if a life estate is renounced the remainder interest accelerates and becomes immediately distributable to the remaindermen of the trust if the remainder is vested but not if the remainder is contingent JESSE DUKEMINIER amp ROBERT H SITKOFF WILLS TRUSTS AND ESTATES 844ndash845 (9th ed 2013) Here because the remainder is contingent upon there being grandchildren who survive the son the remainder will not accelerate It will remain open until the son dies leaving open the possibility that additional grandchildren will be included in the class or the daughterrsquos child could fall out of the class because that child fails to survive the son

And if none of the testatorrsquos grandchildren survive the son the trust principal will be distributed to the testatorrsquos heirs living at the sonrsquos death

15

Trusts and Future Interests Analysis

Point Two(b) (10) Until the trust terminates the trustee must continue to hold the trust assets The distribution of income in the meantime is unclear There are at least three possibilities Income earned on the undistributed assets could be distributed to the son and daughter as the testatorrsquos heirs accumulated and added to principal for distribution to the ultimate remaindermen or distributed from time to time to those persons who are presumptively remaindermen

When trust principal is not immediately distributable the trustee must continue to hold trust assets until the ultimate remaindermen are ascertained During this period trust income will be distributed or retained according to any instructions contained in the trust instrument See WILLIAM M MCGOVERN JR SHELDON F KURTZ amp DAVID M ENGLISH WILLS TRUSTS amp ESTATES sect 102 (4th ed 2010)

Here the testator did not specify what the trustee should do with trust income in the event the sonrsquos disclaimer did not comply with the state statute There are at least three approaches One approach would have the trustee distribute the trust income to the testatorrsquos heirs on the theory that the income represents property that was not disposed of by the testatorrsquos will and which thus passes by partial intestacy to the testatorrsquos heirs A second approach would have the trustee accumulate trust income for distribution to the ultimate remaindermen Under this approach only those individuals ultimately entitled to the principal would share in the income A third approach would have the trustee distribute trust income to those individuals who would be the remaindermen if the trust were to terminate when the income is received by the trustee under this approach trust income would be distributed to the daughterrsquos minor child until another presumptive remainderman is born This approach could result in individuals not ultimately entitled to principal say because they do not survive the son receiving income It could also result in a disproportionate distribution of income among the individuals ultimately entitled to income

[NOTE Examinees should demonstrate a recognition and understanding of the income-allocation problem and the alternatives available to address that issue There is no widely accepted solution to the problem Examinees who cite any of these possible problem-solving approaches may receive credit]

16

SECURED TRANSACTIONS ANALYSIS (Secured Transactions IB IID E amp F IIIB IVA B amp F)

ANALYSIS

Legal Problems

(1)(a) What is the nature of the bankrsquos claim to the businessrsquos equipment

(1)(b) What is the nature of the finance companyrsquos claim to the businessrsquos equipment

(1)(c) As between the bank and the finance company whose claim to the businessrsquos equipment has priority

(2) Do the claims of the bank and the finance company continue in the item of equipment sold by the business to the competitor

DISCUSSION

Summary

The bank and the finance company both have perfected security interests in the businessrsquos equipment Even though the finance companyrsquos perfected security interest was created first the bankrsquos perfected security interest has priority because the bankrsquos financing statement was filed before the finance companyrsquos financing statement The security interests of the bank and the finance company continue in the item of equipment sold by the business to the competitor because their security interests were perfected and the competitor was not a buyer in ordinary course of business

Point One(a) (25) The bank has a perfected security interest in the businessrsquos equipment

The bank has met all criteria necessary for it to have an attached and enforceable security interest in the businessrsquos equipment First value must be given UCC sect 9-203(b)(1) This criterion is fulfilled by the loan made by the bank to the business Second the debtor must have rights in the collateral UCC sect 9-203(b)(2) Clearly the business has rights in its equipment Third either the secured party must take possession of the collateral or the debtor must authenticate a security agreement containing a description of the collateral UCC sect 9-203(b)(3) The agreement that the business owner signed is a ldquosecurity agreementrdquo because it is an agreement that creates or provides for a security interest UCC sect 9-102(a)(74) By signing the security agreement the business owner authenticated it UCC sect 9-102(a)(7) Therefore all three criteria are fulfilled and the bank has an enforceable and attached security interest

A security interest is perfected when it has attached and when any additional steps required for perfection have occurred UCC sect 9-308(a) Generally speaking the additional steps will either be possession of the collateral by the secured party or the filing of a financing statement with respect to the collateral See UCC sectsect 9-310 9-313 In this case the bank filed a financing statement naming the debtor and sufficiently indicating the collateral The collateral indication is sufficient because it identifies the collateral by type of property See UCC sectsect 9-504 9-108 The fact that the financing statement was filed before the security interest was created is

17

Secured Transactions Analysis

not a problem Even though the security agreement had not yet been signed the business had authorized the filing of the financing statement in an authenticated record UCC sect 9-509(a)(1) Moreover the financing statement may be filed before the security agreement is created UCC sect 9-502(d)

Point One(b) (10) The finance company also has a perfected security interest in the businessrsquos equipment

The finance companyrsquos security interest is enforceable and attached for the same reasons as the bankrsquos security interest The loan from the finance company to the business constitutes value the business has rights in the collateral and the business owner has authenticated a security agreement containing a description of the collateral The finance companyrsquos security interest is perfected because the finance company filed a financing statement with respect to it that provides that the business is the debtor and indicates that the collateral is equipment

Point One(c) (30) The bankrsquos security interest has priority over the finance companyrsquos security interest because the bankrsquos financing statement was filed first

As between two perfected security interests the general rule is that the security interest that was the earlier to be either perfected or the subject of a filed financing statement has priority UCC sect 9-322(a)(1) While the finance companyrsquos security interest was perfected before the bankrsquos (March 15 vs March 22) the bankrsquos financing statement was filed even earlier on March 2 Thus under the first-to-file-or-perfect rule of UCC sect 9-322(a)(1) the bankrsquos security interest has priority No exceptions to the general rule apply here

Point Two (35) A security interest in collateral continues notwithstanding its sale unless an exception applies Because the security interests of the bank and the finance company were perfected and the competitor was not a buyer in ordinary course of business no exception applies and the security interests of both creditors continue in the equipment sold to the competitor

As a general rule a security interest in collateral continues notwithstanding the fact that the debtor has sold the collateral to another person UCC sect 9-315(a)(1) Thus unless an exception applies the security interests of the bank and the finance company will continue in the item of equipment sold to the competitor

A buyer of goods will take free of an unperfected security interest in those goods See UCC sect 9-317(a)(2) However when the competitor bought the businessrsquos equipment both the bank and the finance company had perfected security interests in the equipment

A buyer can take free even of a perfected security interest in goods if the buyer is a ldquobuyer in ordinary course of businessrdquo See UCC sect 9-320(a) However the competitor was not a buyer in ordinary course of business To be a ldquobuyer in ordinary course of businessrdquo a buyer must buy goods from a seller that is in the business of selling goods of that kind See UCC sect 1-201(b)(9) The competitor bought this equipment from a seller that is not in the business of selling goods of this kind so the competitor was not a buyer in ordinary course of business with respect to these goods

Because no exception applies the security interests of the bank and the finance company continue even after the item of equipment was sold to the competitor

18

FEDERAL CIVIL PROCEDURE ANALYSIS (Federal Civil Procedure IVD)

ANALYSIS

Legal Problems

(1) Is a document prepared in the course of a contract dispute protected from discovery as ldquowork productrdquo when there is no evidence that the document was prepared in anticipation of litigation

(2)(a) Is a partyrsquos failure to provide relevant electronically stored information excused when the information was destroyed pursuant to a routine document retention scheme at a time when litigation was contemplated by the destroying party

(2)(b) What sanctions should be imposed on a party for allowing the destruction of evidence that is relevant to potential future litigation

DISCUSSION

Summary

The report prepared by the structural engineer is probably not work product and is thus discoverable The engineer examined the foundation of the house at the customerrsquos request and the engineerrsquos findings are potentially relevant to the customerrsquos claim that the foundation is defective The report was not prepared in anticipation of litigation The customer appears to have sought the engineerrsquos opinion in response to the builderrsquos offer to fix any problems with the foundation that an engineer might identify Because the report was not prepared in anticipation of litigation it is not protected by the work-product doctrine

The builder should have taken appropriate steps to preserve evidence including suspending its document retention program as soon as it began planning for litigationmdashie on July 10 Its destruction of potentially relevant material after that date was wrongful However a court is unlikely to impose severe sanctions on the builder because there are no facts indicating that the builder acted in bad faith and the customer can prove that the foundation is defective without the destroyed emails

Point One (40) The customer must turn over the engineerrsquos report because it was not prepared in anticipation of litigation

In general a party to a lawsuit in federal court ldquomay obtain discovery regarding any nonprivileged matter that is relevant to any partyrsquos claim or defenserdquo FED R CIV P 26(b)(1) (2009) This includes the right to inspect and copy documents in the other partyrsquos possession FED R CIV P 34(a)(1) Here the customer hired a structural engineer to examine the foundation of the house The engineerrsquos report on the foundation is likely to include information that would be relevant to the customerrsquos claim that the foundation was defectively constructed

The so-called ldquowork productrdquo rule allows a party to refuse to turn over ldquodocuments that are prepared in anticipation of litigation or for trialrdquo by that partyrsquos representative including

19

Federal Civil Procedure Analysis

a consultant Thus if the customer had hired the structural engineer to prepare a report ldquoin anticipation of litigationrdquo that report might not be discoverable See FED R CIV P 26(b)(3)

In this case however the customer hired the engineer to evaluate the foundation of the house as part of the customerrsquos negotiation with the builder concerning the housersquos flooding problem The builder told the customer that the housersquos landscaping was the reason for the flooding and the builder told the customer ldquoHave an engineer look at the foundation If therersquos a problem wersquoll fix itrdquo The customer appears to have acted in response to that statement There is no indication that the customer anticipated any kind of legal action at the time that the structural engineer was hired Accordingly the structural engineerrsquos report is discoverable and the court should order the customer to turn it over

[NOTE If an examinee concludes that the structural engineerrsquos report was prepared in anticipation of litigation then the examinee should also conclude that the report is not discoverable Documents prepared in anticipation of litigation do not need to be disclosed to an adverse party unless that party can demonstrate a ldquosubstantial needrdquo for the documents and an inability to obtain substantially equivalent information without ldquoundue hardshiprdquo FED R CIV P 26(b)(3)(A)(ii) Furthermore a report prepared by an expert who is not expected to testify is not discoverable in the absence of ldquoexceptional circumstancesrdquo making it ldquoimpracticablerdquo to obtain the information in another way FED R CIV P 26(b)(4)(D)(ii) The builder probably cannot make these showings here unless the engineerrsquos report deals with circumstances that have since changed There is no evidence that the structural engineer would have had access to any information or facts that the builder would not already know as a result of its construction and subsequent inspection of the house In addition if necessary the builder could ask the court for permission to arrange for a further inspection of the house by a structural engineer hired by the builder See FED R CIV P 34(a)(2) Accordingly if an examinee concludes that the report was prepared in anticipation of litigation the examinee should also conclude that the builder is not entitled to see the report]

Point Two(a) (30) Because the builder anticipated that it might be involved in litigation concerning its contract with the customer the builder acted wrongfully in destroying emails that were relevant to the housersquos construction even though the emails were destroyed pursuant to a routine document retention plan

As noted above a party to a lawsuit in federal court ldquomay obtain discovery regarding any nonprivileged matter that is relevant to any partyrsquos claim or defenserdquo FED R CIV P 26(b)(1) This includes emails and other electronically stored information FED R CIV P 34(a)(1)(A) Here the customer has requested all the builderrsquos emails pertaining to work done on the foundation of the house Ordinarily the builder would be obliged to turn over this information which is relevant to the customerrsquos defense that the housersquos foundation was poorly constructed

Unfortunately the emails in question no longer exist because the builder destroyed them on August 2

In general spoliation of evidence (destruction or alteration of evidence) is improper if the party who destroyed or altered the evidence ldquohas notice that the evidence is relevant to litigation or should have known that the evidence may be relevant to future litigationrdquo Fujitsu Ltd v Federal Express Corp 247 F3d 423 436 (2d Cir 2001) It is improper for a party to destroy electronic information relevant to pending litigation even if the destruction occurs before there is any request or order seeking the information See eg Leon v IDX Sys Corp 464 F3d 951 (9th Cir 2006) (plaintiffrsquos intentional destruction of computer files warranted dismissal even

20

In this case the builderrsquos destruction of the emails was pursuant to a routine document retention plan The Federal Rules provide expressly that in the absence of ldquoexceptional circumstancesrdquo parties should not be sanctioned for the loss of electronically stored information when the loss occurs pursuant to ldquoroutine good-faith operation of an electronic information systemrdquo FED R CIV P 37(e) However when a party anticipates litigation ldquoit must suspend its routine document retentiondestruction policy and put in place a lsquolitigation holdrsquo to ensure the preservation of relevant documentsrdquo Zubulake v UBS Warburg LLC 220 FRD 212 218 (SDNY 2003)

Federal Civil Procedure Analysis

though spoliation occurred before order compelling discovery) Similarly the duty to preserve evidence applies to a party who anticipates litigation even if litigation has not yet been commenced See THE SEDONA PRINCIPLES BEST PRACTICES RECOMMENDATIONS amp PRINCIPLES FOR ADDRESSING ELECTRONIC DOCUMENT PRODUCTION 70 cmt 14a (2d ed 2007)

The builder destroyed the emails on August 2 At that time the builder knew that litigation was a possibility because the builder had already directed its attorney to prepare a draft complaint for possible filing Knowing that litigation was a possibility the builder had a duty to take steps to preserve evidence including the emails in question See generally Fujitsu Ltd

Thus the builderrsquos destruction of potentially relevant emails at a time when it knew that litigation was a possibility was improper It had a duty to preserve evidence and it breached that duty

[NOTE Because courts have used different words to describe the test for when evidence must be preserved an examineersquos precise formulation of the test is not critical]

Point Two(b) (30) In determining appropriate sanctions for spoliation courts consider both the level of culpability of the spoliating party and the degree of prejudice the loss of evidence has caused the other party Here the builderrsquos destruction of evidence does not appear to have been willful nor is it likely to pose a significant obstacle to the customerrsquos defense Any sanctions imposed by the court should be modest

Federal courts have inherent power to control the litigation process and can sanction misbehavior including spoliation even when there has been no specific violation of the Federal Rules of Civil Procedure See generally Chambers v NASCO Inc 501 US 32 (1991) (discussing courtrsquos inherent power to control the litigation process) The range of available sanctions is broad It can include such sanctions as the payment of expenses incurred by the other party as a result of the destruction of the evidence an instruction to the jury authorizing it to draw an adverse inference from the destruction of the evidence a shifting of the burden of proof on the relevant issue or even judgment against the responsible party See eg Residential Funding Corp v DeGeorge Financial Corp 306 F3d 99 108 (2d Cir 2002) (adverse inference) Silvestri v General Motors Corp 271 F3d 583 593 (4th Cir 2001) (possibility of dismissal) Cf FED R CIV P 37(b)(2)(A) (listing remedies for failure to comply with discovery obligations)

In determining appropriate sanctions for spoliation courts consider both the level of culpability of the spoliating party and the degree of prejudice the loss of evidence has caused the other party Many courts impose severe sanctions (such as an adverse-inference instruction or the entry of judgment against the spoliating party) only when there is evidence of bad faith in the form of an intentional effort to hide information Eg Greyhound Lines Inc v Wade 485 F3d 1032 1035 (8th Cir 2007) (spoliation sanction requires intentional destruction out of desire ldquoto suppress the truthrdquo) However other courts have said that negligence in preserving evidence can

21

Federal Civil Procedure Analysis

support an adverse-inference instruction See Residential Funding 306 F3d at 108 (negligence enough under some circumstances)

Although a court might well order an evidentiary hearing on the issue of sanctions the facts presented do not seem appropriate for severe sanctions First the evidence was destroyed pursuant to the builderrsquos standard document retention plan and there is no evidence that the builder deliberately failed to suspend its usual procedures with the purpose of allowing the destruction of evidence Second the loss of this evidence will not severely hinder the customerrsquos presentation of his case The central issue is whether the foundation of the house was properly constructed If the construction job was poorly done the customer can present evidence derived from inspection of the premises to prove that point The customer can also depose witnesses about any issues that arose during construction

Under the circumstances a court is not likely to impose particularly severe sanctions although it might shift the burden to the builder to show that the foundation was properly constructed or it might require the builder to reimburse any expenses the customer incurs to discover and prove the facts about issues or disputes that arose during construction of the foundation

[NOTE The result reached by the examinee is less important than the examineersquos recognition that (a) a range of sanctions is available to the court and (b) the appropriate sanction depends both on the culpability of the builder and the prejudice suffered by the customer]

22

CRIMINAL LAW AND PROCEDURE ANALYSIS (Criminal Law and Procedure IIA amp D VE amp F)

ANALYSIS

Legal Problems

(1) Did charging the defendant with both theft and burglary constitute double jeopardy

(2) Did the jury instruction violate the due process clause either by relieving the prosecution of the burden of proving the element of intent or by shifting the burden to the defendant to disprove that element

(3) Did the sentence imposed in this case for the theft conviction unconstitutionally deprive the defendant of his right to a jury trial on the issue of the value of the stolen item

DISCUSSION

Summary

The trial court properly denied the defendantrsquos pretrial motion to dismiss the charges on double jeopardy grounds The defendant may be charged with and convicted of both theft and burglary Each of the charges has an element that the other does not Neither charge is a lesser-included offense nor are they multiplicitous Thus charging both theft and burglary does not violate double jeopardy

The jury instruction on the burglary charge was constitutionally flawed It could have been reasonably understood by the jury as either (1) an irrebuttable conclusive presumption (which relieved the prosecution of proving the element of intent and removed the issue from the jury) or (2) a rebuttable mandatory presumption (which unconstitutionally shifted the burden of proof on an element of a charged offense from the prosecution to the defendant)

Because the four-year sentence imposed by the judge was based on the judgersquos finding by a preponderance of the evidence that the value of the stolen ring exceeded $5000 the sentence violates the defendantrsquos right to a jury determination beyond a reasonable doubt of the value of the ring

Point One (30) Charging the defendant with theft and burglary did not constitute double jeopardy

The Double Jeopardy Clause of the Fifth Amendment provides that a person shall not be twice put in jeopardy for the ldquosame offenserdquo Thus the question is whether the elements of the theft charge are wholly contained in the burglary charge or vice versa If the elements of the lesser charge (theft) are not wholly contained in the greater charge (burglary)mdashie if each charge requires proof of a fact that the other does notmdashthen convicting the defendant of both crimes would not violate double jeopardy even when the two offenses occurred at the same time and are thus arguably part of the ldquosame transactionrdquo Blockburger v United States 284 US 299 304 (1932) See also Albernaz v United States 450 US 333 344 n3 (1981) United States v Dixon 509 US 688 704 (1993)

23

Criminal Law and Procedure Analysis

Here theft and burglary each require proof of an element not required for the other crime Burglary may be defined differently in different jurisdictions However it almost invariably requires entry into a building or dwelling of another with the specific intent to commit a felony therein and the crime of burglary is complete upon the entry into the building or dwelling with such intent See eg Cannon v Oklahoma 827 P2d 1339 1342 (Okla Crim App 1992) In contrast theft which also may be defined differently in different states almost invariably requires the taking and carrying away of an item of personal property belonging to another with the intent to steal or permanently deprive the owner of possession

Here the ldquotakingrdquo or ldquostealingrdquo element is not contained in the definition of burglary and the ldquoentryrdquo element of burglary is not contained in the definition of theft Because theft is not a lesser-included offense of burglary and burglary is not a lesser-included offense of theft charging the defendant for both burglary and theft did not violate double jeopardy and the court properly denied the defense motion on those grounds Yparrea v Dorsey 64 F3d 577 579ndash80 (10th Cir 1995) citing Blockburger 284 US at 304

Finally the defendantrsquos motion to dismiss all the charges on double jeopardy grounds was improper because if both charges were for the same offense the motion should have requested dismissal of one charge not both

Point Two (35) The jury instruction on the burglary charge violated the Due Process Clause because it created either (1) an irrebuttable conclusive presumption (which relieved the prosecution of proving the element of intent and removed that issue from the jury) or (2) a rebuttable mandatory presumption (which unconstitutionally shifted the burden of proof on an element of a charged offense to the defendant)

The Supreme Court has interpreted the Due Process Clause of the US Constitution to require that the prosecution prove all elements of an offense beyond a reasonable doubt See In re Winship 397 US 358 364 (1970) The burden of proof cannot be shifted to the defendant by presuming an essential element upon proof of other elements of the offense because shifting the burden of persuasion with respect to any element of a criminal offense is contrary to the Due Process Clause See Mullaney v Wilbur 421 US 684 (1975)

The crime of burglary includes entry into a building or dwelling with the specific intent to commit a felony therein The requirement that the prosecutor prove beyond a reasonable doubt that the defendant had this specific intent distinguishes burglary from general-intent crimes like trespass See Sandstrom v Montana 442 US 510 523 (1979)

Here the jury was instructed that if ldquoafter consideration of all the evidence presented by the prosecution and defense you find beyond a reasonable doubt that the defendant entered the dwelling without the ownersrsquo consent you may presume that the defendant entered with the intent to commit a felony thereinrdquo This instruction was unconstitutional because it created either an irrebuttable conclusive presumption or a rebuttable mandatory presumption

A conclusive presumption is ldquoan irrebuttable direction by the court to find intent once convinced of the facts triggering the presumptionrdquo Id at 517 Here the jurors were instructed that once the prosecutor established that the defendant entered the neighborsrsquo house without consent they ldquomay presumerdquo that he intended to commit a felony therein The jurors may have reasonably concluded from this instruction that if they found that the defendant intended to enter his neighborsrsquo home without permission they must further find that he entered with the specific intent to commit a felony therein Because this instruction could operate as a conclusive

24

Criminal Law and Procedure Analysis

irrebuttable presumption by eliminating intent ldquoas an ingredient of the offenserdquo it violated due process by relieving the prosecution of the burden of proof for this element Id at 522

In the alternative the jury instruction could have been reasonably understood to create a rebuttable mandatory presumption which ldquotells [the jury] they must find the elemental fact upon proof of the basic fact at least unless the defendant has come forward with some evidence to rebut the presumed connection between the two factsrdquo County Court of Ulster County New York v Allen 442 US 140 157 (1979) The due process problem created by rebuttable mandatory presumptions is that ldquo[t]o the extent that the trier of fact is forced to abide by the presumption and may not reject it based on an independent evaluation of the particular facts presented by the State the analysis of the presumptionrsquos constitutional validity is logically divorced from those facts and based on the presumptionrsquos accuracy in the run of casesrdquo Id at 159

Unlike irrebuttable conclusive presumptions rebuttable mandatory presumptions are not always per se violations of the Due Process Clause However the Supreme Court of the United States has held that jury instructions that could reasonably be understood as shifting the burden of proof to the defendant on an element of the offense are unconstitutional Francis v Franklin 471 US 307 (1985) Here the argument that the jury instruction operated as a rebuttable mandatory presumption is supported by the fact that the judge also instructed the jury to ldquoconsider[ ] all the evidence presented by the prosecution and defenserdquo However even if the instruction created a rebuttable mandatory presumption it would be unconstitutional because it shifted the burden to the defense on an element of the offense Sandstrom 442 US at 524 Mullaney 421 US at 686

[NOTE Whether an examinee identifies the jury instruction as containing a ldquoconclusiverdquo or ldquomandatoryrdquo presumption is less important than the examineersquos analysis of the constitutional infirmities]

Point Three (35) The trial court violated the defendantrsquos Sixth Amendment right to a jury trial on an essential element of the offense when it found by a preponderance of the evidence that the ring was worth over $5000 and increased the defendantrsquos sentence based on this finding

In the statutory scheme under which the defendant was tried and convicted a Class D felony theft is defined as theft of item(s) with a value between $2500 and $10000 The jury found that the value of the diamond ring was at least $2500 and convicted the defendant of felony theft However at sentencing the trial court made a separate finding by a preponderance of the evidence that the value of the ring was greater than $5000 Following the statutersquos two-tiered sentencing scheme the judge then imposed on the defendant a sentence that was one year longer than the maximum that would otherwise have been allowed

The judgersquos sentence was unconstitutional because it violated the defendantrsquos Sixth Amendment right to a jury trial on this question The Supreme Court held in Apprendi v New Jersey 530 US 466 (2000) that ldquo[o]ther than the fact of a prior conviction any fact that increases the penalty for a crime beyond the prescribed statutory maximum must be submitted to a jury and proved beyond a reasonable doubtrdquo because ldquo[i]t is unconstitutional for a legislature to remove from the jury the assessment of facts that increase the prescribed range of penalties to which a criminal defendant is exposed [because] such facts must be established by proof beyond a reasonable doubtrdquo Id The Court reaffirmed Apprendi in Blakely v Washington 542 US 296 (2004) holding that the ldquolsquostatutory maximumrsquo for Apprendi purposes is the maximum sentence a judge may impose solely on the basis of the facts reflected in the jury verdict or admitted by the defendantrdquo Id at 303 (emphasis in original) In United States v Booker 543 US 220 (2005)

25

Criminal Law and Procedure Analysis

the Court relied on Blakely and Apprendi to conclude that protecting a defendantrsquos Sixth Amendment right to a jury trial required that ldquo[a]ny fact which is necessary to support a sentence exceeding the maximum authorized by the facts established by a plea of guilty or a jury verdict must be admitted by the defendant or proved to a jury beyond a reasonable doubtrdquo Id at 244

Thus in order to constitutionally increase a sentence above the statutory maximum of three years the jury must have found beyond a reasonable doubt that the value of the ring exceeded $5000 Here the court made the finding based on an appraisal proffered by the prosecutor only at sentencing and the judgersquos finding was by a preponderance of the evidence rather than beyond a reasonable doubt

26

AGENCY AND PARTNERSHIP ANALYSIS __________ (Agency and Partnership VA amp C VI)

ANALYSIS

Legal Problems

(1) Is a partner in a general partnership personally liable on a claim arising from misrepresentations by another partner made in the course of the partnership business

(2) Does a newly admitted partner in a general partnership become personally liable on existing claims against the partnership

(3) After the filing by a general partnership of a statement of qualification as a limited liability partnership are the partners personally liable as partners on (a) an existing claim against the general partnership and (b) a claim against the partnership that arose after the filing

DISCUSSION

Summary

Adam and Ben formed a general partnership under which they were jointly and severally liable for obligations of the partnership Thus Adam was personally liable for misrepresentations by Ben made in the ordinary course of the partnership business

Upon joining the general partnership Diane became personally liable for the obligations of the partnership arising after her admission but not for obligations pre-existing her admission such as the collectorrsquos claim

By filing a statement of qualification the three partners properly elected limited liability partnership status As partners in an LLP none of the three partners is personally liable as a partner for partnership obligations arising after the election such as the claim by the driverrsquos estate The election however does not change their personal liability on pre-existing claims that arose before the election such as the collectorrsquos claim

Point One (30) As a general partner of Empire a general partnership Adam became personally liable on the collectorrsquos claim a valid claim against the partnership that arose because of Benrsquos wrongful act in the ordinary course of the partnership business

When the collectorrsquos claim arose Empire was a general partnership composed of Adam and Ben Under UPA (1997) sect 306(a) partners of a general partnership are liable jointly and severally for all obligations of the partnership Under UPA (1997) sect 305(a) the partnership could become obligated for the loss caused to the collector as a result of the misrepresentation by Ben provided he was acting in the ordinary course of the partnership business Because there was no statement that limited his partnership authority Ben as partner was ldquoan agent of the partnership for the purpose of its businessrdquo See UPA (1997) sect 301(1) Benrsquos misrepresentation to the collector even if intentional appears to be in the ordinary course of the partnershiprsquos business of dealing

27

Agency and Partnership Analysis

in antique cars Thus Benrsquos wrongful act created a partnership obligation for which Adam was jointly and severally liable

[NOTE Generally a partnership creditor must ldquoexhaust the partnershiprsquos assets before levying on a judgment debtor partnerrsquos individual property where the partner is personally liable for the partnership obligationrdquo as a result of his status as a partner UPA (1997) sect 307 cmt 4 As the UPA comments explain this places Adam more in the position of guarantor than principal debtor on the partnership obligation Id cmt 4 Although an examinee might discuss this point the call focuses on whether Adam is personally liable not how the liability might be enforced]

Point Two (30) Because the collectorrsquos claim arose before Diane joined Empire Diane did not become personally liable on the claim

Diane was admitted to Empire when it was a general partnership and after the collectorrsquos claim arose While the general rule under UPA (1997) sect 306(a) is that the partners of a general partnership are liable jointly and severally for all obligations of the partnership there is a special rule for partners who are admitted during the duration of the partnership Under UPA (1997) sect 306(b) a person admitted to an existing partnership is not personally liable for any partnership obligations incurred before the personrsquos admission Because Diane was admitted to Empire after the collectorrsquos claim arose Diane is not personally liable on the claim

Dianersquos knowledge of the pre-existing claim and her stated concern about becoming liable on the collectorrsquos claim do not change her personal nonliability to the collector Although partners who have a liability shield can assume liability to third parties through private contractual guarantees or modifications to the partnership agreement Dianersquos stated concern constituted neither a guaranty to the collector nor ldquoan intentional waiver of liability protectionsrdquo See UPA (1997) sect 306 cmt 3 (describing methods for waiver of liability protections under sect 306(c) applicable in limited liability partnerships)

At most Diane will lose her investment in the partnership as a result of the collectorrsquos claim Although Diane did not become personally liable on the collectorrsquos claim when she joined the partnership the $250000 she contributed to the partnership is ldquoat risk for the satisfaction of existing partnership debtsrdquo UPA (1997) sect 306 cmt 2

Point Three (40) Filing the statement of qualification was effective to elect limited liability partnership status Despite this new status Adam and Ben remain personally liable on the collectorrsquos claim which arose before the election But as partners in an LLP neither Adam Ben nor Diane is personally liable as a partner on the driverrsquos estatersquos claim which arose after the election

Under UPA (1997) sect 1001 a general partnership can make an election and become a limited liability partnershipmdashif the partners approve the conversion by a vote equivalent to that necessary to amend the partnership agreement and the partnership then files a statement of qualification that specifies the name of the partnership its principal office and its election to be an LLP Here the partners agreed unanimouslymdashsufficient to amend their agreement under UPA (1997) sect 401(j)mdashand the statement of qualification was filed In addition the name of Empire LLP properly included an appropriate ending ldquoLLPrdquo See UPA (1997) sect 1002

Although another way to effectuate a ldquoconversionrdquo (as suggested by Benrsquos lawyer) is to form a new LLP and transfer the assets of the old general partnership to the new LLP the

28

Agency and Partnership Analysis

method used here (approval by the partners and the filing of a statement of qualification) is also sufficient to create LLP status

Thus Empire became Empire LLP as of the date of filing of the statement of qualification See UPA (1997) sect 1001 What effect did this have on the collectorrsquos claim which predated the filing According to UPA (1997) sect 306(c) an obligation incurred while a partnership is an LLP is solely a partnership obligation As the collectorrsquos claim predated the LLP Adam and Ben remain personally liable on the collectorrsquos claim Diane on the other hand was not personally liable on the collectorrsquos claim either before or after the filing of the statement of qualification See Point Two above

The driverrsquos estatersquos claim arose after Empire became Empire LLP Under UPA (1997) sect 306(c) an obligation incurred while a partnership is an LLP is solely a partnership obligationThus Adam Ben and Diane as partners are all protected from personal liability on the driverrsquos estatersquos claim But there may be personal liability if any of them was negligent or otherwise acted wrongfully by not informing the buyer of the bad suspension that caused the accident

29

National Conference of Bar Examiners 302 South Bedford Street | Madison WI 53703-3622 Phone 608-280-8550 | Fax 608-280-8552 | TDD 608-661-1275

wwwncbexorg e-mail contactncbexorg

  • Preface
  • Description of the MEE
  • Instructions
  • February 2014 Questions
    • Constitutinal Law Question
    • Trusts and Future Interests Question
    • Secured Transactions Question
    • Federal Civil Procedure Question
    • Criminal Law and Procedure Question
    • Agency and Partnership Question
      • February 2014 Analyses
        • Constitutional Law Analysis
        • Trust and Future Interests Analysis
        • Secured Transactions Analysis
        • Federal Civil Procedure Analysis
        • Criminal Law and Procedure Analysis
        • Agency and Partnership Analysis
            • ltlt13 ASCII85EncodePages false13 AllowTransparency false13 AutoPositionEPSFiles true13 AutoRotatePages None13 Binding Left13 CalGrayProfile (Dot Gain 20)13 CalRGBProfile (sRGB IEC61966-21)13 CalCMYKProfile (US Web Coated 050SWOP051 v2)13 sRGBProfile (sRGB IEC61966-21)13 CannotEmbedFontPolicy Error13 CompatibilityLevel 1413 CompressObjects Tags13 CompressPages true13 ConvertImagesToIndexed true13 PassThroughJPEGImages true13 CreateJobTicket false13 DefaultRenderingIntent Default13 DetectBlends true13 DetectCurves 0000013 ColorConversionStrategy CMYK13 DoThumbnails false13 EmbedAllFonts true13 EmbedOpenType false13 ParseICCProfilesInComments true13 EmbedJobOptions true13 DSCReportingLevel 013 EmitDSCWarnings false13 EndPage -113 ImageMemory 104857613 LockDistillerParams false13 MaxSubsetPct 10013 Optimize true13 OPM 113 ParseDSCComments true13 ParseDSCCommentsForDocInfo true13 PreserveCopyPage true13 PreserveDICMYKValues true13 PreserveEPSInfo true13 PreserveFlatness true13 PreserveHalftoneInfo false13 PreserveOPIComments true13 PreserveOverprintSettings true13 StartPage 113 SubsetFonts true13 TransferFunctionInfo Apply13 UCRandBGInfo Preserve13 UsePrologue false13 ColorSettingsFile ()13 AlwaysEmbed [ true13 ]13 NeverEmbed [ true13 ]13 AntiAliasColorImages false13 CropColorImages true13 ColorImageMinResolution 30013 ColorImageMinResolutionPolicy OK13 DownsampleColorImages true13 ColorImageDownsampleType Bicubic13 ColorImageResolution 30013 ColorImageDepth -113 ColorImageMinDownsampleDepth 113 ColorImageDownsampleThreshold 15000013 EncodeColorImages true13 ColorImageFilter DCTEncode13 AutoFilterColorImages true13 ColorImageAutoFilterStrategy JPEG13 ColorACSImageDict ltlt13 QFactor 01513 HSamples [1 1 1 1] VSamples [1 1 1 1]13 gtgt13 ColorImageDict ltlt13 QFactor 01513 HSamples [1 1 1 1] VSamples [1 1 1 1]13 gtgt13 JPEG2000ColorACSImageDict ltlt13 TileWidth 25613 TileHeight 25613 Quality 3013 gtgt13 JPEG2000ColorImageDict ltlt13 TileWidth 25613 TileHeight 25613 Quality 3013 gtgt13 AntiAliasGrayImages false13 CropGrayImages true13 GrayImageMinResolution 30013 GrayImageMinResolutionPolicy OK13 DownsampleGrayImages true13 GrayImageDownsampleType Bicubic13 GrayImageResolution 30013 GrayImageDepth -113 GrayImageMinDownsampleDepth 213 GrayImageDownsampleThreshold 15000013 EncodeGrayImages true13 GrayImageFilter DCTEncode13 AutoFilterGrayImages true13 GrayImageAutoFilterStrategy JPEG13 GrayACSImageDict ltlt13 QFactor 01513 HSamples [1 1 1 1] VSamples [1 1 1 1]13 gtgt13 GrayImageDict ltlt13 QFactor 01513 HSamples [1 1 1 1] VSamples [1 1 1 1]13 gtgt13 JPEG2000GrayACSImageDict ltlt13 TileWidth 25613 TileHeight 25613 Quality 3013 gtgt13 JPEG2000GrayImageDict ltlt13 TileWidth 25613 TileHeight 25613 Quality 3013 gtgt13 AntiAliasMonoImages false13 CropMonoImages true13 MonoImageMinResolution 120013 MonoImageMinResolutionPolicy OK13 DownsampleMonoImages true13 MonoImageDownsampleType Bicubic13 MonoImageResolution 120013 MonoImageDepth -113 MonoImageDownsampleThreshold 15000013 EncodeMonoImages true13 MonoImageFilter CCITTFaxEncode13 MonoImageDict ltlt13 K -113 gtgt13 AllowPSXObjects false13 CheckCompliance [13 None13 ]13 PDFX1aCheck false13 PDFX3Check false13 PDFXCompliantPDFOnly false13 PDFXNoTrimBoxError true13 PDFXTrimBoxToMediaBoxOffset [13 00000013 00000013 00000013 00000013 ]13 PDFXSetBleedBoxToMediaBox true13 PDFXBleedBoxToTrimBoxOffset [13 00000013 00000013 00000013 00000013 ]13 PDFXOutputIntentProfile ()13 PDFXOutputConditionIdentifier ()13 PDFXOutputCondition ()13 PDFXRegistryName ()13 PDFXTrapped False1313 CreateJDFFile false13 Description ltlt13 ARA 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 BGR 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 CHS ltFEFF4f7f75288fd94e9b8bbe5b9a521b5efa7684002000410064006f006200650020005000440046002065876863900275284e8e9ad88d2891cf76845370524d53705237300260a853ef4ee54f7f75280020004100630072006f0062006100740020548c002000410064006f00620065002000520065006100640065007200200035002e003000204ee553ca66f49ad87248672c676562535f00521b5efa768400200050004400460020658768633002gt13 CHT ltFEFF4f7f752890194e9b8a2d7f6e5efa7acb7684002000410064006f006200650020005000440046002065874ef69069752865bc9ad854c18cea76845370524d5370523786557406300260a853ef4ee54f7f75280020004100630072006f0062006100740020548c002000410064006f00620065002000520065006100640065007200200035002e003000204ee553ca66f49ad87248672c4f86958b555f5df25efa7acb76840020005000440046002065874ef63002gt13 CZE 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 DAN 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 DEU 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 ESP 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 ETI 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 FRA ltFEFF005500740069006c006900730065007a00200063006500730020006f007000740069006f006e00730020006100660069006e00200064006500200063007200e900650072002000640065007300200064006f00630075006d0065006e00740073002000410064006f00620065002000500044004600200070006f0075007200200075006e00650020007100750061006c0069007400e90020006400270069006d007000720065007300730069006f006e00200070007200e9007000720065007300730065002e0020004c0065007300200064006f00630075006d0065006e00740073002000500044004600200063007200e900e90073002000700065007500760065006e0074002000ea0074007200650020006f007500760065007200740073002000640061006e00730020004100630072006f006200610074002c002000610069006e00730069002000710075002700410064006f00620065002000520065006100640065007200200035002e0030002000650074002000760065007200730069006f006e007300200075006c007400e90072006900650075007200650073002egt13 GRE 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 HEB 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 HRV (Za stvaranje Adobe PDF dokumenata najpogodnijih za visokokvalitetni ispis prije tiskanja koristite ove postavke Stvoreni PDF dokumenti mogu se otvoriti Acrobat i Adobe Reader 50 i kasnijim verzijama)13 HUN 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 ITA 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 JPN ltFEFF9ad854c18cea306a30d730ea30d730ec30b951fa529b7528002000410064006f0062006500200050004400460020658766f8306e4f5c6210306b4f7f75283057307e305930023053306e8a2d5b9a30674f5c62103055308c305f0020005000440046002030d530a130a430eb306f3001004100630072006f0062006100740020304a30883073002000410064006f00620065002000520065006100640065007200200035002e003000204ee5964d3067958b304f30533068304c3067304d307e305930023053306e8a2d5b9a306b306f30d530a930f330c8306e57cb30818fbc307f304c5fc59808306730593002gt13 KOR ltFEFFc7740020c124c815c7440020c0acc6a9d558c5ec0020ace0d488c9c80020c2dcd5d80020c778c1c4c5d00020ac00c7a50020c801d569d55c002000410064006f0062006500200050004400460020bb38c11cb97c0020c791c131d569b2c8b2e4002e0020c774b807ac8c0020c791c131b41c00200050004400460020bb38c11cb2940020004100630072006f0062006100740020bc0f002000410064006f00620065002000520065006100640065007200200035002e00300020c774c0c1c5d0c11c0020c5f40020c2180020c788c2b5b2c8b2e4002egt13 LTH 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 LVI 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 NLD (Gebruik deze instellingen om Adobe PDF-documenten te maken die zijn geoptimaliseerd voor prepress-afdrukken van hoge kwaliteit De gemaakte PDF-documenten kunnen worden geopend met Acrobat en Adobe Reader 50 en hoger)13 NOR ltFEFF004200720075006b00200064006900730073006500200069006e006e007300740069006c006c0069006e00670065006e0065002000740069006c002000e50020006f0070007000720065007400740065002000410064006f006200650020005000440046002d0064006f006b0075006d0065006e00740065007200200073006f006d00200065007200200062006500730074002000650067006e0065007400200066006f00720020006600f80072007400720079006b006b0073007500740073006b00720069006600740020006100760020006800f800790020006b00760061006c0069007400650074002e0020005000440046002d0064006f006b0075006d0065006e00740065006e00650020006b0061006e002000e50070006e00650073002000690020004100630072006f00620061007400200065006c006c00650072002000410064006f00620065002000520065006100640065007200200035002e003000200065006c006c00650072002000730065006e006500720065002egt13 POL 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 PTB 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 RUM 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 RUS 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 SKY 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 SLV 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 SUO 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 SVE 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 TUR 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 UKR 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 ENU (Use these settings to create Adobe PDF documents best suited for high-quality prepress printing Created PDF documents can be opened with Acrobat and Adobe Reader 50 and later)13 gtgt13 Namespace [13 (Adobe)13 (Common)13 (10)13 ]13 OtherNamespaces [13 ltlt13 AsReaderSpreads false13 CropImagesToFrames true13 ErrorControl WarnAndContinue13 FlattenerIgnoreSpreadOverrides false13 IncludeGuidesGrids false13 IncludeNonPrinting false13 IncludeSlug false13 Namespace [13 (Adobe)13 (InDesign)13 (40)13 ]13 OmitPlacedBitmaps false13 OmitPlacedEPS false13 OmitPlacedPDF false13 SimulateOverprint Legacy13 gtgt13 ltlt13 AddBleedMarks false13 AddColorBars false13 AddCropMarks false13 AddPageInfo false13 AddRegMarks false13 ConvertColors ConvertToCMYK13 DestinationProfileName ()13 DestinationProfileSelector DocumentCMYK13 Downsample16BitImages true13 FlattenerPreset ltlt13 PresetSelector MediumResolution13 gtgt13 FormElements false13 GenerateStructure false13 IncludeBookmarks false13 IncludeHyperlinks false13 IncludeInteractive false13 IncludeLayers false13 IncludeProfiles false13 MultimediaHandling UseObjectSettings13 Namespace [13 (Adobe)13 (CreativeSuite)13 (20)13 ]13 PDFXOutputIntentProfileSelector DocumentCMYK13 PreserveEditing true13 UntaggedCMYKHandling LeaveUntagged13 UntaggedRGBHandling UseDocumentProfile13 UseDocumentBleed false13 gtgt13 ]13gtgt setdistillerparams13ltlt13 HWResolution [2400 2400]13 PageSize [612000 792000]13gtgt setpagedevice13

Page 18: February 2014 MEE Questions and AnalysesPreface The Multistate Essay Examination (MEE) is developed by the National Conference of Bar Examiners (NCBE). This publication includes the

Trusts and Future Interests Analysis Sales proceeds (UNIF PRIN amp INC ACT (2000) sect 404(2) UNIF PRIN amp INC ACT (1962)

sect 3(b)(1)) and dividends paid in the stock of the distributing corporation (UNIF PRIN amp INC ACT (2000) sect 401(c)(1) UNIF PRIN amp INC ACT (1962) sect 3(b)(4)) are allocable to principal and added to the principal of the trust

Here the cash dividends and office building rents should have been allocated to income and until the trustee received the sonrsquos letter should have been distributed to him as the sole income beneficiary of the trust The stock dividend and proceeds from the sale of the office building should have been allocated to principal and held by the trustee for future distribution to the ultimate remaindermen of the trust

[NOTE The 2000 Uniform Principal and Income Act has been adopted in Alabama Arkansas Colorado Connecticut the District of Columbia Hawaii Idaho Iowa Kentucky Missouri Montana Nebraska New Mexico North Dakota Oregon South Dakota Utah and West Virginia]

Point Two(a) (45) Because the son did not disclaim within nine months of the testatorrsquos death there is no valid disclaimer under state law Therefore the son is not deemed to have predeceased the testator Furthermore because of the express survivorship contingency in the will the remainder in the trust does not accelerate and become distributable until the son in fact dies When the son dies the trust principal will be distributable to the testatorrsquos then-living grandchildren or if none then to the testatorrsquos then-living heirs

When a trust remainder is given to a class the class closes (ie no new persons can join the class) when there is no outstanding income interest and at least one member of the class is then entitled to demand possession of his or her share of the remainder This principle is called the rule of convenience See generally HERBERT HOVENKAMP amp SHELDON F KURTZ PRINCIPLES OF PROPERTY LAW 199ndash200 (6th ed 2005) A class member may demand possession of his or her share of the remainder upon termination of the income interest only when the class memberrsquos interest is not otherwise subject to a condition precedent See id

When a beneficiary timely disclaims an interest in a trust that beneficiary is treated as if he had predeceased the testator Here had the son disclaimed within nine months of the testatorrsquos death as required by the state statute he would have been deemed to have predeceased the testator This would have closed the class of remaindermen and the testatorrsquos then-living grandchildren (ie the daughterrsquos child) would have been entitled to the trust principal However under the state statute the sonrsquos disclaimer was not timely because he did not disclaim within nine months of the testatorrsquos death Thus because the statute is inapplicable and the son is still alive the class of grandchildren entitled to share in trust principal did not close

Because here the statute is inapplicable due to the sonrsquos failure to comply with the statutory time requirements then presumably the common-law rule allowing disclaimers (aka renunciations) at any time should apply Under the common law if a life estate is renounced the remainder interest accelerates and becomes immediately distributable to the remaindermen of the trust if the remainder is vested but not if the remainder is contingent JESSE DUKEMINIER amp ROBERT H SITKOFF WILLS TRUSTS AND ESTATES 844ndash845 (9th ed 2013) Here because the remainder is contingent upon there being grandchildren who survive the son the remainder will not accelerate It will remain open until the son dies leaving open the possibility that additional grandchildren will be included in the class or the daughterrsquos child could fall out of the class because that child fails to survive the son

And if none of the testatorrsquos grandchildren survive the son the trust principal will be distributed to the testatorrsquos heirs living at the sonrsquos death

15

Trusts and Future Interests Analysis

Point Two(b) (10) Until the trust terminates the trustee must continue to hold the trust assets The distribution of income in the meantime is unclear There are at least three possibilities Income earned on the undistributed assets could be distributed to the son and daughter as the testatorrsquos heirs accumulated and added to principal for distribution to the ultimate remaindermen or distributed from time to time to those persons who are presumptively remaindermen

When trust principal is not immediately distributable the trustee must continue to hold trust assets until the ultimate remaindermen are ascertained During this period trust income will be distributed or retained according to any instructions contained in the trust instrument See WILLIAM M MCGOVERN JR SHELDON F KURTZ amp DAVID M ENGLISH WILLS TRUSTS amp ESTATES sect 102 (4th ed 2010)

Here the testator did not specify what the trustee should do with trust income in the event the sonrsquos disclaimer did not comply with the state statute There are at least three approaches One approach would have the trustee distribute the trust income to the testatorrsquos heirs on the theory that the income represents property that was not disposed of by the testatorrsquos will and which thus passes by partial intestacy to the testatorrsquos heirs A second approach would have the trustee accumulate trust income for distribution to the ultimate remaindermen Under this approach only those individuals ultimately entitled to the principal would share in the income A third approach would have the trustee distribute trust income to those individuals who would be the remaindermen if the trust were to terminate when the income is received by the trustee under this approach trust income would be distributed to the daughterrsquos minor child until another presumptive remainderman is born This approach could result in individuals not ultimately entitled to principal say because they do not survive the son receiving income It could also result in a disproportionate distribution of income among the individuals ultimately entitled to income

[NOTE Examinees should demonstrate a recognition and understanding of the income-allocation problem and the alternatives available to address that issue There is no widely accepted solution to the problem Examinees who cite any of these possible problem-solving approaches may receive credit]

16

SECURED TRANSACTIONS ANALYSIS (Secured Transactions IB IID E amp F IIIB IVA B amp F)

ANALYSIS

Legal Problems

(1)(a) What is the nature of the bankrsquos claim to the businessrsquos equipment

(1)(b) What is the nature of the finance companyrsquos claim to the businessrsquos equipment

(1)(c) As between the bank and the finance company whose claim to the businessrsquos equipment has priority

(2) Do the claims of the bank and the finance company continue in the item of equipment sold by the business to the competitor

DISCUSSION

Summary

The bank and the finance company both have perfected security interests in the businessrsquos equipment Even though the finance companyrsquos perfected security interest was created first the bankrsquos perfected security interest has priority because the bankrsquos financing statement was filed before the finance companyrsquos financing statement The security interests of the bank and the finance company continue in the item of equipment sold by the business to the competitor because their security interests were perfected and the competitor was not a buyer in ordinary course of business

Point One(a) (25) The bank has a perfected security interest in the businessrsquos equipment

The bank has met all criteria necessary for it to have an attached and enforceable security interest in the businessrsquos equipment First value must be given UCC sect 9-203(b)(1) This criterion is fulfilled by the loan made by the bank to the business Second the debtor must have rights in the collateral UCC sect 9-203(b)(2) Clearly the business has rights in its equipment Third either the secured party must take possession of the collateral or the debtor must authenticate a security agreement containing a description of the collateral UCC sect 9-203(b)(3) The agreement that the business owner signed is a ldquosecurity agreementrdquo because it is an agreement that creates or provides for a security interest UCC sect 9-102(a)(74) By signing the security agreement the business owner authenticated it UCC sect 9-102(a)(7) Therefore all three criteria are fulfilled and the bank has an enforceable and attached security interest

A security interest is perfected when it has attached and when any additional steps required for perfection have occurred UCC sect 9-308(a) Generally speaking the additional steps will either be possession of the collateral by the secured party or the filing of a financing statement with respect to the collateral See UCC sectsect 9-310 9-313 In this case the bank filed a financing statement naming the debtor and sufficiently indicating the collateral The collateral indication is sufficient because it identifies the collateral by type of property See UCC sectsect 9-504 9-108 The fact that the financing statement was filed before the security interest was created is

17

Secured Transactions Analysis

not a problem Even though the security agreement had not yet been signed the business had authorized the filing of the financing statement in an authenticated record UCC sect 9-509(a)(1) Moreover the financing statement may be filed before the security agreement is created UCC sect 9-502(d)

Point One(b) (10) The finance company also has a perfected security interest in the businessrsquos equipment

The finance companyrsquos security interest is enforceable and attached for the same reasons as the bankrsquos security interest The loan from the finance company to the business constitutes value the business has rights in the collateral and the business owner has authenticated a security agreement containing a description of the collateral The finance companyrsquos security interest is perfected because the finance company filed a financing statement with respect to it that provides that the business is the debtor and indicates that the collateral is equipment

Point One(c) (30) The bankrsquos security interest has priority over the finance companyrsquos security interest because the bankrsquos financing statement was filed first

As between two perfected security interests the general rule is that the security interest that was the earlier to be either perfected or the subject of a filed financing statement has priority UCC sect 9-322(a)(1) While the finance companyrsquos security interest was perfected before the bankrsquos (March 15 vs March 22) the bankrsquos financing statement was filed even earlier on March 2 Thus under the first-to-file-or-perfect rule of UCC sect 9-322(a)(1) the bankrsquos security interest has priority No exceptions to the general rule apply here

Point Two (35) A security interest in collateral continues notwithstanding its sale unless an exception applies Because the security interests of the bank and the finance company were perfected and the competitor was not a buyer in ordinary course of business no exception applies and the security interests of both creditors continue in the equipment sold to the competitor

As a general rule a security interest in collateral continues notwithstanding the fact that the debtor has sold the collateral to another person UCC sect 9-315(a)(1) Thus unless an exception applies the security interests of the bank and the finance company will continue in the item of equipment sold to the competitor

A buyer of goods will take free of an unperfected security interest in those goods See UCC sect 9-317(a)(2) However when the competitor bought the businessrsquos equipment both the bank and the finance company had perfected security interests in the equipment

A buyer can take free even of a perfected security interest in goods if the buyer is a ldquobuyer in ordinary course of businessrdquo See UCC sect 9-320(a) However the competitor was not a buyer in ordinary course of business To be a ldquobuyer in ordinary course of businessrdquo a buyer must buy goods from a seller that is in the business of selling goods of that kind See UCC sect 1-201(b)(9) The competitor bought this equipment from a seller that is not in the business of selling goods of this kind so the competitor was not a buyer in ordinary course of business with respect to these goods

Because no exception applies the security interests of the bank and the finance company continue even after the item of equipment was sold to the competitor

18

FEDERAL CIVIL PROCEDURE ANALYSIS (Federal Civil Procedure IVD)

ANALYSIS

Legal Problems

(1) Is a document prepared in the course of a contract dispute protected from discovery as ldquowork productrdquo when there is no evidence that the document was prepared in anticipation of litigation

(2)(a) Is a partyrsquos failure to provide relevant electronically stored information excused when the information was destroyed pursuant to a routine document retention scheme at a time when litigation was contemplated by the destroying party

(2)(b) What sanctions should be imposed on a party for allowing the destruction of evidence that is relevant to potential future litigation

DISCUSSION

Summary

The report prepared by the structural engineer is probably not work product and is thus discoverable The engineer examined the foundation of the house at the customerrsquos request and the engineerrsquos findings are potentially relevant to the customerrsquos claim that the foundation is defective The report was not prepared in anticipation of litigation The customer appears to have sought the engineerrsquos opinion in response to the builderrsquos offer to fix any problems with the foundation that an engineer might identify Because the report was not prepared in anticipation of litigation it is not protected by the work-product doctrine

The builder should have taken appropriate steps to preserve evidence including suspending its document retention program as soon as it began planning for litigationmdashie on July 10 Its destruction of potentially relevant material after that date was wrongful However a court is unlikely to impose severe sanctions on the builder because there are no facts indicating that the builder acted in bad faith and the customer can prove that the foundation is defective without the destroyed emails

Point One (40) The customer must turn over the engineerrsquos report because it was not prepared in anticipation of litigation

In general a party to a lawsuit in federal court ldquomay obtain discovery regarding any nonprivileged matter that is relevant to any partyrsquos claim or defenserdquo FED R CIV P 26(b)(1) (2009) This includes the right to inspect and copy documents in the other partyrsquos possession FED R CIV P 34(a)(1) Here the customer hired a structural engineer to examine the foundation of the house The engineerrsquos report on the foundation is likely to include information that would be relevant to the customerrsquos claim that the foundation was defectively constructed

The so-called ldquowork productrdquo rule allows a party to refuse to turn over ldquodocuments that are prepared in anticipation of litigation or for trialrdquo by that partyrsquos representative including

19

Federal Civil Procedure Analysis

a consultant Thus if the customer had hired the structural engineer to prepare a report ldquoin anticipation of litigationrdquo that report might not be discoverable See FED R CIV P 26(b)(3)

In this case however the customer hired the engineer to evaluate the foundation of the house as part of the customerrsquos negotiation with the builder concerning the housersquos flooding problem The builder told the customer that the housersquos landscaping was the reason for the flooding and the builder told the customer ldquoHave an engineer look at the foundation If therersquos a problem wersquoll fix itrdquo The customer appears to have acted in response to that statement There is no indication that the customer anticipated any kind of legal action at the time that the structural engineer was hired Accordingly the structural engineerrsquos report is discoverable and the court should order the customer to turn it over

[NOTE If an examinee concludes that the structural engineerrsquos report was prepared in anticipation of litigation then the examinee should also conclude that the report is not discoverable Documents prepared in anticipation of litigation do not need to be disclosed to an adverse party unless that party can demonstrate a ldquosubstantial needrdquo for the documents and an inability to obtain substantially equivalent information without ldquoundue hardshiprdquo FED R CIV P 26(b)(3)(A)(ii) Furthermore a report prepared by an expert who is not expected to testify is not discoverable in the absence of ldquoexceptional circumstancesrdquo making it ldquoimpracticablerdquo to obtain the information in another way FED R CIV P 26(b)(4)(D)(ii) The builder probably cannot make these showings here unless the engineerrsquos report deals with circumstances that have since changed There is no evidence that the structural engineer would have had access to any information or facts that the builder would not already know as a result of its construction and subsequent inspection of the house In addition if necessary the builder could ask the court for permission to arrange for a further inspection of the house by a structural engineer hired by the builder See FED R CIV P 34(a)(2) Accordingly if an examinee concludes that the report was prepared in anticipation of litigation the examinee should also conclude that the builder is not entitled to see the report]

Point Two(a) (30) Because the builder anticipated that it might be involved in litigation concerning its contract with the customer the builder acted wrongfully in destroying emails that were relevant to the housersquos construction even though the emails were destroyed pursuant to a routine document retention plan

As noted above a party to a lawsuit in federal court ldquomay obtain discovery regarding any nonprivileged matter that is relevant to any partyrsquos claim or defenserdquo FED R CIV P 26(b)(1) This includes emails and other electronically stored information FED R CIV P 34(a)(1)(A) Here the customer has requested all the builderrsquos emails pertaining to work done on the foundation of the house Ordinarily the builder would be obliged to turn over this information which is relevant to the customerrsquos defense that the housersquos foundation was poorly constructed

Unfortunately the emails in question no longer exist because the builder destroyed them on August 2

In general spoliation of evidence (destruction or alteration of evidence) is improper if the party who destroyed or altered the evidence ldquohas notice that the evidence is relevant to litigation or should have known that the evidence may be relevant to future litigationrdquo Fujitsu Ltd v Federal Express Corp 247 F3d 423 436 (2d Cir 2001) It is improper for a party to destroy electronic information relevant to pending litigation even if the destruction occurs before there is any request or order seeking the information See eg Leon v IDX Sys Corp 464 F3d 951 (9th Cir 2006) (plaintiffrsquos intentional destruction of computer files warranted dismissal even

20

In this case the builderrsquos destruction of the emails was pursuant to a routine document retention plan The Federal Rules provide expressly that in the absence of ldquoexceptional circumstancesrdquo parties should not be sanctioned for the loss of electronically stored information when the loss occurs pursuant to ldquoroutine good-faith operation of an electronic information systemrdquo FED R CIV P 37(e) However when a party anticipates litigation ldquoit must suspend its routine document retentiondestruction policy and put in place a lsquolitigation holdrsquo to ensure the preservation of relevant documentsrdquo Zubulake v UBS Warburg LLC 220 FRD 212 218 (SDNY 2003)

Federal Civil Procedure Analysis

though spoliation occurred before order compelling discovery) Similarly the duty to preserve evidence applies to a party who anticipates litigation even if litigation has not yet been commenced See THE SEDONA PRINCIPLES BEST PRACTICES RECOMMENDATIONS amp PRINCIPLES FOR ADDRESSING ELECTRONIC DOCUMENT PRODUCTION 70 cmt 14a (2d ed 2007)

The builder destroyed the emails on August 2 At that time the builder knew that litigation was a possibility because the builder had already directed its attorney to prepare a draft complaint for possible filing Knowing that litigation was a possibility the builder had a duty to take steps to preserve evidence including the emails in question See generally Fujitsu Ltd

Thus the builderrsquos destruction of potentially relevant emails at a time when it knew that litigation was a possibility was improper It had a duty to preserve evidence and it breached that duty

[NOTE Because courts have used different words to describe the test for when evidence must be preserved an examineersquos precise formulation of the test is not critical]

Point Two(b) (30) In determining appropriate sanctions for spoliation courts consider both the level of culpability of the spoliating party and the degree of prejudice the loss of evidence has caused the other party Here the builderrsquos destruction of evidence does not appear to have been willful nor is it likely to pose a significant obstacle to the customerrsquos defense Any sanctions imposed by the court should be modest

Federal courts have inherent power to control the litigation process and can sanction misbehavior including spoliation even when there has been no specific violation of the Federal Rules of Civil Procedure See generally Chambers v NASCO Inc 501 US 32 (1991) (discussing courtrsquos inherent power to control the litigation process) The range of available sanctions is broad It can include such sanctions as the payment of expenses incurred by the other party as a result of the destruction of the evidence an instruction to the jury authorizing it to draw an adverse inference from the destruction of the evidence a shifting of the burden of proof on the relevant issue or even judgment against the responsible party See eg Residential Funding Corp v DeGeorge Financial Corp 306 F3d 99 108 (2d Cir 2002) (adverse inference) Silvestri v General Motors Corp 271 F3d 583 593 (4th Cir 2001) (possibility of dismissal) Cf FED R CIV P 37(b)(2)(A) (listing remedies for failure to comply with discovery obligations)

In determining appropriate sanctions for spoliation courts consider both the level of culpability of the spoliating party and the degree of prejudice the loss of evidence has caused the other party Many courts impose severe sanctions (such as an adverse-inference instruction or the entry of judgment against the spoliating party) only when there is evidence of bad faith in the form of an intentional effort to hide information Eg Greyhound Lines Inc v Wade 485 F3d 1032 1035 (8th Cir 2007) (spoliation sanction requires intentional destruction out of desire ldquoto suppress the truthrdquo) However other courts have said that negligence in preserving evidence can

21

Federal Civil Procedure Analysis

support an adverse-inference instruction See Residential Funding 306 F3d at 108 (negligence enough under some circumstances)

Although a court might well order an evidentiary hearing on the issue of sanctions the facts presented do not seem appropriate for severe sanctions First the evidence was destroyed pursuant to the builderrsquos standard document retention plan and there is no evidence that the builder deliberately failed to suspend its usual procedures with the purpose of allowing the destruction of evidence Second the loss of this evidence will not severely hinder the customerrsquos presentation of his case The central issue is whether the foundation of the house was properly constructed If the construction job was poorly done the customer can present evidence derived from inspection of the premises to prove that point The customer can also depose witnesses about any issues that arose during construction

Under the circumstances a court is not likely to impose particularly severe sanctions although it might shift the burden to the builder to show that the foundation was properly constructed or it might require the builder to reimburse any expenses the customer incurs to discover and prove the facts about issues or disputes that arose during construction of the foundation

[NOTE The result reached by the examinee is less important than the examineersquos recognition that (a) a range of sanctions is available to the court and (b) the appropriate sanction depends both on the culpability of the builder and the prejudice suffered by the customer]

22

CRIMINAL LAW AND PROCEDURE ANALYSIS (Criminal Law and Procedure IIA amp D VE amp F)

ANALYSIS

Legal Problems

(1) Did charging the defendant with both theft and burglary constitute double jeopardy

(2) Did the jury instruction violate the due process clause either by relieving the prosecution of the burden of proving the element of intent or by shifting the burden to the defendant to disprove that element

(3) Did the sentence imposed in this case for the theft conviction unconstitutionally deprive the defendant of his right to a jury trial on the issue of the value of the stolen item

DISCUSSION

Summary

The trial court properly denied the defendantrsquos pretrial motion to dismiss the charges on double jeopardy grounds The defendant may be charged with and convicted of both theft and burglary Each of the charges has an element that the other does not Neither charge is a lesser-included offense nor are they multiplicitous Thus charging both theft and burglary does not violate double jeopardy

The jury instruction on the burglary charge was constitutionally flawed It could have been reasonably understood by the jury as either (1) an irrebuttable conclusive presumption (which relieved the prosecution of proving the element of intent and removed the issue from the jury) or (2) a rebuttable mandatory presumption (which unconstitutionally shifted the burden of proof on an element of a charged offense from the prosecution to the defendant)

Because the four-year sentence imposed by the judge was based on the judgersquos finding by a preponderance of the evidence that the value of the stolen ring exceeded $5000 the sentence violates the defendantrsquos right to a jury determination beyond a reasonable doubt of the value of the ring

Point One (30) Charging the defendant with theft and burglary did not constitute double jeopardy

The Double Jeopardy Clause of the Fifth Amendment provides that a person shall not be twice put in jeopardy for the ldquosame offenserdquo Thus the question is whether the elements of the theft charge are wholly contained in the burglary charge or vice versa If the elements of the lesser charge (theft) are not wholly contained in the greater charge (burglary)mdashie if each charge requires proof of a fact that the other does notmdashthen convicting the defendant of both crimes would not violate double jeopardy even when the two offenses occurred at the same time and are thus arguably part of the ldquosame transactionrdquo Blockburger v United States 284 US 299 304 (1932) See also Albernaz v United States 450 US 333 344 n3 (1981) United States v Dixon 509 US 688 704 (1993)

23

Criminal Law and Procedure Analysis

Here theft and burglary each require proof of an element not required for the other crime Burglary may be defined differently in different jurisdictions However it almost invariably requires entry into a building or dwelling of another with the specific intent to commit a felony therein and the crime of burglary is complete upon the entry into the building or dwelling with such intent See eg Cannon v Oklahoma 827 P2d 1339 1342 (Okla Crim App 1992) In contrast theft which also may be defined differently in different states almost invariably requires the taking and carrying away of an item of personal property belonging to another with the intent to steal or permanently deprive the owner of possession

Here the ldquotakingrdquo or ldquostealingrdquo element is not contained in the definition of burglary and the ldquoentryrdquo element of burglary is not contained in the definition of theft Because theft is not a lesser-included offense of burglary and burglary is not a lesser-included offense of theft charging the defendant for both burglary and theft did not violate double jeopardy and the court properly denied the defense motion on those grounds Yparrea v Dorsey 64 F3d 577 579ndash80 (10th Cir 1995) citing Blockburger 284 US at 304

Finally the defendantrsquos motion to dismiss all the charges on double jeopardy grounds was improper because if both charges were for the same offense the motion should have requested dismissal of one charge not both

Point Two (35) The jury instruction on the burglary charge violated the Due Process Clause because it created either (1) an irrebuttable conclusive presumption (which relieved the prosecution of proving the element of intent and removed that issue from the jury) or (2) a rebuttable mandatory presumption (which unconstitutionally shifted the burden of proof on an element of a charged offense to the defendant)

The Supreme Court has interpreted the Due Process Clause of the US Constitution to require that the prosecution prove all elements of an offense beyond a reasonable doubt See In re Winship 397 US 358 364 (1970) The burden of proof cannot be shifted to the defendant by presuming an essential element upon proof of other elements of the offense because shifting the burden of persuasion with respect to any element of a criminal offense is contrary to the Due Process Clause See Mullaney v Wilbur 421 US 684 (1975)

The crime of burglary includes entry into a building or dwelling with the specific intent to commit a felony therein The requirement that the prosecutor prove beyond a reasonable doubt that the defendant had this specific intent distinguishes burglary from general-intent crimes like trespass See Sandstrom v Montana 442 US 510 523 (1979)

Here the jury was instructed that if ldquoafter consideration of all the evidence presented by the prosecution and defense you find beyond a reasonable doubt that the defendant entered the dwelling without the ownersrsquo consent you may presume that the defendant entered with the intent to commit a felony thereinrdquo This instruction was unconstitutional because it created either an irrebuttable conclusive presumption or a rebuttable mandatory presumption

A conclusive presumption is ldquoan irrebuttable direction by the court to find intent once convinced of the facts triggering the presumptionrdquo Id at 517 Here the jurors were instructed that once the prosecutor established that the defendant entered the neighborsrsquo house without consent they ldquomay presumerdquo that he intended to commit a felony therein The jurors may have reasonably concluded from this instruction that if they found that the defendant intended to enter his neighborsrsquo home without permission they must further find that he entered with the specific intent to commit a felony therein Because this instruction could operate as a conclusive

24

Criminal Law and Procedure Analysis

irrebuttable presumption by eliminating intent ldquoas an ingredient of the offenserdquo it violated due process by relieving the prosecution of the burden of proof for this element Id at 522

In the alternative the jury instruction could have been reasonably understood to create a rebuttable mandatory presumption which ldquotells [the jury] they must find the elemental fact upon proof of the basic fact at least unless the defendant has come forward with some evidence to rebut the presumed connection between the two factsrdquo County Court of Ulster County New York v Allen 442 US 140 157 (1979) The due process problem created by rebuttable mandatory presumptions is that ldquo[t]o the extent that the trier of fact is forced to abide by the presumption and may not reject it based on an independent evaluation of the particular facts presented by the State the analysis of the presumptionrsquos constitutional validity is logically divorced from those facts and based on the presumptionrsquos accuracy in the run of casesrdquo Id at 159

Unlike irrebuttable conclusive presumptions rebuttable mandatory presumptions are not always per se violations of the Due Process Clause However the Supreme Court of the United States has held that jury instructions that could reasonably be understood as shifting the burden of proof to the defendant on an element of the offense are unconstitutional Francis v Franklin 471 US 307 (1985) Here the argument that the jury instruction operated as a rebuttable mandatory presumption is supported by the fact that the judge also instructed the jury to ldquoconsider[ ] all the evidence presented by the prosecution and defenserdquo However even if the instruction created a rebuttable mandatory presumption it would be unconstitutional because it shifted the burden to the defense on an element of the offense Sandstrom 442 US at 524 Mullaney 421 US at 686

[NOTE Whether an examinee identifies the jury instruction as containing a ldquoconclusiverdquo or ldquomandatoryrdquo presumption is less important than the examineersquos analysis of the constitutional infirmities]

Point Three (35) The trial court violated the defendantrsquos Sixth Amendment right to a jury trial on an essential element of the offense when it found by a preponderance of the evidence that the ring was worth over $5000 and increased the defendantrsquos sentence based on this finding

In the statutory scheme under which the defendant was tried and convicted a Class D felony theft is defined as theft of item(s) with a value between $2500 and $10000 The jury found that the value of the diamond ring was at least $2500 and convicted the defendant of felony theft However at sentencing the trial court made a separate finding by a preponderance of the evidence that the value of the ring was greater than $5000 Following the statutersquos two-tiered sentencing scheme the judge then imposed on the defendant a sentence that was one year longer than the maximum that would otherwise have been allowed

The judgersquos sentence was unconstitutional because it violated the defendantrsquos Sixth Amendment right to a jury trial on this question The Supreme Court held in Apprendi v New Jersey 530 US 466 (2000) that ldquo[o]ther than the fact of a prior conviction any fact that increases the penalty for a crime beyond the prescribed statutory maximum must be submitted to a jury and proved beyond a reasonable doubtrdquo because ldquo[i]t is unconstitutional for a legislature to remove from the jury the assessment of facts that increase the prescribed range of penalties to which a criminal defendant is exposed [because] such facts must be established by proof beyond a reasonable doubtrdquo Id The Court reaffirmed Apprendi in Blakely v Washington 542 US 296 (2004) holding that the ldquolsquostatutory maximumrsquo for Apprendi purposes is the maximum sentence a judge may impose solely on the basis of the facts reflected in the jury verdict or admitted by the defendantrdquo Id at 303 (emphasis in original) In United States v Booker 543 US 220 (2005)

25

Criminal Law and Procedure Analysis

the Court relied on Blakely and Apprendi to conclude that protecting a defendantrsquos Sixth Amendment right to a jury trial required that ldquo[a]ny fact which is necessary to support a sentence exceeding the maximum authorized by the facts established by a plea of guilty or a jury verdict must be admitted by the defendant or proved to a jury beyond a reasonable doubtrdquo Id at 244

Thus in order to constitutionally increase a sentence above the statutory maximum of three years the jury must have found beyond a reasonable doubt that the value of the ring exceeded $5000 Here the court made the finding based on an appraisal proffered by the prosecutor only at sentencing and the judgersquos finding was by a preponderance of the evidence rather than beyond a reasonable doubt

26

AGENCY AND PARTNERSHIP ANALYSIS __________ (Agency and Partnership VA amp C VI)

ANALYSIS

Legal Problems

(1) Is a partner in a general partnership personally liable on a claim arising from misrepresentations by another partner made in the course of the partnership business

(2) Does a newly admitted partner in a general partnership become personally liable on existing claims against the partnership

(3) After the filing by a general partnership of a statement of qualification as a limited liability partnership are the partners personally liable as partners on (a) an existing claim against the general partnership and (b) a claim against the partnership that arose after the filing

DISCUSSION

Summary

Adam and Ben formed a general partnership under which they were jointly and severally liable for obligations of the partnership Thus Adam was personally liable for misrepresentations by Ben made in the ordinary course of the partnership business

Upon joining the general partnership Diane became personally liable for the obligations of the partnership arising after her admission but not for obligations pre-existing her admission such as the collectorrsquos claim

By filing a statement of qualification the three partners properly elected limited liability partnership status As partners in an LLP none of the three partners is personally liable as a partner for partnership obligations arising after the election such as the claim by the driverrsquos estate The election however does not change their personal liability on pre-existing claims that arose before the election such as the collectorrsquos claim

Point One (30) As a general partner of Empire a general partnership Adam became personally liable on the collectorrsquos claim a valid claim against the partnership that arose because of Benrsquos wrongful act in the ordinary course of the partnership business

When the collectorrsquos claim arose Empire was a general partnership composed of Adam and Ben Under UPA (1997) sect 306(a) partners of a general partnership are liable jointly and severally for all obligations of the partnership Under UPA (1997) sect 305(a) the partnership could become obligated for the loss caused to the collector as a result of the misrepresentation by Ben provided he was acting in the ordinary course of the partnership business Because there was no statement that limited his partnership authority Ben as partner was ldquoan agent of the partnership for the purpose of its businessrdquo See UPA (1997) sect 301(1) Benrsquos misrepresentation to the collector even if intentional appears to be in the ordinary course of the partnershiprsquos business of dealing

27

Agency and Partnership Analysis

in antique cars Thus Benrsquos wrongful act created a partnership obligation for which Adam was jointly and severally liable

[NOTE Generally a partnership creditor must ldquoexhaust the partnershiprsquos assets before levying on a judgment debtor partnerrsquos individual property where the partner is personally liable for the partnership obligationrdquo as a result of his status as a partner UPA (1997) sect 307 cmt 4 As the UPA comments explain this places Adam more in the position of guarantor than principal debtor on the partnership obligation Id cmt 4 Although an examinee might discuss this point the call focuses on whether Adam is personally liable not how the liability might be enforced]

Point Two (30) Because the collectorrsquos claim arose before Diane joined Empire Diane did not become personally liable on the claim

Diane was admitted to Empire when it was a general partnership and after the collectorrsquos claim arose While the general rule under UPA (1997) sect 306(a) is that the partners of a general partnership are liable jointly and severally for all obligations of the partnership there is a special rule for partners who are admitted during the duration of the partnership Under UPA (1997) sect 306(b) a person admitted to an existing partnership is not personally liable for any partnership obligations incurred before the personrsquos admission Because Diane was admitted to Empire after the collectorrsquos claim arose Diane is not personally liable on the claim

Dianersquos knowledge of the pre-existing claim and her stated concern about becoming liable on the collectorrsquos claim do not change her personal nonliability to the collector Although partners who have a liability shield can assume liability to third parties through private contractual guarantees or modifications to the partnership agreement Dianersquos stated concern constituted neither a guaranty to the collector nor ldquoan intentional waiver of liability protectionsrdquo See UPA (1997) sect 306 cmt 3 (describing methods for waiver of liability protections under sect 306(c) applicable in limited liability partnerships)

At most Diane will lose her investment in the partnership as a result of the collectorrsquos claim Although Diane did not become personally liable on the collectorrsquos claim when she joined the partnership the $250000 she contributed to the partnership is ldquoat risk for the satisfaction of existing partnership debtsrdquo UPA (1997) sect 306 cmt 2

Point Three (40) Filing the statement of qualification was effective to elect limited liability partnership status Despite this new status Adam and Ben remain personally liable on the collectorrsquos claim which arose before the election But as partners in an LLP neither Adam Ben nor Diane is personally liable as a partner on the driverrsquos estatersquos claim which arose after the election

Under UPA (1997) sect 1001 a general partnership can make an election and become a limited liability partnershipmdashif the partners approve the conversion by a vote equivalent to that necessary to amend the partnership agreement and the partnership then files a statement of qualification that specifies the name of the partnership its principal office and its election to be an LLP Here the partners agreed unanimouslymdashsufficient to amend their agreement under UPA (1997) sect 401(j)mdashand the statement of qualification was filed In addition the name of Empire LLP properly included an appropriate ending ldquoLLPrdquo See UPA (1997) sect 1002

Although another way to effectuate a ldquoconversionrdquo (as suggested by Benrsquos lawyer) is to form a new LLP and transfer the assets of the old general partnership to the new LLP the

28

Agency and Partnership Analysis

method used here (approval by the partners and the filing of a statement of qualification) is also sufficient to create LLP status

Thus Empire became Empire LLP as of the date of filing of the statement of qualification See UPA (1997) sect 1001 What effect did this have on the collectorrsquos claim which predated the filing According to UPA (1997) sect 306(c) an obligation incurred while a partnership is an LLP is solely a partnership obligation As the collectorrsquos claim predated the LLP Adam and Ben remain personally liable on the collectorrsquos claim Diane on the other hand was not personally liable on the collectorrsquos claim either before or after the filing of the statement of qualification See Point Two above

The driverrsquos estatersquos claim arose after Empire became Empire LLP Under UPA (1997) sect 306(c) an obligation incurred while a partnership is an LLP is solely a partnership obligationThus Adam Ben and Diane as partners are all protected from personal liability on the driverrsquos estatersquos claim But there may be personal liability if any of them was negligent or otherwise acted wrongfully by not informing the buyer of the bad suspension that caused the accident

29

National Conference of Bar Examiners 302 South Bedford Street | Madison WI 53703-3622 Phone 608-280-8550 | Fax 608-280-8552 | TDD 608-661-1275

wwwncbexorg e-mail contactncbexorg

  • Preface
  • Description of the MEE
  • Instructions
  • February 2014 Questions
    • Constitutinal Law Question
    • Trusts and Future Interests Question
    • Secured Transactions Question
    • Federal Civil Procedure Question
    • Criminal Law and Procedure Question
    • Agency and Partnership Question
      • February 2014 Analyses
        • Constitutional Law Analysis
        • Trust and Future Interests Analysis
        • Secured Transactions Analysis
        • Federal Civil Procedure Analysis
        • Criminal Law and Procedure Analysis
        • Agency and Partnership Analysis
            • ltlt13 ASCII85EncodePages false13 AllowTransparency false13 AutoPositionEPSFiles true13 AutoRotatePages None13 Binding Left13 CalGrayProfile (Dot Gain 20)13 CalRGBProfile (sRGB IEC61966-21)13 CalCMYKProfile (US Web Coated 050SWOP051 v2)13 sRGBProfile (sRGB IEC61966-21)13 CannotEmbedFontPolicy Error13 CompatibilityLevel 1413 CompressObjects Tags13 CompressPages true13 ConvertImagesToIndexed true13 PassThroughJPEGImages true13 CreateJobTicket false13 DefaultRenderingIntent Default13 DetectBlends true13 DetectCurves 0000013 ColorConversionStrategy CMYK13 DoThumbnails false13 EmbedAllFonts true13 EmbedOpenType false13 ParseICCProfilesInComments true13 EmbedJobOptions true13 DSCReportingLevel 013 EmitDSCWarnings false13 EndPage -113 ImageMemory 104857613 LockDistillerParams false13 MaxSubsetPct 10013 Optimize true13 OPM 113 ParseDSCComments true13 ParseDSCCommentsForDocInfo true13 PreserveCopyPage true13 PreserveDICMYKValues true13 PreserveEPSInfo true13 PreserveFlatness true13 PreserveHalftoneInfo false13 PreserveOPIComments true13 PreserveOverprintSettings true13 StartPage 113 SubsetFonts true13 TransferFunctionInfo Apply13 UCRandBGInfo Preserve13 UsePrologue false13 ColorSettingsFile ()13 AlwaysEmbed [ true13 ]13 NeverEmbed [ true13 ]13 AntiAliasColorImages false13 CropColorImages true13 ColorImageMinResolution 30013 ColorImageMinResolutionPolicy OK13 DownsampleColorImages true13 ColorImageDownsampleType Bicubic13 ColorImageResolution 30013 ColorImageDepth -113 ColorImageMinDownsampleDepth 113 ColorImageDownsampleThreshold 15000013 EncodeColorImages true13 ColorImageFilter DCTEncode13 AutoFilterColorImages true13 ColorImageAutoFilterStrategy JPEG13 ColorACSImageDict ltlt13 QFactor 01513 HSamples [1 1 1 1] VSamples [1 1 1 1]13 gtgt13 ColorImageDict ltlt13 QFactor 01513 HSamples [1 1 1 1] VSamples [1 1 1 1]13 gtgt13 JPEG2000ColorACSImageDict ltlt13 TileWidth 25613 TileHeight 25613 Quality 3013 gtgt13 JPEG2000ColorImageDict ltlt13 TileWidth 25613 TileHeight 25613 Quality 3013 gtgt13 AntiAliasGrayImages false13 CropGrayImages true13 GrayImageMinResolution 30013 GrayImageMinResolutionPolicy OK13 DownsampleGrayImages true13 GrayImageDownsampleType Bicubic13 GrayImageResolution 30013 GrayImageDepth -113 GrayImageMinDownsampleDepth 213 GrayImageDownsampleThreshold 15000013 EncodeGrayImages true13 GrayImageFilter DCTEncode13 AutoFilterGrayImages true13 GrayImageAutoFilterStrategy JPEG13 GrayACSImageDict ltlt13 QFactor 01513 HSamples [1 1 1 1] VSamples [1 1 1 1]13 gtgt13 GrayImageDict ltlt13 QFactor 01513 HSamples [1 1 1 1] VSamples [1 1 1 1]13 gtgt13 JPEG2000GrayACSImageDict ltlt13 TileWidth 25613 TileHeight 25613 Quality 3013 gtgt13 JPEG2000GrayImageDict ltlt13 TileWidth 25613 TileHeight 25613 Quality 3013 gtgt13 AntiAliasMonoImages false13 CropMonoImages true13 MonoImageMinResolution 120013 MonoImageMinResolutionPolicy OK13 DownsampleMonoImages true13 MonoImageDownsampleType Bicubic13 MonoImageResolution 120013 MonoImageDepth -113 MonoImageDownsampleThreshold 15000013 EncodeMonoImages true13 MonoImageFilter CCITTFaxEncode13 MonoImageDict ltlt13 K -113 gtgt13 AllowPSXObjects false13 CheckCompliance [13 None13 ]13 PDFX1aCheck false13 PDFX3Check false13 PDFXCompliantPDFOnly false13 PDFXNoTrimBoxError true13 PDFXTrimBoxToMediaBoxOffset [13 00000013 00000013 00000013 00000013 ]13 PDFXSetBleedBoxToMediaBox true13 PDFXBleedBoxToTrimBoxOffset [13 00000013 00000013 00000013 00000013 ]13 PDFXOutputIntentProfile ()13 PDFXOutputConditionIdentifier ()13 PDFXOutputCondition ()13 PDFXRegistryName ()13 PDFXTrapped False1313 CreateJDFFile false13 Description ltlt13 ARA ltFEFF06270633062A062E062F0645002006470630064700200627064406250639062F0627062F0627062A002006440625064606340627062100200648062B062706260642002000410064006F00620065002000500044004600200645062A064806270641064206290020064406440637062806270639062900200641064A00200627064406450637062706280639002006300627062A0020062F0631062C0627062A002006270644062C0648062F0629002006270644063906270644064A0629061B0020064A06450643064600200641062A062D00200648062B0627062606420020005000440046002006270644064506460634062306290020062806270633062A062E062F062706450020004100630072006F0062006100740020064800410064006F006200650020005200650061006400650072002006250635062F0627063100200035002E0030002006480627064406250635062F062706310627062A0020062706440623062D062F062B002E0635062F0627063100200035002E0030002006480627064406250635062F062706310627062A0020062706440623062D062F062B002Egt13 BGR 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 CHS ltFEFF4f7f75288fd94e9b8bbe5b9a521b5efa7684002000410064006f006200650020005000440046002065876863900275284e8e9ad88d2891cf76845370524d53705237300260a853ef4ee54f7f75280020004100630072006f0062006100740020548c002000410064006f00620065002000520065006100640065007200200035002e003000204ee553ca66f49ad87248672c676562535f00521b5efa768400200050004400460020658768633002gt13 CHT ltFEFF4f7f752890194e9b8a2d7f6e5efa7acb7684002000410064006f006200650020005000440046002065874ef69069752865bc9ad854c18cea76845370524d5370523786557406300260a853ef4ee54f7f75280020004100630072006f0062006100740020548c002000410064006f00620065002000520065006100640065007200200035002e003000204ee553ca66f49ad87248672c4f86958b555f5df25efa7acb76840020005000440046002065874ef63002gt13 CZE ltFEFF005400610074006f0020006e006100730074006100760065006e00ed00200070006f0075017e0069006a007400650020006b0020007600790074007600e101590065006e00ed00200064006f006b0075006d0065006e0074016f002000410064006f006200650020005000440046002c0020006b00740065007200e90020007300650020006e0065006a006c00e90070006500200068006f006400ed002000700072006f0020006b00760061006c00690074006e00ed0020007400690073006b00200061002000700072006500700072006500730073002e002000200056007900740076006f01590065006e00e900200064006f006b0075006d0065006e007400790020005000440046002000620075006400650020006d006f017e006e00e90020006f007400650076015900ed007400200076002000700072006f006700720061006d0065006300680020004100630072006f00620061007400200061002000410064006f00620065002000520065006100640065007200200035002e0030002000610020006e006f0076011b006a016100ed00630068002egt13 DAN 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 DEU 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 ESP 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 ETI 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 FRA 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 GRE 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 HEB ltFEFF05D405E905EA05DE05E905D5002005D105D405D205D305E805D505EA002005D005DC05D4002005DB05D305D9002005DC05D905E605D505E8002005DE05E105DE05DB05D9002000410064006F006200650020005000440046002005D405DE05D505EA05D005DE05D905DD002005DC05D405D305E405E105EA002005E705D305DD002D05D305E405D505E1002005D005D905DB05D505EA05D905EA002E002005DE05E105DE05DB05D90020005000440046002005E905E005D505E605E805D5002005E005D905EA05E005D905DD002005DC05E405EA05D905D705D4002005D105D005DE05E605E205D505EA0020004100630072006F006200610074002005D5002D00410064006F00620065002000520065006100640065007200200035002E0030002005D505D205E805E105D005D505EA002005DE05EA05E705D305DE05D505EA002005D905D505EA05E8002E05D005DE05D905DD002005DC002D005000440046002F0058002D0033002C002005E205D905D905E005D5002005D105DE05D305E805D905DA002005DC05DE05E905EA05DE05E9002005E905DC0020004100630072006F006200610074002E002005DE05E105DE05DB05D90020005000440046002005E905E005D505E605E805D5002005E005D905EA05E005D905DD002005DC05E405EA05D905D705D4002005D105D005DE05E605E205D505EA0020004100630072006F006200610074002005D5002D00410064006F00620065002000520065006100640065007200200035002E0030002005D505D205E805E105D005D505EA002005DE05EA05E705D305DE05D505EA002005D905D505EA05E8002Egt13 HRV (Za stvaranje Adobe PDF dokumenata najpogodnijih za visokokvalitetni ispis prije tiskanja koristite ove postavke Stvoreni PDF dokumenti mogu se otvoriti Acrobat i Adobe Reader 50 i kasnijim verzijama)13 HUN 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 ITA 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 JPN ltFEFF9ad854c18cea306a30d730ea30d730ec30b951fa529b7528002000410064006f0062006500200050004400460020658766f8306e4f5c6210306b4f7f75283057307e305930023053306e8a2d5b9a30674f5c62103055308c305f0020005000440046002030d530a130a430eb306f3001004100630072006f0062006100740020304a30883073002000410064006f00620065002000520065006100640065007200200035002e003000204ee5964d3067958b304f30533068304c3067304d307e305930023053306e8a2d5b9a306b306f30d530a930f330c8306e57cb30818fbc307f304c5fc59808306730593002gt13 KOR ltFEFFc7740020c124c815c7440020c0acc6a9d558c5ec0020ace0d488c9c80020c2dcd5d80020c778c1c4c5d00020ac00c7a50020c801d569d55c002000410064006f0062006500200050004400460020bb38c11cb97c0020c791c131d569b2c8b2e4002e0020c774b807ac8c0020c791c131b41c00200050004400460020bb38c11cb2940020004100630072006f0062006100740020bc0f002000410064006f00620065002000520065006100640065007200200035002e00300020c774c0c1c5d0c11c0020c5f40020c2180020c788c2b5b2c8b2e4002egt13 LTH 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 LVI 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 NLD (Gebruik deze instellingen om Adobe PDF-documenten te maken die zijn geoptimaliseerd voor prepress-afdrukken van hoge kwaliteit De gemaakte PDF-documenten kunnen worden geopend met Acrobat en Adobe Reader 50 en hoger)13 NOR 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 POL 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 PTB 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 RUM ltFEFF005500740069006c0069007a00610163006900200061006300650073007400650020007300650074010300720069002000700065006e007400720075002000610020006300720065006100200064006f00630075006d0065006e00740065002000410064006f006200650020005000440046002000610064006500630076006100740065002000700065006e0074007200750020007400690070010300720069007200650061002000700072006500700072006500730073002000640065002000630061006c006900740061007400650020007300750070006500720069006f006100720103002e002000200044006f00630075006d0065006e00740065006c00650020005000440046002000630072006500610074006500200070006f00740020006600690020006400650073006300680069007300650020006300750020004100630072006f006200610074002c002000410064006f00620065002000520065006100640065007200200035002e00300020015f00690020007600650072007300690075006e0069006c006500200075006c0074006500720069006f006100720065002egt13 RUS 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 SKY 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 SLV 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 SUO 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 SVE ltFEFF0041006e007600e4006e00640020006400650020006800e4007200200069006e0073007400e4006c006c006e0069006e006700610072006e00610020006f006d002000640075002000760069006c006c00200073006b006100700061002000410064006f006200650020005000440046002d0064006f006b0075006d0065006e007400200073006f006d002000e400720020006c00e4006d0070006c0069006700610020006600f60072002000700072006500700072006500730073002d007500740073006b00720069006600740020006d006500640020006800f600670020006b00760061006c0069007400650074002e002000200053006b006100700061006400650020005000440046002d0064006f006b0075006d0065006e00740020006b0061006e002000f600700070006e00610073002000690020004100630072006f0062006100740020006f00630068002000410064006f00620065002000520065006100640065007200200035002e00300020006f00630068002000730065006e006100720065002egt13 TUR 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 UKR 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 ENU (Use these settings to create Adobe PDF documents best suited for high-quality prepress printing Created PDF documents can be opened with Acrobat and Adobe Reader 50 and later)13 gtgt13 Namespace [13 (Adobe)13 (Common)13 (10)13 ]13 OtherNamespaces [13 ltlt13 AsReaderSpreads false13 CropImagesToFrames true13 ErrorControl WarnAndContinue13 FlattenerIgnoreSpreadOverrides false13 IncludeGuidesGrids false13 IncludeNonPrinting false13 IncludeSlug false13 Namespace [13 (Adobe)13 (InDesign)13 (40)13 ]13 OmitPlacedBitmaps false13 OmitPlacedEPS false13 OmitPlacedPDF false13 SimulateOverprint Legacy13 gtgt13 ltlt13 AddBleedMarks false13 AddColorBars false13 AddCropMarks false13 AddPageInfo false13 AddRegMarks false13 ConvertColors ConvertToCMYK13 DestinationProfileName ()13 DestinationProfileSelector DocumentCMYK13 Downsample16BitImages true13 FlattenerPreset ltlt13 PresetSelector MediumResolution13 gtgt13 FormElements false13 GenerateStructure false13 IncludeBookmarks false13 IncludeHyperlinks false13 IncludeInteractive false13 IncludeLayers false13 IncludeProfiles false13 MultimediaHandling UseObjectSettings13 Namespace [13 (Adobe)13 (CreativeSuite)13 (20)13 ]13 PDFXOutputIntentProfileSelector DocumentCMYK13 PreserveEditing true13 UntaggedCMYKHandling LeaveUntagged13 UntaggedRGBHandling UseDocumentProfile13 UseDocumentBleed false13 gtgt13 ]13gtgt setdistillerparams13ltlt13 HWResolution [2400 2400]13 PageSize [612000 792000]13gtgt setpagedevice13

Page 19: February 2014 MEE Questions and AnalysesPreface The Multistate Essay Examination (MEE) is developed by the National Conference of Bar Examiners (NCBE). This publication includes the

Trusts and Future Interests Analysis

Point Two(b) (10) Until the trust terminates the trustee must continue to hold the trust assets The distribution of income in the meantime is unclear There are at least three possibilities Income earned on the undistributed assets could be distributed to the son and daughter as the testatorrsquos heirs accumulated and added to principal for distribution to the ultimate remaindermen or distributed from time to time to those persons who are presumptively remaindermen

When trust principal is not immediately distributable the trustee must continue to hold trust assets until the ultimate remaindermen are ascertained During this period trust income will be distributed or retained according to any instructions contained in the trust instrument See WILLIAM M MCGOVERN JR SHELDON F KURTZ amp DAVID M ENGLISH WILLS TRUSTS amp ESTATES sect 102 (4th ed 2010)

Here the testator did not specify what the trustee should do with trust income in the event the sonrsquos disclaimer did not comply with the state statute There are at least three approaches One approach would have the trustee distribute the trust income to the testatorrsquos heirs on the theory that the income represents property that was not disposed of by the testatorrsquos will and which thus passes by partial intestacy to the testatorrsquos heirs A second approach would have the trustee accumulate trust income for distribution to the ultimate remaindermen Under this approach only those individuals ultimately entitled to the principal would share in the income A third approach would have the trustee distribute trust income to those individuals who would be the remaindermen if the trust were to terminate when the income is received by the trustee under this approach trust income would be distributed to the daughterrsquos minor child until another presumptive remainderman is born This approach could result in individuals not ultimately entitled to principal say because they do not survive the son receiving income It could also result in a disproportionate distribution of income among the individuals ultimately entitled to income

[NOTE Examinees should demonstrate a recognition and understanding of the income-allocation problem and the alternatives available to address that issue There is no widely accepted solution to the problem Examinees who cite any of these possible problem-solving approaches may receive credit]

16

SECURED TRANSACTIONS ANALYSIS (Secured Transactions IB IID E amp F IIIB IVA B amp F)

ANALYSIS

Legal Problems

(1)(a) What is the nature of the bankrsquos claim to the businessrsquos equipment

(1)(b) What is the nature of the finance companyrsquos claim to the businessrsquos equipment

(1)(c) As between the bank and the finance company whose claim to the businessrsquos equipment has priority

(2) Do the claims of the bank and the finance company continue in the item of equipment sold by the business to the competitor

DISCUSSION

Summary

The bank and the finance company both have perfected security interests in the businessrsquos equipment Even though the finance companyrsquos perfected security interest was created first the bankrsquos perfected security interest has priority because the bankrsquos financing statement was filed before the finance companyrsquos financing statement The security interests of the bank and the finance company continue in the item of equipment sold by the business to the competitor because their security interests were perfected and the competitor was not a buyer in ordinary course of business

Point One(a) (25) The bank has a perfected security interest in the businessrsquos equipment

The bank has met all criteria necessary for it to have an attached and enforceable security interest in the businessrsquos equipment First value must be given UCC sect 9-203(b)(1) This criterion is fulfilled by the loan made by the bank to the business Second the debtor must have rights in the collateral UCC sect 9-203(b)(2) Clearly the business has rights in its equipment Third either the secured party must take possession of the collateral or the debtor must authenticate a security agreement containing a description of the collateral UCC sect 9-203(b)(3) The agreement that the business owner signed is a ldquosecurity agreementrdquo because it is an agreement that creates or provides for a security interest UCC sect 9-102(a)(74) By signing the security agreement the business owner authenticated it UCC sect 9-102(a)(7) Therefore all three criteria are fulfilled and the bank has an enforceable and attached security interest

A security interest is perfected when it has attached and when any additional steps required for perfection have occurred UCC sect 9-308(a) Generally speaking the additional steps will either be possession of the collateral by the secured party or the filing of a financing statement with respect to the collateral See UCC sectsect 9-310 9-313 In this case the bank filed a financing statement naming the debtor and sufficiently indicating the collateral The collateral indication is sufficient because it identifies the collateral by type of property See UCC sectsect 9-504 9-108 The fact that the financing statement was filed before the security interest was created is

17

Secured Transactions Analysis

not a problem Even though the security agreement had not yet been signed the business had authorized the filing of the financing statement in an authenticated record UCC sect 9-509(a)(1) Moreover the financing statement may be filed before the security agreement is created UCC sect 9-502(d)

Point One(b) (10) The finance company also has a perfected security interest in the businessrsquos equipment

The finance companyrsquos security interest is enforceable and attached for the same reasons as the bankrsquos security interest The loan from the finance company to the business constitutes value the business has rights in the collateral and the business owner has authenticated a security agreement containing a description of the collateral The finance companyrsquos security interest is perfected because the finance company filed a financing statement with respect to it that provides that the business is the debtor and indicates that the collateral is equipment

Point One(c) (30) The bankrsquos security interest has priority over the finance companyrsquos security interest because the bankrsquos financing statement was filed first

As between two perfected security interests the general rule is that the security interest that was the earlier to be either perfected or the subject of a filed financing statement has priority UCC sect 9-322(a)(1) While the finance companyrsquos security interest was perfected before the bankrsquos (March 15 vs March 22) the bankrsquos financing statement was filed even earlier on March 2 Thus under the first-to-file-or-perfect rule of UCC sect 9-322(a)(1) the bankrsquos security interest has priority No exceptions to the general rule apply here

Point Two (35) A security interest in collateral continues notwithstanding its sale unless an exception applies Because the security interests of the bank and the finance company were perfected and the competitor was not a buyer in ordinary course of business no exception applies and the security interests of both creditors continue in the equipment sold to the competitor

As a general rule a security interest in collateral continues notwithstanding the fact that the debtor has sold the collateral to another person UCC sect 9-315(a)(1) Thus unless an exception applies the security interests of the bank and the finance company will continue in the item of equipment sold to the competitor

A buyer of goods will take free of an unperfected security interest in those goods See UCC sect 9-317(a)(2) However when the competitor bought the businessrsquos equipment both the bank and the finance company had perfected security interests in the equipment

A buyer can take free even of a perfected security interest in goods if the buyer is a ldquobuyer in ordinary course of businessrdquo See UCC sect 9-320(a) However the competitor was not a buyer in ordinary course of business To be a ldquobuyer in ordinary course of businessrdquo a buyer must buy goods from a seller that is in the business of selling goods of that kind See UCC sect 1-201(b)(9) The competitor bought this equipment from a seller that is not in the business of selling goods of this kind so the competitor was not a buyer in ordinary course of business with respect to these goods

Because no exception applies the security interests of the bank and the finance company continue even after the item of equipment was sold to the competitor

18

FEDERAL CIVIL PROCEDURE ANALYSIS (Federal Civil Procedure IVD)

ANALYSIS

Legal Problems

(1) Is a document prepared in the course of a contract dispute protected from discovery as ldquowork productrdquo when there is no evidence that the document was prepared in anticipation of litigation

(2)(a) Is a partyrsquos failure to provide relevant electronically stored information excused when the information was destroyed pursuant to a routine document retention scheme at a time when litigation was contemplated by the destroying party

(2)(b) What sanctions should be imposed on a party for allowing the destruction of evidence that is relevant to potential future litigation

DISCUSSION

Summary

The report prepared by the structural engineer is probably not work product and is thus discoverable The engineer examined the foundation of the house at the customerrsquos request and the engineerrsquos findings are potentially relevant to the customerrsquos claim that the foundation is defective The report was not prepared in anticipation of litigation The customer appears to have sought the engineerrsquos opinion in response to the builderrsquos offer to fix any problems with the foundation that an engineer might identify Because the report was not prepared in anticipation of litigation it is not protected by the work-product doctrine

The builder should have taken appropriate steps to preserve evidence including suspending its document retention program as soon as it began planning for litigationmdashie on July 10 Its destruction of potentially relevant material after that date was wrongful However a court is unlikely to impose severe sanctions on the builder because there are no facts indicating that the builder acted in bad faith and the customer can prove that the foundation is defective without the destroyed emails

Point One (40) The customer must turn over the engineerrsquos report because it was not prepared in anticipation of litigation

In general a party to a lawsuit in federal court ldquomay obtain discovery regarding any nonprivileged matter that is relevant to any partyrsquos claim or defenserdquo FED R CIV P 26(b)(1) (2009) This includes the right to inspect and copy documents in the other partyrsquos possession FED R CIV P 34(a)(1) Here the customer hired a structural engineer to examine the foundation of the house The engineerrsquos report on the foundation is likely to include information that would be relevant to the customerrsquos claim that the foundation was defectively constructed

The so-called ldquowork productrdquo rule allows a party to refuse to turn over ldquodocuments that are prepared in anticipation of litigation or for trialrdquo by that partyrsquos representative including

19

Federal Civil Procedure Analysis

a consultant Thus if the customer had hired the structural engineer to prepare a report ldquoin anticipation of litigationrdquo that report might not be discoverable See FED R CIV P 26(b)(3)

In this case however the customer hired the engineer to evaluate the foundation of the house as part of the customerrsquos negotiation with the builder concerning the housersquos flooding problem The builder told the customer that the housersquos landscaping was the reason for the flooding and the builder told the customer ldquoHave an engineer look at the foundation If therersquos a problem wersquoll fix itrdquo The customer appears to have acted in response to that statement There is no indication that the customer anticipated any kind of legal action at the time that the structural engineer was hired Accordingly the structural engineerrsquos report is discoverable and the court should order the customer to turn it over

[NOTE If an examinee concludes that the structural engineerrsquos report was prepared in anticipation of litigation then the examinee should also conclude that the report is not discoverable Documents prepared in anticipation of litigation do not need to be disclosed to an adverse party unless that party can demonstrate a ldquosubstantial needrdquo for the documents and an inability to obtain substantially equivalent information without ldquoundue hardshiprdquo FED R CIV P 26(b)(3)(A)(ii) Furthermore a report prepared by an expert who is not expected to testify is not discoverable in the absence of ldquoexceptional circumstancesrdquo making it ldquoimpracticablerdquo to obtain the information in another way FED R CIV P 26(b)(4)(D)(ii) The builder probably cannot make these showings here unless the engineerrsquos report deals with circumstances that have since changed There is no evidence that the structural engineer would have had access to any information or facts that the builder would not already know as a result of its construction and subsequent inspection of the house In addition if necessary the builder could ask the court for permission to arrange for a further inspection of the house by a structural engineer hired by the builder See FED R CIV P 34(a)(2) Accordingly if an examinee concludes that the report was prepared in anticipation of litigation the examinee should also conclude that the builder is not entitled to see the report]

Point Two(a) (30) Because the builder anticipated that it might be involved in litigation concerning its contract with the customer the builder acted wrongfully in destroying emails that were relevant to the housersquos construction even though the emails were destroyed pursuant to a routine document retention plan

As noted above a party to a lawsuit in federal court ldquomay obtain discovery regarding any nonprivileged matter that is relevant to any partyrsquos claim or defenserdquo FED R CIV P 26(b)(1) This includes emails and other electronically stored information FED R CIV P 34(a)(1)(A) Here the customer has requested all the builderrsquos emails pertaining to work done on the foundation of the house Ordinarily the builder would be obliged to turn over this information which is relevant to the customerrsquos defense that the housersquos foundation was poorly constructed

Unfortunately the emails in question no longer exist because the builder destroyed them on August 2

In general spoliation of evidence (destruction or alteration of evidence) is improper if the party who destroyed or altered the evidence ldquohas notice that the evidence is relevant to litigation or should have known that the evidence may be relevant to future litigationrdquo Fujitsu Ltd v Federal Express Corp 247 F3d 423 436 (2d Cir 2001) It is improper for a party to destroy electronic information relevant to pending litigation even if the destruction occurs before there is any request or order seeking the information See eg Leon v IDX Sys Corp 464 F3d 951 (9th Cir 2006) (plaintiffrsquos intentional destruction of computer files warranted dismissal even

20

In this case the builderrsquos destruction of the emails was pursuant to a routine document retention plan The Federal Rules provide expressly that in the absence of ldquoexceptional circumstancesrdquo parties should not be sanctioned for the loss of electronically stored information when the loss occurs pursuant to ldquoroutine good-faith operation of an electronic information systemrdquo FED R CIV P 37(e) However when a party anticipates litigation ldquoit must suspend its routine document retentiondestruction policy and put in place a lsquolitigation holdrsquo to ensure the preservation of relevant documentsrdquo Zubulake v UBS Warburg LLC 220 FRD 212 218 (SDNY 2003)

Federal Civil Procedure Analysis

though spoliation occurred before order compelling discovery) Similarly the duty to preserve evidence applies to a party who anticipates litigation even if litigation has not yet been commenced See THE SEDONA PRINCIPLES BEST PRACTICES RECOMMENDATIONS amp PRINCIPLES FOR ADDRESSING ELECTRONIC DOCUMENT PRODUCTION 70 cmt 14a (2d ed 2007)

The builder destroyed the emails on August 2 At that time the builder knew that litigation was a possibility because the builder had already directed its attorney to prepare a draft complaint for possible filing Knowing that litigation was a possibility the builder had a duty to take steps to preserve evidence including the emails in question See generally Fujitsu Ltd

Thus the builderrsquos destruction of potentially relevant emails at a time when it knew that litigation was a possibility was improper It had a duty to preserve evidence and it breached that duty

[NOTE Because courts have used different words to describe the test for when evidence must be preserved an examineersquos precise formulation of the test is not critical]

Point Two(b) (30) In determining appropriate sanctions for spoliation courts consider both the level of culpability of the spoliating party and the degree of prejudice the loss of evidence has caused the other party Here the builderrsquos destruction of evidence does not appear to have been willful nor is it likely to pose a significant obstacle to the customerrsquos defense Any sanctions imposed by the court should be modest

Federal courts have inherent power to control the litigation process and can sanction misbehavior including spoliation even when there has been no specific violation of the Federal Rules of Civil Procedure See generally Chambers v NASCO Inc 501 US 32 (1991) (discussing courtrsquos inherent power to control the litigation process) The range of available sanctions is broad It can include such sanctions as the payment of expenses incurred by the other party as a result of the destruction of the evidence an instruction to the jury authorizing it to draw an adverse inference from the destruction of the evidence a shifting of the burden of proof on the relevant issue or even judgment against the responsible party See eg Residential Funding Corp v DeGeorge Financial Corp 306 F3d 99 108 (2d Cir 2002) (adverse inference) Silvestri v General Motors Corp 271 F3d 583 593 (4th Cir 2001) (possibility of dismissal) Cf FED R CIV P 37(b)(2)(A) (listing remedies for failure to comply with discovery obligations)

In determining appropriate sanctions for spoliation courts consider both the level of culpability of the spoliating party and the degree of prejudice the loss of evidence has caused the other party Many courts impose severe sanctions (such as an adverse-inference instruction or the entry of judgment against the spoliating party) only when there is evidence of bad faith in the form of an intentional effort to hide information Eg Greyhound Lines Inc v Wade 485 F3d 1032 1035 (8th Cir 2007) (spoliation sanction requires intentional destruction out of desire ldquoto suppress the truthrdquo) However other courts have said that negligence in preserving evidence can

21

Federal Civil Procedure Analysis

support an adverse-inference instruction See Residential Funding 306 F3d at 108 (negligence enough under some circumstances)

Although a court might well order an evidentiary hearing on the issue of sanctions the facts presented do not seem appropriate for severe sanctions First the evidence was destroyed pursuant to the builderrsquos standard document retention plan and there is no evidence that the builder deliberately failed to suspend its usual procedures with the purpose of allowing the destruction of evidence Second the loss of this evidence will not severely hinder the customerrsquos presentation of his case The central issue is whether the foundation of the house was properly constructed If the construction job was poorly done the customer can present evidence derived from inspection of the premises to prove that point The customer can also depose witnesses about any issues that arose during construction

Under the circumstances a court is not likely to impose particularly severe sanctions although it might shift the burden to the builder to show that the foundation was properly constructed or it might require the builder to reimburse any expenses the customer incurs to discover and prove the facts about issues or disputes that arose during construction of the foundation

[NOTE The result reached by the examinee is less important than the examineersquos recognition that (a) a range of sanctions is available to the court and (b) the appropriate sanction depends both on the culpability of the builder and the prejudice suffered by the customer]

22

CRIMINAL LAW AND PROCEDURE ANALYSIS (Criminal Law and Procedure IIA amp D VE amp F)

ANALYSIS

Legal Problems

(1) Did charging the defendant with both theft and burglary constitute double jeopardy

(2) Did the jury instruction violate the due process clause either by relieving the prosecution of the burden of proving the element of intent or by shifting the burden to the defendant to disprove that element

(3) Did the sentence imposed in this case for the theft conviction unconstitutionally deprive the defendant of his right to a jury trial on the issue of the value of the stolen item

DISCUSSION

Summary

The trial court properly denied the defendantrsquos pretrial motion to dismiss the charges on double jeopardy grounds The defendant may be charged with and convicted of both theft and burglary Each of the charges has an element that the other does not Neither charge is a lesser-included offense nor are they multiplicitous Thus charging both theft and burglary does not violate double jeopardy

The jury instruction on the burglary charge was constitutionally flawed It could have been reasonably understood by the jury as either (1) an irrebuttable conclusive presumption (which relieved the prosecution of proving the element of intent and removed the issue from the jury) or (2) a rebuttable mandatory presumption (which unconstitutionally shifted the burden of proof on an element of a charged offense from the prosecution to the defendant)

Because the four-year sentence imposed by the judge was based on the judgersquos finding by a preponderance of the evidence that the value of the stolen ring exceeded $5000 the sentence violates the defendantrsquos right to a jury determination beyond a reasonable doubt of the value of the ring

Point One (30) Charging the defendant with theft and burglary did not constitute double jeopardy

The Double Jeopardy Clause of the Fifth Amendment provides that a person shall not be twice put in jeopardy for the ldquosame offenserdquo Thus the question is whether the elements of the theft charge are wholly contained in the burglary charge or vice versa If the elements of the lesser charge (theft) are not wholly contained in the greater charge (burglary)mdashie if each charge requires proof of a fact that the other does notmdashthen convicting the defendant of both crimes would not violate double jeopardy even when the two offenses occurred at the same time and are thus arguably part of the ldquosame transactionrdquo Blockburger v United States 284 US 299 304 (1932) See also Albernaz v United States 450 US 333 344 n3 (1981) United States v Dixon 509 US 688 704 (1993)

23

Criminal Law and Procedure Analysis

Here theft and burglary each require proof of an element not required for the other crime Burglary may be defined differently in different jurisdictions However it almost invariably requires entry into a building or dwelling of another with the specific intent to commit a felony therein and the crime of burglary is complete upon the entry into the building or dwelling with such intent See eg Cannon v Oklahoma 827 P2d 1339 1342 (Okla Crim App 1992) In contrast theft which also may be defined differently in different states almost invariably requires the taking and carrying away of an item of personal property belonging to another with the intent to steal or permanently deprive the owner of possession

Here the ldquotakingrdquo or ldquostealingrdquo element is not contained in the definition of burglary and the ldquoentryrdquo element of burglary is not contained in the definition of theft Because theft is not a lesser-included offense of burglary and burglary is not a lesser-included offense of theft charging the defendant for both burglary and theft did not violate double jeopardy and the court properly denied the defense motion on those grounds Yparrea v Dorsey 64 F3d 577 579ndash80 (10th Cir 1995) citing Blockburger 284 US at 304

Finally the defendantrsquos motion to dismiss all the charges on double jeopardy grounds was improper because if both charges were for the same offense the motion should have requested dismissal of one charge not both

Point Two (35) The jury instruction on the burglary charge violated the Due Process Clause because it created either (1) an irrebuttable conclusive presumption (which relieved the prosecution of proving the element of intent and removed that issue from the jury) or (2) a rebuttable mandatory presumption (which unconstitutionally shifted the burden of proof on an element of a charged offense to the defendant)

The Supreme Court has interpreted the Due Process Clause of the US Constitution to require that the prosecution prove all elements of an offense beyond a reasonable doubt See In re Winship 397 US 358 364 (1970) The burden of proof cannot be shifted to the defendant by presuming an essential element upon proof of other elements of the offense because shifting the burden of persuasion with respect to any element of a criminal offense is contrary to the Due Process Clause See Mullaney v Wilbur 421 US 684 (1975)

The crime of burglary includes entry into a building or dwelling with the specific intent to commit a felony therein The requirement that the prosecutor prove beyond a reasonable doubt that the defendant had this specific intent distinguishes burglary from general-intent crimes like trespass See Sandstrom v Montana 442 US 510 523 (1979)

Here the jury was instructed that if ldquoafter consideration of all the evidence presented by the prosecution and defense you find beyond a reasonable doubt that the defendant entered the dwelling without the ownersrsquo consent you may presume that the defendant entered with the intent to commit a felony thereinrdquo This instruction was unconstitutional because it created either an irrebuttable conclusive presumption or a rebuttable mandatory presumption

A conclusive presumption is ldquoan irrebuttable direction by the court to find intent once convinced of the facts triggering the presumptionrdquo Id at 517 Here the jurors were instructed that once the prosecutor established that the defendant entered the neighborsrsquo house without consent they ldquomay presumerdquo that he intended to commit a felony therein The jurors may have reasonably concluded from this instruction that if they found that the defendant intended to enter his neighborsrsquo home without permission they must further find that he entered with the specific intent to commit a felony therein Because this instruction could operate as a conclusive

24

Criminal Law and Procedure Analysis

irrebuttable presumption by eliminating intent ldquoas an ingredient of the offenserdquo it violated due process by relieving the prosecution of the burden of proof for this element Id at 522

In the alternative the jury instruction could have been reasonably understood to create a rebuttable mandatory presumption which ldquotells [the jury] they must find the elemental fact upon proof of the basic fact at least unless the defendant has come forward with some evidence to rebut the presumed connection between the two factsrdquo County Court of Ulster County New York v Allen 442 US 140 157 (1979) The due process problem created by rebuttable mandatory presumptions is that ldquo[t]o the extent that the trier of fact is forced to abide by the presumption and may not reject it based on an independent evaluation of the particular facts presented by the State the analysis of the presumptionrsquos constitutional validity is logically divorced from those facts and based on the presumptionrsquos accuracy in the run of casesrdquo Id at 159

Unlike irrebuttable conclusive presumptions rebuttable mandatory presumptions are not always per se violations of the Due Process Clause However the Supreme Court of the United States has held that jury instructions that could reasonably be understood as shifting the burden of proof to the defendant on an element of the offense are unconstitutional Francis v Franklin 471 US 307 (1985) Here the argument that the jury instruction operated as a rebuttable mandatory presumption is supported by the fact that the judge also instructed the jury to ldquoconsider[ ] all the evidence presented by the prosecution and defenserdquo However even if the instruction created a rebuttable mandatory presumption it would be unconstitutional because it shifted the burden to the defense on an element of the offense Sandstrom 442 US at 524 Mullaney 421 US at 686

[NOTE Whether an examinee identifies the jury instruction as containing a ldquoconclusiverdquo or ldquomandatoryrdquo presumption is less important than the examineersquos analysis of the constitutional infirmities]

Point Three (35) The trial court violated the defendantrsquos Sixth Amendment right to a jury trial on an essential element of the offense when it found by a preponderance of the evidence that the ring was worth over $5000 and increased the defendantrsquos sentence based on this finding

In the statutory scheme under which the defendant was tried and convicted a Class D felony theft is defined as theft of item(s) with a value between $2500 and $10000 The jury found that the value of the diamond ring was at least $2500 and convicted the defendant of felony theft However at sentencing the trial court made a separate finding by a preponderance of the evidence that the value of the ring was greater than $5000 Following the statutersquos two-tiered sentencing scheme the judge then imposed on the defendant a sentence that was one year longer than the maximum that would otherwise have been allowed

The judgersquos sentence was unconstitutional because it violated the defendantrsquos Sixth Amendment right to a jury trial on this question The Supreme Court held in Apprendi v New Jersey 530 US 466 (2000) that ldquo[o]ther than the fact of a prior conviction any fact that increases the penalty for a crime beyond the prescribed statutory maximum must be submitted to a jury and proved beyond a reasonable doubtrdquo because ldquo[i]t is unconstitutional for a legislature to remove from the jury the assessment of facts that increase the prescribed range of penalties to which a criminal defendant is exposed [because] such facts must be established by proof beyond a reasonable doubtrdquo Id The Court reaffirmed Apprendi in Blakely v Washington 542 US 296 (2004) holding that the ldquolsquostatutory maximumrsquo for Apprendi purposes is the maximum sentence a judge may impose solely on the basis of the facts reflected in the jury verdict or admitted by the defendantrdquo Id at 303 (emphasis in original) In United States v Booker 543 US 220 (2005)

25

Criminal Law and Procedure Analysis

the Court relied on Blakely and Apprendi to conclude that protecting a defendantrsquos Sixth Amendment right to a jury trial required that ldquo[a]ny fact which is necessary to support a sentence exceeding the maximum authorized by the facts established by a plea of guilty or a jury verdict must be admitted by the defendant or proved to a jury beyond a reasonable doubtrdquo Id at 244

Thus in order to constitutionally increase a sentence above the statutory maximum of three years the jury must have found beyond a reasonable doubt that the value of the ring exceeded $5000 Here the court made the finding based on an appraisal proffered by the prosecutor only at sentencing and the judgersquos finding was by a preponderance of the evidence rather than beyond a reasonable doubt

26

AGENCY AND PARTNERSHIP ANALYSIS __________ (Agency and Partnership VA amp C VI)

ANALYSIS

Legal Problems

(1) Is a partner in a general partnership personally liable on a claim arising from misrepresentations by another partner made in the course of the partnership business

(2) Does a newly admitted partner in a general partnership become personally liable on existing claims against the partnership

(3) After the filing by a general partnership of a statement of qualification as a limited liability partnership are the partners personally liable as partners on (a) an existing claim against the general partnership and (b) a claim against the partnership that arose after the filing

DISCUSSION

Summary

Adam and Ben formed a general partnership under which they were jointly and severally liable for obligations of the partnership Thus Adam was personally liable for misrepresentations by Ben made in the ordinary course of the partnership business

Upon joining the general partnership Diane became personally liable for the obligations of the partnership arising after her admission but not for obligations pre-existing her admission such as the collectorrsquos claim

By filing a statement of qualification the three partners properly elected limited liability partnership status As partners in an LLP none of the three partners is personally liable as a partner for partnership obligations arising after the election such as the claim by the driverrsquos estate The election however does not change their personal liability on pre-existing claims that arose before the election such as the collectorrsquos claim

Point One (30) As a general partner of Empire a general partnership Adam became personally liable on the collectorrsquos claim a valid claim against the partnership that arose because of Benrsquos wrongful act in the ordinary course of the partnership business

When the collectorrsquos claim arose Empire was a general partnership composed of Adam and Ben Under UPA (1997) sect 306(a) partners of a general partnership are liable jointly and severally for all obligations of the partnership Under UPA (1997) sect 305(a) the partnership could become obligated for the loss caused to the collector as a result of the misrepresentation by Ben provided he was acting in the ordinary course of the partnership business Because there was no statement that limited his partnership authority Ben as partner was ldquoan agent of the partnership for the purpose of its businessrdquo See UPA (1997) sect 301(1) Benrsquos misrepresentation to the collector even if intentional appears to be in the ordinary course of the partnershiprsquos business of dealing

27

Agency and Partnership Analysis

in antique cars Thus Benrsquos wrongful act created a partnership obligation for which Adam was jointly and severally liable

[NOTE Generally a partnership creditor must ldquoexhaust the partnershiprsquos assets before levying on a judgment debtor partnerrsquos individual property where the partner is personally liable for the partnership obligationrdquo as a result of his status as a partner UPA (1997) sect 307 cmt 4 As the UPA comments explain this places Adam more in the position of guarantor than principal debtor on the partnership obligation Id cmt 4 Although an examinee might discuss this point the call focuses on whether Adam is personally liable not how the liability might be enforced]

Point Two (30) Because the collectorrsquos claim arose before Diane joined Empire Diane did not become personally liable on the claim

Diane was admitted to Empire when it was a general partnership and after the collectorrsquos claim arose While the general rule under UPA (1997) sect 306(a) is that the partners of a general partnership are liable jointly and severally for all obligations of the partnership there is a special rule for partners who are admitted during the duration of the partnership Under UPA (1997) sect 306(b) a person admitted to an existing partnership is not personally liable for any partnership obligations incurred before the personrsquos admission Because Diane was admitted to Empire after the collectorrsquos claim arose Diane is not personally liable on the claim

Dianersquos knowledge of the pre-existing claim and her stated concern about becoming liable on the collectorrsquos claim do not change her personal nonliability to the collector Although partners who have a liability shield can assume liability to third parties through private contractual guarantees or modifications to the partnership agreement Dianersquos stated concern constituted neither a guaranty to the collector nor ldquoan intentional waiver of liability protectionsrdquo See UPA (1997) sect 306 cmt 3 (describing methods for waiver of liability protections under sect 306(c) applicable in limited liability partnerships)

At most Diane will lose her investment in the partnership as a result of the collectorrsquos claim Although Diane did not become personally liable on the collectorrsquos claim when she joined the partnership the $250000 she contributed to the partnership is ldquoat risk for the satisfaction of existing partnership debtsrdquo UPA (1997) sect 306 cmt 2

Point Three (40) Filing the statement of qualification was effective to elect limited liability partnership status Despite this new status Adam and Ben remain personally liable on the collectorrsquos claim which arose before the election But as partners in an LLP neither Adam Ben nor Diane is personally liable as a partner on the driverrsquos estatersquos claim which arose after the election

Under UPA (1997) sect 1001 a general partnership can make an election and become a limited liability partnershipmdashif the partners approve the conversion by a vote equivalent to that necessary to amend the partnership agreement and the partnership then files a statement of qualification that specifies the name of the partnership its principal office and its election to be an LLP Here the partners agreed unanimouslymdashsufficient to amend their agreement under UPA (1997) sect 401(j)mdashand the statement of qualification was filed In addition the name of Empire LLP properly included an appropriate ending ldquoLLPrdquo See UPA (1997) sect 1002

Although another way to effectuate a ldquoconversionrdquo (as suggested by Benrsquos lawyer) is to form a new LLP and transfer the assets of the old general partnership to the new LLP the

28

Agency and Partnership Analysis

method used here (approval by the partners and the filing of a statement of qualification) is also sufficient to create LLP status

Thus Empire became Empire LLP as of the date of filing of the statement of qualification See UPA (1997) sect 1001 What effect did this have on the collectorrsquos claim which predated the filing According to UPA (1997) sect 306(c) an obligation incurred while a partnership is an LLP is solely a partnership obligation As the collectorrsquos claim predated the LLP Adam and Ben remain personally liable on the collectorrsquos claim Diane on the other hand was not personally liable on the collectorrsquos claim either before or after the filing of the statement of qualification See Point Two above

The driverrsquos estatersquos claim arose after Empire became Empire LLP Under UPA (1997) sect 306(c) an obligation incurred while a partnership is an LLP is solely a partnership obligationThus Adam Ben and Diane as partners are all protected from personal liability on the driverrsquos estatersquos claim But there may be personal liability if any of them was negligent or otherwise acted wrongfully by not informing the buyer of the bad suspension that caused the accident

29

National Conference of Bar Examiners 302 South Bedford Street | Madison WI 53703-3622 Phone 608-280-8550 | Fax 608-280-8552 | TDD 608-661-1275

wwwncbexorg e-mail contactncbexorg

  • Preface
  • Description of the MEE
  • Instructions
  • February 2014 Questions
    • Constitutinal Law Question
    • Trusts and Future Interests Question
    • Secured Transactions Question
    • Federal Civil Procedure Question
    • Criminal Law and Procedure Question
    • Agency and Partnership Question
      • February 2014 Analyses
        • Constitutional Law Analysis
        • Trust and Future Interests Analysis
        • Secured Transactions Analysis
        • Federal Civil Procedure Analysis
        • Criminal Law and Procedure Analysis
        • Agency and Partnership Analysis
            • ltlt13 ASCII85EncodePages false13 AllowTransparency false13 AutoPositionEPSFiles true13 AutoRotatePages None13 Binding Left13 CalGrayProfile (Dot Gain 20)13 CalRGBProfile (sRGB IEC61966-21)13 CalCMYKProfile (US Web Coated 050SWOP051 v2)13 sRGBProfile (sRGB IEC61966-21)13 CannotEmbedFontPolicy Error13 CompatibilityLevel 1413 CompressObjects Tags13 CompressPages true13 ConvertImagesToIndexed true13 PassThroughJPEGImages true13 CreateJobTicket false13 DefaultRenderingIntent Default13 DetectBlends true13 DetectCurves 0000013 ColorConversionStrategy CMYK13 DoThumbnails false13 EmbedAllFonts true13 EmbedOpenType false13 ParseICCProfilesInComments true13 EmbedJobOptions true13 DSCReportingLevel 013 EmitDSCWarnings false13 EndPage -113 ImageMemory 104857613 LockDistillerParams false13 MaxSubsetPct 10013 Optimize true13 OPM 113 ParseDSCComments true13 ParseDSCCommentsForDocInfo true13 PreserveCopyPage true13 PreserveDICMYKValues true13 PreserveEPSInfo true13 PreserveFlatness true13 PreserveHalftoneInfo false13 PreserveOPIComments true13 PreserveOverprintSettings true13 StartPage 113 SubsetFonts true13 TransferFunctionInfo Apply13 UCRandBGInfo Preserve13 UsePrologue false13 ColorSettingsFile ()13 AlwaysEmbed [ true13 ]13 NeverEmbed [ true13 ]13 AntiAliasColorImages false13 CropColorImages true13 ColorImageMinResolution 30013 ColorImageMinResolutionPolicy OK13 DownsampleColorImages true13 ColorImageDownsampleType Bicubic13 ColorImageResolution 30013 ColorImageDepth -113 ColorImageMinDownsampleDepth 113 ColorImageDownsampleThreshold 15000013 EncodeColorImages true13 ColorImageFilter DCTEncode13 AutoFilterColorImages true13 ColorImageAutoFilterStrategy JPEG13 ColorACSImageDict ltlt13 QFactor 01513 HSamples [1 1 1 1] VSamples [1 1 1 1]13 gtgt13 ColorImageDict ltlt13 QFactor 01513 HSamples [1 1 1 1] VSamples [1 1 1 1]13 gtgt13 JPEG2000ColorACSImageDict ltlt13 TileWidth 25613 TileHeight 25613 Quality 3013 gtgt13 JPEG2000ColorImageDict ltlt13 TileWidth 25613 TileHeight 25613 Quality 3013 gtgt13 AntiAliasGrayImages false13 CropGrayImages true13 GrayImageMinResolution 30013 GrayImageMinResolutionPolicy OK13 DownsampleGrayImages true13 GrayImageDownsampleType Bicubic13 GrayImageResolution 30013 GrayImageDepth -113 GrayImageMinDownsampleDepth 213 GrayImageDownsampleThreshold 15000013 EncodeGrayImages true13 GrayImageFilter DCTEncode13 AutoFilterGrayImages true13 GrayImageAutoFilterStrategy JPEG13 GrayACSImageDict ltlt13 QFactor 01513 HSamples [1 1 1 1] VSamples [1 1 1 1]13 gtgt13 GrayImageDict ltlt13 QFactor 01513 HSamples [1 1 1 1] VSamples [1 1 1 1]13 gtgt13 JPEG2000GrayACSImageDict ltlt13 TileWidth 25613 TileHeight 25613 Quality 3013 gtgt13 JPEG2000GrayImageDict ltlt13 TileWidth 25613 TileHeight 25613 Quality 3013 gtgt13 AntiAliasMonoImages false13 CropMonoImages true13 MonoImageMinResolution 120013 MonoImageMinResolutionPolicy OK13 DownsampleMonoImages true13 MonoImageDownsampleType Bicubic13 MonoImageResolution 120013 MonoImageDepth -113 MonoImageDownsampleThreshold 15000013 EncodeMonoImages true13 MonoImageFilter CCITTFaxEncode13 MonoImageDict ltlt13 K -113 gtgt13 AllowPSXObjects false13 CheckCompliance [13 None13 ]13 PDFX1aCheck false13 PDFX3Check false13 PDFXCompliantPDFOnly false13 PDFXNoTrimBoxError true13 PDFXTrimBoxToMediaBoxOffset [13 00000013 00000013 00000013 00000013 ]13 PDFXSetBleedBoxToMediaBox true13 PDFXBleedBoxToTrimBoxOffset [13 00000013 00000013 00000013 00000013 ]13 PDFXOutputIntentProfile ()13 PDFXOutputConditionIdentifier ()13 PDFXOutputCondition ()13 PDFXRegistryName ()13 PDFXTrapped False1313 CreateJDFFile false13 Description ltlt13 ARA 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 BGR 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 CHS ltFEFF4f7f75288fd94e9b8bbe5b9a521b5efa7684002000410064006f006200650020005000440046002065876863900275284e8e9ad88d2891cf76845370524d53705237300260a853ef4ee54f7f75280020004100630072006f0062006100740020548c002000410064006f00620065002000520065006100640065007200200035002e003000204ee553ca66f49ad87248672c676562535f00521b5efa768400200050004400460020658768633002gt13 CHT ltFEFF4f7f752890194e9b8a2d7f6e5efa7acb7684002000410064006f006200650020005000440046002065874ef69069752865bc9ad854c18cea76845370524d5370523786557406300260a853ef4ee54f7f75280020004100630072006f0062006100740020548c002000410064006f00620065002000520065006100640065007200200035002e003000204ee553ca66f49ad87248672c4f86958b555f5df25efa7acb76840020005000440046002065874ef63002gt13 CZE 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 DAN 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 DEU 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 ESP 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 ETI 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 FRA 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 GRE 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 HEB ltFEFF05D405E905EA05DE05E905D5002005D105D405D205D305E805D505EA002005D005DC05D4002005DB05D305D9002005DC05D905E605D505E8002005DE05E105DE05DB05D9002000410064006F006200650020005000440046002005D405DE05D505EA05D005DE05D905DD002005DC05D405D305E405E105EA002005E705D305DD002D05D305E405D505E1002005D005D905DB05D505EA05D905EA002E002005DE05E105DE05DB05D90020005000440046002005E905E005D505E605E805D5002005E005D905EA05E005D905DD002005DC05E405EA05D905D705D4002005D105D005DE05E605E205D505EA0020004100630072006F006200610074002005D5002D00410064006F00620065002000520065006100640065007200200035002E0030002005D505D205E805E105D005D505EA002005DE05EA05E705D305DE05D505EA002005D905D505EA05E8002E05D005DE05D905DD002005DC002D005000440046002F0058002D0033002C002005E205D905D905E005D5002005D105DE05D305E805D905DA002005DC05DE05E905EA05DE05E9002005E905DC0020004100630072006F006200610074002E002005DE05E105DE05DB05D90020005000440046002005E905E005D505E605E805D5002005E005D905EA05E005D905DD002005DC05E405EA05D905D705D4002005D105D005DE05E605E205D505EA0020004100630072006F006200610074002005D5002D00410064006F00620065002000520065006100640065007200200035002E0030002005D505D205E805E105D005D505EA002005DE05EA05E705D305DE05D505EA002005D905D505EA05E8002Egt13 HRV (Za stvaranje Adobe PDF dokumenata najpogodnijih za visokokvalitetni ispis prije tiskanja koristite ove postavke Stvoreni PDF dokumenti mogu se otvoriti Acrobat i Adobe Reader 50 i kasnijim verzijama)13 HUN 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 ITA 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 JPN ltFEFF9ad854c18cea306a30d730ea30d730ec30b951fa529b7528002000410064006f0062006500200050004400460020658766f8306e4f5c6210306b4f7f75283057307e305930023053306e8a2d5b9a30674f5c62103055308c305f0020005000440046002030d530a130a430eb306f3001004100630072006f0062006100740020304a30883073002000410064006f00620065002000520065006100640065007200200035002e003000204ee5964d3067958b304f30533068304c3067304d307e305930023053306e8a2d5b9a306b306f30d530a930f330c8306e57cb30818fbc307f304c5fc59808306730593002gt13 KOR ltFEFFc7740020c124c815c7440020c0acc6a9d558c5ec0020ace0d488c9c80020c2dcd5d80020c778c1c4c5d00020ac00c7a50020c801d569d55c002000410064006f0062006500200050004400460020bb38c11cb97c0020c791c131d569b2c8b2e4002e0020c774b807ac8c0020c791c131b41c00200050004400460020bb38c11cb2940020004100630072006f0062006100740020bc0f002000410064006f00620065002000520065006100640065007200200035002e00300020c774c0c1c5d0c11c0020c5f40020c2180020c788c2b5b2c8b2e4002egt13 LTH 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 LVI 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 NLD (Gebruik deze instellingen om Adobe PDF-documenten te maken die zijn geoptimaliseerd voor prepress-afdrukken van hoge kwaliteit De gemaakte PDF-documenten kunnen worden geopend met Acrobat en Adobe Reader 50 en hoger)13 NOR ltFEFF004200720075006b00200064006900730073006500200069006e006e007300740069006c006c0069006e00670065006e0065002000740069006c002000e50020006f0070007000720065007400740065002000410064006f006200650020005000440046002d0064006f006b0075006d0065006e00740065007200200073006f006d00200065007200200062006500730074002000650067006e0065007400200066006f00720020006600f80072007400720079006b006b0073007500740073006b00720069006600740020006100760020006800f800790020006b00760061006c0069007400650074002e0020005000440046002d0064006f006b0075006d0065006e00740065006e00650020006b0061006e002000e50070006e00650073002000690020004100630072006f00620061007400200065006c006c00650072002000410064006f00620065002000520065006100640065007200200035002e003000200065006c006c00650072002000730065006e006500720065002egt13 POL 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 PTB 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 RUM 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 RUS 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 SKY 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 SLV 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 SUO 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 SVE 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 TUR ltFEFF005900fc006b00730065006b0020006b0061006c006900740065006c0069002000f6006e002000790061007a006401310072006d00610020006200610073006b013100730131006e006100200065006e0020006900790069002000750079006100620069006c006500630065006b002000410064006f006200650020005000440046002000620065006c00670065006c0065007200690020006f006c0075015f007400750072006d0061006b0020006900e70069006e00200062007500200061007900610072006c0061007201310020006b0075006c006c0061006e0131006e002e00200020004f006c0075015f0074007500720075006c0061006e0020005000440046002000620065006c00670065006c0065007200690020004100630072006f006200610074002000760065002000410064006f00620065002000520065006100640065007200200035002e003000200076006500200073006f006e0072006100730131006e00640061006b00690020007300fc007200fc006d006c00650072006c00650020006100e70131006c006100620069006c00690072002egt13 UKR 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 ENU (Use these settings to create Adobe PDF documents best suited for high-quality prepress printing Created PDF documents can be opened with Acrobat and Adobe Reader 50 and later)13 gtgt13 Namespace [13 (Adobe)13 (Common)13 (10)13 ]13 OtherNamespaces [13 ltlt13 AsReaderSpreads false13 CropImagesToFrames true13 ErrorControl WarnAndContinue13 FlattenerIgnoreSpreadOverrides false13 IncludeGuidesGrids false13 IncludeNonPrinting false13 IncludeSlug false13 Namespace [13 (Adobe)13 (InDesign)13 (40)13 ]13 OmitPlacedBitmaps false13 OmitPlacedEPS false13 OmitPlacedPDF false13 SimulateOverprint Legacy13 gtgt13 ltlt13 AddBleedMarks false13 AddColorBars false13 AddCropMarks false13 AddPageInfo false13 AddRegMarks false13 ConvertColors ConvertToCMYK13 DestinationProfileName ()13 DestinationProfileSelector DocumentCMYK13 Downsample16BitImages true13 FlattenerPreset ltlt13 PresetSelector MediumResolution13 gtgt13 FormElements false13 GenerateStructure false13 IncludeBookmarks false13 IncludeHyperlinks false13 IncludeInteractive false13 IncludeLayers false13 IncludeProfiles false13 MultimediaHandling UseObjectSettings13 Namespace [13 (Adobe)13 (CreativeSuite)13 (20)13 ]13 PDFXOutputIntentProfileSelector DocumentCMYK13 PreserveEditing true13 UntaggedCMYKHandling LeaveUntagged13 UntaggedRGBHandling UseDocumentProfile13 UseDocumentBleed false13 gtgt13 ]13gtgt setdistillerparams13ltlt13 HWResolution [2400 2400]13 PageSize [612000 792000]13gtgt setpagedevice13

Page 20: February 2014 MEE Questions and AnalysesPreface The Multistate Essay Examination (MEE) is developed by the National Conference of Bar Examiners (NCBE). This publication includes the

SECURED TRANSACTIONS ANALYSIS (Secured Transactions IB IID E amp F IIIB IVA B amp F)

ANALYSIS

Legal Problems

(1)(a) What is the nature of the bankrsquos claim to the businessrsquos equipment

(1)(b) What is the nature of the finance companyrsquos claim to the businessrsquos equipment

(1)(c) As between the bank and the finance company whose claim to the businessrsquos equipment has priority

(2) Do the claims of the bank and the finance company continue in the item of equipment sold by the business to the competitor

DISCUSSION

Summary

The bank and the finance company both have perfected security interests in the businessrsquos equipment Even though the finance companyrsquos perfected security interest was created first the bankrsquos perfected security interest has priority because the bankrsquos financing statement was filed before the finance companyrsquos financing statement The security interests of the bank and the finance company continue in the item of equipment sold by the business to the competitor because their security interests were perfected and the competitor was not a buyer in ordinary course of business

Point One(a) (25) The bank has a perfected security interest in the businessrsquos equipment

The bank has met all criteria necessary for it to have an attached and enforceable security interest in the businessrsquos equipment First value must be given UCC sect 9-203(b)(1) This criterion is fulfilled by the loan made by the bank to the business Second the debtor must have rights in the collateral UCC sect 9-203(b)(2) Clearly the business has rights in its equipment Third either the secured party must take possession of the collateral or the debtor must authenticate a security agreement containing a description of the collateral UCC sect 9-203(b)(3) The agreement that the business owner signed is a ldquosecurity agreementrdquo because it is an agreement that creates or provides for a security interest UCC sect 9-102(a)(74) By signing the security agreement the business owner authenticated it UCC sect 9-102(a)(7) Therefore all three criteria are fulfilled and the bank has an enforceable and attached security interest

A security interest is perfected when it has attached and when any additional steps required for perfection have occurred UCC sect 9-308(a) Generally speaking the additional steps will either be possession of the collateral by the secured party or the filing of a financing statement with respect to the collateral See UCC sectsect 9-310 9-313 In this case the bank filed a financing statement naming the debtor and sufficiently indicating the collateral The collateral indication is sufficient because it identifies the collateral by type of property See UCC sectsect 9-504 9-108 The fact that the financing statement was filed before the security interest was created is

17

Secured Transactions Analysis

not a problem Even though the security agreement had not yet been signed the business had authorized the filing of the financing statement in an authenticated record UCC sect 9-509(a)(1) Moreover the financing statement may be filed before the security agreement is created UCC sect 9-502(d)

Point One(b) (10) The finance company also has a perfected security interest in the businessrsquos equipment

The finance companyrsquos security interest is enforceable and attached for the same reasons as the bankrsquos security interest The loan from the finance company to the business constitutes value the business has rights in the collateral and the business owner has authenticated a security agreement containing a description of the collateral The finance companyrsquos security interest is perfected because the finance company filed a financing statement with respect to it that provides that the business is the debtor and indicates that the collateral is equipment

Point One(c) (30) The bankrsquos security interest has priority over the finance companyrsquos security interest because the bankrsquos financing statement was filed first

As between two perfected security interests the general rule is that the security interest that was the earlier to be either perfected or the subject of a filed financing statement has priority UCC sect 9-322(a)(1) While the finance companyrsquos security interest was perfected before the bankrsquos (March 15 vs March 22) the bankrsquos financing statement was filed even earlier on March 2 Thus under the first-to-file-or-perfect rule of UCC sect 9-322(a)(1) the bankrsquos security interest has priority No exceptions to the general rule apply here

Point Two (35) A security interest in collateral continues notwithstanding its sale unless an exception applies Because the security interests of the bank and the finance company were perfected and the competitor was not a buyer in ordinary course of business no exception applies and the security interests of both creditors continue in the equipment sold to the competitor

As a general rule a security interest in collateral continues notwithstanding the fact that the debtor has sold the collateral to another person UCC sect 9-315(a)(1) Thus unless an exception applies the security interests of the bank and the finance company will continue in the item of equipment sold to the competitor

A buyer of goods will take free of an unperfected security interest in those goods See UCC sect 9-317(a)(2) However when the competitor bought the businessrsquos equipment both the bank and the finance company had perfected security interests in the equipment

A buyer can take free even of a perfected security interest in goods if the buyer is a ldquobuyer in ordinary course of businessrdquo See UCC sect 9-320(a) However the competitor was not a buyer in ordinary course of business To be a ldquobuyer in ordinary course of businessrdquo a buyer must buy goods from a seller that is in the business of selling goods of that kind See UCC sect 1-201(b)(9) The competitor bought this equipment from a seller that is not in the business of selling goods of this kind so the competitor was not a buyer in ordinary course of business with respect to these goods

Because no exception applies the security interests of the bank and the finance company continue even after the item of equipment was sold to the competitor

18

FEDERAL CIVIL PROCEDURE ANALYSIS (Federal Civil Procedure IVD)

ANALYSIS

Legal Problems

(1) Is a document prepared in the course of a contract dispute protected from discovery as ldquowork productrdquo when there is no evidence that the document was prepared in anticipation of litigation

(2)(a) Is a partyrsquos failure to provide relevant electronically stored information excused when the information was destroyed pursuant to a routine document retention scheme at a time when litigation was contemplated by the destroying party

(2)(b) What sanctions should be imposed on a party for allowing the destruction of evidence that is relevant to potential future litigation

DISCUSSION

Summary

The report prepared by the structural engineer is probably not work product and is thus discoverable The engineer examined the foundation of the house at the customerrsquos request and the engineerrsquos findings are potentially relevant to the customerrsquos claim that the foundation is defective The report was not prepared in anticipation of litigation The customer appears to have sought the engineerrsquos opinion in response to the builderrsquos offer to fix any problems with the foundation that an engineer might identify Because the report was not prepared in anticipation of litigation it is not protected by the work-product doctrine

The builder should have taken appropriate steps to preserve evidence including suspending its document retention program as soon as it began planning for litigationmdashie on July 10 Its destruction of potentially relevant material after that date was wrongful However a court is unlikely to impose severe sanctions on the builder because there are no facts indicating that the builder acted in bad faith and the customer can prove that the foundation is defective without the destroyed emails

Point One (40) The customer must turn over the engineerrsquos report because it was not prepared in anticipation of litigation

In general a party to a lawsuit in federal court ldquomay obtain discovery regarding any nonprivileged matter that is relevant to any partyrsquos claim or defenserdquo FED R CIV P 26(b)(1) (2009) This includes the right to inspect and copy documents in the other partyrsquos possession FED R CIV P 34(a)(1) Here the customer hired a structural engineer to examine the foundation of the house The engineerrsquos report on the foundation is likely to include information that would be relevant to the customerrsquos claim that the foundation was defectively constructed

The so-called ldquowork productrdquo rule allows a party to refuse to turn over ldquodocuments that are prepared in anticipation of litigation or for trialrdquo by that partyrsquos representative including

19

Federal Civil Procedure Analysis

a consultant Thus if the customer had hired the structural engineer to prepare a report ldquoin anticipation of litigationrdquo that report might not be discoverable See FED R CIV P 26(b)(3)

In this case however the customer hired the engineer to evaluate the foundation of the house as part of the customerrsquos negotiation with the builder concerning the housersquos flooding problem The builder told the customer that the housersquos landscaping was the reason for the flooding and the builder told the customer ldquoHave an engineer look at the foundation If therersquos a problem wersquoll fix itrdquo The customer appears to have acted in response to that statement There is no indication that the customer anticipated any kind of legal action at the time that the structural engineer was hired Accordingly the structural engineerrsquos report is discoverable and the court should order the customer to turn it over

[NOTE If an examinee concludes that the structural engineerrsquos report was prepared in anticipation of litigation then the examinee should also conclude that the report is not discoverable Documents prepared in anticipation of litigation do not need to be disclosed to an adverse party unless that party can demonstrate a ldquosubstantial needrdquo for the documents and an inability to obtain substantially equivalent information without ldquoundue hardshiprdquo FED R CIV P 26(b)(3)(A)(ii) Furthermore a report prepared by an expert who is not expected to testify is not discoverable in the absence of ldquoexceptional circumstancesrdquo making it ldquoimpracticablerdquo to obtain the information in another way FED R CIV P 26(b)(4)(D)(ii) The builder probably cannot make these showings here unless the engineerrsquos report deals with circumstances that have since changed There is no evidence that the structural engineer would have had access to any information or facts that the builder would not already know as a result of its construction and subsequent inspection of the house In addition if necessary the builder could ask the court for permission to arrange for a further inspection of the house by a structural engineer hired by the builder See FED R CIV P 34(a)(2) Accordingly if an examinee concludes that the report was prepared in anticipation of litigation the examinee should also conclude that the builder is not entitled to see the report]

Point Two(a) (30) Because the builder anticipated that it might be involved in litigation concerning its contract with the customer the builder acted wrongfully in destroying emails that were relevant to the housersquos construction even though the emails were destroyed pursuant to a routine document retention plan

As noted above a party to a lawsuit in federal court ldquomay obtain discovery regarding any nonprivileged matter that is relevant to any partyrsquos claim or defenserdquo FED R CIV P 26(b)(1) This includes emails and other electronically stored information FED R CIV P 34(a)(1)(A) Here the customer has requested all the builderrsquos emails pertaining to work done on the foundation of the house Ordinarily the builder would be obliged to turn over this information which is relevant to the customerrsquos defense that the housersquos foundation was poorly constructed

Unfortunately the emails in question no longer exist because the builder destroyed them on August 2

In general spoliation of evidence (destruction or alteration of evidence) is improper if the party who destroyed or altered the evidence ldquohas notice that the evidence is relevant to litigation or should have known that the evidence may be relevant to future litigationrdquo Fujitsu Ltd v Federal Express Corp 247 F3d 423 436 (2d Cir 2001) It is improper for a party to destroy electronic information relevant to pending litigation even if the destruction occurs before there is any request or order seeking the information See eg Leon v IDX Sys Corp 464 F3d 951 (9th Cir 2006) (plaintiffrsquos intentional destruction of computer files warranted dismissal even

20

In this case the builderrsquos destruction of the emails was pursuant to a routine document retention plan The Federal Rules provide expressly that in the absence of ldquoexceptional circumstancesrdquo parties should not be sanctioned for the loss of electronically stored information when the loss occurs pursuant to ldquoroutine good-faith operation of an electronic information systemrdquo FED R CIV P 37(e) However when a party anticipates litigation ldquoit must suspend its routine document retentiondestruction policy and put in place a lsquolitigation holdrsquo to ensure the preservation of relevant documentsrdquo Zubulake v UBS Warburg LLC 220 FRD 212 218 (SDNY 2003)

Federal Civil Procedure Analysis

though spoliation occurred before order compelling discovery) Similarly the duty to preserve evidence applies to a party who anticipates litigation even if litigation has not yet been commenced See THE SEDONA PRINCIPLES BEST PRACTICES RECOMMENDATIONS amp PRINCIPLES FOR ADDRESSING ELECTRONIC DOCUMENT PRODUCTION 70 cmt 14a (2d ed 2007)

The builder destroyed the emails on August 2 At that time the builder knew that litigation was a possibility because the builder had already directed its attorney to prepare a draft complaint for possible filing Knowing that litigation was a possibility the builder had a duty to take steps to preserve evidence including the emails in question See generally Fujitsu Ltd

Thus the builderrsquos destruction of potentially relevant emails at a time when it knew that litigation was a possibility was improper It had a duty to preserve evidence and it breached that duty

[NOTE Because courts have used different words to describe the test for when evidence must be preserved an examineersquos precise formulation of the test is not critical]

Point Two(b) (30) In determining appropriate sanctions for spoliation courts consider both the level of culpability of the spoliating party and the degree of prejudice the loss of evidence has caused the other party Here the builderrsquos destruction of evidence does not appear to have been willful nor is it likely to pose a significant obstacle to the customerrsquos defense Any sanctions imposed by the court should be modest

Federal courts have inherent power to control the litigation process and can sanction misbehavior including spoliation even when there has been no specific violation of the Federal Rules of Civil Procedure See generally Chambers v NASCO Inc 501 US 32 (1991) (discussing courtrsquos inherent power to control the litigation process) The range of available sanctions is broad It can include such sanctions as the payment of expenses incurred by the other party as a result of the destruction of the evidence an instruction to the jury authorizing it to draw an adverse inference from the destruction of the evidence a shifting of the burden of proof on the relevant issue or even judgment against the responsible party See eg Residential Funding Corp v DeGeorge Financial Corp 306 F3d 99 108 (2d Cir 2002) (adverse inference) Silvestri v General Motors Corp 271 F3d 583 593 (4th Cir 2001) (possibility of dismissal) Cf FED R CIV P 37(b)(2)(A) (listing remedies for failure to comply with discovery obligations)

In determining appropriate sanctions for spoliation courts consider both the level of culpability of the spoliating party and the degree of prejudice the loss of evidence has caused the other party Many courts impose severe sanctions (such as an adverse-inference instruction or the entry of judgment against the spoliating party) only when there is evidence of bad faith in the form of an intentional effort to hide information Eg Greyhound Lines Inc v Wade 485 F3d 1032 1035 (8th Cir 2007) (spoliation sanction requires intentional destruction out of desire ldquoto suppress the truthrdquo) However other courts have said that negligence in preserving evidence can

21

Federal Civil Procedure Analysis

support an adverse-inference instruction See Residential Funding 306 F3d at 108 (negligence enough under some circumstances)

Although a court might well order an evidentiary hearing on the issue of sanctions the facts presented do not seem appropriate for severe sanctions First the evidence was destroyed pursuant to the builderrsquos standard document retention plan and there is no evidence that the builder deliberately failed to suspend its usual procedures with the purpose of allowing the destruction of evidence Second the loss of this evidence will not severely hinder the customerrsquos presentation of his case The central issue is whether the foundation of the house was properly constructed If the construction job was poorly done the customer can present evidence derived from inspection of the premises to prove that point The customer can also depose witnesses about any issues that arose during construction

Under the circumstances a court is not likely to impose particularly severe sanctions although it might shift the burden to the builder to show that the foundation was properly constructed or it might require the builder to reimburse any expenses the customer incurs to discover and prove the facts about issues or disputes that arose during construction of the foundation

[NOTE The result reached by the examinee is less important than the examineersquos recognition that (a) a range of sanctions is available to the court and (b) the appropriate sanction depends both on the culpability of the builder and the prejudice suffered by the customer]

22

CRIMINAL LAW AND PROCEDURE ANALYSIS (Criminal Law and Procedure IIA amp D VE amp F)

ANALYSIS

Legal Problems

(1) Did charging the defendant with both theft and burglary constitute double jeopardy

(2) Did the jury instruction violate the due process clause either by relieving the prosecution of the burden of proving the element of intent or by shifting the burden to the defendant to disprove that element

(3) Did the sentence imposed in this case for the theft conviction unconstitutionally deprive the defendant of his right to a jury trial on the issue of the value of the stolen item

DISCUSSION

Summary

The trial court properly denied the defendantrsquos pretrial motion to dismiss the charges on double jeopardy grounds The defendant may be charged with and convicted of both theft and burglary Each of the charges has an element that the other does not Neither charge is a lesser-included offense nor are they multiplicitous Thus charging both theft and burglary does not violate double jeopardy

The jury instruction on the burglary charge was constitutionally flawed It could have been reasonably understood by the jury as either (1) an irrebuttable conclusive presumption (which relieved the prosecution of proving the element of intent and removed the issue from the jury) or (2) a rebuttable mandatory presumption (which unconstitutionally shifted the burden of proof on an element of a charged offense from the prosecution to the defendant)

Because the four-year sentence imposed by the judge was based on the judgersquos finding by a preponderance of the evidence that the value of the stolen ring exceeded $5000 the sentence violates the defendantrsquos right to a jury determination beyond a reasonable doubt of the value of the ring

Point One (30) Charging the defendant with theft and burglary did not constitute double jeopardy

The Double Jeopardy Clause of the Fifth Amendment provides that a person shall not be twice put in jeopardy for the ldquosame offenserdquo Thus the question is whether the elements of the theft charge are wholly contained in the burglary charge or vice versa If the elements of the lesser charge (theft) are not wholly contained in the greater charge (burglary)mdashie if each charge requires proof of a fact that the other does notmdashthen convicting the defendant of both crimes would not violate double jeopardy even when the two offenses occurred at the same time and are thus arguably part of the ldquosame transactionrdquo Blockburger v United States 284 US 299 304 (1932) See also Albernaz v United States 450 US 333 344 n3 (1981) United States v Dixon 509 US 688 704 (1993)

23

Criminal Law and Procedure Analysis

Here theft and burglary each require proof of an element not required for the other crime Burglary may be defined differently in different jurisdictions However it almost invariably requires entry into a building or dwelling of another with the specific intent to commit a felony therein and the crime of burglary is complete upon the entry into the building or dwelling with such intent See eg Cannon v Oklahoma 827 P2d 1339 1342 (Okla Crim App 1992) In contrast theft which also may be defined differently in different states almost invariably requires the taking and carrying away of an item of personal property belonging to another with the intent to steal or permanently deprive the owner of possession

Here the ldquotakingrdquo or ldquostealingrdquo element is not contained in the definition of burglary and the ldquoentryrdquo element of burglary is not contained in the definition of theft Because theft is not a lesser-included offense of burglary and burglary is not a lesser-included offense of theft charging the defendant for both burglary and theft did not violate double jeopardy and the court properly denied the defense motion on those grounds Yparrea v Dorsey 64 F3d 577 579ndash80 (10th Cir 1995) citing Blockburger 284 US at 304

Finally the defendantrsquos motion to dismiss all the charges on double jeopardy grounds was improper because if both charges were for the same offense the motion should have requested dismissal of one charge not both

Point Two (35) The jury instruction on the burglary charge violated the Due Process Clause because it created either (1) an irrebuttable conclusive presumption (which relieved the prosecution of proving the element of intent and removed that issue from the jury) or (2) a rebuttable mandatory presumption (which unconstitutionally shifted the burden of proof on an element of a charged offense to the defendant)

The Supreme Court has interpreted the Due Process Clause of the US Constitution to require that the prosecution prove all elements of an offense beyond a reasonable doubt See In re Winship 397 US 358 364 (1970) The burden of proof cannot be shifted to the defendant by presuming an essential element upon proof of other elements of the offense because shifting the burden of persuasion with respect to any element of a criminal offense is contrary to the Due Process Clause See Mullaney v Wilbur 421 US 684 (1975)

The crime of burglary includes entry into a building or dwelling with the specific intent to commit a felony therein The requirement that the prosecutor prove beyond a reasonable doubt that the defendant had this specific intent distinguishes burglary from general-intent crimes like trespass See Sandstrom v Montana 442 US 510 523 (1979)

Here the jury was instructed that if ldquoafter consideration of all the evidence presented by the prosecution and defense you find beyond a reasonable doubt that the defendant entered the dwelling without the ownersrsquo consent you may presume that the defendant entered with the intent to commit a felony thereinrdquo This instruction was unconstitutional because it created either an irrebuttable conclusive presumption or a rebuttable mandatory presumption

A conclusive presumption is ldquoan irrebuttable direction by the court to find intent once convinced of the facts triggering the presumptionrdquo Id at 517 Here the jurors were instructed that once the prosecutor established that the defendant entered the neighborsrsquo house without consent they ldquomay presumerdquo that he intended to commit a felony therein The jurors may have reasonably concluded from this instruction that if they found that the defendant intended to enter his neighborsrsquo home without permission they must further find that he entered with the specific intent to commit a felony therein Because this instruction could operate as a conclusive

24

Criminal Law and Procedure Analysis

irrebuttable presumption by eliminating intent ldquoas an ingredient of the offenserdquo it violated due process by relieving the prosecution of the burden of proof for this element Id at 522

In the alternative the jury instruction could have been reasonably understood to create a rebuttable mandatory presumption which ldquotells [the jury] they must find the elemental fact upon proof of the basic fact at least unless the defendant has come forward with some evidence to rebut the presumed connection between the two factsrdquo County Court of Ulster County New York v Allen 442 US 140 157 (1979) The due process problem created by rebuttable mandatory presumptions is that ldquo[t]o the extent that the trier of fact is forced to abide by the presumption and may not reject it based on an independent evaluation of the particular facts presented by the State the analysis of the presumptionrsquos constitutional validity is logically divorced from those facts and based on the presumptionrsquos accuracy in the run of casesrdquo Id at 159

Unlike irrebuttable conclusive presumptions rebuttable mandatory presumptions are not always per se violations of the Due Process Clause However the Supreme Court of the United States has held that jury instructions that could reasonably be understood as shifting the burden of proof to the defendant on an element of the offense are unconstitutional Francis v Franklin 471 US 307 (1985) Here the argument that the jury instruction operated as a rebuttable mandatory presumption is supported by the fact that the judge also instructed the jury to ldquoconsider[ ] all the evidence presented by the prosecution and defenserdquo However even if the instruction created a rebuttable mandatory presumption it would be unconstitutional because it shifted the burden to the defense on an element of the offense Sandstrom 442 US at 524 Mullaney 421 US at 686

[NOTE Whether an examinee identifies the jury instruction as containing a ldquoconclusiverdquo or ldquomandatoryrdquo presumption is less important than the examineersquos analysis of the constitutional infirmities]

Point Three (35) The trial court violated the defendantrsquos Sixth Amendment right to a jury trial on an essential element of the offense when it found by a preponderance of the evidence that the ring was worth over $5000 and increased the defendantrsquos sentence based on this finding

In the statutory scheme under which the defendant was tried and convicted a Class D felony theft is defined as theft of item(s) with a value between $2500 and $10000 The jury found that the value of the diamond ring was at least $2500 and convicted the defendant of felony theft However at sentencing the trial court made a separate finding by a preponderance of the evidence that the value of the ring was greater than $5000 Following the statutersquos two-tiered sentencing scheme the judge then imposed on the defendant a sentence that was one year longer than the maximum that would otherwise have been allowed

The judgersquos sentence was unconstitutional because it violated the defendantrsquos Sixth Amendment right to a jury trial on this question The Supreme Court held in Apprendi v New Jersey 530 US 466 (2000) that ldquo[o]ther than the fact of a prior conviction any fact that increases the penalty for a crime beyond the prescribed statutory maximum must be submitted to a jury and proved beyond a reasonable doubtrdquo because ldquo[i]t is unconstitutional for a legislature to remove from the jury the assessment of facts that increase the prescribed range of penalties to which a criminal defendant is exposed [because] such facts must be established by proof beyond a reasonable doubtrdquo Id The Court reaffirmed Apprendi in Blakely v Washington 542 US 296 (2004) holding that the ldquolsquostatutory maximumrsquo for Apprendi purposes is the maximum sentence a judge may impose solely on the basis of the facts reflected in the jury verdict or admitted by the defendantrdquo Id at 303 (emphasis in original) In United States v Booker 543 US 220 (2005)

25

Criminal Law and Procedure Analysis

the Court relied on Blakely and Apprendi to conclude that protecting a defendantrsquos Sixth Amendment right to a jury trial required that ldquo[a]ny fact which is necessary to support a sentence exceeding the maximum authorized by the facts established by a plea of guilty or a jury verdict must be admitted by the defendant or proved to a jury beyond a reasonable doubtrdquo Id at 244

Thus in order to constitutionally increase a sentence above the statutory maximum of three years the jury must have found beyond a reasonable doubt that the value of the ring exceeded $5000 Here the court made the finding based on an appraisal proffered by the prosecutor only at sentencing and the judgersquos finding was by a preponderance of the evidence rather than beyond a reasonable doubt

26

AGENCY AND PARTNERSHIP ANALYSIS __________ (Agency and Partnership VA amp C VI)

ANALYSIS

Legal Problems

(1) Is a partner in a general partnership personally liable on a claim arising from misrepresentations by another partner made in the course of the partnership business

(2) Does a newly admitted partner in a general partnership become personally liable on existing claims against the partnership

(3) After the filing by a general partnership of a statement of qualification as a limited liability partnership are the partners personally liable as partners on (a) an existing claim against the general partnership and (b) a claim against the partnership that arose after the filing

DISCUSSION

Summary

Adam and Ben formed a general partnership under which they were jointly and severally liable for obligations of the partnership Thus Adam was personally liable for misrepresentations by Ben made in the ordinary course of the partnership business

Upon joining the general partnership Diane became personally liable for the obligations of the partnership arising after her admission but not for obligations pre-existing her admission such as the collectorrsquos claim

By filing a statement of qualification the three partners properly elected limited liability partnership status As partners in an LLP none of the three partners is personally liable as a partner for partnership obligations arising after the election such as the claim by the driverrsquos estate The election however does not change their personal liability on pre-existing claims that arose before the election such as the collectorrsquos claim

Point One (30) As a general partner of Empire a general partnership Adam became personally liable on the collectorrsquos claim a valid claim against the partnership that arose because of Benrsquos wrongful act in the ordinary course of the partnership business

When the collectorrsquos claim arose Empire was a general partnership composed of Adam and Ben Under UPA (1997) sect 306(a) partners of a general partnership are liable jointly and severally for all obligations of the partnership Under UPA (1997) sect 305(a) the partnership could become obligated for the loss caused to the collector as a result of the misrepresentation by Ben provided he was acting in the ordinary course of the partnership business Because there was no statement that limited his partnership authority Ben as partner was ldquoan agent of the partnership for the purpose of its businessrdquo See UPA (1997) sect 301(1) Benrsquos misrepresentation to the collector even if intentional appears to be in the ordinary course of the partnershiprsquos business of dealing

27

Agency and Partnership Analysis

in antique cars Thus Benrsquos wrongful act created a partnership obligation for which Adam was jointly and severally liable

[NOTE Generally a partnership creditor must ldquoexhaust the partnershiprsquos assets before levying on a judgment debtor partnerrsquos individual property where the partner is personally liable for the partnership obligationrdquo as a result of his status as a partner UPA (1997) sect 307 cmt 4 As the UPA comments explain this places Adam more in the position of guarantor than principal debtor on the partnership obligation Id cmt 4 Although an examinee might discuss this point the call focuses on whether Adam is personally liable not how the liability might be enforced]

Point Two (30) Because the collectorrsquos claim arose before Diane joined Empire Diane did not become personally liable on the claim

Diane was admitted to Empire when it was a general partnership and after the collectorrsquos claim arose While the general rule under UPA (1997) sect 306(a) is that the partners of a general partnership are liable jointly and severally for all obligations of the partnership there is a special rule for partners who are admitted during the duration of the partnership Under UPA (1997) sect 306(b) a person admitted to an existing partnership is not personally liable for any partnership obligations incurred before the personrsquos admission Because Diane was admitted to Empire after the collectorrsquos claim arose Diane is not personally liable on the claim

Dianersquos knowledge of the pre-existing claim and her stated concern about becoming liable on the collectorrsquos claim do not change her personal nonliability to the collector Although partners who have a liability shield can assume liability to third parties through private contractual guarantees or modifications to the partnership agreement Dianersquos stated concern constituted neither a guaranty to the collector nor ldquoan intentional waiver of liability protectionsrdquo See UPA (1997) sect 306 cmt 3 (describing methods for waiver of liability protections under sect 306(c) applicable in limited liability partnerships)

At most Diane will lose her investment in the partnership as a result of the collectorrsquos claim Although Diane did not become personally liable on the collectorrsquos claim when she joined the partnership the $250000 she contributed to the partnership is ldquoat risk for the satisfaction of existing partnership debtsrdquo UPA (1997) sect 306 cmt 2

Point Three (40) Filing the statement of qualification was effective to elect limited liability partnership status Despite this new status Adam and Ben remain personally liable on the collectorrsquos claim which arose before the election But as partners in an LLP neither Adam Ben nor Diane is personally liable as a partner on the driverrsquos estatersquos claim which arose after the election

Under UPA (1997) sect 1001 a general partnership can make an election and become a limited liability partnershipmdashif the partners approve the conversion by a vote equivalent to that necessary to amend the partnership agreement and the partnership then files a statement of qualification that specifies the name of the partnership its principal office and its election to be an LLP Here the partners agreed unanimouslymdashsufficient to amend their agreement under UPA (1997) sect 401(j)mdashand the statement of qualification was filed In addition the name of Empire LLP properly included an appropriate ending ldquoLLPrdquo See UPA (1997) sect 1002

Although another way to effectuate a ldquoconversionrdquo (as suggested by Benrsquos lawyer) is to form a new LLP and transfer the assets of the old general partnership to the new LLP the

28

Agency and Partnership Analysis

method used here (approval by the partners and the filing of a statement of qualification) is also sufficient to create LLP status

Thus Empire became Empire LLP as of the date of filing of the statement of qualification See UPA (1997) sect 1001 What effect did this have on the collectorrsquos claim which predated the filing According to UPA (1997) sect 306(c) an obligation incurred while a partnership is an LLP is solely a partnership obligation As the collectorrsquos claim predated the LLP Adam and Ben remain personally liable on the collectorrsquos claim Diane on the other hand was not personally liable on the collectorrsquos claim either before or after the filing of the statement of qualification See Point Two above

The driverrsquos estatersquos claim arose after Empire became Empire LLP Under UPA (1997) sect 306(c) an obligation incurred while a partnership is an LLP is solely a partnership obligationThus Adam Ben and Diane as partners are all protected from personal liability on the driverrsquos estatersquos claim But there may be personal liability if any of them was negligent or otherwise acted wrongfully by not informing the buyer of the bad suspension that caused the accident

29

National Conference of Bar Examiners 302 South Bedford Street | Madison WI 53703-3622 Phone 608-280-8550 | Fax 608-280-8552 | TDD 608-661-1275

wwwncbexorg e-mail contactncbexorg

  • Preface
  • Description of the MEE
  • Instructions
  • February 2014 Questions
    • Constitutinal Law Question
    • Trusts and Future Interests Question
    • Secured Transactions Question
    • Federal Civil Procedure Question
    • Criminal Law and Procedure Question
    • Agency and Partnership Question
      • February 2014 Analyses
        • Constitutional Law Analysis
        • Trust and Future Interests Analysis
        • Secured Transactions Analysis
        • Federal Civil Procedure Analysis
        • Criminal Law and Procedure Analysis
        • Agency and Partnership Analysis
            • ltlt13 ASCII85EncodePages false13 AllowTransparency false13 AutoPositionEPSFiles true13 AutoRotatePages None13 Binding Left13 CalGrayProfile (Dot Gain 20)13 CalRGBProfile (sRGB IEC61966-21)13 CalCMYKProfile (US Web Coated 050SWOP051 v2)13 sRGBProfile (sRGB IEC61966-21)13 CannotEmbedFontPolicy Error13 CompatibilityLevel 1413 CompressObjects Tags13 CompressPages true13 ConvertImagesToIndexed true13 PassThroughJPEGImages true13 CreateJobTicket false13 DefaultRenderingIntent Default13 DetectBlends true13 DetectCurves 0000013 ColorConversionStrategy CMYK13 DoThumbnails false13 EmbedAllFonts true13 EmbedOpenType false13 ParseICCProfilesInComments true13 EmbedJobOptions true13 DSCReportingLevel 013 EmitDSCWarnings false13 EndPage -113 ImageMemory 104857613 LockDistillerParams false13 MaxSubsetPct 10013 Optimize true13 OPM 113 ParseDSCComments true13 ParseDSCCommentsForDocInfo true13 PreserveCopyPage true13 PreserveDICMYKValues true13 PreserveEPSInfo true13 PreserveFlatness true13 PreserveHalftoneInfo false13 PreserveOPIComments true13 PreserveOverprintSettings true13 StartPage 113 SubsetFonts true13 TransferFunctionInfo Apply13 UCRandBGInfo Preserve13 UsePrologue false13 ColorSettingsFile ()13 AlwaysEmbed [ true13 ]13 NeverEmbed [ true13 ]13 AntiAliasColorImages false13 CropColorImages true13 ColorImageMinResolution 30013 ColorImageMinResolutionPolicy OK13 DownsampleColorImages true13 ColorImageDownsampleType Bicubic13 ColorImageResolution 30013 ColorImageDepth -113 ColorImageMinDownsampleDepth 113 ColorImageDownsampleThreshold 15000013 EncodeColorImages true13 ColorImageFilter DCTEncode13 AutoFilterColorImages true13 ColorImageAutoFilterStrategy JPEG13 ColorACSImageDict ltlt13 QFactor 01513 HSamples [1 1 1 1] VSamples [1 1 1 1]13 gtgt13 ColorImageDict ltlt13 QFactor 01513 HSamples [1 1 1 1] VSamples [1 1 1 1]13 gtgt13 JPEG2000ColorACSImageDict ltlt13 TileWidth 25613 TileHeight 25613 Quality 3013 gtgt13 JPEG2000ColorImageDict ltlt13 TileWidth 25613 TileHeight 25613 Quality 3013 gtgt13 AntiAliasGrayImages false13 CropGrayImages true13 GrayImageMinResolution 30013 GrayImageMinResolutionPolicy OK13 DownsampleGrayImages true13 GrayImageDownsampleType Bicubic13 GrayImageResolution 30013 GrayImageDepth -113 GrayImageMinDownsampleDepth 213 GrayImageDownsampleThreshold 15000013 EncodeGrayImages true13 GrayImageFilter DCTEncode13 AutoFilterGrayImages true13 GrayImageAutoFilterStrategy JPEG13 GrayACSImageDict ltlt13 QFactor 01513 HSamples [1 1 1 1] VSamples [1 1 1 1]13 gtgt13 GrayImageDict ltlt13 QFactor 01513 HSamples [1 1 1 1] VSamples [1 1 1 1]13 gtgt13 JPEG2000GrayACSImageDict ltlt13 TileWidth 25613 TileHeight 25613 Quality 3013 gtgt13 JPEG2000GrayImageDict ltlt13 TileWidth 25613 TileHeight 25613 Quality 3013 gtgt13 AntiAliasMonoImages false13 CropMonoImages true13 MonoImageMinResolution 120013 MonoImageMinResolutionPolicy OK13 DownsampleMonoImages true13 MonoImageDownsampleType Bicubic13 MonoImageResolution 120013 MonoImageDepth -113 MonoImageDownsampleThreshold 15000013 EncodeMonoImages true13 MonoImageFilter CCITTFaxEncode13 MonoImageDict ltlt13 K -113 gtgt13 AllowPSXObjects false13 CheckCompliance [13 None13 ]13 PDFX1aCheck false13 PDFX3Check false13 PDFXCompliantPDFOnly false13 PDFXNoTrimBoxError true13 PDFXTrimBoxToMediaBoxOffset [13 00000013 00000013 00000013 00000013 ]13 PDFXSetBleedBoxToMediaBox true13 PDFXBleedBoxToTrimBoxOffset [13 00000013 00000013 00000013 00000013 ]13 PDFXOutputIntentProfile ()13 PDFXOutputConditionIdentifier ()13 PDFXOutputCondition ()13 PDFXRegistryName ()13 PDFXTrapped False1313 CreateJDFFile false13 Description ltlt13 ARA 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 BGR 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 CHS ltFEFF4f7f75288fd94e9b8bbe5b9a521b5efa7684002000410064006f006200650020005000440046002065876863900275284e8e9ad88d2891cf76845370524d53705237300260a853ef4ee54f7f75280020004100630072006f0062006100740020548c002000410064006f00620065002000520065006100640065007200200035002e003000204ee553ca66f49ad87248672c676562535f00521b5efa768400200050004400460020658768633002gt13 CHT ltFEFF4f7f752890194e9b8a2d7f6e5efa7acb7684002000410064006f006200650020005000440046002065874ef69069752865bc9ad854c18cea76845370524d5370523786557406300260a853ef4ee54f7f75280020004100630072006f0062006100740020548c002000410064006f00620065002000520065006100640065007200200035002e003000204ee553ca66f49ad87248672c4f86958b555f5df25efa7acb76840020005000440046002065874ef63002gt13 CZE 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 DAN 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 DEU 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 ESP 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 ETI 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 FRA 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 GRE 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 HEB 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 HRV (Za stvaranje Adobe PDF dokumenata najpogodnijih za visokokvalitetni ispis prije tiskanja koristite ove postavke Stvoreni PDF dokumenti mogu se otvoriti Acrobat i Adobe Reader 50 i kasnijim verzijama)13 HUN 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 ITA ltFEFF005500740069006c0069007a007a006100720065002000710075006500730074006500200069006d0070006f007300740061007a0069006f006e00690020007000650072002000630072006500610072006500200064006f00630075006d0065006e00740069002000410064006f00620065002000500044004600200070006900f900200061006400610074007400690020006100200075006e00610020007000720065007300740061006d0070006100200064006900200061006c007400610020007100750061006c0069007400e0002e0020004900200064006f00630075006d0065006e007400690020005000440046002000630072006500610074006900200070006f00730073006f006e006f0020006500730073006500720065002000610070006500720074006900200063006f006e0020004100630072006f00620061007400200065002000410064006f00620065002000520065006100640065007200200035002e003000200065002000760065007200730069006f006e006900200073007500630063006500730073006900760065002egt13 JPN ltFEFF9ad854c18cea306a30d730ea30d730ec30b951fa529b7528002000410064006f0062006500200050004400460020658766f8306e4f5c6210306b4f7f75283057307e305930023053306e8a2d5b9a30674f5c62103055308c305f0020005000440046002030d530a130a430eb306f3001004100630072006f0062006100740020304a30883073002000410064006f00620065002000520065006100640065007200200035002e003000204ee5964d3067958b304f30533068304c3067304d307e305930023053306e8a2d5b9a306b306f30d530a930f330c8306e57cb30818fbc307f304c5fc59808306730593002gt13 KOR ltFEFFc7740020c124c815c7440020c0acc6a9d558c5ec0020ace0d488c9c80020c2dcd5d80020c778c1c4c5d00020ac00c7a50020c801d569d55c002000410064006f0062006500200050004400460020bb38c11cb97c0020c791c131d569b2c8b2e4002e0020c774b807ac8c0020c791c131b41c00200050004400460020bb38c11cb2940020004100630072006f0062006100740020bc0f002000410064006f00620065002000520065006100640065007200200035002e00300020c774c0c1c5d0c11c0020c5f40020c2180020c788c2b5b2c8b2e4002egt13 LTH 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 LVI 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 NLD (Gebruik deze instellingen om Adobe PDF-documenten te maken die zijn geoptimaliseerd voor prepress-afdrukken van hoge kwaliteit De gemaakte PDF-documenten kunnen worden geopend met Acrobat en Adobe Reader 50 en hoger)13 NOR 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 POL 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 PTB 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 RUM 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 RUS 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 SKY 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 SLV 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 SUO 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 SVE ltFEFF0041006e007600e4006e00640020006400650020006800e4007200200069006e0073007400e4006c006c006e0069006e006700610072006e00610020006f006d002000640075002000760069006c006c00200073006b006100700061002000410064006f006200650020005000440046002d0064006f006b0075006d0065006e007400200073006f006d002000e400720020006c00e4006d0070006c0069006700610020006600f60072002000700072006500700072006500730073002d007500740073006b00720069006600740020006d006500640020006800f600670020006b00760061006c0069007400650074002e002000200053006b006100700061006400650020005000440046002d0064006f006b0075006d0065006e00740020006b0061006e002000f600700070006e00610073002000690020004100630072006f0062006100740020006f00630068002000410064006f00620065002000520065006100640065007200200035002e00300020006f00630068002000730065006e006100720065002egt13 TUR 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 UKR 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 ENU (Use these settings to create Adobe PDF documents best suited for high-quality prepress printing Created PDF documents can be opened with Acrobat and Adobe Reader 50 and later)13 gtgt13 Namespace [13 (Adobe)13 (Common)13 (10)13 ]13 OtherNamespaces [13 ltlt13 AsReaderSpreads false13 CropImagesToFrames true13 ErrorControl WarnAndContinue13 FlattenerIgnoreSpreadOverrides false13 IncludeGuidesGrids false13 IncludeNonPrinting false13 IncludeSlug false13 Namespace [13 (Adobe)13 (InDesign)13 (40)13 ]13 OmitPlacedBitmaps false13 OmitPlacedEPS false13 OmitPlacedPDF false13 SimulateOverprint Legacy13 gtgt13 ltlt13 AddBleedMarks false13 AddColorBars false13 AddCropMarks false13 AddPageInfo false13 AddRegMarks false13 ConvertColors ConvertToCMYK13 DestinationProfileName ()13 DestinationProfileSelector DocumentCMYK13 Downsample16BitImages true13 FlattenerPreset ltlt13 PresetSelector MediumResolution13 gtgt13 FormElements false13 GenerateStructure false13 IncludeBookmarks false13 IncludeHyperlinks false13 IncludeInteractive false13 IncludeLayers false13 IncludeProfiles false13 MultimediaHandling UseObjectSettings13 Namespace [13 (Adobe)13 (CreativeSuite)13 (20)13 ]13 PDFXOutputIntentProfileSelector DocumentCMYK13 PreserveEditing true13 UntaggedCMYKHandling LeaveUntagged13 UntaggedRGBHandling UseDocumentProfile13 UseDocumentBleed false13 gtgt13 ]13gtgt setdistillerparams13ltlt13 HWResolution [2400 2400]13 PageSize [612000 792000]13gtgt setpagedevice13

Page 21: February 2014 MEE Questions and AnalysesPreface The Multistate Essay Examination (MEE) is developed by the National Conference of Bar Examiners (NCBE). This publication includes the

Secured Transactions Analysis

not a problem Even though the security agreement had not yet been signed the business had authorized the filing of the financing statement in an authenticated record UCC sect 9-509(a)(1) Moreover the financing statement may be filed before the security agreement is created UCC sect 9-502(d)

Point One(b) (10) The finance company also has a perfected security interest in the businessrsquos equipment

The finance companyrsquos security interest is enforceable and attached for the same reasons as the bankrsquos security interest The loan from the finance company to the business constitutes value the business has rights in the collateral and the business owner has authenticated a security agreement containing a description of the collateral The finance companyrsquos security interest is perfected because the finance company filed a financing statement with respect to it that provides that the business is the debtor and indicates that the collateral is equipment

Point One(c) (30) The bankrsquos security interest has priority over the finance companyrsquos security interest because the bankrsquos financing statement was filed first

As between two perfected security interests the general rule is that the security interest that was the earlier to be either perfected or the subject of a filed financing statement has priority UCC sect 9-322(a)(1) While the finance companyrsquos security interest was perfected before the bankrsquos (March 15 vs March 22) the bankrsquos financing statement was filed even earlier on March 2 Thus under the first-to-file-or-perfect rule of UCC sect 9-322(a)(1) the bankrsquos security interest has priority No exceptions to the general rule apply here

Point Two (35) A security interest in collateral continues notwithstanding its sale unless an exception applies Because the security interests of the bank and the finance company were perfected and the competitor was not a buyer in ordinary course of business no exception applies and the security interests of both creditors continue in the equipment sold to the competitor

As a general rule a security interest in collateral continues notwithstanding the fact that the debtor has sold the collateral to another person UCC sect 9-315(a)(1) Thus unless an exception applies the security interests of the bank and the finance company will continue in the item of equipment sold to the competitor

A buyer of goods will take free of an unperfected security interest in those goods See UCC sect 9-317(a)(2) However when the competitor bought the businessrsquos equipment both the bank and the finance company had perfected security interests in the equipment

A buyer can take free even of a perfected security interest in goods if the buyer is a ldquobuyer in ordinary course of businessrdquo See UCC sect 9-320(a) However the competitor was not a buyer in ordinary course of business To be a ldquobuyer in ordinary course of businessrdquo a buyer must buy goods from a seller that is in the business of selling goods of that kind See UCC sect 1-201(b)(9) The competitor bought this equipment from a seller that is not in the business of selling goods of this kind so the competitor was not a buyer in ordinary course of business with respect to these goods

Because no exception applies the security interests of the bank and the finance company continue even after the item of equipment was sold to the competitor

18

FEDERAL CIVIL PROCEDURE ANALYSIS (Federal Civil Procedure IVD)

ANALYSIS

Legal Problems

(1) Is a document prepared in the course of a contract dispute protected from discovery as ldquowork productrdquo when there is no evidence that the document was prepared in anticipation of litigation

(2)(a) Is a partyrsquos failure to provide relevant electronically stored information excused when the information was destroyed pursuant to a routine document retention scheme at a time when litigation was contemplated by the destroying party

(2)(b) What sanctions should be imposed on a party for allowing the destruction of evidence that is relevant to potential future litigation

DISCUSSION

Summary

The report prepared by the structural engineer is probably not work product and is thus discoverable The engineer examined the foundation of the house at the customerrsquos request and the engineerrsquos findings are potentially relevant to the customerrsquos claim that the foundation is defective The report was not prepared in anticipation of litigation The customer appears to have sought the engineerrsquos opinion in response to the builderrsquos offer to fix any problems with the foundation that an engineer might identify Because the report was not prepared in anticipation of litigation it is not protected by the work-product doctrine

The builder should have taken appropriate steps to preserve evidence including suspending its document retention program as soon as it began planning for litigationmdashie on July 10 Its destruction of potentially relevant material after that date was wrongful However a court is unlikely to impose severe sanctions on the builder because there are no facts indicating that the builder acted in bad faith and the customer can prove that the foundation is defective without the destroyed emails

Point One (40) The customer must turn over the engineerrsquos report because it was not prepared in anticipation of litigation

In general a party to a lawsuit in federal court ldquomay obtain discovery regarding any nonprivileged matter that is relevant to any partyrsquos claim or defenserdquo FED R CIV P 26(b)(1) (2009) This includes the right to inspect and copy documents in the other partyrsquos possession FED R CIV P 34(a)(1) Here the customer hired a structural engineer to examine the foundation of the house The engineerrsquos report on the foundation is likely to include information that would be relevant to the customerrsquos claim that the foundation was defectively constructed

The so-called ldquowork productrdquo rule allows a party to refuse to turn over ldquodocuments that are prepared in anticipation of litigation or for trialrdquo by that partyrsquos representative including

19

Federal Civil Procedure Analysis

a consultant Thus if the customer had hired the structural engineer to prepare a report ldquoin anticipation of litigationrdquo that report might not be discoverable See FED R CIV P 26(b)(3)

In this case however the customer hired the engineer to evaluate the foundation of the house as part of the customerrsquos negotiation with the builder concerning the housersquos flooding problem The builder told the customer that the housersquos landscaping was the reason for the flooding and the builder told the customer ldquoHave an engineer look at the foundation If therersquos a problem wersquoll fix itrdquo The customer appears to have acted in response to that statement There is no indication that the customer anticipated any kind of legal action at the time that the structural engineer was hired Accordingly the structural engineerrsquos report is discoverable and the court should order the customer to turn it over

[NOTE If an examinee concludes that the structural engineerrsquos report was prepared in anticipation of litigation then the examinee should also conclude that the report is not discoverable Documents prepared in anticipation of litigation do not need to be disclosed to an adverse party unless that party can demonstrate a ldquosubstantial needrdquo for the documents and an inability to obtain substantially equivalent information without ldquoundue hardshiprdquo FED R CIV P 26(b)(3)(A)(ii) Furthermore a report prepared by an expert who is not expected to testify is not discoverable in the absence of ldquoexceptional circumstancesrdquo making it ldquoimpracticablerdquo to obtain the information in another way FED R CIV P 26(b)(4)(D)(ii) The builder probably cannot make these showings here unless the engineerrsquos report deals with circumstances that have since changed There is no evidence that the structural engineer would have had access to any information or facts that the builder would not already know as a result of its construction and subsequent inspection of the house In addition if necessary the builder could ask the court for permission to arrange for a further inspection of the house by a structural engineer hired by the builder See FED R CIV P 34(a)(2) Accordingly if an examinee concludes that the report was prepared in anticipation of litigation the examinee should also conclude that the builder is not entitled to see the report]

Point Two(a) (30) Because the builder anticipated that it might be involved in litigation concerning its contract with the customer the builder acted wrongfully in destroying emails that were relevant to the housersquos construction even though the emails were destroyed pursuant to a routine document retention plan

As noted above a party to a lawsuit in federal court ldquomay obtain discovery regarding any nonprivileged matter that is relevant to any partyrsquos claim or defenserdquo FED R CIV P 26(b)(1) This includes emails and other electronically stored information FED R CIV P 34(a)(1)(A) Here the customer has requested all the builderrsquos emails pertaining to work done on the foundation of the house Ordinarily the builder would be obliged to turn over this information which is relevant to the customerrsquos defense that the housersquos foundation was poorly constructed

Unfortunately the emails in question no longer exist because the builder destroyed them on August 2

In general spoliation of evidence (destruction or alteration of evidence) is improper if the party who destroyed or altered the evidence ldquohas notice that the evidence is relevant to litigation or should have known that the evidence may be relevant to future litigationrdquo Fujitsu Ltd v Federal Express Corp 247 F3d 423 436 (2d Cir 2001) It is improper for a party to destroy electronic information relevant to pending litigation even if the destruction occurs before there is any request or order seeking the information See eg Leon v IDX Sys Corp 464 F3d 951 (9th Cir 2006) (plaintiffrsquos intentional destruction of computer files warranted dismissal even

20

In this case the builderrsquos destruction of the emails was pursuant to a routine document retention plan The Federal Rules provide expressly that in the absence of ldquoexceptional circumstancesrdquo parties should not be sanctioned for the loss of electronically stored information when the loss occurs pursuant to ldquoroutine good-faith operation of an electronic information systemrdquo FED R CIV P 37(e) However when a party anticipates litigation ldquoit must suspend its routine document retentiondestruction policy and put in place a lsquolitigation holdrsquo to ensure the preservation of relevant documentsrdquo Zubulake v UBS Warburg LLC 220 FRD 212 218 (SDNY 2003)

Federal Civil Procedure Analysis

though spoliation occurred before order compelling discovery) Similarly the duty to preserve evidence applies to a party who anticipates litigation even if litigation has not yet been commenced See THE SEDONA PRINCIPLES BEST PRACTICES RECOMMENDATIONS amp PRINCIPLES FOR ADDRESSING ELECTRONIC DOCUMENT PRODUCTION 70 cmt 14a (2d ed 2007)

The builder destroyed the emails on August 2 At that time the builder knew that litigation was a possibility because the builder had already directed its attorney to prepare a draft complaint for possible filing Knowing that litigation was a possibility the builder had a duty to take steps to preserve evidence including the emails in question See generally Fujitsu Ltd

Thus the builderrsquos destruction of potentially relevant emails at a time when it knew that litigation was a possibility was improper It had a duty to preserve evidence and it breached that duty

[NOTE Because courts have used different words to describe the test for when evidence must be preserved an examineersquos precise formulation of the test is not critical]

Point Two(b) (30) In determining appropriate sanctions for spoliation courts consider both the level of culpability of the spoliating party and the degree of prejudice the loss of evidence has caused the other party Here the builderrsquos destruction of evidence does not appear to have been willful nor is it likely to pose a significant obstacle to the customerrsquos defense Any sanctions imposed by the court should be modest

Federal courts have inherent power to control the litigation process and can sanction misbehavior including spoliation even when there has been no specific violation of the Federal Rules of Civil Procedure See generally Chambers v NASCO Inc 501 US 32 (1991) (discussing courtrsquos inherent power to control the litigation process) The range of available sanctions is broad It can include such sanctions as the payment of expenses incurred by the other party as a result of the destruction of the evidence an instruction to the jury authorizing it to draw an adverse inference from the destruction of the evidence a shifting of the burden of proof on the relevant issue or even judgment against the responsible party See eg Residential Funding Corp v DeGeorge Financial Corp 306 F3d 99 108 (2d Cir 2002) (adverse inference) Silvestri v General Motors Corp 271 F3d 583 593 (4th Cir 2001) (possibility of dismissal) Cf FED R CIV P 37(b)(2)(A) (listing remedies for failure to comply with discovery obligations)

In determining appropriate sanctions for spoliation courts consider both the level of culpability of the spoliating party and the degree of prejudice the loss of evidence has caused the other party Many courts impose severe sanctions (such as an adverse-inference instruction or the entry of judgment against the spoliating party) only when there is evidence of bad faith in the form of an intentional effort to hide information Eg Greyhound Lines Inc v Wade 485 F3d 1032 1035 (8th Cir 2007) (spoliation sanction requires intentional destruction out of desire ldquoto suppress the truthrdquo) However other courts have said that negligence in preserving evidence can

21

Federal Civil Procedure Analysis

support an adverse-inference instruction See Residential Funding 306 F3d at 108 (negligence enough under some circumstances)

Although a court might well order an evidentiary hearing on the issue of sanctions the facts presented do not seem appropriate for severe sanctions First the evidence was destroyed pursuant to the builderrsquos standard document retention plan and there is no evidence that the builder deliberately failed to suspend its usual procedures with the purpose of allowing the destruction of evidence Second the loss of this evidence will not severely hinder the customerrsquos presentation of his case The central issue is whether the foundation of the house was properly constructed If the construction job was poorly done the customer can present evidence derived from inspection of the premises to prove that point The customer can also depose witnesses about any issues that arose during construction

Under the circumstances a court is not likely to impose particularly severe sanctions although it might shift the burden to the builder to show that the foundation was properly constructed or it might require the builder to reimburse any expenses the customer incurs to discover and prove the facts about issues or disputes that arose during construction of the foundation

[NOTE The result reached by the examinee is less important than the examineersquos recognition that (a) a range of sanctions is available to the court and (b) the appropriate sanction depends both on the culpability of the builder and the prejudice suffered by the customer]

22

CRIMINAL LAW AND PROCEDURE ANALYSIS (Criminal Law and Procedure IIA amp D VE amp F)

ANALYSIS

Legal Problems

(1) Did charging the defendant with both theft and burglary constitute double jeopardy

(2) Did the jury instruction violate the due process clause either by relieving the prosecution of the burden of proving the element of intent or by shifting the burden to the defendant to disprove that element

(3) Did the sentence imposed in this case for the theft conviction unconstitutionally deprive the defendant of his right to a jury trial on the issue of the value of the stolen item

DISCUSSION

Summary

The trial court properly denied the defendantrsquos pretrial motion to dismiss the charges on double jeopardy grounds The defendant may be charged with and convicted of both theft and burglary Each of the charges has an element that the other does not Neither charge is a lesser-included offense nor are they multiplicitous Thus charging both theft and burglary does not violate double jeopardy

The jury instruction on the burglary charge was constitutionally flawed It could have been reasonably understood by the jury as either (1) an irrebuttable conclusive presumption (which relieved the prosecution of proving the element of intent and removed the issue from the jury) or (2) a rebuttable mandatory presumption (which unconstitutionally shifted the burden of proof on an element of a charged offense from the prosecution to the defendant)

Because the four-year sentence imposed by the judge was based on the judgersquos finding by a preponderance of the evidence that the value of the stolen ring exceeded $5000 the sentence violates the defendantrsquos right to a jury determination beyond a reasonable doubt of the value of the ring

Point One (30) Charging the defendant with theft and burglary did not constitute double jeopardy

The Double Jeopardy Clause of the Fifth Amendment provides that a person shall not be twice put in jeopardy for the ldquosame offenserdquo Thus the question is whether the elements of the theft charge are wholly contained in the burglary charge or vice versa If the elements of the lesser charge (theft) are not wholly contained in the greater charge (burglary)mdashie if each charge requires proof of a fact that the other does notmdashthen convicting the defendant of both crimes would not violate double jeopardy even when the two offenses occurred at the same time and are thus arguably part of the ldquosame transactionrdquo Blockburger v United States 284 US 299 304 (1932) See also Albernaz v United States 450 US 333 344 n3 (1981) United States v Dixon 509 US 688 704 (1993)

23

Criminal Law and Procedure Analysis

Here theft and burglary each require proof of an element not required for the other crime Burglary may be defined differently in different jurisdictions However it almost invariably requires entry into a building or dwelling of another with the specific intent to commit a felony therein and the crime of burglary is complete upon the entry into the building or dwelling with such intent See eg Cannon v Oklahoma 827 P2d 1339 1342 (Okla Crim App 1992) In contrast theft which also may be defined differently in different states almost invariably requires the taking and carrying away of an item of personal property belonging to another with the intent to steal or permanently deprive the owner of possession

Here the ldquotakingrdquo or ldquostealingrdquo element is not contained in the definition of burglary and the ldquoentryrdquo element of burglary is not contained in the definition of theft Because theft is not a lesser-included offense of burglary and burglary is not a lesser-included offense of theft charging the defendant for both burglary and theft did not violate double jeopardy and the court properly denied the defense motion on those grounds Yparrea v Dorsey 64 F3d 577 579ndash80 (10th Cir 1995) citing Blockburger 284 US at 304

Finally the defendantrsquos motion to dismiss all the charges on double jeopardy grounds was improper because if both charges were for the same offense the motion should have requested dismissal of one charge not both

Point Two (35) The jury instruction on the burglary charge violated the Due Process Clause because it created either (1) an irrebuttable conclusive presumption (which relieved the prosecution of proving the element of intent and removed that issue from the jury) or (2) a rebuttable mandatory presumption (which unconstitutionally shifted the burden of proof on an element of a charged offense to the defendant)

The Supreme Court has interpreted the Due Process Clause of the US Constitution to require that the prosecution prove all elements of an offense beyond a reasonable doubt See In re Winship 397 US 358 364 (1970) The burden of proof cannot be shifted to the defendant by presuming an essential element upon proof of other elements of the offense because shifting the burden of persuasion with respect to any element of a criminal offense is contrary to the Due Process Clause See Mullaney v Wilbur 421 US 684 (1975)

The crime of burglary includes entry into a building or dwelling with the specific intent to commit a felony therein The requirement that the prosecutor prove beyond a reasonable doubt that the defendant had this specific intent distinguishes burglary from general-intent crimes like trespass See Sandstrom v Montana 442 US 510 523 (1979)

Here the jury was instructed that if ldquoafter consideration of all the evidence presented by the prosecution and defense you find beyond a reasonable doubt that the defendant entered the dwelling without the ownersrsquo consent you may presume that the defendant entered with the intent to commit a felony thereinrdquo This instruction was unconstitutional because it created either an irrebuttable conclusive presumption or a rebuttable mandatory presumption

A conclusive presumption is ldquoan irrebuttable direction by the court to find intent once convinced of the facts triggering the presumptionrdquo Id at 517 Here the jurors were instructed that once the prosecutor established that the defendant entered the neighborsrsquo house without consent they ldquomay presumerdquo that he intended to commit a felony therein The jurors may have reasonably concluded from this instruction that if they found that the defendant intended to enter his neighborsrsquo home without permission they must further find that he entered with the specific intent to commit a felony therein Because this instruction could operate as a conclusive

24

Criminal Law and Procedure Analysis

irrebuttable presumption by eliminating intent ldquoas an ingredient of the offenserdquo it violated due process by relieving the prosecution of the burden of proof for this element Id at 522

In the alternative the jury instruction could have been reasonably understood to create a rebuttable mandatory presumption which ldquotells [the jury] they must find the elemental fact upon proof of the basic fact at least unless the defendant has come forward with some evidence to rebut the presumed connection between the two factsrdquo County Court of Ulster County New York v Allen 442 US 140 157 (1979) The due process problem created by rebuttable mandatory presumptions is that ldquo[t]o the extent that the trier of fact is forced to abide by the presumption and may not reject it based on an independent evaluation of the particular facts presented by the State the analysis of the presumptionrsquos constitutional validity is logically divorced from those facts and based on the presumptionrsquos accuracy in the run of casesrdquo Id at 159

Unlike irrebuttable conclusive presumptions rebuttable mandatory presumptions are not always per se violations of the Due Process Clause However the Supreme Court of the United States has held that jury instructions that could reasonably be understood as shifting the burden of proof to the defendant on an element of the offense are unconstitutional Francis v Franklin 471 US 307 (1985) Here the argument that the jury instruction operated as a rebuttable mandatory presumption is supported by the fact that the judge also instructed the jury to ldquoconsider[ ] all the evidence presented by the prosecution and defenserdquo However even if the instruction created a rebuttable mandatory presumption it would be unconstitutional because it shifted the burden to the defense on an element of the offense Sandstrom 442 US at 524 Mullaney 421 US at 686

[NOTE Whether an examinee identifies the jury instruction as containing a ldquoconclusiverdquo or ldquomandatoryrdquo presumption is less important than the examineersquos analysis of the constitutional infirmities]

Point Three (35) The trial court violated the defendantrsquos Sixth Amendment right to a jury trial on an essential element of the offense when it found by a preponderance of the evidence that the ring was worth over $5000 and increased the defendantrsquos sentence based on this finding

In the statutory scheme under which the defendant was tried and convicted a Class D felony theft is defined as theft of item(s) with a value between $2500 and $10000 The jury found that the value of the diamond ring was at least $2500 and convicted the defendant of felony theft However at sentencing the trial court made a separate finding by a preponderance of the evidence that the value of the ring was greater than $5000 Following the statutersquos two-tiered sentencing scheme the judge then imposed on the defendant a sentence that was one year longer than the maximum that would otherwise have been allowed

The judgersquos sentence was unconstitutional because it violated the defendantrsquos Sixth Amendment right to a jury trial on this question The Supreme Court held in Apprendi v New Jersey 530 US 466 (2000) that ldquo[o]ther than the fact of a prior conviction any fact that increases the penalty for a crime beyond the prescribed statutory maximum must be submitted to a jury and proved beyond a reasonable doubtrdquo because ldquo[i]t is unconstitutional for a legislature to remove from the jury the assessment of facts that increase the prescribed range of penalties to which a criminal defendant is exposed [because] such facts must be established by proof beyond a reasonable doubtrdquo Id The Court reaffirmed Apprendi in Blakely v Washington 542 US 296 (2004) holding that the ldquolsquostatutory maximumrsquo for Apprendi purposes is the maximum sentence a judge may impose solely on the basis of the facts reflected in the jury verdict or admitted by the defendantrdquo Id at 303 (emphasis in original) In United States v Booker 543 US 220 (2005)

25

Criminal Law and Procedure Analysis

the Court relied on Blakely and Apprendi to conclude that protecting a defendantrsquos Sixth Amendment right to a jury trial required that ldquo[a]ny fact which is necessary to support a sentence exceeding the maximum authorized by the facts established by a plea of guilty or a jury verdict must be admitted by the defendant or proved to a jury beyond a reasonable doubtrdquo Id at 244

Thus in order to constitutionally increase a sentence above the statutory maximum of three years the jury must have found beyond a reasonable doubt that the value of the ring exceeded $5000 Here the court made the finding based on an appraisal proffered by the prosecutor only at sentencing and the judgersquos finding was by a preponderance of the evidence rather than beyond a reasonable doubt

26

AGENCY AND PARTNERSHIP ANALYSIS __________ (Agency and Partnership VA amp C VI)

ANALYSIS

Legal Problems

(1) Is a partner in a general partnership personally liable on a claim arising from misrepresentations by another partner made in the course of the partnership business

(2) Does a newly admitted partner in a general partnership become personally liable on existing claims against the partnership

(3) After the filing by a general partnership of a statement of qualification as a limited liability partnership are the partners personally liable as partners on (a) an existing claim against the general partnership and (b) a claim against the partnership that arose after the filing

DISCUSSION

Summary

Adam and Ben formed a general partnership under which they were jointly and severally liable for obligations of the partnership Thus Adam was personally liable for misrepresentations by Ben made in the ordinary course of the partnership business

Upon joining the general partnership Diane became personally liable for the obligations of the partnership arising after her admission but not for obligations pre-existing her admission such as the collectorrsquos claim

By filing a statement of qualification the three partners properly elected limited liability partnership status As partners in an LLP none of the three partners is personally liable as a partner for partnership obligations arising after the election such as the claim by the driverrsquos estate The election however does not change their personal liability on pre-existing claims that arose before the election such as the collectorrsquos claim

Point One (30) As a general partner of Empire a general partnership Adam became personally liable on the collectorrsquos claim a valid claim against the partnership that arose because of Benrsquos wrongful act in the ordinary course of the partnership business

When the collectorrsquos claim arose Empire was a general partnership composed of Adam and Ben Under UPA (1997) sect 306(a) partners of a general partnership are liable jointly and severally for all obligations of the partnership Under UPA (1997) sect 305(a) the partnership could become obligated for the loss caused to the collector as a result of the misrepresentation by Ben provided he was acting in the ordinary course of the partnership business Because there was no statement that limited his partnership authority Ben as partner was ldquoan agent of the partnership for the purpose of its businessrdquo See UPA (1997) sect 301(1) Benrsquos misrepresentation to the collector even if intentional appears to be in the ordinary course of the partnershiprsquos business of dealing

27

Agency and Partnership Analysis

in antique cars Thus Benrsquos wrongful act created a partnership obligation for which Adam was jointly and severally liable

[NOTE Generally a partnership creditor must ldquoexhaust the partnershiprsquos assets before levying on a judgment debtor partnerrsquos individual property where the partner is personally liable for the partnership obligationrdquo as a result of his status as a partner UPA (1997) sect 307 cmt 4 As the UPA comments explain this places Adam more in the position of guarantor than principal debtor on the partnership obligation Id cmt 4 Although an examinee might discuss this point the call focuses on whether Adam is personally liable not how the liability might be enforced]

Point Two (30) Because the collectorrsquos claim arose before Diane joined Empire Diane did not become personally liable on the claim

Diane was admitted to Empire when it was a general partnership and after the collectorrsquos claim arose While the general rule under UPA (1997) sect 306(a) is that the partners of a general partnership are liable jointly and severally for all obligations of the partnership there is a special rule for partners who are admitted during the duration of the partnership Under UPA (1997) sect 306(b) a person admitted to an existing partnership is not personally liable for any partnership obligations incurred before the personrsquos admission Because Diane was admitted to Empire after the collectorrsquos claim arose Diane is not personally liable on the claim

Dianersquos knowledge of the pre-existing claim and her stated concern about becoming liable on the collectorrsquos claim do not change her personal nonliability to the collector Although partners who have a liability shield can assume liability to third parties through private contractual guarantees or modifications to the partnership agreement Dianersquos stated concern constituted neither a guaranty to the collector nor ldquoan intentional waiver of liability protectionsrdquo See UPA (1997) sect 306 cmt 3 (describing methods for waiver of liability protections under sect 306(c) applicable in limited liability partnerships)

At most Diane will lose her investment in the partnership as a result of the collectorrsquos claim Although Diane did not become personally liable on the collectorrsquos claim when she joined the partnership the $250000 she contributed to the partnership is ldquoat risk for the satisfaction of existing partnership debtsrdquo UPA (1997) sect 306 cmt 2

Point Three (40) Filing the statement of qualification was effective to elect limited liability partnership status Despite this new status Adam and Ben remain personally liable on the collectorrsquos claim which arose before the election But as partners in an LLP neither Adam Ben nor Diane is personally liable as a partner on the driverrsquos estatersquos claim which arose after the election

Under UPA (1997) sect 1001 a general partnership can make an election and become a limited liability partnershipmdashif the partners approve the conversion by a vote equivalent to that necessary to amend the partnership agreement and the partnership then files a statement of qualification that specifies the name of the partnership its principal office and its election to be an LLP Here the partners agreed unanimouslymdashsufficient to amend their agreement under UPA (1997) sect 401(j)mdashand the statement of qualification was filed In addition the name of Empire LLP properly included an appropriate ending ldquoLLPrdquo See UPA (1997) sect 1002

Although another way to effectuate a ldquoconversionrdquo (as suggested by Benrsquos lawyer) is to form a new LLP and transfer the assets of the old general partnership to the new LLP the

28

Agency and Partnership Analysis

method used here (approval by the partners and the filing of a statement of qualification) is also sufficient to create LLP status

Thus Empire became Empire LLP as of the date of filing of the statement of qualification See UPA (1997) sect 1001 What effect did this have on the collectorrsquos claim which predated the filing According to UPA (1997) sect 306(c) an obligation incurred while a partnership is an LLP is solely a partnership obligation As the collectorrsquos claim predated the LLP Adam and Ben remain personally liable on the collectorrsquos claim Diane on the other hand was not personally liable on the collectorrsquos claim either before or after the filing of the statement of qualification See Point Two above

The driverrsquos estatersquos claim arose after Empire became Empire LLP Under UPA (1997) sect 306(c) an obligation incurred while a partnership is an LLP is solely a partnership obligationThus Adam Ben and Diane as partners are all protected from personal liability on the driverrsquos estatersquos claim But there may be personal liability if any of them was negligent or otherwise acted wrongfully by not informing the buyer of the bad suspension that caused the accident

29

National Conference of Bar Examiners 302 South Bedford Street | Madison WI 53703-3622 Phone 608-280-8550 | Fax 608-280-8552 | TDD 608-661-1275

wwwncbexorg e-mail contactncbexorg

  • Preface
  • Description of the MEE
  • Instructions
  • February 2014 Questions
    • Constitutinal Law Question
    • Trusts and Future Interests Question
    • Secured Transactions Question
    • Federal Civil Procedure Question
    • Criminal Law and Procedure Question
    • Agency and Partnership Question
      • February 2014 Analyses
        • Constitutional Law Analysis
        • Trust and Future Interests Analysis
        • Secured Transactions Analysis
        • Federal Civil Procedure Analysis
        • Criminal Law and Procedure Analysis
        • Agency and Partnership Analysis
            • ltlt13 ASCII85EncodePages false13 AllowTransparency false13 AutoPositionEPSFiles true13 AutoRotatePages None13 Binding Left13 CalGrayProfile (Dot Gain 20)13 CalRGBProfile (sRGB IEC61966-21)13 CalCMYKProfile (US Web Coated 050SWOP051 v2)13 sRGBProfile (sRGB IEC61966-21)13 CannotEmbedFontPolicy Error13 CompatibilityLevel 1413 CompressObjects Tags13 CompressPages true13 ConvertImagesToIndexed true13 PassThroughJPEGImages true13 CreateJobTicket false13 DefaultRenderingIntent Default13 DetectBlends true13 DetectCurves 0000013 ColorConversionStrategy CMYK13 DoThumbnails false13 EmbedAllFonts true13 EmbedOpenType false13 ParseICCProfilesInComments true13 EmbedJobOptions true13 DSCReportingLevel 013 EmitDSCWarnings false13 EndPage -113 ImageMemory 104857613 LockDistillerParams false13 MaxSubsetPct 10013 Optimize true13 OPM 113 ParseDSCComments true13 ParseDSCCommentsForDocInfo true13 PreserveCopyPage true13 PreserveDICMYKValues true13 PreserveEPSInfo true13 PreserveFlatness true13 PreserveHalftoneInfo false13 PreserveOPIComments true13 PreserveOverprintSettings true13 StartPage 113 SubsetFonts true13 TransferFunctionInfo Apply13 UCRandBGInfo Preserve13 UsePrologue false13 ColorSettingsFile ()13 AlwaysEmbed [ true13 ]13 NeverEmbed [ true13 ]13 AntiAliasColorImages false13 CropColorImages true13 ColorImageMinResolution 30013 ColorImageMinResolutionPolicy OK13 DownsampleColorImages true13 ColorImageDownsampleType Bicubic13 ColorImageResolution 30013 ColorImageDepth -113 ColorImageMinDownsampleDepth 113 ColorImageDownsampleThreshold 15000013 EncodeColorImages true13 ColorImageFilter DCTEncode13 AutoFilterColorImages true13 ColorImageAutoFilterStrategy JPEG13 ColorACSImageDict ltlt13 QFactor 01513 HSamples [1 1 1 1] VSamples [1 1 1 1]13 gtgt13 ColorImageDict ltlt13 QFactor 01513 HSamples [1 1 1 1] VSamples [1 1 1 1]13 gtgt13 JPEG2000ColorACSImageDict ltlt13 TileWidth 25613 TileHeight 25613 Quality 3013 gtgt13 JPEG2000ColorImageDict ltlt13 TileWidth 25613 TileHeight 25613 Quality 3013 gtgt13 AntiAliasGrayImages false13 CropGrayImages true13 GrayImageMinResolution 30013 GrayImageMinResolutionPolicy OK13 DownsampleGrayImages true13 GrayImageDownsampleType Bicubic13 GrayImageResolution 30013 GrayImageDepth -113 GrayImageMinDownsampleDepth 213 GrayImageDownsampleThreshold 15000013 EncodeGrayImages true13 GrayImageFilter DCTEncode13 AutoFilterGrayImages true13 GrayImageAutoFilterStrategy JPEG13 GrayACSImageDict ltlt13 QFactor 01513 HSamples [1 1 1 1] VSamples [1 1 1 1]13 gtgt13 GrayImageDict ltlt13 QFactor 01513 HSamples [1 1 1 1] VSamples [1 1 1 1]13 gtgt13 JPEG2000GrayACSImageDict ltlt13 TileWidth 25613 TileHeight 25613 Quality 3013 gtgt13 JPEG2000GrayImageDict ltlt13 TileWidth 25613 TileHeight 25613 Quality 3013 gtgt13 AntiAliasMonoImages false13 CropMonoImages true13 MonoImageMinResolution 120013 MonoImageMinResolutionPolicy OK13 DownsampleMonoImages true13 MonoImageDownsampleType Bicubic13 MonoImageResolution 120013 MonoImageDepth -113 MonoImageDownsampleThreshold 15000013 EncodeMonoImages true13 MonoImageFilter CCITTFaxEncode13 MonoImageDict ltlt13 K -113 gtgt13 AllowPSXObjects false13 CheckCompliance [13 None13 ]13 PDFX1aCheck false13 PDFX3Check false13 PDFXCompliantPDFOnly false13 PDFXNoTrimBoxError true13 PDFXTrimBoxToMediaBoxOffset [13 00000013 00000013 00000013 00000013 ]13 PDFXSetBleedBoxToMediaBox true13 PDFXBleedBoxToTrimBoxOffset [13 00000013 00000013 00000013 00000013 ]13 PDFXOutputIntentProfile ()13 PDFXOutputConditionIdentifier ()13 PDFXOutputCondition ()13 PDFXRegistryName ()13 PDFXTrapped False1313 CreateJDFFile false13 Description ltlt13 ARA 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 BGR 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 CHS ltFEFF4f7f75288fd94e9b8bbe5b9a521b5efa7684002000410064006f006200650020005000440046002065876863900275284e8e9ad88d2891cf76845370524d53705237300260a853ef4ee54f7f75280020004100630072006f0062006100740020548c002000410064006f00620065002000520065006100640065007200200035002e003000204ee553ca66f49ad87248672c676562535f00521b5efa768400200050004400460020658768633002gt13 CHT ltFEFF4f7f752890194e9b8a2d7f6e5efa7acb7684002000410064006f006200650020005000440046002065874ef69069752865bc9ad854c18cea76845370524d5370523786557406300260a853ef4ee54f7f75280020004100630072006f0062006100740020548c002000410064006f00620065002000520065006100640065007200200035002e003000204ee553ca66f49ad87248672c4f86958b555f5df25efa7acb76840020005000440046002065874ef63002gt13 CZE 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 DAN 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 DEU ltFEFF00560065007200770065006e00640065006e0020005300690065002000640069006500730065002000450069006e007300740065006c006c0075006e00670065006e0020007a0075006d002000450072007300740065006c006c0065006e00200076006f006e002000410064006f006200650020005000440046002d0044006f006b0075006d0065006e00740065006e002c00200076006f006e002000640065006e0065006e002000530069006500200068006f006300680077006500720074006900670065002000500072006500700072006500730073002d0044007200750063006b0065002000650072007a0065007500670065006e0020006d00f60063006800740065006e002e002000450072007300740065006c006c007400650020005000440046002d0044006f006b0075006d0065006e007400650020006b00f6006e006e0065006e0020006d006900740020004100630072006f00620061007400200075006e0064002000410064006f00620065002000520065006100640065007200200035002e00300020006f0064006500720020006800f600680065007200200067006500f600660066006e00650074002000770065007200640065006e002egt13 ESP ltFEFF005500740069006c0069006300650020006500730074006100200063006f006e0066006900670075007200610063006900f3006e0020007000610072006100200063007200650061007200200064006f00630075006d0065006e0074006f00730020005000440046002000640065002000410064006f0062006500200061006400650063007500610064006f00730020007000610072006100200069006d0070007200650073006900f3006e0020007000720065002d0065006400690074006f007200690061006c00200064006500200061006c00740061002000630061006c0069006400610064002e002000530065002000700075006500640065006e00200061006200720069007200200064006f00630075006d0065006e0074006f00730020005000440046002000630072006500610064006f007300200063006f006e0020004100630072006f006200610074002c002000410064006f00620065002000520065006100640065007200200035002e003000200079002000760065007200730069006f006e0065007300200070006f00730074006500720069006f007200650073002egt13 ETI 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 FRA 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 GRE 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 HEB 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 HRV (Za stvaranje Adobe PDF dokumenata najpogodnijih za visokokvalitetni ispis prije tiskanja koristite ove postavke Stvoreni PDF dokumenti mogu se otvoriti Acrobat i Adobe Reader 50 i kasnijim verzijama)13 HUN 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 ITA 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 JPN ltFEFF9ad854c18cea306a30d730ea30d730ec30b951fa529b7528002000410064006f0062006500200050004400460020658766f8306e4f5c6210306b4f7f75283057307e305930023053306e8a2d5b9a30674f5c62103055308c305f0020005000440046002030d530a130a430eb306f3001004100630072006f0062006100740020304a30883073002000410064006f00620065002000520065006100640065007200200035002e003000204ee5964d3067958b304f30533068304c3067304d307e305930023053306e8a2d5b9a306b306f30d530a930f330c8306e57cb30818fbc307f304c5fc59808306730593002gt13 KOR ltFEFFc7740020c124c815c7440020c0acc6a9d558c5ec0020ace0d488c9c80020c2dcd5d80020c778c1c4c5d00020ac00c7a50020c801d569d55c002000410064006f0062006500200050004400460020bb38c11cb97c0020c791c131d569b2c8b2e4002e0020c774b807ac8c0020c791c131b41c00200050004400460020bb38c11cb2940020004100630072006f0062006100740020bc0f002000410064006f00620065002000520065006100640065007200200035002e00300020c774c0c1c5d0c11c0020c5f40020c2180020c788c2b5b2c8b2e4002egt13 LTH 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 LVI 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 NLD (Gebruik deze instellingen om Adobe PDF-documenten te maken die zijn geoptimaliseerd voor prepress-afdrukken van hoge kwaliteit De gemaakte PDF-documenten kunnen worden geopend met Acrobat en Adobe Reader 50 en hoger)13 NOR ltFEFF004200720075006b00200064006900730073006500200069006e006e007300740069006c006c0069006e00670065006e0065002000740069006c002000e50020006f0070007000720065007400740065002000410064006f006200650020005000440046002d0064006f006b0075006d0065006e00740065007200200073006f006d00200065007200200062006500730074002000650067006e0065007400200066006f00720020006600f80072007400720079006b006b0073007500740073006b00720069006600740020006100760020006800f800790020006b00760061006c0069007400650074002e0020005000440046002d0064006f006b0075006d0065006e00740065006e00650020006b0061006e002000e50070006e00650073002000690020004100630072006f00620061007400200065006c006c00650072002000410064006f00620065002000520065006100640065007200200035002e003000200065006c006c00650072002000730065006e006500720065002egt13 POL 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 PTB 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 RUM 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 RUS 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 SKY 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 SLV ltFEFF005400650020006e006100730074006100760069007400760065002000750070006f0072006100620069007400650020007a00610020007500730074007600610072006a0061006e006a006500200064006f006b0075006d0065006e0074006f0076002000410064006f006200650020005000440046002c0020006b006900200073006f0020006e0061006a007000720069006d00650072006e0065006a016100690020007a00610020006b0061006b006f0076006f00730074006e006f0020007400690073006b0061006e006a00650020007300200070007200690070007200610076006f0020006e00610020007400690073006b002e00200020005500730074007600610072006a0065006e006500200064006f006b0075006d0065006e0074006500200050004400460020006a00650020006d006f0067006f010d00650020006f0064007000720065007400690020007a0020004100630072006f00620061007400200069006e002000410064006f00620065002000520065006100640065007200200035002e003000200069006e0020006e006f00760065006a01610069006d002egt13 SUO 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 SVE ltFEFF0041006e007600e4006e00640020006400650020006800e4007200200069006e0073007400e4006c006c006e0069006e006700610072006e00610020006f006d002000640075002000760069006c006c00200073006b006100700061002000410064006f006200650020005000440046002d0064006f006b0075006d0065006e007400200073006f006d002000e400720020006c00e4006d0070006c0069006700610020006600f60072002000700072006500700072006500730073002d007500740073006b00720069006600740020006d006500640020006800f600670020006b00760061006c0069007400650074002e002000200053006b006100700061006400650020005000440046002d0064006f006b0075006d0065006e00740020006b0061006e002000f600700070006e00610073002000690020004100630072006f0062006100740020006f00630068002000410064006f00620065002000520065006100640065007200200035002e00300020006f00630068002000730065006e006100720065002egt13 TUR 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 UKR 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 ENU (Use these settings to create Adobe PDF documents best suited for high-quality prepress printing Created PDF documents can be opened with Acrobat and Adobe Reader 50 and later)13 gtgt13 Namespace [13 (Adobe)13 (Common)13 (10)13 ]13 OtherNamespaces [13 ltlt13 AsReaderSpreads false13 CropImagesToFrames true13 ErrorControl WarnAndContinue13 FlattenerIgnoreSpreadOverrides false13 IncludeGuidesGrids false13 IncludeNonPrinting false13 IncludeSlug false13 Namespace [13 (Adobe)13 (InDesign)13 (40)13 ]13 OmitPlacedBitmaps false13 OmitPlacedEPS false13 OmitPlacedPDF false13 SimulateOverprint Legacy13 gtgt13 ltlt13 AddBleedMarks false13 AddColorBars false13 AddCropMarks false13 AddPageInfo false13 AddRegMarks false13 ConvertColors ConvertToCMYK13 DestinationProfileName ()13 DestinationProfileSelector DocumentCMYK13 Downsample16BitImages true13 FlattenerPreset ltlt13 PresetSelector MediumResolution13 gtgt13 FormElements false13 GenerateStructure false13 IncludeBookmarks false13 IncludeHyperlinks false13 IncludeInteractive false13 IncludeLayers false13 IncludeProfiles false13 MultimediaHandling UseObjectSettings13 Namespace [13 (Adobe)13 (CreativeSuite)13 (20)13 ]13 PDFXOutputIntentProfileSelector DocumentCMYK13 PreserveEditing true13 UntaggedCMYKHandling LeaveUntagged13 UntaggedRGBHandling UseDocumentProfile13 UseDocumentBleed false13 gtgt13 ]13gtgt setdistillerparams13ltlt13 HWResolution [2400 2400]13 PageSize [612000 792000]13gtgt setpagedevice13

Page 22: February 2014 MEE Questions and AnalysesPreface The Multistate Essay Examination (MEE) is developed by the National Conference of Bar Examiners (NCBE). This publication includes the

FEDERAL CIVIL PROCEDURE ANALYSIS (Federal Civil Procedure IVD)

ANALYSIS

Legal Problems

(1) Is a document prepared in the course of a contract dispute protected from discovery as ldquowork productrdquo when there is no evidence that the document was prepared in anticipation of litigation

(2)(a) Is a partyrsquos failure to provide relevant electronically stored information excused when the information was destroyed pursuant to a routine document retention scheme at a time when litigation was contemplated by the destroying party

(2)(b) What sanctions should be imposed on a party for allowing the destruction of evidence that is relevant to potential future litigation

DISCUSSION

Summary

The report prepared by the structural engineer is probably not work product and is thus discoverable The engineer examined the foundation of the house at the customerrsquos request and the engineerrsquos findings are potentially relevant to the customerrsquos claim that the foundation is defective The report was not prepared in anticipation of litigation The customer appears to have sought the engineerrsquos opinion in response to the builderrsquos offer to fix any problems with the foundation that an engineer might identify Because the report was not prepared in anticipation of litigation it is not protected by the work-product doctrine

The builder should have taken appropriate steps to preserve evidence including suspending its document retention program as soon as it began planning for litigationmdashie on July 10 Its destruction of potentially relevant material after that date was wrongful However a court is unlikely to impose severe sanctions on the builder because there are no facts indicating that the builder acted in bad faith and the customer can prove that the foundation is defective without the destroyed emails

Point One (40) The customer must turn over the engineerrsquos report because it was not prepared in anticipation of litigation

In general a party to a lawsuit in federal court ldquomay obtain discovery regarding any nonprivileged matter that is relevant to any partyrsquos claim or defenserdquo FED R CIV P 26(b)(1) (2009) This includes the right to inspect and copy documents in the other partyrsquos possession FED R CIV P 34(a)(1) Here the customer hired a structural engineer to examine the foundation of the house The engineerrsquos report on the foundation is likely to include information that would be relevant to the customerrsquos claim that the foundation was defectively constructed

The so-called ldquowork productrdquo rule allows a party to refuse to turn over ldquodocuments that are prepared in anticipation of litigation or for trialrdquo by that partyrsquos representative including

19

Federal Civil Procedure Analysis

a consultant Thus if the customer had hired the structural engineer to prepare a report ldquoin anticipation of litigationrdquo that report might not be discoverable See FED R CIV P 26(b)(3)

In this case however the customer hired the engineer to evaluate the foundation of the house as part of the customerrsquos negotiation with the builder concerning the housersquos flooding problem The builder told the customer that the housersquos landscaping was the reason for the flooding and the builder told the customer ldquoHave an engineer look at the foundation If therersquos a problem wersquoll fix itrdquo The customer appears to have acted in response to that statement There is no indication that the customer anticipated any kind of legal action at the time that the structural engineer was hired Accordingly the structural engineerrsquos report is discoverable and the court should order the customer to turn it over

[NOTE If an examinee concludes that the structural engineerrsquos report was prepared in anticipation of litigation then the examinee should also conclude that the report is not discoverable Documents prepared in anticipation of litigation do not need to be disclosed to an adverse party unless that party can demonstrate a ldquosubstantial needrdquo for the documents and an inability to obtain substantially equivalent information without ldquoundue hardshiprdquo FED R CIV P 26(b)(3)(A)(ii) Furthermore a report prepared by an expert who is not expected to testify is not discoverable in the absence of ldquoexceptional circumstancesrdquo making it ldquoimpracticablerdquo to obtain the information in another way FED R CIV P 26(b)(4)(D)(ii) The builder probably cannot make these showings here unless the engineerrsquos report deals with circumstances that have since changed There is no evidence that the structural engineer would have had access to any information or facts that the builder would not already know as a result of its construction and subsequent inspection of the house In addition if necessary the builder could ask the court for permission to arrange for a further inspection of the house by a structural engineer hired by the builder See FED R CIV P 34(a)(2) Accordingly if an examinee concludes that the report was prepared in anticipation of litigation the examinee should also conclude that the builder is not entitled to see the report]

Point Two(a) (30) Because the builder anticipated that it might be involved in litigation concerning its contract with the customer the builder acted wrongfully in destroying emails that were relevant to the housersquos construction even though the emails were destroyed pursuant to a routine document retention plan

As noted above a party to a lawsuit in federal court ldquomay obtain discovery regarding any nonprivileged matter that is relevant to any partyrsquos claim or defenserdquo FED R CIV P 26(b)(1) This includes emails and other electronically stored information FED R CIV P 34(a)(1)(A) Here the customer has requested all the builderrsquos emails pertaining to work done on the foundation of the house Ordinarily the builder would be obliged to turn over this information which is relevant to the customerrsquos defense that the housersquos foundation was poorly constructed

Unfortunately the emails in question no longer exist because the builder destroyed them on August 2

In general spoliation of evidence (destruction or alteration of evidence) is improper if the party who destroyed or altered the evidence ldquohas notice that the evidence is relevant to litigation or should have known that the evidence may be relevant to future litigationrdquo Fujitsu Ltd v Federal Express Corp 247 F3d 423 436 (2d Cir 2001) It is improper for a party to destroy electronic information relevant to pending litigation even if the destruction occurs before there is any request or order seeking the information See eg Leon v IDX Sys Corp 464 F3d 951 (9th Cir 2006) (plaintiffrsquos intentional destruction of computer files warranted dismissal even

20

In this case the builderrsquos destruction of the emails was pursuant to a routine document retention plan The Federal Rules provide expressly that in the absence of ldquoexceptional circumstancesrdquo parties should not be sanctioned for the loss of electronically stored information when the loss occurs pursuant to ldquoroutine good-faith operation of an electronic information systemrdquo FED R CIV P 37(e) However when a party anticipates litigation ldquoit must suspend its routine document retentiondestruction policy and put in place a lsquolitigation holdrsquo to ensure the preservation of relevant documentsrdquo Zubulake v UBS Warburg LLC 220 FRD 212 218 (SDNY 2003)

Federal Civil Procedure Analysis

though spoliation occurred before order compelling discovery) Similarly the duty to preserve evidence applies to a party who anticipates litigation even if litigation has not yet been commenced See THE SEDONA PRINCIPLES BEST PRACTICES RECOMMENDATIONS amp PRINCIPLES FOR ADDRESSING ELECTRONIC DOCUMENT PRODUCTION 70 cmt 14a (2d ed 2007)

The builder destroyed the emails on August 2 At that time the builder knew that litigation was a possibility because the builder had already directed its attorney to prepare a draft complaint for possible filing Knowing that litigation was a possibility the builder had a duty to take steps to preserve evidence including the emails in question See generally Fujitsu Ltd

Thus the builderrsquos destruction of potentially relevant emails at a time when it knew that litigation was a possibility was improper It had a duty to preserve evidence and it breached that duty

[NOTE Because courts have used different words to describe the test for when evidence must be preserved an examineersquos precise formulation of the test is not critical]

Point Two(b) (30) In determining appropriate sanctions for spoliation courts consider both the level of culpability of the spoliating party and the degree of prejudice the loss of evidence has caused the other party Here the builderrsquos destruction of evidence does not appear to have been willful nor is it likely to pose a significant obstacle to the customerrsquos defense Any sanctions imposed by the court should be modest

Federal courts have inherent power to control the litigation process and can sanction misbehavior including spoliation even when there has been no specific violation of the Federal Rules of Civil Procedure See generally Chambers v NASCO Inc 501 US 32 (1991) (discussing courtrsquos inherent power to control the litigation process) The range of available sanctions is broad It can include such sanctions as the payment of expenses incurred by the other party as a result of the destruction of the evidence an instruction to the jury authorizing it to draw an adverse inference from the destruction of the evidence a shifting of the burden of proof on the relevant issue or even judgment against the responsible party See eg Residential Funding Corp v DeGeorge Financial Corp 306 F3d 99 108 (2d Cir 2002) (adverse inference) Silvestri v General Motors Corp 271 F3d 583 593 (4th Cir 2001) (possibility of dismissal) Cf FED R CIV P 37(b)(2)(A) (listing remedies for failure to comply with discovery obligations)

In determining appropriate sanctions for spoliation courts consider both the level of culpability of the spoliating party and the degree of prejudice the loss of evidence has caused the other party Many courts impose severe sanctions (such as an adverse-inference instruction or the entry of judgment against the spoliating party) only when there is evidence of bad faith in the form of an intentional effort to hide information Eg Greyhound Lines Inc v Wade 485 F3d 1032 1035 (8th Cir 2007) (spoliation sanction requires intentional destruction out of desire ldquoto suppress the truthrdquo) However other courts have said that negligence in preserving evidence can

21

Federal Civil Procedure Analysis

support an adverse-inference instruction See Residential Funding 306 F3d at 108 (negligence enough under some circumstances)

Although a court might well order an evidentiary hearing on the issue of sanctions the facts presented do not seem appropriate for severe sanctions First the evidence was destroyed pursuant to the builderrsquos standard document retention plan and there is no evidence that the builder deliberately failed to suspend its usual procedures with the purpose of allowing the destruction of evidence Second the loss of this evidence will not severely hinder the customerrsquos presentation of his case The central issue is whether the foundation of the house was properly constructed If the construction job was poorly done the customer can present evidence derived from inspection of the premises to prove that point The customer can also depose witnesses about any issues that arose during construction

Under the circumstances a court is not likely to impose particularly severe sanctions although it might shift the burden to the builder to show that the foundation was properly constructed or it might require the builder to reimburse any expenses the customer incurs to discover and prove the facts about issues or disputes that arose during construction of the foundation

[NOTE The result reached by the examinee is less important than the examineersquos recognition that (a) a range of sanctions is available to the court and (b) the appropriate sanction depends both on the culpability of the builder and the prejudice suffered by the customer]

22

CRIMINAL LAW AND PROCEDURE ANALYSIS (Criminal Law and Procedure IIA amp D VE amp F)

ANALYSIS

Legal Problems

(1) Did charging the defendant with both theft and burglary constitute double jeopardy

(2) Did the jury instruction violate the due process clause either by relieving the prosecution of the burden of proving the element of intent or by shifting the burden to the defendant to disprove that element

(3) Did the sentence imposed in this case for the theft conviction unconstitutionally deprive the defendant of his right to a jury trial on the issue of the value of the stolen item

DISCUSSION

Summary

The trial court properly denied the defendantrsquos pretrial motion to dismiss the charges on double jeopardy grounds The defendant may be charged with and convicted of both theft and burglary Each of the charges has an element that the other does not Neither charge is a lesser-included offense nor are they multiplicitous Thus charging both theft and burglary does not violate double jeopardy

The jury instruction on the burglary charge was constitutionally flawed It could have been reasonably understood by the jury as either (1) an irrebuttable conclusive presumption (which relieved the prosecution of proving the element of intent and removed the issue from the jury) or (2) a rebuttable mandatory presumption (which unconstitutionally shifted the burden of proof on an element of a charged offense from the prosecution to the defendant)

Because the four-year sentence imposed by the judge was based on the judgersquos finding by a preponderance of the evidence that the value of the stolen ring exceeded $5000 the sentence violates the defendantrsquos right to a jury determination beyond a reasonable doubt of the value of the ring

Point One (30) Charging the defendant with theft and burglary did not constitute double jeopardy

The Double Jeopardy Clause of the Fifth Amendment provides that a person shall not be twice put in jeopardy for the ldquosame offenserdquo Thus the question is whether the elements of the theft charge are wholly contained in the burglary charge or vice versa If the elements of the lesser charge (theft) are not wholly contained in the greater charge (burglary)mdashie if each charge requires proof of a fact that the other does notmdashthen convicting the defendant of both crimes would not violate double jeopardy even when the two offenses occurred at the same time and are thus arguably part of the ldquosame transactionrdquo Blockburger v United States 284 US 299 304 (1932) See also Albernaz v United States 450 US 333 344 n3 (1981) United States v Dixon 509 US 688 704 (1993)

23

Criminal Law and Procedure Analysis

Here theft and burglary each require proof of an element not required for the other crime Burglary may be defined differently in different jurisdictions However it almost invariably requires entry into a building or dwelling of another with the specific intent to commit a felony therein and the crime of burglary is complete upon the entry into the building or dwelling with such intent See eg Cannon v Oklahoma 827 P2d 1339 1342 (Okla Crim App 1992) In contrast theft which also may be defined differently in different states almost invariably requires the taking and carrying away of an item of personal property belonging to another with the intent to steal or permanently deprive the owner of possession

Here the ldquotakingrdquo or ldquostealingrdquo element is not contained in the definition of burglary and the ldquoentryrdquo element of burglary is not contained in the definition of theft Because theft is not a lesser-included offense of burglary and burglary is not a lesser-included offense of theft charging the defendant for both burglary and theft did not violate double jeopardy and the court properly denied the defense motion on those grounds Yparrea v Dorsey 64 F3d 577 579ndash80 (10th Cir 1995) citing Blockburger 284 US at 304

Finally the defendantrsquos motion to dismiss all the charges on double jeopardy grounds was improper because if both charges were for the same offense the motion should have requested dismissal of one charge not both

Point Two (35) The jury instruction on the burglary charge violated the Due Process Clause because it created either (1) an irrebuttable conclusive presumption (which relieved the prosecution of proving the element of intent and removed that issue from the jury) or (2) a rebuttable mandatory presumption (which unconstitutionally shifted the burden of proof on an element of a charged offense to the defendant)

The Supreme Court has interpreted the Due Process Clause of the US Constitution to require that the prosecution prove all elements of an offense beyond a reasonable doubt See In re Winship 397 US 358 364 (1970) The burden of proof cannot be shifted to the defendant by presuming an essential element upon proof of other elements of the offense because shifting the burden of persuasion with respect to any element of a criminal offense is contrary to the Due Process Clause See Mullaney v Wilbur 421 US 684 (1975)

The crime of burglary includes entry into a building or dwelling with the specific intent to commit a felony therein The requirement that the prosecutor prove beyond a reasonable doubt that the defendant had this specific intent distinguishes burglary from general-intent crimes like trespass See Sandstrom v Montana 442 US 510 523 (1979)

Here the jury was instructed that if ldquoafter consideration of all the evidence presented by the prosecution and defense you find beyond a reasonable doubt that the defendant entered the dwelling without the ownersrsquo consent you may presume that the defendant entered with the intent to commit a felony thereinrdquo This instruction was unconstitutional because it created either an irrebuttable conclusive presumption or a rebuttable mandatory presumption

A conclusive presumption is ldquoan irrebuttable direction by the court to find intent once convinced of the facts triggering the presumptionrdquo Id at 517 Here the jurors were instructed that once the prosecutor established that the defendant entered the neighborsrsquo house without consent they ldquomay presumerdquo that he intended to commit a felony therein The jurors may have reasonably concluded from this instruction that if they found that the defendant intended to enter his neighborsrsquo home without permission they must further find that he entered with the specific intent to commit a felony therein Because this instruction could operate as a conclusive

24

Criminal Law and Procedure Analysis

irrebuttable presumption by eliminating intent ldquoas an ingredient of the offenserdquo it violated due process by relieving the prosecution of the burden of proof for this element Id at 522

In the alternative the jury instruction could have been reasonably understood to create a rebuttable mandatory presumption which ldquotells [the jury] they must find the elemental fact upon proof of the basic fact at least unless the defendant has come forward with some evidence to rebut the presumed connection between the two factsrdquo County Court of Ulster County New York v Allen 442 US 140 157 (1979) The due process problem created by rebuttable mandatory presumptions is that ldquo[t]o the extent that the trier of fact is forced to abide by the presumption and may not reject it based on an independent evaluation of the particular facts presented by the State the analysis of the presumptionrsquos constitutional validity is logically divorced from those facts and based on the presumptionrsquos accuracy in the run of casesrdquo Id at 159

Unlike irrebuttable conclusive presumptions rebuttable mandatory presumptions are not always per se violations of the Due Process Clause However the Supreme Court of the United States has held that jury instructions that could reasonably be understood as shifting the burden of proof to the defendant on an element of the offense are unconstitutional Francis v Franklin 471 US 307 (1985) Here the argument that the jury instruction operated as a rebuttable mandatory presumption is supported by the fact that the judge also instructed the jury to ldquoconsider[ ] all the evidence presented by the prosecution and defenserdquo However even if the instruction created a rebuttable mandatory presumption it would be unconstitutional because it shifted the burden to the defense on an element of the offense Sandstrom 442 US at 524 Mullaney 421 US at 686

[NOTE Whether an examinee identifies the jury instruction as containing a ldquoconclusiverdquo or ldquomandatoryrdquo presumption is less important than the examineersquos analysis of the constitutional infirmities]

Point Three (35) The trial court violated the defendantrsquos Sixth Amendment right to a jury trial on an essential element of the offense when it found by a preponderance of the evidence that the ring was worth over $5000 and increased the defendantrsquos sentence based on this finding

In the statutory scheme under which the defendant was tried and convicted a Class D felony theft is defined as theft of item(s) with a value between $2500 and $10000 The jury found that the value of the diamond ring was at least $2500 and convicted the defendant of felony theft However at sentencing the trial court made a separate finding by a preponderance of the evidence that the value of the ring was greater than $5000 Following the statutersquos two-tiered sentencing scheme the judge then imposed on the defendant a sentence that was one year longer than the maximum that would otherwise have been allowed

The judgersquos sentence was unconstitutional because it violated the defendantrsquos Sixth Amendment right to a jury trial on this question The Supreme Court held in Apprendi v New Jersey 530 US 466 (2000) that ldquo[o]ther than the fact of a prior conviction any fact that increases the penalty for a crime beyond the prescribed statutory maximum must be submitted to a jury and proved beyond a reasonable doubtrdquo because ldquo[i]t is unconstitutional for a legislature to remove from the jury the assessment of facts that increase the prescribed range of penalties to which a criminal defendant is exposed [because] such facts must be established by proof beyond a reasonable doubtrdquo Id The Court reaffirmed Apprendi in Blakely v Washington 542 US 296 (2004) holding that the ldquolsquostatutory maximumrsquo for Apprendi purposes is the maximum sentence a judge may impose solely on the basis of the facts reflected in the jury verdict or admitted by the defendantrdquo Id at 303 (emphasis in original) In United States v Booker 543 US 220 (2005)

25

Criminal Law and Procedure Analysis

the Court relied on Blakely and Apprendi to conclude that protecting a defendantrsquos Sixth Amendment right to a jury trial required that ldquo[a]ny fact which is necessary to support a sentence exceeding the maximum authorized by the facts established by a plea of guilty or a jury verdict must be admitted by the defendant or proved to a jury beyond a reasonable doubtrdquo Id at 244

Thus in order to constitutionally increase a sentence above the statutory maximum of three years the jury must have found beyond a reasonable doubt that the value of the ring exceeded $5000 Here the court made the finding based on an appraisal proffered by the prosecutor only at sentencing and the judgersquos finding was by a preponderance of the evidence rather than beyond a reasonable doubt

26

AGENCY AND PARTNERSHIP ANALYSIS __________ (Agency and Partnership VA amp C VI)

ANALYSIS

Legal Problems

(1) Is a partner in a general partnership personally liable on a claim arising from misrepresentations by another partner made in the course of the partnership business

(2) Does a newly admitted partner in a general partnership become personally liable on existing claims against the partnership

(3) After the filing by a general partnership of a statement of qualification as a limited liability partnership are the partners personally liable as partners on (a) an existing claim against the general partnership and (b) a claim against the partnership that arose after the filing

DISCUSSION

Summary

Adam and Ben formed a general partnership under which they were jointly and severally liable for obligations of the partnership Thus Adam was personally liable for misrepresentations by Ben made in the ordinary course of the partnership business

Upon joining the general partnership Diane became personally liable for the obligations of the partnership arising after her admission but not for obligations pre-existing her admission such as the collectorrsquos claim

By filing a statement of qualification the three partners properly elected limited liability partnership status As partners in an LLP none of the three partners is personally liable as a partner for partnership obligations arising after the election such as the claim by the driverrsquos estate The election however does not change their personal liability on pre-existing claims that arose before the election such as the collectorrsquos claim

Point One (30) As a general partner of Empire a general partnership Adam became personally liable on the collectorrsquos claim a valid claim against the partnership that arose because of Benrsquos wrongful act in the ordinary course of the partnership business

When the collectorrsquos claim arose Empire was a general partnership composed of Adam and Ben Under UPA (1997) sect 306(a) partners of a general partnership are liable jointly and severally for all obligations of the partnership Under UPA (1997) sect 305(a) the partnership could become obligated for the loss caused to the collector as a result of the misrepresentation by Ben provided he was acting in the ordinary course of the partnership business Because there was no statement that limited his partnership authority Ben as partner was ldquoan agent of the partnership for the purpose of its businessrdquo See UPA (1997) sect 301(1) Benrsquos misrepresentation to the collector even if intentional appears to be in the ordinary course of the partnershiprsquos business of dealing

27

Agency and Partnership Analysis

in antique cars Thus Benrsquos wrongful act created a partnership obligation for which Adam was jointly and severally liable

[NOTE Generally a partnership creditor must ldquoexhaust the partnershiprsquos assets before levying on a judgment debtor partnerrsquos individual property where the partner is personally liable for the partnership obligationrdquo as a result of his status as a partner UPA (1997) sect 307 cmt 4 As the UPA comments explain this places Adam more in the position of guarantor than principal debtor on the partnership obligation Id cmt 4 Although an examinee might discuss this point the call focuses on whether Adam is personally liable not how the liability might be enforced]

Point Two (30) Because the collectorrsquos claim arose before Diane joined Empire Diane did not become personally liable on the claim

Diane was admitted to Empire when it was a general partnership and after the collectorrsquos claim arose While the general rule under UPA (1997) sect 306(a) is that the partners of a general partnership are liable jointly and severally for all obligations of the partnership there is a special rule for partners who are admitted during the duration of the partnership Under UPA (1997) sect 306(b) a person admitted to an existing partnership is not personally liable for any partnership obligations incurred before the personrsquos admission Because Diane was admitted to Empire after the collectorrsquos claim arose Diane is not personally liable on the claim

Dianersquos knowledge of the pre-existing claim and her stated concern about becoming liable on the collectorrsquos claim do not change her personal nonliability to the collector Although partners who have a liability shield can assume liability to third parties through private contractual guarantees or modifications to the partnership agreement Dianersquos stated concern constituted neither a guaranty to the collector nor ldquoan intentional waiver of liability protectionsrdquo See UPA (1997) sect 306 cmt 3 (describing methods for waiver of liability protections under sect 306(c) applicable in limited liability partnerships)

At most Diane will lose her investment in the partnership as a result of the collectorrsquos claim Although Diane did not become personally liable on the collectorrsquos claim when she joined the partnership the $250000 she contributed to the partnership is ldquoat risk for the satisfaction of existing partnership debtsrdquo UPA (1997) sect 306 cmt 2

Point Three (40) Filing the statement of qualification was effective to elect limited liability partnership status Despite this new status Adam and Ben remain personally liable on the collectorrsquos claim which arose before the election But as partners in an LLP neither Adam Ben nor Diane is personally liable as a partner on the driverrsquos estatersquos claim which arose after the election

Under UPA (1997) sect 1001 a general partnership can make an election and become a limited liability partnershipmdashif the partners approve the conversion by a vote equivalent to that necessary to amend the partnership agreement and the partnership then files a statement of qualification that specifies the name of the partnership its principal office and its election to be an LLP Here the partners agreed unanimouslymdashsufficient to amend their agreement under UPA (1997) sect 401(j)mdashand the statement of qualification was filed In addition the name of Empire LLP properly included an appropriate ending ldquoLLPrdquo See UPA (1997) sect 1002

Although another way to effectuate a ldquoconversionrdquo (as suggested by Benrsquos lawyer) is to form a new LLP and transfer the assets of the old general partnership to the new LLP the

28

Agency and Partnership Analysis

method used here (approval by the partners and the filing of a statement of qualification) is also sufficient to create LLP status

Thus Empire became Empire LLP as of the date of filing of the statement of qualification See UPA (1997) sect 1001 What effect did this have on the collectorrsquos claim which predated the filing According to UPA (1997) sect 306(c) an obligation incurred while a partnership is an LLP is solely a partnership obligation As the collectorrsquos claim predated the LLP Adam and Ben remain personally liable on the collectorrsquos claim Diane on the other hand was not personally liable on the collectorrsquos claim either before or after the filing of the statement of qualification See Point Two above

The driverrsquos estatersquos claim arose after Empire became Empire LLP Under UPA (1997) sect 306(c) an obligation incurred while a partnership is an LLP is solely a partnership obligationThus Adam Ben and Diane as partners are all protected from personal liability on the driverrsquos estatersquos claim But there may be personal liability if any of them was negligent or otherwise acted wrongfully by not informing the buyer of the bad suspension that caused the accident

29

National Conference of Bar Examiners 302 South Bedford Street | Madison WI 53703-3622 Phone 608-280-8550 | Fax 608-280-8552 | TDD 608-661-1275

wwwncbexorg e-mail contactncbexorg

  • Preface
  • Description of the MEE
  • Instructions
  • February 2014 Questions
    • Constitutinal Law Question
    • Trusts and Future Interests Question
    • Secured Transactions Question
    • Federal Civil Procedure Question
    • Criminal Law and Procedure Question
    • Agency and Partnership Question
      • February 2014 Analyses
        • Constitutional Law Analysis
        • Trust and Future Interests Analysis
        • Secured Transactions Analysis
        • Federal Civil Procedure Analysis
        • Criminal Law and Procedure Analysis
        • Agency and Partnership Analysis
            • ltlt13 ASCII85EncodePages false13 AllowTransparency false13 AutoPositionEPSFiles true13 AutoRotatePages None13 Binding Left13 CalGrayProfile (Dot Gain 20)13 CalRGBProfile (sRGB IEC61966-21)13 CalCMYKProfile (US Web Coated 050SWOP051 v2)13 sRGBProfile (sRGB IEC61966-21)13 CannotEmbedFontPolicy Error13 CompatibilityLevel 1413 CompressObjects Tags13 CompressPages true13 ConvertImagesToIndexed true13 PassThroughJPEGImages true13 CreateJobTicket false13 DefaultRenderingIntent Default13 DetectBlends true13 DetectCurves 0000013 ColorConversionStrategy CMYK13 DoThumbnails false13 EmbedAllFonts true13 EmbedOpenType false13 ParseICCProfilesInComments true13 EmbedJobOptions true13 DSCReportingLevel 013 EmitDSCWarnings false13 EndPage -113 ImageMemory 104857613 LockDistillerParams false13 MaxSubsetPct 10013 Optimize true13 OPM 113 ParseDSCComments true13 ParseDSCCommentsForDocInfo true13 PreserveCopyPage true13 PreserveDICMYKValues true13 PreserveEPSInfo true13 PreserveFlatness true13 PreserveHalftoneInfo false13 PreserveOPIComments true13 PreserveOverprintSettings true13 StartPage 113 SubsetFonts true13 TransferFunctionInfo Apply13 UCRandBGInfo Preserve13 UsePrologue false13 ColorSettingsFile ()13 AlwaysEmbed [ true13 ]13 NeverEmbed [ true13 ]13 AntiAliasColorImages false13 CropColorImages true13 ColorImageMinResolution 30013 ColorImageMinResolutionPolicy OK13 DownsampleColorImages true13 ColorImageDownsampleType Bicubic13 ColorImageResolution 30013 ColorImageDepth -113 ColorImageMinDownsampleDepth 113 ColorImageDownsampleThreshold 15000013 EncodeColorImages true13 ColorImageFilter DCTEncode13 AutoFilterColorImages true13 ColorImageAutoFilterStrategy JPEG13 ColorACSImageDict ltlt13 QFactor 01513 HSamples [1 1 1 1] VSamples [1 1 1 1]13 gtgt13 ColorImageDict ltlt13 QFactor 01513 HSamples [1 1 1 1] VSamples [1 1 1 1]13 gtgt13 JPEG2000ColorACSImageDict ltlt13 TileWidth 25613 TileHeight 25613 Quality 3013 gtgt13 JPEG2000ColorImageDict ltlt13 TileWidth 25613 TileHeight 25613 Quality 3013 gtgt13 AntiAliasGrayImages false13 CropGrayImages true13 GrayImageMinResolution 30013 GrayImageMinResolutionPolicy OK13 DownsampleGrayImages true13 GrayImageDownsampleType Bicubic13 GrayImageResolution 30013 GrayImageDepth -113 GrayImageMinDownsampleDepth 213 GrayImageDownsampleThreshold 15000013 EncodeGrayImages true13 GrayImageFilter DCTEncode13 AutoFilterGrayImages true13 GrayImageAutoFilterStrategy JPEG13 GrayACSImageDict ltlt13 QFactor 01513 HSamples [1 1 1 1] VSamples [1 1 1 1]13 gtgt13 GrayImageDict ltlt13 QFactor 01513 HSamples [1 1 1 1] VSamples [1 1 1 1]13 gtgt13 JPEG2000GrayACSImageDict ltlt13 TileWidth 25613 TileHeight 25613 Quality 3013 gtgt13 JPEG2000GrayImageDict ltlt13 TileWidth 25613 TileHeight 25613 Quality 3013 gtgt13 AntiAliasMonoImages false13 CropMonoImages true13 MonoImageMinResolution 120013 MonoImageMinResolutionPolicy OK13 DownsampleMonoImages true13 MonoImageDownsampleType Bicubic13 MonoImageResolution 120013 MonoImageDepth -113 MonoImageDownsampleThreshold 15000013 EncodeMonoImages true13 MonoImageFilter CCITTFaxEncode13 MonoImageDict ltlt13 K -113 gtgt13 AllowPSXObjects false13 CheckCompliance [13 None13 ]13 PDFX1aCheck false13 PDFX3Check false13 PDFXCompliantPDFOnly false13 PDFXNoTrimBoxError true13 PDFXTrimBoxToMediaBoxOffset [13 00000013 00000013 00000013 00000013 ]13 PDFXSetBleedBoxToMediaBox true13 PDFXBleedBoxToTrimBoxOffset [13 00000013 00000013 00000013 00000013 ]13 PDFXOutputIntentProfile ()13 PDFXOutputConditionIdentifier ()13 PDFXOutputCondition ()13 PDFXRegistryName ()13 PDFXTrapped False1313 CreateJDFFile false13 Description ltlt13 ARA 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 BGR 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 CHS ltFEFF4f7f75288fd94e9b8bbe5b9a521b5efa7684002000410064006f006200650020005000440046002065876863900275284e8e9ad88d2891cf76845370524d53705237300260a853ef4ee54f7f75280020004100630072006f0062006100740020548c002000410064006f00620065002000520065006100640065007200200035002e003000204ee553ca66f49ad87248672c676562535f00521b5efa768400200050004400460020658768633002gt13 CHT ltFEFF4f7f752890194e9b8a2d7f6e5efa7acb7684002000410064006f006200650020005000440046002065874ef69069752865bc9ad854c18cea76845370524d5370523786557406300260a853ef4ee54f7f75280020004100630072006f0062006100740020548c002000410064006f00620065002000520065006100640065007200200035002e003000204ee553ca66f49ad87248672c4f86958b555f5df25efa7acb76840020005000440046002065874ef63002gt13 CZE 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 DAN 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 DEU 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 ESP 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 ETI 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 FRA 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 GRE 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 HEB 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 HRV (Za stvaranje Adobe PDF dokumenata najpogodnijih za visokokvalitetni ispis prije tiskanja koristite ove postavke Stvoreni PDF dokumenti mogu se otvoriti Acrobat i Adobe Reader 50 i kasnijim verzijama)13 HUN 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 ITA 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 JPN ltFEFF9ad854c18cea306a30d730ea30d730ec30b951fa529b7528002000410064006f0062006500200050004400460020658766f8306e4f5c6210306b4f7f75283057307e305930023053306e8a2d5b9a30674f5c62103055308c305f0020005000440046002030d530a130a430eb306f3001004100630072006f0062006100740020304a30883073002000410064006f00620065002000520065006100640065007200200035002e003000204ee5964d3067958b304f30533068304c3067304d307e305930023053306e8a2d5b9a306b306f30d530a930f330c8306e57cb30818fbc307f304c5fc59808306730593002gt13 KOR ltFEFFc7740020c124c815c7440020c0acc6a9d558c5ec0020ace0d488c9c80020c2dcd5d80020c778c1c4c5d00020ac00c7a50020c801d569d55c002000410064006f0062006500200050004400460020bb38c11cb97c0020c791c131d569b2c8b2e4002e0020c774b807ac8c0020c791c131b41c00200050004400460020bb38c11cb2940020004100630072006f0062006100740020bc0f002000410064006f00620065002000520065006100640065007200200035002e00300020c774c0c1c5d0c11c0020c5f40020c2180020c788c2b5b2c8b2e4002egt13 LTH 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 LVI 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 NLD (Gebruik deze instellingen om Adobe PDF-documenten te maken die zijn geoptimaliseerd voor prepress-afdrukken van hoge kwaliteit De gemaakte PDF-documenten kunnen worden geopend met Acrobat en Adobe Reader 50 en hoger)13 NOR 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 POL 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 PTB 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 RUM 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 RUS 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 SKY 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 SLV 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 SUO 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 SVE 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 TUR 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 UKR ltFEFF04120438043a043e0440043804410442043e043204430439044204350020044604560020043f043004400430043c043504420440043800200434043b044f0020044104420432043e04400435043d043d044f00200434043e043a0443043c0435043d044204560432002000410064006f006200650020005000440046002c0020044f043a04560020043d04300439043a04400430044904350020043f045604340445043e0434044f0442044c00200434043b044f0020043204380441043e043a043e044f043a04560441043d043e0433043e0020043f0435044004350434043404400443043a043e0432043e0433043e0020043404400443043a0443002e00200020042104420432043e04400435043d045600200434043e043a0443043c0435043d0442043800200050004400460020043c043e0436043d04300020043204560434043a0440043804420438002004430020004100630072006f006200610074002004420430002000410064006f00620065002000520065006100640065007200200035002e0030002004300431043e0020043f04560437043d04560448043e04570020043204350440044104560457002egt13 ENU (Use these settings to create Adobe PDF documents best suited for high-quality prepress printing Created PDF documents can be opened with Acrobat and Adobe Reader 50 and later)13 gtgt13 Namespace [13 (Adobe)13 (Common)13 (10)13 ]13 OtherNamespaces [13 ltlt13 AsReaderSpreads false13 CropImagesToFrames true13 ErrorControl WarnAndContinue13 FlattenerIgnoreSpreadOverrides false13 IncludeGuidesGrids false13 IncludeNonPrinting false13 IncludeSlug false13 Namespace [13 (Adobe)13 (InDesign)13 (40)13 ]13 OmitPlacedBitmaps false13 OmitPlacedEPS false13 OmitPlacedPDF false13 SimulateOverprint Legacy13 gtgt13 ltlt13 AddBleedMarks false13 AddColorBars false13 AddCropMarks false13 AddPageInfo false13 AddRegMarks false13 ConvertColors ConvertToCMYK13 DestinationProfileName ()13 DestinationProfileSelector DocumentCMYK13 Downsample16BitImages true13 FlattenerPreset ltlt13 PresetSelector MediumResolution13 gtgt13 FormElements false13 GenerateStructure false13 IncludeBookmarks false13 IncludeHyperlinks false13 IncludeInteractive false13 IncludeLayers false13 IncludeProfiles false13 MultimediaHandling UseObjectSettings13 Namespace [13 (Adobe)13 (CreativeSuite)13 (20)13 ]13 PDFXOutputIntentProfileSelector DocumentCMYK13 PreserveEditing true13 UntaggedCMYKHandling LeaveUntagged13 UntaggedRGBHandling UseDocumentProfile13 UseDocumentBleed false13 gtgt13 ]13gtgt setdistillerparams13ltlt13 HWResolution [2400 2400]13 PageSize [612000 792000]13gtgt setpagedevice13

Page 23: February 2014 MEE Questions and AnalysesPreface The Multistate Essay Examination (MEE) is developed by the National Conference of Bar Examiners (NCBE). This publication includes the

Federal Civil Procedure Analysis

a consultant Thus if the customer had hired the structural engineer to prepare a report ldquoin anticipation of litigationrdquo that report might not be discoverable See FED R CIV P 26(b)(3)

In this case however the customer hired the engineer to evaluate the foundation of the house as part of the customerrsquos negotiation with the builder concerning the housersquos flooding problem The builder told the customer that the housersquos landscaping was the reason for the flooding and the builder told the customer ldquoHave an engineer look at the foundation If therersquos a problem wersquoll fix itrdquo The customer appears to have acted in response to that statement There is no indication that the customer anticipated any kind of legal action at the time that the structural engineer was hired Accordingly the structural engineerrsquos report is discoverable and the court should order the customer to turn it over

[NOTE If an examinee concludes that the structural engineerrsquos report was prepared in anticipation of litigation then the examinee should also conclude that the report is not discoverable Documents prepared in anticipation of litigation do not need to be disclosed to an adverse party unless that party can demonstrate a ldquosubstantial needrdquo for the documents and an inability to obtain substantially equivalent information without ldquoundue hardshiprdquo FED R CIV P 26(b)(3)(A)(ii) Furthermore a report prepared by an expert who is not expected to testify is not discoverable in the absence of ldquoexceptional circumstancesrdquo making it ldquoimpracticablerdquo to obtain the information in another way FED R CIV P 26(b)(4)(D)(ii) The builder probably cannot make these showings here unless the engineerrsquos report deals with circumstances that have since changed There is no evidence that the structural engineer would have had access to any information or facts that the builder would not already know as a result of its construction and subsequent inspection of the house In addition if necessary the builder could ask the court for permission to arrange for a further inspection of the house by a structural engineer hired by the builder See FED R CIV P 34(a)(2) Accordingly if an examinee concludes that the report was prepared in anticipation of litigation the examinee should also conclude that the builder is not entitled to see the report]

Point Two(a) (30) Because the builder anticipated that it might be involved in litigation concerning its contract with the customer the builder acted wrongfully in destroying emails that were relevant to the housersquos construction even though the emails were destroyed pursuant to a routine document retention plan

As noted above a party to a lawsuit in federal court ldquomay obtain discovery regarding any nonprivileged matter that is relevant to any partyrsquos claim or defenserdquo FED R CIV P 26(b)(1) This includes emails and other electronically stored information FED R CIV P 34(a)(1)(A) Here the customer has requested all the builderrsquos emails pertaining to work done on the foundation of the house Ordinarily the builder would be obliged to turn over this information which is relevant to the customerrsquos defense that the housersquos foundation was poorly constructed

Unfortunately the emails in question no longer exist because the builder destroyed them on August 2

In general spoliation of evidence (destruction or alteration of evidence) is improper if the party who destroyed or altered the evidence ldquohas notice that the evidence is relevant to litigation or should have known that the evidence may be relevant to future litigationrdquo Fujitsu Ltd v Federal Express Corp 247 F3d 423 436 (2d Cir 2001) It is improper for a party to destroy electronic information relevant to pending litigation even if the destruction occurs before there is any request or order seeking the information See eg Leon v IDX Sys Corp 464 F3d 951 (9th Cir 2006) (plaintiffrsquos intentional destruction of computer files warranted dismissal even

20

In this case the builderrsquos destruction of the emails was pursuant to a routine document retention plan The Federal Rules provide expressly that in the absence of ldquoexceptional circumstancesrdquo parties should not be sanctioned for the loss of electronically stored information when the loss occurs pursuant to ldquoroutine good-faith operation of an electronic information systemrdquo FED R CIV P 37(e) However when a party anticipates litigation ldquoit must suspend its routine document retentiondestruction policy and put in place a lsquolitigation holdrsquo to ensure the preservation of relevant documentsrdquo Zubulake v UBS Warburg LLC 220 FRD 212 218 (SDNY 2003)

Federal Civil Procedure Analysis

though spoliation occurred before order compelling discovery) Similarly the duty to preserve evidence applies to a party who anticipates litigation even if litigation has not yet been commenced See THE SEDONA PRINCIPLES BEST PRACTICES RECOMMENDATIONS amp PRINCIPLES FOR ADDRESSING ELECTRONIC DOCUMENT PRODUCTION 70 cmt 14a (2d ed 2007)

The builder destroyed the emails on August 2 At that time the builder knew that litigation was a possibility because the builder had already directed its attorney to prepare a draft complaint for possible filing Knowing that litigation was a possibility the builder had a duty to take steps to preserve evidence including the emails in question See generally Fujitsu Ltd

Thus the builderrsquos destruction of potentially relevant emails at a time when it knew that litigation was a possibility was improper It had a duty to preserve evidence and it breached that duty

[NOTE Because courts have used different words to describe the test for when evidence must be preserved an examineersquos precise formulation of the test is not critical]

Point Two(b) (30) In determining appropriate sanctions for spoliation courts consider both the level of culpability of the spoliating party and the degree of prejudice the loss of evidence has caused the other party Here the builderrsquos destruction of evidence does not appear to have been willful nor is it likely to pose a significant obstacle to the customerrsquos defense Any sanctions imposed by the court should be modest

Federal courts have inherent power to control the litigation process and can sanction misbehavior including spoliation even when there has been no specific violation of the Federal Rules of Civil Procedure See generally Chambers v NASCO Inc 501 US 32 (1991) (discussing courtrsquos inherent power to control the litigation process) The range of available sanctions is broad It can include such sanctions as the payment of expenses incurred by the other party as a result of the destruction of the evidence an instruction to the jury authorizing it to draw an adverse inference from the destruction of the evidence a shifting of the burden of proof on the relevant issue or even judgment against the responsible party See eg Residential Funding Corp v DeGeorge Financial Corp 306 F3d 99 108 (2d Cir 2002) (adverse inference) Silvestri v General Motors Corp 271 F3d 583 593 (4th Cir 2001) (possibility of dismissal) Cf FED R CIV P 37(b)(2)(A) (listing remedies for failure to comply with discovery obligations)

In determining appropriate sanctions for spoliation courts consider both the level of culpability of the spoliating party and the degree of prejudice the loss of evidence has caused the other party Many courts impose severe sanctions (such as an adverse-inference instruction or the entry of judgment against the spoliating party) only when there is evidence of bad faith in the form of an intentional effort to hide information Eg Greyhound Lines Inc v Wade 485 F3d 1032 1035 (8th Cir 2007) (spoliation sanction requires intentional destruction out of desire ldquoto suppress the truthrdquo) However other courts have said that negligence in preserving evidence can

21

Federal Civil Procedure Analysis

support an adverse-inference instruction See Residential Funding 306 F3d at 108 (negligence enough under some circumstances)

Although a court might well order an evidentiary hearing on the issue of sanctions the facts presented do not seem appropriate for severe sanctions First the evidence was destroyed pursuant to the builderrsquos standard document retention plan and there is no evidence that the builder deliberately failed to suspend its usual procedures with the purpose of allowing the destruction of evidence Second the loss of this evidence will not severely hinder the customerrsquos presentation of his case The central issue is whether the foundation of the house was properly constructed If the construction job was poorly done the customer can present evidence derived from inspection of the premises to prove that point The customer can also depose witnesses about any issues that arose during construction

Under the circumstances a court is not likely to impose particularly severe sanctions although it might shift the burden to the builder to show that the foundation was properly constructed or it might require the builder to reimburse any expenses the customer incurs to discover and prove the facts about issues or disputes that arose during construction of the foundation

[NOTE The result reached by the examinee is less important than the examineersquos recognition that (a) a range of sanctions is available to the court and (b) the appropriate sanction depends both on the culpability of the builder and the prejudice suffered by the customer]

22

CRIMINAL LAW AND PROCEDURE ANALYSIS (Criminal Law and Procedure IIA amp D VE amp F)

ANALYSIS

Legal Problems

(1) Did charging the defendant with both theft and burglary constitute double jeopardy

(2) Did the jury instruction violate the due process clause either by relieving the prosecution of the burden of proving the element of intent or by shifting the burden to the defendant to disprove that element

(3) Did the sentence imposed in this case for the theft conviction unconstitutionally deprive the defendant of his right to a jury trial on the issue of the value of the stolen item

DISCUSSION

Summary

The trial court properly denied the defendantrsquos pretrial motion to dismiss the charges on double jeopardy grounds The defendant may be charged with and convicted of both theft and burglary Each of the charges has an element that the other does not Neither charge is a lesser-included offense nor are they multiplicitous Thus charging both theft and burglary does not violate double jeopardy

The jury instruction on the burglary charge was constitutionally flawed It could have been reasonably understood by the jury as either (1) an irrebuttable conclusive presumption (which relieved the prosecution of proving the element of intent and removed the issue from the jury) or (2) a rebuttable mandatory presumption (which unconstitutionally shifted the burden of proof on an element of a charged offense from the prosecution to the defendant)

Because the four-year sentence imposed by the judge was based on the judgersquos finding by a preponderance of the evidence that the value of the stolen ring exceeded $5000 the sentence violates the defendantrsquos right to a jury determination beyond a reasonable doubt of the value of the ring

Point One (30) Charging the defendant with theft and burglary did not constitute double jeopardy

The Double Jeopardy Clause of the Fifth Amendment provides that a person shall not be twice put in jeopardy for the ldquosame offenserdquo Thus the question is whether the elements of the theft charge are wholly contained in the burglary charge or vice versa If the elements of the lesser charge (theft) are not wholly contained in the greater charge (burglary)mdashie if each charge requires proof of a fact that the other does notmdashthen convicting the defendant of both crimes would not violate double jeopardy even when the two offenses occurred at the same time and are thus arguably part of the ldquosame transactionrdquo Blockburger v United States 284 US 299 304 (1932) See also Albernaz v United States 450 US 333 344 n3 (1981) United States v Dixon 509 US 688 704 (1993)

23

Criminal Law and Procedure Analysis

Here theft and burglary each require proof of an element not required for the other crime Burglary may be defined differently in different jurisdictions However it almost invariably requires entry into a building or dwelling of another with the specific intent to commit a felony therein and the crime of burglary is complete upon the entry into the building or dwelling with such intent See eg Cannon v Oklahoma 827 P2d 1339 1342 (Okla Crim App 1992) In contrast theft which also may be defined differently in different states almost invariably requires the taking and carrying away of an item of personal property belonging to another with the intent to steal or permanently deprive the owner of possession

Here the ldquotakingrdquo or ldquostealingrdquo element is not contained in the definition of burglary and the ldquoentryrdquo element of burglary is not contained in the definition of theft Because theft is not a lesser-included offense of burglary and burglary is not a lesser-included offense of theft charging the defendant for both burglary and theft did not violate double jeopardy and the court properly denied the defense motion on those grounds Yparrea v Dorsey 64 F3d 577 579ndash80 (10th Cir 1995) citing Blockburger 284 US at 304

Finally the defendantrsquos motion to dismiss all the charges on double jeopardy grounds was improper because if both charges were for the same offense the motion should have requested dismissal of one charge not both

Point Two (35) The jury instruction on the burglary charge violated the Due Process Clause because it created either (1) an irrebuttable conclusive presumption (which relieved the prosecution of proving the element of intent and removed that issue from the jury) or (2) a rebuttable mandatory presumption (which unconstitutionally shifted the burden of proof on an element of a charged offense to the defendant)

The Supreme Court has interpreted the Due Process Clause of the US Constitution to require that the prosecution prove all elements of an offense beyond a reasonable doubt See In re Winship 397 US 358 364 (1970) The burden of proof cannot be shifted to the defendant by presuming an essential element upon proof of other elements of the offense because shifting the burden of persuasion with respect to any element of a criminal offense is contrary to the Due Process Clause See Mullaney v Wilbur 421 US 684 (1975)

The crime of burglary includes entry into a building or dwelling with the specific intent to commit a felony therein The requirement that the prosecutor prove beyond a reasonable doubt that the defendant had this specific intent distinguishes burglary from general-intent crimes like trespass See Sandstrom v Montana 442 US 510 523 (1979)

Here the jury was instructed that if ldquoafter consideration of all the evidence presented by the prosecution and defense you find beyond a reasonable doubt that the defendant entered the dwelling without the ownersrsquo consent you may presume that the defendant entered with the intent to commit a felony thereinrdquo This instruction was unconstitutional because it created either an irrebuttable conclusive presumption or a rebuttable mandatory presumption

A conclusive presumption is ldquoan irrebuttable direction by the court to find intent once convinced of the facts triggering the presumptionrdquo Id at 517 Here the jurors were instructed that once the prosecutor established that the defendant entered the neighborsrsquo house without consent they ldquomay presumerdquo that he intended to commit a felony therein The jurors may have reasonably concluded from this instruction that if they found that the defendant intended to enter his neighborsrsquo home without permission they must further find that he entered with the specific intent to commit a felony therein Because this instruction could operate as a conclusive

24

Criminal Law and Procedure Analysis

irrebuttable presumption by eliminating intent ldquoas an ingredient of the offenserdquo it violated due process by relieving the prosecution of the burden of proof for this element Id at 522

In the alternative the jury instruction could have been reasonably understood to create a rebuttable mandatory presumption which ldquotells [the jury] they must find the elemental fact upon proof of the basic fact at least unless the defendant has come forward with some evidence to rebut the presumed connection between the two factsrdquo County Court of Ulster County New York v Allen 442 US 140 157 (1979) The due process problem created by rebuttable mandatory presumptions is that ldquo[t]o the extent that the trier of fact is forced to abide by the presumption and may not reject it based on an independent evaluation of the particular facts presented by the State the analysis of the presumptionrsquos constitutional validity is logically divorced from those facts and based on the presumptionrsquos accuracy in the run of casesrdquo Id at 159

Unlike irrebuttable conclusive presumptions rebuttable mandatory presumptions are not always per se violations of the Due Process Clause However the Supreme Court of the United States has held that jury instructions that could reasonably be understood as shifting the burden of proof to the defendant on an element of the offense are unconstitutional Francis v Franklin 471 US 307 (1985) Here the argument that the jury instruction operated as a rebuttable mandatory presumption is supported by the fact that the judge also instructed the jury to ldquoconsider[ ] all the evidence presented by the prosecution and defenserdquo However even if the instruction created a rebuttable mandatory presumption it would be unconstitutional because it shifted the burden to the defense on an element of the offense Sandstrom 442 US at 524 Mullaney 421 US at 686

[NOTE Whether an examinee identifies the jury instruction as containing a ldquoconclusiverdquo or ldquomandatoryrdquo presumption is less important than the examineersquos analysis of the constitutional infirmities]

Point Three (35) The trial court violated the defendantrsquos Sixth Amendment right to a jury trial on an essential element of the offense when it found by a preponderance of the evidence that the ring was worth over $5000 and increased the defendantrsquos sentence based on this finding

In the statutory scheme under which the defendant was tried and convicted a Class D felony theft is defined as theft of item(s) with a value between $2500 and $10000 The jury found that the value of the diamond ring was at least $2500 and convicted the defendant of felony theft However at sentencing the trial court made a separate finding by a preponderance of the evidence that the value of the ring was greater than $5000 Following the statutersquos two-tiered sentencing scheme the judge then imposed on the defendant a sentence that was one year longer than the maximum that would otherwise have been allowed

The judgersquos sentence was unconstitutional because it violated the defendantrsquos Sixth Amendment right to a jury trial on this question The Supreme Court held in Apprendi v New Jersey 530 US 466 (2000) that ldquo[o]ther than the fact of a prior conviction any fact that increases the penalty for a crime beyond the prescribed statutory maximum must be submitted to a jury and proved beyond a reasonable doubtrdquo because ldquo[i]t is unconstitutional for a legislature to remove from the jury the assessment of facts that increase the prescribed range of penalties to which a criminal defendant is exposed [because] such facts must be established by proof beyond a reasonable doubtrdquo Id The Court reaffirmed Apprendi in Blakely v Washington 542 US 296 (2004) holding that the ldquolsquostatutory maximumrsquo for Apprendi purposes is the maximum sentence a judge may impose solely on the basis of the facts reflected in the jury verdict or admitted by the defendantrdquo Id at 303 (emphasis in original) In United States v Booker 543 US 220 (2005)

25

Criminal Law and Procedure Analysis

the Court relied on Blakely and Apprendi to conclude that protecting a defendantrsquos Sixth Amendment right to a jury trial required that ldquo[a]ny fact which is necessary to support a sentence exceeding the maximum authorized by the facts established by a plea of guilty or a jury verdict must be admitted by the defendant or proved to a jury beyond a reasonable doubtrdquo Id at 244

Thus in order to constitutionally increase a sentence above the statutory maximum of three years the jury must have found beyond a reasonable doubt that the value of the ring exceeded $5000 Here the court made the finding based on an appraisal proffered by the prosecutor only at sentencing and the judgersquos finding was by a preponderance of the evidence rather than beyond a reasonable doubt

26

AGENCY AND PARTNERSHIP ANALYSIS __________ (Agency and Partnership VA amp C VI)

ANALYSIS

Legal Problems

(1) Is a partner in a general partnership personally liable on a claim arising from misrepresentations by another partner made in the course of the partnership business

(2) Does a newly admitted partner in a general partnership become personally liable on existing claims against the partnership

(3) After the filing by a general partnership of a statement of qualification as a limited liability partnership are the partners personally liable as partners on (a) an existing claim against the general partnership and (b) a claim against the partnership that arose after the filing

DISCUSSION

Summary

Adam and Ben formed a general partnership under which they were jointly and severally liable for obligations of the partnership Thus Adam was personally liable for misrepresentations by Ben made in the ordinary course of the partnership business

Upon joining the general partnership Diane became personally liable for the obligations of the partnership arising after her admission but not for obligations pre-existing her admission such as the collectorrsquos claim

By filing a statement of qualification the three partners properly elected limited liability partnership status As partners in an LLP none of the three partners is personally liable as a partner for partnership obligations arising after the election such as the claim by the driverrsquos estate The election however does not change their personal liability on pre-existing claims that arose before the election such as the collectorrsquos claim

Point One (30) As a general partner of Empire a general partnership Adam became personally liable on the collectorrsquos claim a valid claim against the partnership that arose because of Benrsquos wrongful act in the ordinary course of the partnership business

When the collectorrsquos claim arose Empire was a general partnership composed of Adam and Ben Under UPA (1997) sect 306(a) partners of a general partnership are liable jointly and severally for all obligations of the partnership Under UPA (1997) sect 305(a) the partnership could become obligated for the loss caused to the collector as a result of the misrepresentation by Ben provided he was acting in the ordinary course of the partnership business Because there was no statement that limited his partnership authority Ben as partner was ldquoan agent of the partnership for the purpose of its businessrdquo See UPA (1997) sect 301(1) Benrsquos misrepresentation to the collector even if intentional appears to be in the ordinary course of the partnershiprsquos business of dealing

27

Agency and Partnership Analysis

in antique cars Thus Benrsquos wrongful act created a partnership obligation for which Adam was jointly and severally liable

[NOTE Generally a partnership creditor must ldquoexhaust the partnershiprsquos assets before levying on a judgment debtor partnerrsquos individual property where the partner is personally liable for the partnership obligationrdquo as a result of his status as a partner UPA (1997) sect 307 cmt 4 As the UPA comments explain this places Adam more in the position of guarantor than principal debtor on the partnership obligation Id cmt 4 Although an examinee might discuss this point the call focuses on whether Adam is personally liable not how the liability might be enforced]

Point Two (30) Because the collectorrsquos claim arose before Diane joined Empire Diane did not become personally liable on the claim

Diane was admitted to Empire when it was a general partnership and after the collectorrsquos claim arose While the general rule under UPA (1997) sect 306(a) is that the partners of a general partnership are liable jointly and severally for all obligations of the partnership there is a special rule for partners who are admitted during the duration of the partnership Under UPA (1997) sect 306(b) a person admitted to an existing partnership is not personally liable for any partnership obligations incurred before the personrsquos admission Because Diane was admitted to Empire after the collectorrsquos claim arose Diane is not personally liable on the claim

Dianersquos knowledge of the pre-existing claim and her stated concern about becoming liable on the collectorrsquos claim do not change her personal nonliability to the collector Although partners who have a liability shield can assume liability to third parties through private contractual guarantees or modifications to the partnership agreement Dianersquos stated concern constituted neither a guaranty to the collector nor ldquoan intentional waiver of liability protectionsrdquo See UPA (1997) sect 306 cmt 3 (describing methods for waiver of liability protections under sect 306(c) applicable in limited liability partnerships)

At most Diane will lose her investment in the partnership as a result of the collectorrsquos claim Although Diane did not become personally liable on the collectorrsquos claim when she joined the partnership the $250000 she contributed to the partnership is ldquoat risk for the satisfaction of existing partnership debtsrdquo UPA (1997) sect 306 cmt 2

Point Three (40) Filing the statement of qualification was effective to elect limited liability partnership status Despite this new status Adam and Ben remain personally liable on the collectorrsquos claim which arose before the election But as partners in an LLP neither Adam Ben nor Diane is personally liable as a partner on the driverrsquos estatersquos claim which arose after the election

Under UPA (1997) sect 1001 a general partnership can make an election and become a limited liability partnershipmdashif the partners approve the conversion by a vote equivalent to that necessary to amend the partnership agreement and the partnership then files a statement of qualification that specifies the name of the partnership its principal office and its election to be an LLP Here the partners agreed unanimouslymdashsufficient to amend their agreement under UPA (1997) sect 401(j)mdashand the statement of qualification was filed In addition the name of Empire LLP properly included an appropriate ending ldquoLLPrdquo See UPA (1997) sect 1002

Although another way to effectuate a ldquoconversionrdquo (as suggested by Benrsquos lawyer) is to form a new LLP and transfer the assets of the old general partnership to the new LLP the

28

Agency and Partnership Analysis

method used here (approval by the partners and the filing of a statement of qualification) is also sufficient to create LLP status

Thus Empire became Empire LLP as of the date of filing of the statement of qualification See UPA (1997) sect 1001 What effect did this have on the collectorrsquos claim which predated the filing According to UPA (1997) sect 306(c) an obligation incurred while a partnership is an LLP is solely a partnership obligation As the collectorrsquos claim predated the LLP Adam and Ben remain personally liable on the collectorrsquos claim Diane on the other hand was not personally liable on the collectorrsquos claim either before or after the filing of the statement of qualification See Point Two above

The driverrsquos estatersquos claim arose after Empire became Empire LLP Under UPA (1997) sect 306(c) an obligation incurred while a partnership is an LLP is solely a partnership obligationThus Adam Ben and Diane as partners are all protected from personal liability on the driverrsquos estatersquos claim But there may be personal liability if any of them was negligent or otherwise acted wrongfully by not informing the buyer of the bad suspension that caused the accident

29

National Conference of Bar Examiners 302 South Bedford Street | Madison WI 53703-3622 Phone 608-280-8550 | Fax 608-280-8552 | TDD 608-661-1275

wwwncbexorg e-mail contactncbexorg

  • Preface
  • Description of the MEE
  • Instructions
  • February 2014 Questions
    • Constitutinal Law Question
    • Trusts and Future Interests Question
    • Secured Transactions Question
    • Federal Civil Procedure Question
    • Criminal Law and Procedure Question
    • Agency and Partnership Question
      • February 2014 Analyses
        • Constitutional Law Analysis
        • Trust and Future Interests Analysis
        • Secured Transactions Analysis
        • Federal Civil Procedure Analysis
        • Criminal Law and Procedure Analysis
        • Agency and Partnership Analysis
            • ltlt13 ASCII85EncodePages false13 AllowTransparency false13 AutoPositionEPSFiles true13 AutoRotatePages None13 Binding Left13 CalGrayProfile (Dot Gain 20)13 CalRGBProfile (sRGB IEC61966-21)13 CalCMYKProfile (US Web Coated 050SWOP051 v2)13 sRGBProfile (sRGB IEC61966-21)13 CannotEmbedFontPolicy Error13 CompatibilityLevel 1413 CompressObjects Tags13 CompressPages true13 ConvertImagesToIndexed true13 PassThroughJPEGImages true13 CreateJobTicket false13 DefaultRenderingIntent Default13 DetectBlends true13 DetectCurves 0000013 ColorConversionStrategy CMYK13 DoThumbnails false13 EmbedAllFonts true13 EmbedOpenType false13 ParseICCProfilesInComments true13 EmbedJobOptions true13 DSCReportingLevel 013 EmitDSCWarnings false13 EndPage -113 ImageMemory 104857613 LockDistillerParams false13 MaxSubsetPct 10013 Optimize true13 OPM 113 ParseDSCComments true13 ParseDSCCommentsForDocInfo true13 PreserveCopyPage true13 PreserveDICMYKValues true13 PreserveEPSInfo true13 PreserveFlatness true13 PreserveHalftoneInfo false13 PreserveOPIComments true13 PreserveOverprintSettings true13 StartPage 113 SubsetFonts true13 TransferFunctionInfo Apply13 UCRandBGInfo Preserve13 UsePrologue false13 ColorSettingsFile ()13 AlwaysEmbed [ true13 ]13 NeverEmbed [ true13 ]13 AntiAliasColorImages false13 CropColorImages true13 ColorImageMinResolution 30013 ColorImageMinResolutionPolicy OK13 DownsampleColorImages true13 ColorImageDownsampleType Bicubic13 ColorImageResolution 30013 ColorImageDepth -113 ColorImageMinDownsampleDepth 113 ColorImageDownsampleThreshold 15000013 EncodeColorImages true13 ColorImageFilter DCTEncode13 AutoFilterColorImages true13 ColorImageAutoFilterStrategy JPEG13 ColorACSImageDict ltlt13 QFactor 01513 HSamples [1 1 1 1] VSamples [1 1 1 1]13 gtgt13 ColorImageDict ltlt13 QFactor 01513 HSamples [1 1 1 1] VSamples [1 1 1 1]13 gtgt13 JPEG2000ColorACSImageDict ltlt13 TileWidth 25613 TileHeight 25613 Quality 3013 gtgt13 JPEG2000ColorImageDict ltlt13 TileWidth 25613 TileHeight 25613 Quality 3013 gtgt13 AntiAliasGrayImages false13 CropGrayImages true13 GrayImageMinResolution 30013 GrayImageMinResolutionPolicy OK13 DownsampleGrayImages true13 GrayImageDownsampleType Bicubic13 GrayImageResolution 30013 GrayImageDepth -113 GrayImageMinDownsampleDepth 213 GrayImageDownsampleThreshold 15000013 EncodeGrayImages true13 GrayImageFilter DCTEncode13 AutoFilterGrayImages true13 GrayImageAutoFilterStrategy JPEG13 GrayACSImageDict ltlt13 QFactor 01513 HSamples [1 1 1 1] VSamples [1 1 1 1]13 gtgt13 GrayImageDict ltlt13 QFactor 01513 HSamples [1 1 1 1] VSamples [1 1 1 1]13 gtgt13 JPEG2000GrayACSImageDict ltlt13 TileWidth 25613 TileHeight 25613 Quality 3013 gtgt13 JPEG2000GrayImageDict ltlt13 TileWidth 25613 TileHeight 25613 Quality 3013 gtgt13 AntiAliasMonoImages false13 CropMonoImages true13 MonoImageMinResolution 120013 MonoImageMinResolutionPolicy OK13 DownsampleMonoImages true13 MonoImageDownsampleType Bicubic13 MonoImageResolution 120013 MonoImageDepth -113 MonoImageDownsampleThreshold 15000013 EncodeMonoImages true13 MonoImageFilter CCITTFaxEncode13 MonoImageDict ltlt13 K -113 gtgt13 AllowPSXObjects false13 CheckCompliance [13 None13 ]13 PDFX1aCheck false13 PDFX3Check false13 PDFXCompliantPDFOnly false13 PDFXNoTrimBoxError true13 PDFXTrimBoxToMediaBoxOffset [13 00000013 00000013 00000013 00000013 ]13 PDFXSetBleedBoxToMediaBox true13 PDFXBleedBoxToTrimBoxOffset [13 00000013 00000013 00000013 00000013 ]13 PDFXOutputIntentProfile ()13 PDFXOutputConditionIdentifier ()13 PDFXOutputCondition ()13 PDFXRegistryName ()13 PDFXTrapped False1313 CreateJDFFile false13 Description ltlt13 ARA 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 BGR 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 CHS ltFEFF4f7f75288fd94e9b8bbe5b9a521b5efa7684002000410064006f006200650020005000440046002065876863900275284e8e9ad88d2891cf76845370524d53705237300260a853ef4ee54f7f75280020004100630072006f0062006100740020548c002000410064006f00620065002000520065006100640065007200200035002e003000204ee553ca66f49ad87248672c676562535f00521b5efa768400200050004400460020658768633002gt13 CHT ltFEFF4f7f752890194e9b8a2d7f6e5efa7acb7684002000410064006f006200650020005000440046002065874ef69069752865bc9ad854c18cea76845370524d5370523786557406300260a853ef4ee54f7f75280020004100630072006f0062006100740020548c002000410064006f00620065002000520065006100640065007200200035002e003000204ee553ca66f49ad87248672c4f86958b555f5df25efa7acb76840020005000440046002065874ef63002gt13 CZE 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 DAN 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 DEU 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 ESP ltFEFF005500740069006c0069006300650020006500730074006100200063006f006e0066006900670075007200610063006900f3006e0020007000610072006100200063007200650061007200200064006f00630075006d0065006e0074006f00730020005000440046002000640065002000410064006f0062006500200061006400650063007500610064006f00730020007000610072006100200069006d0070007200650073006900f3006e0020007000720065002d0065006400690074006f007200690061006c00200064006500200061006c00740061002000630061006c0069006400610064002e002000530065002000700075006500640065006e00200061006200720069007200200064006f00630075006d0065006e0074006f00730020005000440046002000630072006500610064006f007300200063006f006e0020004100630072006f006200610074002c002000410064006f00620065002000520065006100640065007200200035002e003000200079002000760065007200730069006f006e0065007300200070006f00730074006500720069006f007200650073002egt13 ETI ltFEFF004b00610073007500740061006700650020006e0065006900640020007300e4007400740065006900640020006b00760061006c006900740065006500740073006500200074007200fc006b006900650065006c007300650020007000720069006e00740069006d0069007300650020006a0061006f006b007300200073006f00620069006c0069006b0065002000410064006f006200650020005000440046002d0064006f006b0075006d0065006e00740069006400650020006c006f006f006d006900730065006b0073002e00200020004c006f006f0064007500640020005000440046002d0064006f006b0075006d0065006e00740065002000730061006100740065002000610076006100640061002000700072006f006700720061006d006d006900640065006700610020004100630072006f0062006100740020006e0069006e0067002000410064006f00620065002000520065006100640065007200200035002e00300020006a00610020007500750065006d006100740065002000760065007200730069006f006f006e00690064006500670061002e000d000agt13 FRA 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 GRE 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 HEB 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 HRV (Za stvaranje Adobe PDF dokumenata najpogodnijih za visokokvalitetni ispis prije tiskanja koristite ove postavke Stvoreni PDF dokumenti mogu se otvoriti Acrobat i Adobe Reader 50 i kasnijim verzijama)13 HUN 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 ITA 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 JPN ltFEFF9ad854c18cea306a30d730ea30d730ec30b951fa529b7528002000410064006f0062006500200050004400460020658766f8306e4f5c6210306b4f7f75283057307e305930023053306e8a2d5b9a30674f5c62103055308c305f0020005000440046002030d530a130a430eb306f3001004100630072006f0062006100740020304a30883073002000410064006f00620065002000520065006100640065007200200035002e003000204ee5964d3067958b304f30533068304c3067304d307e305930023053306e8a2d5b9a306b306f30d530a930f330c8306e57cb30818fbc307f304c5fc59808306730593002gt13 KOR ltFEFFc7740020c124c815c7440020c0acc6a9d558c5ec0020ace0d488c9c80020c2dcd5d80020c778c1c4c5d00020ac00c7a50020c801d569d55c002000410064006f0062006500200050004400460020bb38c11cb97c0020c791c131d569b2c8b2e4002e0020c774b807ac8c0020c791c131b41c00200050004400460020bb38c11cb2940020004100630072006f0062006100740020bc0f002000410064006f00620065002000520065006100640065007200200035002e00300020c774c0c1c5d0c11c0020c5f40020c2180020c788c2b5b2c8b2e4002egt13 LTH 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 LVI 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 NLD (Gebruik deze instellingen om Adobe PDF-documenten te maken die zijn geoptimaliseerd voor prepress-afdrukken van hoge kwaliteit De gemaakte PDF-documenten kunnen worden geopend met Acrobat en Adobe Reader 50 en hoger)13 NOR 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 POL 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 PTB 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 RUM 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 RUS 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 SKY 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 SLV 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 SUO ltFEFF004b00e40079007400e40020006e00e40069007400e4002000610073006500740075006b007300690061002c0020006b0075006e0020006c0075006f00740020006c00e400680069006e006e00e4002000760061006100740069007600610061006e0020007000610069006e006100740075006b00730065006e002000760061006c006d0069007300740065006c00750074007900f6006800f6006e00200073006f00700069007600690061002000410064006f0062006500200050004400460020002d0064006f006b0075006d0065006e007400740065006a0061002e0020004c0075006f0064007500740020005000440046002d0064006f006b0075006d0065006e00740069007400200076006f0069006400610061006e0020006100760061007400610020004100630072006f0062006100740069006c006c00610020006a0061002000410064006f00620065002000520065006100640065007200200035002e0030003a006c006c00610020006a006100200075007500640065006d006d0069006c006c0061002egt13 SVE 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 TUR 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 UKR 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 ENU (Use these settings to create Adobe PDF documents best suited for high-quality prepress printing Created PDF documents can be opened with Acrobat and Adobe Reader 50 and later)13 gtgt13 Namespace [13 (Adobe)13 (Common)13 (10)13 ]13 OtherNamespaces [13 ltlt13 AsReaderSpreads false13 CropImagesToFrames true13 ErrorControl WarnAndContinue13 FlattenerIgnoreSpreadOverrides false13 IncludeGuidesGrids false13 IncludeNonPrinting false13 IncludeSlug false13 Namespace [13 (Adobe)13 (InDesign)13 (40)13 ]13 OmitPlacedBitmaps false13 OmitPlacedEPS false13 OmitPlacedPDF false13 SimulateOverprint Legacy13 gtgt13 ltlt13 AddBleedMarks false13 AddColorBars false13 AddCropMarks false13 AddPageInfo false13 AddRegMarks false13 ConvertColors ConvertToCMYK13 DestinationProfileName ()13 DestinationProfileSelector DocumentCMYK13 Downsample16BitImages true13 FlattenerPreset ltlt13 PresetSelector MediumResolution13 gtgt13 FormElements false13 GenerateStructure false13 IncludeBookmarks false13 IncludeHyperlinks false13 IncludeInteractive false13 IncludeLayers false13 IncludeProfiles false13 MultimediaHandling UseObjectSettings13 Namespace [13 (Adobe)13 (CreativeSuite)13 (20)13 ]13 PDFXOutputIntentProfileSelector DocumentCMYK13 PreserveEditing true13 UntaggedCMYKHandling LeaveUntagged13 UntaggedRGBHandling UseDocumentProfile13 UseDocumentBleed false13 gtgt13 ]13gtgt setdistillerparams13ltlt13 HWResolution [2400 2400]13 PageSize [612000 792000]13gtgt setpagedevice13

Page 24: February 2014 MEE Questions and AnalysesPreface The Multistate Essay Examination (MEE) is developed by the National Conference of Bar Examiners (NCBE). This publication includes the

In this case the builderrsquos destruction of the emails was pursuant to a routine document retention plan The Federal Rules provide expressly that in the absence of ldquoexceptional circumstancesrdquo parties should not be sanctioned for the loss of electronically stored information when the loss occurs pursuant to ldquoroutine good-faith operation of an electronic information systemrdquo FED R CIV P 37(e) However when a party anticipates litigation ldquoit must suspend its routine document retentiondestruction policy and put in place a lsquolitigation holdrsquo to ensure the preservation of relevant documentsrdquo Zubulake v UBS Warburg LLC 220 FRD 212 218 (SDNY 2003)

Federal Civil Procedure Analysis

though spoliation occurred before order compelling discovery) Similarly the duty to preserve evidence applies to a party who anticipates litigation even if litigation has not yet been commenced See THE SEDONA PRINCIPLES BEST PRACTICES RECOMMENDATIONS amp PRINCIPLES FOR ADDRESSING ELECTRONIC DOCUMENT PRODUCTION 70 cmt 14a (2d ed 2007)

The builder destroyed the emails on August 2 At that time the builder knew that litigation was a possibility because the builder had already directed its attorney to prepare a draft complaint for possible filing Knowing that litigation was a possibility the builder had a duty to take steps to preserve evidence including the emails in question See generally Fujitsu Ltd

Thus the builderrsquos destruction of potentially relevant emails at a time when it knew that litigation was a possibility was improper It had a duty to preserve evidence and it breached that duty

[NOTE Because courts have used different words to describe the test for when evidence must be preserved an examineersquos precise formulation of the test is not critical]

Point Two(b) (30) In determining appropriate sanctions for spoliation courts consider both the level of culpability of the spoliating party and the degree of prejudice the loss of evidence has caused the other party Here the builderrsquos destruction of evidence does not appear to have been willful nor is it likely to pose a significant obstacle to the customerrsquos defense Any sanctions imposed by the court should be modest

Federal courts have inherent power to control the litigation process and can sanction misbehavior including spoliation even when there has been no specific violation of the Federal Rules of Civil Procedure See generally Chambers v NASCO Inc 501 US 32 (1991) (discussing courtrsquos inherent power to control the litigation process) The range of available sanctions is broad It can include such sanctions as the payment of expenses incurred by the other party as a result of the destruction of the evidence an instruction to the jury authorizing it to draw an adverse inference from the destruction of the evidence a shifting of the burden of proof on the relevant issue or even judgment against the responsible party See eg Residential Funding Corp v DeGeorge Financial Corp 306 F3d 99 108 (2d Cir 2002) (adverse inference) Silvestri v General Motors Corp 271 F3d 583 593 (4th Cir 2001) (possibility of dismissal) Cf FED R CIV P 37(b)(2)(A) (listing remedies for failure to comply with discovery obligations)

In determining appropriate sanctions for spoliation courts consider both the level of culpability of the spoliating party and the degree of prejudice the loss of evidence has caused the other party Many courts impose severe sanctions (such as an adverse-inference instruction or the entry of judgment against the spoliating party) only when there is evidence of bad faith in the form of an intentional effort to hide information Eg Greyhound Lines Inc v Wade 485 F3d 1032 1035 (8th Cir 2007) (spoliation sanction requires intentional destruction out of desire ldquoto suppress the truthrdquo) However other courts have said that negligence in preserving evidence can

21

Federal Civil Procedure Analysis

support an adverse-inference instruction See Residential Funding 306 F3d at 108 (negligence enough under some circumstances)

Although a court might well order an evidentiary hearing on the issue of sanctions the facts presented do not seem appropriate for severe sanctions First the evidence was destroyed pursuant to the builderrsquos standard document retention plan and there is no evidence that the builder deliberately failed to suspend its usual procedures with the purpose of allowing the destruction of evidence Second the loss of this evidence will not severely hinder the customerrsquos presentation of his case The central issue is whether the foundation of the house was properly constructed If the construction job was poorly done the customer can present evidence derived from inspection of the premises to prove that point The customer can also depose witnesses about any issues that arose during construction

Under the circumstances a court is not likely to impose particularly severe sanctions although it might shift the burden to the builder to show that the foundation was properly constructed or it might require the builder to reimburse any expenses the customer incurs to discover and prove the facts about issues or disputes that arose during construction of the foundation

[NOTE The result reached by the examinee is less important than the examineersquos recognition that (a) a range of sanctions is available to the court and (b) the appropriate sanction depends both on the culpability of the builder and the prejudice suffered by the customer]

22

CRIMINAL LAW AND PROCEDURE ANALYSIS (Criminal Law and Procedure IIA amp D VE amp F)

ANALYSIS

Legal Problems

(1) Did charging the defendant with both theft and burglary constitute double jeopardy

(2) Did the jury instruction violate the due process clause either by relieving the prosecution of the burden of proving the element of intent or by shifting the burden to the defendant to disprove that element

(3) Did the sentence imposed in this case for the theft conviction unconstitutionally deprive the defendant of his right to a jury trial on the issue of the value of the stolen item

DISCUSSION

Summary

The trial court properly denied the defendantrsquos pretrial motion to dismiss the charges on double jeopardy grounds The defendant may be charged with and convicted of both theft and burglary Each of the charges has an element that the other does not Neither charge is a lesser-included offense nor are they multiplicitous Thus charging both theft and burglary does not violate double jeopardy

The jury instruction on the burglary charge was constitutionally flawed It could have been reasonably understood by the jury as either (1) an irrebuttable conclusive presumption (which relieved the prosecution of proving the element of intent and removed the issue from the jury) or (2) a rebuttable mandatory presumption (which unconstitutionally shifted the burden of proof on an element of a charged offense from the prosecution to the defendant)

Because the four-year sentence imposed by the judge was based on the judgersquos finding by a preponderance of the evidence that the value of the stolen ring exceeded $5000 the sentence violates the defendantrsquos right to a jury determination beyond a reasonable doubt of the value of the ring

Point One (30) Charging the defendant with theft and burglary did not constitute double jeopardy

The Double Jeopardy Clause of the Fifth Amendment provides that a person shall not be twice put in jeopardy for the ldquosame offenserdquo Thus the question is whether the elements of the theft charge are wholly contained in the burglary charge or vice versa If the elements of the lesser charge (theft) are not wholly contained in the greater charge (burglary)mdashie if each charge requires proof of a fact that the other does notmdashthen convicting the defendant of both crimes would not violate double jeopardy even when the two offenses occurred at the same time and are thus arguably part of the ldquosame transactionrdquo Blockburger v United States 284 US 299 304 (1932) See also Albernaz v United States 450 US 333 344 n3 (1981) United States v Dixon 509 US 688 704 (1993)

23

Criminal Law and Procedure Analysis

Here theft and burglary each require proof of an element not required for the other crime Burglary may be defined differently in different jurisdictions However it almost invariably requires entry into a building or dwelling of another with the specific intent to commit a felony therein and the crime of burglary is complete upon the entry into the building or dwelling with such intent See eg Cannon v Oklahoma 827 P2d 1339 1342 (Okla Crim App 1992) In contrast theft which also may be defined differently in different states almost invariably requires the taking and carrying away of an item of personal property belonging to another with the intent to steal or permanently deprive the owner of possession

Here the ldquotakingrdquo or ldquostealingrdquo element is not contained in the definition of burglary and the ldquoentryrdquo element of burglary is not contained in the definition of theft Because theft is not a lesser-included offense of burglary and burglary is not a lesser-included offense of theft charging the defendant for both burglary and theft did not violate double jeopardy and the court properly denied the defense motion on those grounds Yparrea v Dorsey 64 F3d 577 579ndash80 (10th Cir 1995) citing Blockburger 284 US at 304

Finally the defendantrsquos motion to dismiss all the charges on double jeopardy grounds was improper because if both charges were for the same offense the motion should have requested dismissal of one charge not both

Point Two (35) The jury instruction on the burglary charge violated the Due Process Clause because it created either (1) an irrebuttable conclusive presumption (which relieved the prosecution of proving the element of intent and removed that issue from the jury) or (2) a rebuttable mandatory presumption (which unconstitutionally shifted the burden of proof on an element of a charged offense to the defendant)

The Supreme Court has interpreted the Due Process Clause of the US Constitution to require that the prosecution prove all elements of an offense beyond a reasonable doubt See In re Winship 397 US 358 364 (1970) The burden of proof cannot be shifted to the defendant by presuming an essential element upon proof of other elements of the offense because shifting the burden of persuasion with respect to any element of a criminal offense is contrary to the Due Process Clause See Mullaney v Wilbur 421 US 684 (1975)

The crime of burglary includes entry into a building or dwelling with the specific intent to commit a felony therein The requirement that the prosecutor prove beyond a reasonable doubt that the defendant had this specific intent distinguishes burglary from general-intent crimes like trespass See Sandstrom v Montana 442 US 510 523 (1979)

Here the jury was instructed that if ldquoafter consideration of all the evidence presented by the prosecution and defense you find beyond a reasonable doubt that the defendant entered the dwelling without the ownersrsquo consent you may presume that the defendant entered with the intent to commit a felony thereinrdquo This instruction was unconstitutional because it created either an irrebuttable conclusive presumption or a rebuttable mandatory presumption

A conclusive presumption is ldquoan irrebuttable direction by the court to find intent once convinced of the facts triggering the presumptionrdquo Id at 517 Here the jurors were instructed that once the prosecutor established that the defendant entered the neighborsrsquo house without consent they ldquomay presumerdquo that he intended to commit a felony therein The jurors may have reasonably concluded from this instruction that if they found that the defendant intended to enter his neighborsrsquo home without permission they must further find that he entered with the specific intent to commit a felony therein Because this instruction could operate as a conclusive

24

Criminal Law and Procedure Analysis

irrebuttable presumption by eliminating intent ldquoas an ingredient of the offenserdquo it violated due process by relieving the prosecution of the burden of proof for this element Id at 522

In the alternative the jury instruction could have been reasonably understood to create a rebuttable mandatory presumption which ldquotells [the jury] they must find the elemental fact upon proof of the basic fact at least unless the defendant has come forward with some evidence to rebut the presumed connection between the two factsrdquo County Court of Ulster County New York v Allen 442 US 140 157 (1979) The due process problem created by rebuttable mandatory presumptions is that ldquo[t]o the extent that the trier of fact is forced to abide by the presumption and may not reject it based on an independent evaluation of the particular facts presented by the State the analysis of the presumptionrsquos constitutional validity is logically divorced from those facts and based on the presumptionrsquos accuracy in the run of casesrdquo Id at 159

Unlike irrebuttable conclusive presumptions rebuttable mandatory presumptions are not always per se violations of the Due Process Clause However the Supreme Court of the United States has held that jury instructions that could reasonably be understood as shifting the burden of proof to the defendant on an element of the offense are unconstitutional Francis v Franklin 471 US 307 (1985) Here the argument that the jury instruction operated as a rebuttable mandatory presumption is supported by the fact that the judge also instructed the jury to ldquoconsider[ ] all the evidence presented by the prosecution and defenserdquo However even if the instruction created a rebuttable mandatory presumption it would be unconstitutional because it shifted the burden to the defense on an element of the offense Sandstrom 442 US at 524 Mullaney 421 US at 686

[NOTE Whether an examinee identifies the jury instruction as containing a ldquoconclusiverdquo or ldquomandatoryrdquo presumption is less important than the examineersquos analysis of the constitutional infirmities]

Point Three (35) The trial court violated the defendantrsquos Sixth Amendment right to a jury trial on an essential element of the offense when it found by a preponderance of the evidence that the ring was worth over $5000 and increased the defendantrsquos sentence based on this finding

In the statutory scheme under which the defendant was tried and convicted a Class D felony theft is defined as theft of item(s) with a value between $2500 and $10000 The jury found that the value of the diamond ring was at least $2500 and convicted the defendant of felony theft However at sentencing the trial court made a separate finding by a preponderance of the evidence that the value of the ring was greater than $5000 Following the statutersquos two-tiered sentencing scheme the judge then imposed on the defendant a sentence that was one year longer than the maximum that would otherwise have been allowed

The judgersquos sentence was unconstitutional because it violated the defendantrsquos Sixth Amendment right to a jury trial on this question The Supreme Court held in Apprendi v New Jersey 530 US 466 (2000) that ldquo[o]ther than the fact of a prior conviction any fact that increases the penalty for a crime beyond the prescribed statutory maximum must be submitted to a jury and proved beyond a reasonable doubtrdquo because ldquo[i]t is unconstitutional for a legislature to remove from the jury the assessment of facts that increase the prescribed range of penalties to which a criminal defendant is exposed [because] such facts must be established by proof beyond a reasonable doubtrdquo Id The Court reaffirmed Apprendi in Blakely v Washington 542 US 296 (2004) holding that the ldquolsquostatutory maximumrsquo for Apprendi purposes is the maximum sentence a judge may impose solely on the basis of the facts reflected in the jury verdict or admitted by the defendantrdquo Id at 303 (emphasis in original) In United States v Booker 543 US 220 (2005)

25

Criminal Law and Procedure Analysis

the Court relied on Blakely and Apprendi to conclude that protecting a defendantrsquos Sixth Amendment right to a jury trial required that ldquo[a]ny fact which is necessary to support a sentence exceeding the maximum authorized by the facts established by a plea of guilty or a jury verdict must be admitted by the defendant or proved to a jury beyond a reasonable doubtrdquo Id at 244

Thus in order to constitutionally increase a sentence above the statutory maximum of three years the jury must have found beyond a reasonable doubt that the value of the ring exceeded $5000 Here the court made the finding based on an appraisal proffered by the prosecutor only at sentencing and the judgersquos finding was by a preponderance of the evidence rather than beyond a reasonable doubt

26

AGENCY AND PARTNERSHIP ANALYSIS __________ (Agency and Partnership VA amp C VI)

ANALYSIS

Legal Problems

(1) Is a partner in a general partnership personally liable on a claim arising from misrepresentations by another partner made in the course of the partnership business

(2) Does a newly admitted partner in a general partnership become personally liable on existing claims against the partnership

(3) After the filing by a general partnership of a statement of qualification as a limited liability partnership are the partners personally liable as partners on (a) an existing claim against the general partnership and (b) a claim against the partnership that arose after the filing

DISCUSSION

Summary

Adam and Ben formed a general partnership under which they were jointly and severally liable for obligations of the partnership Thus Adam was personally liable for misrepresentations by Ben made in the ordinary course of the partnership business

Upon joining the general partnership Diane became personally liable for the obligations of the partnership arising after her admission but not for obligations pre-existing her admission such as the collectorrsquos claim

By filing a statement of qualification the three partners properly elected limited liability partnership status As partners in an LLP none of the three partners is personally liable as a partner for partnership obligations arising after the election such as the claim by the driverrsquos estate The election however does not change their personal liability on pre-existing claims that arose before the election such as the collectorrsquos claim

Point One (30) As a general partner of Empire a general partnership Adam became personally liable on the collectorrsquos claim a valid claim against the partnership that arose because of Benrsquos wrongful act in the ordinary course of the partnership business

When the collectorrsquos claim arose Empire was a general partnership composed of Adam and Ben Under UPA (1997) sect 306(a) partners of a general partnership are liable jointly and severally for all obligations of the partnership Under UPA (1997) sect 305(a) the partnership could become obligated for the loss caused to the collector as a result of the misrepresentation by Ben provided he was acting in the ordinary course of the partnership business Because there was no statement that limited his partnership authority Ben as partner was ldquoan agent of the partnership for the purpose of its businessrdquo See UPA (1997) sect 301(1) Benrsquos misrepresentation to the collector even if intentional appears to be in the ordinary course of the partnershiprsquos business of dealing

27

Agency and Partnership Analysis

in antique cars Thus Benrsquos wrongful act created a partnership obligation for which Adam was jointly and severally liable

[NOTE Generally a partnership creditor must ldquoexhaust the partnershiprsquos assets before levying on a judgment debtor partnerrsquos individual property where the partner is personally liable for the partnership obligationrdquo as a result of his status as a partner UPA (1997) sect 307 cmt 4 As the UPA comments explain this places Adam more in the position of guarantor than principal debtor on the partnership obligation Id cmt 4 Although an examinee might discuss this point the call focuses on whether Adam is personally liable not how the liability might be enforced]

Point Two (30) Because the collectorrsquos claim arose before Diane joined Empire Diane did not become personally liable on the claim

Diane was admitted to Empire when it was a general partnership and after the collectorrsquos claim arose While the general rule under UPA (1997) sect 306(a) is that the partners of a general partnership are liable jointly and severally for all obligations of the partnership there is a special rule for partners who are admitted during the duration of the partnership Under UPA (1997) sect 306(b) a person admitted to an existing partnership is not personally liable for any partnership obligations incurred before the personrsquos admission Because Diane was admitted to Empire after the collectorrsquos claim arose Diane is not personally liable on the claim

Dianersquos knowledge of the pre-existing claim and her stated concern about becoming liable on the collectorrsquos claim do not change her personal nonliability to the collector Although partners who have a liability shield can assume liability to third parties through private contractual guarantees or modifications to the partnership agreement Dianersquos stated concern constituted neither a guaranty to the collector nor ldquoan intentional waiver of liability protectionsrdquo See UPA (1997) sect 306 cmt 3 (describing methods for waiver of liability protections under sect 306(c) applicable in limited liability partnerships)

At most Diane will lose her investment in the partnership as a result of the collectorrsquos claim Although Diane did not become personally liable on the collectorrsquos claim when she joined the partnership the $250000 she contributed to the partnership is ldquoat risk for the satisfaction of existing partnership debtsrdquo UPA (1997) sect 306 cmt 2

Point Three (40) Filing the statement of qualification was effective to elect limited liability partnership status Despite this new status Adam and Ben remain personally liable on the collectorrsquos claim which arose before the election But as partners in an LLP neither Adam Ben nor Diane is personally liable as a partner on the driverrsquos estatersquos claim which arose after the election

Under UPA (1997) sect 1001 a general partnership can make an election and become a limited liability partnershipmdashif the partners approve the conversion by a vote equivalent to that necessary to amend the partnership agreement and the partnership then files a statement of qualification that specifies the name of the partnership its principal office and its election to be an LLP Here the partners agreed unanimouslymdashsufficient to amend their agreement under UPA (1997) sect 401(j)mdashand the statement of qualification was filed In addition the name of Empire LLP properly included an appropriate ending ldquoLLPrdquo See UPA (1997) sect 1002

Although another way to effectuate a ldquoconversionrdquo (as suggested by Benrsquos lawyer) is to form a new LLP and transfer the assets of the old general partnership to the new LLP the

28

Agency and Partnership Analysis

method used here (approval by the partners and the filing of a statement of qualification) is also sufficient to create LLP status

Thus Empire became Empire LLP as of the date of filing of the statement of qualification See UPA (1997) sect 1001 What effect did this have on the collectorrsquos claim which predated the filing According to UPA (1997) sect 306(c) an obligation incurred while a partnership is an LLP is solely a partnership obligation As the collectorrsquos claim predated the LLP Adam and Ben remain personally liable on the collectorrsquos claim Diane on the other hand was not personally liable on the collectorrsquos claim either before or after the filing of the statement of qualification See Point Two above

The driverrsquos estatersquos claim arose after Empire became Empire LLP Under UPA (1997) sect 306(c) an obligation incurred while a partnership is an LLP is solely a partnership obligationThus Adam Ben and Diane as partners are all protected from personal liability on the driverrsquos estatersquos claim But there may be personal liability if any of them was negligent or otherwise acted wrongfully by not informing the buyer of the bad suspension that caused the accident

29

National Conference of Bar Examiners 302 South Bedford Street | Madison WI 53703-3622 Phone 608-280-8550 | Fax 608-280-8552 | TDD 608-661-1275

wwwncbexorg e-mail contactncbexorg

  • Preface
  • Description of the MEE
  • Instructions
  • February 2014 Questions
    • Constitutinal Law Question
    • Trusts and Future Interests Question
    • Secured Transactions Question
    • Federal Civil Procedure Question
    • Criminal Law and Procedure Question
    • Agency and Partnership Question
      • February 2014 Analyses
        • Constitutional Law Analysis
        • Trust and Future Interests Analysis
        • Secured Transactions Analysis
        • Federal Civil Procedure Analysis
        • Criminal Law and Procedure Analysis
        • Agency and Partnership Analysis
            • ltlt13 ASCII85EncodePages false13 AllowTransparency false13 AutoPositionEPSFiles true13 AutoRotatePages None13 Binding Left13 CalGrayProfile (Dot Gain 20)13 CalRGBProfile (sRGB IEC61966-21)13 CalCMYKProfile (US Web Coated 050SWOP051 v2)13 sRGBProfile (sRGB IEC61966-21)13 CannotEmbedFontPolicy Error13 CompatibilityLevel 1413 CompressObjects Tags13 CompressPages true13 ConvertImagesToIndexed true13 PassThroughJPEGImages true13 CreateJobTicket false13 DefaultRenderingIntent Default13 DetectBlends true13 DetectCurves 0000013 ColorConversionStrategy CMYK13 DoThumbnails false13 EmbedAllFonts true13 EmbedOpenType false13 ParseICCProfilesInComments true13 EmbedJobOptions true13 DSCReportingLevel 013 EmitDSCWarnings false13 EndPage -113 ImageMemory 104857613 LockDistillerParams false13 MaxSubsetPct 10013 Optimize true13 OPM 113 ParseDSCComments true13 ParseDSCCommentsForDocInfo true13 PreserveCopyPage true13 PreserveDICMYKValues true13 PreserveEPSInfo true13 PreserveFlatness true13 PreserveHalftoneInfo false13 PreserveOPIComments true13 PreserveOverprintSettings true13 StartPage 113 SubsetFonts true13 TransferFunctionInfo Apply13 UCRandBGInfo Preserve13 UsePrologue false13 ColorSettingsFile ()13 AlwaysEmbed [ true13 ]13 NeverEmbed [ true13 ]13 AntiAliasColorImages false13 CropColorImages true13 ColorImageMinResolution 30013 ColorImageMinResolutionPolicy OK13 DownsampleColorImages true13 ColorImageDownsampleType Bicubic13 ColorImageResolution 30013 ColorImageDepth -113 ColorImageMinDownsampleDepth 113 ColorImageDownsampleThreshold 15000013 EncodeColorImages true13 ColorImageFilter DCTEncode13 AutoFilterColorImages true13 ColorImageAutoFilterStrategy JPEG13 ColorACSImageDict ltlt13 QFactor 01513 HSamples [1 1 1 1] VSamples [1 1 1 1]13 gtgt13 ColorImageDict ltlt13 QFactor 01513 HSamples [1 1 1 1] VSamples [1 1 1 1]13 gtgt13 JPEG2000ColorACSImageDict ltlt13 TileWidth 25613 TileHeight 25613 Quality 3013 gtgt13 JPEG2000ColorImageDict ltlt13 TileWidth 25613 TileHeight 25613 Quality 3013 gtgt13 AntiAliasGrayImages false13 CropGrayImages true13 GrayImageMinResolution 30013 GrayImageMinResolutionPolicy OK13 DownsampleGrayImages true13 GrayImageDownsampleType Bicubic13 GrayImageResolution 30013 GrayImageDepth -113 GrayImageMinDownsampleDepth 213 GrayImageDownsampleThreshold 15000013 EncodeGrayImages true13 GrayImageFilter DCTEncode13 AutoFilterGrayImages true13 GrayImageAutoFilterStrategy JPEG13 GrayACSImageDict ltlt13 QFactor 01513 HSamples [1 1 1 1] VSamples [1 1 1 1]13 gtgt13 GrayImageDict ltlt13 QFactor 01513 HSamples [1 1 1 1] VSamples [1 1 1 1]13 gtgt13 JPEG2000GrayACSImageDict ltlt13 TileWidth 25613 TileHeight 25613 Quality 3013 gtgt13 JPEG2000GrayImageDict ltlt13 TileWidth 25613 TileHeight 25613 Quality 3013 gtgt13 AntiAliasMonoImages false13 CropMonoImages true13 MonoImageMinResolution 120013 MonoImageMinResolutionPolicy OK13 DownsampleMonoImages true13 MonoImageDownsampleType Bicubic13 MonoImageResolution 120013 MonoImageDepth -113 MonoImageDownsampleThreshold 15000013 EncodeMonoImages true13 MonoImageFilter CCITTFaxEncode13 MonoImageDict ltlt13 K -113 gtgt13 AllowPSXObjects false13 CheckCompliance [13 None13 ]13 PDFX1aCheck false13 PDFX3Check false13 PDFXCompliantPDFOnly false13 PDFXNoTrimBoxError true13 PDFXTrimBoxToMediaBoxOffset [13 00000013 00000013 00000013 00000013 ]13 PDFXSetBleedBoxToMediaBox true13 PDFXBleedBoxToTrimBoxOffset [13 00000013 00000013 00000013 00000013 ]13 PDFXOutputIntentProfile ()13 PDFXOutputConditionIdentifier ()13 PDFXOutputCondition ()13 PDFXRegistryName ()13 PDFXTrapped False1313 CreateJDFFile false13 Description ltlt13 ARA ltFEFF06270633062A062E062F0645002006470630064700200627064406250639062F0627062F0627062A002006440625064606340627062100200648062B062706260642002000410064006F00620065002000500044004600200645062A064806270641064206290020064406440637062806270639062900200641064A00200627064406450637062706280639002006300627062A0020062F0631062C0627062A002006270644062C0648062F0629002006270644063906270644064A0629061B0020064A06450643064600200641062A062D00200648062B0627062606420020005000440046002006270644064506460634062306290020062806270633062A062E062F062706450020004100630072006F0062006100740020064800410064006F006200650020005200650061006400650072002006250635062F0627063100200035002E0030002006480627064406250635062F062706310627062A0020062706440623062D062F062B002E0635062F0627063100200035002E0030002006480627064406250635062F062706310627062A0020062706440623062D062F062B002Egt13 BGR 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 CHS ltFEFF4f7f75288fd94e9b8bbe5b9a521b5efa7684002000410064006f006200650020005000440046002065876863900275284e8e9ad88d2891cf76845370524d53705237300260a853ef4ee54f7f75280020004100630072006f0062006100740020548c002000410064006f00620065002000520065006100640065007200200035002e003000204ee553ca66f49ad87248672c676562535f00521b5efa768400200050004400460020658768633002gt13 CHT ltFEFF4f7f752890194e9b8a2d7f6e5efa7acb7684002000410064006f006200650020005000440046002065874ef69069752865bc9ad854c18cea76845370524d5370523786557406300260a853ef4ee54f7f75280020004100630072006f0062006100740020548c002000410064006f00620065002000520065006100640065007200200035002e003000204ee553ca66f49ad87248672c4f86958b555f5df25efa7acb76840020005000440046002065874ef63002gt13 CZE 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 DAN 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 DEU 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 ESP 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 ETI 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 FRA 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 GRE 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 HEB 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 HRV (Za stvaranje Adobe PDF dokumenata najpogodnijih za visokokvalitetni ispis prije tiskanja koristite ove postavke Stvoreni PDF dokumenti mogu se otvoriti Acrobat i Adobe Reader 50 i kasnijim verzijama)13 HUN 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 ITA 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 JPN ltFEFF9ad854c18cea306a30d730ea30d730ec30b951fa529b7528002000410064006f0062006500200050004400460020658766f8306e4f5c6210306b4f7f75283057307e305930023053306e8a2d5b9a30674f5c62103055308c305f0020005000440046002030d530a130a430eb306f3001004100630072006f0062006100740020304a30883073002000410064006f00620065002000520065006100640065007200200035002e003000204ee5964d3067958b304f30533068304c3067304d307e305930023053306e8a2d5b9a306b306f30d530a930f330c8306e57cb30818fbc307f304c5fc59808306730593002gt13 KOR ltFEFFc7740020c124c815c7440020c0acc6a9d558c5ec0020ace0d488c9c80020c2dcd5d80020c778c1c4c5d00020ac00c7a50020c801d569d55c002000410064006f0062006500200050004400460020bb38c11cb97c0020c791c131d569b2c8b2e4002e0020c774b807ac8c0020c791c131b41c00200050004400460020bb38c11cb2940020004100630072006f0062006100740020bc0f002000410064006f00620065002000520065006100640065007200200035002e00300020c774c0c1c5d0c11c0020c5f40020c2180020c788c2b5b2c8b2e4002egt13 LTH 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 LVI 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 NLD (Gebruik deze instellingen om Adobe PDF-documenten te maken die zijn geoptimaliseerd voor prepress-afdrukken van hoge kwaliteit De gemaakte PDF-documenten kunnen worden geopend met Acrobat en Adobe Reader 50 en hoger)13 NOR 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 POL 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 PTB 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 RUM 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 RUS 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 SKY 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 SLV 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 SUO 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 SVE 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 TUR ltFEFF005900fc006b00730065006b0020006b0061006c006900740065006c0069002000f6006e002000790061007a006401310072006d00610020006200610073006b013100730131006e006100200065006e0020006900790069002000750079006100620069006c006500630065006b002000410064006f006200650020005000440046002000620065006c00670065006c0065007200690020006f006c0075015f007400750072006d0061006b0020006900e70069006e00200062007500200061007900610072006c0061007201310020006b0075006c006c0061006e0131006e002e00200020004f006c0075015f0074007500720075006c0061006e0020005000440046002000620065006c00670065006c0065007200690020004100630072006f006200610074002000760065002000410064006f00620065002000520065006100640065007200200035002e003000200076006500200073006f006e0072006100730131006e00640061006b00690020007300fc007200fc006d006c00650072006c00650020006100e70131006c006100620069006c00690072002egt13 UKR 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 ENU (Use these settings to create Adobe PDF documents best suited for high-quality prepress printing Created PDF documents can be opened with Acrobat and Adobe Reader 50 and later)13 gtgt13 Namespace [13 (Adobe)13 (Common)13 (10)13 ]13 OtherNamespaces [13 ltlt13 AsReaderSpreads false13 CropImagesToFrames true13 ErrorControl WarnAndContinue13 FlattenerIgnoreSpreadOverrides false13 IncludeGuidesGrids false13 IncludeNonPrinting false13 IncludeSlug false13 Namespace [13 (Adobe)13 (InDesign)13 (40)13 ]13 OmitPlacedBitmaps false13 OmitPlacedEPS false13 OmitPlacedPDF false13 SimulateOverprint Legacy13 gtgt13 ltlt13 AddBleedMarks false13 AddColorBars false13 AddCropMarks false13 AddPageInfo false13 AddRegMarks false13 ConvertColors ConvertToCMYK13 DestinationProfileName ()13 DestinationProfileSelector DocumentCMYK13 Downsample16BitImages true13 FlattenerPreset ltlt13 PresetSelector MediumResolution13 gtgt13 FormElements false13 GenerateStructure false13 IncludeBookmarks false13 IncludeHyperlinks false13 IncludeInteractive false13 IncludeLayers false13 IncludeProfiles false13 MultimediaHandling UseObjectSettings13 Namespace [13 (Adobe)13 (CreativeSuite)13 (20)13 ]13 PDFXOutputIntentProfileSelector DocumentCMYK13 PreserveEditing true13 UntaggedCMYKHandling LeaveUntagged13 UntaggedRGBHandling UseDocumentProfile13 UseDocumentBleed false13 gtgt13 ]13gtgt setdistillerparams13ltlt13 HWResolution [2400 2400]13 PageSize [612000 792000]13gtgt setpagedevice13

Page 25: February 2014 MEE Questions and AnalysesPreface The Multistate Essay Examination (MEE) is developed by the National Conference of Bar Examiners (NCBE). This publication includes the

Federal Civil Procedure Analysis

support an adverse-inference instruction See Residential Funding 306 F3d at 108 (negligence enough under some circumstances)

Although a court might well order an evidentiary hearing on the issue of sanctions the facts presented do not seem appropriate for severe sanctions First the evidence was destroyed pursuant to the builderrsquos standard document retention plan and there is no evidence that the builder deliberately failed to suspend its usual procedures with the purpose of allowing the destruction of evidence Second the loss of this evidence will not severely hinder the customerrsquos presentation of his case The central issue is whether the foundation of the house was properly constructed If the construction job was poorly done the customer can present evidence derived from inspection of the premises to prove that point The customer can also depose witnesses about any issues that arose during construction

Under the circumstances a court is not likely to impose particularly severe sanctions although it might shift the burden to the builder to show that the foundation was properly constructed or it might require the builder to reimburse any expenses the customer incurs to discover and prove the facts about issues or disputes that arose during construction of the foundation

[NOTE The result reached by the examinee is less important than the examineersquos recognition that (a) a range of sanctions is available to the court and (b) the appropriate sanction depends both on the culpability of the builder and the prejudice suffered by the customer]

22

CRIMINAL LAW AND PROCEDURE ANALYSIS (Criminal Law and Procedure IIA amp D VE amp F)

ANALYSIS

Legal Problems

(1) Did charging the defendant with both theft and burglary constitute double jeopardy

(2) Did the jury instruction violate the due process clause either by relieving the prosecution of the burden of proving the element of intent or by shifting the burden to the defendant to disprove that element

(3) Did the sentence imposed in this case for the theft conviction unconstitutionally deprive the defendant of his right to a jury trial on the issue of the value of the stolen item

DISCUSSION

Summary

The trial court properly denied the defendantrsquos pretrial motion to dismiss the charges on double jeopardy grounds The defendant may be charged with and convicted of both theft and burglary Each of the charges has an element that the other does not Neither charge is a lesser-included offense nor are they multiplicitous Thus charging both theft and burglary does not violate double jeopardy

The jury instruction on the burglary charge was constitutionally flawed It could have been reasonably understood by the jury as either (1) an irrebuttable conclusive presumption (which relieved the prosecution of proving the element of intent and removed the issue from the jury) or (2) a rebuttable mandatory presumption (which unconstitutionally shifted the burden of proof on an element of a charged offense from the prosecution to the defendant)

Because the four-year sentence imposed by the judge was based on the judgersquos finding by a preponderance of the evidence that the value of the stolen ring exceeded $5000 the sentence violates the defendantrsquos right to a jury determination beyond a reasonable doubt of the value of the ring

Point One (30) Charging the defendant with theft and burglary did not constitute double jeopardy

The Double Jeopardy Clause of the Fifth Amendment provides that a person shall not be twice put in jeopardy for the ldquosame offenserdquo Thus the question is whether the elements of the theft charge are wholly contained in the burglary charge or vice versa If the elements of the lesser charge (theft) are not wholly contained in the greater charge (burglary)mdashie if each charge requires proof of a fact that the other does notmdashthen convicting the defendant of both crimes would not violate double jeopardy even when the two offenses occurred at the same time and are thus arguably part of the ldquosame transactionrdquo Blockburger v United States 284 US 299 304 (1932) See also Albernaz v United States 450 US 333 344 n3 (1981) United States v Dixon 509 US 688 704 (1993)

23

Criminal Law and Procedure Analysis

Here theft and burglary each require proof of an element not required for the other crime Burglary may be defined differently in different jurisdictions However it almost invariably requires entry into a building or dwelling of another with the specific intent to commit a felony therein and the crime of burglary is complete upon the entry into the building or dwelling with such intent See eg Cannon v Oklahoma 827 P2d 1339 1342 (Okla Crim App 1992) In contrast theft which also may be defined differently in different states almost invariably requires the taking and carrying away of an item of personal property belonging to another with the intent to steal or permanently deprive the owner of possession

Here the ldquotakingrdquo or ldquostealingrdquo element is not contained in the definition of burglary and the ldquoentryrdquo element of burglary is not contained in the definition of theft Because theft is not a lesser-included offense of burglary and burglary is not a lesser-included offense of theft charging the defendant for both burglary and theft did not violate double jeopardy and the court properly denied the defense motion on those grounds Yparrea v Dorsey 64 F3d 577 579ndash80 (10th Cir 1995) citing Blockburger 284 US at 304

Finally the defendantrsquos motion to dismiss all the charges on double jeopardy grounds was improper because if both charges were for the same offense the motion should have requested dismissal of one charge not both

Point Two (35) The jury instruction on the burglary charge violated the Due Process Clause because it created either (1) an irrebuttable conclusive presumption (which relieved the prosecution of proving the element of intent and removed that issue from the jury) or (2) a rebuttable mandatory presumption (which unconstitutionally shifted the burden of proof on an element of a charged offense to the defendant)

The Supreme Court has interpreted the Due Process Clause of the US Constitution to require that the prosecution prove all elements of an offense beyond a reasonable doubt See In re Winship 397 US 358 364 (1970) The burden of proof cannot be shifted to the defendant by presuming an essential element upon proof of other elements of the offense because shifting the burden of persuasion with respect to any element of a criminal offense is contrary to the Due Process Clause See Mullaney v Wilbur 421 US 684 (1975)

The crime of burglary includes entry into a building or dwelling with the specific intent to commit a felony therein The requirement that the prosecutor prove beyond a reasonable doubt that the defendant had this specific intent distinguishes burglary from general-intent crimes like trespass See Sandstrom v Montana 442 US 510 523 (1979)

Here the jury was instructed that if ldquoafter consideration of all the evidence presented by the prosecution and defense you find beyond a reasonable doubt that the defendant entered the dwelling without the ownersrsquo consent you may presume that the defendant entered with the intent to commit a felony thereinrdquo This instruction was unconstitutional because it created either an irrebuttable conclusive presumption or a rebuttable mandatory presumption

A conclusive presumption is ldquoan irrebuttable direction by the court to find intent once convinced of the facts triggering the presumptionrdquo Id at 517 Here the jurors were instructed that once the prosecutor established that the defendant entered the neighborsrsquo house without consent they ldquomay presumerdquo that he intended to commit a felony therein The jurors may have reasonably concluded from this instruction that if they found that the defendant intended to enter his neighborsrsquo home without permission they must further find that he entered with the specific intent to commit a felony therein Because this instruction could operate as a conclusive

24

Criminal Law and Procedure Analysis

irrebuttable presumption by eliminating intent ldquoas an ingredient of the offenserdquo it violated due process by relieving the prosecution of the burden of proof for this element Id at 522

In the alternative the jury instruction could have been reasonably understood to create a rebuttable mandatory presumption which ldquotells [the jury] they must find the elemental fact upon proof of the basic fact at least unless the defendant has come forward with some evidence to rebut the presumed connection between the two factsrdquo County Court of Ulster County New York v Allen 442 US 140 157 (1979) The due process problem created by rebuttable mandatory presumptions is that ldquo[t]o the extent that the trier of fact is forced to abide by the presumption and may not reject it based on an independent evaluation of the particular facts presented by the State the analysis of the presumptionrsquos constitutional validity is logically divorced from those facts and based on the presumptionrsquos accuracy in the run of casesrdquo Id at 159

Unlike irrebuttable conclusive presumptions rebuttable mandatory presumptions are not always per se violations of the Due Process Clause However the Supreme Court of the United States has held that jury instructions that could reasonably be understood as shifting the burden of proof to the defendant on an element of the offense are unconstitutional Francis v Franklin 471 US 307 (1985) Here the argument that the jury instruction operated as a rebuttable mandatory presumption is supported by the fact that the judge also instructed the jury to ldquoconsider[ ] all the evidence presented by the prosecution and defenserdquo However even if the instruction created a rebuttable mandatory presumption it would be unconstitutional because it shifted the burden to the defense on an element of the offense Sandstrom 442 US at 524 Mullaney 421 US at 686

[NOTE Whether an examinee identifies the jury instruction as containing a ldquoconclusiverdquo or ldquomandatoryrdquo presumption is less important than the examineersquos analysis of the constitutional infirmities]

Point Three (35) The trial court violated the defendantrsquos Sixth Amendment right to a jury trial on an essential element of the offense when it found by a preponderance of the evidence that the ring was worth over $5000 and increased the defendantrsquos sentence based on this finding

In the statutory scheme under which the defendant was tried and convicted a Class D felony theft is defined as theft of item(s) with a value between $2500 and $10000 The jury found that the value of the diamond ring was at least $2500 and convicted the defendant of felony theft However at sentencing the trial court made a separate finding by a preponderance of the evidence that the value of the ring was greater than $5000 Following the statutersquos two-tiered sentencing scheme the judge then imposed on the defendant a sentence that was one year longer than the maximum that would otherwise have been allowed

The judgersquos sentence was unconstitutional because it violated the defendantrsquos Sixth Amendment right to a jury trial on this question The Supreme Court held in Apprendi v New Jersey 530 US 466 (2000) that ldquo[o]ther than the fact of a prior conviction any fact that increases the penalty for a crime beyond the prescribed statutory maximum must be submitted to a jury and proved beyond a reasonable doubtrdquo because ldquo[i]t is unconstitutional for a legislature to remove from the jury the assessment of facts that increase the prescribed range of penalties to which a criminal defendant is exposed [because] such facts must be established by proof beyond a reasonable doubtrdquo Id The Court reaffirmed Apprendi in Blakely v Washington 542 US 296 (2004) holding that the ldquolsquostatutory maximumrsquo for Apprendi purposes is the maximum sentence a judge may impose solely on the basis of the facts reflected in the jury verdict or admitted by the defendantrdquo Id at 303 (emphasis in original) In United States v Booker 543 US 220 (2005)

25

Criminal Law and Procedure Analysis

the Court relied on Blakely and Apprendi to conclude that protecting a defendantrsquos Sixth Amendment right to a jury trial required that ldquo[a]ny fact which is necessary to support a sentence exceeding the maximum authorized by the facts established by a plea of guilty or a jury verdict must be admitted by the defendant or proved to a jury beyond a reasonable doubtrdquo Id at 244

Thus in order to constitutionally increase a sentence above the statutory maximum of three years the jury must have found beyond a reasonable doubt that the value of the ring exceeded $5000 Here the court made the finding based on an appraisal proffered by the prosecutor only at sentencing and the judgersquos finding was by a preponderance of the evidence rather than beyond a reasonable doubt

26

AGENCY AND PARTNERSHIP ANALYSIS __________ (Agency and Partnership VA amp C VI)

ANALYSIS

Legal Problems

(1) Is a partner in a general partnership personally liable on a claim arising from misrepresentations by another partner made in the course of the partnership business

(2) Does a newly admitted partner in a general partnership become personally liable on existing claims against the partnership

(3) After the filing by a general partnership of a statement of qualification as a limited liability partnership are the partners personally liable as partners on (a) an existing claim against the general partnership and (b) a claim against the partnership that arose after the filing

DISCUSSION

Summary

Adam and Ben formed a general partnership under which they were jointly and severally liable for obligations of the partnership Thus Adam was personally liable for misrepresentations by Ben made in the ordinary course of the partnership business

Upon joining the general partnership Diane became personally liable for the obligations of the partnership arising after her admission but not for obligations pre-existing her admission such as the collectorrsquos claim

By filing a statement of qualification the three partners properly elected limited liability partnership status As partners in an LLP none of the three partners is personally liable as a partner for partnership obligations arising after the election such as the claim by the driverrsquos estate The election however does not change their personal liability on pre-existing claims that arose before the election such as the collectorrsquos claim

Point One (30) As a general partner of Empire a general partnership Adam became personally liable on the collectorrsquos claim a valid claim against the partnership that arose because of Benrsquos wrongful act in the ordinary course of the partnership business

When the collectorrsquos claim arose Empire was a general partnership composed of Adam and Ben Under UPA (1997) sect 306(a) partners of a general partnership are liable jointly and severally for all obligations of the partnership Under UPA (1997) sect 305(a) the partnership could become obligated for the loss caused to the collector as a result of the misrepresentation by Ben provided he was acting in the ordinary course of the partnership business Because there was no statement that limited his partnership authority Ben as partner was ldquoan agent of the partnership for the purpose of its businessrdquo See UPA (1997) sect 301(1) Benrsquos misrepresentation to the collector even if intentional appears to be in the ordinary course of the partnershiprsquos business of dealing

27

Agency and Partnership Analysis

in antique cars Thus Benrsquos wrongful act created a partnership obligation for which Adam was jointly and severally liable

[NOTE Generally a partnership creditor must ldquoexhaust the partnershiprsquos assets before levying on a judgment debtor partnerrsquos individual property where the partner is personally liable for the partnership obligationrdquo as a result of his status as a partner UPA (1997) sect 307 cmt 4 As the UPA comments explain this places Adam more in the position of guarantor than principal debtor on the partnership obligation Id cmt 4 Although an examinee might discuss this point the call focuses on whether Adam is personally liable not how the liability might be enforced]

Point Two (30) Because the collectorrsquos claim arose before Diane joined Empire Diane did not become personally liable on the claim

Diane was admitted to Empire when it was a general partnership and after the collectorrsquos claim arose While the general rule under UPA (1997) sect 306(a) is that the partners of a general partnership are liable jointly and severally for all obligations of the partnership there is a special rule for partners who are admitted during the duration of the partnership Under UPA (1997) sect 306(b) a person admitted to an existing partnership is not personally liable for any partnership obligations incurred before the personrsquos admission Because Diane was admitted to Empire after the collectorrsquos claim arose Diane is not personally liable on the claim

Dianersquos knowledge of the pre-existing claim and her stated concern about becoming liable on the collectorrsquos claim do not change her personal nonliability to the collector Although partners who have a liability shield can assume liability to third parties through private contractual guarantees or modifications to the partnership agreement Dianersquos stated concern constituted neither a guaranty to the collector nor ldquoan intentional waiver of liability protectionsrdquo See UPA (1997) sect 306 cmt 3 (describing methods for waiver of liability protections under sect 306(c) applicable in limited liability partnerships)

At most Diane will lose her investment in the partnership as a result of the collectorrsquos claim Although Diane did not become personally liable on the collectorrsquos claim when she joined the partnership the $250000 she contributed to the partnership is ldquoat risk for the satisfaction of existing partnership debtsrdquo UPA (1997) sect 306 cmt 2

Point Three (40) Filing the statement of qualification was effective to elect limited liability partnership status Despite this new status Adam and Ben remain personally liable on the collectorrsquos claim which arose before the election But as partners in an LLP neither Adam Ben nor Diane is personally liable as a partner on the driverrsquos estatersquos claim which arose after the election

Under UPA (1997) sect 1001 a general partnership can make an election and become a limited liability partnershipmdashif the partners approve the conversion by a vote equivalent to that necessary to amend the partnership agreement and the partnership then files a statement of qualification that specifies the name of the partnership its principal office and its election to be an LLP Here the partners agreed unanimouslymdashsufficient to amend their agreement under UPA (1997) sect 401(j)mdashand the statement of qualification was filed In addition the name of Empire LLP properly included an appropriate ending ldquoLLPrdquo See UPA (1997) sect 1002

Although another way to effectuate a ldquoconversionrdquo (as suggested by Benrsquos lawyer) is to form a new LLP and transfer the assets of the old general partnership to the new LLP the

28

Agency and Partnership Analysis

method used here (approval by the partners and the filing of a statement of qualification) is also sufficient to create LLP status

Thus Empire became Empire LLP as of the date of filing of the statement of qualification See UPA (1997) sect 1001 What effect did this have on the collectorrsquos claim which predated the filing According to UPA (1997) sect 306(c) an obligation incurred while a partnership is an LLP is solely a partnership obligation As the collectorrsquos claim predated the LLP Adam and Ben remain personally liable on the collectorrsquos claim Diane on the other hand was not personally liable on the collectorrsquos claim either before or after the filing of the statement of qualification See Point Two above

The driverrsquos estatersquos claim arose after Empire became Empire LLP Under UPA (1997) sect 306(c) an obligation incurred while a partnership is an LLP is solely a partnership obligationThus Adam Ben and Diane as partners are all protected from personal liability on the driverrsquos estatersquos claim But there may be personal liability if any of them was negligent or otherwise acted wrongfully by not informing the buyer of the bad suspension that caused the accident

29

National Conference of Bar Examiners 302 South Bedford Street | Madison WI 53703-3622 Phone 608-280-8550 | Fax 608-280-8552 | TDD 608-661-1275

wwwncbexorg e-mail contactncbexorg

  • Preface
  • Description of the MEE
  • Instructions
  • February 2014 Questions
    • Constitutinal Law Question
    • Trusts and Future Interests Question
    • Secured Transactions Question
    • Federal Civil Procedure Question
    • Criminal Law and Procedure Question
    • Agency and Partnership Question
      • February 2014 Analyses
        • Constitutional Law Analysis
        • Trust and Future Interests Analysis
        • Secured Transactions Analysis
        • Federal Civil Procedure Analysis
        • Criminal Law and Procedure Analysis
        • Agency and Partnership Analysis
            • ltlt13 ASCII85EncodePages false13 AllowTransparency false13 AutoPositionEPSFiles true13 AutoRotatePages None13 Binding Left13 CalGrayProfile (Dot Gain 20)13 CalRGBProfile (sRGB IEC61966-21)13 CalCMYKProfile (US Web Coated 050SWOP051 v2)13 sRGBProfile (sRGB IEC61966-21)13 CannotEmbedFontPolicy Error13 CompatibilityLevel 1413 CompressObjects Tags13 CompressPages true13 ConvertImagesToIndexed true13 PassThroughJPEGImages true13 CreateJobTicket false13 DefaultRenderingIntent Default13 DetectBlends true13 DetectCurves 0000013 ColorConversionStrategy CMYK13 DoThumbnails false13 EmbedAllFonts true13 EmbedOpenType false13 ParseICCProfilesInComments true13 EmbedJobOptions true13 DSCReportingLevel 013 EmitDSCWarnings false13 EndPage -113 ImageMemory 104857613 LockDistillerParams false13 MaxSubsetPct 10013 Optimize true13 OPM 113 ParseDSCComments true13 ParseDSCCommentsForDocInfo true13 PreserveCopyPage true13 PreserveDICMYKValues true13 PreserveEPSInfo true13 PreserveFlatness true13 PreserveHalftoneInfo false13 PreserveOPIComments true13 PreserveOverprintSettings true13 StartPage 113 SubsetFonts true13 TransferFunctionInfo Apply13 UCRandBGInfo Preserve13 UsePrologue false13 ColorSettingsFile ()13 AlwaysEmbed [ true13 ]13 NeverEmbed [ true13 ]13 AntiAliasColorImages false13 CropColorImages true13 ColorImageMinResolution 30013 ColorImageMinResolutionPolicy OK13 DownsampleColorImages true13 ColorImageDownsampleType Bicubic13 ColorImageResolution 30013 ColorImageDepth -113 ColorImageMinDownsampleDepth 113 ColorImageDownsampleThreshold 15000013 EncodeColorImages true13 ColorImageFilter DCTEncode13 AutoFilterColorImages true13 ColorImageAutoFilterStrategy JPEG13 ColorACSImageDict ltlt13 QFactor 01513 HSamples [1 1 1 1] VSamples [1 1 1 1]13 gtgt13 ColorImageDict ltlt13 QFactor 01513 HSamples [1 1 1 1] VSamples [1 1 1 1]13 gtgt13 JPEG2000ColorACSImageDict ltlt13 TileWidth 25613 TileHeight 25613 Quality 3013 gtgt13 JPEG2000ColorImageDict ltlt13 TileWidth 25613 TileHeight 25613 Quality 3013 gtgt13 AntiAliasGrayImages false13 CropGrayImages true13 GrayImageMinResolution 30013 GrayImageMinResolutionPolicy OK13 DownsampleGrayImages true13 GrayImageDownsampleType Bicubic13 GrayImageResolution 30013 GrayImageDepth -113 GrayImageMinDownsampleDepth 213 GrayImageDownsampleThreshold 15000013 EncodeGrayImages true13 GrayImageFilter DCTEncode13 AutoFilterGrayImages true13 GrayImageAutoFilterStrategy JPEG13 GrayACSImageDict ltlt13 QFactor 01513 HSamples [1 1 1 1] VSamples [1 1 1 1]13 gtgt13 GrayImageDict ltlt13 QFactor 01513 HSamples [1 1 1 1] VSamples [1 1 1 1]13 gtgt13 JPEG2000GrayACSImageDict ltlt13 TileWidth 25613 TileHeight 25613 Quality 3013 gtgt13 JPEG2000GrayImageDict ltlt13 TileWidth 25613 TileHeight 25613 Quality 3013 gtgt13 AntiAliasMonoImages false13 CropMonoImages true13 MonoImageMinResolution 120013 MonoImageMinResolutionPolicy OK13 DownsampleMonoImages true13 MonoImageDownsampleType Bicubic13 MonoImageResolution 120013 MonoImageDepth -113 MonoImageDownsampleThreshold 15000013 EncodeMonoImages true13 MonoImageFilter CCITTFaxEncode13 MonoImageDict ltlt13 K -113 gtgt13 AllowPSXObjects false13 CheckCompliance [13 None13 ]13 PDFX1aCheck false13 PDFX3Check false13 PDFXCompliantPDFOnly false13 PDFXNoTrimBoxError true13 PDFXTrimBoxToMediaBoxOffset [13 00000013 00000013 00000013 00000013 ]13 PDFXSetBleedBoxToMediaBox true13 PDFXBleedBoxToTrimBoxOffset [13 00000013 00000013 00000013 00000013 ]13 PDFXOutputIntentProfile ()13 PDFXOutputConditionIdentifier ()13 PDFXOutputCondition ()13 PDFXRegistryName ()13 PDFXTrapped False1313 CreateJDFFile false13 Description ltlt13 ARA 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 BGR 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 CHS ltFEFF4f7f75288fd94e9b8bbe5b9a521b5efa7684002000410064006f006200650020005000440046002065876863900275284e8e9ad88d2891cf76845370524d53705237300260a853ef4ee54f7f75280020004100630072006f0062006100740020548c002000410064006f00620065002000520065006100640065007200200035002e003000204ee553ca66f49ad87248672c676562535f00521b5efa768400200050004400460020658768633002gt13 CHT ltFEFF4f7f752890194e9b8a2d7f6e5efa7acb7684002000410064006f006200650020005000440046002065874ef69069752865bc9ad854c18cea76845370524d5370523786557406300260a853ef4ee54f7f75280020004100630072006f0062006100740020548c002000410064006f00620065002000520065006100640065007200200035002e003000204ee553ca66f49ad87248672c4f86958b555f5df25efa7acb76840020005000440046002065874ef63002gt13 CZE 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 DAN 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 DEU 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 ESP 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 ETI 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 FRA 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 GRE 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 HEB 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 HRV (Za stvaranje Adobe PDF dokumenata najpogodnijih za visokokvalitetni ispis prije tiskanja koristite ove postavke Stvoreni PDF dokumenti mogu se otvoriti Acrobat i Adobe Reader 50 i kasnijim verzijama)13 HUN 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 ITA 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 JPN ltFEFF9ad854c18cea306a30d730ea30d730ec30b951fa529b7528002000410064006f0062006500200050004400460020658766f8306e4f5c6210306b4f7f75283057307e305930023053306e8a2d5b9a30674f5c62103055308c305f0020005000440046002030d530a130a430eb306f3001004100630072006f0062006100740020304a30883073002000410064006f00620065002000520065006100640065007200200035002e003000204ee5964d3067958b304f30533068304c3067304d307e305930023053306e8a2d5b9a306b306f30d530a930f330c8306e57cb30818fbc307f304c5fc59808306730593002gt13 KOR ltFEFFc7740020c124c815c7440020c0acc6a9d558c5ec0020ace0d488c9c80020c2dcd5d80020c778c1c4c5d00020ac00c7a50020c801d569d55c002000410064006f0062006500200050004400460020bb38c11cb97c0020c791c131d569b2c8b2e4002e0020c774b807ac8c0020c791c131b41c00200050004400460020bb38c11cb2940020004100630072006f0062006100740020bc0f002000410064006f00620065002000520065006100640065007200200035002e00300020c774c0c1c5d0c11c0020c5f40020c2180020c788c2b5b2c8b2e4002egt13 LTH 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 LVI 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 NLD (Gebruik deze instellingen om Adobe PDF-documenten te maken die zijn geoptimaliseerd voor prepress-afdrukken van hoge kwaliteit De gemaakte PDF-documenten kunnen worden geopend met Acrobat en Adobe Reader 50 en hoger)13 NOR 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 POL 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 PTB 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 RUM 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 RUS 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 SKY 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 SLV 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 SUO 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 SVE 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 TUR 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 UKR ltFEFF04120438043a043e0440043804410442043e043204430439044204350020044604560020043f043004400430043c043504420440043800200434043b044f0020044104420432043e04400435043d043d044f00200434043e043a0443043c0435043d044204560432002000410064006f006200650020005000440046002c0020044f043a04560020043d04300439043a04400430044904350020043f045604340445043e0434044f0442044c00200434043b044f0020043204380441043e043a043e044f043a04560441043d043e0433043e0020043f0435044004350434043404400443043a043e0432043e0433043e0020043404400443043a0443002e00200020042104420432043e04400435043d045600200434043e043a0443043c0435043d0442043800200050004400460020043c043e0436043d04300020043204560434043a0440043804420438002004430020004100630072006f006200610074002004420430002000410064006f00620065002000520065006100640065007200200035002e0030002004300431043e0020043f04560437043d04560448043e04570020043204350440044104560457002egt13 ENU (Use these settings to create Adobe PDF documents best suited for high-quality prepress printing Created PDF documents can be opened with Acrobat and Adobe Reader 50 and later)13 gtgt13 Namespace [13 (Adobe)13 (Common)13 (10)13 ]13 OtherNamespaces [13 ltlt13 AsReaderSpreads false13 CropImagesToFrames true13 ErrorControl WarnAndContinue13 FlattenerIgnoreSpreadOverrides false13 IncludeGuidesGrids false13 IncludeNonPrinting false13 IncludeSlug false13 Namespace [13 (Adobe)13 (InDesign)13 (40)13 ]13 OmitPlacedBitmaps false13 OmitPlacedEPS false13 OmitPlacedPDF false13 SimulateOverprint Legacy13 gtgt13 ltlt13 AddBleedMarks false13 AddColorBars false13 AddCropMarks false13 AddPageInfo false13 AddRegMarks false13 ConvertColors ConvertToCMYK13 DestinationProfileName ()13 DestinationProfileSelector DocumentCMYK13 Downsample16BitImages true13 FlattenerPreset ltlt13 PresetSelector MediumResolution13 gtgt13 FormElements false13 GenerateStructure false13 IncludeBookmarks false13 IncludeHyperlinks false13 IncludeInteractive false13 IncludeLayers false13 IncludeProfiles false13 MultimediaHandling UseObjectSettings13 Namespace [13 (Adobe)13 (CreativeSuite)13 (20)13 ]13 PDFXOutputIntentProfileSelector DocumentCMYK13 PreserveEditing true13 UntaggedCMYKHandling LeaveUntagged13 UntaggedRGBHandling UseDocumentProfile13 UseDocumentBleed false13 gtgt13 ]13gtgt setdistillerparams13ltlt13 HWResolution [2400 2400]13 PageSize [612000 792000]13gtgt setpagedevice13

Page 26: February 2014 MEE Questions and AnalysesPreface The Multistate Essay Examination (MEE) is developed by the National Conference of Bar Examiners (NCBE). This publication includes the

CRIMINAL LAW AND PROCEDURE ANALYSIS (Criminal Law and Procedure IIA amp D VE amp F)

ANALYSIS

Legal Problems

(1) Did charging the defendant with both theft and burglary constitute double jeopardy

(2) Did the jury instruction violate the due process clause either by relieving the prosecution of the burden of proving the element of intent or by shifting the burden to the defendant to disprove that element

(3) Did the sentence imposed in this case for the theft conviction unconstitutionally deprive the defendant of his right to a jury trial on the issue of the value of the stolen item

DISCUSSION

Summary

The trial court properly denied the defendantrsquos pretrial motion to dismiss the charges on double jeopardy grounds The defendant may be charged with and convicted of both theft and burglary Each of the charges has an element that the other does not Neither charge is a lesser-included offense nor are they multiplicitous Thus charging both theft and burglary does not violate double jeopardy

The jury instruction on the burglary charge was constitutionally flawed It could have been reasonably understood by the jury as either (1) an irrebuttable conclusive presumption (which relieved the prosecution of proving the element of intent and removed the issue from the jury) or (2) a rebuttable mandatory presumption (which unconstitutionally shifted the burden of proof on an element of a charged offense from the prosecution to the defendant)

Because the four-year sentence imposed by the judge was based on the judgersquos finding by a preponderance of the evidence that the value of the stolen ring exceeded $5000 the sentence violates the defendantrsquos right to a jury determination beyond a reasonable doubt of the value of the ring

Point One (30) Charging the defendant with theft and burglary did not constitute double jeopardy

The Double Jeopardy Clause of the Fifth Amendment provides that a person shall not be twice put in jeopardy for the ldquosame offenserdquo Thus the question is whether the elements of the theft charge are wholly contained in the burglary charge or vice versa If the elements of the lesser charge (theft) are not wholly contained in the greater charge (burglary)mdashie if each charge requires proof of a fact that the other does notmdashthen convicting the defendant of both crimes would not violate double jeopardy even when the two offenses occurred at the same time and are thus arguably part of the ldquosame transactionrdquo Blockburger v United States 284 US 299 304 (1932) See also Albernaz v United States 450 US 333 344 n3 (1981) United States v Dixon 509 US 688 704 (1993)

23

Criminal Law and Procedure Analysis

Here theft and burglary each require proof of an element not required for the other crime Burglary may be defined differently in different jurisdictions However it almost invariably requires entry into a building or dwelling of another with the specific intent to commit a felony therein and the crime of burglary is complete upon the entry into the building or dwelling with such intent See eg Cannon v Oklahoma 827 P2d 1339 1342 (Okla Crim App 1992) In contrast theft which also may be defined differently in different states almost invariably requires the taking and carrying away of an item of personal property belonging to another with the intent to steal or permanently deprive the owner of possession

Here the ldquotakingrdquo or ldquostealingrdquo element is not contained in the definition of burglary and the ldquoentryrdquo element of burglary is not contained in the definition of theft Because theft is not a lesser-included offense of burglary and burglary is not a lesser-included offense of theft charging the defendant for both burglary and theft did not violate double jeopardy and the court properly denied the defense motion on those grounds Yparrea v Dorsey 64 F3d 577 579ndash80 (10th Cir 1995) citing Blockburger 284 US at 304

Finally the defendantrsquos motion to dismiss all the charges on double jeopardy grounds was improper because if both charges were for the same offense the motion should have requested dismissal of one charge not both

Point Two (35) The jury instruction on the burglary charge violated the Due Process Clause because it created either (1) an irrebuttable conclusive presumption (which relieved the prosecution of proving the element of intent and removed that issue from the jury) or (2) a rebuttable mandatory presumption (which unconstitutionally shifted the burden of proof on an element of a charged offense to the defendant)

The Supreme Court has interpreted the Due Process Clause of the US Constitution to require that the prosecution prove all elements of an offense beyond a reasonable doubt See In re Winship 397 US 358 364 (1970) The burden of proof cannot be shifted to the defendant by presuming an essential element upon proof of other elements of the offense because shifting the burden of persuasion with respect to any element of a criminal offense is contrary to the Due Process Clause See Mullaney v Wilbur 421 US 684 (1975)

The crime of burglary includes entry into a building or dwelling with the specific intent to commit a felony therein The requirement that the prosecutor prove beyond a reasonable doubt that the defendant had this specific intent distinguishes burglary from general-intent crimes like trespass See Sandstrom v Montana 442 US 510 523 (1979)

Here the jury was instructed that if ldquoafter consideration of all the evidence presented by the prosecution and defense you find beyond a reasonable doubt that the defendant entered the dwelling without the ownersrsquo consent you may presume that the defendant entered with the intent to commit a felony thereinrdquo This instruction was unconstitutional because it created either an irrebuttable conclusive presumption or a rebuttable mandatory presumption

A conclusive presumption is ldquoan irrebuttable direction by the court to find intent once convinced of the facts triggering the presumptionrdquo Id at 517 Here the jurors were instructed that once the prosecutor established that the defendant entered the neighborsrsquo house without consent they ldquomay presumerdquo that he intended to commit a felony therein The jurors may have reasonably concluded from this instruction that if they found that the defendant intended to enter his neighborsrsquo home without permission they must further find that he entered with the specific intent to commit a felony therein Because this instruction could operate as a conclusive

24

Criminal Law and Procedure Analysis

irrebuttable presumption by eliminating intent ldquoas an ingredient of the offenserdquo it violated due process by relieving the prosecution of the burden of proof for this element Id at 522

In the alternative the jury instruction could have been reasonably understood to create a rebuttable mandatory presumption which ldquotells [the jury] they must find the elemental fact upon proof of the basic fact at least unless the defendant has come forward with some evidence to rebut the presumed connection between the two factsrdquo County Court of Ulster County New York v Allen 442 US 140 157 (1979) The due process problem created by rebuttable mandatory presumptions is that ldquo[t]o the extent that the trier of fact is forced to abide by the presumption and may not reject it based on an independent evaluation of the particular facts presented by the State the analysis of the presumptionrsquos constitutional validity is logically divorced from those facts and based on the presumptionrsquos accuracy in the run of casesrdquo Id at 159

Unlike irrebuttable conclusive presumptions rebuttable mandatory presumptions are not always per se violations of the Due Process Clause However the Supreme Court of the United States has held that jury instructions that could reasonably be understood as shifting the burden of proof to the defendant on an element of the offense are unconstitutional Francis v Franklin 471 US 307 (1985) Here the argument that the jury instruction operated as a rebuttable mandatory presumption is supported by the fact that the judge also instructed the jury to ldquoconsider[ ] all the evidence presented by the prosecution and defenserdquo However even if the instruction created a rebuttable mandatory presumption it would be unconstitutional because it shifted the burden to the defense on an element of the offense Sandstrom 442 US at 524 Mullaney 421 US at 686

[NOTE Whether an examinee identifies the jury instruction as containing a ldquoconclusiverdquo or ldquomandatoryrdquo presumption is less important than the examineersquos analysis of the constitutional infirmities]

Point Three (35) The trial court violated the defendantrsquos Sixth Amendment right to a jury trial on an essential element of the offense when it found by a preponderance of the evidence that the ring was worth over $5000 and increased the defendantrsquos sentence based on this finding

In the statutory scheme under which the defendant was tried and convicted a Class D felony theft is defined as theft of item(s) with a value between $2500 and $10000 The jury found that the value of the diamond ring was at least $2500 and convicted the defendant of felony theft However at sentencing the trial court made a separate finding by a preponderance of the evidence that the value of the ring was greater than $5000 Following the statutersquos two-tiered sentencing scheme the judge then imposed on the defendant a sentence that was one year longer than the maximum that would otherwise have been allowed

The judgersquos sentence was unconstitutional because it violated the defendantrsquos Sixth Amendment right to a jury trial on this question The Supreme Court held in Apprendi v New Jersey 530 US 466 (2000) that ldquo[o]ther than the fact of a prior conviction any fact that increases the penalty for a crime beyond the prescribed statutory maximum must be submitted to a jury and proved beyond a reasonable doubtrdquo because ldquo[i]t is unconstitutional for a legislature to remove from the jury the assessment of facts that increase the prescribed range of penalties to which a criminal defendant is exposed [because] such facts must be established by proof beyond a reasonable doubtrdquo Id The Court reaffirmed Apprendi in Blakely v Washington 542 US 296 (2004) holding that the ldquolsquostatutory maximumrsquo for Apprendi purposes is the maximum sentence a judge may impose solely on the basis of the facts reflected in the jury verdict or admitted by the defendantrdquo Id at 303 (emphasis in original) In United States v Booker 543 US 220 (2005)

25

Criminal Law and Procedure Analysis

the Court relied on Blakely and Apprendi to conclude that protecting a defendantrsquos Sixth Amendment right to a jury trial required that ldquo[a]ny fact which is necessary to support a sentence exceeding the maximum authorized by the facts established by a plea of guilty or a jury verdict must be admitted by the defendant or proved to a jury beyond a reasonable doubtrdquo Id at 244

Thus in order to constitutionally increase a sentence above the statutory maximum of three years the jury must have found beyond a reasonable doubt that the value of the ring exceeded $5000 Here the court made the finding based on an appraisal proffered by the prosecutor only at sentencing and the judgersquos finding was by a preponderance of the evidence rather than beyond a reasonable doubt

26

AGENCY AND PARTNERSHIP ANALYSIS __________ (Agency and Partnership VA amp C VI)

ANALYSIS

Legal Problems

(1) Is a partner in a general partnership personally liable on a claim arising from misrepresentations by another partner made in the course of the partnership business

(2) Does a newly admitted partner in a general partnership become personally liable on existing claims against the partnership

(3) After the filing by a general partnership of a statement of qualification as a limited liability partnership are the partners personally liable as partners on (a) an existing claim against the general partnership and (b) a claim against the partnership that arose after the filing

DISCUSSION

Summary

Adam and Ben formed a general partnership under which they were jointly and severally liable for obligations of the partnership Thus Adam was personally liable for misrepresentations by Ben made in the ordinary course of the partnership business

Upon joining the general partnership Diane became personally liable for the obligations of the partnership arising after her admission but not for obligations pre-existing her admission such as the collectorrsquos claim

By filing a statement of qualification the three partners properly elected limited liability partnership status As partners in an LLP none of the three partners is personally liable as a partner for partnership obligations arising after the election such as the claim by the driverrsquos estate The election however does not change their personal liability on pre-existing claims that arose before the election such as the collectorrsquos claim

Point One (30) As a general partner of Empire a general partnership Adam became personally liable on the collectorrsquos claim a valid claim against the partnership that arose because of Benrsquos wrongful act in the ordinary course of the partnership business

When the collectorrsquos claim arose Empire was a general partnership composed of Adam and Ben Under UPA (1997) sect 306(a) partners of a general partnership are liable jointly and severally for all obligations of the partnership Under UPA (1997) sect 305(a) the partnership could become obligated for the loss caused to the collector as a result of the misrepresentation by Ben provided he was acting in the ordinary course of the partnership business Because there was no statement that limited his partnership authority Ben as partner was ldquoan agent of the partnership for the purpose of its businessrdquo See UPA (1997) sect 301(1) Benrsquos misrepresentation to the collector even if intentional appears to be in the ordinary course of the partnershiprsquos business of dealing

27

Agency and Partnership Analysis

in antique cars Thus Benrsquos wrongful act created a partnership obligation for which Adam was jointly and severally liable

[NOTE Generally a partnership creditor must ldquoexhaust the partnershiprsquos assets before levying on a judgment debtor partnerrsquos individual property where the partner is personally liable for the partnership obligationrdquo as a result of his status as a partner UPA (1997) sect 307 cmt 4 As the UPA comments explain this places Adam more in the position of guarantor than principal debtor on the partnership obligation Id cmt 4 Although an examinee might discuss this point the call focuses on whether Adam is personally liable not how the liability might be enforced]

Point Two (30) Because the collectorrsquos claim arose before Diane joined Empire Diane did not become personally liable on the claim

Diane was admitted to Empire when it was a general partnership and after the collectorrsquos claim arose While the general rule under UPA (1997) sect 306(a) is that the partners of a general partnership are liable jointly and severally for all obligations of the partnership there is a special rule for partners who are admitted during the duration of the partnership Under UPA (1997) sect 306(b) a person admitted to an existing partnership is not personally liable for any partnership obligations incurred before the personrsquos admission Because Diane was admitted to Empire after the collectorrsquos claim arose Diane is not personally liable on the claim

Dianersquos knowledge of the pre-existing claim and her stated concern about becoming liable on the collectorrsquos claim do not change her personal nonliability to the collector Although partners who have a liability shield can assume liability to third parties through private contractual guarantees or modifications to the partnership agreement Dianersquos stated concern constituted neither a guaranty to the collector nor ldquoan intentional waiver of liability protectionsrdquo See UPA (1997) sect 306 cmt 3 (describing methods for waiver of liability protections under sect 306(c) applicable in limited liability partnerships)

At most Diane will lose her investment in the partnership as a result of the collectorrsquos claim Although Diane did not become personally liable on the collectorrsquos claim when she joined the partnership the $250000 she contributed to the partnership is ldquoat risk for the satisfaction of existing partnership debtsrdquo UPA (1997) sect 306 cmt 2

Point Three (40) Filing the statement of qualification was effective to elect limited liability partnership status Despite this new status Adam and Ben remain personally liable on the collectorrsquos claim which arose before the election But as partners in an LLP neither Adam Ben nor Diane is personally liable as a partner on the driverrsquos estatersquos claim which arose after the election

Under UPA (1997) sect 1001 a general partnership can make an election and become a limited liability partnershipmdashif the partners approve the conversion by a vote equivalent to that necessary to amend the partnership agreement and the partnership then files a statement of qualification that specifies the name of the partnership its principal office and its election to be an LLP Here the partners agreed unanimouslymdashsufficient to amend their agreement under UPA (1997) sect 401(j)mdashand the statement of qualification was filed In addition the name of Empire LLP properly included an appropriate ending ldquoLLPrdquo See UPA (1997) sect 1002

Although another way to effectuate a ldquoconversionrdquo (as suggested by Benrsquos lawyer) is to form a new LLP and transfer the assets of the old general partnership to the new LLP the

28

Agency and Partnership Analysis

method used here (approval by the partners and the filing of a statement of qualification) is also sufficient to create LLP status

Thus Empire became Empire LLP as of the date of filing of the statement of qualification See UPA (1997) sect 1001 What effect did this have on the collectorrsquos claim which predated the filing According to UPA (1997) sect 306(c) an obligation incurred while a partnership is an LLP is solely a partnership obligation As the collectorrsquos claim predated the LLP Adam and Ben remain personally liable on the collectorrsquos claim Diane on the other hand was not personally liable on the collectorrsquos claim either before or after the filing of the statement of qualification See Point Two above

The driverrsquos estatersquos claim arose after Empire became Empire LLP Under UPA (1997) sect 306(c) an obligation incurred while a partnership is an LLP is solely a partnership obligationThus Adam Ben and Diane as partners are all protected from personal liability on the driverrsquos estatersquos claim But there may be personal liability if any of them was negligent or otherwise acted wrongfully by not informing the buyer of the bad suspension that caused the accident

29

National Conference of Bar Examiners 302 South Bedford Street | Madison WI 53703-3622 Phone 608-280-8550 | Fax 608-280-8552 | TDD 608-661-1275

wwwncbexorg e-mail contactncbexorg

  • Preface
  • Description of the MEE
  • Instructions
  • February 2014 Questions
    • Constitutinal Law Question
    • Trusts and Future Interests Question
    • Secured Transactions Question
    • Federal Civil Procedure Question
    • Criminal Law and Procedure Question
    • Agency and Partnership Question
      • February 2014 Analyses
        • Constitutional Law Analysis
        • Trust and Future Interests Analysis
        • Secured Transactions Analysis
        • Federal Civil Procedure Analysis
        • Criminal Law and Procedure Analysis
        • Agency and Partnership Analysis
            • ltlt13 ASCII85EncodePages false13 AllowTransparency false13 AutoPositionEPSFiles true13 AutoRotatePages None13 Binding Left13 CalGrayProfile (Dot Gain 20)13 CalRGBProfile (sRGB IEC61966-21)13 CalCMYKProfile (US Web Coated 050SWOP051 v2)13 sRGBProfile (sRGB IEC61966-21)13 CannotEmbedFontPolicy Error13 CompatibilityLevel 1413 CompressObjects Tags13 CompressPages true13 ConvertImagesToIndexed true13 PassThroughJPEGImages true13 CreateJobTicket false13 DefaultRenderingIntent Default13 DetectBlends true13 DetectCurves 0000013 ColorConversionStrategy CMYK13 DoThumbnails false13 EmbedAllFonts true13 EmbedOpenType false13 ParseICCProfilesInComments true13 EmbedJobOptions true13 DSCReportingLevel 013 EmitDSCWarnings false13 EndPage -113 ImageMemory 104857613 LockDistillerParams false13 MaxSubsetPct 10013 Optimize true13 OPM 113 ParseDSCComments true13 ParseDSCCommentsForDocInfo true13 PreserveCopyPage true13 PreserveDICMYKValues true13 PreserveEPSInfo true13 PreserveFlatness true13 PreserveHalftoneInfo false13 PreserveOPIComments true13 PreserveOverprintSettings true13 StartPage 113 SubsetFonts true13 TransferFunctionInfo Apply13 UCRandBGInfo Preserve13 UsePrologue false13 ColorSettingsFile ()13 AlwaysEmbed [ true13 ]13 NeverEmbed [ true13 ]13 AntiAliasColorImages false13 CropColorImages true13 ColorImageMinResolution 30013 ColorImageMinResolutionPolicy OK13 DownsampleColorImages true13 ColorImageDownsampleType Bicubic13 ColorImageResolution 30013 ColorImageDepth -113 ColorImageMinDownsampleDepth 113 ColorImageDownsampleThreshold 15000013 EncodeColorImages true13 ColorImageFilter DCTEncode13 AutoFilterColorImages true13 ColorImageAutoFilterStrategy JPEG13 ColorACSImageDict ltlt13 QFactor 01513 HSamples [1 1 1 1] VSamples [1 1 1 1]13 gtgt13 ColorImageDict ltlt13 QFactor 01513 HSamples [1 1 1 1] VSamples [1 1 1 1]13 gtgt13 JPEG2000ColorACSImageDict ltlt13 TileWidth 25613 TileHeight 25613 Quality 3013 gtgt13 JPEG2000ColorImageDict ltlt13 TileWidth 25613 TileHeight 25613 Quality 3013 gtgt13 AntiAliasGrayImages false13 CropGrayImages true13 GrayImageMinResolution 30013 GrayImageMinResolutionPolicy OK13 DownsampleGrayImages true13 GrayImageDownsampleType Bicubic13 GrayImageResolution 30013 GrayImageDepth -113 GrayImageMinDownsampleDepth 213 GrayImageDownsampleThreshold 15000013 EncodeGrayImages true13 GrayImageFilter DCTEncode13 AutoFilterGrayImages true13 GrayImageAutoFilterStrategy JPEG13 GrayACSImageDict ltlt13 QFactor 01513 HSamples [1 1 1 1] VSamples [1 1 1 1]13 gtgt13 GrayImageDict ltlt13 QFactor 01513 HSamples [1 1 1 1] VSamples [1 1 1 1]13 gtgt13 JPEG2000GrayACSImageDict ltlt13 TileWidth 25613 TileHeight 25613 Quality 3013 gtgt13 JPEG2000GrayImageDict ltlt13 TileWidth 25613 TileHeight 25613 Quality 3013 gtgt13 AntiAliasMonoImages false13 CropMonoImages true13 MonoImageMinResolution 120013 MonoImageMinResolutionPolicy OK13 DownsampleMonoImages true13 MonoImageDownsampleType Bicubic13 MonoImageResolution 120013 MonoImageDepth -113 MonoImageDownsampleThreshold 15000013 EncodeMonoImages true13 MonoImageFilter CCITTFaxEncode13 MonoImageDict ltlt13 K -113 gtgt13 AllowPSXObjects false13 CheckCompliance [13 None13 ]13 PDFX1aCheck false13 PDFX3Check false13 PDFXCompliantPDFOnly false13 PDFXNoTrimBoxError true13 PDFXTrimBoxToMediaBoxOffset [13 00000013 00000013 00000013 00000013 ]13 PDFXSetBleedBoxToMediaBox true13 PDFXBleedBoxToTrimBoxOffset [13 00000013 00000013 00000013 00000013 ]13 PDFXOutputIntentProfile ()13 PDFXOutputConditionIdentifier ()13 PDFXOutputCondition ()13 PDFXRegistryName ()13 PDFXTrapped False1313 CreateJDFFile false13 Description ltlt13 ARA 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 BGR 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 CHS ltFEFF4f7f75288fd94e9b8bbe5b9a521b5efa7684002000410064006f006200650020005000440046002065876863900275284e8e9ad88d2891cf76845370524d53705237300260a853ef4ee54f7f75280020004100630072006f0062006100740020548c002000410064006f00620065002000520065006100640065007200200035002e003000204ee553ca66f49ad87248672c676562535f00521b5efa768400200050004400460020658768633002gt13 CHT ltFEFF4f7f752890194e9b8a2d7f6e5efa7acb7684002000410064006f006200650020005000440046002065874ef69069752865bc9ad854c18cea76845370524d5370523786557406300260a853ef4ee54f7f75280020004100630072006f0062006100740020548c002000410064006f00620065002000520065006100640065007200200035002e003000204ee553ca66f49ad87248672c4f86958b555f5df25efa7acb76840020005000440046002065874ef63002gt13 CZE 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 DAN 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 DEU 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 ESP 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 ETI 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 FRA 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 GRE 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 HEB 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 HRV (Za stvaranje Adobe PDF dokumenata najpogodnijih za visokokvalitetni ispis prije tiskanja koristite ove postavke Stvoreni PDF dokumenti mogu se otvoriti Acrobat i Adobe Reader 50 i kasnijim verzijama)13 HUN 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 ITA 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 JPN ltFEFF9ad854c18cea306a30d730ea30d730ec30b951fa529b7528002000410064006f0062006500200050004400460020658766f8306e4f5c6210306b4f7f75283057307e305930023053306e8a2d5b9a30674f5c62103055308c305f0020005000440046002030d530a130a430eb306f3001004100630072006f0062006100740020304a30883073002000410064006f00620065002000520065006100640065007200200035002e003000204ee5964d3067958b304f30533068304c3067304d307e305930023053306e8a2d5b9a306b306f30d530a930f330c8306e57cb30818fbc307f304c5fc59808306730593002gt13 KOR ltFEFFc7740020c124c815c7440020c0acc6a9d558c5ec0020ace0d488c9c80020c2dcd5d80020c778c1c4c5d00020ac00c7a50020c801d569d55c002000410064006f0062006500200050004400460020bb38c11cb97c0020c791c131d569b2c8b2e4002e0020c774b807ac8c0020c791c131b41c00200050004400460020bb38c11cb2940020004100630072006f0062006100740020bc0f002000410064006f00620065002000520065006100640065007200200035002e00300020c774c0c1c5d0c11c0020c5f40020c2180020c788c2b5b2c8b2e4002egt13 LTH 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 LVI 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 NLD (Gebruik deze instellingen om Adobe PDF-documenten te maken die zijn geoptimaliseerd voor prepress-afdrukken van hoge kwaliteit De gemaakte PDF-documenten kunnen worden geopend met Acrobat en Adobe Reader 50 en hoger)13 NOR 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 POL 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 PTB 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 RUM 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 RUS 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 SKY ltFEFF0054006900650074006f0020006e006100730074006100760065006e0069006100200070006f0075017e0069007400650020006e00610020007600790074007600e100720061006e0069006500200064006f006b0075006d0065006e0074006f0076002000410064006f006200650020005000440046002c0020006b0074006f007200e90020007300610020006e0061006a006c0065007001610069006500200068006f0064006900610020006e00610020006b00760061006c00690074006e00fa00200074006c0061010d00200061002000700072006500700072006500730073002e00200056007900740076006f00720065006e00e900200064006f006b0075006d0065006e007400790020005000440046002000620075006400650020006d006f017e006e00e90020006f00740076006f00720069016500200076002000700072006f006700720061006d006f006300680020004100630072006f00620061007400200061002000410064006f00620065002000520065006100640065007200200035002e0030002000610020006e006f0076016100ed00630068002egt13 SLV 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 SUO 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 SVE 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 TUR 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 UKR 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 ENU (Use these settings to create Adobe PDF documents best suited for high-quality prepress printing Created PDF documents can be opened with Acrobat and Adobe Reader 50 and later)13 gtgt13 Namespace [13 (Adobe)13 (Common)13 (10)13 ]13 OtherNamespaces [13 ltlt13 AsReaderSpreads false13 CropImagesToFrames true13 ErrorControl WarnAndContinue13 FlattenerIgnoreSpreadOverrides false13 IncludeGuidesGrids false13 IncludeNonPrinting false13 IncludeSlug false13 Namespace [13 (Adobe)13 (InDesign)13 (40)13 ]13 OmitPlacedBitmaps false13 OmitPlacedEPS false13 OmitPlacedPDF false13 SimulateOverprint Legacy13 gtgt13 ltlt13 AddBleedMarks false13 AddColorBars false13 AddCropMarks false13 AddPageInfo false13 AddRegMarks false13 ConvertColors ConvertToCMYK13 DestinationProfileName ()13 DestinationProfileSelector DocumentCMYK13 Downsample16BitImages true13 FlattenerPreset ltlt13 PresetSelector MediumResolution13 gtgt13 FormElements false13 GenerateStructure false13 IncludeBookmarks false13 IncludeHyperlinks false13 IncludeInteractive false13 IncludeLayers false13 IncludeProfiles false13 MultimediaHandling UseObjectSettings13 Namespace [13 (Adobe)13 (CreativeSuite)13 (20)13 ]13 PDFXOutputIntentProfileSelector DocumentCMYK13 PreserveEditing true13 UntaggedCMYKHandling LeaveUntagged13 UntaggedRGBHandling UseDocumentProfile13 UseDocumentBleed false13 gtgt13 ]13gtgt setdistillerparams13ltlt13 HWResolution [2400 2400]13 PageSize [612000 792000]13gtgt setpagedevice13

Page 27: February 2014 MEE Questions and AnalysesPreface The Multistate Essay Examination (MEE) is developed by the National Conference of Bar Examiners (NCBE). This publication includes the

Criminal Law and Procedure Analysis

Here theft and burglary each require proof of an element not required for the other crime Burglary may be defined differently in different jurisdictions However it almost invariably requires entry into a building or dwelling of another with the specific intent to commit a felony therein and the crime of burglary is complete upon the entry into the building or dwelling with such intent See eg Cannon v Oklahoma 827 P2d 1339 1342 (Okla Crim App 1992) In contrast theft which also may be defined differently in different states almost invariably requires the taking and carrying away of an item of personal property belonging to another with the intent to steal or permanently deprive the owner of possession

Here the ldquotakingrdquo or ldquostealingrdquo element is not contained in the definition of burglary and the ldquoentryrdquo element of burglary is not contained in the definition of theft Because theft is not a lesser-included offense of burglary and burglary is not a lesser-included offense of theft charging the defendant for both burglary and theft did not violate double jeopardy and the court properly denied the defense motion on those grounds Yparrea v Dorsey 64 F3d 577 579ndash80 (10th Cir 1995) citing Blockburger 284 US at 304

Finally the defendantrsquos motion to dismiss all the charges on double jeopardy grounds was improper because if both charges were for the same offense the motion should have requested dismissal of one charge not both

Point Two (35) The jury instruction on the burglary charge violated the Due Process Clause because it created either (1) an irrebuttable conclusive presumption (which relieved the prosecution of proving the element of intent and removed that issue from the jury) or (2) a rebuttable mandatory presumption (which unconstitutionally shifted the burden of proof on an element of a charged offense to the defendant)

The Supreme Court has interpreted the Due Process Clause of the US Constitution to require that the prosecution prove all elements of an offense beyond a reasonable doubt See In re Winship 397 US 358 364 (1970) The burden of proof cannot be shifted to the defendant by presuming an essential element upon proof of other elements of the offense because shifting the burden of persuasion with respect to any element of a criminal offense is contrary to the Due Process Clause See Mullaney v Wilbur 421 US 684 (1975)

The crime of burglary includes entry into a building or dwelling with the specific intent to commit a felony therein The requirement that the prosecutor prove beyond a reasonable doubt that the defendant had this specific intent distinguishes burglary from general-intent crimes like trespass See Sandstrom v Montana 442 US 510 523 (1979)

Here the jury was instructed that if ldquoafter consideration of all the evidence presented by the prosecution and defense you find beyond a reasonable doubt that the defendant entered the dwelling without the ownersrsquo consent you may presume that the defendant entered with the intent to commit a felony thereinrdquo This instruction was unconstitutional because it created either an irrebuttable conclusive presumption or a rebuttable mandatory presumption

A conclusive presumption is ldquoan irrebuttable direction by the court to find intent once convinced of the facts triggering the presumptionrdquo Id at 517 Here the jurors were instructed that once the prosecutor established that the defendant entered the neighborsrsquo house without consent they ldquomay presumerdquo that he intended to commit a felony therein The jurors may have reasonably concluded from this instruction that if they found that the defendant intended to enter his neighborsrsquo home without permission they must further find that he entered with the specific intent to commit a felony therein Because this instruction could operate as a conclusive

24

Criminal Law and Procedure Analysis

irrebuttable presumption by eliminating intent ldquoas an ingredient of the offenserdquo it violated due process by relieving the prosecution of the burden of proof for this element Id at 522

In the alternative the jury instruction could have been reasonably understood to create a rebuttable mandatory presumption which ldquotells [the jury] they must find the elemental fact upon proof of the basic fact at least unless the defendant has come forward with some evidence to rebut the presumed connection between the two factsrdquo County Court of Ulster County New York v Allen 442 US 140 157 (1979) The due process problem created by rebuttable mandatory presumptions is that ldquo[t]o the extent that the trier of fact is forced to abide by the presumption and may not reject it based on an independent evaluation of the particular facts presented by the State the analysis of the presumptionrsquos constitutional validity is logically divorced from those facts and based on the presumptionrsquos accuracy in the run of casesrdquo Id at 159

Unlike irrebuttable conclusive presumptions rebuttable mandatory presumptions are not always per se violations of the Due Process Clause However the Supreme Court of the United States has held that jury instructions that could reasonably be understood as shifting the burden of proof to the defendant on an element of the offense are unconstitutional Francis v Franklin 471 US 307 (1985) Here the argument that the jury instruction operated as a rebuttable mandatory presumption is supported by the fact that the judge also instructed the jury to ldquoconsider[ ] all the evidence presented by the prosecution and defenserdquo However even if the instruction created a rebuttable mandatory presumption it would be unconstitutional because it shifted the burden to the defense on an element of the offense Sandstrom 442 US at 524 Mullaney 421 US at 686

[NOTE Whether an examinee identifies the jury instruction as containing a ldquoconclusiverdquo or ldquomandatoryrdquo presumption is less important than the examineersquos analysis of the constitutional infirmities]

Point Three (35) The trial court violated the defendantrsquos Sixth Amendment right to a jury trial on an essential element of the offense when it found by a preponderance of the evidence that the ring was worth over $5000 and increased the defendantrsquos sentence based on this finding

In the statutory scheme under which the defendant was tried and convicted a Class D felony theft is defined as theft of item(s) with a value between $2500 and $10000 The jury found that the value of the diamond ring was at least $2500 and convicted the defendant of felony theft However at sentencing the trial court made a separate finding by a preponderance of the evidence that the value of the ring was greater than $5000 Following the statutersquos two-tiered sentencing scheme the judge then imposed on the defendant a sentence that was one year longer than the maximum that would otherwise have been allowed

The judgersquos sentence was unconstitutional because it violated the defendantrsquos Sixth Amendment right to a jury trial on this question The Supreme Court held in Apprendi v New Jersey 530 US 466 (2000) that ldquo[o]ther than the fact of a prior conviction any fact that increases the penalty for a crime beyond the prescribed statutory maximum must be submitted to a jury and proved beyond a reasonable doubtrdquo because ldquo[i]t is unconstitutional for a legislature to remove from the jury the assessment of facts that increase the prescribed range of penalties to which a criminal defendant is exposed [because] such facts must be established by proof beyond a reasonable doubtrdquo Id The Court reaffirmed Apprendi in Blakely v Washington 542 US 296 (2004) holding that the ldquolsquostatutory maximumrsquo for Apprendi purposes is the maximum sentence a judge may impose solely on the basis of the facts reflected in the jury verdict or admitted by the defendantrdquo Id at 303 (emphasis in original) In United States v Booker 543 US 220 (2005)

25

Criminal Law and Procedure Analysis

the Court relied on Blakely and Apprendi to conclude that protecting a defendantrsquos Sixth Amendment right to a jury trial required that ldquo[a]ny fact which is necessary to support a sentence exceeding the maximum authorized by the facts established by a plea of guilty or a jury verdict must be admitted by the defendant or proved to a jury beyond a reasonable doubtrdquo Id at 244

Thus in order to constitutionally increase a sentence above the statutory maximum of three years the jury must have found beyond a reasonable doubt that the value of the ring exceeded $5000 Here the court made the finding based on an appraisal proffered by the prosecutor only at sentencing and the judgersquos finding was by a preponderance of the evidence rather than beyond a reasonable doubt

26

AGENCY AND PARTNERSHIP ANALYSIS __________ (Agency and Partnership VA amp C VI)

ANALYSIS

Legal Problems

(1) Is a partner in a general partnership personally liable on a claim arising from misrepresentations by another partner made in the course of the partnership business

(2) Does a newly admitted partner in a general partnership become personally liable on existing claims against the partnership

(3) After the filing by a general partnership of a statement of qualification as a limited liability partnership are the partners personally liable as partners on (a) an existing claim against the general partnership and (b) a claim against the partnership that arose after the filing

DISCUSSION

Summary

Adam and Ben formed a general partnership under which they were jointly and severally liable for obligations of the partnership Thus Adam was personally liable for misrepresentations by Ben made in the ordinary course of the partnership business

Upon joining the general partnership Diane became personally liable for the obligations of the partnership arising after her admission but not for obligations pre-existing her admission such as the collectorrsquos claim

By filing a statement of qualification the three partners properly elected limited liability partnership status As partners in an LLP none of the three partners is personally liable as a partner for partnership obligations arising after the election such as the claim by the driverrsquos estate The election however does not change their personal liability on pre-existing claims that arose before the election such as the collectorrsquos claim

Point One (30) As a general partner of Empire a general partnership Adam became personally liable on the collectorrsquos claim a valid claim against the partnership that arose because of Benrsquos wrongful act in the ordinary course of the partnership business

When the collectorrsquos claim arose Empire was a general partnership composed of Adam and Ben Under UPA (1997) sect 306(a) partners of a general partnership are liable jointly and severally for all obligations of the partnership Under UPA (1997) sect 305(a) the partnership could become obligated for the loss caused to the collector as a result of the misrepresentation by Ben provided he was acting in the ordinary course of the partnership business Because there was no statement that limited his partnership authority Ben as partner was ldquoan agent of the partnership for the purpose of its businessrdquo See UPA (1997) sect 301(1) Benrsquos misrepresentation to the collector even if intentional appears to be in the ordinary course of the partnershiprsquos business of dealing

27

Agency and Partnership Analysis

in antique cars Thus Benrsquos wrongful act created a partnership obligation for which Adam was jointly and severally liable

[NOTE Generally a partnership creditor must ldquoexhaust the partnershiprsquos assets before levying on a judgment debtor partnerrsquos individual property where the partner is personally liable for the partnership obligationrdquo as a result of his status as a partner UPA (1997) sect 307 cmt 4 As the UPA comments explain this places Adam more in the position of guarantor than principal debtor on the partnership obligation Id cmt 4 Although an examinee might discuss this point the call focuses on whether Adam is personally liable not how the liability might be enforced]

Point Two (30) Because the collectorrsquos claim arose before Diane joined Empire Diane did not become personally liable on the claim

Diane was admitted to Empire when it was a general partnership and after the collectorrsquos claim arose While the general rule under UPA (1997) sect 306(a) is that the partners of a general partnership are liable jointly and severally for all obligations of the partnership there is a special rule for partners who are admitted during the duration of the partnership Under UPA (1997) sect 306(b) a person admitted to an existing partnership is not personally liable for any partnership obligations incurred before the personrsquos admission Because Diane was admitted to Empire after the collectorrsquos claim arose Diane is not personally liable on the claim

Dianersquos knowledge of the pre-existing claim and her stated concern about becoming liable on the collectorrsquos claim do not change her personal nonliability to the collector Although partners who have a liability shield can assume liability to third parties through private contractual guarantees or modifications to the partnership agreement Dianersquos stated concern constituted neither a guaranty to the collector nor ldquoan intentional waiver of liability protectionsrdquo See UPA (1997) sect 306 cmt 3 (describing methods for waiver of liability protections under sect 306(c) applicable in limited liability partnerships)

At most Diane will lose her investment in the partnership as a result of the collectorrsquos claim Although Diane did not become personally liable on the collectorrsquos claim when she joined the partnership the $250000 she contributed to the partnership is ldquoat risk for the satisfaction of existing partnership debtsrdquo UPA (1997) sect 306 cmt 2

Point Three (40) Filing the statement of qualification was effective to elect limited liability partnership status Despite this new status Adam and Ben remain personally liable on the collectorrsquos claim which arose before the election But as partners in an LLP neither Adam Ben nor Diane is personally liable as a partner on the driverrsquos estatersquos claim which arose after the election

Under UPA (1997) sect 1001 a general partnership can make an election and become a limited liability partnershipmdashif the partners approve the conversion by a vote equivalent to that necessary to amend the partnership agreement and the partnership then files a statement of qualification that specifies the name of the partnership its principal office and its election to be an LLP Here the partners agreed unanimouslymdashsufficient to amend their agreement under UPA (1997) sect 401(j)mdashand the statement of qualification was filed In addition the name of Empire LLP properly included an appropriate ending ldquoLLPrdquo See UPA (1997) sect 1002

Although another way to effectuate a ldquoconversionrdquo (as suggested by Benrsquos lawyer) is to form a new LLP and transfer the assets of the old general partnership to the new LLP the

28

Agency and Partnership Analysis

method used here (approval by the partners and the filing of a statement of qualification) is also sufficient to create LLP status

Thus Empire became Empire LLP as of the date of filing of the statement of qualification See UPA (1997) sect 1001 What effect did this have on the collectorrsquos claim which predated the filing According to UPA (1997) sect 306(c) an obligation incurred while a partnership is an LLP is solely a partnership obligation As the collectorrsquos claim predated the LLP Adam and Ben remain personally liable on the collectorrsquos claim Diane on the other hand was not personally liable on the collectorrsquos claim either before or after the filing of the statement of qualification See Point Two above

The driverrsquos estatersquos claim arose after Empire became Empire LLP Under UPA (1997) sect 306(c) an obligation incurred while a partnership is an LLP is solely a partnership obligationThus Adam Ben and Diane as partners are all protected from personal liability on the driverrsquos estatersquos claim But there may be personal liability if any of them was negligent or otherwise acted wrongfully by not informing the buyer of the bad suspension that caused the accident

29

National Conference of Bar Examiners 302 South Bedford Street | Madison WI 53703-3622 Phone 608-280-8550 | Fax 608-280-8552 | TDD 608-661-1275

wwwncbexorg e-mail contactncbexorg

  • Preface
  • Description of the MEE
  • Instructions
  • February 2014 Questions
    • Constitutinal Law Question
    • Trusts and Future Interests Question
    • Secured Transactions Question
    • Federal Civil Procedure Question
    • Criminal Law and Procedure Question
    • Agency and Partnership Question
      • February 2014 Analyses
        • Constitutional Law Analysis
        • Trust and Future Interests Analysis
        • Secured Transactions Analysis
        • Federal Civil Procedure Analysis
        • Criminal Law and Procedure Analysis
        • Agency and Partnership Analysis
            • ltlt13 ASCII85EncodePages false13 AllowTransparency false13 AutoPositionEPSFiles true13 AutoRotatePages None13 Binding Left13 CalGrayProfile (Dot Gain 20)13 CalRGBProfile (sRGB IEC61966-21)13 CalCMYKProfile (US Web Coated 050SWOP051 v2)13 sRGBProfile (sRGB IEC61966-21)13 CannotEmbedFontPolicy Error13 CompatibilityLevel 1413 CompressObjects Tags13 CompressPages true13 ConvertImagesToIndexed true13 PassThroughJPEGImages true13 CreateJobTicket false13 DefaultRenderingIntent Default13 DetectBlends true13 DetectCurves 0000013 ColorConversionStrategy CMYK13 DoThumbnails false13 EmbedAllFonts true13 EmbedOpenType false13 ParseICCProfilesInComments true13 EmbedJobOptions true13 DSCReportingLevel 013 EmitDSCWarnings false13 EndPage -113 ImageMemory 104857613 LockDistillerParams false13 MaxSubsetPct 10013 Optimize true13 OPM 113 ParseDSCComments true13 ParseDSCCommentsForDocInfo true13 PreserveCopyPage true13 PreserveDICMYKValues true13 PreserveEPSInfo true13 PreserveFlatness true13 PreserveHalftoneInfo false13 PreserveOPIComments true13 PreserveOverprintSettings true13 StartPage 113 SubsetFonts true13 TransferFunctionInfo Apply13 UCRandBGInfo Preserve13 UsePrologue false13 ColorSettingsFile ()13 AlwaysEmbed [ true13 ]13 NeverEmbed [ true13 ]13 AntiAliasColorImages false13 CropColorImages true13 ColorImageMinResolution 30013 ColorImageMinResolutionPolicy OK13 DownsampleColorImages true13 ColorImageDownsampleType Bicubic13 ColorImageResolution 30013 ColorImageDepth -113 ColorImageMinDownsampleDepth 113 ColorImageDownsampleThreshold 15000013 EncodeColorImages true13 ColorImageFilter DCTEncode13 AutoFilterColorImages true13 ColorImageAutoFilterStrategy JPEG13 ColorACSImageDict ltlt13 QFactor 01513 HSamples [1 1 1 1] VSamples [1 1 1 1]13 gtgt13 ColorImageDict ltlt13 QFactor 01513 HSamples [1 1 1 1] VSamples [1 1 1 1]13 gtgt13 JPEG2000ColorACSImageDict ltlt13 TileWidth 25613 TileHeight 25613 Quality 3013 gtgt13 JPEG2000ColorImageDict ltlt13 TileWidth 25613 TileHeight 25613 Quality 3013 gtgt13 AntiAliasGrayImages false13 CropGrayImages true13 GrayImageMinResolution 30013 GrayImageMinResolutionPolicy OK13 DownsampleGrayImages true13 GrayImageDownsampleType Bicubic13 GrayImageResolution 30013 GrayImageDepth -113 GrayImageMinDownsampleDepth 213 GrayImageDownsampleThreshold 15000013 EncodeGrayImages true13 GrayImageFilter DCTEncode13 AutoFilterGrayImages true13 GrayImageAutoFilterStrategy JPEG13 GrayACSImageDict ltlt13 QFactor 01513 HSamples [1 1 1 1] VSamples [1 1 1 1]13 gtgt13 GrayImageDict ltlt13 QFactor 01513 HSamples [1 1 1 1] VSamples [1 1 1 1]13 gtgt13 JPEG2000GrayACSImageDict ltlt13 TileWidth 25613 TileHeight 25613 Quality 3013 gtgt13 JPEG2000GrayImageDict ltlt13 TileWidth 25613 TileHeight 25613 Quality 3013 gtgt13 AntiAliasMonoImages false13 CropMonoImages true13 MonoImageMinResolution 120013 MonoImageMinResolutionPolicy OK13 DownsampleMonoImages true13 MonoImageDownsampleType Bicubic13 MonoImageResolution 120013 MonoImageDepth -113 MonoImageDownsampleThreshold 15000013 EncodeMonoImages true13 MonoImageFilter CCITTFaxEncode13 MonoImageDict ltlt13 K -113 gtgt13 AllowPSXObjects false13 CheckCompliance [13 None13 ]13 PDFX1aCheck false13 PDFX3Check false13 PDFXCompliantPDFOnly false13 PDFXNoTrimBoxError true13 PDFXTrimBoxToMediaBoxOffset [13 00000013 00000013 00000013 00000013 ]13 PDFXSetBleedBoxToMediaBox true13 PDFXBleedBoxToTrimBoxOffset [13 00000013 00000013 00000013 00000013 ]13 PDFXOutputIntentProfile ()13 PDFXOutputConditionIdentifier ()13 PDFXOutputCondition ()13 PDFXRegistryName ()13 PDFXTrapped False1313 CreateJDFFile false13 Description ltlt13 ARA 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 BGR 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 CHS ltFEFF4f7f75288fd94e9b8bbe5b9a521b5efa7684002000410064006f006200650020005000440046002065876863900275284e8e9ad88d2891cf76845370524d53705237300260a853ef4ee54f7f75280020004100630072006f0062006100740020548c002000410064006f00620065002000520065006100640065007200200035002e003000204ee553ca66f49ad87248672c676562535f00521b5efa768400200050004400460020658768633002gt13 CHT ltFEFF4f7f752890194e9b8a2d7f6e5efa7acb7684002000410064006f006200650020005000440046002065874ef69069752865bc9ad854c18cea76845370524d5370523786557406300260a853ef4ee54f7f75280020004100630072006f0062006100740020548c002000410064006f00620065002000520065006100640065007200200035002e003000204ee553ca66f49ad87248672c4f86958b555f5df25efa7acb76840020005000440046002065874ef63002gt13 CZE 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 DAN ltFEFF004200720075006700200069006e0064007300740069006c006c0069006e006700650072006e0065002000740069006c0020006100740020006f007000720065007400740065002000410064006f006200650020005000440046002d0064006f006b0075006d0065006e007400650072002c0020006400650072002000620065006400730074002000650067006e006500720020007300690067002000740069006c002000700072006500700072006500730073002d007500640073006b007200690076006e0069006e00670020006100660020006800f8006a0020006b00760061006c0069007400650074002e0020004400650020006f007000720065007400740065006400650020005000440046002d0064006f006b0075006d0065006e0074006500720020006b0061006e002000e50062006e00650073002000690020004100630072006f00620061007400200065006c006c006500720020004100630072006f006200610074002000520065006100640065007200200035002e00300020006f00670020006e0079006500720065002egt13 DEU ltFEFF00560065007200770065006e00640065006e0020005300690065002000640069006500730065002000450069006e007300740065006c006c0075006e00670065006e0020007a0075006d002000450072007300740065006c006c0065006e00200076006f006e002000410064006f006200650020005000440046002d0044006f006b0075006d0065006e00740065006e002c00200076006f006e002000640065006e0065006e002000530069006500200068006f006300680077006500720074006900670065002000500072006500700072006500730073002d0044007200750063006b0065002000650072007a0065007500670065006e0020006d00f60063006800740065006e002e002000450072007300740065006c006c007400650020005000440046002d0044006f006b0075006d0065006e007400650020006b00f6006e006e0065006e0020006d006900740020004100630072006f00620061007400200075006e0064002000410064006f00620065002000520065006100640065007200200035002e00300020006f0064006500720020006800f600680065007200200067006500f600660066006e00650074002000770065007200640065006e002egt13 ESP 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 ETI ltFEFF004b00610073007500740061006700650020006e0065006900640020007300e4007400740065006900640020006b00760061006c006900740065006500740073006500200074007200fc006b006900650065006c007300650020007000720069006e00740069006d0069007300650020006a0061006f006b007300200073006f00620069006c0069006b0065002000410064006f006200650020005000440046002d0064006f006b0075006d0065006e00740069006400650020006c006f006f006d006900730065006b0073002e00200020004c006f006f0064007500640020005000440046002d0064006f006b0075006d0065006e00740065002000730061006100740065002000610076006100640061002000700072006f006700720061006d006d006900640065006700610020004100630072006f0062006100740020006e0069006e0067002000410064006f00620065002000520065006100640065007200200035002e00300020006a00610020007500750065006d006100740065002000760065007200730069006f006f006e00690064006500670061002e000d000agt13 FRA 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 GRE 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 HEB ltFEFF05D405E905EA05DE05E905D5002005D105D405D205D305E805D505EA002005D005DC05D4002005DB05D305D9002005DC05D905E605D505E8002005DE05E105DE05DB05D9002000410064006F006200650020005000440046002005D405DE05D505EA05D005DE05D905DD002005DC05D405D305E405E105EA002005E705D305DD002D05D305E405D505E1002005D005D905DB05D505EA05D905EA002E002005DE05E105DE05DB05D90020005000440046002005E905E005D505E605E805D5002005E005D905EA05E005D905DD002005DC05E405EA05D905D705D4002005D105D005DE05E605E205D505EA0020004100630072006F006200610074002005D5002D00410064006F00620065002000520065006100640065007200200035002E0030002005D505D205E805E105D005D505EA002005DE05EA05E705D305DE05D505EA002005D905D505EA05E8002E05D005DE05D905DD002005DC002D005000440046002F0058002D0033002C002005E205D905D905E005D5002005D105DE05D305E805D905DA002005DC05DE05E905EA05DE05E9002005E905DC0020004100630072006F006200610074002E002005DE05E105DE05DB05D90020005000440046002005E905E005D505E605E805D5002005E005D905EA05E005D905DD002005DC05E405EA05D905D705D4002005D105D005DE05E605E205D505EA0020004100630072006F006200610074002005D5002D00410064006F00620065002000520065006100640065007200200035002E0030002005D505D205E805E105D005D505EA002005DE05EA05E705D305DE05D505EA002005D905D505EA05E8002Egt13 HRV (Za stvaranje Adobe PDF dokumenata najpogodnijih za visokokvalitetni ispis prije tiskanja koristite ove postavke Stvoreni PDF dokumenti mogu se otvoriti Acrobat i Adobe Reader 50 i kasnijim verzijama)13 HUN 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 ITA 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 JPN ltFEFF9ad854c18cea306a30d730ea30d730ec30b951fa529b7528002000410064006f0062006500200050004400460020658766f8306e4f5c6210306b4f7f75283057307e305930023053306e8a2d5b9a30674f5c62103055308c305f0020005000440046002030d530a130a430eb306f3001004100630072006f0062006100740020304a30883073002000410064006f00620065002000520065006100640065007200200035002e003000204ee5964d3067958b304f30533068304c3067304d307e305930023053306e8a2d5b9a306b306f30d530a930f330c8306e57cb30818fbc307f304c5fc59808306730593002gt13 KOR ltFEFFc7740020c124c815c7440020c0acc6a9d558c5ec0020ace0d488c9c80020c2dcd5d80020c778c1c4c5d00020ac00c7a50020c801d569d55c002000410064006f0062006500200050004400460020bb38c11cb97c0020c791c131d569b2c8b2e4002e0020c774b807ac8c0020c791c131b41c00200050004400460020bb38c11cb2940020004100630072006f0062006100740020bc0f002000410064006f00620065002000520065006100640065007200200035002e00300020c774c0c1c5d0c11c0020c5f40020c2180020c788c2b5b2c8b2e4002egt13 LTH 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 LVI 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 NLD (Gebruik deze instellingen om Adobe PDF-documenten te maken die zijn geoptimaliseerd voor prepress-afdrukken van hoge kwaliteit De gemaakte PDF-documenten kunnen worden geopend met Acrobat en Adobe Reader 50 en hoger)13 NOR 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 POL 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 PTB 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 RUM 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 RUS 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 SKY ltFEFF0054006900650074006f0020006e006100730074006100760065006e0069006100200070006f0075017e0069007400650020006e00610020007600790074007600e100720061006e0069006500200064006f006b0075006d0065006e0074006f0076002000410064006f006200650020005000440046002c0020006b0074006f007200e90020007300610020006e0061006a006c0065007001610069006500200068006f0064006900610020006e00610020006b00760061006c00690074006e00fa00200074006c0061010d00200061002000700072006500700072006500730073002e00200056007900740076006f00720065006e00e900200064006f006b0075006d0065006e007400790020005000440046002000620075006400650020006d006f017e006e00e90020006f00740076006f00720069016500200076002000700072006f006700720061006d006f006300680020004100630072006f00620061007400200061002000410064006f00620065002000520065006100640065007200200035002e0030002000610020006e006f0076016100ed00630068002egt13 SLV 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 SUO 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 SVE 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 TUR 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 UKR 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 ENU (Use these settings to create Adobe PDF documents best suited for high-quality prepress printing Created PDF documents can be opened with Acrobat and Adobe Reader 50 and later)13 gtgt13 Namespace [13 (Adobe)13 (Common)13 (10)13 ]13 OtherNamespaces [13 ltlt13 AsReaderSpreads false13 CropImagesToFrames true13 ErrorControl WarnAndContinue13 FlattenerIgnoreSpreadOverrides false13 IncludeGuidesGrids false13 IncludeNonPrinting false13 IncludeSlug false13 Namespace [13 (Adobe)13 (InDesign)13 (40)13 ]13 OmitPlacedBitmaps false13 OmitPlacedEPS false13 OmitPlacedPDF false13 SimulateOverprint Legacy13 gtgt13 ltlt13 AddBleedMarks false13 AddColorBars false13 AddCropMarks false13 AddPageInfo false13 AddRegMarks false13 ConvertColors ConvertToCMYK13 DestinationProfileName ()13 DestinationProfileSelector DocumentCMYK13 Downsample16BitImages true13 FlattenerPreset ltlt13 PresetSelector MediumResolution13 gtgt13 FormElements false13 GenerateStructure false13 IncludeBookmarks false13 IncludeHyperlinks false13 IncludeInteractive false13 IncludeLayers false13 IncludeProfiles false13 MultimediaHandling UseObjectSettings13 Namespace [13 (Adobe)13 (CreativeSuite)13 (20)13 ]13 PDFXOutputIntentProfileSelector DocumentCMYK13 PreserveEditing true13 UntaggedCMYKHandling LeaveUntagged13 UntaggedRGBHandling UseDocumentProfile13 UseDocumentBleed false13 gtgt13 ]13gtgt setdistillerparams13ltlt13 HWResolution [2400 2400]13 PageSize [612000 792000]13gtgt setpagedevice13

Page 28: February 2014 MEE Questions and AnalysesPreface The Multistate Essay Examination (MEE) is developed by the National Conference of Bar Examiners (NCBE). This publication includes the

Criminal Law and Procedure Analysis

irrebuttable presumption by eliminating intent ldquoas an ingredient of the offenserdquo it violated due process by relieving the prosecution of the burden of proof for this element Id at 522

In the alternative the jury instruction could have been reasonably understood to create a rebuttable mandatory presumption which ldquotells [the jury] they must find the elemental fact upon proof of the basic fact at least unless the defendant has come forward with some evidence to rebut the presumed connection between the two factsrdquo County Court of Ulster County New York v Allen 442 US 140 157 (1979) The due process problem created by rebuttable mandatory presumptions is that ldquo[t]o the extent that the trier of fact is forced to abide by the presumption and may not reject it based on an independent evaluation of the particular facts presented by the State the analysis of the presumptionrsquos constitutional validity is logically divorced from those facts and based on the presumptionrsquos accuracy in the run of casesrdquo Id at 159

Unlike irrebuttable conclusive presumptions rebuttable mandatory presumptions are not always per se violations of the Due Process Clause However the Supreme Court of the United States has held that jury instructions that could reasonably be understood as shifting the burden of proof to the defendant on an element of the offense are unconstitutional Francis v Franklin 471 US 307 (1985) Here the argument that the jury instruction operated as a rebuttable mandatory presumption is supported by the fact that the judge also instructed the jury to ldquoconsider[ ] all the evidence presented by the prosecution and defenserdquo However even if the instruction created a rebuttable mandatory presumption it would be unconstitutional because it shifted the burden to the defense on an element of the offense Sandstrom 442 US at 524 Mullaney 421 US at 686

[NOTE Whether an examinee identifies the jury instruction as containing a ldquoconclusiverdquo or ldquomandatoryrdquo presumption is less important than the examineersquos analysis of the constitutional infirmities]

Point Three (35) The trial court violated the defendantrsquos Sixth Amendment right to a jury trial on an essential element of the offense when it found by a preponderance of the evidence that the ring was worth over $5000 and increased the defendantrsquos sentence based on this finding

In the statutory scheme under which the defendant was tried and convicted a Class D felony theft is defined as theft of item(s) with a value between $2500 and $10000 The jury found that the value of the diamond ring was at least $2500 and convicted the defendant of felony theft However at sentencing the trial court made a separate finding by a preponderance of the evidence that the value of the ring was greater than $5000 Following the statutersquos two-tiered sentencing scheme the judge then imposed on the defendant a sentence that was one year longer than the maximum that would otherwise have been allowed

The judgersquos sentence was unconstitutional because it violated the defendantrsquos Sixth Amendment right to a jury trial on this question The Supreme Court held in Apprendi v New Jersey 530 US 466 (2000) that ldquo[o]ther than the fact of a prior conviction any fact that increases the penalty for a crime beyond the prescribed statutory maximum must be submitted to a jury and proved beyond a reasonable doubtrdquo because ldquo[i]t is unconstitutional for a legislature to remove from the jury the assessment of facts that increase the prescribed range of penalties to which a criminal defendant is exposed [because] such facts must be established by proof beyond a reasonable doubtrdquo Id The Court reaffirmed Apprendi in Blakely v Washington 542 US 296 (2004) holding that the ldquolsquostatutory maximumrsquo for Apprendi purposes is the maximum sentence a judge may impose solely on the basis of the facts reflected in the jury verdict or admitted by the defendantrdquo Id at 303 (emphasis in original) In United States v Booker 543 US 220 (2005)

25

Criminal Law and Procedure Analysis

the Court relied on Blakely and Apprendi to conclude that protecting a defendantrsquos Sixth Amendment right to a jury trial required that ldquo[a]ny fact which is necessary to support a sentence exceeding the maximum authorized by the facts established by a plea of guilty or a jury verdict must be admitted by the defendant or proved to a jury beyond a reasonable doubtrdquo Id at 244

Thus in order to constitutionally increase a sentence above the statutory maximum of three years the jury must have found beyond a reasonable doubt that the value of the ring exceeded $5000 Here the court made the finding based on an appraisal proffered by the prosecutor only at sentencing and the judgersquos finding was by a preponderance of the evidence rather than beyond a reasonable doubt

26

AGENCY AND PARTNERSHIP ANALYSIS __________ (Agency and Partnership VA amp C VI)

ANALYSIS

Legal Problems

(1) Is a partner in a general partnership personally liable on a claim arising from misrepresentations by another partner made in the course of the partnership business

(2) Does a newly admitted partner in a general partnership become personally liable on existing claims against the partnership

(3) After the filing by a general partnership of a statement of qualification as a limited liability partnership are the partners personally liable as partners on (a) an existing claim against the general partnership and (b) a claim against the partnership that arose after the filing

DISCUSSION

Summary

Adam and Ben formed a general partnership under which they were jointly and severally liable for obligations of the partnership Thus Adam was personally liable for misrepresentations by Ben made in the ordinary course of the partnership business

Upon joining the general partnership Diane became personally liable for the obligations of the partnership arising after her admission but not for obligations pre-existing her admission such as the collectorrsquos claim

By filing a statement of qualification the three partners properly elected limited liability partnership status As partners in an LLP none of the three partners is personally liable as a partner for partnership obligations arising after the election such as the claim by the driverrsquos estate The election however does not change their personal liability on pre-existing claims that arose before the election such as the collectorrsquos claim

Point One (30) As a general partner of Empire a general partnership Adam became personally liable on the collectorrsquos claim a valid claim against the partnership that arose because of Benrsquos wrongful act in the ordinary course of the partnership business

When the collectorrsquos claim arose Empire was a general partnership composed of Adam and Ben Under UPA (1997) sect 306(a) partners of a general partnership are liable jointly and severally for all obligations of the partnership Under UPA (1997) sect 305(a) the partnership could become obligated for the loss caused to the collector as a result of the misrepresentation by Ben provided he was acting in the ordinary course of the partnership business Because there was no statement that limited his partnership authority Ben as partner was ldquoan agent of the partnership for the purpose of its businessrdquo See UPA (1997) sect 301(1) Benrsquos misrepresentation to the collector even if intentional appears to be in the ordinary course of the partnershiprsquos business of dealing

27

Agency and Partnership Analysis

in antique cars Thus Benrsquos wrongful act created a partnership obligation for which Adam was jointly and severally liable

[NOTE Generally a partnership creditor must ldquoexhaust the partnershiprsquos assets before levying on a judgment debtor partnerrsquos individual property where the partner is personally liable for the partnership obligationrdquo as a result of his status as a partner UPA (1997) sect 307 cmt 4 As the UPA comments explain this places Adam more in the position of guarantor than principal debtor on the partnership obligation Id cmt 4 Although an examinee might discuss this point the call focuses on whether Adam is personally liable not how the liability might be enforced]

Point Two (30) Because the collectorrsquos claim arose before Diane joined Empire Diane did not become personally liable on the claim

Diane was admitted to Empire when it was a general partnership and after the collectorrsquos claim arose While the general rule under UPA (1997) sect 306(a) is that the partners of a general partnership are liable jointly and severally for all obligations of the partnership there is a special rule for partners who are admitted during the duration of the partnership Under UPA (1997) sect 306(b) a person admitted to an existing partnership is not personally liable for any partnership obligations incurred before the personrsquos admission Because Diane was admitted to Empire after the collectorrsquos claim arose Diane is not personally liable on the claim

Dianersquos knowledge of the pre-existing claim and her stated concern about becoming liable on the collectorrsquos claim do not change her personal nonliability to the collector Although partners who have a liability shield can assume liability to third parties through private contractual guarantees or modifications to the partnership agreement Dianersquos stated concern constituted neither a guaranty to the collector nor ldquoan intentional waiver of liability protectionsrdquo See UPA (1997) sect 306 cmt 3 (describing methods for waiver of liability protections under sect 306(c) applicable in limited liability partnerships)

At most Diane will lose her investment in the partnership as a result of the collectorrsquos claim Although Diane did not become personally liable on the collectorrsquos claim when she joined the partnership the $250000 she contributed to the partnership is ldquoat risk for the satisfaction of existing partnership debtsrdquo UPA (1997) sect 306 cmt 2

Point Three (40) Filing the statement of qualification was effective to elect limited liability partnership status Despite this new status Adam and Ben remain personally liable on the collectorrsquos claim which arose before the election But as partners in an LLP neither Adam Ben nor Diane is personally liable as a partner on the driverrsquos estatersquos claim which arose after the election

Under UPA (1997) sect 1001 a general partnership can make an election and become a limited liability partnershipmdashif the partners approve the conversion by a vote equivalent to that necessary to amend the partnership agreement and the partnership then files a statement of qualification that specifies the name of the partnership its principal office and its election to be an LLP Here the partners agreed unanimouslymdashsufficient to amend their agreement under UPA (1997) sect 401(j)mdashand the statement of qualification was filed In addition the name of Empire LLP properly included an appropriate ending ldquoLLPrdquo See UPA (1997) sect 1002

Although another way to effectuate a ldquoconversionrdquo (as suggested by Benrsquos lawyer) is to form a new LLP and transfer the assets of the old general partnership to the new LLP the

28

Agency and Partnership Analysis

method used here (approval by the partners and the filing of a statement of qualification) is also sufficient to create LLP status

Thus Empire became Empire LLP as of the date of filing of the statement of qualification See UPA (1997) sect 1001 What effect did this have on the collectorrsquos claim which predated the filing According to UPA (1997) sect 306(c) an obligation incurred while a partnership is an LLP is solely a partnership obligation As the collectorrsquos claim predated the LLP Adam and Ben remain personally liable on the collectorrsquos claim Diane on the other hand was not personally liable on the collectorrsquos claim either before or after the filing of the statement of qualification See Point Two above

The driverrsquos estatersquos claim arose after Empire became Empire LLP Under UPA (1997) sect 306(c) an obligation incurred while a partnership is an LLP is solely a partnership obligationThus Adam Ben and Diane as partners are all protected from personal liability on the driverrsquos estatersquos claim But there may be personal liability if any of them was negligent or otherwise acted wrongfully by not informing the buyer of the bad suspension that caused the accident

29

National Conference of Bar Examiners 302 South Bedford Street | Madison WI 53703-3622 Phone 608-280-8550 | Fax 608-280-8552 | TDD 608-661-1275

wwwncbexorg e-mail contactncbexorg

  • Preface
  • Description of the MEE
  • Instructions
  • February 2014 Questions
    • Constitutinal Law Question
    • Trusts and Future Interests Question
    • Secured Transactions Question
    • Federal Civil Procedure Question
    • Criminal Law and Procedure Question
    • Agency and Partnership Question
      • February 2014 Analyses
        • Constitutional Law Analysis
        • Trust and Future Interests Analysis
        • Secured Transactions Analysis
        • Federal Civil Procedure Analysis
        • Criminal Law and Procedure Analysis
        • Agency and Partnership Analysis
            • ltlt13 ASCII85EncodePages false13 AllowTransparency false13 AutoPositionEPSFiles true13 AutoRotatePages None13 Binding Left13 CalGrayProfile (Dot Gain 20)13 CalRGBProfile (sRGB IEC61966-21)13 CalCMYKProfile (US Web Coated 050SWOP051 v2)13 sRGBProfile (sRGB IEC61966-21)13 CannotEmbedFontPolicy Error13 CompatibilityLevel 1413 CompressObjects Tags13 CompressPages true13 ConvertImagesToIndexed true13 PassThroughJPEGImages true13 CreateJobTicket false13 DefaultRenderingIntent Default13 DetectBlends true13 DetectCurves 0000013 ColorConversionStrategy CMYK13 DoThumbnails false13 EmbedAllFonts true13 EmbedOpenType false13 ParseICCProfilesInComments true13 EmbedJobOptions true13 DSCReportingLevel 013 EmitDSCWarnings false13 EndPage -113 ImageMemory 104857613 LockDistillerParams false13 MaxSubsetPct 10013 Optimize true13 OPM 113 ParseDSCComments true13 ParseDSCCommentsForDocInfo true13 PreserveCopyPage true13 PreserveDICMYKValues true13 PreserveEPSInfo true13 PreserveFlatness true13 PreserveHalftoneInfo false13 PreserveOPIComments true13 PreserveOverprintSettings true13 StartPage 113 SubsetFonts true13 TransferFunctionInfo Apply13 UCRandBGInfo Preserve13 UsePrologue false13 ColorSettingsFile ()13 AlwaysEmbed [ true13 ]13 NeverEmbed [ true13 ]13 AntiAliasColorImages false13 CropColorImages true13 ColorImageMinResolution 30013 ColorImageMinResolutionPolicy OK13 DownsampleColorImages true13 ColorImageDownsampleType Bicubic13 ColorImageResolution 30013 ColorImageDepth -113 ColorImageMinDownsampleDepth 113 ColorImageDownsampleThreshold 15000013 EncodeColorImages true13 ColorImageFilter DCTEncode13 AutoFilterColorImages true13 ColorImageAutoFilterStrategy JPEG13 ColorACSImageDict ltlt13 QFactor 01513 HSamples [1 1 1 1] VSamples [1 1 1 1]13 gtgt13 ColorImageDict ltlt13 QFactor 01513 HSamples [1 1 1 1] VSamples [1 1 1 1]13 gtgt13 JPEG2000ColorACSImageDict ltlt13 TileWidth 25613 TileHeight 25613 Quality 3013 gtgt13 JPEG2000ColorImageDict ltlt13 TileWidth 25613 TileHeight 25613 Quality 3013 gtgt13 AntiAliasGrayImages false13 CropGrayImages true13 GrayImageMinResolution 30013 GrayImageMinResolutionPolicy OK13 DownsampleGrayImages true13 GrayImageDownsampleType Bicubic13 GrayImageResolution 30013 GrayImageDepth -113 GrayImageMinDownsampleDepth 213 GrayImageDownsampleThreshold 15000013 EncodeGrayImages true13 GrayImageFilter DCTEncode13 AutoFilterGrayImages true13 GrayImageAutoFilterStrategy JPEG13 GrayACSImageDict ltlt13 QFactor 01513 HSamples [1 1 1 1] VSamples [1 1 1 1]13 gtgt13 GrayImageDict ltlt13 QFactor 01513 HSamples [1 1 1 1] VSamples [1 1 1 1]13 gtgt13 JPEG2000GrayACSImageDict ltlt13 TileWidth 25613 TileHeight 25613 Quality 3013 gtgt13 JPEG2000GrayImageDict ltlt13 TileWidth 25613 TileHeight 25613 Quality 3013 gtgt13 AntiAliasMonoImages false13 CropMonoImages true13 MonoImageMinResolution 120013 MonoImageMinResolutionPolicy OK13 DownsampleMonoImages true13 MonoImageDownsampleType Bicubic13 MonoImageResolution 120013 MonoImageDepth -113 MonoImageDownsampleThreshold 15000013 EncodeMonoImages true13 MonoImageFilter CCITTFaxEncode13 MonoImageDict ltlt13 K -113 gtgt13 AllowPSXObjects false13 CheckCompliance [13 None13 ]13 PDFX1aCheck false13 PDFX3Check false13 PDFXCompliantPDFOnly false13 PDFXNoTrimBoxError true13 PDFXTrimBoxToMediaBoxOffset [13 00000013 00000013 00000013 00000013 ]13 PDFXSetBleedBoxToMediaBox true13 PDFXBleedBoxToTrimBoxOffset [13 00000013 00000013 00000013 00000013 ]13 PDFXOutputIntentProfile ()13 PDFXOutputConditionIdentifier ()13 PDFXOutputCondition ()13 PDFXRegistryName ()13 PDFXTrapped False1313 CreateJDFFile false13 Description ltlt13 ARA 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 BGR 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 CHS ltFEFF4f7f75288fd94e9b8bbe5b9a521b5efa7684002000410064006f006200650020005000440046002065876863900275284e8e9ad88d2891cf76845370524d53705237300260a853ef4ee54f7f75280020004100630072006f0062006100740020548c002000410064006f00620065002000520065006100640065007200200035002e003000204ee553ca66f49ad87248672c676562535f00521b5efa768400200050004400460020658768633002gt13 CHT ltFEFF4f7f752890194e9b8a2d7f6e5efa7acb7684002000410064006f006200650020005000440046002065874ef69069752865bc9ad854c18cea76845370524d5370523786557406300260a853ef4ee54f7f75280020004100630072006f0062006100740020548c002000410064006f00620065002000520065006100640065007200200035002e003000204ee553ca66f49ad87248672c4f86958b555f5df25efa7acb76840020005000440046002065874ef63002gt13 CZE ltFEFF005400610074006f0020006e006100730074006100760065006e00ed00200070006f0075017e0069006a007400650020006b0020007600790074007600e101590065006e00ed00200064006f006b0075006d0065006e0074016f002000410064006f006200650020005000440046002c0020006b00740065007200e90020007300650020006e0065006a006c00e90070006500200068006f006400ed002000700072006f0020006b00760061006c00690074006e00ed0020007400690073006b00200061002000700072006500700072006500730073002e002000200056007900740076006f01590065006e00e900200064006f006b0075006d0065006e007400790020005000440046002000620075006400650020006d006f017e006e00e90020006f007400650076015900ed007400200076002000700072006f006700720061006d0065006300680020004100630072006f00620061007400200061002000410064006f00620065002000520065006100640065007200200035002e0030002000610020006e006f0076011b006a016100ed00630068002egt13 DAN 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 DEU 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 ESP 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 ETI 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 FRA 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 GRE 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 HEB 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 HRV (Za stvaranje Adobe PDF dokumenata najpogodnijih za visokokvalitetni ispis prije tiskanja koristite ove postavke Stvoreni PDF dokumenti mogu se otvoriti Acrobat i Adobe Reader 50 i kasnijim verzijama)13 HUN 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 ITA ltFEFF005500740069006c0069007a007a006100720065002000710075006500730074006500200069006d0070006f007300740061007a0069006f006e00690020007000650072002000630072006500610072006500200064006f00630075006d0065006e00740069002000410064006f00620065002000500044004600200070006900f900200061006400610074007400690020006100200075006e00610020007000720065007300740061006d0070006100200064006900200061006c007400610020007100750061006c0069007400e0002e0020004900200064006f00630075006d0065006e007400690020005000440046002000630072006500610074006900200070006f00730073006f006e006f0020006500730073006500720065002000610070006500720074006900200063006f006e0020004100630072006f00620061007400200065002000410064006f00620065002000520065006100640065007200200035002e003000200065002000760065007200730069006f006e006900200073007500630063006500730073006900760065002egt13 JPN ltFEFF9ad854c18cea306a30d730ea30d730ec30b951fa529b7528002000410064006f0062006500200050004400460020658766f8306e4f5c6210306b4f7f75283057307e305930023053306e8a2d5b9a30674f5c62103055308c305f0020005000440046002030d530a130a430eb306f3001004100630072006f0062006100740020304a30883073002000410064006f00620065002000520065006100640065007200200035002e003000204ee5964d3067958b304f30533068304c3067304d307e305930023053306e8a2d5b9a306b306f30d530a930f330c8306e57cb30818fbc307f304c5fc59808306730593002gt13 KOR ltFEFFc7740020c124c815c7440020c0acc6a9d558c5ec0020ace0d488c9c80020c2dcd5d80020c778c1c4c5d00020ac00c7a50020c801d569d55c002000410064006f0062006500200050004400460020bb38c11cb97c0020c791c131d569b2c8b2e4002e0020c774b807ac8c0020c791c131b41c00200050004400460020bb38c11cb2940020004100630072006f0062006100740020bc0f002000410064006f00620065002000520065006100640065007200200035002e00300020c774c0c1c5d0c11c0020c5f40020c2180020c788c2b5b2c8b2e4002egt13 LTH 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 LVI 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 NLD (Gebruik deze instellingen om Adobe PDF-documenten te maken die zijn geoptimaliseerd voor prepress-afdrukken van hoge kwaliteit De gemaakte PDF-documenten kunnen worden geopend met Acrobat en Adobe Reader 50 en hoger)13 NOR 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 POL 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 PTB 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 RUM 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 RUS ltFEFF04180441043f043e043b044c04370443043904420435002004340430043d043d044b04350020043d0430044104420440043e0439043a043800200434043b044f00200441043e043704340430043d0438044f00200434043e043a0443043c0435043d0442043e0432002000410064006f006200650020005000440046002c0020043c0430043a04410438043c0430043b044c043d043e0020043f043e04340445043e0434044f04490438044500200434043b044f00200432044b0441043e043a043e043a0430044704350441044204320435043d043d043e0433043e00200434043e043f0435044704300442043d043e0433043e00200432044b0432043e04340430002e002000200421043e043704340430043d043d044b04350020005000440046002d0434043e043a0443043c0435043d0442044b0020043c043e0436043d043e0020043e0442043a0440044b043204300442044c002004410020043f043e043c043e0449044c044e0020004100630072006f00620061007400200438002000410064006f00620065002000520065006100640065007200200035002e00300020043800200431043e043b043504350020043f043e04370434043d043804450020043204350440044104380439002egt13 SKY 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 SLV 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 SUO 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 SVE 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 TUR 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 UKR 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 ENU (Use these settings to create Adobe PDF documents best suited for high-quality prepress printing Created PDF documents can be opened with Acrobat and Adobe Reader 50 and later)13 gtgt13 Namespace [13 (Adobe)13 (Common)13 (10)13 ]13 OtherNamespaces [13 ltlt13 AsReaderSpreads false13 CropImagesToFrames true13 ErrorControl WarnAndContinue13 FlattenerIgnoreSpreadOverrides false13 IncludeGuidesGrids false13 IncludeNonPrinting false13 IncludeSlug false13 Namespace [13 (Adobe)13 (InDesign)13 (40)13 ]13 OmitPlacedBitmaps false13 OmitPlacedEPS false13 OmitPlacedPDF false13 SimulateOverprint Legacy13 gtgt13 ltlt13 AddBleedMarks false13 AddColorBars false13 AddCropMarks false13 AddPageInfo false13 AddRegMarks false13 ConvertColors ConvertToCMYK13 DestinationProfileName ()13 DestinationProfileSelector DocumentCMYK13 Downsample16BitImages true13 FlattenerPreset ltlt13 PresetSelector MediumResolution13 gtgt13 FormElements false13 GenerateStructure false13 IncludeBookmarks false13 IncludeHyperlinks false13 IncludeInteractive false13 IncludeLayers false13 IncludeProfiles false13 MultimediaHandling UseObjectSettings13 Namespace [13 (Adobe)13 (CreativeSuite)13 (20)13 ]13 PDFXOutputIntentProfileSelector DocumentCMYK13 PreserveEditing true13 UntaggedCMYKHandling LeaveUntagged13 UntaggedRGBHandling UseDocumentProfile13 UseDocumentBleed false13 gtgt13 ]13gtgt setdistillerparams13ltlt13 HWResolution [2400 2400]13 PageSize [612000 792000]13gtgt setpagedevice13

Page 29: February 2014 MEE Questions and AnalysesPreface The Multistate Essay Examination (MEE) is developed by the National Conference of Bar Examiners (NCBE). This publication includes the

Criminal Law and Procedure Analysis

the Court relied on Blakely and Apprendi to conclude that protecting a defendantrsquos Sixth Amendment right to a jury trial required that ldquo[a]ny fact which is necessary to support a sentence exceeding the maximum authorized by the facts established by a plea of guilty or a jury verdict must be admitted by the defendant or proved to a jury beyond a reasonable doubtrdquo Id at 244

Thus in order to constitutionally increase a sentence above the statutory maximum of three years the jury must have found beyond a reasonable doubt that the value of the ring exceeded $5000 Here the court made the finding based on an appraisal proffered by the prosecutor only at sentencing and the judgersquos finding was by a preponderance of the evidence rather than beyond a reasonable doubt

26

AGENCY AND PARTNERSHIP ANALYSIS __________ (Agency and Partnership VA amp C VI)

ANALYSIS

Legal Problems

(1) Is a partner in a general partnership personally liable on a claim arising from misrepresentations by another partner made in the course of the partnership business

(2) Does a newly admitted partner in a general partnership become personally liable on existing claims against the partnership

(3) After the filing by a general partnership of a statement of qualification as a limited liability partnership are the partners personally liable as partners on (a) an existing claim against the general partnership and (b) a claim against the partnership that arose after the filing

DISCUSSION

Summary

Adam and Ben formed a general partnership under which they were jointly and severally liable for obligations of the partnership Thus Adam was personally liable for misrepresentations by Ben made in the ordinary course of the partnership business

Upon joining the general partnership Diane became personally liable for the obligations of the partnership arising after her admission but not for obligations pre-existing her admission such as the collectorrsquos claim

By filing a statement of qualification the three partners properly elected limited liability partnership status As partners in an LLP none of the three partners is personally liable as a partner for partnership obligations arising after the election such as the claim by the driverrsquos estate The election however does not change their personal liability on pre-existing claims that arose before the election such as the collectorrsquos claim

Point One (30) As a general partner of Empire a general partnership Adam became personally liable on the collectorrsquos claim a valid claim against the partnership that arose because of Benrsquos wrongful act in the ordinary course of the partnership business

When the collectorrsquos claim arose Empire was a general partnership composed of Adam and Ben Under UPA (1997) sect 306(a) partners of a general partnership are liable jointly and severally for all obligations of the partnership Under UPA (1997) sect 305(a) the partnership could become obligated for the loss caused to the collector as a result of the misrepresentation by Ben provided he was acting in the ordinary course of the partnership business Because there was no statement that limited his partnership authority Ben as partner was ldquoan agent of the partnership for the purpose of its businessrdquo See UPA (1997) sect 301(1) Benrsquos misrepresentation to the collector even if intentional appears to be in the ordinary course of the partnershiprsquos business of dealing

27

Agency and Partnership Analysis

in antique cars Thus Benrsquos wrongful act created a partnership obligation for which Adam was jointly and severally liable

[NOTE Generally a partnership creditor must ldquoexhaust the partnershiprsquos assets before levying on a judgment debtor partnerrsquos individual property where the partner is personally liable for the partnership obligationrdquo as a result of his status as a partner UPA (1997) sect 307 cmt 4 As the UPA comments explain this places Adam more in the position of guarantor than principal debtor on the partnership obligation Id cmt 4 Although an examinee might discuss this point the call focuses on whether Adam is personally liable not how the liability might be enforced]

Point Two (30) Because the collectorrsquos claim arose before Diane joined Empire Diane did not become personally liable on the claim

Diane was admitted to Empire when it was a general partnership and after the collectorrsquos claim arose While the general rule under UPA (1997) sect 306(a) is that the partners of a general partnership are liable jointly and severally for all obligations of the partnership there is a special rule for partners who are admitted during the duration of the partnership Under UPA (1997) sect 306(b) a person admitted to an existing partnership is not personally liable for any partnership obligations incurred before the personrsquos admission Because Diane was admitted to Empire after the collectorrsquos claim arose Diane is not personally liable on the claim

Dianersquos knowledge of the pre-existing claim and her stated concern about becoming liable on the collectorrsquos claim do not change her personal nonliability to the collector Although partners who have a liability shield can assume liability to third parties through private contractual guarantees or modifications to the partnership agreement Dianersquos stated concern constituted neither a guaranty to the collector nor ldquoan intentional waiver of liability protectionsrdquo See UPA (1997) sect 306 cmt 3 (describing methods for waiver of liability protections under sect 306(c) applicable in limited liability partnerships)

At most Diane will lose her investment in the partnership as a result of the collectorrsquos claim Although Diane did not become personally liable on the collectorrsquos claim when she joined the partnership the $250000 she contributed to the partnership is ldquoat risk for the satisfaction of existing partnership debtsrdquo UPA (1997) sect 306 cmt 2

Point Three (40) Filing the statement of qualification was effective to elect limited liability partnership status Despite this new status Adam and Ben remain personally liable on the collectorrsquos claim which arose before the election But as partners in an LLP neither Adam Ben nor Diane is personally liable as a partner on the driverrsquos estatersquos claim which arose after the election

Under UPA (1997) sect 1001 a general partnership can make an election and become a limited liability partnershipmdashif the partners approve the conversion by a vote equivalent to that necessary to amend the partnership agreement and the partnership then files a statement of qualification that specifies the name of the partnership its principal office and its election to be an LLP Here the partners agreed unanimouslymdashsufficient to amend their agreement under UPA (1997) sect 401(j)mdashand the statement of qualification was filed In addition the name of Empire LLP properly included an appropriate ending ldquoLLPrdquo See UPA (1997) sect 1002

Although another way to effectuate a ldquoconversionrdquo (as suggested by Benrsquos lawyer) is to form a new LLP and transfer the assets of the old general partnership to the new LLP the

28

Agency and Partnership Analysis

method used here (approval by the partners and the filing of a statement of qualification) is also sufficient to create LLP status

Thus Empire became Empire LLP as of the date of filing of the statement of qualification See UPA (1997) sect 1001 What effect did this have on the collectorrsquos claim which predated the filing According to UPA (1997) sect 306(c) an obligation incurred while a partnership is an LLP is solely a partnership obligation As the collectorrsquos claim predated the LLP Adam and Ben remain personally liable on the collectorrsquos claim Diane on the other hand was not personally liable on the collectorrsquos claim either before or after the filing of the statement of qualification See Point Two above

The driverrsquos estatersquos claim arose after Empire became Empire LLP Under UPA (1997) sect 306(c) an obligation incurred while a partnership is an LLP is solely a partnership obligationThus Adam Ben and Diane as partners are all protected from personal liability on the driverrsquos estatersquos claim But there may be personal liability if any of them was negligent or otherwise acted wrongfully by not informing the buyer of the bad suspension that caused the accident

29

National Conference of Bar Examiners 302 South Bedford Street | Madison WI 53703-3622 Phone 608-280-8550 | Fax 608-280-8552 | TDD 608-661-1275

wwwncbexorg e-mail contactncbexorg

  • Preface
  • Description of the MEE
  • Instructions
  • February 2014 Questions
    • Constitutinal Law Question
    • Trusts and Future Interests Question
    • Secured Transactions Question
    • Federal Civil Procedure Question
    • Criminal Law and Procedure Question
    • Agency and Partnership Question
      • February 2014 Analyses
        • Constitutional Law Analysis
        • Trust and Future Interests Analysis
        • Secured Transactions Analysis
        • Federal Civil Procedure Analysis
        • Criminal Law and Procedure Analysis
        • Agency and Partnership Analysis
            • ltlt13 ASCII85EncodePages false13 AllowTransparency false13 AutoPositionEPSFiles true13 AutoRotatePages None13 Binding Left13 CalGrayProfile (Dot Gain 20)13 CalRGBProfile (sRGB IEC61966-21)13 CalCMYKProfile (US Web Coated 050SWOP051 v2)13 sRGBProfile (sRGB IEC61966-21)13 CannotEmbedFontPolicy Error13 CompatibilityLevel 1413 CompressObjects Tags13 CompressPages true13 ConvertImagesToIndexed true13 PassThroughJPEGImages true13 CreateJobTicket false13 DefaultRenderingIntent Default13 DetectBlends true13 DetectCurves 0000013 ColorConversionStrategy CMYK13 DoThumbnails false13 EmbedAllFonts true13 EmbedOpenType false13 ParseICCProfilesInComments true13 EmbedJobOptions true13 DSCReportingLevel 013 EmitDSCWarnings false13 EndPage -113 ImageMemory 104857613 LockDistillerParams false13 MaxSubsetPct 10013 Optimize true13 OPM 113 ParseDSCComments true13 ParseDSCCommentsForDocInfo true13 PreserveCopyPage true13 PreserveDICMYKValues true13 PreserveEPSInfo true13 PreserveFlatness true13 PreserveHalftoneInfo false13 PreserveOPIComments true13 PreserveOverprintSettings true13 StartPage 113 SubsetFonts true13 TransferFunctionInfo Apply13 UCRandBGInfo Preserve13 UsePrologue false13 ColorSettingsFile ()13 AlwaysEmbed [ true13 ]13 NeverEmbed [ true13 ]13 AntiAliasColorImages false13 CropColorImages true13 ColorImageMinResolution 30013 ColorImageMinResolutionPolicy OK13 DownsampleColorImages true13 ColorImageDownsampleType Bicubic13 ColorImageResolution 30013 ColorImageDepth -113 ColorImageMinDownsampleDepth 113 ColorImageDownsampleThreshold 15000013 EncodeColorImages true13 ColorImageFilter DCTEncode13 AutoFilterColorImages true13 ColorImageAutoFilterStrategy JPEG13 ColorACSImageDict ltlt13 QFactor 01513 HSamples [1 1 1 1] VSamples [1 1 1 1]13 gtgt13 ColorImageDict ltlt13 QFactor 01513 HSamples [1 1 1 1] VSamples [1 1 1 1]13 gtgt13 JPEG2000ColorACSImageDict ltlt13 TileWidth 25613 TileHeight 25613 Quality 3013 gtgt13 JPEG2000ColorImageDict ltlt13 TileWidth 25613 TileHeight 25613 Quality 3013 gtgt13 AntiAliasGrayImages false13 CropGrayImages true13 GrayImageMinResolution 30013 GrayImageMinResolutionPolicy OK13 DownsampleGrayImages true13 GrayImageDownsampleType Bicubic13 GrayImageResolution 30013 GrayImageDepth -113 GrayImageMinDownsampleDepth 213 GrayImageDownsampleThreshold 15000013 EncodeGrayImages true13 GrayImageFilter DCTEncode13 AutoFilterGrayImages true13 GrayImageAutoFilterStrategy JPEG13 GrayACSImageDict ltlt13 QFactor 01513 HSamples [1 1 1 1] VSamples [1 1 1 1]13 gtgt13 GrayImageDict ltlt13 QFactor 01513 HSamples [1 1 1 1] VSamples [1 1 1 1]13 gtgt13 JPEG2000GrayACSImageDict ltlt13 TileWidth 25613 TileHeight 25613 Quality 3013 gtgt13 JPEG2000GrayImageDict ltlt13 TileWidth 25613 TileHeight 25613 Quality 3013 gtgt13 AntiAliasMonoImages false13 CropMonoImages true13 MonoImageMinResolution 120013 MonoImageMinResolutionPolicy OK13 DownsampleMonoImages true13 MonoImageDownsampleType Bicubic13 MonoImageResolution 120013 MonoImageDepth -113 MonoImageDownsampleThreshold 15000013 EncodeMonoImages true13 MonoImageFilter CCITTFaxEncode13 MonoImageDict ltlt13 K -113 gtgt13 AllowPSXObjects false13 CheckCompliance [13 None13 ]13 PDFX1aCheck false13 PDFX3Check false13 PDFXCompliantPDFOnly false13 PDFXNoTrimBoxError true13 PDFXTrimBoxToMediaBoxOffset [13 00000013 00000013 00000013 00000013 ]13 PDFXSetBleedBoxToMediaBox true13 PDFXBleedBoxToTrimBoxOffset [13 00000013 00000013 00000013 00000013 ]13 PDFXOutputIntentProfile ()13 PDFXOutputConditionIdentifier ()13 PDFXOutputCondition ()13 PDFXRegistryName ()13 PDFXTrapped False1313 CreateJDFFile false13 Description ltlt13 ARA 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 BGR ltFEFF04180437043f043e043b043704320430043904420435002004420435043704380020043d0430044104420440043e0439043a0438002c00200437043000200434043000200441044a0437043404300432043004420435002000410064006f00620065002000500044004600200434043e043a0443043c0435043d04420438002c0020043c0430043a04410438043c0430043b043d043e0020043f044004380433043e04340435043d04380020043704300020043204380441043e043a043e043a0430044704350441044204320435043d0020043f04350447043004420020043704300020043f044004350434043f0435044704300442043d04300020043f043e04340433043e0442043e0432043a0430002e002000200421044a04370434043004340435043d043804420435002000500044004600200434043e043a0443043c0435043d044204380020043c043e0433043004420020043404300020044104350020043e0442043204300440044f0442002004410020004100630072006f00620061007400200438002000410064006f00620065002000520065006100640065007200200035002e00300020043800200441043b0435043404320430044904380020043204350440044104380438002egt13 CHS ltFEFF4f7f75288fd94e9b8bbe5b9a521b5efa7684002000410064006f006200650020005000440046002065876863900275284e8e9ad88d2891cf76845370524d53705237300260a853ef4ee54f7f75280020004100630072006f0062006100740020548c002000410064006f00620065002000520065006100640065007200200035002e003000204ee553ca66f49ad87248672c676562535f00521b5efa768400200050004400460020658768633002gt13 CHT ltFEFF4f7f752890194e9b8a2d7f6e5efa7acb7684002000410064006f006200650020005000440046002065874ef69069752865bc9ad854c18cea76845370524d5370523786557406300260a853ef4ee54f7f75280020004100630072006f0062006100740020548c002000410064006f00620065002000520065006100640065007200200035002e003000204ee553ca66f49ad87248672c4f86958b555f5df25efa7acb76840020005000440046002065874ef63002gt13 CZE 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 DAN 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 DEU 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 ESP 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 ETI 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 FRA 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 GRE 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 HEB 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 HRV (Za stvaranje Adobe PDF dokumenata najpogodnijih za visokokvalitetni ispis prije tiskanja koristite ove postavke Stvoreni PDF dokumenti mogu se otvoriti Acrobat i Adobe Reader 50 i kasnijim verzijama)13 HUN 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 ITA 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 JPN ltFEFF9ad854c18cea306a30d730ea30d730ec30b951fa529b7528002000410064006f0062006500200050004400460020658766f8306e4f5c6210306b4f7f75283057307e305930023053306e8a2d5b9a30674f5c62103055308c305f0020005000440046002030d530a130a430eb306f3001004100630072006f0062006100740020304a30883073002000410064006f00620065002000520065006100640065007200200035002e003000204ee5964d3067958b304f30533068304c3067304d307e305930023053306e8a2d5b9a306b306f30d530a930f330c8306e57cb30818fbc307f304c5fc59808306730593002gt13 KOR ltFEFFc7740020c124c815c7440020c0acc6a9d558c5ec0020ace0d488c9c80020c2dcd5d80020c778c1c4c5d00020ac00c7a50020c801d569d55c002000410064006f0062006500200050004400460020bb38c11cb97c0020c791c131d569b2c8b2e4002e0020c774b807ac8c0020c791c131b41c00200050004400460020bb38c11cb2940020004100630072006f0062006100740020bc0f002000410064006f00620065002000520065006100640065007200200035002e00300020c774c0c1c5d0c11c0020c5f40020c2180020c788c2b5b2c8b2e4002egt13 LTH 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 LVI 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 NLD (Gebruik deze instellingen om Adobe PDF-documenten te maken die zijn geoptimaliseerd voor prepress-afdrukken van hoge kwaliteit De gemaakte PDF-documenten kunnen worden geopend met Acrobat en Adobe Reader 50 en hoger)13 NOR 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 POL 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 PTB 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 RUM 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 RUS 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 SKY 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 SLV 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 SUO 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 SVE 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 TUR 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 UKR ltFEFF04120438043a043e0440043804410442043e043204430439044204350020044604560020043f043004400430043c043504420440043800200434043b044f0020044104420432043e04400435043d043d044f00200434043e043a0443043c0435043d044204560432002000410064006f006200650020005000440046002c0020044f043a04560020043d04300439043a04400430044904350020043f045604340445043e0434044f0442044c00200434043b044f0020043204380441043e043a043e044f043a04560441043d043e0433043e0020043f0435044004350434043404400443043a043e0432043e0433043e0020043404400443043a0443002e00200020042104420432043e04400435043d045600200434043e043a0443043c0435043d0442043800200050004400460020043c043e0436043d04300020043204560434043a0440043804420438002004430020004100630072006f006200610074002004420430002000410064006f00620065002000520065006100640065007200200035002e0030002004300431043e0020043f04560437043d04560448043e04570020043204350440044104560457002egt13 ENU (Use these settings to create Adobe PDF documents best suited for high-quality prepress printing Created PDF documents can be opened with Acrobat and Adobe Reader 50 and later)13 gtgt13 Namespace [13 (Adobe)13 (Common)13 (10)13 ]13 OtherNamespaces [13 ltlt13 AsReaderSpreads false13 CropImagesToFrames true13 ErrorControl WarnAndContinue13 FlattenerIgnoreSpreadOverrides false13 IncludeGuidesGrids false13 IncludeNonPrinting false13 IncludeSlug false13 Namespace [13 (Adobe)13 (InDesign)13 (40)13 ]13 OmitPlacedBitmaps false13 OmitPlacedEPS false13 OmitPlacedPDF false13 SimulateOverprint Legacy13 gtgt13 ltlt13 AddBleedMarks false13 AddColorBars false13 AddCropMarks false13 AddPageInfo false13 AddRegMarks false13 ConvertColors ConvertToCMYK13 DestinationProfileName ()13 DestinationProfileSelector DocumentCMYK13 Downsample16BitImages true13 FlattenerPreset ltlt13 PresetSelector MediumResolution13 gtgt13 FormElements false13 GenerateStructure false13 IncludeBookmarks false13 IncludeHyperlinks false13 IncludeInteractive false13 IncludeLayers false13 IncludeProfiles false13 MultimediaHandling UseObjectSettings13 Namespace [13 (Adobe)13 (CreativeSuite)13 (20)13 ]13 PDFXOutputIntentProfileSelector DocumentCMYK13 PreserveEditing true13 UntaggedCMYKHandling LeaveUntagged13 UntaggedRGBHandling UseDocumentProfile13 UseDocumentBleed false13 gtgt13 ]13gtgt setdistillerparams13ltlt13 HWResolution [2400 2400]13 PageSize [612000 792000]13gtgt setpagedevice13

Page 30: February 2014 MEE Questions and AnalysesPreface The Multistate Essay Examination (MEE) is developed by the National Conference of Bar Examiners (NCBE). This publication includes the

AGENCY AND PARTNERSHIP ANALYSIS __________ (Agency and Partnership VA amp C VI)

ANALYSIS

Legal Problems

(1) Is a partner in a general partnership personally liable on a claim arising from misrepresentations by another partner made in the course of the partnership business

(2) Does a newly admitted partner in a general partnership become personally liable on existing claims against the partnership

(3) After the filing by a general partnership of a statement of qualification as a limited liability partnership are the partners personally liable as partners on (a) an existing claim against the general partnership and (b) a claim against the partnership that arose after the filing

DISCUSSION

Summary

Adam and Ben formed a general partnership under which they were jointly and severally liable for obligations of the partnership Thus Adam was personally liable for misrepresentations by Ben made in the ordinary course of the partnership business

Upon joining the general partnership Diane became personally liable for the obligations of the partnership arising after her admission but not for obligations pre-existing her admission such as the collectorrsquos claim

By filing a statement of qualification the three partners properly elected limited liability partnership status As partners in an LLP none of the three partners is personally liable as a partner for partnership obligations arising after the election such as the claim by the driverrsquos estate The election however does not change their personal liability on pre-existing claims that arose before the election such as the collectorrsquos claim

Point One (30) As a general partner of Empire a general partnership Adam became personally liable on the collectorrsquos claim a valid claim against the partnership that arose because of Benrsquos wrongful act in the ordinary course of the partnership business

When the collectorrsquos claim arose Empire was a general partnership composed of Adam and Ben Under UPA (1997) sect 306(a) partners of a general partnership are liable jointly and severally for all obligations of the partnership Under UPA (1997) sect 305(a) the partnership could become obligated for the loss caused to the collector as a result of the misrepresentation by Ben provided he was acting in the ordinary course of the partnership business Because there was no statement that limited his partnership authority Ben as partner was ldquoan agent of the partnership for the purpose of its businessrdquo See UPA (1997) sect 301(1) Benrsquos misrepresentation to the collector even if intentional appears to be in the ordinary course of the partnershiprsquos business of dealing

27

Agency and Partnership Analysis

in antique cars Thus Benrsquos wrongful act created a partnership obligation for which Adam was jointly and severally liable

[NOTE Generally a partnership creditor must ldquoexhaust the partnershiprsquos assets before levying on a judgment debtor partnerrsquos individual property where the partner is personally liable for the partnership obligationrdquo as a result of his status as a partner UPA (1997) sect 307 cmt 4 As the UPA comments explain this places Adam more in the position of guarantor than principal debtor on the partnership obligation Id cmt 4 Although an examinee might discuss this point the call focuses on whether Adam is personally liable not how the liability might be enforced]

Point Two (30) Because the collectorrsquos claim arose before Diane joined Empire Diane did not become personally liable on the claim

Diane was admitted to Empire when it was a general partnership and after the collectorrsquos claim arose While the general rule under UPA (1997) sect 306(a) is that the partners of a general partnership are liable jointly and severally for all obligations of the partnership there is a special rule for partners who are admitted during the duration of the partnership Under UPA (1997) sect 306(b) a person admitted to an existing partnership is not personally liable for any partnership obligations incurred before the personrsquos admission Because Diane was admitted to Empire after the collectorrsquos claim arose Diane is not personally liable on the claim

Dianersquos knowledge of the pre-existing claim and her stated concern about becoming liable on the collectorrsquos claim do not change her personal nonliability to the collector Although partners who have a liability shield can assume liability to third parties through private contractual guarantees or modifications to the partnership agreement Dianersquos stated concern constituted neither a guaranty to the collector nor ldquoan intentional waiver of liability protectionsrdquo See UPA (1997) sect 306 cmt 3 (describing methods for waiver of liability protections under sect 306(c) applicable in limited liability partnerships)

At most Diane will lose her investment in the partnership as a result of the collectorrsquos claim Although Diane did not become personally liable on the collectorrsquos claim when she joined the partnership the $250000 she contributed to the partnership is ldquoat risk for the satisfaction of existing partnership debtsrdquo UPA (1997) sect 306 cmt 2

Point Three (40) Filing the statement of qualification was effective to elect limited liability partnership status Despite this new status Adam and Ben remain personally liable on the collectorrsquos claim which arose before the election But as partners in an LLP neither Adam Ben nor Diane is personally liable as a partner on the driverrsquos estatersquos claim which arose after the election

Under UPA (1997) sect 1001 a general partnership can make an election and become a limited liability partnershipmdashif the partners approve the conversion by a vote equivalent to that necessary to amend the partnership agreement and the partnership then files a statement of qualification that specifies the name of the partnership its principal office and its election to be an LLP Here the partners agreed unanimouslymdashsufficient to amend their agreement under UPA (1997) sect 401(j)mdashand the statement of qualification was filed In addition the name of Empire LLP properly included an appropriate ending ldquoLLPrdquo See UPA (1997) sect 1002

Although another way to effectuate a ldquoconversionrdquo (as suggested by Benrsquos lawyer) is to form a new LLP and transfer the assets of the old general partnership to the new LLP the

28

Agency and Partnership Analysis

method used here (approval by the partners and the filing of a statement of qualification) is also sufficient to create LLP status

Thus Empire became Empire LLP as of the date of filing of the statement of qualification See UPA (1997) sect 1001 What effect did this have on the collectorrsquos claim which predated the filing According to UPA (1997) sect 306(c) an obligation incurred while a partnership is an LLP is solely a partnership obligation As the collectorrsquos claim predated the LLP Adam and Ben remain personally liable on the collectorrsquos claim Diane on the other hand was not personally liable on the collectorrsquos claim either before or after the filing of the statement of qualification See Point Two above

The driverrsquos estatersquos claim arose after Empire became Empire LLP Under UPA (1997) sect 306(c) an obligation incurred while a partnership is an LLP is solely a partnership obligationThus Adam Ben and Diane as partners are all protected from personal liability on the driverrsquos estatersquos claim But there may be personal liability if any of them was negligent or otherwise acted wrongfully by not informing the buyer of the bad suspension that caused the accident

29

National Conference of Bar Examiners 302 South Bedford Street | Madison WI 53703-3622 Phone 608-280-8550 | Fax 608-280-8552 | TDD 608-661-1275

wwwncbexorg e-mail contactncbexorg

  • Preface
  • Description of the MEE
  • Instructions
  • February 2014 Questions
    • Constitutinal Law Question
    • Trusts and Future Interests Question
    • Secured Transactions Question
    • Federal Civil Procedure Question
    • Criminal Law and Procedure Question
    • Agency and Partnership Question
      • February 2014 Analyses
        • Constitutional Law Analysis
        • Trust and Future Interests Analysis
        • Secured Transactions Analysis
        • Federal Civil Procedure Analysis
        • Criminal Law and Procedure Analysis
        • Agency and Partnership Analysis
            • ltlt13 ASCII85EncodePages false13 AllowTransparency false13 AutoPositionEPSFiles true13 AutoRotatePages None13 Binding Left13 CalGrayProfile (Dot Gain 20)13 CalRGBProfile (sRGB IEC61966-21)13 CalCMYKProfile (US Web Coated 050SWOP051 v2)13 sRGBProfile (sRGB IEC61966-21)13 CannotEmbedFontPolicy Error13 CompatibilityLevel 1413 CompressObjects Tags13 CompressPages true13 ConvertImagesToIndexed true13 PassThroughJPEGImages true13 CreateJobTicket false13 DefaultRenderingIntent Default13 DetectBlends true13 DetectCurves 0000013 ColorConversionStrategy CMYK13 DoThumbnails false13 EmbedAllFonts true13 EmbedOpenType false13 ParseICCProfilesInComments true13 EmbedJobOptions true13 DSCReportingLevel 013 EmitDSCWarnings false13 EndPage -113 ImageMemory 104857613 LockDistillerParams false13 MaxSubsetPct 10013 Optimize true13 OPM 113 ParseDSCComments true13 ParseDSCCommentsForDocInfo true13 PreserveCopyPage true13 PreserveDICMYKValues true13 PreserveEPSInfo true13 PreserveFlatness true13 PreserveHalftoneInfo false13 PreserveOPIComments true13 PreserveOverprintSettings true13 StartPage 113 SubsetFonts true13 TransferFunctionInfo Apply13 UCRandBGInfo Preserve13 UsePrologue false13 ColorSettingsFile ()13 AlwaysEmbed [ true13 ]13 NeverEmbed [ true13 ]13 AntiAliasColorImages false13 CropColorImages true13 ColorImageMinResolution 30013 ColorImageMinResolutionPolicy OK13 DownsampleColorImages true13 ColorImageDownsampleType Bicubic13 ColorImageResolution 30013 ColorImageDepth -113 ColorImageMinDownsampleDepth 113 ColorImageDownsampleThreshold 15000013 EncodeColorImages true13 ColorImageFilter DCTEncode13 AutoFilterColorImages true13 ColorImageAutoFilterStrategy JPEG13 ColorACSImageDict ltlt13 QFactor 01513 HSamples [1 1 1 1] VSamples [1 1 1 1]13 gtgt13 ColorImageDict ltlt13 QFactor 01513 HSamples [1 1 1 1] VSamples [1 1 1 1]13 gtgt13 JPEG2000ColorACSImageDict ltlt13 TileWidth 25613 TileHeight 25613 Quality 3013 gtgt13 JPEG2000ColorImageDict ltlt13 TileWidth 25613 TileHeight 25613 Quality 3013 gtgt13 AntiAliasGrayImages false13 CropGrayImages true13 GrayImageMinResolution 30013 GrayImageMinResolutionPolicy OK13 DownsampleGrayImages true13 GrayImageDownsampleType Bicubic13 GrayImageResolution 30013 GrayImageDepth -113 GrayImageMinDownsampleDepth 213 GrayImageDownsampleThreshold 15000013 EncodeGrayImages true13 GrayImageFilter DCTEncode13 AutoFilterGrayImages true13 GrayImageAutoFilterStrategy JPEG13 GrayACSImageDict ltlt13 QFactor 01513 HSamples [1 1 1 1] VSamples [1 1 1 1]13 gtgt13 GrayImageDict ltlt13 QFactor 01513 HSamples [1 1 1 1] VSamples [1 1 1 1]13 gtgt13 JPEG2000GrayACSImageDict ltlt13 TileWidth 25613 TileHeight 25613 Quality 3013 gtgt13 JPEG2000GrayImageDict ltlt13 TileWidth 25613 TileHeight 25613 Quality 3013 gtgt13 AntiAliasMonoImages false13 CropMonoImages true13 MonoImageMinResolution 120013 MonoImageMinResolutionPolicy OK13 DownsampleMonoImages true13 MonoImageDownsampleType Bicubic13 MonoImageResolution 120013 MonoImageDepth -113 MonoImageDownsampleThreshold 15000013 EncodeMonoImages true13 MonoImageFilter CCITTFaxEncode13 MonoImageDict ltlt13 K -113 gtgt13 AllowPSXObjects false13 CheckCompliance [13 None13 ]13 PDFX1aCheck false13 PDFX3Check false13 PDFXCompliantPDFOnly false13 PDFXNoTrimBoxError true13 PDFXTrimBoxToMediaBoxOffset [13 00000013 00000013 00000013 00000013 ]13 PDFXSetBleedBoxToMediaBox true13 PDFXBleedBoxToTrimBoxOffset [13 00000013 00000013 00000013 00000013 ]13 PDFXOutputIntentProfile ()13 PDFXOutputConditionIdentifier ()13 PDFXOutputCondition ()13 PDFXRegistryName ()13 PDFXTrapped False1313 CreateJDFFile false13 Description ltlt13 ARA 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 BGR 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 CHS ltFEFF4f7f75288fd94e9b8bbe5b9a521b5efa7684002000410064006f006200650020005000440046002065876863900275284e8e9ad88d2891cf76845370524d53705237300260a853ef4ee54f7f75280020004100630072006f0062006100740020548c002000410064006f00620065002000520065006100640065007200200035002e003000204ee553ca66f49ad87248672c676562535f00521b5efa768400200050004400460020658768633002gt13 CHT ltFEFF4f7f752890194e9b8a2d7f6e5efa7acb7684002000410064006f006200650020005000440046002065874ef69069752865bc9ad854c18cea76845370524d5370523786557406300260a853ef4ee54f7f75280020004100630072006f0062006100740020548c002000410064006f00620065002000520065006100640065007200200035002e003000204ee553ca66f49ad87248672c4f86958b555f5df25efa7acb76840020005000440046002065874ef63002gt13 CZE 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 DAN ltFEFF004200720075006700200069006e0064007300740069006c006c0069006e006700650072006e0065002000740069006c0020006100740020006f007000720065007400740065002000410064006f006200650020005000440046002d0064006f006b0075006d0065006e007400650072002c0020006400650072002000620065006400730074002000650067006e006500720020007300690067002000740069006c002000700072006500700072006500730073002d007500640073006b007200690076006e0069006e00670020006100660020006800f8006a0020006b00760061006c0069007400650074002e0020004400650020006f007000720065007400740065006400650020005000440046002d0064006f006b0075006d0065006e0074006500720020006b0061006e002000e50062006e00650073002000690020004100630072006f00620061007400200065006c006c006500720020004100630072006f006200610074002000520065006100640065007200200035002e00300020006f00670020006e0079006500720065002egt13 DEU 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 ESP ltFEFF005500740069006c0069006300650020006500730074006100200063006f006e0066006900670075007200610063006900f3006e0020007000610072006100200063007200650061007200200064006f00630075006d0065006e0074006f00730020005000440046002000640065002000410064006f0062006500200061006400650063007500610064006f00730020007000610072006100200069006d0070007200650073006900f3006e0020007000720065002d0065006400690074006f007200690061006c00200064006500200061006c00740061002000630061006c0069006400610064002e002000530065002000700075006500640065006e00200061006200720069007200200064006f00630075006d0065006e0074006f00730020005000440046002000630072006500610064006f007300200063006f006e0020004100630072006f006200610074002c002000410064006f00620065002000520065006100640065007200200035002e003000200079002000760065007200730069006f006e0065007300200070006f00730074006500720069006f007200650073002egt13 ETI 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 FRA 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 GRE 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 HEB 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 HRV (Za stvaranje Adobe PDF dokumenata najpogodnijih za visokokvalitetni ispis prije tiskanja koristite ove postavke Stvoreni PDF dokumenti mogu se otvoriti Acrobat i Adobe Reader 50 i kasnijim verzijama)13 HUN 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 ITA 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 JPN ltFEFF9ad854c18cea306a30d730ea30d730ec30b951fa529b7528002000410064006f0062006500200050004400460020658766f8306e4f5c6210306b4f7f75283057307e305930023053306e8a2d5b9a30674f5c62103055308c305f0020005000440046002030d530a130a430eb306f3001004100630072006f0062006100740020304a30883073002000410064006f00620065002000520065006100640065007200200035002e003000204ee5964d3067958b304f30533068304c3067304d307e305930023053306e8a2d5b9a306b306f30d530a930f330c8306e57cb30818fbc307f304c5fc59808306730593002gt13 KOR ltFEFFc7740020c124c815c7440020c0acc6a9d558c5ec0020ace0d488c9c80020c2dcd5d80020c778c1c4c5d00020ac00c7a50020c801d569d55c002000410064006f0062006500200050004400460020bb38c11cb97c0020c791c131d569b2c8b2e4002e0020c774b807ac8c0020c791c131b41c00200050004400460020bb38c11cb2940020004100630072006f0062006100740020bc0f002000410064006f00620065002000520065006100640065007200200035002e00300020c774c0c1c5d0c11c0020c5f40020c2180020c788c2b5b2c8b2e4002egt13 LTH 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 LVI 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 NLD (Gebruik deze instellingen om Adobe PDF-documenten te maken die zijn geoptimaliseerd voor prepress-afdrukken van hoge kwaliteit De gemaakte PDF-documenten kunnen worden geopend met Acrobat en Adobe Reader 50 en hoger)13 NOR ltFEFF004200720075006b00200064006900730073006500200069006e006e007300740069006c006c0069006e00670065006e0065002000740069006c002000e50020006f0070007000720065007400740065002000410064006f006200650020005000440046002d0064006f006b0075006d0065006e00740065007200200073006f006d00200065007200200062006500730074002000650067006e0065007400200066006f00720020006600f80072007400720079006b006b0073007500740073006b00720069006600740020006100760020006800f800790020006b00760061006c0069007400650074002e0020005000440046002d0064006f006b0075006d0065006e00740065006e00650020006b0061006e002000e50070006e00650073002000690020004100630072006f00620061007400200065006c006c00650072002000410064006f00620065002000520065006100640065007200200035002e003000200065006c006c00650072002000730065006e006500720065002egt13 POL ltFEFF0055007300740061007700690065006e0069006100200064006f002000740077006f0072007a0065006e0069006100200064006f006b0075006d0065006e007400f300770020005000440046002000700072007a0065007a006e00610063007a006f006e00790063006800200064006f002000770079006400720075006b00f30077002000770020007700790073006f006b00690065006a0020006a0061006b006f015b00630069002e002000200044006f006b0075006d0065006e0074007900200050004400460020006d006f017c006e00610020006f007400770069006500720061010700200077002000700072006f006700720061006d006900650020004100630072006f00620061007400200069002000410064006f00620065002000520065006100640065007200200035002e0030002000690020006e006f00770073007a0079006d002egt13 PTB 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 RUM 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 RUS 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 SKY ltFEFF0054006900650074006f0020006e006100730074006100760065006e0069006100200070006f0075017e0069007400650020006e00610020007600790074007600e100720061006e0069006500200064006f006b0075006d0065006e0074006f0076002000410064006f006200650020005000440046002c0020006b0074006f007200e90020007300610020006e0061006a006c0065007001610069006500200068006f0064006900610020006e00610020006b00760061006c00690074006e00fa00200074006c0061010d00200061002000700072006500700072006500730073002e00200056007900740076006f00720065006e00e900200064006f006b0075006d0065006e007400790020005000440046002000620075006400650020006d006f017e006e00e90020006f00740076006f00720069016500200076002000700072006f006700720061006d006f006300680020004100630072006f00620061007400200061002000410064006f00620065002000520065006100640065007200200035002e0030002000610020006e006f0076016100ed00630068002egt13 SLV 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 SUO 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 SVE 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 TUR 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 UKR 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 ENU (Use these settings to create Adobe PDF documents best suited for high-quality prepress printing Created PDF documents can be opened with Acrobat and Adobe Reader 50 and later)13 gtgt13 Namespace [13 (Adobe)13 (Common)13 (10)13 ]13 OtherNamespaces [13 ltlt13 AsReaderSpreads false13 CropImagesToFrames true13 ErrorControl WarnAndContinue13 FlattenerIgnoreSpreadOverrides false13 IncludeGuidesGrids false13 IncludeNonPrinting false13 IncludeSlug false13 Namespace [13 (Adobe)13 (InDesign)13 (40)13 ]13 OmitPlacedBitmaps false13 OmitPlacedEPS false13 OmitPlacedPDF false13 SimulateOverprint Legacy13 gtgt13 ltlt13 AddBleedMarks false13 AddColorBars false13 AddCropMarks false13 AddPageInfo false13 AddRegMarks false13 ConvertColors ConvertToCMYK13 DestinationProfileName ()13 DestinationProfileSelector DocumentCMYK13 Downsample16BitImages true13 FlattenerPreset ltlt13 PresetSelector MediumResolution13 gtgt13 FormElements false13 GenerateStructure false13 IncludeBookmarks false13 IncludeHyperlinks false13 IncludeInteractive false13 IncludeLayers false13 IncludeProfiles false13 MultimediaHandling UseObjectSettings13 Namespace [13 (Adobe)13 (CreativeSuite)13 (20)13 ]13 PDFXOutputIntentProfileSelector DocumentCMYK13 PreserveEditing true13 UntaggedCMYKHandling LeaveUntagged13 UntaggedRGBHandling UseDocumentProfile13 UseDocumentBleed false13 gtgt13 ]13gtgt setdistillerparams13ltlt13 HWResolution [2400 2400]13 PageSize [612000 792000]13gtgt setpagedevice13

Page 31: February 2014 MEE Questions and AnalysesPreface The Multistate Essay Examination (MEE) is developed by the National Conference of Bar Examiners (NCBE). This publication includes the

Agency and Partnership Analysis

in antique cars Thus Benrsquos wrongful act created a partnership obligation for which Adam was jointly and severally liable

[NOTE Generally a partnership creditor must ldquoexhaust the partnershiprsquos assets before levying on a judgment debtor partnerrsquos individual property where the partner is personally liable for the partnership obligationrdquo as a result of his status as a partner UPA (1997) sect 307 cmt 4 As the UPA comments explain this places Adam more in the position of guarantor than principal debtor on the partnership obligation Id cmt 4 Although an examinee might discuss this point the call focuses on whether Adam is personally liable not how the liability might be enforced]

Point Two (30) Because the collectorrsquos claim arose before Diane joined Empire Diane did not become personally liable on the claim

Diane was admitted to Empire when it was a general partnership and after the collectorrsquos claim arose While the general rule under UPA (1997) sect 306(a) is that the partners of a general partnership are liable jointly and severally for all obligations of the partnership there is a special rule for partners who are admitted during the duration of the partnership Under UPA (1997) sect 306(b) a person admitted to an existing partnership is not personally liable for any partnership obligations incurred before the personrsquos admission Because Diane was admitted to Empire after the collectorrsquos claim arose Diane is not personally liable on the claim

Dianersquos knowledge of the pre-existing claim and her stated concern about becoming liable on the collectorrsquos claim do not change her personal nonliability to the collector Although partners who have a liability shield can assume liability to third parties through private contractual guarantees or modifications to the partnership agreement Dianersquos stated concern constituted neither a guaranty to the collector nor ldquoan intentional waiver of liability protectionsrdquo See UPA (1997) sect 306 cmt 3 (describing methods for waiver of liability protections under sect 306(c) applicable in limited liability partnerships)

At most Diane will lose her investment in the partnership as a result of the collectorrsquos claim Although Diane did not become personally liable on the collectorrsquos claim when she joined the partnership the $250000 she contributed to the partnership is ldquoat risk for the satisfaction of existing partnership debtsrdquo UPA (1997) sect 306 cmt 2

Point Three (40) Filing the statement of qualification was effective to elect limited liability partnership status Despite this new status Adam and Ben remain personally liable on the collectorrsquos claim which arose before the election But as partners in an LLP neither Adam Ben nor Diane is personally liable as a partner on the driverrsquos estatersquos claim which arose after the election

Under UPA (1997) sect 1001 a general partnership can make an election and become a limited liability partnershipmdashif the partners approve the conversion by a vote equivalent to that necessary to amend the partnership agreement and the partnership then files a statement of qualification that specifies the name of the partnership its principal office and its election to be an LLP Here the partners agreed unanimouslymdashsufficient to amend their agreement under UPA (1997) sect 401(j)mdashand the statement of qualification was filed In addition the name of Empire LLP properly included an appropriate ending ldquoLLPrdquo See UPA (1997) sect 1002

Although another way to effectuate a ldquoconversionrdquo (as suggested by Benrsquos lawyer) is to form a new LLP and transfer the assets of the old general partnership to the new LLP the

28

Agency and Partnership Analysis

method used here (approval by the partners and the filing of a statement of qualification) is also sufficient to create LLP status

Thus Empire became Empire LLP as of the date of filing of the statement of qualification See UPA (1997) sect 1001 What effect did this have on the collectorrsquos claim which predated the filing According to UPA (1997) sect 306(c) an obligation incurred while a partnership is an LLP is solely a partnership obligation As the collectorrsquos claim predated the LLP Adam and Ben remain personally liable on the collectorrsquos claim Diane on the other hand was not personally liable on the collectorrsquos claim either before or after the filing of the statement of qualification See Point Two above

The driverrsquos estatersquos claim arose after Empire became Empire LLP Under UPA (1997) sect 306(c) an obligation incurred while a partnership is an LLP is solely a partnership obligationThus Adam Ben and Diane as partners are all protected from personal liability on the driverrsquos estatersquos claim But there may be personal liability if any of them was negligent or otherwise acted wrongfully by not informing the buyer of the bad suspension that caused the accident

29

National Conference of Bar Examiners 302 South Bedford Street | Madison WI 53703-3622 Phone 608-280-8550 | Fax 608-280-8552 | TDD 608-661-1275

wwwncbexorg e-mail contactncbexorg

  • Preface
  • Description of the MEE
  • Instructions
  • February 2014 Questions
    • Constitutinal Law Question
    • Trusts and Future Interests Question
    • Secured Transactions Question
    • Federal Civil Procedure Question
    • Criminal Law and Procedure Question
    • Agency and Partnership Question
      • February 2014 Analyses
        • Constitutional Law Analysis
        • Trust and Future Interests Analysis
        • Secured Transactions Analysis
        • Federal Civil Procedure Analysis
        • Criminal Law and Procedure Analysis
        • Agency and Partnership Analysis
            • ltlt13 ASCII85EncodePages false13 AllowTransparency false13 AutoPositionEPSFiles true13 AutoRotatePages None13 Binding Left13 CalGrayProfile (Dot Gain 20)13 CalRGBProfile (sRGB IEC61966-21)13 CalCMYKProfile (US Web Coated 050SWOP051 v2)13 sRGBProfile (sRGB IEC61966-21)13 CannotEmbedFontPolicy Error13 CompatibilityLevel 1413 CompressObjects Tags13 CompressPages true13 ConvertImagesToIndexed true13 PassThroughJPEGImages true13 CreateJobTicket false13 DefaultRenderingIntent Default13 DetectBlends true13 DetectCurves 0000013 ColorConversionStrategy CMYK13 DoThumbnails false13 EmbedAllFonts true13 EmbedOpenType false13 ParseICCProfilesInComments true13 EmbedJobOptions true13 DSCReportingLevel 013 EmitDSCWarnings false13 EndPage -113 ImageMemory 104857613 LockDistillerParams false13 MaxSubsetPct 10013 Optimize true13 OPM 113 ParseDSCComments true13 ParseDSCCommentsForDocInfo true13 PreserveCopyPage true13 PreserveDICMYKValues true13 PreserveEPSInfo true13 PreserveFlatness true13 PreserveHalftoneInfo false13 PreserveOPIComments true13 PreserveOverprintSettings true13 StartPage 113 SubsetFonts true13 TransferFunctionInfo Apply13 UCRandBGInfo Preserve13 UsePrologue false13 ColorSettingsFile ()13 AlwaysEmbed [ true13 ]13 NeverEmbed [ true13 ]13 AntiAliasColorImages false13 CropColorImages true13 ColorImageMinResolution 30013 ColorImageMinResolutionPolicy OK13 DownsampleColorImages true13 ColorImageDownsampleType Bicubic13 ColorImageResolution 30013 ColorImageDepth -113 ColorImageMinDownsampleDepth 113 ColorImageDownsampleThreshold 15000013 EncodeColorImages true13 ColorImageFilter DCTEncode13 AutoFilterColorImages true13 ColorImageAutoFilterStrategy JPEG13 ColorACSImageDict ltlt13 QFactor 01513 HSamples [1 1 1 1] VSamples [1 1 1 1]13 gtgt13 ColorImageDict ltlt13 QFactor 01513 HSamples [1 1 1 1] VSamples [1 1 1 1]13 gtgt13 JPEG2000ColorACSImageDict ltlt13 TileWidth 25613 TileHeight 25613 Quality 3013 gtgt13 JPEG2000ColorImageDict ltlt13 TileWidth 25613 TileHeight 25613 Quality 3013 gtgt13 AntiAliasGrayImages false13 CropGrayImages true13 GrayImageMinResolution 30013 GrayImageMinResolutionPolicy OK13 DownsampleGrayImages true13 GrayImageDownsampleType Bicubic13 GrayImageResolution 30013 GrayImageDepth -113 GrayImageMinDownsampleDepth 213 GrayImageDownsampleThreshold 15000013 EncodeGrayImages true13 GrayImageFilter DCTEncode13 AutoFilterGrayImages true13 GrayImageAutoFilterStrategy JPEG13 GrayACSImageDict ltlt13 QFactor 01513 HSamples [1 1 1 1] VSamples [1 1 1 1]13 gtgt13 GrayImageDict ltlt13 QFactor 01513 HSamples [1 1 1 1] VSamples [1 1 1 1]13 gtgt13 JPEG2000GrayACSImageDict ltlt13 TileWidth 25613 TileHeight 25613 Quality 3013 gtgt13 JPEG2000GrayImageDict ltlt13 TileWidth 25613 TileHeight 25613 Quality 3013 gtgt13 AntiAliasMonoImages false13 CropMonoImages true13 MonoImageMinResolution 120013 MonoImageMinResolutionPolicy OK13 DownsampleMonoImages true13 MonoImageDownsampleType Bicubic13 MonoImageResolution 120013 MonoImageDepth -113 MonoImageDownsampleThreshold 15000013 EncodeMonoImages true13 MonoImageFilter CCITTFaxEncode13 MonoImageDict ltlt13 K -113 gtgt13 AllowPSXObjects false13 CheckCompliance [13 None13 ]13 PDFX1aCheck false13 PDFX3Check false13 PDFXCompliantPDFOnly false13 PDFXNoTrimBoxError true13 PDFXTrimBoxToMediaBoxOffset [13 00000013 00000013 00000013 00000013 ]13 PDFXSetBleedBoxToMediaBox true13 PDFXBleedBoxToTrimBoxOffset [13 00000013 00000013 00000013 00000013 ]13 PDFXOutputIntentProfile ()13 PDFXOutputConditionIdentifier ()13 PDFXOutputCondition ()13 PDFXRegistryName ()13 PDFXTrapped False1313 CreateJDFFile false13 Description ltlt13 ARA 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 BGR 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 CHS ltFEFF4f7f75288fd94e9b8bbe5b9a521b5efa7684002000410064006f006200650020005000440046002065876863900275284e8e9ad88d2891cf76845370524d53705237300260a853ef4ee54f7f75280020004100630072006f0062006100740020548c002000410064006f00620065002000520065006100640065007200200035002e003000204ee553ca66f49ad87248672c676562535f00521b5efa768400200050004400460020658768633002gt13 CHT ltFEFF4f7f752890194e9b8a2d7f6e5efa7acb7684002000410064006f006200650020005000440046002065874ef69069752865bc9ad854c18cea76845370524d5370523786557406300260a853ef4ee54f7f75280020004100630072006f0062006100740020548c002000410064006f00620065002000520065006100640065007200200035002e003000204ee553ca66f49ad87248672c4f86958b555f5df25efa7acb76840020005000440046002065874ef63002gt13 CZE 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 DAN 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 DEU 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 ESP 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 ETI 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 FRA 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 GRE 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 HEB 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 HRV (Za stvaranje Adobe PDF dokumenata najpogodnijih za visokokvalitetni ispis prije tiskanja koristite ove postavke Stvoreni PDF dokumenti mogu se otvoriti Acrobat i Adobe Reader 50 i kasnijim verzijama)13 HUN ltFEFF004b0069007600e1006c00f30020006d0069006e0151007300e9006701710020006e0079006f006d00640061006900200065006c0151006b00e90073007a00ed007401510020006e0079006f006d00740061007400e100730068006f007a0020006c006500670069006e006b00e1006200620020006d0065006700660065006c0065006c0151002000410064006f00620065002000500044004600200064006f006b0075006d0065006e00740075006d006f006b0061007400200065007a0065006b006b0065006c0020006100200062006500e1006c006c00ed007400e10073006f006b006b0061006c0020006b00e90073007a00ed0074006800650074002e0020002000410020006c00e90074007200650068006f007a006f00740074002000500044004600200064006f006b0075006d0065006e00740075006d006f006b00200061007a0020004100630072006f006200610074002000e9007300200061007a002000410064006f00620065002000520065006100640065007200200035002e0030002c0020007600610067007900200061007a002000610074007400f3006c0020006b00e9007301510062006200690020007600650072007a006900f3006b006b0061006c0020006e00790069007400680061007400f3006b0020006d00650067002egt13 ITA 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 JPN ltFEFF9ad854c18cea306a30d730ea30d730ec30b951fa529b7528002000410064006f0062006500200050004400460020658766f8306e4f5c6210306b4f7f75283057307e305930023053306e8a2d5b9a30674f5c62103055308c305f0020005000440046002030d530a130a430eb306f3001004100630072006f0062006100740020304a30883073002000410064006f00620065002000520065006100640065007200200035002e003000204ee5964d3067958b304f30533068304c3067304d307e305930023053306e8a2d5b9a306b306f30d530a930f330c8306e57cb30818fbc307f304c5fc59808306730593002gt13 KOR ltFEFFc7740020c124c815c7440020c0acc6a9d558c5ec0020ace0d488c9c80020c2dcd5d80020c778c1c4c5d00020ac00c7a50020c801d569d55c002000410064006f0062006500200050004400460020bb38c11cb97c0020c791c131d569b2c8b2e4002e0020c774b807ac8c0020c791c131b41c00200050004400460020bb38c11cb2940020004100630072006f0062006100740020bc0f002000410064006f00620065002000520065006100640065007200200035002e00300020c774c0c1c5d0c11c0020c5f40020c2180020c788c2b5b2c8b2e4002egt13 LTH 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 LVI 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 NLD (Gebruik deze instellingen om Adobe PDF-documenten te maken die zijn geoptimaliseerd voor prepress-afdrukken van hoge kwaliteit De gemaakte PDF-documenten kunnen worden geopend met Acrobat en Adobe Reader 50 en hoger)13 NOR 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 POL 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 PTB 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 RUM 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 RUS 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 SKY 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 SLV 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 SUO 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 SVE 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 TUR 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 UKR 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 ENU (Use these settings to create Adobe PDF documents best suited for high-quality prepress printing Created PDF documents can be opened with Acrobat and Adobe Reader 50 and later)13 gtgt13 Namespace [13 (Adobe)13 (Common)13 (10)13 ]13 OtherNamespaces [13 ltlt13 AsReaderSpreads false13 CropImagesToFrames true13 ErrorControl WarnAndContinue13 FlattenerIgnoreSpreadOverrides false13 IncludeGuidesGrids false13 IncludeNonPrinting false13 IncludeSlug false13 Namespace [13 (Adobe)13 (InDesign)13 (40)13 ]13 OmitPlacedBitmaps false13 OmitPlacedEPS false13 OmitPlacedPDF false13 SimulateOverprint Legacy13 gtgt13 ltlt13 AddBleedMarks false13 AddColorBars false13 AddCropMarks false13 AddPageInfo false13 AddRegMarks false13 ConvertColors ConvertToCMYK13 DestinationProfileName ()13 DestinationProfileSelector DocumentCMYK13 Downsample16BitImages true13 FlattenerPreset ltlt13 PresetSelector MediumResolution13 gtgt13 FormElements false13 GenerateStructure false13 IncludeBookmarks false13 IncludeHyperlinks false13 IncludeInteractive false13 IncludeLayers false13 IncludeProfiles false13 MultimediaHandling UseObjectSettings13 Namespace [13 (Adobe)13 (CreativeSuite)13 (20)13 ]13 PDFXOutputIntentProfileSelector DocumentCMYK13 PreserveEditing true13 UntaggedCMYKHandling LeaveUntagged13 UntaggedRGBHandling UseDocumentProfile13 UseDocumentBleed false13 gtgt13 ]13gtgt setdistillerparams13ltlt13 HWResolution [2400 2400]13 PageSize [612000 792000]13gtgt setpagedevice13

Page 32: February 2014 MEE Questions and AnalysesPreface The Multistate Essay Examination (MEE) is developed by the National Conference of Bar Examiners (NCBE). This publication includes the

Agency and Partnership Analysis

method used here (approval by the partners and the filing of a statement of qualification) is also sufficient to create LLP status

Thus Empire became Empire LLP as of the date of filing of the statement of qualification See UPA (1997) sect 1001 What effect did this have on the collectorrsquos claim which predated the filing According to UPA (1997) sect 306(c) an obligation incurred while a partnership is an LLP is solely a partnership obligation As the collectorrsquos claim predated the LLP Adam and Ben remain personally liable on the collectorrsquos claim Diane on the other hand was not personally liable on the collectorrsquos claim either before or after the filing of the statement of qualification See Point Two above

The driverrsquos estatersquos claim arose after Empire became Empire LLP Under UPA (1997) sect 306(c) an obligation incurred while a partnership is an LLP is solely a partnership obligationThus Adam Ben and Diane as partners are all protected from personal liability on the driverrsquos estatersquos claim But there may be personal liability if any of them was negligent or otherwise acted wrongfully by not informing the buyer of the bad suspension that caused the accident

29

National Conference of Bar Examiners 302 South Bedford Street | Madison WI 53703-3622 Phone 608-280-8550 | Fax 608-280-8552 | TDD 608-661-1275

wwwncbexorg e-mail contactncbexorg

  • Preface
  • Description of the MEE
  • Instructions
  • February 2014 Questions
    • Constitutinal Law Question
    • Trusts and Future Interests Question
    • Secured Transactions Question
    • Federal Civil Procedure Question
    • Criminal Law and Procedure Question
    • Agency and Partnership Question
      • February 2014 Analyses
        • Constitutional Law Analysis
        • Trust and Future Interests Analysis
        • Secured Transactions Analysis
        • Federal Civil Procedure Analysis
        • Criminal Law and Procedure Analysis
        • Agency and Partnership Analysis
            • ltlt13 ASCII85EncodePages false13 AllowTransparency false13 AutoPositionEPSFiles true13 AutoRotatePages None13 Binding Left13 CalGrayProfile (Dot Gain 20)13 CalRGBProfile (sRGB IEC61966-21)13 CalCMYKProfile (US Web Coated 050SWOP051 v2)13 sRGBProfile (sRGB IEC61966-21)13 CannotEmbedFontPolicy Error13 CompatibilityLevel 1413 CompressObjects Tags13 CompressPages true13 ConvertImagesToIndexed true13 PassThroughJPEGImages true13 CreateJobTicket false13 DefaultRenderingIntent Default13 DetectBlends true13 DetectCurves 0000013 ColorConversionStrategy CMYK13 DoThumbnails false13 EmbedAllFonts true13 EmbedOpenType false13 ParseICCProfilesInComments true13 EmbedJobOptions true13 DSCReportingLevel 013 EmitDSCWarnings false13 EndPage -113 ImageMemory 104857613 LockDistillerParams false13 MaxSubsetPct 10013 Optimize true13 OPM 113 ParseDSCComments true13 ParseDSCCommentsForDocInfo true13 PreserveCopyPage true13 PreserveDICMYKValues true13 PreserveEPSInfo true13 PreserveFlatness true13 PreserveHalftoneInfo false13 PreserveOPIComments true13 PreserveOverprintSettings true13 StartPage 113 SubsetFonts true13 TransferFunctionInfo Apply13 UCRandBGInfo Preserve13 UsePrologue false13 ColorSettingsFile ()13 AlwaysEmbed [ true13 ]13 NeverEmbed [ true13 ]13 AntiAliasColorImages false13 CropColorImages true13 ColorImageMinResolution 30013 ColorImageMinResolutionPolicy OK13 DownsampleColorImages true13 ColorImageDownsampleType Bicubic13 ColorImageResolution 30013 ColorImageDepth -113 ColorImageMinDownsampleDepth 113 ColorImageDownsampleThreshold 15000013 EncodeColorImages true13 ColorImageFilter DCTEncode13 AutoFilterColorImages true13 ColorImageAutoFilterStrategy JPEG13 ColorACSImageDict ltlt13 QFactor 01513 HSamples [1 1 1 1] VSamples [1 1 1 1]13 gtgt13 ColorImageDict ltlt13 QFactor 01513 HSamples [1 1 1 1] VSamples [1 1 1 1]13 gtgt13 JPEG2000ColorACSImageDict ltlt13 TileWidth 25613 TileHeight 25613 Quality 3013 gtgt13 JPEG2000ColorImageDict ltlt13 TileWidth 25613 TileHeight 25613 Quality 3013 gtgt13 AntiAliasGrayImages false13 CropGrayImages true13 GrayImageMinResolution 30013 GrayImageMinResolutionPolicy OK13 DownsampleGrayImages true13 GrayImageDownsampleType Bicubic13 GrayImageResolution 30013 GrayImageDepth -113 GrayImageMinDownsampleDepth 213 GrayImageDownsampleThreshold 15000013 EncodeGrayImages true13 GrayImageFilter DCTEncode13 AutoFilterGrayImages true13 GrayImageAutoFilterStrategy JPEG13 GrayACSImageDict ltlt13 QFactor 01513 HSamples [1 1 1 1] VSamples [1 1 1 1]13 gtgt13 GrayImageDict ltlt13 QFactor 01513 HSamples [1 1 1 1] VSamples [1 1 1 1]13 gtgt13 JPEG2000GrayACSImageDict ltlt13 TileWidth 25613 TileHeight 25613 Quality 3013 gtgt13 JPEG2000GrayImageDict ltlt13 TileWidth 25613 TileHeight 25613 Quality 3013 gtgt13 AntiAliasMonoImages false13 CropMonoImages true13 MonoImageMinResolution 120013 MonoImageMinResolutionPolicy OK13 DownsampleMonoImages true13 MonoImageDownsampleType Bicubic13 MonoImageResolution 120013 MonoImageDepth -113 MonoImageDownsampleThreshold 15000013 EncodeMonoImages true13 MonoImageFilter CCITTFaxEncode13 MonoImageDict ltlt13 K -113 gtgt13 AllowPSXObjects false13 CheckCompliance [13 None13 ]13 PDFX1aCheck false13 PDFX3Check false13 PDFXCompliantPDFOnly false13 PDFXNoTrimBoxError true13 PDFXTrimBoxToMediaBoxOffset [13 00000013 00000013 00000013 00000013 ]13 PDFXSetBleedBoxToMediaBox true13 PDFXBleedBoxToTrimBoxOffset [13 00000013 00000013 00000013 00000013 ]13 PDFXOutputIntentProfile ()13 PDFXOutputConditionIdentifier ()13 PDFXOutputCondition ()13 PDFXRegistryName ()13 PDFXTrapped False1313 CreateJDFFile false13 Description ltlt13 ARA 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 BGR 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 CHS ltFEFF4f7f75288fd94e9b8bbe5b9a521b5efa7684002000410064006f006200650020005000440046002065876863900275284e8e9ad88d2891cf76845370524d53705237300260a853ef4ee54f7f75280020004100630072006f0062006100740020548c002000410064006f00620065002000520065006100640065007200200035002e003000204ee553ca66f49ad87248672c676562535f00521b5efa768400200050004400460020658768633002gt13 CHT ltFEFF4f7f752890194e9b8a2d7f6e5efa7acb7684002000410064006f006200650020005000440046002065874ef69069752865bc9ad854c18cea76845370524d5370523786557406300260a853ef4ee54f7f75280020004100630072006f0062006100740020548c002000410064006f00620065002000520065006100640065007200200035002e003000204ee553ca66f49ad87248672c4f86958b555f5df25efa7acb76840020005000440046002065874ef63002gt13 CZE 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 DAN ltFEFF004200720075006700200069006e0064007300740069006c006c0069006e006700650072006e0065002000740069006c0020006100740020006f007000720065007400740065002000410064006f006200650020005000440046002d0064006f006b0075006d0065006e007400650072002c0020006400650072002000620065006400730074002000650067006e006500720020007300690067002000740069006c002000700072006500700072006500730073002d007500640073006b007200690076006e0069006e00670020006100660020006800f8006a0020006b00760061006c0069007400650074002e0020004400650020006f007000720065007400740065006400650020005000440046002d0064006f006b0075006d0065006e0074006500720020006b0061006e002000e50062006e00650073002000690020004100630072006f00620061007400200065006c006c006500720020004100630072006f006200610074002000520065006100640065007200200035002e00300020006f00670020006e0079006500720065002egt13 DEU 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 ESP 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 ETI 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 FRA 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 GRE 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 HEB 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 HRV (Za stvaranje Adobe PDF dokumenata najpogodnijih za visokokvalitetni ispis prije tiskanja koristite ove postavke Stvoreni PDF dokumenti mogu se otvoriti Acrobat i Adobe Reader 50 i kasnijim verzijama)13 HUN 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 ITA 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 JPN ltFEFF9ad854c18cea306a30d730ea30d730ec30b951fa529b7528002000410064006f0062006500200050004400460020658766f8306e4f5c6210306b4f7f75283057307e305930023053306e8a2d5b9a30674f5c62103055308c305f0020005000440046002030d530a130a430eb306f3001004100630072006f0062006100740020304a30883073002000410064006f00620065002000520065006100640065007200200035002e003000204ee5964d3067958b304f30533068304c3067304d307e305930023053306e8a2d5b9a306b306f30d530a930f330c8306e57cb30818fbc307f304c5fc59808306730593002gt13 KOR ltFEFFc7740020c124c815c7440020c0acc6a9d558c5ec0020ace0d488c9c80020c2dcd5d80020c778c1c4c5d00020ac00c7a50020c801d569d55c002000410064006f0062006500200050004400460020bb38c11cb97c0020c791c131d569b2c8b2e4002e0020c774b807ac8c0020c791c131b41c00200050004400460020bb38c11cb2940020004100630072006f0062006100740020bc0f002000410064006f00620065002000520065006100640065007200200035002e00300020c774c0c1c5d0c11c0020c5f40020c2180020c788c2b5b2c8b2e4002egt13 LTH 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 LVI 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 NLD (Gebruik deze instellingen om Adobe PDF-documenten te maken die zijn geoptimaliseerd voor prepress-afdrukken van hoge kwaliteit De gemaakte PDF-documenten kunnen worden geopend met Acrobat en Adobe Reader 50 en hoger)13 NOR ltFEFF004200720075006b00200064006900730073006500200069006e006e007300740069006c006c0069006e00670065006e0065002000740069006c002000e50020006f0070007000720065007400740065002000410064006f006200650020005000440046002d0064006f006b0075006d0065006e00740065007200200073006f006d00200065007200200062006500730074002000650067006e0065007400200066006f00720020006600f80072007400720079006b006b0073007500740073006b00720069006600740020006100760020006800f800790020006b00760061006c0069007400650074002e0020005000440046002d0064006f006b0075006d0065006e00740065006e00650020006b0061006e002000e50070006e00650073002000690020004100630072006f00620061007400200065006c006c00650072002000410064006f00620065002000520065006100640065007200200035002e003000200065006c006c00650072002000730065006e006500720065002egt13 POL 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 PTB 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 RUM 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 RUS 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 SKY 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 SLV 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 SUO 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 SVE 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 TUR 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 UKR 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 ENU (Use these settings to create Adobe PDF documents best suited for high-quality prepress printing Created PDF documents can be opened with Acrobat and Adobe Reader 50 and later)13 gtgt13 Namespace [13 (Adobe)13 (Common)13 (10)13 ]13 OtherNamespaces [13 ltlt13 AsReaderSpreads false13 CropImagesToFrames true13 ErrorControl WarnAndContinue13 FlattenerIgnoreSpreadOverrides false13 IncludeGuidesGrids false13 IncludeNonPrinting false13 IncludeSlug false13 Namespace [13 (Adobe)13 (InDesign)13 (40)13 ]13 OmitPlacedBitmaps false13 OmitPlacedEPS false13 OmitPlacedPDF false13 SimulateOverprint Legacy13 gtgt13 ltlt13 AddBleedMarks false13 AddColorBars false13 AddCropMarks false13 AddPageInfo false13 AddRegMarks false13 ConvertColors ConvertToCMYK13 DestinationProfileName ()13 DestinationProfileSelector DocumentCMYK13 Downsample16BitImages true13 FlattenerPreset ltlt13 PresetSelector MediumResolution13 gtgt13 FormElements false13 GenerateStructure false13 IncludeBookmarks false13 IncludeHyperlinks false13 IncludeInteractive false13 IncludeLayers false13 IncludeProfiles false13 MultimediaHandling UseObjectSettings13 Namespace [13 (Adobe)13 (CreativeSuite)13 (20)13 ]13 PDFXOutputIntentProfileSelector DocumentCMYK13 PreserveEditing true13 UntaggedCMYKHandling LeaveUntagged13 UntaggedRGBHandling UseDocumentProfile13 UseDocumentBleed false13 gtgt13 ]13gtgt setdistillerparams13ltlt13 HWResolution [2400 2400]13 PageSize [612000 792000]13gtgt setpagedevice13

Page 33: February 2014 MEE Questions and AnalysesPreface The Multistate Essay Examination (MEE) is developed by the National Conference of Bar Examiners (NCBE). This publication includes the

National Conference of Bar Examiners 302 South Bedford Street | Madison WI 53703-3622 Phone 608-280-8550 | Fax 608-280-8552 | TDD 608-661-1275

wwwncbexorg e-mail contactncbexorg

  • Preface
  • Description of the MEE
  • Instructions
  • February 2014 Questions
    • Constitutinal Law Question
    • Trusts and Future Interests Question
    • Secured Transactions Question
    • Federal Civil Procedure Question
    • Criminal Law and Procedure Question
    • Agency and Partnership Question
      • February 2014 Analyses
        • Constitutional Law Analysis
        • Trust and Future Interests Analysis
        • Secured Transactions Analysis
        • Federal Civil Procedure Analysis
        • Criminal Law and Procedure Analysis
        • Agency and Partnership Analysis
            • ltlt13 ASCII85EncodePages false13 AllowTransparency false13 AutoPositionEPSFiles true13 AutoRotatePages None13 Binding Left13 CalGrayProfile (Dot Gain 20)13 CalRGBProfile (sRGB IEC61966-21)13 CalCMYKProfile (US Web Coated 050SWOP051 v2)13 sRGBProfile (sRGB IEC61966-21)13 CannotEmbedFontPolicy Error13 CompatibilityLevel 1413 CompressObjects Tags13 CompressPages true13 ConvertImagesToIndexed true13 PassThroughJPEGImages true13 CreateJobTicket false13 DefaultRenderingIntent Default13 DetectBlends true13 DetectCurves 0000013 ColorConversionStrategy CMYK13 DoThumbnails false13 EmbedAllFonts true13 EmbedOpenType false13 ParseICCProfilesInComments true13 EmbedJobOptions true13 DSCReportingLevel 013 EmitDSCWarnings false13 EndPage -113 ImageMemory 104857613 LockDistillerParams false13 MaxSubsetPct 10013 Optimize true13 OPM 113 ParseDSCComments true13 ParseDSCCommentsForDocInfo true13 PreserveCopyPage true13 PreserveDICMYKValues true13 PreserveEPSInfo true13 PreserveFlatness true13 PreserveHalftoneInfo false13 PreserveOPIComments true13 PreserveOverprintSettings true13 StartPage 113 SubsetFonts true13 TransferFunctionInfo Apply13 UCRandBGInfo Preserve13 UsePrologue false13 ColorSettingsFile ()13 AlwaysEmbed [ true13 ]13 NeverEmbed [ true13 ]13 AntiAliasColorImages false13 CropColorImages true13 ColorImageMinResolution 30013 ColorImageMinResolutionPolicy OK13 DownsampleColorImages true13 ColorImageDownsampleType Bicubic13 ColorImageResolution 30013 ColorImageDepth -113 ColorImageMinDownsampleDepth 113 ColorImageDownsampleThreshold 15000013 EncodeColorImages true13 ColorImageFilter DCTEncode13 AutoFilterColorImages true13 ColorImageAutoFilterStrategy JPEG13 ColorACSImageDict ltlt13 QFactor 01513 HSamples [1 1 1 1] VSamples [1 1 1 1]13 gtgt13 ColorImageDict ltlt13 QFactor 01513 HSamples [1 1 1 1] VSamples [1 1 1 1]13 gtgt13 JPEG2000ColorACSImageDict ltlt13 TileWidth 25613 TileHeight 25613 Quality 3013 gtgt13 JPEG2000ColorImageDict ltlt13 TileWidth 25613 TileHeight 25613 Quality 3013 gtgt13 AntiAliasGrayImages false13 CropGrayImages true13 GrayImageMinResolution 30013 GrayImageMinResolutionPolicy OK13 DownsampleGrayImages true13 GrayImageDownsampleType Bicubic13 GrayImageResolution 30013 GrayImageDepth -113 GrayImageMinDownsampleDepth 213 GrayImageDownsampleThreshold 15000013 EncodeGrayImages true13 GrayImageFilter DCTEncode13 AutoFilterGrayImages true13 GrayImageAutoFilterStrategy JPEG13 GrayACSImageDict ltlt13 QFactor 01513 HSamples [1 1 1 1] VSamples [1 1 1 1]13 gtgt13 GrayImageDict ltlt13 QFactor 01513 HSamples [1 1 1 1] VSamples [1 1 1 1]13 gtgt13 JPEG2000GrayACSImageDict ltlt13 TileWidth 25613 TileHeight 25613 Quality 3013 gtgt13 JPEG2000GrayImageDict ltlt13 TileWidth 25613 TileHeight 25613 Quality 3013 gtgt13 AntiAliasMonoImages false13 CropMonoImages true13 MonoImageMinResolution 120013 MonoImageMinResolutionPolicy OK13 DownsampleMonoImages true13 MonoImageDownsampleType Bicubic13 MonoImageResolution 120013 MonoImageDepth -113 MonoImageDownsampleThreshold 15000013 EncodeMonoImages true13 MonoImageFilter CCITTFaxEncode13 MonoImageDict ltlt13 K -113 gtgt13 AllowPSXObjects false13 CheckCompliance [13 None13 ]13 PDFX1aCheck false13 PDFX3Check false13 PDFXCompliantPDFOnly false13 PDFXNoTrimBoxError true13 PDFXTrimBoxToMediaBoxOffset [13 00000013 00000013 00000013 00000013 ]13 PDFXSetBleedBoxToMediaBox true13 PDFXBleedBoxToTrimBoxOffset [13 00000013 00000013 00000013 00000013 ]13 PDFXOutputIntentProfile ()13 PDFXOutputConditionIdentifier ()13 PDFXOutputCondition ()13 PDFXRegistryName ()13 PDFXTrapped False1313 CreateJDFFile false13 Description ltlt13 ARA 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 BGR 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 CHS ltFEFF4f7f75288fd94e9b8bbe5b9a521b5efa7684002000410064006f006200650020005000440046002065876863900275284e8e9ad88d2891cf76845370524d53705237300260a853ef4ee54f7f75280020004100630072006f0062006100740020548c002000410064006f00620065002000520065006100640065007200200035002e003000204ee553ca66f49ad87248672c676562535f00521b5efa768400200050004400460020658768633002gt13 CHT ltFEFF4f7f752890194e9b8a2d7f6e5efa7acb7684002000410064006f006200650020005000440046002065874ef69069752865bc9ad854c18cea76845370524d5370523786557406300260a853ef4ee54f7f75280020004100630072006f0062006100740020548c002000410064006f00620065002000520065006100640065007200200035002e003000204ee553ca66f49ad87248672c4f86958b555f5df25efa7acb76840020005000440046002065874ef63002gt13 CZE 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 DAN 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 DEU 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 ESP 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 ETI 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 FRA 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 GRE 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 HEB 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 HRV (Za stvaranje Adobe PDF dokumenata najpogodnijih za visokokvalitetni ispis prije tiskanja koristite ove postavke Stvoreni PDF dokumenti mogu se otvoriti Acrobat i Adobe Reader 50 i kasnijim verzijama)13 HUN ltFEFF004b0069007600e1006c00f30020006d0069006e0151007300e9006701710020006e0079006f006d00640061006900200065006c0151006b00e90073007a00ed007401510020006e0079006f006d00740061007400e100730068006f007a0020006c006500670069006e006b00e1006200620020006d0065006700660065006c0065006c0151002000410064006f00620065002000500044004600200064006f006b0075006d0065006e00740075006d006f006b0061007400200065007a0065006b006b0065006c0020006100200062006500e1006c006c00ed007400e10073006f006b006b0061006c0020006b00e90073007a00ed0074006800650074002e0020002000410020006c00e90074007200650068006f007a006f00740074002000500044004600200064006f006b0075006d0065006e00740075006d006f006b00200061007a0020004100630072006f006200610074002000e9007300200061007a002000410064006f00620065002000520065006100640065007200200035002e0030002c0020007600610067007900200061007a002000610074007400f3006c0020006b00e9007301510062006200690020007600650072007a006900f3006b006b0061006c0020006e00790069007400680061007400f3006b0020006d00650067002egt13 ITA 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 JPN ltFEFF9ad854c18cea306a30d730ea30d730ec30b951fa529b7528002000410064006f0062006500200050004400460020658766f8306e4f5c6210306b4f7f75283057307e305930023053306e8a2d5b9a30674f5c62103055308c305f0020005000440046002030d530a130a430eb306f3001004100630072006f0062006100740020304a30883073002000410064006f00620065002000520065006100640065007200200035002e003000204ee5964d3067958b304f30533068304c3067304d307e305930023053306e8a2d5b9a306b306f30d530a930f330c8306e57cb30818fbc307f304c5fc59808306730593002gt13 KOR ltFEFFc7740020c124c815c7440020c0acc6a9d558c5ec0020ace0d488c9c80020c2dcd5d80020c778c1c4c5d00020ac00c7a50020c801d569d55c002000410064006f0062006500200050004400460020bb38c11cb97c0020c791c131d569b2c8b2e4002e0020c774b807ac8c0020c791c131b41c00200050004400460020bb38c11cb2940020004100630072006f0062006100740020bc0f002000410064006f00620065002000520065006100640065007200200035002e00300020c774c0c1c5d0c11c0020c5f40020c2180020c788c2b5b2c8b2e4002egt13 LTH 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 LVI 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 NLD (Gebruik deze instellingen om Adobe PDF-documenten te maken die zijn geoptimaliseerd voor prepress-afdrukken van hoge kwaliteit De gemaakte PDF-documenten kunnen worden geopend met Acrobat en Adobe Reader 50 en hoger)13 NOR ltFEFF004200720075006b00200064006900730073006500200069006e006e007300740069006c006c0069006e00670065006e0065002000740069006c002000e50020006f0070007000720065007400740065002000410064006f006200650020005000440046002d0064006f006b0075006d0065006e00740065007200200073006f006d00200065007200200062006500730074002000650067006e0065007400200066006f00720020006600f80072007400720079006b006b0073007500740073006b00720069006600740020006100760020006800f800790020006b00760061006c0069007400650074002e0020005000440046002d0064006f006b0075006d0065006e00740065006e00650020006b0061006e002000e50070006e00650073002000690020004100630072006f00620061007400200065006c006c00650072002000410064006f00620065002000520065006100640065007200200035002e003000200065006c006c00650072002000730065006e006500720065002egt13 POL 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 PTB ltFEFF005500740069006c0069007a006500200065007300730061007300200063006f006e00660069006700750072006100e700f50065007300200064006500200066006f0072006d00610020006100200063007200690061007200200064006f00630075006d0065006e0074006f0073002000410064006f0062006500200050004400460020006d00610069007300200061006400650071007500610064006f00730020007000610072006100200070007200e9002d0069006d0070007200650073007300f50065007300200064006500200061006c007400610020007100750061006c00690064006100640065002e0020004f007300200064006f00630075006d0065006e0074006f00730020005000440046002000630072006900610064006f007300200070006f00640065006d0020007300650072002000610062006500720074006f007300200063006f006d0020006f0020004100630072006f006200610074002000650020006f002000410064006f00620065002000520065006100640065007200200035002e0030002000650020007600650072007300f50065007300200070006f00730074006500720069006f007200650073002egt13 RUM 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 RUS ltFEFF04180441043f043e043b044c04370443043904420435002004340430043d043d044b04350020043d0430044104420440043e0439043a043800200434043b044f00200441043e043704340430043d0438044f00200434043e043a0443043c0435043d0442043e0432002000410064006f006200650020005000440046002c0020043c0430043a04410438043c0430043b044c043d043e0020043f043e04340445043e0434044f04490438044500200434043b044f00200432044b0441043e043a043e043a0430044704350441044204320435043d043d043e0433043e00200434043e043f0435044704300442043d043e0433043e00200432044b0432043e04340430002e002000200421043e043704340430043d043d044b04350020005000440046002d0434043e043a0443043c0435043d0442044b0020043c043e0436043d043e0020043e0442043a0440044b043204300442044c002004410020043f043e043c043e0449044c044e0020004100630072006f00620061007400200438002000410064006f00620065002000520065006100640065007200200035002e00300020043800200431043e043b043504350020043f043e04370434043d043804450020043204350440044104380439002egt13 SKY 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 SLV 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 SUO ltFEFF004b00e40079007400e40020006e00e40069007400e4002000610073006500740075006b007300690061002c0020006b0075006e0020006c0075006f00740020006c00e400680069006e006e00e4002000760061006100740069007600610061006e0020007000610069006e006100740075006b00730065006e002000760061006c006d0069007300740065006c00750074007900f6006800f6006e00200073006f00700069007600690061002000410064006f0062006500200050004400460020002d0064006f006b0075006d0065006e007400740065006a0061002e0020004c0075006f0064007500740020005000440046002d0064006f006b0075006d0065006e00740069007400200076006f0069006400610061006e0020006100760061007400610020004100630072006f0062006100740069006c006c00610020006a0061002000410064006f00620065002000520065006100640065007200200035002e0030003a006c006c00610020006a006100200075007500640065006d006d0069006c006c0061002egt13 SVE 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 TUR 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 UKR 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 ENU (Use these settings to create Adobe PDF documents best suited for high-quality prepress printing Created PDF documents can be opened with Acrobat and Adobe Reader 50 and later)13 gtgt13 Namespace [13 (Adobe)13 (Common)13 (10)13 ]13 OtherNamespaces [13 ltlt13 AsReaderSpreads false13 CropImagesToFrames true13 ErrorControl WarnAndContinue13 FlattenerIgnoreSpreadOverrides false13 IncludeGuidesGrids false13 IncludeNonPrinting false13 IncludeSlug false13 Namespace [13 (Adobe)13 (InDesign)13 (40)13 ]13 OmitPlacedBitmaps false13 OmitPlacedEPS false13 OmitPlacedPDF false13 SimulateOverprint Legacy13 gtgt13 ltlt13 AddBleedMarks false13 AddColorBars false13 AddCropMarks false13 AddPageInfo false13 AddRegMarks false13 ConvertColors ConvertToCMYK13 DestinationProfileName ()13 DestinationProfileSelector DocumentCMYK13 Downsample16BitImages true13 FlattenerPreset ltlt13 PresetSelector MediumResolution13 gtgt13 FormElements false13 GenerateStructure false13 IncludeBookmarks false13 IncludeHyperlinks false13 IncludeInteractive false13 IncludeLayers false13 IncludeProfiles false13 MultimediaHandling UseObjectSettings13 Namespace [13 (Adobe)13 (CreativeSuite)13 (20)13 ]13 PDFXOutputIntentProfileSelector DocumentCMYK13 PreserveEditing true13 UntaggedCMYKHandling LeaveUntagged13 UntaggedRGBHandling UseDocumentProfile13 UseDocumentBleed false13 gtgt13 ]13gtgt setdistillerparams13ltlt13 HWResolution [2400 2400]13 PageSize [612000 792000]13gtgt setpagedevice13